You are on page 1of 955

KINEMATICS

DHANALAKSHMI NAGAR

NEAR ANNAMAIAH CIRCLE,

TIRUPATI.

PH NO. 9440025125
KINEMATICS
THEORY OF KINEMATICS
Kinematics : The study of the motion of an object without taking into consideration cause of its motion is called
kinematics.

NOTE : The word kinematics comes from the Greek word Kinema which means
motion. The word dynamics comes from the Greek word dynamics which means power.

BASIC DEFINITIONS
Distance and Displacement
Suppose an insect is at a point A(x1, y1, z1) at t = t1. It reaches at point B(x2, y2, z2) at t = t2 through path ACB
with respect to the frame shown in fig. The actual length of curved path ACB is the distance travelled by the
insect in time t = t2 t1.
Y C
A B
rA rB

X
Z

If we connect point A (initial position) and point B (final position) by a straight line, then the length of straight line
AB gives the magnitude of displacement of insect in time interval t = t2 t1.
The direction of displacement is directed from A to B through the straight line AB from the concept of vector,

the position vector of A is r A = x1i + y1j+ z1k and that of B is r B = x 2i + y 2j+ z 2 k .
According to addition law of vectors,

r A + AB = r B

AB = r B - r A

AB = ( x 2 - x1 ) i + ( y 2 - y1 ) j+ ( z 2 - z1 ) k
The magnitude of displacement is
2 2 2
| AB |= ( x 2 - x1 ) + ( y 2 - y1 ) + ( z 2 - z1 )

NOTE : Distance covered by the body is always equal to or greater


than its magnitude of displacement.

Example 1. A man walks 3m in east direction, then 4m in north direction. Find distance covered and the
displacement covered by man.
Sol. The distance covered by man is the length of path = 3m + 4m = 7m. N
Let the man starts from O and reaches finally at B (shown in figure). B

OB represents the displacement of man. From figure, 4m
2 2
| OB |= ( OA) + ( AB) O 3m A
2 2
W E
| OB |= (3m ) + (4 m ) = 5 m

KINEMATICS www.physicsashok.in 1
KINEMATICS

4m 4
and tan q = =
3m 3
4
q = tan- 1
3

- 1 4
The displacement is directed at an angle tan north of east.
3
Average Speed and Average Velocity
Suppose we wish to calculate the average speed and average velocity of the insect (in section (i)) between
t = t1 and t = t2. From the path (shown in fig.) we see that at t = t1, the position of the insect is A(x1, y1, z1) and
at t = t2, the position of the insect is B(x2, y2, z2).
Y C
A B
rA rB

X
Z

The average speed is defined as total distance travelled by a body in a particular time interval divided by the
time interval. Thus, the average speed of the insect is
The length of curve ACB
v av =
t 2 - t1
The average velocity is defined as total displacement of the body in a particular time interval divided by the time
interval.
Thus, the average velocity of the insect in the time interval t2 t1 is

AB
v av =
t - t
2 1
rB - rA
v av =
t 2 - t1
( x - x1 ) i + ( y2 - y1 ) j+ ( z 2 - z1 ) k
v av = 2
t 2 - t1
Example 2. In one second a particle goes from point A to point B moving in a semicircle (fig.).
Find the magnitude of average velocity. A
AB
Sol. | vav |=
Dt 1.0m
2.0
| vav |= m/s
1.0
B
| vav |= 2 m / s

Example 3. A particle goes from A to B with a speed of 40 km/h and B to C with a speed of 60 km/h. If AB =
6BC the average speed in km/h between A and C is _______
Sol. AB = 40t1 ...(1)
BC = 60t2 ...(2)

KINEMATICS www.physicsashok.in 2
KINEMATICS
total distance travelled
Average speed =
time taken
AB + BC
Vav =
t1 + t 2
From eqn. (1) and (2) A B C
40t1 + 60t 2
Vav = ...(3)
t1 + t 2
According to question
AB = 6BC
40t1 = 6 60t2 From eqn (1) and (2)
t1 = 9t2
40 9t 2 60t 2
From eqn (3) Vav from eqn (3)
9t 2 t 2
420t 2
Vav = Vav = 42 km / h
10t 2

Instantaneous Velocity
Instantaneous velocity is defined as the average velocity over smaller and smaller interval of time.

Suppose position of a particle at t is r and at t + t is r + Dr . The average velocity of the particle for time

Dr
interval t is v av = .
Dt
From our definition of instantaneous velocity, t should be smaller and smaller. Thus, instantaneous velocity is

Dr dr
v = lim =
Dt 0 Dt dt


Example 4. Let at any time t, the position vector of a particle is r = x i + y j+ z k . Find the velocity of the particle.
dx
Sol. x-component of velocity, v x =
dt
dy
y-component of velocity, v y =
dt
dz
z-component of velocity, vz =
dt
Thus, velocity of particle

v = v x i + v y j+ v 2 k
dx
v= i + dy j+ dz k
dt dt dt

Average and Instantaneous Acceleration


In general, when a body is moving, its velocity is not always the same. A body whose velocity is increasing is
said to be accelerated.
Average acceleration is defined as change in velocity divided by the time interval.

Let us consider the motion of a particle. Suppose that the particle has velocity v1 at t = t1 and at a later time

KINEMATICS www.physicsashok.in 3
KINEMATICS

t = t2 it has velocity v 2 . Thus, the average acceleration during time interval t = t2 t1 is

v 2 - v1 Dv
a av = =
t 2 - t1 Dt
If the time interval approaches to zero, average acceleration is known as instantaneous acceleration.
Mathematically,

Dv dv
a = lim =
Dt 0 Dt dt


Example 5. The velocity of a point depends on time t, as v = c + t b where c and b are constant vectors.
Find the instantaneous acceleration at any time t.
Sol. Acceleration at any time t,

dv d
a= = (c + t b )
dt dt

a = 0+ b = b

IMPORTANT FEATURES
1. If a body is moving continuously in a given direction on a straight line, then the magnitude of displacement is
equal to distance.
2. Generally, the magnitude of displacement is less or equal to distance.
3. Many paths are possible between two points. For different paths between two points, distances are different
but magnitudes of displacement are same.
4. The slope of distance-time graph is always greater or equal to zero.
5. The slope of displacement-time graph may be negative.
6. If a particle travels equal distances at speeds v1, v2, v3, ...... etc. respectively, then the average speed is
harmonic mean of individual speeds.
2v1 v 2
7. If a particle moves a distance at speed v1 and comes back with speed v2, then v av =
v1 + v 2

but v av = 0
8. If a particle moves in two equal intervals of time at different speeds v1 and v2 respectively, then
v + v2
v av = 1
2
9. The average velocity between two points in a time interval can be obtained from a position versus time graph
by calculating the slope of the straight line joining the co-ordinates of the two points.

x2 x2 B
x1 x1 A
( x2 x1)
( t 2 t1)

t1 t2 t1 t2
(a) (b)
The graph [shown in fig.], describes the motion of a particle moving along x-axis (along a straight line).
Suppose we wish to calculate the average velocity between t = t1 and t = t2. the slope of chord AB [shown in
fig.(b)] gives the average velocity.

KINEMATICS www.physicsashok.in 4
KINEMATICS
x 2 - x1
Mathematically, v av = tan q =
t 2 - t1
10. If a body moves with constant velocity, the instantaneous velocity is equal to average velocity. The instantaneous
speed is equal to modulus of instantaneous velocity.
11. x-component of displacement is Dx = v dtx

y-component of displacement is Dy = v dty

z-component of displacement is Dz = v dt
z

Thus, displacement of particle is



Dr = Dx i + Dy j+ Dz k
dx
12. If particle moves on a straight line, (along x-axis), then v = .
dt
13. The area of velocity-time graph gives displacement.
14. The area of speed-time graph gives distance.
15. The slope of tangent at position-time graph at a particular instant gives instantaneous velocity at that instant.
16. The slope of velocity-time graph gives acceleration.
17. The area of acceleration-time graph in a particular time interval gives change in velocity in that time interval.

ONE, TWO AND THREE DIMENSIONAL MOTION


One Dimensional Motion
If only one of the three co-ordinates is required to specify the position of an object in space changing w.r.t.
time, then the motion of the object is called one dimensional motion. Motion of a particle in a straight line can
be described by only one component of its velocity and acceleration. For example, motion of a block in a
straight line, motion of a train along a straight track, a man walking on a level and narrow road, an object falling
under gravity, etc.

Two Dimensional Motion


If two of the three co-ordinates are required to spacify the position of an object on space changing w.r.t. time,
then the motion of the object is called two dimensional motion. The motion of a particle through its vertical
plane at some angle with horizontal. ( 90) is an example of two dimensional (2D) motion.
This is a projectile motion. Similarly, a circular motion is an example of 2D motion. A 2D motion takes place
in a plane and its velocity (or acceleration) can be described by two components in any two mutually
perpendicular directions (vx and vy).

Three Dimensional Motion


If all the three co-ordinates are required to specify the position of an object in space changing w.r.t. time, then
the motion of an object is called three dimensional motion. Such a motion is not restricted to a straight line or
plane but takes place in space. In a 3D motion velocity and acceleration of a particle can be resolved in three
components (vx, vy, vz, ax, ay, az). A few examples of 3D motion are a flying bird, a flying kite, a flying
aeroplane, the random motion of gas molecules, etc.

UNIFORMLY ACCELERATED MOTION


A motion, in which change in velocity in each unit of time is constant, is called uniformly accelerated motion. So,
for uniformly accelerated motion, acceleration is constant or approximately. So for uniformly accelerated motion

( a = constant) , equations of motion are as under,,

v = u+ a t ...(i)

KINEMATICS www.physicsashok.in 5
KINEMATICS
1
s = u t + a t2 ...(ii)
2

and v . v = u . u + 2a . s ...(iii)

where u = initial velocity of particle

v = velocity at time t

s = displacement of particle at time t
If motion is described in one dimension, so vector sign () need not to be used.
Normally, vertical upward motion is taken negative and vertical downward motion is taken positive. Similarly,
for horizontal rightward motion is taken positive and leftward motion is taken negative.
Sign convention : Sign convention for (a) motion in vertical (b) motion in horizontal, is shown in Fig.

+ve ve +ve ve
(a) (b)

Example 6. At a distance L = 400m from the traffic light brakes are applied to a locomotive
moving at a velocity v = 54 km/hr. Determine the position of the locomotive relative to the traffic light 1 min after
the application of the brakes if its acceleration is 0.3 m/sec2.
5
Sol. u 54 15 m / s
18
a = 0.3 m/s2
v = u + at
0 = 15 0.3 t0
15
t0 = = 50 sec
0.3
After 50 second, locomotive comes in rest permanently.
v2 = u2 + 2as
O2 = 152 2 0.3 S0
225 2250
S0 = = = 375m
0.6 6
the distance of the locomotive from traffic light
= 400 375 = 25 metre

Example 7. A car moves in the xy plane with acceleration (3i + 4 j) m / s 2 .


(a) Assuming that the car is at rest at the origin at t = 0, derive expression for the velocity as function of time.
(b) Find the equation of path of car and find the position vector as function of time.
Sol. Here, ux = 0, uy = 0, uz = 0
2 2
ax = 3 m/s , ay = 4 m/s
(a) vx = ux + axt
or vx = 3t
and vy = uy + ayt

KINEMATICS www.physicsashok.in 6
KINEMATICS
or vy = 4t

v = v x i + v y j

v = (3ti + 4tj)
1 2
(b) x = uxt + a t
2 x
1 3
or x= 3t 2 = t 2
2 2
1 2
and y= uyt + a yt
2
1
or y= (4)t 2 = 2t 2
2
2 4 3 2
y= 2 x = x x= t
3 3 2
4
y= x
3
Hence, the path is straight line.
The position of car is
3
r = x i + y j = t 2 i + 2t 2 j
2

Motion Under Gravity


The most familiar example of motion with costant acceleration on a straight line is motion in a vertical direction
near the surface of earth. If air resistance is neglected, the acceleration of such type of particle is gravitational
acceleration which is nearly constant for a height negligible with respect to the radius of earth. The magnitude of
gravitational acceleration near surface of earth is g = 9.8 m/s2 = 32 ft/s2.
Discussion :
Case-I : If particle is moving upwards :
In this case applicable kinematics relations are : g
v = u gt ...(i)
1
h = ut - gt 2 ...(ii) u
2
v2 = u2 2gh ...(iii)
Here, h is the vertical height of the particle in upward direction.

NOTE : For maximum height attained by projectile


h = hmax, v=0
2 2
i.e., (0) = u 2ghmax
u2
hmax =
2g

Case-II : If particle is moving vertically downwards : u


In this case :
v = u + gt ...(i) g
v2 = u2 + 2gh ...(ii)

KINEMATICS www.physicsashok.in 7
KINEMATICS
1 2
h = ut + gt ...(iii)
2
Here, h is the vertical height of the particle in downward direction.

Example 8. A particle is projected vertically upwards with velocity 40 m/s. Find the displacement and distance
travelled by the particle in
(a) 2s (b) 4s (c) 6s [Take g = 10 m/s2]
Sol. Here, u is positive (upwards) and a is negative (downwards). So, first we will find t0, the time when velocity
becomes zero.
u 40
t0 = = = 4s
a 10
(a) t < t0. Therefore, distance and displacement are equal.
1 2
d = s = ut + at
2
1
d = 40 2 - 10 4 = 60 m
2
(b) t = t0. So, again distance and displacement are equal.
1
d = s = ut + at 2
2
1
d = 40 4- 10 16 = 80 m
2
(c) t > t0. Hence, d > s
1
s = 40 6 - 10 36 = 60 m
2
u2 1 2
While d= + a (t - t 0 )
2a 2
(40)2 1
d= + 10 (6 - 4)2 = 100 m
2 10 2

Example 9. A ball is thrown upwards from the ground with an initial speed of u. The ball is at a height of 80 m at two
times, the time interval being 6s. Find u. Take g = 10 m/s2.
Sol. Here, u = u m/s, a = g = 10 m/s2 and s = 80 m.
1
Substituting the value in s = ut + at 2 , we have
2
2
80 = ut 5t
or 5t2 ut + 80 = 0
u+ u 2 - 1600
or t=
10 +ve
s = 80 m
u- u 2 - 1600
and t=
10 ve u
Now, it is given that

KINEMATICS www.physicsashok.in 8
KINEMATICS

u+ u 2 - 1600 u - u 2 - 1600
- =6
10 10
u 2 - 1600
or =6 or u 2 - 1600 = 30
5
or u2 1600 = 900
u2 = 2500
or u = 50 m/s
Ignoring the negative sign, we have
u = 50 m/s
A
C
Example 10. A disc arranged in a vertical plane has two groves of same length directed 60
along the vertical chord AB and CD as shown in the fig. The same particles slide
down along AB and CD. The ratio of the time tAB/t CD is : D
(A) 1 : 2 (B) 1 : 2 (C) 2 : 1 (D) 2 : 1
B
1 2
Sol. SAB = g t AB
2
1
SCD = g cos 60 t 2CD
2 A
C
But SAB = SCD 60
60
1 2 60
g t gcos g
SAB 2 AB
= D g
SCD 1 g cos 60 t 2
CD
2
t 2AB t AB B
or 1 = 2 = 1 : 2
t 2CD t CD

Example 11. A stone is dropped from a height h. Simultaneously another stone is thrown
up from the ground with such a velocity that it can reach a height of 4h. Find the time when two stones cross
each other.
Sol. For second stone,
v 2 = v02 - 2g (4h)
0 2 = v 20 - 8gh
v 0 8gh
but they meth at height H in time t0. v=0
Displacement of 1st stone is
1
h H gt 20 ...(1) (1)
2 4h
(2)
and that of second stone is h
1 v0
H = v0 t 0 - gt 02 ...(2)
2
After solving eqn (1) and (2)
1 1
or h v 0 t 0 g t 02 g t 02
2 2
h = v0t0

KINEMATICS www.physicsashok.in 9
KINEMATICS

h h h
t0 = = t0 =
v0 8gh 8g

Example 12. A rocket is launched at an angle 53 to the horizontal with an initial speed of 100 ms1. It
moves along its initial line of motion with an acceleration of 30 ms2 for 3 seconds. At this time its engine
fails & the rocket proceeds like a free body. Find :
(i) the maximum altitude reached by the rocket
(ii) total time of flight.
Sol. S0 = ut + at 2
= 100 3 + 30 9
= 300 + 135 = 435 m
In OAB
h
sin 53 =
S0
4
h S0 sin 53 435 87 4 348 m
5
v0 = u + at = 100 + 30 3 = 190 m/s (velocity at the time of switch off)
After engin switch off
3
v0x v0 sin 37 190 114 m / s
5
4
v0y v0 cos37 190 152 m / s
5
ay = 10 m/s2, ax = 0
(i) At maximum altitudes vy = 0
v 2y = v 20y + 2a y h 0
37 v0
0 = v 20y + 2a y h 0 A
S0 Engin fail x
v20y 37 h
h0 u
2a y
53
O B
152 152
h0
2 10
11552
h0 = h 0 = 1155.2 m
10
The maximum altitude reached by the rocket is
= h0 + h

= (1155.2 + 348) m 1503.2 m

(ii)Total time of flight.


1
y = v0 y t + a y t 2
2
1
- h = v 0y t 0 + a y t 02
2
1
1155.2 152t 0 10 t 20
2

KINEMATICS www.physicsashok.in 10
KINEMATICS

- 1155.2 = 152t 0 - 5t 20
5 t 20 - 152 t 0 - 1155.2 = 0
t 02 - 30.4 t 0 - 231.04 = 0
t 0 = 35.54 sec.

NON-UNIFORMLY ACCELERATED MOTION


When motion of a particle is not uniform i.e., acceleration of particle is not constant or acceleration is a function
of time, then following relations hold for one dimensional motion :
ds
(i) v=
dt
dv dv
(ii) a= = v
dt ds
(iii) ds = vdt and
(iv) dv = adt or vdv = ads
Such problems can be solved either by differentiation or integration applying some boundary conditions.

Example 13. A particle is moving with a velocity of v = (3 + 6t + 9t2) cm/s. Find out :
(a) the acceleration of the particle at t = 3 s.
(b) the displacement of the particle in the interval t = 5 s to t = 8 s.
Sol. (a) Acceleration of particle
dv
a= = ( 6 + 18t ) cm / s 2
dt
At t = 3 s,
a = (6 + 18 3) cm/s2
a = 60 cm/s2
(b) Given, v = (3 + 6t + 9t2) cm/s
ds
or = (3+ 6t + 9t 2 )
dt
or ds = (3 + 6t + 9t2)dt
8 8
5
ds = (3+ 6t + 9t ) dt
5
2

8
s = 3t + 3t 2 + 3t 3 5 or s = 1287 cm

Example 14 : The motion of a particle along a straight line is described by the function x = (2t 3)2 where x is in
metres and t is in seconds.
(a) Find the position, velocity and acceleration at t = 2 s.
(b) Find the velocity of the particle at origin.
Sol. (a) Position, x = (2t 3)2
dx
Velocity, v 4 2t 3 m / s
dt
dv
and acceleration, a = = 8 m / s2
dt

KINEMATICS www.physicsashok.in 11
KINEMATICS
At t = 2 s,
x = (2 2 3)2 = 1.0 m
v = 4(2 2 3) = 4 m/s and a = 8 m/s2
(b) At origin, x = 0
or (2t 3) = 0
v=40=0

EQUATIONS OF MOTION FOR VARIABLE ACCELERATION


Case-I : When acceleration a of the particle is a function of time :
Since, acceleration of a particle is a function of time, i.e., a = f(t)
dv
= f (t)
dt
dv = f(t)dt
Integrating within the proper limits, we get
t
v = u+ 0 f (t)dt
Case-II : When acceleration a of the particle is a function of distance :
Since, acceleration of a particle is a function of distance, i.e., a = f(x)
dv
= f (x)
dt
dv dx
. = f (x)
dx dt
vdv = f(x)dx
Integrating with in proper limits, we get
x
v 2 = u 2 + 2 f (x)dx
x0

Case-III : When acceleration a of the particle is a function of velocity :


Since, acceleration of a particle is a function of velocity, i.e., a = f(v)
dv
= f (v)
dt
dv
dt =
f (v)
Integrating within proper limits, we get
v dv
t= u f (v)
Therefore, we get v as a function of t i.e., v(t).
dv
Again = f (v)
dt
dv dx
. = f (v)
dx dt
dv
v = f (v)
dx
vdv
dx =
f (v)

KINEMATICS www.physicsashok.in 12
KINEMATICS
x v vdv
x 0
dx = u f (v)
v vdv
x - x0 = u f (v)
v vdv
x = x0 + u f (v)
Therefore, we get x as a function of v i.e., x(v).

GRAPHS IN ONE DIMENSIONAL MOTION


The tabular forms of st and vt graphs are given for one-dimensional motion with uniform velocity or with
constant acceleration.
Table-1 : Displacement-Time Graph
S.No. Different Cases st Graph The main Features of Graph
s

1. At rest Slope = v = 0
t
s

2. Uniform motion s = ut s = 0 at t = 0
t
s

1
3. Uniformly accelerated motion s = 2 at 2 u = 0 i.e., slope of st graph at t = 0,
with u = 0, s = 0 at t = 0 t should be zero.
s

1
4. Uniformly accelerated motion s = ut + 2 at2 Slope of st graph gradually goes on
with u 0 but s = 0 at t = 0 t increasing.
s

s0
5. Uniformly accelerated motion s = s0 + ut + 1
2 at
2
s = s0 at t = 0
with u 0 but s = s0 at t = 0 t
s

6. Uniformly retarded motion At t = t0, slope of st graph becomes


t zero.
t0
Table-2 : Velocity-Time Graph
S.No. Different Cases vt Graph The main Features of Graph
v v = constant
a=0
1. Uniform motion (i) Slope of vt graph = a = 0
t (ii) v = constant

KINEMATICS www.physicsashok.in 13
KINEMATICS
v

2. Uniformly accelerated motion v = at (i) u = 0 i.e., v = 0 at t = 0


with u = 0, s = 0 at t = 0 t (ii) a or slope of vt graph is constant.
v
v = u + at
u
3. Uniformly accelerated motion At t = 0, v 0 and slope of vt graph
with u 0 but s = 0 at t = 0 t is not zero.
v
v = u + at
u
4. Uniformly accelerated motion v = u at t = 0
with u 0 but s = s0 at t = 0 t
v

u v = u at
5. Uniformly decelerated motion Slope of vt graph = a (retardation)
t
t0
v

6. Non-uniformly accelerated motion Slope of vt graph increases with


t time

Acceleration-Time Graph
(i) When at graph is a straight line parallel to time-axis, then acceleration = a = constant.
a Slope = 0

t
(a)
(ii) When at graph is a straight line passing through origin, then acceleration of particle is increasing uniformly.
a
e
+v
e=
op
Sl
t
(b)
(iii) When at graph is a straight line of negative slope, then acceleration is decreasing uniformly.
a
Sl
op
e=
v
e
t
(c)

KINEMATICS www.physicsashok.in 14
KINEMATICS
v
Example 15. The fig. shows the vt graph of a particle moving in straight line. 20
Find the time when particle returns to the starting point. 10

Sol. When the particle comes at initial position, total displacement is zero.
25 t
Since, the area of v t graph gives displacement. In this case area of 10 20
v t graph should be zero.
1 1
S 20 25 t 25 v 0
2 2
v0
or 0 250 t 25
2
500
or t 25 v ...(1)
0
v
20 v
Also tan 0 from figure 20
25 20 t 25
v0
or 4
t 25
v 0 4 t 25 ...(2) 20 25 t
From eqn (1) and (2) v0
500 125
t 25
4 t 25 t 25

t 25 2 125
t 25 125 5 5
t = (25 + 5 5 ) sec. t = 36.2 sec

V
Example 16. From the velocity-time plot shown in figure, find the distance 5m/s
travelled by the particle during the first 40 seconds. Also find the average t(s)
velocity during this period. 20 40
Sol. The distance fravelled by the particle during the first 20 second. 5m/s
1
S1 v t
2
1
S1 5 20 S1 = 50 m
2
The distance travelled by the partile during next 20 second is
1
S2 5 20
2
S2 = 50 m
Since distance is a sclar quantity therefore
total distance = S1 + S2 = 50 + 50
S = 100 m

Example 17. A ball is dropped from a height of 80m on a floor. At each collision the ball losses half of its speed.
Plot the speed-time graph and velocity-time graph of its motion till two collisions with the floor,
[Take g = 10 ms2].

KINEMATICS www.physicsashok.in 15
KINEMATICS
Sol. The time in first collision
1 2
h = ut + gt (during downward motion)
2
1
or 80 = 0 + 10 t 2 ( u = 0)
2
2 80
or t= = 4s
10
Final speed just before first collision
v = 0 + 10 4 = 40 m/s
40
It now loses half of initial speed after the collision i.e., when it first bounces its initial speed is 20 = m / s.
2
So, the time is loosing half of its speed.
0 = 20 10 t (during upward motion)
20
t= = 2s (final speed = 0)
10
In 2 s, it attains height
1
h= 20 2 - 10 (2) 2
2
h = 40 20 = 20 m
Now, it is dropped again from 20m with zero initial speed. Time taken in reaching the ground
1
20 = 0 + 10(t) 2
2
t=2s
Also final speed v2 = 0 + 2 10 20 (from v2 = u2 + 2gh)
v = 20 m/s
Thus, with the above data, we can draw the speed-time graph.
Speed (m/s)

40
20
Time(s)
4 6 8
Since, velocity is a vector quantity so from the above graph, we can now draw the velocity-time graph.
[Take downward motion positive and upward motion negative in case of vt graph]
Velocity (m/s)

40
20

4 Time(s)
6 8
20

Example 18. The velocity-time graph of an object moving along a straight line is as shown in the fig.
Calculate the distance covered by the object :
20 ms1 A B

v
O A B C
0 2 5 10
t (s)
KINEMATICS www.physicsashok.in 16
KINEMATICS
(a) between t = 0 to t = 5 s and
(b) between t = 0 to t = 10 s.
Sol. (a) Let x1 be the distance covered in the time interval between t = 0 to t = 5 s. Then, x1 = area of the trapezium
OABB
AB + OB 3+ 5
= AA= 20 = 80 m
2 2
(b) Let x2 be the distance covered in the time interval between t = 0 to t = 10 s. Then, x2 = area of the
trapezium OABC.
AB + OC 3 + 10
= AA= 20 = 130 m
2 2

IMPORTANT FEATURES
1. For a particle having zero initial velocity if v t, s t2 and v2 s then acceleration of particle must be
constant i.e., particle is moving rectilinearly with uniform acceleration.
2. For a particle having zero initial velocity if s t, where > 2, then particles acceleration increases with time.
3. For a particle having zero initial velocity if s t, where < 0, then particles acceleration decreases with time.
4. When a body is non-uniformly accelerated, then problem can be solved either be differentiation or integration
(applying some boundary conditions).
Differentiation : s t v t a t
Integration : atvtst
By boundary condition we mean that velocity or displacement at some time (usually at t = 0) should be known
to us. Otherwise we cannot find constant of integration.
dv
5. Equation a = v or vdv = ads is useful when acceleration displacement equation is known and velocity
ds
displacement equation is required.

6. When either u = 0 or u a , motion is only accelerated.

7. It can be observed when either u = 0, u a or u a .

8. When u a motion is first retarded (till the velocity becomes zero) and then accelerated in opposite direction.
9. Following are few points we may conclude in case of a one dimensional motion :
ds
(a) slope of displacement-time graph gives velocity as v = .
dt
dv
(b) slope of velocity-time graph gives acceleration as a = .
dt
(c) area under velocity-time graph gives displacement as (ds = vdt).
(d) area under acceleration-time graph gives change in velocity (as dv = adt).
(e) displacement-time graph in uniform motion is a straight line passing through origin, if displacement is zero at
time t = 0 (as s = vt).
(f) velocity-time graph is a straight line passing through origin in a uniformly accelerated motion if initial velocity
u = 0 and a straight line not passing through origin if initial velocity u 0 (as v = u + at).
1 2
(g) displacement-time graph in uniformly accelerated or retarded motion is a parabola as s = ut at .
2
10. Slopes of vt or st graphs can never be infinite at any point, because infinite slope of vt graph means infinite
acceleration. Similarly, infinite slope of st graph means infinite velocity. Hence, the following graphs are not
possible :

KINEMATICS www.physicsashok.in 17
KINEMATICS

v s

t t

11. At one time, two values of velocity or displacement are not possible. Hence, the following graphs are not
acceptable :
v s

v1 s1
v2 s2

t0 t t0 t

RELATIVE MOTION
If the velocities of two bodies are known w.r.t a common frame of reference, then the velocity of a body can be

measured w.r.t. the second body. Therefore, if the velocities of two bodies A and B w.r.t. the ground are v A

and v B then the relative velocity of A w.r.t. B is

v AB = v A - v B
Similarly, we see that

v AB = - vBA
Also, relative acceleration of A w.r.t. B is

and a AB = a A - a B

a AB = - a BA

Example 19. Car A has an acceleration of 6 m/s2 due east and car B, 8 m/s2 due north. What is the acceleration of
car B w.r.t. to car A ?
Sol. It is a two dimensional motion.
aBA aB = 8 m/s 2
Therefore, a BA = acceleration of car B w.r.t. car A
N
= aB - aA
W E
a BA = (6) 2 - (8)2 = 10 m / s 2 S
8 4
and a = tan- 1 = tan- 1 aA = 6 m/s 2
6 3
-1 4
Thus, a BA is 10 m/s2 at an angle of = tan from west towards north.
3
Discussion

(a) If a satellite is moving in equatorial plane with velocity v and a point on the surface of earth with velocity u
relative to the centre of earth, the velocity of satellite relative to the surface of earth

vSE = v - u

(b) If a car is moving at equator on the earths surface with a velocity v CE relative to earths surface and a point on
the surface of earth with velocity vE relative to its centre, then

v CE = vC - v E
(c) If the car moves from west to east (the direction of motion of earth)

KINEMATICS www.physicsashok.in 18
KINEMATICS

vC = vCE + vE
and if the car moves from east to west (opposite to the motion to earth)
vC = vCE vE
(d) For crossing the river in shortest time, the boat should sail perpendicular to
the flow. If the width of river is d and v the velocity of boat in still water, B vr C
then, vr
d v
t=
v
The position of boat at the other bank is C (not B). O

The displacement of the boat = OC = OB + BC

OC = (OB) 2 + (BC) 2

2 2 d 2
2
OC = d + (v r t) = d + v r
v
(e) For crossing the river in shortest distance, the boat moves as such its horizontal component of velocity balances
the speed of flow.
A vr B
OB = the shortest path = d
v vr
vr = v sin sin = r v
v
cos = 1 sin 2 O
v 2 v 2 - v 2
cos = 1 r = r

v v
d d
t= =
v cos q v v 2 - v 2
r
v
d
t=
v - v 2r
2

In this case, the magnitude of displacement = d.


(f) If boat crosses the river along the shortest path, then time is not least.

Example 20. A police van moving on a highway with a speed of 30 km/h fires a bullet at a thiefs car speeding away
in the same direction with a speed of 192 km/h. If the muzzle speed of the bullet is 150 m/s, with what speed
does the bullet hits the thiefs car ?

Sol. Let v p , v b and v t represent velocity of the police van, muzzle velocity of bullet and the velocity of the thiefs
s
car respectively. This is a one dimensional motion.
25
v p = 30km / h = m/s
3
v b = 150 m / s
160
and v t = 192 km / h = m/s
3
Since, bullet is fired from the moving police van, the effective velocity of the bullet will be
25 475
vt = v b + v p = 150 + = m/s
3 3

KINEMATICS www.physicsashok.in 19
KINEMATICS

if the relative velocity of the bullet w.r.t the thiefs car is v bt , then

v bt = vb - v t
475 160 315
v bt = - = = 105 m / s
3 3 3

Example 21. A, B & C are three objects each moving wit constant velocity. As speed is 10m/sec in a

direction PQ . The velocity of B relative to A is 6 m/sec at an angle of cos1(15/24) to PQ. The velocity

of C relative to B is 12 m/sec in a direction QP , then find the magnitude of the velocity of C.

Sol. v A = 10 i

v B 6 cos i 6sin j
15 351
Here cos , sin
24 24

And v CB = - 12 i

v BA = v B - vA

or 6cos i 6sin j v B 10 i

v B cos i 6sin j 10 i
15 351
6 i6 j 10 i
24 24
15 351
= 4 10 i 4 j

55 351
i j
4 4

But v CB = vC - v B
55 351
v C = vCB + v B = - 12 i + i + j
4 4
7 351
= i + j
4 4
2 2
7 351
vC
4 4

49 + 351 400 20
= = = Ans. 5 m/s
4 4 4

Example 22. A man crosses a river in a boat. If he crosses the river in minimum time he takes 10 min with a drift
120 m. If he crosses the river taking shortest path, he takes 12.5 min, find :
(a) width of the river
(b) velocity of the boat with respect to water, B
(c) speed of the current.
Sol. Let vr = velocity of river v d
v = velocity of river in still water and
A
d = width of river For minimum time

KINEMATICS www.physicsashok.in 20
KINEMATICS

Given, tmin = 10 min


d
or = 10 ...(i)
v
Drift in this case will be,
x = vrt
120 = 10vr ...(ii)
Shortest path is taken when vb is along AB. In this case,

vb = v 2 - v 2r

d d vr B
Now, 125 = = ...(iii)
vb v 2 - v 2r v vb
Solving these three equations we get,
v = 20 m/min, vr = 12 m/min and d = 200 m. A
Shortest path

Example 23. A man with some passengers in his boat, starts perpendicular to flow of river 200m wide and
flowing with 2m/s. boat speed in still water is 4m/s. When he reaches half the width of river the passengers
asked him they want to reach the just opposite end from where they have started.
(a) Find the direction due to which he must row to reach the required end.
(b) How many times more total time, it would take to that if he would have denied the passengers.
Sol. (a) Event (1) From A to B,
Time taken by boat to recent from A to B is
100 100
t= = = 25sec.
vy 4
Also, AD = vx t = 2t = 50 m
Event (2) From B to C,
Actual velocity of boat should be along BC. This actual velocity is found by resultant of vrel and vr.

v rel = v b - vr
xcomput of actual velocity is
y
vx = vr vrel sin = 2 4sin C E
and y comput of actual velocity is vb
vy = vrel cos vrel B 200m
vrel
The time taken to go from B to C is vr
A D
BE
t0 =
vy
100 100

v rel cos 4 cos
25

cos
Also, EC = vx t 0
= (2 4 sin ) t 0
25
4 sin 2
cos
But EC = AD = 50 m

KINEMATICS www.physicsashok.in 21
KINEMATICS
25
EC 4 sin 2
cos
25
50 4 sin 2
cos
50 cos = 100 sin 50
cos = 2 sin 1
1 + cos = 2 sin

2 cos 2 4sin cos
2 2 2
a 1
tan =
2 2
1
tan 1
2 2
1
2 tan 1
2
(b) If boat crosses the river with initial condition
200 200
t1 = =
vy vrel
200
t1 = = 50sec.
4
If boat cross the river with final condition,
t 2 = t AB + t BC
t2 = t + t0
25
t 2 25
cos
1
tan
2 2
2
cos
2 5

cos 2 cos 2 1
2
4
2 1
5
8 3
= - 1=
5 5
25 125 200
t 2 = 25 + = 25 + =
3 3 3
5
t2 200 4 t2 4
=
t1 3 50 3 t1 3

Example 24. To a person going west wards with a speed of 6 km/h rain appears to fall vertically downwards with
a speed of 8 km/h. Find the actual direction of rain.

Sol. Let v M = velocity of man = 6 km/h

v = relative velocity of rain w.r.t. man = 8 km/h
KINEMATICS www.physicsashok.in 22
KINEMATICS
Vertical
v R = actual velocity of rain N
In this case

v = v R + (- v M ) vM O
W E
v = vR - vM A
v
vR B
or vR = vM + v C S

v R = (6) 2 + (8)2 = 10 km / h S

The velocity of rain ( v R ) is given by the vector OC , the resultant of vectors OA and OB as shown in figure.

If is the angle that v R makes with the vertical, then

BC | v M | 6
tan 0.75
OB | v | 8
or = 36 52 (east of vertical)

Example 25. Rain is falling vertically with a speed of 20 ms1 relative to air. A person is running in the rain
with a velocity of 5 ms1 and a wind is also blowing with a speed of 15 ms1 (both towards east). Find the
angle with the vertical at which the person should hold his umbrella so that he may not get drenched.

Sol. v ra = - 20 k z
N
v m = 5 i

v a = 15 i W x
E

v ra = v r - v a
S
v r = - 20 k + 15 i

v rm = v r - v m
= - 20k +15i - 5i
= - 20 k + 10 i
10 1
tan
20 2
1
tan1
2
vrm
vertically

Example 26. An aircraft flies at 400 km/h in still air. A wind of 200 2 km / h is blowing from the south. The pilot
wishes to travel from A to a point B north east of A. Find the direction he must steer and time of his journey if
AB = 1000 km.

Sol. Given that vw = 200 2 km / h . vaw = 400 km/h and v a should be along AB or in north-east direction. Thus,

the direction of v aw should be such as the resultant of v w and v aw is along AB or in north-east direction.
N
B
va 45 vw = 200 2 km/h
45 C
A vaw = 400 km/h
E

KINEMATICS www.physicsashok.in 23
KINEMATICS

Let v aw makes an angle with AB as shown in fig. Applying sine law in triangle ABC, we get
AC BC

sin 45 sin
BC
or sin sin 45
AC
200 2 1 1
sin
400 2 2
= 30
therefore, the pilot should steer in a direction at an angle of (45 + ) or 75 from north towards east.

| va | 400
Further,
sin(180 45 30 ) sin 45
sin105 km
or | v a | (400)
sin 45 h
cos15 km 0.9659 km
| va | (400) (400)
sin 45 h 0.707 h
km
| va | 546.47
h
The time of journey from A to B is
AB 1000
t h t = 1.83 h
| va | 546.67

Example 27. A glass wind screen whose inclination with the vertical can
be changed, is mounted on a cart as shown in figure. The cart moves
uniformly along the horizontal path with a speed of 6 m/s. At what v=6m/s
maximum angle to the vertical can the wind screen be placed so
that the rain drops falling vertically downwards with velocity 2 m/s,
do not enter the cart ?
D vc
Sol. v c = 6 i x

v r = 2 j
vr
v rc = v r - vc
v 6 vrc y
tan c 3
vr 2
AE
cos

AE cos
ED sin A
tan cos
BE cos E
sin
D
sin
3 B
2sin 2 / 2 vr C

2sin cos
3 2 2 cot
2
2sin 2
2

KINEMATICS www.physicsashok.in 24
KINEMATICS

or cot 3
2
1
or tan
2 3
1
or tan 1
2 3
1
2 tan 1
3

MOTION IN TWO AND THREE DIMENSIONS


INTRODUCTION
A body is free to move in space. In this case, the initial position of body is taken as origin.
Any convenient co-ordinate system is chosen. Let us suppose that at an instant t, the body is at point P(x, y, z).

The position vector of the body is r = x i + y j+ z k . Thus, velocity

dr dx
v= = i + dy j+ dz k
dt dt dt dt
dx
The velocity along x-axis is, v x =
dt
dv x
and acceleration along x-axis is a x = .
dt
dy
The velocity along y-axis is v y =
dt
dv y
and the acceleration along y-axis is a y =
dt
dz dv z
Similarly, v z = and a z =
dt dt

The acceleration of the body a = a i + a j+ a k .
x y z

Discussion
(a) If ax is constant,
vx = ux + axt
1
x = uxt + axt2
2
v 2x = u 2x + 2a x x
If ax is a variable,
x= v x dx
dv x = a x dt
(b) If ay is constant
1 2
y = u yt + a t
2 y
vy = uy + ayt
v 2y = u 2y + 2a y y

KINEMATICS www.physicsashok.in 25
KINEMATICS

If ay is variable,
y= v ydt
dv y = a y dt
(c) If az is constant,
vz = uz + azt
1
z = uzt + azt2
2
2 2
v z = u z + 2a z z
If az is variable,
z= vz dt
dvz = a z dt
If the motion of the body takes place in xy plane, then az = 0, vz = 0, uz = 0


Example 28. A bird flies in the xy plane with a velocity v = t 2 i + 3t j . At t = 0, bird is at origin.
Calculate position and acceleration of bird as function of time.
Sol. We have given

v = t 2 i + 3t j
Here, vx = t2, vy = 3t and vz = 0
Since, vx = t2
dx
or t2
dt
x t 2
or 0 dx 0 t dt
t3
or x
3
Also, vy = 3t
dy
or 3t
dt
y t
or 0 dy 0 3t dt
3t 2
y
2

Thus, position of bird is r x i y j
t3 3t 2
r i j
3 2
vx = t2
dv x d(t) 2
ax 2t
dt dt
and vy = 3t

KINEMATICS www.physicsashok.in 26
KINEMATICS

dv y dt
or 3
dt dt
ay = 3 unit

Thus, acceleration of bird is a a x i a y j

a 2t i 3 j

PROJECTILE MOTION
We next consider a special case of two dimensional motion : A particle moves in a vertical plane with initial

velocity v 0 but its acceleration is always the free fall acceleration g , which is downward. Such a particle is
referred to as a projectile (meaning that is projected or launched) and its motion is called projectile motion. A
projectile might be a baseball in flight or a golf ball, but it is not air plane or a duck in flight. Or goal here is to
analyze projectile motion using the tools for two-dimensional motion and making the assumption that air has no
effect on the projectile.
Now let us consider a projectile launched so that its initial velocity v0 makes an angle with with the horizontal
(shown in figure). For discussion of motion, we take origin at the point of projection. Horizontal direction as x-
axis and vertical direction as y-axis is taken.
y

g v0

O x
A
The initial velocity of projectile along x-axis is ux = u cos .
The component of gravitational acceleration along x-axis is ax = g cos 90 = 0.
The component of initial velocity along y-axis is uy = u sin .
The acceleration along y-axis is ay = g

NOTE : In projectile motion, the horizontal and the vertical motions are
independent of each other, that is, neither motion affects the other.

Discussion
(a) The instantaneous velocity of the projectile as function of time : Let projectile reaches at point (x, y)
after time t [Fig.].
vx = ux = u cos y
vy
and vy = uy gt = u sin gt
u g
vx

v vx i vy j P(x,y)

O x
v u cos i (u sin gt) j A
The instantaneous speed

| v | (u cos ) 2 (u sin gt) 2
Also, x = uxt = (u cos ) t = ut cos

KINEMATICS www.physicsashok.in 27
KINEMATICS

1
y u y t gt 2
2
1
or y u sin t gt 2
2
The position of the projectile is

r x i y j
1
r ut cos i ut sin gt 2 j
2
(b) Trajectory of projectile : The yx graph gives the path or trajectory of the projectile.
From discussion of instantaneous velocity of projectile.
x = ut cos ...(i)
1
and y ut sin gt 2 ...(ii)
2
x
t ...(iii)
u cos
Putting the value of t from Eq. (iii) into Eq. (ii).
2
x 1 x
y u sin g
u cos 2 u cos
gx 2
or y x tan 2 ...(iv)
2u cos 2
This is the required path or projectile.
2u 2 cos 2
Multiplying the eq. (iv) by to both sides, we get
g

2u 2 sin cos 2u 2 cos 2


x2 x y
g g
2
u 2 sin cos
Adding to both sides, we get
g
2
u 2 sin cos 2u 2 cos 2 u 2 sin 2
x y
g g 2g
This is of the form,
(x a)2 = c(y b) ...(v)
which is the equation of a parabola. Hence, the equation of the path of the projectile is a parabola.

NOTE : The trajectory of a projectile will be parabolic when direction of velocity of


projectile is different from direction of acceleration and its acceleration is
constant both in magnitude and direction.

(c) Time of flight : In Fig., the time taken by projectile to reach at point A from point O is known as time of flight.
Here, OA = vxT, where T is time of flight. The total displacement along y-axis during motion of projectile from
O to A is zero. So, y = 0

KINEMATICS www.physicsashok.in 28
KINEMATICS

1
but y u y T gT 2
2
1
or 0 u sin T gT 2
2
2u sin
T ...(vi)
g
(d) Range of projectile : Distance OA is known as range of projectile [Fig.]
The time taken to reach point A from point O is
2u sin
T
g

2u sin
The range R u x T u cos
g
2sin cos
R u
g

u 2 sin 2
R ...(vii)
g
Caution : This equation does not give the horizontal distance travelled by a projectile when the final height is not the
launch height.
Regarding range of projectile, two cases are discussed below :
Case-I : Range will be maximum if sin 2 = 1 or = 45
u2
R max. (at = 45)
g
Case-II : For given velue of v, the range of projectile will be same for angle and 90 though their times of flight
and maximum heights are different.
u 2 sin 2 90
R 90
g

u 2 sin 180 2
R 90
g

u 2 sin 2 y
R 90 R u
g
Thus, for the case as shown in fig, u
60
R30 = R60 O
30
x

(e) Height attained by projectile : At the maximum height (at point B) the vertical component of velocity is zero.
v 2y u 2y 2gH y vy = 0
2 2 B
or (0) = (u sin ) 2gH ux
2 2
u sin H
H
2g O x
A

KINEMATICS www.physicsashok.in 29
KINEMATICS

NOTE : If air resists or opposes the projectile motion [Fig.], then


y
u I

II

0 0
O x
> 0
ITrajectory in vacuum
IITrajectory in presence of air resistance

Time taken by projectile during upward motion < Time taken during downward motion.
The values of height attained and of range of a projectile decrease.
The projectile returns to the ground with less speed. At its trajectory its horizontal velocity also decreases.
Time of flight also decreases.
At which angle, the projectile returns to the ground, increases.

Example 29. Prove that the maximum horizontal range is four times the maximum height attained by the projectile;
when fired at an inclination so as to have maximum horizontal range.
Sol. For = 45, the horizontal range is maximum and is given by
u2
R max
g
Maximum height attained
u 2 sin 2 45 u 2 R max
H max
2g 4g 4
or Rmax = 4 Hmax

Example 30. There are two angles of projection for which the horizontal range is the same. Show that the sum of
the maximum heights for these two angles is independent of the angle of projection.
Sol. There are two angles of projection and 90 for which the horizontal range R is same.
u 2 sin 2
Now, H1
2g
u 2 sin 2 90
and H2
2g
2 2
u cos
H2
2g
2
u u2
Therefore, 1H H 2
2g
sin 2
cos 2

2g
Clearly, the sum of the heights for the two angles of projection is independent of the angle of projection.

Example 31. A particle is projected upwards with a velocity of 100 m/sec at an angle of 60 with the vertical.
Find the time when the particle will move perpendicular to its initial direction, taking g = 10 m/sec2.
Sol. Here ax = 0
ay = g
ux = 100 sin60 = 50 3
uy = 100 cos60 = 50

KINEMATICS www.physicsashok.in 30
KINEMATICS

u 0 = u x i + u y j = 50 3 i + 50j
y

v y = u y + a y t = 50 gt
v x = u x = 50 3 m / s
60 x
v = 50 3 i + (50 - gt )j

But u and v are perpendicular..

u v0
or 50
3 i 50 j 50 3 i 50 gt j 0
or 7500 + 2500 500 t = 0
10000
t= t 0 = 20 second
500

Example 32. Fig. shows a pirate ship 560 m from a fort defending the harbor entrance of an island. A defense
cannon, located at sea level, fires balls at initial speed v0 = 82 m/s.
(a) At what angle 0 from the horizontal must a ball be fired to hit the ship ?
(b) How far should the pirate ship be from the cannon if it is to be beyond the maximum range of the cannonballs?
y

6 3
27 x

R = 560 m

Sol. (a) Because the cannon and the ship are at the same height, the horizontal displacement is the range.
The horizontal range is
v 20
R sin 2 ...(i)
g
which gives us
gR
20 sin 1 2
v0
9.8 560
20 sin 1
(82)2
20 = sin1(0.816) ...(ii)
1
0 (46.7 ) 23
2
and = 90 0
= 90 23 = 67
The commandant of the fort can elevate the cannon to either of these two angles and (if only there were no
intervening air!) hit the pirate ship.
(b) We have seen that maximum range corresponds to an elevation angle 0 of 45. Thus, from Eq. (i) with

KINEMATICS www.physicsashok.in 31
KINEMATICS

0 = 45.
v02 (82)2
R sin 20 sin(2 45 )
g 9.8
R = 686 m 690 m
As the pirate ship sails away, the two elevation angles at which the ship can be hit draw together, eventually
merging at 0 = 45 when the ship is 690 m away. Beyond that distance the ship is safe.

IMPORTANT FEATURES
1. As we have seen in the above derivations that ax = 0, i.e., motion of the projectile in horizontal direction is
uniform. Hence, horizontal component of velocity u cos does not change during its motion.
2. Motion in vertical direction is first retarded then accelerated in opposite direction. Because uy is upwards and
ay is downwards. Hence, vertical component of its velocity first decreases from O to A and then increases from
A to B. This can be shown as in fig.
y

A
uy u

x
O ux B
3. The co-ordinates and velocity components of the projectile at time t are
x = sx = uxt = (u cos ) t
1 1
y s y u y t a y t 2 u sin t gt 2
2 2
vx = ux = u cos
and vy = uy + ayt = u sin gt

Therefore, speed of projectile at time t is v v 2x v 2y and the angle made by its velocity vector with
positive x-axis is
vy
tan 1
vx
4. Equation of trajectory of projectile
x = (u cos )t
x
t
u cos
Substituting this value of t in,
1
y u sin t gt 2 , we get
2
gx 2
y x tan 2
2u cos 2
gx 2
y x tan 2 sec 2
2u
gx 2

y x tan 2 1 tan 2
2u

These are the standard equations of trajectory of a projectile.
The equation is quadratic in x. This is why the path of a projectile is a parabola. The above equation can also

KINEMATICS www.physicsashok.in 32
KINEMATICS
be written in terms of range (R) of projectile as
x
y x 1 tan
R
5. Range R is given by
2
R u cos u sin
g

2 Horizontal component Vertical component


R
g of initial velocity of initial velocity
6. There are two unique times at which the projectile is at the same height h(< H) and the sum of these two times
1 2
equals the time of flight T. Since, h u sin t gt is quadratic in time, so it has two unique roots t1 and
2
2u sin 2h
t2 (say) such that sum of roots (t1 + t2) is and product (t1t2) is . The time lapse (t1 t2) between
g g
these two events is
(t1 t2)2 = (t1 + t2)2 4t1 t2
4u 2 sin 2 8h
t1 t 2
g g
7. If K is the kinetic energy at the point of launch then kinetic energy at the highest point is
1 1
mv 2x mu 2 cos 2
K
2 2
2
K = K cos
8. For complementary angles and 90 , if T and T90 are the times of flight and R is the range, then
2R 2R 90 2R
T T90
g g g
2R1 2R 89
e.g., T1 T89
g g

Example 33. A particle is projected in the XY plane. 2 sec after projection the velocity of the particle
makes an angle 45 with the X-axis. 4 sec after projection, it moves horizontally. Find the velocity of
projection.
Sol. After 4 sec the particle reach at maximum hight.
At maximum height it move horizontally.
So that,
T y
=4
2 v0
2u sin 2u sin
T8 T
g g O x
80
u sin 40 ...(1)
2
u cos = v cos 45
v sin45 = 45 sin 10 2

KINEMATICS www.physicsashok.in 33
KINEMATICS
u sin 20 20
1 ucos = 20 ...(2)
u cos u cos
From eqn (1) and (2)
u2 sin2 + u2 cos2 = 402 + 202
u2 = 1600 + 400 = 2000
u 20 100 10 20
20 5 m / s

PROJECTILE PROJECTED FROM SOME HEIGHT


Projectile Projected in Horizontal Direction

Let a projectile is projected with velocity u. Take observation point O at a height h from ground [Fig.].
y
u x
O
r y(x, y)
x a y = g

vx
h
vy v
P A
Here, ux = u, uy = 0
and ax = 0, ay = g
(a) Let at time t, the co-ordinates of position of projectile is (x, y), then
1 2
x = ut and y 0 gt
2
Therefore, at time t position vector

r x i y j
1
r (ut) i gt 2 j
2
2 2 2 1 2
2
| r | x y (ut) gt
2
y
and tan
x
(b) Let at time t the horizontal and vertical velocities of projectile be vx and vy, so
vx = u and vy = 0 + (g)t = gt

v v i v j u i ( gt) j
x y

and v v 2x v 2y
v u 2 ( gt) 2
v
and tan x
vy
(c) Let time taken by projectile from O to point A at ground is T, then
1
h 0 ( g)T 2
2

KINEMATICS www.physicsashok.in 34
KINEMATICS

2h
T
g
The horizontal distance in time T
2h
PA uT u
g
Therefore, the bomb dropped from an aeroplane moving with velocity u horizontally at height h, covers a
2h
horizontal distance u on the ground.
g

Projectile Projected Upwards at an Angle



Let projectile is projected upward at angle with horizontal velocity u [Fig.].
u vy = 0, vx = u x = u cos
A ay = g
u sin

B u cos
x
O u cos
u sin
h u

P D C
ux = u cos and uy = u sin
ax = 0 and ay = g
Now from 2nd equation of motion,
1
h u sin T g T 2
2
or gT2 (2u sin )T 2h = 0
Solving this equation, we get horizontal distance covered in time T
PC = (u cos )T
2u sin
Time taken in covering path OAB
g
and horizontal distance covered in this time
u 2 sin 2
OB
g
In such case for range PC to become maximum, should be just less than 45.

NOTE : In such case on earths surface for maximum range,


u
sin = .
2
2u + 2gh
1
If h = 0, then = = 45.
2

Projectile Projected Downward at an Angle



Let projectile is projected downward at an angle with horizontal velocity u [Fig.].

KINEMATICS www.physicsashok.in 35
KINEMATICS
y u x = u cos
O x

u y = u sin
u
a y = g

P A
ux = u cos
and uy = u sin
ax = 0
and ay = g
From 2nd equation of motion,
1
h = u sin T g T 2
2
2
or gT + (2u sin )T 2h = 0
To solve this euation, value of T can be evaluated. In this time the horizontal distance covered on the earth,
PA = (u cos )T

Example 34. A projectile is fired horizontal with a velocity of 98 m/s from the u = 98 m/s
O
top of a hill 490 m high. Find x
(a) the time taken by the projectile to reach the ground
(b) the distance of the point where the particle hits the ground from foot y
of the hill and
(c) the velocity with which the projectile hits the ground (g = 9.8 m/s2) B A vx
Sol. Here, ux = 98 m/s, ax = 0, uy = 0 and ay = g
vy
(a) At A, sy = 490 m. So, applying
1
sy u y t a y t2
2
1
490 0 (9.8)t 2
2
t = 10 s
1
(b) BA s x u x t a x t 2
2
or BA = (98) (10) + (0) or BA = 980 m
(c) vx = ux = 98 m/s
vy = uy + ayt = 0 + (9.8) (10) = 98 m/s

v v 2x v 2y (98)2 (98)2 98 2 m / s

vy 98
and tan 1
vx 98
= 45
Thus, the projectile hits the ground with a velocity 98 2 m / s at an angle of = 45 with horizontal.

Example 35. A train starts from rest and moves with a constant acceleration of 2.0 m/s2 for half a minute. The
brakes are then applied and the train comes to rest in one minute. Find

KINEMATICS www.physicsashok.in 36
KINEMATICS
(a) the total distance moved by the train,
(b) the maximum speed attained by the train and
(c) the position(s) of the train at half the maximum speed.
Sol. (a) S1 = The distance moved by the train in first half minute.
1
S1 = u1t1 + a1t12
2
1 1
S1 = 0 t1 + 2 (30)2 { t1 = minute}
2 2
1
S1 = 2 900 S1 = 900 m
2
The distance moved by the train after brakes applied.
1
S2 = u2t2 + a2t22
2
u2 = u1 + a1t1
u2 = 0 + 2 30
u2 = 60 m/s
v2 = u2 a2t2
0 = 60 a2 60 a 2 = 1 m / sec2
v22 = u22 + 2 a2S2
0 = 60 60 2 1 S2
S2 = 1800 m
Total distance(s) = S1 + S2
S = (900 + 1800) m
S = 2700 m S = 2.7 km
(b) the maximum speed attained by the train is
u 2 = 60 m / s
(c) the position of the train at half of minimum speed.
u22 = u12 + 2a1S
(30)2 = 0 + 2 2 S
900
= S' S' = 225 m
4

Example 36. A car is moving along a straight line. It is taken from rest to a velocity of 20 ms1 by a constant
acceleration of 5 ms2. It maintains a constant velocity of 20 ms2 for 5 seconds and then is brought to rest
again by a constant acceleration of 2 ms2. Draw a velocity-time graph and find the distance covered by the
car.
Sol. v22 = u2 + 2aS
(20)2 = 0 + 2 5 S
400
S= S = 40 m
10
Car maintains a constant velocity of 20 m/s for 5 Second.
S = v t
S = 20 5 S '' = 100 m

KINEMATICS www.physicsashok.in 37
KINEMATICS
The car comes to rest by a constant acceleration of 2 m/s2
(v)2 = v2 + 2 aS
0 = (20)2 2 2 S
400
S '' = S'' = 100 m
4
The total distance covered by the car.
= S + S + S
= 40 + 100 + 100
= 240 m

Example 37. A ball rolls off the edge of a horizontal table top 4m high. If it strikes the floor at a point 5m horizontally
away from the edge of the table, what was its speed at the instant it left the table ?
1 2
Sol. Using h gt we have,
2
v
1 A
h AB gt 2AC
2
4m
2h AB 24
or t AC 0.9 s
g 9.8 B C
BC 5.0 5m
Further, BC = vtAC or v 5.55 m / s
t AC 0.9

Example 38. A ball is projected at an angle of 30 above with the horizontal from the top of a tower and
strikes the ground in 5 sec at an angle of 45 with the horizontal. Find the height of the tower and the
speed with which it was projected.
Sol. ux = v0 cos30
uy = v0 sin30
Vx = uX = v0 cos30
y
vy = u y + a y t v0
vy = v0 sin30 gt 30
x
v0 H
v y 10 5
2
v0 ax=0, ay= g
vy 50
But tan 45 2
v x v 0 cos 30
v
v 0 cos 30 0 50
2
1 3
v0 50
2 2
100
v0 =
3 +1


100 3 1
3 1 3 1
= 50 3 1
KINEMATICS www.physicsashok.in 38
KINEMATICS
1
Also, y = - H = u yt + a yt2
2
1
H v0 sin 30 5 10 52
2

H 125 2 3 m
Example 39. A ball is thrown horizontally from a cliff such that it strikes ground
37
after 5 sec. The line of sight from the point of projection to the point of
hitting makes an angle of 37 with the horizontal. What is the initial velocity
of projection.
Sol. vx = v0cos37
vy = v0sin37 y
ax = gsin37 = 6 m/s2 O v0
2 37
ay = gcos37 = 8 m/s
From O to A, displacement along y-axis is zero.
37
y = uyt + ayt 2
A x
0 = uy 5 + 8 25 { t = 5 sec.}
5uy = 8 25
100
uy = = 20 m / s
5
3
20 v0
5
100
v0 = m/s
3

Example 40. A ball is projected from top of a tower with a velocity of 5 m/s at an angle of 53 to horizontal.
Its speed when it is at a height of 0.45 m from the point of projection is :
(A) 2 m/s (B) 3 m/s (C) 4 m/s (D) data insufficient
Sol. According to conservation principle of machenic energy
Ui + T i = Uf + T f
1 1
0 + mu 2 = mgh + mv 2
2 2
2 2
u = 2gh + v
v = u 2 - 2gh

v 52 2 10 0.45
v 25 9 16 v 4 m/s

u
Example 41. In the figure shown, the two projectiles are fired simultaneously. 20m/s
What should be the initial speed of the left side projectile for the two 60 45
projectile to hit in mid-air ?
10 m
Sol. When two projectiles are projected from same height, then for collision, vertical component of velocities
of both projectiles should be same

KINEMATICS www.physicsashok.in 39
KINEMATICS
u sin 60 = 20 sin45
20sin 45
u=
sin 60
20 2 2
u u = 20 m/s
2 3 3

Example 42. The speed of a particle when it is at its greatest height is 2 / 5 times of its speed when it is at
its half the maximum height. the angle of projection is _____ and the velocity vector angle at half the
maximum height is ______.
Sol. uy = usin
u u cos
ux = ucos
vx = ucos ...(1)
2 2
vy = uy +2ayS
H
v 2y u 2 sin 2 2g ...(2)
2
2
u cos v 2x v 2y
5
2 H
u cos u 2 cos 2 u 2 sin 2 2g
5 2
2 u 2 sin 2
u cos u2 g
5 2g

2 sin 2
u 1
5 2
2 sin
cos 1
5 2
2
2 sin
cos2 1
5 2
2 sin 2 sin 2 2
or cos 2 , 1 sin 2
5 5 5 5
or 5 5sin 2 sin 2 2
or 3 4 sin2 = 0
3
sin 60
2
vy u 2 sin 2 gH
and tan {from eqn. (1) and (2)}
vx u cos
u 2 sin 2 sin 2
u 2 sin 2 g sin 2
2g 2
tan tan
u cos cos
sin 2
tan tan 60
2 tan
tan 2 2
cos
KINEMATICS www.physicsashok.in 40
KINEMATICS

3 3
tan tan 1
2 2

Example 43. A projectile is to be thrown horizontally from the top of a wall of height 1.7 m. Calculate the
initial velocity of projection if its hits perpendicularly an incline of angle 37 which starts from the ground
at the bottom of the wall. The line of greatest slope of incline lies in the plane of motion of projectile.
Sol. ux = v0
uy = 0
ay = g = 10 m/s2
vx = v1 cos53 = 0.6 v1
vy = v1 sin53 = 0.8 v1
vx = u x = v0
or 0.6 v1 = v0
v 10v0 5v 0
v1 = 0 = = ...(1) O v0
0.6 6 3 x
vy = u y + a y t 1 2
gt
2
0.8v1 = 10 t C
h = 1.7 m A 37
v1
5v
0.8 0 10 t
3 37 53
6 60 B grad x 53
v0 = t= t ...(2) y v1
0.8 8
In BAC
1
AB 1.7 - gt 2 1.7 - 5t 2
tan 37 = 3 2
= =
AC 4 uxt v0 t
2
or 3v0t = 6.8 20t
8v 64 v02
or 3v 0 0 6.8 20
60 3600
2
24 2 64v 0 72 64 2
or v0 + = 6.8 v 0 6.8
60 180 180
6.8 180
or v 20 9 v0 = 3 m / s
136

Example 44. A hunter is riding an elephant of height 4m moving in straight line with uniform speed of 2 m/
sec. A dear running with a speed V in front at a distance of 4 5m moving perpendicular to the direction
of motion of the elephant. If hunter can throw his spear with a speed of 10 m/sec, relative to the elephant,
then at what angle to its direction of motion must he thrown his spear horizontally for a successful hit.
Find also the speed V of the dear.
1 2
Sol. h = gt
2
2h 24 8
t
g 10 10

KINEMATICS www.physicsashok.in 41
KINEMATICS
Assume horizontal plane at xyplane.

v rel 10 cos i 10sin j

v rel u 2 i

u v rel 2 i 10 cos 2 i 10 sin j
The deer is moving along yaxis.
So, displacement of deer and displacement of spear along yaxis will be same in time t.
v t = u yt
v = 10 sin ...(1)
Also, along xaxis :
ux t = 4 5
8 4m
or 10cos 2 4 5 2m/s
10
x
10 4 5
or 10cos 2 4 5
2
10
8
10 cos = 8
8 4
cos = 37
10 5
From eqn (1)
3
v = 10 sin = 10 sin37 = 10 = 6 m/s
5

y
Example 45. An object A is kept fixed at the point x = 3 m and y = 1.25 m on a A
1.25m P
plank P raised above the ground. At time t = 0 the plank starts moving along
the + x direction with an acceleration 1.5 m/s2. At the same instant a stone is u
x
projected from the origin with a velocity u as shown. A stationary person on O 3.0m
the ground observes the stone hitting the object during its downward motion at an angle of 45 to the horizontal.
All the motions are in x y plane. Find u and the time after which the stone hits the object. Take g = 10 m/s2.
Sol. ux = u cos
uy = u sin
ax = 0 u u =ucos
x

ay = g 45
u sin

If the stone hitt the object after time t. 1.25


So that virtical displacement of stone is 1.25 m u cos
1 2
y = uyt + a yt
2
1 2
therefore 1.25 (u sin )t gt
2
1
1.25 (u sin )t 10 t 2
2
1.25 = (u sin) t 5 t 2
(u sin)t = 1.25 + 5t 2 ...(1)
Hotizontal displacement of stone is
x = 3 + displacement of object A.
Initial velocity of object is zero.

KINEMATICS www.physicsashok.in 42
KINEMATICS
so displacement of object is
1 1
1.25 at 2 1.5 t 2 0.75 t 2
2 2
x = 3 + 0.75 t 2
(u cos)t = 3 + 0.75 t 2 ...(2)
Since velocity vector inclined at 45 with horizontals.
u y u sin gt
tan(45 )
ux u cos
u cos = (u sin gt)
u cos = gt u sin ...(3)
(u cos) t + (u sin) t = 10 t2
Add eqn (1) and (2)
(u cos) t + (u sin) t = 4.25 + 5.75 t2 ...(4)
from eqn (3) and (4)
10 t2 = 4.25 + 5 .27 t2
4.25 t2 = 4.25
t2 = 1 t = 1 sec.
From eqn (1) and (2)
uy = u sin = 6.25 m/s
uy = 6.25 m/s
ux = 3.75 m/s
u = u 2x + u 2y

u = (6.25) 2 + (3.75) 2 u = 7.29 m / s

IMPORTANT FEATURES
Projectile motion is a two dimensional motion with constant acceleration (g). So, we can use

v uat
1
a u t a t2
2
etc. in projectile motion as well. Here,

u u cos i u sin j and a g j y
Now, suppose we want to find velocity at time t.
u
v u a t

v u cos i u sin j gt j
O x
or v u cos i u sin gt j g
Similarly, displacement at time t will be,
1
S u t a t2
2
1

S u cos i u sin j t gt 2 j
2

KINEMATICS www.physicsashok.in 43
KINEMATICS

1
S u cos i ut sin gt 2 j
2

PROJECTILE MOTION ON AN INCLINED PLANE


One an inclined plane projectile is projected into two cases, one upwards and the other downwards.

Up the Plane
A projectile is projected up the inclined plane from the point O with an initial velocity u at an angle with
horizontal. The angle of inclination of the plane with horizontal is [Fig.].
x
x
u A s in
) =g
9 0
o s(
gc
y
y g sin(90 ) = g cos
90

O B x O
y
x y
ux = u cos ( ) and ax = g sin
uy = u sin ( ) and ay = g cos
(i) Time of flight : During motion from point O to A, the displacement along y-axis is zero.
sy = 0 at t = T
1
sy u y t a y t2
2
1
or 0 u sin T g cos T 2
2
2u sin
T
g cos

2u sin
NOTE : Substituting = 0, in the above expression, we get T = .
g
Which is quite obvious because = 0 is the situation shown in Fig.
y

O x
g

(ii) Range : As shown in Fig, OA is the range of proectile.


Horizontal component of initial velocity uH = u cos
OB = uHT (as aH = 0)

=
u cos 2u sin 2u 2 sin cos
g cos g cos

KINEMATICS www.physicsashok.in 44
KINEMATICS

OB 2u 2 sin cos
R OA
cos g cos 2
Using, 2 sin A cos B = sin (A + B) + sin (A B)
Range can also be written as,
u2
R sin 2 sin
g cos 2
This range will be maximum when
u2
2 R
or and max 1 sin
2 4 2 g cos 2

we see that for = 0, range will be maximum for or 45.
4
u2
R max 1 sin 0
g cos 2 0
u2
R max
g
Alternative method :
For range, sx = R, t = T
1
sx u x t a x t 2
2
1
or R u cos T g sin T 2
2
2u sin
Substituting the value of T , in above equation for R.
g cos
u2
R sin 2 sin
g cos 2

Down the Plane


A projectile is projected down the plane from the point O with an initial velocity u at an angle with horizontal
[Fig.]. The angle of inclination of plane with horizontal is .
y

u x
g sin
)=
O
( 90
cos
g g sin(90 ) = g cos
90

A
Therefore, ux = u cos ( + ), ax = g sin
uy = u sin ( + ), ay = g cos
Proceeding in the similar manner, we get the following results :
2u sin
T
g cos

KINEMATICS www.physicsashok.in 45
KINEMATICS

u2
R sin 2 sin
g cos 2

IMPORTANT FEATURES
If two particles are projected at angles 1 and 2 respectively as shown in fig., then the relative motion of 1
with respect to 2 is a straight line at an angle.
y
y u2

u1

1 2
x x

u12y
tan 1 with positive x-axis.
u12x
where u12x = u1x u2x = u1 cos 1 u2 cos 2
u12y = u1y u2y = u1 sin 1 u2 sin 2

Example 46. A particle is projected with a velocity of 20 m/s at an angle of 30 to an inclined plane of inclination
30 to the horizontal. The particle hits the inclined plane at an angle of 30, during its journey. Find the
(a) time of impact,
(b) the height of the point of impact from the horizontal plane passing through the point of projection.
Sol. The particle hits the plane at 30 (the angle of inclination of plane). It means particle hits the plane horizontally.
T u sin u
(a) t
2 g
20sin 30 30
t 1.76 s
9.8 30
u 2 sin 2 30
(b) H
2g
2
20 sin 2 60
H 15.3 m
2 9.8

Example 47. A particle is projected up an inclined plane with initial speed v = 20 m/s at an angle = 30 with plane.
Find the component of its velocity perpendicular to plane when it strikes the plane.
Sol. Component of velocity perpendicular to plane remains the same (in opposite direction)
i.e., u sin = 20 sin 30 = 10 m/s

Example 48. A particle is thrown horizontally with relative velocity 10 m/s from an
10m/s2
inclined plane, which is also moving with acceleration 10 m/s2 vertically upward.
Find the time after which it lands on the plane (g = 10 m/s2). 30

Sol. ux = 10 cos30 = 5 3 m/s


uy = 10 sin30 = 5 m/s
arel =20 m/s2

KINEMATICS www.physicsashok.in 46
y
10sin30 10 m/s2
v0= 10m/s
KINEMATICS
30 O
0
ax = arel sin30 = 10 m/s2 co s3
ay = arel cos30 A 10
30
3 x
20 10 3 m / s 2
2
when ball lands on inclined plane, y = 0
1
y = uyt + ay t 2
2
1
0 5 t 10 3 t 2
2
5 3 t =5
1
t= sec.
3

Example 49. A particle is projected from point P with velocity 5 2 m/s perpendicular to y
the surface of a hollow right angle cone whose axis is vertical. It collides at Q
normally. Find the time of the flight of the particle. Q
P
Sol. u0x = 5 2m / s 45 x
u0y = 0
ax = g sin45
ay = g cos45 y x
At point Q, vx = 0 { the particle collied normally at point Q} P Q
The time taken by particle to go from P to Q is t0.

45
vx = u x + a x t g 45 45
0 = 5 2 g sin45 t 0
5 2 5 2 2
t0 1 sec.
g sin 45 10
t 0 = 1 sec.

Example 50. A ball is projected on smoot inclined plane in direction perpendicular to 8 m/s
line of greatest slope with velocity of 8m/s. Find its speed after 1 sec.
37
Sol. vy = uy + ayt
vy = 0 + 10 sin37 1
3
vy = 10 1
5 7
in3
gs
vy = 6 m/s 37
vx = ux = 8 m/s y x
v = v 2x + v 2y
= 82 + 6 2
Ans. 10 m / s

Example 51. Two inclined planes OA and OB having inclination (with horizontal) 30 and 60 respectively,
intersect each other at O as shown in fig.
A particle is projected from point P with velocity u = 10 3 ms - 1
KINEMATICS www.physicsashok.in 47
B
u KINEMATICS
A Q
along a direction perpendicular to plane OA. If the particle strikes P
plane OB perpendicularly at Q, calculate h
(a) time of flight, 30 60
(b) velocity with which particle strikes the plane OB, O
(c) vertical height h of P from O,
(d) maximum height from O attained by the particle.
Sol. (a) u x = 10 3 m / s
uy = 0
ax = g sin60 B
u
A Q
ay = g cos60
P
At point Q h
vx = 0
30 60
The time taken by particle to go from P to Q is to O
vx = ux + axt
0 = 10 3 - g sin 60 t 0
10 3
t0 = t 0 = 2sec
g sin 60
(b) vy = uy ayt 0
vy = 0 g cos60 2
1
v y 10 2 10 m / sec
2
h u X
sin 30 = Y
(c)
x
1 h h
= x = 2h m x
2 x
30 60
1 O
2h = u y t + a y t 2
2
1
2h 0 g sin 30 (2) 2
2
1 1
2h 10 4 h= 5m
2 2
(d) At maximum height uy = u cos30
vy = 0 X
30
v 2y = u 2y - 2gH Y
2
0 = (u cos30) 2gH h 60
2 2
u cos 30 30
H=
2g
2
3 2

10 3
2
H
2g
100 3 3 4
H
2 10

KINEMATICS www.physicsashok.in 48
KINEMATICS

25 9 45
H= = 11.25 m
20 4
Maximum height from O attained by the particle
= H+h
= 11.25 + 5
Ans. 16.25 m

Example 52. A large heavy box is sliding without friction down a smooth plane of inclination
. From a point P on the bottom of a box, a particle is projected. inside the box. The
initial speed of the particle with respect to box is u and the direction of projection makes P Q
an angle with the bottom as shown in figure.
(a) Find PQ if particle lands on Q.
(b) If horizontal displacement of particle with respect to ground is zero. Find the velocity of box.
Sol. ux = u cos
uy = usin
ax = g sin
ay = g cos
When the particle hitt the inclined plane then displacement in
y-direction is zero. y
1 u =usin
y
x
y = u yt + a yt2 Q
2 s
o
1 P uc s
0 (u sin )t g cos t 2 ax = gsin u = gco
x

2 90
=
ay
1
(u sin )t g cos t 2 g
2
2u sin
t ...(1)
g cos
with respect to box
ux = u cos
ax = 0
1
x = uxt + axt2
2
x = u cost
(2u sin )
x (u cos ) (from eqn 1)
g cos
u 2 sin 2
x
g cos
According to question
ux = U
ax = gsin
1
x = uxt + axt2
2
2 2
u sin 2 2u sin 1 2u sin
U g sin
g cos g cos 2 g cos

KINEMATICS www.physicsashok.in 49
KINEMATICS
u sin cos
u cos U
cos
u sin cos
u cos U
cos
u sin cos
U u cos
cos
cos cos cos sin
U u
cos
u cos( )
U
cos

THINKING PROBLEMS

1. Can you use the equations of kinematics to find the height attained by a body projected upwards with any
velocity ?
2. Can the relative velocity of two bodies be greater than the absolute velocity of either ?
3. A boy sitting in a car moving with a constant velocity throws a ball straight up into the air. Will the ball fall
behind him, in front of him or into his hand? What would happen if the car accelerated forward or went round
a curve while the ball was in the air ?

v f vi
4. A student argues that the mean velocity during an interval of time can also be expressed as v
2
r f ri
and this should always be equal to v f . Is he right .
t 2 t1
5. Consider a collection of a large number of particles moving with the same speed v in random directions. Could
you, by using simple logic, show that the magnitude of the relative velocity of a pair of particles averaged over
all the pairs of the collection is greater than v ?
6. The barrel of a gun and a target lie along the same horizontal. If the target
is released and the gun is fired at the same time, the bullet will always

hit the target whatever be the distance between the gun and the target. Is this true or false ?

7. A black dot is made at the tip of an aerofoil of an aeroplane. What is the trajectory of the black dot as it
appears to the pilot and to an observer on the ground?
8. A body is dropped fron the window of a train. W ill the time of the free fall be equal if the train is stationary,
moves with constant velocity, moves with constant acceleration?
9. We can order events in time, such as past, present and future. Hence there is a sense of time. So is time a
vector? If not, why not?

10. Average speed can mean the magnitude of the average velocity vector. Another meaning given to it is that the
average speed is the total length of the path traversed divided by the elapsed time. Are these meanings
different? if so, give an example.
11. Can a body have zero velocity and still be accelerating? can a body have a constant speed and still have a
varying velocity? Can a body have a constant velocity and still have a varying speed?
12. Can an object have an eastward velocity while experiencing westward acceleration?

KINEMATICS www.physicsashok.in 50
KINEMATICS
THINKING PROBLEMS SOLUTION

1. No, because the equations are applicable only so long as the acceleration is uniform. The acceleration due to
gravity is uniform only near the surface of the earth.

2. Yes, e.g., when two bodies move in opposite directions, the relative velocity of each is greater than the individual
velocity of either.
3. It will fall into his hand because of the physical independence of vectors or the fact that the forward velocity of
the ball in the air is the same as that of the car. But if the car accelerated, the ball would fall behind him. If the
car went round a curve the ball would fall in front of him because it would move along the tangent to the curved
path through a certain distance while the car moved along the curved path through the same distance.

4. No, he is not right. The correct definition of average velocity is the latter one. the first one can be used only
when there is uniform acceleration.

5. v rel v 2 v2 2v 2 cos 2v sin / 2. In other words between 0 and 60 the relative velocity is less
than v, while between 60 and 180 it is greater than v. Hence the relative velocity averaged over all directions
v rel v.
6. True, provided the target is within the range of the bullet, because the downward acceleration of both is the
same.
7. A circular path relative to the pilot and a helical path relative to the observer fixed to the earth.

8. Same in all the three cases due to the physical independence of vectors.

9. Time is not a vector though it may have a sense because in order to be a vector it must add and subtract by
vector algebra rules and it does not.
10. The meanings are different. The point can be made clear by considering the motion of a particle along a
circle. Let a particle describe half a circle of radius R in time .
R i R i 2R
v = average velocity vector i

2R R
| v | v = average speed

Obviously, v | v | , that is, average speed as the modulus of the average velocity vector is not the same
as the average speed defined as the total length of the path traversed divided by time elapsed.
11. Yes, for example a particle in SHM at the extreme position has zero velocity but its acceleration is maximum.
Yes, a particle in circular motion has constant speed but varying velocity. No, a body cannot have a constant
velocity while having a varying speed.

12. Yes, a body can have eastward velocity and westward acceleration, e.g., SHM.

KINEMATICS www.physicsashok.in 51
KINEMATICS
ASSERTION & REASON
THE NEXT QUESTIONS REFER TO THE FOLLOWING INSTRUCTIONS
A statement of assertion (A) is given and a Corresponding statement of reason (R) is given just below
it of the statements, mark the correct answer as
(A) If both A and R are true and R is the correct explanation of A.
(B) If both A and R are true but R is not the correct explanation of A.
(C) If A is true but R is false.
(D) If both A and R are false.
(E) If A is false but R is true.

1. Assertion (A) : Horizontal component of the velocity of angular projectile remains unchange during its
flight.
Reason (R) : At highest point the velocity reduces to zero.
(A) (B) (C) (D) (E)

2. Assertion (A) : Suppose a particle starts moving in a straight line with initial velocity +u and an acceleration
a, then velocity at displacement s comes out to be, v 2 u 2 2as . If we draw a graph
between v 2 and s, it will be a straight line as shown in figure.

2
v

S0 S

Reason (R) : v 2 versus s graph is a straight line passing through origin with positive intercept and
negative slope.
(A) (B) (C) (D) (E)

3. Assertion (A) : A body can have acceleration even if its velocity is zero at a given instant of time.
Reason (R) : A body is momentarily at rest when it reverses its direction of motion.
(A) (B) (C) (D) (E)

4. Assertion (A) : When a body is projected with an angle 45, its range is maximum.
Reason (R) : For maximisation of range sin 2 should be equal to one.
(A) (B) (C) (D) (E)

5. Assertion (A) : Rocket in flight is not an illustration of projectile.


Reason (R) : Rocket takes flight due to combustion of fuel and does not move under the gravity effect
alone.
(A) (B) (C) (D) (E)

6. Assertion (A) : The slope of displacement-time graph of a body moving with high velocity is steeper
than the slope of displacement-time graph of a body with low velocity.
Reason (R) : Slope of displacement-time graph = Velocity of the body.
(A) (B) (C) (D) (E)

7. Assertion (A) : Displacement of a body may be zero when distance travelled by it is not zero.
Reason (R) : The displacement is the longest distance between initial and final position.
(A) (B) (C) (D) (E)

KINEMATICS www.physicsashok.in 52
KINEMATICS
8. Assertion (A) : The relative velocity between any two bodies moving in opposite direction is equal to sum
of the velocities of two bodies.
Reason (R) : Sometimes relative velocity between two bodies is equal to difference in velocities of the
two.
(A) (B) (C) (D) (E)

9. Assertion (A) : The displacement-time graph of a body moving with uniform acceleration is a straight line.
Reason (R) : The displacement is proportional to time for uniformly accelerated motion.
(A) (B) (C) (D) (E)

10. Assertion (A) : A body falling freely may do so with constant velocity.
Reason (R) : The body falls freely, when acceleration of a body is equal to acceleration due to gravity.
(A) (B) (C) (D) (E)

11. Assertion (A) : The speedometer of an automobile measure the average speed of the automobile.
Reason (R) : Average velocity is equal to total displacement per total time taken.
(A) (B) (C) (D) (E)

KINEMATICS www.physicsashok.in 53
KINEMATICS

Level # 1
1. A train starting from rest travels the first part of its journey with constant acceleration a, second part with
constant velocity v and third part with constant retardation a, being brought to rest. The average speed for
7v
the whole journey is . The train travels with constant velocity for .... of the total time
8
3 7 5 9
(A) (B) (C) (D)
4 8 6 7

2. A particle moving in a straight line with uniform acceleration is observed to be at a distance a from a fixed
point initially. It is at distances b, c, d from the same point after n, 2n, 3n second. The acceleration of the
particle is
c 2b a cba c 2b a c ba
(A) (B) (C) (D)
n2 9 n2 4 n2 n2

3. If a, b and c be the distances travelled by the body during xth, yth and zth second from start, then which of
the following relations is true ?
(A) a(y z) + b(z x) + c(x y) = 0 (B) a(x y) + b(y z) + c(z x) = 0
(C) a(z x) + b(x y) + c(y z) = 0 (D) ax + by + cz = 0

4. A body of mass 3 kg falls from the multi-storeyed building 100 m high and buries itself 2m deep in the sand.
The time of penetration will be
(A) 0.09 s (B) 0.9 s (C) 9 s (D) 10 s

5. Two cars A and B, each having a speed of 30 km/hr are heading towards each other along a straight path. A
bird that can fly at 60 km/hr flies off car A when the distance between the cars is 60 km, heads directly
towards car B, on reaching B, the bird directly flies back to A and so forth, then the total distance the bird
travels till the cars meet is.
(A) infinite (B) 30 km (C) 60 km (D) 120 km

6. Two cars A and B, each having a speed of 30 km/hr are heading towards each other along a straight path. A
bird that can fly at 60 km/hr flies off car A when the distance between the cars is 60 km, heads directly
towards car B, on reaching B, the bird directly flies back to A and so forth, the total no. of trips which the bird
makes till the cars meet is
(A) Four (B) Eight (C) Sixteen (D) Infinite

7. Three particles start from the origin at the same time, one with a velocity v 1 along x-axis the second along
the y-axis with a velocity v 2 and the third along x = y line. The velocity of the third so that the three may
always lie on the same line.
v1 v 2 2 v1 v 2 3 v1 v 2
(A) (B) (C) (D) Zero
v1 v 2 v1 v 2 v1 v 2

8. Choose the correct statement


(A) A body starts from rest and moving with constant acceleration travels a distance y1 and the 3rd second
y1 5
and y2 in 5th second. The ratio y 9 .
2
(B) A ball falls from the top of a tower in 8 second in 4 second, it till cover the first quarter of the distance
starting from top.
(C) The distance traveled by a freely falling stone released. With zero velocity in the last second of its
motion to that traveled by it in the last second of its motion to that traveled by it in the last but one
second is 7 : 5. The stone strike the ground with velocity 39.2 m/s.
(D) None of these

KINEMATICS www.physicsashok.in 54
KINEMATICS
9. A steam boat goes across a lake and comes back (i) on a quiet day when the water is still, and (ii) on a
rough day when there is a uniform current so as to help the journey onward and to impede the journey back.
If the speed of the launch, on both days, was same, the time required for the complete journey on the rough
day, as compared to that on the quiet day, will be.
(A) Less (B) Same (C) More (D) Cannot be predicted

10. A car is moving along a straight road with a uniform acceleration. It passes through two points P and Q
separated by a distance with velocity 30 km/hr and 40 km/hr respectively. The velocity of the car midway
between P and Q is
(A) 33.3 km/hr (B) 20 3 km/hr (C) 25 2 km/hr (D) 35 km/hr

11. Starting from rest a particle moves in a straight line with acceleration a = (25 t 2)1/2 m/s2 for 0 t 5 s,
3
a= m/s2 for t > 5 s. The velocity of particle at t = 7 s is.
8
(A) 11 m/s (B) 22 m/s (C) 33 m/s (D) 44 m/s

12. A body is released from a great height and falls freely towards the earth. Exactly one sec later another body
is released. What is the distance between the two bodies 2 sec after the release of the second body.
(A) 4.9 m (B) 9.8 m (C) 24.5 m (D) 50 m

13. The distance moved by a freely falling body (starting from rest) during 1st, 2nd, 3rd, .... nth s of its motion
are proportional to.
(A) Even numbers (B) Odd numbers
(C) All integral numbers (D) Square of integral numbers

14. A ball is thrown vertically upwards with a speed of 10 m/s from the top of a tower 200 m high and another is
thrown vertically downwards with the same speed simultaneously. the time difference between them in
reaching the ground in s(g = 10 m/s 2) is
(A) 12 (B) 6 (C) 2 (D) 1

15. Between two stations a train first accelerates uniformly, then moves with uniform speed and finally, retards
uniformly. If the ratios of the time taken for acceleration, uniform speed and retarded motions are 1 : 8 : 1
and the maximum speed of the train is 60 km/h, the average speed of the train over the whole journey is.
(A) 25 km/h (B) 54 km/h (C) 40 km/h (D) 50 km/h

16. A ball is thrown vertically upwards. It was observed at a height h twice with a time interval t. The initial
velocity of the ball is.
2
g t 1
(A) 2
8 gh g ( t ) 2
(B) 8 gh (C) 2 2
(D) 8 gh 4g2 ( t )2
2 2 8 gh g ( t )

17. A target is made of two plates, one of wood and the other of iron. The thickness of the wooden plate is 4 cm
and that of iron plate is 2 cm. A bullet fired goes through the wood first and then penetrates 1 cm into iron.
a similar bullet fired with the same velocity from opposite direction goes through iron first and then penetrates
2 cm into wood. If a1 and a2 be the retardation offered to the bullet by wood and iron plates respectively then
(A) a1 = 2a2 (B) a2 = 2a1 (C) a1 = a2 (D) Data insufficient

dv
18. The motion of a body falling from rest in a resisting medium is described by the equation = a bv where
dt
a and b are constants. The velocity at any time t is
a b a b bt
(A) v t = b (1 ebt) (B) v t = ebt (C) v t = b (1 + ebt) (D) v t = e
a a
P
19. A self-propelled vehicle of mass M whose engine delivers constant power P has an acceleration a = . To
o
Mv
increase the velocity of the vehicle from v 1 to v 2, the distance travelled by it (assuming no friction) is
3P 2 M 2 M 3 3P 3
(A) s = (v 2 v 12 ) (B) s = ( v 2 v 12 ) (C) s = ( v 2 v 13 ) (D) s = ( v 2 v 13 )
M 3P 3P M

KINEMATICS www.physicsashok.in 55
KINEMATICS

20. For an airplane to take-off it accelerates according to the A B


graph shown and takes 12 s to take-off from the rest 5
position. The distance travelled by the airplane is.

m/s2
(A) 21 m (B) 210 m
(C) 2100 m (D) 120 m
6 12
t (in s)

21. A river is flowing from west to east at a speed of 5 meters per minute. A man on the south bank of the river,
capable of swimming at 10 meters per minute in still water, wants to swim across the river in the shortest
time. He should swim in a direction.
(A) due north (B) 30 east or north (C) 30 west of north (D) 60 east of north

22. A boat which has a speed of 5 km/hr in still water crosses a river of width 1 km along the shortest possible
path in 15 minutes. The velocity of the river water in km/hr is A
(A) 1 (B) 3 (C) 4 (D) 41

23. In 1, 0, s, a particle goes from point A to point B, moving in a semicircle of


1.0 m
radius 1.0 m (see Figure). The magnitude of the average velocity
(A) 3.14 m/s (B) 2.0 m/s
(C) 1.0 m/s (D) Zero

24. A ball is dropped vertically from a height d above the ground. It hits the ground and B

bounces up vertically to a height d 2 . Neglecting subsequent motion and air resistance, its velocity v varies
with the height h above the ground as
v v v v

d d d
(A) h (B) h (C) h (D) h
d

Acceleration
2
(m/s )
25. A body starts from rest at time t = 0, the acceleration time graph is shown 10
in the figure. The maximum velocity attained by the body will be
(A) 110 m/s (B) 55 m/s
(C) 650 m/s (D) 550 m/s
Time
11 (Sec.)
26. The velocity is displacement graph of a particle moving along a straight line is shown
v
v0

x
x0
The most suitable acceleration-displacement graph will be
a a a a
x x x
(A) (B) (C) (D)

KINEMATICS www.physicsashok.in 56
KINEMATICS

Multiple Choice Question


27. A particle is moving eastwards with a velocity of 5 m/s. In 10s the velocity changes to 5 m/s northwards. The
average acceleration in this time is
(A) Zero (B) 1 2 m s 2 towards north-west
1
(C) 1 2 m s 2 towards north-east (D) m s 2 towards north-west
2
28. A particle of mass m moves on the x-axis as follows : it starts from rest at t = 0 from the point x = 0, and
comes to rest at t = 1 at the point x = 1. NO other information is available about its motion at intermediate
times (0 < t < 1). If denotes the instantaneous acceleration of the particle, then:
(A) cannot remain positive for all t in the interval 0 t 1.
(B) cannot exceed 2 at any point in its path.
(C) must be 4 at some point or points in its path.
(D) must change sign during the motion, but no other assertion can be made with the information given.

29. The coordinates of a particle moving in a plane are given by x(t) = a cos(pt) and y(t) = b sin(pt) where a, b(<a)
and p are positive constants of appropriate dimensions. Then
(A) the path of the particle is an ellipse
(B) the velocity and acceleration of the particle are normal to each other at t 2 p .
(C) the acceleration of the particle is always directed towards a focus
(D) the distance travelled by the particle in time interval t = 0 to t 2 p is a
30. At t = 0, an arrow is fired vertically upwards with a speed of 98 ms1
A second arrow is fired vertically upwards with the same speed at t = 5s. Then
(A) The two arrows will be at the same height above the ground at t = 12.5 s.
(B) The two arrows will reach back their starting points at t = 20 s and at t = 25 s.
(C) The ratio of the speeds of the first and the second arrows at t = 20 s will be 2 : 1.
(D) The maximum height attained by either arrow will be 980 m.

31. A projectile is fired with a constant speed at two different angles of projection, say, and that give it the
same range. Then, and are such that
(A) cosec = sec (B) tan ( + ) (C) sin2 cos2 = sin2 cos2 (D) cot = cos sec

Fill in the blanks


1. A particle moves in a circle of radius R. In half the period of revolution its displacement is _________ and
distance covered is ___________.

2. Four persons K, L, M, N are initially at the four corners of a square of side d. Each person now moves with a
uniform speed v in such a way that K always moves directly towards L, L directly towards M, M directly
towards N, and N directly towards K. The four persons will meet at a time __________.

3. Spotlight S rotates in a horizontal plane with constant angular velocity of 0.1 radian/second. The spot of light
P moves along the wall at a distance of 3 m. The velocity of the spot P when 45 (see figure) is
__________ ms/.
S
2
4. The trajectory of a projectile in a vertical plane is y = ax bx , where a, b are
constants, and x and y are respectively the horizontal and vertical distances of 3m
the projectile from the point of projection. The maximum height attained is
________ and the angle of projection from the horizontal is ________. P

KINEMATICS www.physicsashok.in 57
KINEMATICS

True / False
5. Two balls of different masses are thrown vertically upwards with the same speed. They pass through the point
of projection in their downward motion with the same speed (Neglect air resistance).

6. A projectile fired from the ground follows a parabolic path. The speed of the projectile is minimum at the top
of its path.

7. Two identical trains are moving on rails along the equator on the earth in opposite directions with the same
speed. They will exert the same pressure on the rails.

8. An electric line of forces in the x-y plane is given by the equation x 2 + y2 = 1. A particle with unit positive
charge, initially at rest at the point x = 1, y = 0 in the x-y plane, will move along the circular line of force.

Table Match
9. Match List I and List II and select the correct answer using the codes given below in the lists:
Column-I Column-II

I. Deceleration decreasing A.

time

II. Deceleration increasing B.

time

velocity

III.Acceleration decreasing C.

time

velocity

IV.Uniform acceleration D.

time

velocity

E.

time
(A) ID, IIE, IIIC, IVA (B) IE, IIB, IIIC, IVD
(C) IC, IIE, IIIB, IVA (D) ID, IIB, IIIA, IVC

KINEMATICS www.physicsashok.in 58
KINEMATICS

Passage Type Questions


THE NEXT QUESTIONS REFER TO THE FOLLOWING PASSAGE
When an airplane flies, its total velocity with respect to the ground is

vtotal v plane vwind

Where v plane , denotes the planes velocity through motionless air, and vwind denotes the winds velocity..
Crucially, all the quantities in this equation are vectors. The magnitude of a velocity vector is often called the
speed.
Consider an airplane whose speed through motionless air is 100 m/s. To reach its destination, the plane
must fly east.
The heading of a plane is the direction in which the nose of the plane points. So, it is the direction in which
the engines propel the plane.

1. If the plane has an eastward heading, and a 20 m/s wind blows towards the southwest, then the planes
speed is
(A) 80 m/s (B) more than 80 m/s but less than 100 m/s
(C) 100 m/s (D) more than 100 m/s

2. The pilot maintains an eastward heading while a 20 m/s wind blows northward. The planes velocity is deflected
from due east by what angle?
1 1 1
(A) sin 1 (B) cos 1 (C) tan 1 (D) None of these
5 5 5

3. Let denote the answer to previous question. The plane in question 2 has what speed with respect to the
ground?

100 m s 100 m s
(A) 100 m s sin (B) 100 m s cos (C) (D)
sin cos

4. Because the 20 m/s northward wind persists, the pilot adjusts the heading so that the planes total velocity
is eastward. By what angle does the new heading differ from due east?

1 1 1 1 1 1
(A) sin (B) cos (C) tan (D) None of these
5 5 5

5. Let denote the answer to previous question. What is the total speed, with respect to the ground, of the
plane in previous question?

100 m s 100 m s
(A) 100 m s sin (B) 100 m s cos (C) (D)
sin cos

THE NEXT QUESTIONS REFER TO THE FOLLOWING PASSAGE


During a car crash, the more rapidly a person decelerates, the more likely she is to be injured. A large
deceleration is dangerous, even if it lasts for a short time.
Airbags are designed to decrease the magnitude of the deceleration. Before the airbag inflates, the driver
continues forward at constant speed. But once the airbag inflates, the driver decelerates gradually, instead of
getting thrown into the windshield or steering wheel.
20 v Airbag 1 20 v Airbag 2
(m/s) (m/s)

.01 .02 .03 .04 .05 t(s) .01 .02 .03 .04 .05 t(s)

KINEMATICS www.physicsashok.in 59
KINEMATICS

20 v Airbag 3
(m/s)

.01 .02 .03 .04 .05 t(s)


Three different models of airbags were tested using identical crash test dummies. Sensors measured the
velocity of the crash test dummy as a function of time, when the car crashed at 20 m/s into a brick wall.
The shown velocity vs, time graphs resulted. Time t = 0 is the moment the car crashes.

6. All three airbags are the same size and shape. Which one inflates most quickly?
(A) Airbag 1 (B) Airbag 2
(C) Airbag 3 (D) We cannot determine the answer from the given information.

7. Let amax denote the largest instantaneous acceleration that the crash test dummy experiences during the

crash. The best airbag is the one for which amax is as small as possible. Which airbag is best?
(A) Airbag 1 (B) Airbag 2
(C) Airbag 3 (D) We cannot determine the answer from the given information.
8. For airbag 2, which of the following graphs best represents the position of the crash test dummy as a function
of time? Let x = 0 m be the dummys position at time t = 0s.
A B C D
Position

Position

Position

Position
.01 time(s) .01 time(s) .01 time(s) .01 time(s)
(A) A (B) B (C) C (D) D
9. For airbag 2, approximately how much distance does the dummy cover between the moment the car crashes
and the moment the dummy first makes contact with the airbag?
(A) 0.2 m (B) 0.4 m (C) 0.6 m (D) 1.0 m

THE NEXT QUESTIONS REFER TO THE FOLLOWING PASSAGE


Recently, college teams from all over the country sent tennis players to participate in a series of experiments
conducted by the Physical Education Department of a major university. A variety of coaching methods was
used to improve the players serves, described below.
Experiment 1
Two groups of 50 tennis players worked on the speed of their basic serves for two weeks. one group consisted
solely of right-handed players; the other consisted solely of left-handed players. Half of each group watched
videos of a right-handed tennis coach. Each player was told to pattern his or her serve on that of the coach in
the video. The players received no verbal or physical guidance. the average speed of each players serve was
measured at the beginning and end of the two-week period, and changes were recorded in Table 1.

Table 1
Players' Coach's Average change in
handedness handedness speed (mph)
Right Right 5
Right Left 2
Left Right -1
Left Left 8

Experiment 2
For two weeks, a second group of 100 right-handed tennis players watched the same videos of the right-
handed tennis coach. The coach also physically guided 50 of the those players through the motions of the
serve. Again, no verbal instruction was given during the experiment. The average speed and accuracy of each
players serves were recorded at the beginning and end of this two-week period. The results are recorded in
Table - 2.

KINEMATICS www.physicsashok.in 60
KINEMATICS

Table 2
Average Change in Average change in
Guided speed (mph) Accuracy
No 5 15%
Yes 9 25%
Experiment 3
For two weeks, a third group of 100 right-handed tennis players worked on their basis serves. 50 players
received no verbal instruction; they watched the same video of the right handed tennis coach, who also
physically guided them through the motions of the serve. The other 50 players did not observe the video but
received verbal instruction from the coach, who then physically guided them thought the motions of the serve.
The results are shown in Table 3.

Table 3
Average Change in
Guided Plus
speed (mph)
Video 7
Verbal
Coaching 10

10. Which of the following results would the expected if Experiment 3 were repeated using left-handed tennis
players and a left-handed coach
(A) The average service accuracy of all the players would increases by at least 30%.
(B) The average service speed of all the players would decrease slightly.
(C) Verbal coaching would improve average service speed less than would watching the video.
(D) The average service speed of the players who watched the video would increase by at least 8 mph.

11. Which of the following conclusions could NOT be supported by the results of Experiment
(A) Imitating someone whose handedness is the opposite of ones own will cause ones skills to deteriorate.
(B) Left-handed people are better than right-handed people at imitating the movement of someone with
similar handedness.
(C) People learn more easily by observing someone with similar handedness than by observing someone
with handedness opposite their own.
(D) Right-handed people are better than left-handed people at imitating the movement of someone whose
handedness is opposite their own.

12. Which of the following hypotheses is best supported by the results of Experiment 2
(A) Instructional videos are more helpful for right-handed tennis players than is verbal instruction.
(B) Instructional videos are more helpful for left-handed tennis players than for right-handed tennis players.
(C) Physical guidance by a coach improves both speed and accuracy of service for right-handed tennis
players.
(D) Physical guidance by a coach improves service accuracy for right-handed tennis players more than for
left-handed players.

13. Suppose 50 left-handed tennis players watch a video of a left-handed coach and are also physically guided
by that coach. The results of the experiments suggest that the players average change in service speed will
most closely approximate
(A) 1 mph (B) +5 mph (C) +8 mph (D) +12 mph

14. Which of the following hypotheses is best supported by the results of Experiment 1 alone
(A) Tennis players improve less by observing coaches whose handedness is the opposite of their own than
by observing those with similar handedness.
(B) Right-handed tennis players are coached by left-handed coaches more frequently than left-handed
players are coached by right-handed coaches.
(C) Right-handed coaches are better models for all tennis players than are left-handed coaches.
(D) People learn much better from physical contact plus a visual stimulus than from the visual stimulus
alone.

KINEMATICS www.physicsashok.in 61
KINEMATICS
THE NEXT QUESTIONS REFER TO THE FOLLOWING PASSAGE
It is known that if a steel ball and a feather are dropped from the same height, the steel ball falls faster than
the feather. The following scientists have two different views on falling bodies:
Scientist 1 :
The force of gravity makes things fall. The greater the gravitational force on an object, the faster it falls. The
steel ball falls faster because it is more massive than the feather, and thus is attracted more strongly by the
earths gravitational field.
Scientist 2 :
The mass of an object does not determine how fast an object falls, but shape does. Two identical pieces of
paper will fall at different speeds, if one of them is crumpled into a small ball. That is because gravity is not
the only force acting on a falling object. Air buoys up objects falling through it. Since the shape of the steel
ball gives it less air resistance than a feather, it falls faster.

15. According to Scientist 1, the property that determines how fast an object will fall is its
(A) chemical composition (B) mass (C) shape (D) gravity

16. According to Scientist 1, the speed of the falling ball could be increased by
(A) dropping the ball from a greater height (B) making the ball out of aluminum
(C) reshaping the ball (D) using a larger, steel ball

17. According to Scientist 2, a crumpled piece of paper falls at a different speed from a flat piece of paper
because of its
(A) mass (B) air resistance (C) gravitational attraction (D) texture

18. Both scientists agree that the rate at which an object falls is affected by the
(A) force of gravity (B) mass of the object
(C) objects resistance of air (D) shape of object

19. Scientist 2 would predict that in a vacuum, two objects would fall at the same speed if they had the same
(A) shape and different masses (B) air resistance and shape
(C) composition and air resistance (D) mass and different shapes

20. Suppose a small ball and a feather having the same weight are dropped from the same height. Which of the
following would Scientist 1 predict?
(A) the feather would fall faster than the ball (B) the ball and feather would fall at the same speed.
(C) the ball would fall faster than the feather (D) none of the above.

21. Two balls of identical shape, one made of lead and one made of aluminum, are both dropped from the same
height. Which scientists prediction and reasoning is incorrect based on the arguments presented?
(A) Scientist 1 : the lead ball falls faster because the balls have different masses.
(B) Scientist 1 : the lead ball falls faster because the balls have different gravitational attraction.
(C) Scientist 2: The lead ball falls faster because the balls have the same air resistance.
(D) Scientist 2: The balls fall at the same rate because they have the same air resistance.

THE NEXT QUESTIONS REFER TO THE FOLLOWING PASSAGE


A student performs an experiment to determine how the range of a ball depends on the velocity with which
it is released. The range is the distance between where the ball lands and where it was released, assuming
it lands at the same height from which it was released.
In each trial, the student uses the same baseball, and launches it at the same angle. Table 1 shows the
experimental results.

Table 1
Trial Launch speed (m/s) Range (m)
1 10 8
2 20 31.8
3 30 70.7
4 40 122.5

KINEMATICS www.physicsashok.in 62
KINEMATICS

Based on this data, the student then hypothesizes that the range, R, depends on the initial speed, 0 ,
according to the following to the following equation: R Cv0n , where C is a constant, and n is another
constant.

22. Based on this data, the best guess for the value of n is
1
(A) (B) 1 (C) 2 (D) 3
2

23. The student speculates that he constant C depends on:


I. The angle at which the ball was launch.
II. The balls mass
III. The balls diameter
If we neglect air resistance, then C actually depends on
(A) I only (B) I and II (C) I and III (D) I, II and III.

24. The student performs another trial in which the ball is launched at speed 5.0 m/s. Its range is approximately
(A) 1.0 meters (B) 2.0 meters (C) 3.0 meters (D) 4.0 meters

25. Let denote the angle of the balls initial velocity, as measured from the horizontal. Neglect air resistance.
At the peak (highest point) of its trajectory, the balls speed is
(A) 0 (B) 0 sin (C) 0 cos (D) 0

26. For trial 2, which of the following graphs best represents the vertical component of the balls velocity as a
function of time, assuming upward is positive?
+ + + +
vy vy vy vy
0 0 0 0
(A) time (B) time (C) time (D) time

THE NEXT QUESTIONS REFER TO THE FOLLOWING PASSAGE


Aristotle developed a system of physics based on what he thought occurred in nature. For example, he
thought that if a stone is released from rest, it instantaneously reaches a speed that remains constant as the
stone falls. He also believed that the speed attained by a stone falling in air varies directly with the weight of
the stone. A 5pound stone, for example, falls with a constant speed 5 times as great as that of a 1-pound
stone. Aristotle also noted that stones dropped into water continue to fall, but at a slower rate than stones
falling through air. To account for this, he explained that the resistance of the medium through which an object
falls also affects the speed. Therefore, he said, the speed of a falling object also varies inversely with the
resistance of the medium, and this resistance is the same for all objects.
Galielo disagreed with Aristotles explanation. He generated the following arguments to refute. Aristotle.
Consider a stake partially driven into the ground and a heavy stone falling from various heights onto the stake.
If the stone falls from a height of 4 cubits, the stake will be driven into the ground, say, 4 finger breadths. But
if the stone falls from a height of 1 cubit, the stake will be driven in a much smaller amount. Certainly, Galileo
argued, if the stone is raised above the stake by only the thickness of a leaf, then the effect of the stones
falling on the stake will be altogether unnoticeable.
On the basis of a careful set of experiments, Galielo argued that the speed of an object released from rest
varies directly with the time of fall. Also, the distance the object falls varies directly with the square of the time
of fall if the effect of air resistance on the object is negligible. Thus, according to Galileo, objects actually fall
with constant acceleration, and if air resistance is negligible, all object exactly the same acceleration.

27. Which graph accurately represents Galileos theory of the relationship between speed and time for an object
falling from rest under conditions of negligible air resistance

KINEMATICS www.physicsashok.in 63
KINEMATICS

Speed

Speed

Speed

Speed
(A) (B) (C) (D)

Time Time Time Time

28. A book dropped from a height of 1 meter falls to the floor in t seconds. To be consistent with Aristotles views,
from what height, in meters, should a book 3 times as heavy be dropped so that it will fall to the floor in the
same amount of time
(A) 1 9 (B) 1 3 (C) 1 (D) 3

29. Suppose a heavy object falls to the ground in t seconds when dropped from shoulder height. According to
Galileo, if air resistance were negligible, how many seconds would it take an object half as heavy to fall to the
ground from the same height
(A) 0.5 t (B) 1.0 t (C) 1.5 t (D) 2.0 t

30. A piece of putty weighing 2 pounds is dropped down a shaft from the top of a tall building. 1 second later, a 3
pound piece of putty si dropped down the shaft. According to Aristotle, what happens to the 2 pieces of putty
if they fall for a relatively long time
(A) The separation between the 2 piece constantly increases until they strike the ground.
(B) The separation between the 2 pieces is constant until the strike the ground.
(C) The heavier piece catches up to the smaller piece. and the 2 pieces travel together with the speed of the
heavier piece.
(D) The heavier piece catches up to the smaller piece, and the 2 piece travel together with a speed faster
than the speed of either.

THE NEXT QUESTIONS REFER TO THE FOLLOWING PASSAGE


Abhishek and Sweta Bachhan are two friends Abhishek is Joint to IIT-Pawai and Sweta come to see off
him at Indore railway station. Non Abhishek drops ball 1 from 1 mt height, just when the train starts to move.
Abhishek observes that ball doesnt hit the floor point exactly below the dropped point, it gets deviated by 20
cm. For 50 seconds train accelerates and attains the maximum speed and starts moving with this velocity.

50 m 50 m

Initial Final

Now Abhishek drops ball 2 at the same time (t = 50 sec.) Abhishek throw ball 3 such that it just reach Sweta.

31. Path of ball 1 seen by Sweta will be


(A) Parabolic (B) Straight line (C) Circle (D) Cant predict

32. Path of ball 1 seen by Abhishek will be


(A) Parabolic (B) Straight line (C) Circle (D) Cant predict

33. The acceleration of train is


(A) g/2 (B) g/4 (C) g/5 (D) Cant predict

34. The path of ball 2 as seen by Sweta will be


(A) Parabolic (B) Straight line (C) Circle (D) Cant predict

35. The path of ball 2 as seen by Abhishek will be


(A) Parabolic (B) Straight line (C) Circle (D) Cant predict

36. The speed with which Abhishek throw ball 3 will be


(A) 30 (B) 20 (C) 15 (D) None of these

KINEMATICS www.physicsashok.in 64
KINEMATICS
THE NEXT QUESTIONS REFER TO THE FOLLOWING PASSAGE
A circus wishes to develop a new clown act. Figure 1 shows a diagram of the proposed setup. A clown will
be shot out of a cannon with velocity v0 at a trajectory that makes an angle 45 with the ground. At this
angle, the clown will travel a maximum horizontal distance. The cannon will accelerate the clown by applying
a constant force of 10,000 N over a very short time of 0.24 sec. The height above he ground at which the clown
begins his trajectory is 10 m.
A large hoop is to be suspended from the ceiling by a massless cable at just the right place so that the
clown will be able to dive through it when he reaches a maximum height above the ground. After passing
through the hoop he will then continue on his trajectory until arriving at the safety net.
vy (m/s)
42

28

21 40
60 kg
80 kg
kg
45
t(s)
Figure 1 2.1 2.8 4.2
Figure 2
Figure 2 shows a graph of the vertical component of the clowns velocity as a function of time between the
cannon and the hoop. Since the velocity depends on the mass of the particular clown performing the act, the
graph shows data for several different masses.
37. If the angle the cannon makes with the horizontal is increased from 45, the hoop will have to be:
(A) moved farther away from the cannon and lowered.
(B) moved farther away from the cannon and raised.
(C) moved closer to the cannon and lowered.
(D) moved closer to the cannon and raised.

38. If the clowns mass is 80 kg, what initial velocity v0 will he have as he leaves the cannon?

(A) 3m s (B) 15 m s (C) 30 m s (D) 300 m s

39. The slope of the line segments plotted in figure 2 is a constant. Which one of the following physical quantities
does this slope represent?
(A) g (B) v0 (C) y y0 (D) sin

40. From Figure 2, approximately how much time will it take for clown with a mass of 60 kg to reach the safety
net located 10 m below the height of the cannon?
(A) 4.3 s (B) 6.4 s (C) 5.9 s (D) 7.2 s

41. If the mass of a clown doubles, his initial kinetic energy, mv02 2 , will:
(A) remain the same (B) be reduced to half
(C) double (D) four times

42. If a clown holds on to the hoop instead of passing through it, what is the expression for minimum length of the
cable so that he doesnt hit is head on the ceiling as he swings upward?
2v02 v02 v02 v02
(A) (B) (C) (D)
g g 2g 4g

KINEMATICS www.physicsashok.in 65
KINEMATICS

Level # 2
M O TI O N I N O N E D I M E N SI O N
1. Two intersecting straight lines moves out parallel to themselves with the speed v 1 and v 2. Calculate the
speed of the point of intersection of the lines if the angle between them is .

2. A base ball player hits a pitched ball so that its velocity reverses direction and its speed changes from 30
m/s to 40 m/s. The bat moves at an average velocity of 30 m/s. and it is in contact with the ball for a
distance of 0.05 m.
(A) For how long are the bat and ball in contact ?
(B) What is the average acceleration of the ball while it is in contact with the bat ?

3. A car starts from rest with an acceleration of 6 m/s2 which decreases to zero linearly with time, in 10 sec.,
after which car continues at a constant speed. Find the time required for the car to travel 400 m from the
start.
4. The speed of a motor launch with respect to the water is v = 7 m/s, the speed of the stream u = 3 m/s. When
the launch began traveling upstream, a float was dropped from it. The launch travelled 4.2 km upstream, turned
about and caught up with the float. How long is it before the launch reaches the float again ? Assume that float
is moving with the speed of stream.

5. The speed of a train increases at a constant rate from zero to v and then remains constant for an interval
and finally decreases to zero at a constant rate . If is the total distance described, prove that the total
v 1 1
time taken is t = . At what value of v is the time of travel the shortest ? What is the value of the
v 2
shortest time ?

6. A passenger is running at his maximum velocity of 8 m/s to catch a train. When he is a distance d from the
nearest entry to the train, the train starts from rest with constant acceleration a = 1 m/s2 away from the
passengers.
(A) If d = 30 m and he keeps running, will he be able to jump onto the train.
(B) Sketch the position function x(t) for the train, choosing x = 0 at t = 0. On the same graph sketch x (t)
for the passenger for various value of initial separation distance d, including d = 30 m and the critical
separation distance dc, such that he just catches the train,
(C) For the critical separation distance d0, what is the speed of the train when the passenger catches it ?
What is its average speed for the time interval from t = 0 until he catches it ? What is the value of dc.

7. The acceleration of a particle is given by a = 4t 30, where a is in meters per second squared and t is in
seconds. Determine the velocity and displacement as functions of time. The initial displacement at t = 0 is
s0 = 5m, and the initial velocity is v 0 = 3 m/s. a ,m/s
2
x

8. The acceleration of a particle that moves in the positive 0.4


x-direction varies with its position as shown. If the velocity
of the particle is 0.8 m/s when x = 0, determine the velocity 0.2
v of the particle when x = 1.4 m.

0
0 0.4 0.8 1.2

9. A particle starts from rest at x = 2 m and moves v, m/s


along the x-axis with the velocity history shown.
3
Plot the corresponding acceleration and displacement
histories for the two seconds. Find the time t when
the particle crosses the origin. 0 2.0
0 0.5 1.0 1.5
-1

KINEMATICS www.physicsashok.in 66
KINEMATICS
MOTION UNDER GRAVITY
10. One student throws a ball vertically upward with an initial speed of 9.8 m/s. Another student standing. 5m
away starts running towards the ball on release and catches it at the same height. What was the students
acceleration ? (Assume a uniform acceleration).

11. An aluminium ball with a mass of 4 kg and an iron ball of the same size with a mass of 11.6 kg are dropped
simultaneously from a height of 49 m.
(A) Neglecting air resistance, how long does it take the aluminium ball to gall to the ground.
(B) How much later does the heavier iron ball strike the ground ?

12. Ball A is dropped from the top of a building at the same instant that ball B is thrown vertically upward from
the ground. When balls collide, they are moving in opposite directions and the speed of A is twice the speed
of B. At what fraction of the height of the building did the collision occur ?

13. A ball is thrown vertically upward from the 12 m level in an elevator shaft with an initial velocity of 18 m/s. At
the same instant an open platform elevator passes the 5m level, moving upward with a constant velocity of
2 m/s. Determine (A) When and where the ball will hit the elevator, (B) the relative velocity of the ball with
respect to the elevator when the ball hits the elevator.

14. An object is thrown upward with an initial velocity v 0. The drag on the object is assumed to be proportional
to the velocity. What time will it take the object to move upward and what maximal altitude will it reach ?

15. A nut comes loose from a bolt on the bottom of an elevator as the elevator is moving up the shaft at 3 m/s.
The nut strikes the bottom of the shaft in 2 sec.
(A) How far from the bottom of the shaft was the elevator when the nut fell off.
(B) How far above the bottom was the nut 0.25 s after it fell off ?

16. A helicopter is descending vertically downward with a uniform velocity. At a certain instant, a food packet is
dropped from it which takes 5 seconds to reach the ground. As this packet strikes the ground, another food
packet is dropped from it, which takes 4 seconds to reach the ground. Find the velocity with which the
helicopter is descending and its height, when second packet is dropped. Also find the distance travelled by
the helicopter during the interval of dropping the packets.

RELATIVE MOTION
17. Two bodies moves in a straight line towards each other at initial velocities v 1 and v 2 and with constant
accelerations a1 and a2 directed against the corresponding velocities at the initial instant. What must be the
maximum initial separation max. between the bodies for which they meet during the motion ?

18. The velocity of a ship in still water is 20 km/h. What is the velocity of a motor boat approaching the ship at
right angle to its course if it appears to people on board the ship, that the motor boat heads towards the ship
at 60 ?

19. A man running on a horizontal road at 8 km/h, find the rain falling vertically. He increase the speed to 13 km/
h and find the drops make angle 30 with the vertical. Find the speed and the direction of the rain with
respect to the read.

20. A person riding in the back of a pickup truck traveling at 60 km/h. On a straight, level road throws a ball with
a speed of 20 km/h relative to the truck in the direction opposite to its motion.
(A) What is the velocity of the ball relative to a stationary observer by the side of the read ?
(B) What is the velocity of the ball relative to the driver of a car moving in the same direction as the truck
at a speed of 90 km/h. ?

KINEMATICS www.physicsashok.in 67
KINEMATICS
21. The speed of a boat in still water is v. The boat is to make a round trip in a river whose current travels at
speed u. Derive a formula for a time needed to make a round trip of total distance D if the boat makes the
round trip by moving
(A) Upstream and back down stream.
(B) Directly across the river and back. We must assume u < v ; Why ?
O

22. In the adjacent figure, a vertical cemented circular plane of


radius R having frictionless slots along the chords OA, OB,
OC, OD and DE is shown. OGC is the diameter of the circle.
G
Five Boys starts sliding down from rest along the slots OA,
OB, OC, OD and OE simultaneously. Find the time taken by A E
each boy to reach at the point A, B, C, D and E. D
B C

PROJECTILE MOTION
23. A body is dropped from a stationary balloon at a height h above the ground. At the same time a bullet is fired
from a gun on the ground with a velocity u. If the angle of elevation of the balloon from the position of the gun
is , in which direction should the bullet be fired so that it strikes the body before reaching the ground. Also
find the minimum value of u required for this.

24. Particles P and Q of mass 20 gms and 40 gms respectively are simultaneously projected from points A and
B on the ground. The initial velocities of P and Q makes 45 and 135 angles respectively with the horizontal
as shown in the figure. Each particle has in
initial speed of 49 m/s. The separation AB is 245 m.
Both particles travel in the same vertical plane and
under go a collision. After the collision P retraces its
path. Determine the position of Q when it hits the 45 135
ground. How much time after the collision does the
particle Q take to reach the ground. Take g = 9.8 m/s2. A 245 m B

25. A boy throws a ball horizontally with a speed of v0=12 m/s


v 0 = 12 m/s from the gandhi setu bridge c of patna
in an effort to hit the top surface AB of a truck
travelling directly underneath the boy on the bridge. s 8m
If the truck maintains a constant speed u = 15 m/s, B A
u
and the ball is projected at the instant B on the top C
of the truck appears at point C, determine the pos-
ition s where the ball strikes the top of the truck.
10m
u
26. Two inclined planes intersect in a horizontal plane. Their
inclinations to the horizontal being and . If a particle P
a
is projected at right angle to the former from a point in it
so as to strike the other at right angles, then find the
velocity of projection.
O

KINEMATICS www.physicsashok.in 68
KINEMATICS

27. A cannon fires from under a shelter inclined at an angle


to the horizontal. The canon is located at a point at a
distance from the base of the shelter. The initial velo-
city of the shell is v 0 and its trajectory lies in the plane
of the figure. Determine the maximum range of the shell.

28. A juggler manages to keep five balls in motion, throwing each sequentially up a distance of 3m. (a) Determine
the time interval between successive throws. (B) Give the positions of the other balls at the instant when
one reaches her hand. (Neglect the time taken to transfer balls from one hand to the other.)

29. A ball is shot from the ground into the air. At a height of 9.1 m., its velocity is observed to be v 7.6 i 6.1j
in water per second ( i horizontal, j upward).
(A) To what maximum height does the ball rise ?
(B) What total horizontal distance does the ball travel ? What are
(C) The magnitude and
(D) The direction of the balls velocity just before it hits the ground ?

30. A bomb bursts on contact with the ground and pieces fly off in all directions with speed up to 30 m/s. A girl
is standing 40 m away. What is the time duration over which she can be hit by a piece ?

31. A boy sitting at the rear end of a railway compartment of a train running at a constant acceleration a on
horizontal rails fires a shot towards the fore end of the compartment with a muzzle velocity u = 20 m/s at an
angle = 37 above the horizontal when the trains velocity v = 10 m/s. If the boy catches the shot without
moving from his seat at the same height as that of projection find (a) Speed of the train at the time when he
catches the shot and (B) the acceleration of the train (g = 10 m/s2).
y
32. A projectile of mass m is fired into a liquid at an angle 0 with
v0
an initial velocity v 0 as shown. If the liquid develops a frictional
or drag resistance on the projectile which is proportional to its 0
x
velocity. i.e., F = kv, where k is a positive constant, determine
the x and y components of its velocity at any instant. Also, what
is the maximum distance x max. that it travels ?

33. An elevator is going up with an upward acceleration of 1 m/s2. At the instant when its velocity is 2 m/s, a stone
is projected upward from its floor with a speed of 2 m/s relative to the elevator, at an elevation of 30.
(A) Calculate the time taken by the stone to return the floor.
(B) Sketch the path of the projectile as observed by an observer outside the elevator.
(C) If the elevator was moving with a downward acceleration equal to g, how would the motion be altered ?


34. A particle is moving in the plane with velocity given by, u u0 i a cos t j , where i and j are unit vectors
along x and y axes respectively. If particle is at the origin at t = 0 ;
3
(A) Calculate the trajectory of the particle. (B) Find its distance from the origin at time .
200

35. A man is travelling on a flat car which is moving up a plane inclined at cos1 (4/5) to the horizontal with a
speed 5 m/s. He throws a ball towards a stationary hoop located perpendicular to the inclined in such a way
that the ball moves parallel to the slope of the incline while going through the hoop. If the distance d of the
hoop from the level of the mans hand is such that its component perpendicular to the incline is 4 m,
calculate the time taken by the ball to reach the hoop.

KINEMATICS www.physicsashok.in 69
KINEMATICS
36. A rifleman on a train moving with a speed of 60 km/hr. fires at an object running away from the train at right
angles with a speed of 45 km/hr. If the line connecting the man and the object makes an angle of 30 to the
train at the instant of shooting to what angle to the train should be aim in order to hit the object if the muzzle
velocity is 850 km/hr. ?
37. Water is ejected from the water nozzle with a speed v 0 = 14 m/s. For what value of the angle will the water
land closest to the wall after clearing the top ? Neglect the effects of wall thickness and air resistance.
Where does the water land ?
38. A bullet is projected so as to graze the tops of two walls each of height 20 m located at distances of 30 and
170 m in the same line from the point of projection as shown in figure. Find the angle and the speed of
projection of the bullet.

V0 P1 P2
O
20 m 20 m
30 m 170 m
30 B

39. Two guns are projected at each other, one upward


at an angle of 30 and the other at the same angle
of depression, the muzzles being 30 m apart as 30 m
shown in figure problem. If the guns are shot with P
velocities of 350 m/s upward and 300 m/s down- y
ward respectively, find when and where the bullets 30
may meet. A
x

Bomber Line of Flight


40. A bomber is flying horizontally at a speed of 500 km/h
at an altitude of 3 km such that a ship lies in a vertical
plane through the line of sight as shown in figure. Trajectory
3 km
Determine the angle of the line of sight of the bomber
Line of Sight
with the ship at the instant a bomb is released so as
to hit the ship. Where would the bomber be at the
instant the ship is wrecked ?
ship

41. Two guns are pointed at each other one upwards at an angle of elevation 30 and the other downwards at the
same angle of depression, the muzzles being 42 m apart. If the charges leave the guns with velocities 400
m/s and 300 m/s respectively, find when and where they meet.
42. A particle is projected up an inclined plane of inclination with an initial velocity u at an angle to the
horizontal. Find the maximum distance of the projectile from the inclined plane.
43. On a frictions horizontal surface, assumed to be the x-y plane, a small y
trolley A is moving along a straight line parallel to the y-axis (see figure)
with a constant velocity of 3 1 m s . At a particular instant, when the A
line OA makes an angle of 45 with the x-axis, a ball is thrown along the
surface from the origin O. Its velocity makes an angle with the x-axis
and it hits the trolley. 45
x
(a) The motion of the ball is observed from the frame of the trolley. O
Calculate the angle made by the velocity vector of the ball
with the x-axis in this frame.
4
(B) Find the speed of the ball with respect to the surface, if . [IIT 2002]
3

KINEMATICS www.physicsashok.in 70
KINEMATICS

44. Two guns, situated on the top of a hill of height 10 m, fire one shot each with the same speed 5 3 ms1 at
some interval of time. One gun fires horizontally and other fires upwards at an angle of 60 with the horizontal.
The shots collide in air at a point P. Find
(a) the time-interval between the firings, and (b) the coordinates of the point P. [IIT 1996]
Take origin of the coordinate system at the foot of the hill right below the muzzle and trajectories in x-y
plane.

45. Two towers AB and CD are situated a distance d apart as shown


in figure AB is 20 m high and CD is 30 m high from the ground. An 2m
object of mass m is thrown from the top of AB horizontally with a C
60
velocity of 10 m s towards CD.
Simultaneously another object of mass 2 m is thrown from the A
top of CD at an angle of 60 to the horizontal towards AB with
the same magnitude of initial velocity as that of the first object.
The two objects move in the same vertical plane, collide in
mid-air and stick to each other.
(i) Calculate the distance d between the towers and, d
B D
(ii) Find the position where the objects hit the ground.

Answer Key
Assertion & Reason
Que. 1 2 3 4 5 6 7 8 9 10 11

Ans. C D A A A A C B D E E

L ev e l 1
Q. 1 2 3 4 5 6 7 8 9
A n s. A A A A C D B AB C
Q. 10 11 12 13 14 15 16 17 18
A n s. C B C B C B C B A
Q. 19 20 21 22 23 24 25 26 27
A n s. C B A B B A B A B
Q. 28 29 30 31
A n s. AD ABC ABC ABD

Fill in the Blanks / TrueFalse / Match Table

1. 2 r , r 2. d v 3. 0.6 4. a 2 4b , tan 1 a
5. T 6. T 7. F 8. F 9. A

KINEMATICS www.physicsashok.in 71
KINEMATICS

Passage Type
Que. 1 2 3 4 5 6 7 8 9 10 11
Ans. B C D A A A B C A D A
Que. 12 13 14 15 16 17 18 19 20 21 22
Ans. C D A B D B A B B C C
Que. 23 24 25 26 27 28 29 30 31 32 33
Ans. A B C A A D B D B B C
Que. 34 35 36 37 38 39 40 41 42
Ans. A B D D C A C B D

Level # 2
M O TI O N I N O N E D I M E N SI O N
1. V v 12 v 22 2 v 1v 0 cos . cos ec 2. (a) 0.001667 s (b) 42000 m/s2 3. 16.678

2 ( )
4. 5. t min. = 6. (a) yes (b) 8 m/s, 4 m/s 32 m 7. v = 3 30t + 2t 2, S = 5 + 3t
( v u) 2
15 t 2 + 2/3 t 3
8. V = 1.166 m/s 9. t = 0.917 s

MOTION UNDER GRAVITY

10. 2.5 m/s2 11. 3.2 s, O 12. 2/3 13. (a) 3.65 s, 12.30 m (b) 19.81 m/s

1 r v0 v2
14. t m = r n 1 g , h = 0 15. (a) 13.6 m (b) 14 m
2g
16. 11.04 m/s downward at 177.8 k height, 55.2 m downward.

RELATIVE MOTION
( v 1 v 2 )2
17. max 18. 34.6 km/h 19. 4 7 km/h
2 (a1 a 2 )
DV D
20. (a) 40 km/h (b) 50 km/h 21. (a) (b)
v u2
2
v u2
2

R
22. t = 2 g , every boy will take same time

PROJECTILE MOTION
gh
23. , v min. = .cosec 24. 122.5 m, 3.53 sec. 25. 3.84 m
2

1 g sin 2
v 02
2 ag
26. u sin
27. g sin 2 sin v0
sin sin cos( )

KINEMATICS www.physicsashok.in 72
KINEMATICS

28. (a) 310 m/s (b) 1.9 m and 2.9 m above the hands

29. (a) 11m (b) 23 m (c) 17 m/s (d) 63 below horizontal. 30. 4.6 sec 31. 42 m/s

m k k / m
32. v x = v 0 cos . e( km) t, v y = k m v 0 sin g e t g , x = mv 0 cos 0 33. (a) 0.18 sec
m k

x 9 2 u02 a 2
34. (a) y = a sin u (b) 35. 1 sec. 36. 34.5'
0 4 2
37. = 50.7, 0.835 to the right of B 38. 48 m/s 39. 0.0462 s, (14 m, 8.07 m)

40. 48.9, over the ship.

41. In 0.068 s, at a point whose horizontal of vertical dist. from the 1st gun are 20.78 and 11.98 m

u 2 sin 2 ( )
42. s
2g cos
43. (a) 45 (b) 2 m/s. 44. (a) t 1 sec ond
(b) 5 3 m, 5m
45. 17.32, 11.547 m from B

XXXX

KINEMATICS www.physicsashok.in 73
DYNAMICS

DHANALAKSHMI NAGAR

NEAR ANNAMAIAH CIRCLE,

TIRUPATI.

PH NO. 9440025125
DYNAMICS
THEORY OF LAWS OF MOTION
REFERENCE FRAME
An axis is a number line. Origin is the point where zeros of the axes coincide. Position of an object at a time is
described by drawing one axis for motion along a straighline, two intersecting axes for motion in a plane and
three perppendicular axes for motion in space. More than one axis is a system of axes. At any instant of time,
the position of a particle is described by noting co-ordinates on axes.
To describe motion, we need an axis system (frame) that is fixed on some body, called reference body. This
axis system is called reference frame. If we sit in an aeroplane and look at a car moving on road, the
aeroplane in the reference body and the car is the object in motion. To locate the car, we set up xyz-axes in the
aeroplane. This frame is here the reference frame. A reference frame is also attached with a clock to measure
time.
Example : A car is moving on road. Mile stones give us the positions of the car. We can describe it by choosing a
point as origin and laying down number line along the road. (Observer is on the road.) This number line is called
an axis and is a reference frame.
mile stones

4 5 6 7
1 2 3
3 2 1
Curved Axis
O (Origin)
Position (at time t)

Example : The coint on a carrom board may be located at any time by drawing two axes in the plane of the board.
Two numbers x and y locate it at a moment t.
Y

Origin (x, y)
y
x
X

Example : An insect (P) flying in a room may be located at a time by drawing three axes along the edges of the
room, meeting at a corner. Insect P has three co-ordinates (x, y, z) at time t.
Z

P(x, y, z)
Origin
x Y
y
X
In all the examples above, axis systems are reference frames as they are fixed in the body at which rests the
observer. The frame fixed to the ground is called ground frame that fixed to a car is called car frame, that
fixed to the sun is called heliocentric frame etc.

www.physicsashok.in 1
DYNAMICS

CONCEPT OF FORCE
Force is familiar word in science. From your own experience, you know that forces can produce changes in
motion. If a force is applied to a stationary body, the body comes in motion. It can speed up and slow down a
moving body or change the direction of its motion. In nut shell, the force is cause of change in velocity of the
body. In other words, force is the cause of acceleration of the body.
Force is a vector quantity and if more than one forces act on a particle, we can find the resultant force using the
laws of vector addition.
When a heavy block is suspended by a rope, the rope exerts a force on the block to hold it and the block
exerts a force on the rope to make it tight and stretched. This is according to Newtons third law of motion
which may be stated as followed :

If a body A exerts a force F on another body B, then B exerts a force F on A, the two forces acting along
the same line.

Some Particular Forces


(i) Gravitational force : The force of attraction between bodies by virtue of their masses is known as gravitational
force.
Let two blocks of mass m1 and m2 are separated by a distance r.

The force on block 1 by block 2 is F12 acting towards m2 along line joining m1 and m2. Similarly, the force on

block 2 by block 1 is F21 acting towards m1 along line joining m2 and m1 (as shown in figure)
From the concept of Newtons third law, F12 F21
m1 m2
F12 F21
r
In the sense of magnitude,
Gm1m 2
F12 F21 F
r2
Here, G = gravitational constant = 6.7 1011 Nm2/kg2.
(ii) Weight of body (mg) : It is defined as the force by which earth attracts a body
towards its centre. If body is situated either on the surface of earth or near the m
surface of earth, then gravitational acceleration is nearly constant and is equal to
mg
g = 9.8 m/s2. The force of gravity (weight) on a block of mass m is w = mg
acting centre of earth (shown in figure).
Caution : The weight of a body is not the mass of the body. Weight is the magnitude of force and is related to the
mass by w = mg.
(iii) The normal reaction force : When a body presses against a surface, the surface (even a seemingly rigid

surface) deforms and pushes on the body with a normal reaction force N that is perpendiuclar to the surface.
For a simple introduction to the normal reaction force, consider a situation in which you put a book on your
head and continue your stationary position. In this case, the pain you feel is due to the force that the book exerts
on your head. In the language of physics, the book exerts a force on your head normal to the surface of contact
in downward direction. According to Newtons third law of motion, the head exerts a force of same magnitude
on the book normal to the surface of contact in upward direction.
These forces are known as normal reaction forces.
Normal reaction forces in difference situations are shown below :
N

N
(a) Direction of normal Direction of normal
reaction on reaction on
the blocks the surface
www.physicsashok.in 2
DYNAMICS

N N
m m

N1
(b)
Inclined plane Direction of normal Direction of normal reaction Normal reaction on
reaction on the blocks on the inclined plane horizontal surface

The number of normal reaction pairs is equal to number of contact surfaces.

m2 B NB NB

(c) A m1 B A NA
NA

The normal reaction on upper block is in upward direction and normal reaction on lower block is in downward
direction.
N2 N1
Wall
(d) Sphere N1
N2

N2 N1

C C
(e) N1 N2

C N2
C

N2
(f) N1

N1

N2
N2
(g) N1 N1

(iv) Friction : If we slide or attempt to slide over a surface, the moton is


resisted by a bonding between the body and the surface. The resistance Direction of

is considered to be single force f , called the frictional force or simply attempted slide
friction. This force is directed along the surface, opposite the direction f
of the intended motion.
Some times to simplify a situation, friction is assumed to be negligible (the surface is frictionless).
(v) Tension : When a string or cord is attached to a body and pulled taut, the

cord pulls on the body with a force T directed away from the body and
T T
along the cord Fig. The force is often called a tension force because the cord Force
is said to be in a state of tension (or to be under tension), which means that it
is being pulled taut. The tension in the cord is the magnitude T of the force on the body.

www.physicsashok.in 3
DYNAMICS

If the cord (string) is massless, the tension in cord everywhere remains the same. The cord then exists only as
a connection between two bodies.
If the cord (string) is massive, the tension in it varies point to point.

NOTE : If string, slacks, tension in string becomes zero.


The direction of tension on a body or pulley is always away from the body or pulley.

The direction of tensions in some cases are shown below :


T T
(a) m2 m2 m2 m1

(b) m2 T T
String is massless T T
and pulley is m1 m2 m2
light and smooth
T T
m1 m1

String is massless T
and pulley is
(c) light and smooth T
m2
m2

T1
T1 T1

T1 T1
m1
m
T1 1
(d) Pulleys are light and
smooth and string
is massless T2
m1
T2
m2

T
T
T
m1 m1
(e) m2 T
m2

www.physicsashok.in 4
DYNAMICS

T1 T2

m1 T1
T2
(f) m2 m1
m2
String is massless
and there is friction
between string and pulley
T1 T1
m1 m1
C C
String is massless T2
and there is friction
(g) between pulley and string m2 T2
m2
If m2 tends to move downwards, then
T 2 > T1 and T2 = T1e
where = coefficient of friction
= angle subtended by string at the centre of pulley

T2 T1

m1 T3
(h) m2 m1 T4
String is not massless m2
and there is friction
between pulley and string
(vi) Spring force : Coiled metallic wire is known as spring. The distance between
two successive collisions in a spring remains the same. If a spring is placed on
a smooth surface, the length between ends of spring is known as natural length L
(shown in figure).
As you may have discovered itself, springs resist attempts to change their
length. If the length of spring is greater than its natural length, the spring is in the condition of elongation (shown
in figure ahead). If the length of spring is lesser than its natural length, the spring is in the condition of compression
(shown in figure ahead). In fact, the more you alter a springs length, the harder it resists. From this point of
view, spring force increases, when elongation or compression increases.
For small elongation or compression of spring, spring force is proportional to its elongation or compression.
i.e., Fx
F = kx
where k is proportionality constant known as spring constant or stiffness constant. Its unit is N/m. The
direction of spring force is always towards the natural length of spring.

L L+ x L x

Spring in natural Spring in the Spring in the


length condition of elongation condition of compression
(a) (b) (c)

www.physicsashok.in 5
DYNAMICS
L L+x L x

F=0 F = kx F = kx
m m m
No spring force Spring force on the Spring force on the
on body body is leftward body is rightward
(d) (e) (f)

FREE BODY DIAGRAM


A free body diagram is helpful in solving problems like several bodies connected by string, springs, surfaces of
contact, with the Newtons second law.
It is a striped-down diagram in which one body is considered. That body is represented by a sketch or simply
a dot. The external forces on the body are drawn, and a co-ordinate system is superimposed, oriented so as to
simplify the solution. While sketching a free body diagram, the following points should be kept in mind.
1. Normal reaction (N) always acts normal to the surface on which body is kept [Fig.].

N
N

Vertical surface
Block
Block Block
N

W Horizontal
surface W
Normal reaction = N
Weight (action) = w Inclined plane
(a) (b) (c)

2. When two blocks A and B are connected by a string the tension for block A is towards B and for block B, it
is towards A.
T T
B
A

NOTE : Always remember that while drawing a FBD, tension in each


branch must form an action-reaction pair.

3. If the pulley is light and frictionless and a string passes over it without any links, then tension on either side of the
string is the same.
4. While drawing FBD of a body, always take into account the forces which are acting on the body not those
which the body exerts on others.
5. The friction is a tangential force acting tangentially to the surfaces in contact.
A
Example 1. Three blocks A, B and C are placed one over the other as shown in figure.
Draw free body diagarms of all the three blocks. B
Sol. Free body diagrams of A, B and C are shown below :
C
N1 N2

wA wB wC

N1 N2 N3
FBD of A FBD of B FBD of C

www.physicsashok.in 6
DYNAMICS
Here, N1 = normal reaction between A and B
N2 = normal reaction between B and C
N3 = normal reaction between C and ground

Example 2. A block of mass m is attached with two strings as shown in figure. Draw the free body diagram of the
block.

Sol. The free body diagram of the block is as shown in fig.


T1

T2

mg

NEWTONS LAWS OF MOTION


First Law
If no force acts on a body, then the bodys velocity cannot change; that is the body cannot accelerate.
In other words, if the body is at rest it stays at rest and if it is moving, it will continue to move with the same
velocity (same magnitude and same direction) unless it experiences a net external force.
In simpler terms, we can say that the net force on Q body is zero, its acceleration is zero. That is, where
F = 0, then a = 0. From the first law. We conclude that an isolated body (a body that does not interact with
its environment) is either at rest or moving with constant velocity.

Second Law
The change of motion is proportional to the magnitude of force applied and is made in the direction of the
straight line in which that force is applied.
According to this law, the net force on a body is equal to the product of the bodys mass and the acceleration
of the body.
In equation form

Fnet
Fnet m a or a
m

Third Law
To every action there is always an equal and opposite reaction or the mutual actions of two bodies upon each
other are always directed to contrary parts.
According to this law, when two bodies interact, the forces on the bodies from each other are always equal in
magnitude and opposite in direction.
2 F12 = F21
This is, F12 F21
This law, which is illustrated in fig., is equivalent to stating that forces always F21 F12
occurs in pairs, or that a single isolated force cannot exist.
1
Problems Solving Steps :
Normally problem based on Newtons law can be solved in following steps :
1. Concentrate your mind on the system (body) which is considered by you. The considered body may be a
single particle, a block or a combination of two or more blocks, two blocks connected by a string. etc.

www.physicsashok.in 7
DYNAMICS
2. Show all forces acting on the system but it is restricted that the forces that the system exerts on other should not
be shown here.
3. Thus, we draw, a free body diagram of the system and indicate the magnitude and directions of all the forces as
discussed in step 2.
4. Choose a co-ordinate system including mutually perpendicular x and y axes in the plane of the forces.
Write the components of all the forcs acting along the acceleration (assumed as x-axis) and perpendicular to
acceleration (assumed as y-axis). That is
Fx = ma ...(i)
and for y-axis, Fy = 0 ...(ii)

NOTE : Above Eqs (i) and (ii) hold in case of coplanar forces.
If the forces are collinear, the Eq. (ii) is not needed.

Example 3. In the arrangement shown in figure, the strings are light


4kg F = 28 N
and inextensible. The surface over which blocks are placed is 2kg 1kg
smooth. Find :
(a) the acceleration of each block ; (b) the tension in each string.
Sol. (a) Let a be the acceleration of each block and T1 and T2 be the tensions, in the wo strings as shown in figure.
y
T2 T1 F = 28 N
4kg 2kg 1kg
x

Taking the three blocks and the two strings as the system.

4kg F = 28 N
2kg 1kg

Using Fx = max
or 28 = (4 + 2 + 1)a
28
or a 4 m / s2
7
(b) Free body diagram (showing the forces in x-direction only) of 4 kg block and 1 kg block are shown in
figure.
a = 4 m/s 2
y
T2 T1 F = 28 N
4kg 1kg x

Using Fx = max
For 1 kg block, F T1 = (1) (a)
or 28 T1 = (1) (4) = 4
T1 = 28 4 = 24 N
For 4 kg blocks,
T2 = (4) (a)
T2 = (4) (4) = 16 N

Example 4. Two blocks of mass 4 kg and 2 kg are placed side by side on a smooth
horizontal surface as shown in the figure. A horizontal force of 20 N is applied 4kg 2kg
on 4 kg block. Find the acceleration of each block.
www.physicsashok.in 8
DYNAMICS
Sol. Since, both the blocks will move with same acceleration (say a) in horizontal direction.
a
y
20 N
4kg 2kg
x

Let us take both the blocks as a system. Net external force on the system is 20 N in horizontal direction.
Using Fx = max
20 = (4 + 2) a = 6a
10
or a m / s2
3

Example 5. Find the acceleration of 3 kg mass when acceleration of 2 kg 10 N


3 kg 2 kg
mass is 2 ms2 as shown in figure.
2 ms 2
(A) 3 ms2 (B) 2 ms2 (C) 0.5 ms2 (D) Zero
Sol. From force diagram of 2 kg block :
3 kg 2 kg 10 N
10 kx = 2a = 4
kx = 6
2 ms 2
From force diagram of 3kg block, 2 kg 10 N
kx = 3a kx
or 6 = 3a a
3 kg kx
6
a 2m / s 2
3

Example 6. The system shown adjacent is in equilibrium. Find the acceleration of the K
blocks A, B & C all of equal masses m at the instant when (Assume springs
A
to be ideal)
B
(a) The spring between ceiling & A is cut.
(b) The string (inextensible) between A and B is cut. K
(c) The spring between B & C is cut. C
Also find the tension in the string when the system is at rest and in the above 3 cases.
kx1 kx3
Sol. The system is in equilibrium :
kx3 = mg ...(1) from fig.(1)
A+B C
2mg + kx3 = kx1 ...(2) from fig.(2)
3mg = kx1 kx3 2mg mg
Fig(2) Fig(1)
(a) when spring between ceiling and block is cut. then, elongation of spring between B and C remains same just
after cutting.
ac = 0 ( kx3 = mg) 2m
For (A + B) A+B
kx3 + 2mg = 3mg
3mg = 2ma
3 2
a = g = 15 m / s
2
T
a A = a B = 15 m / s 2
B
For tension,
mg + kx3 T = maB kx3 mg
www.physicsashok.in 9
DYNAMICS
3mg
mg + mg T =
2
mg
T=
2
(b) when string between A and B is cut. the elongation in springs do not change just
after cutting the string. kx1
mg kx1 = maA
mg 3mg = maA { kx1 = 3mg} A
2mg = maA mg
aA = 2g
aA = 2g (upward)
For B B
mg + kx3 = m aB mg kx3
or mg + mg = m aB { kx3 = mg}
a A = 2g (downward)
For C, kx3
mg kx3 = maC
C
or mg mg = maC { kx3 = mg}
aC = 0 mg
T = 0
(c)when spring between B and C is cut. C
mg = maC
aC = g (downward) mg

2mg kx1 = 2maB { aA = aB}


kx1
2mg 3mg = 2maB { kx1 = 3mg}
g A+B
aB = -
2
2mg
g
aA = aB = (upward)
2
T (mg) = maB
mg
T (mg) = T
2
mg B
T = mg +
2
mg kx3
3mg
T=
2

PULLEYS
Using pulling force method, the problems based on pulleys can be made simple.
We can understand this method using an example.
Let two enequal masses m and 2m are attached to the ends of a light inextensible a a
string passing over a smooth massless pulley.
We have to find the acceleration of the system. m 2m

mg 2mg
www.physicsashok.in 10
DYNAMICS
Net pulling force on the system is 2mg mg = mg and total mass being pulled
is 2m + m = 3m
Therefore, acceleration of the system
Net pulling force
a
Total mass to be pulled
mg g
a
3m 3

Motion of Connected Bodies


(i) Unequal masses (m1 > m2) suspended from a pulley :

m2g m1g m2g m1g


T T m2 m1 m1 + m2

m2
m1
m2g
m1g

Acceleration a
m1 m2 g
m1 m2
2m1m 2
Tension T g
m1 m 2
(ii) Bodies accelerated on a horizontal surface by a falling body :
T = m2a ...(i)
m1g T = m1a ...(ii)
N
a
m1 T
Acceleration a g m2
m1 m 2
Smooth
T
mm m2g
Tension T 1 2 g m1
m1 m 2
a
(iii) Motion on a smooth inclined plane :
m1g
m1g T = m1a ...(i) T
N
T m2g sin = m2a ...(ii) T
m2
m m 2 sin m1
a 1 g m2g sin
m1 m 2 m2gcos a
m1g
m1m 2 1 sin g m2g
and T
m1 m 2

Example 7. A rope of mass 5 kg is moving vertically in vertical position with an upwards force of 100 N acting at
the upper end and a downwards force of 70 N acting at the lower end. The tension at midpoint of the rope is
(A) 100 N (B) 85 N (C) 75 N (D) 105 N

www.physicsashok.in 11
DYNAMICS
100N
Sol. From force diagram of rope,
mg + 70 100 = ma
or 50 + 70 100 = 5a
or 20 = 5a m=5kg

a = 4 m/s2 a

Let tension at middle point is T. T mg 70N

From force diagram of half lower portion : 2.5g


25 + 70 T = 2.5a
or 95 T = 10 a
T = 95 10 = 85 N. 70N P
2.5g

Example 8. What should be the minimum force P to be applied to the string so that
block of mass m just begins to move up the frictionless plane.
M
Mg tan Mg cot Mg cos
(A) (B) (C) (D) None
2 2 1 sin
Sol. From force diagram of pulley, P
P + P cos = T
From force diagram of block : P
N T
T > mg sin T
or P + P cos > mg sin mgsin mgcos
mg sin
or P mg
1 cos
mg sin
Pmin
1 cos

Example 9. In the fig., mass of A, B and C are 1 kg, 3 kg and 2 kg


respectively. Find : C
B

(a) the acceleration of the system and


A

(b) tension in the string. Neglect friction. (g = 10 m/s2) 60 30


Sol. (a) In this case net pulling force
= mAg sin 60 + mBg sin 60 mCg sin 30

3 3 1
110 310 210 24.64 N
2 2 2
Total mass being pulled = 1 + 3 + 2 = 6 kg
Acceleration of the system
24.64
a 4.11 m / s 2
6 T1
(b) For the tension in the string between A and B. FBD of A
mAg sin 60 T1 = mAa A
T1 = mAg sin 60 mAa
a
T1 = mA (g sin 60 a)
mAg sin 60

www.physicsashok.in 12
DYNAMICS

3
T1 1 10 4.11 4.55 N
2
For the tension in the string between B and C.
FBD of C T2
a
T2 mCg sin 30 = mCa
T2 = mC (a + g sin 30) C

1
T2 2 4.11 10 18.22 N
2 mCg sin 30

Example 10. Two masses m and M are attached to the strings as shown in the
figure. If the system is in equilibrium, then
2M 2m
(A) tan 1 (B) tan 1
m M M
2M 2m 45 45
(C) cot 1 (D) cot 1
m M m
Sol. At point A,
2T sin 45 = mg
T1
mg mg
T
2sin 45 2 T B
At point B, T T M
T1 sin = Mg + T cos 45 45 45
Mg
A
mg
or T1 sin Mg cos 45 m
2
mg
mg
or T1 sin Mg
2
Also, T1 cos = T sin 45
mg mg
or T1 cos sin 45
2 2

mg
Mg
T1 sin 2 2M 1

T1 cos mg m
2
Example 11. In the figure shown the velocity of different blocks is shown. The velocity
of C is
(A) 6 m/s (B) 4 m/s
(C) 0 m/s (D) none of these
6m/s

6m/s

4m/s

Sol. A B C D
66 V1
Here v1 0 P1
2 P2
6m/s

Since, pulley P1 is in rest. so, pulley P1 is also in rest. B


A D
So, velocity of C is 4 m/s in upward direction. C

www.physicsashok.in 13
DYNAMICS

Example 12. At what value of m1 will 8 kg mass be at rest.

Sol. Here T0 = 8g = 80 N
Also, T0 = 2T 8kg
T = 40 N
From force diagram of 5 kg block,
50 T = 5a 5kg
50 40 = 5a m1
10 = 5a
a = 2 m/s2
From force diagram of m1,
T m1g = m1a T0
or 40 m1g = m1 2 8kg
T0
or 40 = 10m1 + 2m1 = 12 m1
40 10 T
m1 kg . T
12 3 T
T 5kg a
m1
5g
a m1g

Example 13. Two masses A and B, lie on a frictionless table. They are attached P1
B
to either end of a light rope which passes around a horizontal movable pulley P2
A
of negligible mass. Find the acceleration of each mass MA = 1 kg, MB = 2 kg, C
MC = 4 kg. The pulley P2 is vertical.
aB
aC
Sol.
aA
From contstant relation,
aA aB T
aC ...(i)
2
T3
From force diagram of pulley,
T3 = 2T ...(ii) T
Here T = mBaB ...(iii)
and T = mAaA ...(iv)
and mCg T3 = mCaC ...(v)
After solving,
4g 2g 3g
aA , aB , aC
5 5 5

Example 14. Block A of mass m/2 is connected to one end of light rope which passes
over a pulley as shown in the Fig. Man of mass m climbs the other end of rope with
a relative acceleration of g/6 with respect to rope find acceleration of block A g/6
m/2 m
and tension in the rope.
Sol. The acceleration of man with respect to ground is a. A
g
a a0
6

www.physicsashok.in 14
DYNAMICS

g
a a0
6 a0
From force diagram of man,
T mg = ma m
2 A m
g
or T mg m a 0 ...(i)
6
From force diagram of block A,
m m
T g a0 ...(ii)
2 2
Subtracting eqn (i) and (ii),
m m mg
g a0 ma 0
2 2 6
3 mg mg
or ma 0
2 2 6
3 4mg
or ma 0
2 6
4mg 2
or a0
6 3m
4
a0 g
9
From eqn (ii)
m m
T a0 g
2 2
m 4 m
T g g
2 9 2
13mg
T
18

Example 15. Inclined plane is moved towards right with an acceleration of 5 ms2 5kg
as shown in figure. Find force in newton which block of mass 5 kg exerts on
the incline plane. (All surfaces are smooth) 37
Sol. For discussion of the block inclined plane is taken as reference frame.
ma 5 m/s2
0s
in5
3 N

ma
ma0 53 sin
37

37

37
mgco
s37+
ma 0 cos53 mg

From the force diagram,
N = mg cos 37 + ma0 cos 53
N = 5 10 0.8 + 5 5 0.6
N = 40 + 15 = 55 N
www.physicsashok.in 15
DYNAMICS
2 .5k
spring g
Example 16. Find the reading of spring balance as shown in figure. balance
Assume that mass M is in equilibrium. (All surfaces are smooth) M
37

kx0
53 37
Sol. 53
N0
N mg

From force diagram of wedge,


N cos 53 = kx0 ...(i)
N

mg
sin
37

mgcos37

From force diagram of smaller block :


N = mg cos 37
kx0 = N cos 53
kx0 = mg cos 37 cos 53
kx0 = 2.5 10 0.8 0.6 = 12 N
Hence, reading is kx0 = 12 N.

Example 17. A flexible chain of weight W hangs between two fixed points A & B which are at he same
horizontal level. The inclination of the chain with the horizontal at both the points of support is . What is the
tension of the chain at the mid point ?
W W W
(A) cosec (B) tan (C) cot (D) none
2 2 T
2
T
W
Sol. Tsin
2
W
T
2 sin
T
Also, T cos = T0 T sin
T0 = T cos T cos
W W
T0 cos cot
2sin 2
T0
W
2
Comprehension Type [18 to 20]
A paticle of mass m is constrained to move on x-axis. A force F acts on
the particle. F always points toward the position labeled E. For example, A +ve
when the particle is to the left of E, F points to the right. The magnitude of
m E x
F is a constant F except at point E where it is zero.
The system is horizontal. F is the net force acting on the particle. The particle is displaced a distance A
towards left from the equilibrium position E and released from rest at t = 0.

www.physicsashok.in 16
DYNAMICS
Example 18. What is the period of the motion ?
2Am 2Am 2Am
(A) 4 F (B) 2 F (C) F (D) None

Sol. The time period is four times the time taken by particle to cover the distance A.
F
Here the acceleration of the particle is a .
m
1 2
A at 0
2
2A 2mA
t0
a F
The time period is
2mA
T 4t 0 4
F

Example 19. Velocity-time graph of the particle is


v v

(A) t (B) t

v v

(C) t (D) t

Sol. Since, acceleration is constant in magnitude.


So, graph is straight line.
Since, acceleration is always directed towards E.
F
If the particle is in leftward acceleration is . So, slope of v t graph is positive.
m
F
But when the particle is rightward, the acceleration is a i.e. negative.
m
Hence, option (A) is correct.

A
Example 20. Find minimum time it will take to reach from x to 0.
2
3 mA mA mA
(A)
2 F
2 1 (B)
F
2 1 (C) 2
F

2 1 (D) None

1 2 Ft 02
Sol. Here A at
2 0 2m
2mA
t0
F
But the time taken x = A to x = A/2 is t1

www.physicsashok.in 17
DYNAMICS
A 1 F 2
Here t1
2 2 m
mA
or t1
F
mA
t t 0 t1
F
2 1
Example 21. Two blocks are connected by a spring. The combination is suspended, at rest, 2m
from a string attatched to the ceiling, as shown in the figure. The string breaks suddenly.
Immediately after the string breaks, what is the initial downward acceleration of the
upper block of mass 2m ?
(A) 0 (B) 3g/2 (C) g (D) 2g m
Sol. Step-I : Discuss the problem before cutting the string : kx0
From force diagram of lower block,
kx0 = mg B
From force diagram of upper block :
T = 2mg + kx0 mg
Step-II : Discuss the problem after cutting the string, T
2mg + kx0 = 2ma
or 2mg + mg = 2ma
2m
or 3mg = 2ma
3 kx0
a g 2mg
2

Example 22. A block of mass m1 = 3.70 kg on a frictionless inclined plane of angle


30 is connected by a cord over a massless, frictionless pulley to a second block
of mass m2 = 2.30 kg hanging vertically (fig.). m1
m2
what are
(a) the magnitude of the acceleration of each block and 30
(b) the direction of the acceleration of the hanging block?
(c) What is the tension in the cord ?
Sol. (a) The free body diagram for each block is shown in fig.
x
y

m1gcos
in
gs
1
m

T
m1g
For block 1,
Fx = m1a
a
to T m1g sin = m1a ...(i)
and Fy = 0
m2g
or N m1g cos = 0 ...(ii)
For block 2,

www.physicsashok.in 18
DYNAMICS
m2g T = m2a ...(iii)
Eliminating T from Eqs. (i) and (iii), we have

a
m 2 m1 sin g
m1 m 2

a
2.30 3.70sin 30 9.8 0.735 m / s 2

3.70 2.30
(b) the result is positive, indicating that the acceleration of block 1 is up the incline and the acceleration of block
2 is downward.
(c) The tension force of the cord is
T = m1a + m1g sin
T = 3.70 0.735 + 3.70 9.8 sin 30
T = 20.8 N F = 30t N

Example 23. Force F is applied on upper pulley. If F = 30t where t is time in second.
Find the time when m1 loses contact with floor.
Sol. From force diagram of pulley :
F = 3T
30t = 3T m1 m2
T = 10 t m1=4kg m2=1kg
For lossing contact,
2T m1g
or 20t 40
40
or t 2 sec.
20

IMPORTANT FEATURES
1. Newtons first law is not true in all reference frames, but we can always find reference frames in which it is true.
such frames are called inertial reference frames, or simply inertial frames. Thus, an inertial reference frame is
one in which Newtons laws hold.
2. Newtons laws are not valid in the non-inertial frames. They are to be modified by introducing the concept of
pseudo force.
3. The pseudo force is always directed in a direction opposite to the direction of the acceleration of the non-
inertial frame.
4. While drawing free body diagrams (FBD) in which pseudo force is involved, we must first see the acceleration
of the non-inertial frame and then in the FBD, plot the pseudo force with a value ma in a direction opposite to
the acceleration of non-inertial frame.
5. If a pulley is massless, net force on it is zero even if it is accelerated. For example, in the following figure :
T1

T1
T1

T2 T2 T2
T2
FBD of pulley P

www.physicsashok.in 19
DYNAMICS

T1 = 2T2 whether the pulley is accelerated or not provided the pulley is massless. This because

Fnet mass acceleration and Fnet will be zero if pulley is massless.

FRICTION

When a force f tends to slide a body along a surface, a frictional force from the surface acts on the body. The
frictional force is parallel to the surface and directed so as to oppose the sliding. It is due to bonding between

the body and the surface. If the body does not slide the frictional force is a Static Frictional Force f s . If there

is sliding the frictional force is kinetic frictional force f k .

Three Properties of Friction



1. If the body does not move, then the static frictional force f s and the component of F that is parallel to the

surface are equal in magnitude, and f s is directed opposite that component. If that parallel component increases,
magnitude fs also increases. If the applied parallel component exceeds a certain (maximum) value, the body
slides on the surface.

2. The magnitude of f s has a maximum value called limiting value fs,max or fL that is given by fL = sN where s
is the coefficient of static friction and N is the magnitude of the normal force.
3. If the body begins to slide on the surface, the magnitude of the frictional force rapidly decreases to a constant
value fk given by fk = kN
where k is coefficient of kinetic friction.
The value of s is greater or equal to k.

NOTE : In problems if s and k are separately not given. But only is given. Then
= s = k

Example 24. A baseball player with mass m = 79 kg, sliding into second base, is retarded by a frictional force of
magnitude 470 N. What is the coefficient of kinetic friction k between the player and the ground ?
Sol. The free body diagram from the player is shown in the right. The force of friction is related to the normal force
by f = kN. N
The vertical component of acceleration is zero, so the vertical component of
Newtons second law is
N mg = 0 N = mg f
f 470
Thus, k 0.61 mg
N (79) (9.8)

Angle of Friction
As shown in fig, a body A is in contact with surface B. The forces acting on A is shown. Surface B applies two
contact forces on body A.
P N

A
F
fL
Angle of friction O B
w =M g
www.physicsashok.in 20
DYNAMICS
1. Normal reaction force N, whose magnitude is equal to weight w( = mg) of body and is directed upwards.
2. Maximum static frictional force fL (= N), which is tangential to surface of A and directed opposite to the

direction of applied force F .

The resultant force of these two forces is represented by the vector OP . The angle between vector OP and
normal reaction force N is the angle of friction ().
f N
tan L
N N
or a = tan1()
The magnitude of net tangential force applied by surface B on A,

| OP | N2 f L2

| OP | N 2 (N)2

| OP | N 1 2

Angle of Repose
Suppose inclined plane BC, makes an angle with the horizontal and a body A is placed on it is position of rest
[Fig.].

N = mg cos N
A
mg cos
mg sin
mg C
A

B H

Angle is gradually increased, till the body placed on its surface just begins to slide down. If is the inclination
at which the body just begins to slide down, then it is called angle of repose. In the figure is the angle of
repose.
From figure,
mg sin = N
= mg cos
or = tan
But = tan , wher is angle of friction.
tan = tan
or =
or angle of repose is equal to angle of friction.

Example 25. A particle of mass 1 kg rests on rough conact with a plane inclined at 30 to the horizontal and is just
about to slip. Find the coefficient of friction between the plane and the particle.
Sol. The given angle 30 is really the angle of repose . Hence,
1
tan 30
3

www.physicsashok.in 21
DYNAMICS
F
Example 26. A force F i 4 j acts on block shown. The force of friction y
acting on the block is : x 1 Kg
(A) i (B) 1.8 i (C) 2.4 i (D) 3 i = 0.3

Sol. The maximum value of friction is fmax = N


Here N + 4 = mg = 10
N=6N
fmax = 0.4 6 = 2.4 N
But applied force in horizontal direction is Fh = 1 N.
Fh < fmax. So, static friction comes into play.
Fh = f
f=1N

f i

Some Particular Cases


(i) If force is applied on lower block : The maximum friction acting on m2 is f2 = 2m2g. If the system moves
with the common acceleration, then 2
m2
F 1(m1 + m2)g = (m1 + m2)a
1
and f2 = m2a m1 F
2m2g = m2a
a = 2g
(ii) If force is applied on upper block :
f2 = limiting friction between m1 and m2.
f1 = limiting friction between the surface and m1. 2
m2 F
If F > f2, then both blocks move with different acceleration and the
1
maximum friction acts between the blocks. m1
F f2 = m2a2
F 2m2g = m2a2
and N2 = m2g N2
N1 = N2 + m1g
N1 = (m1 + m2)g f2 m2 F
f1 = 1N1
f1 = 1(m1 + m2)g
m2g
f2 = 2m2g
If f2 < f1, then m1 remains at rest.
If f2 > f1, then m1 moves in the direction of f2.
f2 f1 = m1a1
N1
N2
f2
m1
f1

m1 g
If F < f2, then no relation is found between m1 and m2. i..e, m1 and m2 move together.

www.physicsashok.in 22
DYNAMICS

If F < f1 then the system is in rest.


If F > f1, the system moves with the common acceleration a. In this case,
F f1 = (m1 + m2)a
or F 1(m1 + m2)g = (m1 + m2)a
(iii) The ratio of masses on an inclined plane : The coefficient of friction = .
N T T

T m2
m2 sin m1
m1 mg
2
m2gcos
m1 g
(a) When m1 starts moving downwards, then
m1
sin cos
m2
(b) When m2 starts moving downwards,
m1
sin cos
m2
(c) When no motion takes place,
m1
sin
m2
(iv) Blocks in contact on an inclined plane.
m2
m1 2
1

In this type of problem, find the accelerations of blocks without contact.

os os
gc N2 gc
N1
1m 2m
1 2
a1 a2
m1 m2
in in
gs gs
m1 m1gcos m2 m2gcos

(a) If a1 > a2, then both blocks move separately with respective accelerations a1 and a2.
(b) If a1 < a2, then both blocks move together with a common acceleration a.
In this case, both blocks are treated as a system of mass (m1 + m2)
(m1 + m2)g sin 1m1g cos 2m2g cos = (m1 + m2)a

Example 27. Figure shown two blocks in contact sliding down an inclined surface of
inclination 30. The friction coefficient between the block of mass 2.0 kg and the kg
kg 2
4
incline is 1 = 0.20 and that between the block of mass 4.0 kg and the incline is
2 = 0.30. Find he acceleration of 2.0 kg block. g = 10 m/s2. 30
Sol. Since, 1 < 2, acceleration of 2 kg block down the plane will be more than the acceleration of 4 kg block if
allowed to move separately.

www.physicsashok.in 23
DYNAMICS
In this case both blocks are treated as a system of mass (4 + 2) = 6 kg and will move down with the same
acceleration.
Net force down the plane is
F = (m1 + m2)g sin 1m1g cos 2m2g cos
F = (4 + 2)g sin 30 (0.2) (2) g cos 30 (0.3) (4) g cos 30

1 3 3
F (6) (10) (0.4) (10) (1.2) (10)
2 2 2
F = 30 13.76 = 16.24 N
Therefore, acceleration of both the blocks down the plane will be
F 16.24
a 2.7 m / s 2
m1 m 2 4 2

Comprehension Type 28 to 32 refer figure


Example 28. When F = 2N, the frictional force between 5 kg block and 10 kg F
ground is = 0.1
(A) 2N (B) 0 5 kg
(C) 8 N (D) 10 N = 0.3
Sol. f1 max = 0.1 10 10 = 10 N
f2 max = 0.3 15 10 = 45 N
For F = 2N, static friction come in to play at both surfaces.
F = f2
f2 = 2N

Example 29. When F = 2N, the frictional force between 10 kg block and 5 kg block is
(A) 2 N (B) 15 N (C) 10 N (D) None
Sol. From force diagram of
10 kg block,
F = f1
f1 = 2N

Example 30. The maximum F which will not cause motion of any of the blocks.
(A) 10 N (B) 15 N (C) data insufficient (D) None
Sol. F = f1
F = f1max = 10 N

Example 31. The maximum acceleration of 5 kg block


(A) 1 m/s2 (B) 3 m/s2 (C) 0 (D) None
Sol.
10N = f 1 max
5 kg
45N = f 2 max
f1max < f2max
Hence, static friction comes into play between ground and 5 kg block.
Acceleration of 5 kg block is zero.

www.physicsashok.in 24
DYNAMICS
Example 32. The acceleration of 10 kg block when F = 30 N
(A) 2 m/s2 (B) 3 m/s2 (C) 1 m/s2 (D) None
Sol.26. F f1 max = 10 a
F 10 = 10 a
or 30 10 = 10 a
20
a 2 m / s2
10

Example 33. The coefficient of static and kinetic friction between the two blocks
( s =0.6, k =0.4)
and also between the lower block and the ground are s = 0.6 and k = 0.4. M = 2kg
Find the value of tenstion T applied on the lower block at which the T
M = 2kg
upper block begins to slip relative to lower block.
Sol. The maximum static friction on upper block and lower block is
f1 max = Sm1g
= 0.6 2 10 = 12 N
The maximum static friction between ground and lower block is
F2 max = S(m1 + m2)g
= 0.6 4 10 = 24 N
Assume that both blocks move together.
T k(m1 + m2)g = (m1 + m2)a
T 0.4 4 10 = 4a
or T 16 = 4a ...(1)
For upper block :
T 16
f1 2a 2
4

T 16
or f1
2
For just sliding,
f1 = f1 max
T 16
or 12
2
or T 16 = 24
T = 24 + 16 = 40 N

Example 34. A block of mass 1 kg is horizontally thrown with a velocity of 10 m/s on a stationary long plank of
mass 2 kg whose surface has a = 0.5. Plank rests on frictionless surface. Find the time when m1 comes to rest
w.r.t. plank.
m1
V0
Sol. m2

m1g
a1 g
m1
m1g
a2
m2
www.physicsashok.in 25
DYNAMICS

m1g
a rel a1 a 2 g
m2
1
a rel 5 0.5 10
2
5
a rel 5 7.5 m / s 2
2
vrel = 0, urel = v0
vrel = urel + arel t
or 0 = v0 7.5 t
v0 10 4
t sec.
7.5 7.5 3

Example 35. A thin rod of length 1 m is fixed in a vertical position inside a train, which is moving horizontally with
constant acceleration 4 m/s2. A bead can slide on the rod, and friction coefficient between them is 1/2. If the
bead is released from rest at the top of the rod, find the time when it will reach at the bottom.
Sol. From force diagram of bead,
N = ma0 = 4m N
mg N = ma N
mg
1
or mg 4m ma a0 = 4m/s2
2
or 10 m 2m = ma
a = 8 m/s2
1 2
L at
2
1
or 1 8 t2
2
1 1
or t2 t= second
4 2

Example 36. A car begins to move at time t = 0 and then accelerates along a straight track with a speed given by
V(t) = 2t2 ms1 for 0 t 2.
After the end of acceleration, the car continues to move at a constant speed. A small block initially at rest on the
floor of the car begins to slip at t = 1 sec. and stops slipping at t = 3 sec. Find the coefficient of static and kinetic
friction between the block and the floor.
Sol. v = 2t2
dv
a 4t
dt
at t = 1, a = 4 m/s2
Since, block moves with the car upto t = 1 sec. Hence, static friction comes into play between block and car.
fs = ma
But fs = smg
or a = sg
www.physicsashok.in 26
DYNAMICS

or 4 = sg
4 4
s 0.4
g 10
Let the velocity of block at t = 3 second is v0.
The velocity of block at t = 1 second is
u0 = 2 12 = 2 m/s
The velocity of car at t = 2 second is
uC = 2 22 = 8 m/s
The acceleration of car is kg
at 1<t3
v0 = u0 + kg (3 1)
or v0 = u0 + 2kg
or vC = u0 + 2kg
or 8 = 2 + 2 k 10
or 6 = 20 k
6
k 0.3
20

Example 37. A body of mass 2 kg rests on a horizontal plane having coefficient of friction = 0.5. At t = 0 a

horizontal force F is applied that varies with time F = 2t. The time constant t0 at whcih motion starts and
distance moved in t = 2t0 second will be ........... and .............. respectively.
Sol. Here fmax = mg = 0.5 2 10 = 10 N
for sliding starts,
F > mg
F > 10
or 2t > 10
or t>5
t0 = 5 second.
2t mg 2t 0.5 2 10
Here a
m 2
2t 10
or a
2
or a=t5
dv
or t 5
dt
t
v t t2
or dv t 5 dt 5t
0 t0 5
2 5
t 2 25
v 5t 25
2 2
t2 25
v 5t
2 2
www.physicsashok.in 27
DYNAMICS

ds t 2 25
or 5t
dt 2 2

2t 0 10 t2 25
or s ds 2 5t 2 dt
t 0 5

10
t 3 5t 2 25
t
6 2 2 5
1000 125 125 125 125
250 125 m
6 6 2 2 6
m V0
Example 38. With what minimum velocity should block be projected A B
V
from left end A towards end B such that it reaches the other
end B of conveyer belt moving with constant velocity v.
Friction coefficient between block and belt is . L
(A) gh (B) 2gL (C) 3gL (D) 2 gL
Sol. a = g
02 = v02 2 gL
or v02 = 2 gL
v 0 2gL
m
Example 39. Block M slides down on frictionless incline as shown. Find the minimum M
friction coefficient so that m does not slide with respect to M.
Sol. Since, m does not slide on the block M. So, both block move together. 37
From force diagram of combined mass (m + M),
(M + m)g sin 37 = (m + M)a0

a0
M m g sin 37

m M
3
a 0 10 6m / s 2 along inclined surface. N+ma0 sin37
5
ma0
From force diagram of smaller block in the frame of triangular block,
37
N + ma0 sin 37 = mg
f mg ma0 cos37
and ma0 cos 37 = f
But f N
37
or ma0 cos 37 (mg ma0 sin 37)
or a0 cos 37 (10 6 0.6)
or 6 0.8 6.4
or 4.8 6.4
4.8
or
6.4
6
or
8
6 3
or min 0.75
8 4

www.physicsashok.in 28
DYNAMICS
IMPORTANT FEATURES
1. The direction of friction force on each of the blocks in Fig.(a) is such as it either stops the relative motion or
attempts to do so. For example, if a force F is applied on block A of a two block system, the direction of
frictional forces at different contacts on different bodies will be as shown :
A F
A f1
f1
B
B f2
f2
(a) (b)
Here, f1 = force of friction between A and B
f2 = force of friction between B and ground
2. Force of friction f = 0, if no driving force is applied.
f fL (= sN)
If driving force is applied but no relative motion is there.
and f = kN
if relative motion is there.
3. A common mistake which the students do in hurry is that they always write fL = mg (in case of horizontal
ground) or fL = mg cos (in inclined surface). The actual formula is fL = N. Here, N is equal to mg or mg cos
upto when no force is acting at some angle ( 0) with the plane.
F
F
F F

F or

N = mg N = mg N = mg cos N = mg cos
fL = mg fL = mg fL = mg cos fL = mg cos

www.physicsashok.in 29
DYNAMICS

THINKING PROBLEMS

1. Explain why a man getting out of a moving train must run in the same direction for a certain distance.

2. During a high jump event, it hurts less when an athlete lands on a heap of sand. Explain.

3. An athlete runs over a certain distance before taking a long jump. Explain.

4. Ripe mangoes fall down when the three is shaken. Why?

5. A heavy picture frame is suspended by a string passing over a peg in the vertical wall and attached at its
two ends to two points in the upper horizontal edge of the frame. Explain why the picture is likely to fall
if the string is too short.

6. Out of the coefficient of static friction, coefficient of kinetic friction and coefficient of rolling friction, the
last is the least. True or false ?

7. When we walk we rub our footwear opposite to the direction of motion. True or false ?

8. Action and reaction are equal and opposite and so they balance each other. True or false ?

9. The motion of the cm of a system is determined by external forces only and not the internal forces of the
system. True or false ?

10. The momentum of a system of particles is always conserved. True or false?

11. What will be the trajectory of the bob of a pendulum if the string is cut when the bob is (a) at one of its
extreme positions, (b) at its mean position.

12. Figure shows the position-time graph of a particle of mass m = 0.5 kg.
Suggest a suitable example to fit the curve. What is the interval between
ten consecutive impulses ? What is the magnitude of each impulse ?

13. A rod not reaching the rough floor is inserted between two identical blocks. A
horizontal force F is applied to the upper end of the rod. Which of the blocks
will move first ?

14. A massive homogeneous cylinder that can revolve without friction around a
horizontal axis is secured on a flat car. A bullet flying horizontally with a velocity
v strikes the cylinder and drops on to the car. Does the speed acquired by the
car after the impact depend on the point where the bullet strikes the cylinder ?

15. The application of brakes on the driving wheels of a car means the reversal of the direction of the force of
friction between the wheel and the ground. True or false ?

www.physicsashok.in 30
DYNAMICS

SOLUTION OF THINKING PROBLEMS

1. A man runs for a certain distance after alighting from a moving train to maintain the same velocity of all
parts of his body relative to the train so that he may not fall forward due to the inertia of rest of the lower
part of his body and inertia of motion of the upper of his body.

2. Because of greater distance and hence greater time over which him motion is stopped, he experiences less
force :
F = change in momentum = p / t. For a given p, t 1/F

3. An athlete runs over a certain distance before taking a long jump to gain inertia of motion so that he may
take a longer jump.

4. Due to inertia of rest ripe mangoes get detached from their weak stem.

5. Suppose 2l is the length of the string and 2a is the distance between the points on the upper edge of the
frame where the ends of the string are attached. Then the inclination of each part of the string to the
1 a
horizontal is cos . If T is the tension of the string, then for the equilibrium of the picture,
l
W 2T sin 2T 1 a 2 / l 2
Thus it is seen that a shorter length means a greater tension. So the picture is likely to fall if the length is too
short.

6. True 7. True 8. False 9. True

10. False, momentum is conserved only when the external force on the system is zero.

11.(a) The bob will fall vertically downward as the velocity of the bob is zero.

12. The motion of a particle between two parallel walls; 2s; 8 kg ms1.

13.The right block will move first. To justify the answer let us suppose that F is the
force applied at the end of the rod. Let F1 be the magnitude of the action and
reaction between the rod and the light block and F2 that of the rod and and left
block. Considering translational equilibrium of the rod.
F1 F2 = F
Therefore, F1 > F2
Therefore, the right block will tend to move first.

14. No, the speed of the cart will not depend on the point of impact. The momentum of the revolving cylinder
is zero because one half has as much negative linear momentum as the other half has positive linear momentum.
For this reason the bullet will impart to the car + cylinder system the same momentum as it would be a
cylinder rigidly fixed to the cart.

15.Tru. In a car the driving force and the frictional force are in the same direction. This is not in violation of the
law of friction. The application of brakes means the application of force on the axle in the opposite direction.
Consequently the frictional force is also reversed.

www.physicsashok.in 31
DYNAMICS

ASSERTION & REASON


THE NEXT QUESTIONS REFER TO THE FOLLOWING INSTRUCTIONS
A statement of Statement-1 is given and a Corresponding statement of Statement-2 is given just below it of
the statements, mark the correct answer as
(A) If both Statement-1 and Statement-2 are true and Statement-2 is the correct explanation of Statement-1.
(B) If both Statement-1 and Statement-2 are true and Statement-2 is NOT correct explanation of Statement-1.
(C) If Statement-1 is true but Statement-2 is false.
(D) If both Statement-1 and Statement-2 are false.
(E) If Statement-1 is false but Statement-2 is true.

1. Statement-1 : Aeroplanes always fly at low altitudes.


Statement-2 : According to Newtons third law of motion.
2. Statement-1 : The slope of momentum versus time curve give us the acceleration.
Statement-2 : Acceleration is given by the rate of change of momentum.
3. Statement-1 : The apparent weight of a body in an elevator moving with some downward acceleration is
less than the actual weight of body.
Statement-2 : The part of the weight is spent in producing downward acceleration, when body is in elevator.
4. Statement-1 : When the lift moves with uniform velocity the man in the lift will feel weightlessness.
Statement-2 : In downward accelerated motion of lift, apparent weight of a body decreases.
5. Statement-1 : A player lowers his hands while catching a cricket ball and suffers less reaction force.
Statement-2 : The time of catch increases when cricketer lowers its hand while catching a ball.
6. Statement-1 : A reference frame attached to earth is an inertial frame of reference.
Statement-2 : The reference frame which has zero acceleration is called a non inertial frame of reference.
7. Statement-1 : A table cloth can be pulled from a table without dislodging the dishes.
Statement-2 : To every action there is an equal and opposite reaction.
8. Statement-1 : A body subjected to three concurrent forces cannot be in equilibrium.
Statement-2 : If large number of concurrent forces acting on the same point, then the point will be in
equilibrium, if sum of all the forces is equal to zero.
9. Statement-1 : Impulse and momentum have different dimensions.
Statement-2 : From Newtons second law of motion, impulse is equal to change in momentum.
10. Statement-1 : On a rainy day, it is difficult to drive a car or bus at high speed.
Statement-2 : The value of coefficient of friction is lowered due to wetting of the surface.
11. Statement-1 : When a bicycle is in motion, the force of friction exerted by the ground on the two wheels
is always in forward direction.
Statement-2 : The frictional force acts only when the bodies are in contact.
12. Statement-1 : Pulling a lawn roller is easier than pushing it.
Statement-2 : Pushing increases the apparent weight and hence the force of friction.
13. Statement-1 : Two bodies of masses M and m (M > m) are allowed to fall from the same height if the air
resistance for each be the same then both the bodies will reach the earth simultaneously.
Statement-2 : For same air resistance, acceleration of both the bodies will be same.
14. Statement-1 : The acceleration of a body down a rough inclined plane is greater than the acceleration due
to gravity.
Statement-2 : The body is able to slide on a inclined plane only when its acceleration is greater than
acceleration due to gravity.
15. Statement-1 : A cloth covers a table. Some dishes are kept on it. The cloth can be pulled out without
dislodging the dishes from the table.
Statement-2 : For every there is an equal and opposite reaction.

www.physicsashok.in 32
DYNAMICS

Level # 1
1. The upper half on an inclined plane of inclination is perfectly smooth while the lower half is rough. A block
starting from rest at the top of the plane will come to rest at the bottom if the coefficient of friction between the
block and the lower half of the plane is given by :
2 1
(A) 2 tan (B) tan (C) (D)
tan tan

2. A rope which can withstand a maximum tension of 400 N is hanging from a tree. If a monkey of mass 30 kg
climbs on the rope, in which of the following cases will the rope brake? Take g = 10 m/s2 and neglect the
mass of the rope
(A) the monkey climbs up with a uniform speed of 5 m/s
(B) the monkey climbs up with a uniform acceleration of 2 m/s2
(C) the monkey climbs up with a uniform acceleration of 5 m/s2
(D) the monkey climbs down with a uniform acceleration of 5 m/s2

3. A block of mass m is projected up an inclined plane of inclination with an initial velocity u. If the coefficient
of kinetic friction between the block and the plane is , the distance up to which the block will rise up the
plane , before coming to rest, is given by :
u2 u2 u2 u2
(A) (B) (C) (D)
2g sin 2g cos 4g sin 4g cos

4. A flat car given an acceleration a0 = 2 m/s2 starting from rest. A cable


is connected to a crate A of weight 50 kg as shown. Neglect friction
a0
between the floor and the car wheels and also the mass of the pulley. A
Calculate corresponding tension in the cable if 0.30 between the
crate and the floor of the car.
(A) 350 N (B) 300 N (C) 450 N (D) none of these

5. A 40 kg trunk sliding across a floor slows down from 5 to 2 m/s in 6 seconds. Assuming that the force acting
on the trunk is constant, find is magnitude and its direction relative to the velocity vector of the trunk
(A) 20 N in the direction opposite to the velocity (B) 30 N in the direction opposite to the velocity
(C) 20 N in the same direction to the velocity (D) none of these

6. If the coefficient of friction between a cars wheels and the roadway is 0.70, what is the least distance in
which a car can accelerate from rest to a speed of 15 m/s?
(A) 20.8 m (B) 14.6 m (C) 16.4 m (D) none of these

7. In figure , when m is 3 kg, the acceleration of block m is


0.60 m/s2, while a = 1.6 m/s2 If m = 4 kg. Find the frictional
force on the block M as well as its mass .(neglect mass &
friction of pulley)
(A) 12.2 N (B) 10.11 N (C) 8.66 N (D) none of these

8. Find the frictional force on block 30 kg.


(A) 20 N (B) 30 N
(C) 40 N (D) 45 N

9. A heavy homogeneous sphere is suspended by a light string, one end of which is attached to a vertical wall
and the other, to a point on the vertical line through the centre of sphere. What should be the coefficient of
friction between the sphere and the wall for the sphere to remain in equilibrium.
(A) < 0.5 (B) > 1 (C) = 0 (D) none of these

www.physicsashok.in 33
DYNAMICS
10. A short, right circular cylinder of weight W rest in a horizontal,
V shaped notch of angle 2 . If the coefficicent of friction is , Find
the horizontal force P parallel the axis necessary for slipping to occur.
2
W W W
(A) (B) (C) (D) none of these
cos tan sin

11. When a cube is in limiting equilibrium on an inclined plane, it is also about to topple. The coefficient of friction
between the cube and the plane is
(A) 1 (B) 1/2 (C)1 / 2 (D) 1 / 3

12. A scooter is moving on a straight horizontal surface with a velocity u, calculate the shortest distance in which
a scooter can be stopped, if the coefficient of friction between tires and the road is

u2 u 2g u2
(A) (B) (C) (D) none of these
2g 2 4g

13. In the figure shown the force with which the man should pull the rope to
hold the plank in position is F. If weight of the man is 60 kgf, the plank
and the pulleys have negligible masses,then
(A) F = 150 N (B) F = 300 N
(C) F = 600 N (D) F = 1200 N

14. A given object takes n times as much time to slide down a 450 rough inclined as a takes to slide down the
perfectly smooth 450 incline. The coefficient of kinetic friction between the object and the incline is given by

1 1 1 1
(A) k (B) k 1 (C) k (D) 1
(1 n 2 ) n2 (1 n 2 ) n2

15. For the system in figure, the pulleys are light and frictionless.
The tension in the string will be
2 3
(A) mg sin (B) mg sin
3 2
(C) mg sin (D) 2 mg sin

16. A particle is lying at rest on a rough horizontal plane. The plane is tilted till its starts moving & then kept
fixed. If the static and the dynamic coefficient of friction are s and k , Find the velocity of particle after it
has travelled a distance d.
1/ 2
2( s k )gd 2 s k 2( s k ) 2( s k )
(A) (B) (C) (D)
2 1/ 2
(1 s ) (1 s2 )1/ 2 1 s2 (1 s2 )1/ 2

17. In the arrangement shown, the pulleys are fixed an ideal,


the strings are light, m 1 > m 2, and S is a spring balance
which is itself massless. The reading of S (in units of
mass) is
(A) m 1 - m 2 (B) (m 1 + m 2)
m1m2 2m1m2
(C) (D)
m1 m2 m1 m2

www.physicsashok.in 34
DYNAMICS
18. A uniform chain of length l is placed on a rough table, with length l/n, where n > 1, hanging over the edge. If the
chain just begin to slide off the table by itself from this position, the coefficient of friction between the chain
and the table is
1 1 1 n 1
(A) (B) (C) (D)
n n 1 n 1 n 1

19. Two masses M, m (M > m) are joined by a light spring passing over a smooth
light pulley. If the blocks are allowed to move for some time, after which M is
stopped momentarily (brought to rest and released at once). After this
(A) both blocks will move with different acceleration
(B) the string will become taut (under tension) again when the blocks aquire m
the same speed m
(C) the string will become taut again when the blocks cover equal distances M
(D) none of these

20. A boy B drags a wedge A by an in extensible string passing over


the pulleys 1,2,3 and 4 as shown in figure. If all the pulleys are
smooth and boy walks with constant velocity of magnitude V, the
magnitude of relative velocity between the boy and the wedge is =
(A) V (B) 2V
(C) 1.5 V (D) 1.25 V

21. In the given figure all the surfaces are smooth. Find
the time taken by the block to reach from the free end
to the pulley attached to the plank. Distance between
free end and pulley is

2Mm Mm Mm
(A) (B) (C) (D) none of these
(2m M)F (2m M)F (m M)F

MORE THAN ONE ANSWER IS CORRECT

22. A block of mass 0.1 is held against a wall applying a horizontal force of 5 N on the block. If the coefficient of
friction between the block and the wall is 0.5, the magnitude of the frictional force acting on the block is:
(A) 2.5 N (B) 0.98 N (C) 4.9 N (D) 0.49 N

23. The pulley and strings shown in the figure are smooth and of negligible
mass. For the system to remain in equilibrium, the angle should be
(Shown in figure) [JEE (Scr.) 2001] 2m
(A) 0 (B) 30 (C) 45 (D) 60
m m

24. A string of negligible mass going over a clamped pulley of mass m supports a block
of mass M as shown in the figure. The force on the pulley by the clamp is given by m

(A) 2 Mg (B) 2 mg [JEE (Scr.) 2001]

(C) M m
2
m2 g (D) M m 2

M2 g M

F
1
25. What is the maximum value of the force F such that the block =
shown in the arrangement, does not move? 60 2 3
m = 3 kg
(A) 20 N (B) 10 N (C) 12 N (D) 15 N [JEE (Scr.) 2003]

www.physicsashok.in 35
DYNAMICS

26. A block P of mass m is placed on a horizontal frictionless plane. A


second block of same mass m is placed on it and is connected to a k Q s
spring of spring constant k, the two blocks are pulled by distance A.
Block Q oscillates without slipping. P
What is the maximum value of frictional force between the two blocks.
(A) kA 2 (B) KA (C) s mg (D) Zero

27. A particle starts sliding down a frictionless inclined plane. If S n is the distance traveled by it from time

t n 1 sec to t n sec , the ratio S n S n 1 is


2n 1 2n 1 2n 2n 1
(A) (B) (C) (D)
2n 1 2n 2n 1 2n 1

28. A reference frame attached to the earth


(A) is an inertial frame by definition.
(B) cannot be an inertial frame because the earth is revolving round the sun.
(C) is an inertial frame because Newtons laws are applicable in this frame.
(D) cannot be an inertial frame because the earth is rotating about its own axis.

29. A spring of force constant k is cut into two pieces such that one piece is double the length of the other. Then
the long piece will have a force constant of [JEE-1999]
(A) (2/3) k (B) (3/2) k (C) 3 k (D) 6 k

30. The string between blocks of mass m and 2m is massless and inextensible. The system
is suspended by a massless and inextensible. The system is suspended by a massless
spring as shown. If the string is cut find the magnitudes of accelerations of mass 2m
and m (immediately after cutting) [JEE-2006] 2m
m
g g g g
(A) g, g (B) g, (C) , g (D) ,
2 2 2 2
31. Two particles of mass m each are tied at the ends of a light string of length 2a. F
The whole system is kept on a frictionless horizontal surface with the string held
tight so that each mass is at a distance a from the centre P (as shown in the m P m
figure). Now, the mid-point of the string is pulled vertically upwards with a small a a
but constant force F. As a result, the particles move towards each other on the surface. The magnitude of
acceleration, when the separation between them becomes 2x, is [JEE-2007]
F a F x F x F a2 x2
(A) 2m 2 (B) 2m 2 (C) (D)
a x2 a x2 2m a 2m x
32. A particle moves in the XY plane under the influence of a force such that its linear momentum is

pt A i cos kt jsin kt , where Aand k are constants. The angle between the force and the momentum
is [JEE-2007]
(A) 0 (B) 30 (C) 45 (D) 90

www.physicsashok.in 36
DYNAMICS

FILL IN THE BLANKS

1. A block of mass 1 kg lies on a horizontal surface in a truck. The coefficient of static friction between the block
and the surface is 0.6. If the acceleration of the truck is 5 m/s2, the frictional force acting on the block is
................ newtons.
L
2. A uniform rod of length L and density is being pulled along a smooth floor with
a horizontal acceleration (see Figure).
The magnitude of the stress at the transverse cross-section through the mid-point
of the rod is ...............

3. Imagine that you are holding a book weighing 4 N at rest on the palm of your hand. Complete the following
sentences:
(A) A downward force of magnitude 4 N is exerted on the book by __________ (B) An upward force of
magnitude __________ is exerted on __________ by the hand. (C) Is the upward force in part (B) the
reaction to the downward force in part (A)? (D) The reaction to the force in part (A) is a force of magnitude
__________ exerted on __________ by __________ Its direction is __________ (E) The reaction to the
force in part (B) is a force of magnitude __________ exerted on __________ by __________ Its direction is
_____ (F) The forces in parts (A) and (B) are equal and opposite because of Newtons __________ law. (G)
The forces in parts (B) and (E) are equal and opposite because of Newtons __________ law.

TRUE / FALSE

4. When a person walks on a rough surface, the frictional force exerted by the surface on the person is opposite
to the direction of his motion.
5. A simple pendulum with a bob of mass m swings with an angular amplitude of 40. When its angular
displacement is 20, the tension in the string is greater than mg cos20.
6. The pulley arrangements of Figure (A) and (B) are identical. The mass of
the rope is negligible. In (A) the mass m is lifted up by attaching a mass
2m to the other end of the rope. In (B), m is lifted up by pulling the other
end of the rope with a constant downward force F = 2 mg.
The acceleration of m is the same in both cases.
m 2m m F = 2mg
(A) (B)
7. Two particles of mass 1 kg and 3 kg move towards each other under their mutual force of attraction. No other
force acts on them. When the relative velocity of approach of the two particles is 2 m/s, their centre of mass
has a velocity of 0.5 m/s. When the relative velocity of approach becomes 3 m/s, the velocity of the centre of
mass is 0.75 m/s.
TABLE MATCH
8. Match Table I and Table II and select the correct answer using the codes given below the lists:
Column-I Column-II
A. Normal reaction P. Electromagnetic force
B. Frictional force Q. [MLT2]
C. Nuclear force R. Super position principle
D. Force of tension S. Short range force

9. Match Table I and Table II and select the correct answer using the codes given below the lists:
Column-I Column-II
A. Thrust on a body of mass m floats inside a liquid P. mg
B. Normal reaction on a body of mass m placed Q. Zero
on rough horizontal surface stationary
C. Friction on a body of mass m placed on a R. mg
rough horizontal surface in the absence of
pulling and pushing force
D. Friction on a body of mass m moving on S. greater than zero
horizontal rough surface.

www.physicsashok.in 37
DYNAMICS

PASSAGE TYPE QUESTIONS

PASSAGE # 1
A moving company uses the pulley system in figure 1 to lift heavy crates up a ramp. The ramp is coated
with rollers that make the crates motion essentially frictionless. A worker piles cinder blocks onto the plate
until the plate moves down, pulling the crate up the ramp. Each cinder block has mass 10 kg. The plate has
mass 5 kg. The rope is nearly massless, and the pulley is essentially frictionless. The ramp makes a 30
angle with the ground. the crate has mass 100 kg.

Cr
ate

Plate 30

Let W1 denote the combined weight of the plate and the cinder blocks piled on the plate. Let T denote

the tension in the rope. And let W2 denote the crates weight.

1. What is the smallest number of cinder blocks that need to be placed on the plate in order to lift the crate up
the ramp?
(A) 3 (B) 5 (C) 7 (D) 10

2. Ten cinder blocks are placed on the plate. As a result, the crate accelerates up the ramp. W hich of the
following is true?
(A) W1 T W2 sin 30 (B) W1 T W2 sin 30

(C) W1 T W2 sin 30 (D) W1 T W2 sin 30

3. The ramp exerts a normal force on the crate, directed perpendicular to the ramps surface. This normal force
has magnitude:
(A) W2 (B) W2 sin 30 (C) W2 cos 30 (D) W2 sin 30 cos 30

4. The net force on the crate has magnitude:


(A) W1 W2 sin 30 (B) W1 W2 (C) T W2 sin 30 (D) T W2

5. After the crate is already moving, the cinder blocks suddenly fall off the plate. Which of the following graphs
best shows the subsequent velocity of the crate, after the cinder blocks have fallen off the plate? (Up the-
ramp is the positive direction.)

+ A + B + C + D
v v v v
0 0 0 0
time time time time

(A) A (B) B (C) C (D) D

www.physicsashok.in 38
DYNAMICS

PASSAGE # 2
A safety engineering firm is producing a film for high school driver education classes. The firm uses
skilled test drivers driving both small cars and larger vans. The vans weigh three times as much as the cars
and have larger tires with twice the tread width. In a demonstration that tests reaction times and skid-to-stop
distances and shows them on the film, three guns that fire a yellow paint onto the road are mounted on the
bumpers and fired electrically. When the driver hears the report of the first gun, he locks the breaks, and the
touch of his foot on the brake pedal fires a second yellow pellet. The third pellet is fired when the car stops.
The safety engineers also design several remote controlled cars and vans in order to film crash results.

6. Drivers of a car and a van brake hard and skid to a stop from 50 mph. The skid marks are measured to be the
same length for both. Why are the stopping distances the same length?
(A) The mechanical work done by friction to stop both is the same
(B) The frictional force between tires and road is three times greater for the heavier van so it slides the same
distance as the car
(C) The frictional force for the car and van are the same
(D) The wider tires on the van require less friction force than the narrow tires on the car.

7. Two drivers in identical cars skid to a stop from speeds of 20 mph and 40 mph. How do the lengths of the skid
marks compare?
(A) They are the same length (B) The 40 mph mark is twice as long
(C) The 40 mph mark is four times as long (D) The 40 mph mark is eight times as long.

8. A remote controlled car and van are crashed head on at the same speed. Why does the car suffer more
damage in the collision?
(A) The car and van had the same momentum
(B) The forces during collision are equal and opposite, so the smaller and weaker car suffers more damage.
(C) The van exerts a larger force on the car
(D) The mechanical work down in stopping the car is greater.

9. The van going at a speed of 15 mph collides head on with a car going at a speed of 45 mph. Because the van
weighs three times as much, their vector momenta are equal and opposite. However, examination shows that
the car suffers more damage than the van. Why?
(A) The massive van exerts a larger force on the car.
(B) The lighter car exerts a smaller force on the van
(C) the forces exerted during the collision are equal and opposite, so the weaker car suffers more damage.
(D) The mechanical work required to stop the van is smaller.

10. A driver in a van skids to a stop from 20 mph and a driver in a car skids to a stop from 60 mph. How do the
length of the skid marks compare?
(A) The van skids three times further than the car, because it is three times heavier.
(B) The car skids three times further because it is lighter an si going three times faster.
(C) The car will skid nine times as fast as the van
(D) They skid the same length because the speed ratio is 1:3, whereas the weight ratio is 3:1.

11. For the reaction time test, one driver is tested at 20 mph and 60 mph. It is noted that the distance between
the first two paint marks is three times farther for the 60 mph test than the 20 mph test. How do his reaction
times compare at 20 mph and 60 mph?
(A) His reaction time at 60 mph is three times faster.
(B) His reaction times remain the same
(C) His reaction time at 20 mph is three times faster.
(D) His reaction time at 20 mph is one-third as long as at 60 mph.

www.physicsashok.in 39
DYNAMICS

PASSAGE # 3
A student performs a series of experiments to determine the coefficient of static friction and the coefficient
of kinetic friction between a large crate and the floor. The magnitude of the force of static friction is always
less than or equal to s N . where s denotes the coefficient of static friction and N denotes the normal
force exerted by the floor on the crate f s s N
Static friction exists only when the crate is not sliding across the floor. The force of kinetic friction is given by
f k K N , where K denotes the coefficient of kinetic friction. Kinetic friction exists only when the crate
is sliding across the floor. The crate has mass 100 kg. In this situation, the normal force points upward.
Experiment 1 The student pushes horizontally (rightward) on the crate and gradually increases the strength
of this push force. The crate does not begin to move until the push force reaches 400 N.
Experiment 2 The student applies a constant horizontal (rightward) push force for 1.0 second and measures
how far the crate moves during that time interval. In each trial the crate starts at rest, and the student stops
pushing after the 1.0 second interval. The following table summarizes the results.
Trial Push force (N) Distance (m)
1 500 1.5
2 600 1.5
3 700 2

12. The coefficient of static friction between the crate and floor is approximately
(A) 0.25 (B) 0.40 (C) 2.5 (D) 4.0

13. In experiment 1, when the rightward push force was 50 N the crate didnt it move?
(A) The push force was weaker than the frictional force on the crate.
(B) The push force had the same strength as the gravitational force on the crate.
(C) The push force was weaker than the frictional force on the crate.
(D) The push force had the same strength as the frictional force on the crate.

14. The coefficient of kinetic friction between the crate and the floor is approximately
(A) 0.20 (B) 0.30 (C) 0.40 (D) 0.50

15. In trial 3, what is the crates speed at the moment the student stops pushing it?
(A) 1.0 m/s (B) 2.0 m/s (C) 3.0 m/s (D) 4.0 m/s

16. For trial 3, which of the following graphs best shows the positions of the crate as a function of time? The
student first starts pushing the crate at time t = 0.
position

position

position

position

(A) (B) (C) (D)

1 time(s) 1 time(s) 1 time(s) 1 time(s)

www.physicsashok.in 40
DYNAMICS

Level # 2

Section-A
1. In figure, block 1 is one-fourth the length of block 2 and weighs 1
one-fourth as much. Assume that there is no friction between m
block 2 and the surface on which it moves and that the coefficient 4m 2
of sliding friction between blocks 1 and 2 is k = 0.2. After the
system is released, find the distance block 2 has moved when m 3
only one-fourth of block 1 is still on block 2. Block 1 and block 3 have the same mass.

2. A dinner plate rests on a tablecloth, with its center 0.3m


from the edge of the table. The tablecloth is suddenly yanked
horizontally with a constant acceleration of 9.2 m/s2. The 2
a = 9.2 m/s
coefficient of sliding friction between the tablecloth and the
plate is k = 0.75. Find (a) the acceleration, (b) the velocity,,
x
and (c) the distance of the plate from the edge of the table,
when the edge of the tablecloth passes under the center of
the plate. Assume that the tablecloth just fits the tabletop.

3. A research balloon of total mass M is descending vertically with downward


acceleration a. How much balalst must be thrown from the car to give the
balloon an upward acceleration a, presuming that the upward life of the air
on the balloon does not change?
F

4. Someone exerts a force F directly up on the axie of the pulley shown in figure.
Consider the pulley and string to be massless and the bearing frictionless. Two
objects, m 1 of mass 1.2 kg. and m 2 of mass 1.9 kg, are attached as shown to
the opposite ends of the string, which passes over the pulley. The object m 2 is
in contact with the floor.
(a) What is the largest value the force F may have so that m 2 will remain at m1
rest on the floor?
(b) What is the tension in the string if the upward force F is 110 N?
(c) With the tension determined in part (b), what is the acceleration of m 1? m2

5. Two particles, each of mass m, are connected by a light string of 2L


length 2L, as shown in figure. A steady force F is applied at the midpoint
of the string (x = 0) at a right angle to the initial position of the string.
Show that the acceleration of each mass in the direction at 900 to F is m m
F x
given by a x 2m 2 2 1/ 2 in which x is the perpendicular distance of F
(L x )
one of the particles from the line of action of F. Discuss the situation when x = L.

6. A worker wishes to pile sand onto a circular area in his yard. The radius of the
circle is R. No sand is to spill onto the surrounding area; Figure show that the
h
greatest volume of sand that can be stored in this manner is s R 3 / 3 where

s is the cofficient of static friction of sand on sand. (The volume of a cone R


is Ah/3, where A is the base area and h is the height)

www.physicsashok.in 41
DYNAMICS

7. A block of mass M is connected with a particle of mass m by


a light inextensisble string as shown in figure. Assuming all
contacting surfaces as smooth, find the acceleration of the M
wedge after releasing the system. m

BAR B
8. Blocks A and B in figure are connected with a bar of negligible weight.
If A and B each weight 170 N and A = 0.2 and B = 0.4, calculate the
A 8
acceleration of the system and the force in the bar.
15

9. In the system of connected bodies in figure


the coefficient of friction is 0.2 under bodies
B and C. Determine the acceleration of each
body and the tension in this cord.

10. A smooth ring of mass M, is threaded on a string whose ends


are then threaded over two smooth fixed pulleys with masses
m and m tied on-to them respectively, the various portions of
the strings being vertical. The system being free to move,
4 1 1 m m
show that the ring will remain at rest if :-
M m m' M

11. Two men, of masses M and M + m, start simultaneously from the ground and climb with uniform accelerations
up the free ends of a weightless inextensible rope which passes over a smooth pulley at a height h from the
ground. If the lighter of the two men reaches the pulley in t second, show that the heavier cannot get nearer

m gt 2
to it than h .
Mm 2

12. A 25 kg block A rests on an inclined surface and a 15 kg


counter weight B is attached to a cable as shown. Neglecting
friction, determine the acceleration of A, acceleration of B
and tension in the cable after the system is released from rest.
Cable is parallel to the plane Take g = 10 m/s2.

Section-B
1. The 10 kg solid cylinder is resting in the inclined V-block. If the
coefficient of static friction between the cylinder and the block
is 0.5, determine (a) the frictional force F acting on the cylinder
at each side before force P is applied (b) the value of P required
to start sliding the cylinder up the incline (g = 9.8 m/s2)

www.physicsashok.in 42
DYNAMICS

2. A block A of mass m and length l is placed on a horizontal floor.


A rectangular box B is used to cover A. The distance between
interior of the walls of B is L (>1) and the mass of B is also m.
The coefficient of friction between A and floor is 1 and that
between B and floor is 2 ( 2 1 ) . Initially the left end of A
touches the left wall of B as shown in figure and both A and B
moves with velocity 0 towards the right. All collisions between
A ad B are elastic and contact time during each collision is very
short. Find an expression for the period between two consecutive collisions.

3. Consider the situation shown in figure the block B


moves on a frictionless surface, while the coefficient
of friction between A and the surface on which it moves
is 0.2. Find the acceleration with which the masses
move and also the tension in the strings. (Take = 10 m/s2)

4. Figure shows a man standing stationary with respect to a


horizontal conveyor belt that is accelerating with 1 ms-2.
What is the net force on the man? If the coefficient of static
friction between the mans shoes and the belt is 0.2, upto
what acceleration of the belt can the man continue to be
stationary relative to the belt?
Mass of the man = 65 kg. (g = 9.8 m/s2) 2
a = 2 ms 5m
5. The rear side of truck is open and a box of 40 kg. mass is
placed 5 m away from the open end as shown in figure. The
coefficient of friction between the box and the surface below
it is 0.15. On a straight road, the truck starts from rest and
accelerates with 2 ms-2 , find the time when box falls off
the truck. (g = 9.8 m/s2)

6. A small disc A is placed on an inclined plane forming an


angle with the horizontal and is imparted an initial
velocity v 0. Find how the velocity of the disc depends on
the angle if the friction cofficient tan and at the
initial moment . 0 / 2

7. A monkey of mass m clings to a rope slung over a fixed pulley. The


M
opposite end of the rope is tied to a weight of mass M lying on a
horizontal plate. The coefficient of friction between the weight and
the plate is . Find the acceleration of weight and the tension of
m
the rope for three cases.(a) the monkey does not move with respect
to the rope (b) the monkey moves upwards with respect to the rope
with an acceleration b (c) the monkey moves downwards with respect
to the rope with an acceleration b.

8. Each of the three plates has a mass of 10 kg. If the coefficients of 18 N D


static and kinetic friction at each surface of contact are s = 0.3 and C 100 N
k 0.2 , respectively, determine the acceleration of each plate when 15 N B
the three horizontal forces are applied. (Take g = 10 m/s2)
A

www.physicsashok.in 43
DYNAMICS

9. In the arrangement shown in figure, the rod of mass m


held by two smooth walls, remians always perpendicular
to the surface of the wedge of mass M. Assuming all the
surfaces are frictionless, find the acceleration of the rod
and that of the wedge.

10. A plank of mass M is placed on a rough horizontal surface and a m


constant horizontal force F is applied on it. A man of mass m runs
on the plank. Find the acceleration of the man so that the plank does
M F
not move on the surface. Co-efficient of friction between the plank
and the surface is . Assume that the man does not slip on the plank.

11. A certain constant force starts acting on a body moving at a constant velocity v. After a time interval t , the
velocity of the body is reduced by half and after the same time interval the velocity is again reduced by half.
Determine the velocity of the body after a time interval 3 t from the moment when the constant force starts
acting.
V0
M
12. A cart with a mass of M = 0.5 kg is connected by a string to
a weight having a mass m = 0.2 kg. At the initial moment the
cart moves to the right along a horizontal plane at a speed of
v0 = 7 m/s. Find the magnitude and direction of the velocity
of the cart, the place it will be at and the distance it covers in m
t = 5 seconds.
F(N)
F
13. A 20 kg block is originally at rest on a horizontal surface for
which the coefficient of friction is 0.6. If a horizontal force F
is applied such that it varies with time as shown. Determine 200
the speed of the block in 10s.
0 t(s)
5 10

14. For the system at rest shown in the figure, determine the accelerations
of all the loads immediately after the lower thread keeping the system in
equilibrium has been cut. Assume that the threads are weightless and
inextensible, the strings are weightless, the mass of the pulley is negligibly m1 m3
small and there is no friction at the point of suspension.
k k
15. m2 m4
(a) In the arrangement shown in the figure the floor
is smooth and the friction exists only between the
blocks. The coefficient of static friction s = 0.6 m1 m1 F
and coeffecient of kinetic friction k = 0.4, the F
m2 m2
masses of the block are m 1 = 20 kg and m 2 = 30kg.
Find the acceleration of each block if
(1) F = 180 N (2) F = 200 N
(b) Do the previous for F = 180 N if F is directed up as shown in the figure.

16. A body with zero initial slips from the top of an inclined plane forming an angle with the horizontal.The
coefficient of friction between the body and the plane increases with the distance s from the top according
to the law = bs. The body stops before it reaches the end of the plane. Determine the time t from the
beginning of motion of the body to the moment when it comes to rest.

www.physicsashok.in 44
DYNAMICS

Level # 3
K
1. A particle of mass 102 kg is moving along the positive x-axis under the influence of a force F(x) =
where K = 102 Nm 2. At time t = 0 it is at x = 1.0 m and its velocity is v = 0.
2 x 2

(a) Find its velocity when it reaches x = 0.50 m.


(b) Find the time at which it reaches x = 0.25 m (IIT 1998)

2. A smooth semicircular wire-track of radius R is fixed in a vertical plane. One end


3R
of a massless spring of natural length is attached to the lowest point O of the
4
wire-track. A small ring is held stationary at point P such that the spring makes an
P
angle of 60 with the vertical. The spring constant K = mg/R. Consider the instant
60
when the ring is released, and
(a) draw the free body diagram of the ring. O
(b) determine the tangential acceleration of the ring and the normal reaction. (IIT 1996)

3. Two masses m and 2m are connected by a massless string which


passes over a light frictionless pulley as shown in the figure. The
masses are initially held with equal lengths of strings on either side
of the pulley. Find the velocity of the masses at the instant the lighter
mass moves up a distance of 6.54 m. The string is suddenly cut at m 2m
that instant. Calculate the time taken by each mass to reach the
13.08 m
ground. (g = 981 cm/s2) (IIT 1977)
Ground

4. Two cubes of masses m 1 and m 2 be on two frictionless slopes of m2


block A which rests on a horizontal table. The cubes are connected
by a string which passes over a pulley as shown in the figure. To
what horizontal acceleration should the whole system (that is blocks

and cubes) be subjected so that the cubes do not slide down the planes.
what is the tension of the string in this situation? (IIT 1978)
A m1

5. A lift is going up. The total mass of the lift and the
passengers is 1500 Kg. The variation in speed of v(m/s)
the lift is as given in the graph.
(a) What will be the tension in the rope pulling the
lift at t equal to 3.6
(i) 1 sec (ii) 6 sec (iii) 11 sec.
(b) What is the height to which the lift takes the passengers?
(c) What will be the average velocity and the average
acceleration during the course of the entire motion? O 2 10 12 t (sec)
( g = 9.8 m/s2) (IIT 1976)

6. Two balls A and B of masses 100 gm and 250 gm respectively and connected by a stretched string of
negligible mass, and placed on a smooth table. When the balls are released simultaneously, the initial
acceleration of ball B is 10 cm/s2 westward. What is the magnitude and direction of the initial acceleration of
the ball A? (IIT 1975)

7. A spring weighing machine kept inside a stationary elevator reads 50 kg when a man stands on it. What would
happen to the scale reading if the elevator is moving upward with
(a) constant velocity
(b) constant acceleration ? (IIT 1972)

www.physicsashok.in 45
DYNAMICS
8. Two identical blocks A and B are placed on a rough inclined plane 2m
of inclination 45. The coefficient of friction between block A and
incline is 0.2 and that of between B and incline is 0.3. the initial
A B
separation between the two blocks is 2m . The two blocks are
released from rest, then find
(a) the time after which front faces of both blocks come in B
same line and
(b) the distance moved by each block for attaining above A
position. (IIT 2004) 45

9. In the figure masses m 1, m 2 and M are 20 kg, 5 kg and 50 kg


P1
respectively. The coefficient of friction between M and ground m1
is zero. The coefficient of friction between m 1 and M and that
between m 2 and ground is 0.3. The pulley and the string are
massless. The string is perfectly horizontal between P1 and F
M
m 1 and also between P2 and m 2. The string is perfectly vertical P2
between P1 and P2. An external horizontal force F is applies to m2
the mass M. Take g = 10 m/s2.
(a) Draw a free body diagram for mass M, clearly showing the forces.
(b) Let the magnitude of the force of friction between m 1 and M be 1 and that between m 2 and ground be 2.
For a particular F it is found that
1 = 2 2.
Find 1 and 2. Write down equations of motion of all the masses.
Find F, tension in the string and accelerations of the masses. (IIT 2000)

10. Block A of mass m block B of mass 2m are placed on


a fixed triangular wedge by means of a massless, in
extensible string and a frictionless pulley as shown in A B
Figure. The wedge is inclined at 45 to the horizontal
on both sides. The coefficient of friction between block
A and the wedge is 2/3 and that between block B and
m
2m
the wedge is 1/3. If the system A and B is released 45 45
from rest, find
(a) The acceleration of A,
(b) tension in the string,
(c) the magnitude and direction of friction acting on A. (IIT 1997, May)

11. Masses M1, M2 and M3 are connected by strings of negligible


mass which pass over massless and frictionless pulleys P1 and P1
P2 as shown in the figure. Th masses move such that the portion
of the string between P1 and P2 is parallel to the inclined plane
and the portion of th string between and the coefficient of M2 P2
kinetic friction between the masses and the surfaces is 0.25. M1
The inclined plane makes an angle of 37 with the horizontal. M3
37
If the mass M1 moves downward with a uniform velocity, find:
(a) the mass of M1
(b) tension in the horizontal portion of the string.
3
(g = 9.8 m/s2, sin 37 = ) (IIT 1981)
5
A D
12. In the Figure (a) and (b), AC, DG and GF are fixed
inclined planes. BC = EF = x and AB = DE = y. A small
block of mass M is released from the point A. It slides G
down AC and reaches C with a speed VC. The same block
is released from rest from the point D. It slides down DGF
and reaches the point F with speed VF . The coefficient of
kinetic frictions between the block and both the surfaces B (a) C E (b)
AC and DGF are . Calculate VC and VF . (IIT 1980)

www.physicsashok.in 46
DYNAMICS

13. Two blocks connected by a massless sting slide down an inclined m


1
plane having an inclination of 37. The masses of two blocks are
m 1 = 4 kg m 2 = 2 kg respectively and the coefficient of friction of m 1
and m 2 with inclined plane are 0.75 and 0.25 respectively. Assuming m
2
the string to be taut, find
(a) the common acceleration of two masses and
(b) the tension in the string (sin 37 = 0.6, g = 9.8 m/s2) (IIT 1979) 37

14. In the diagram shown, the blocks A, B and C weight 3 kg, 4 kg


and 5 kg respectively. The coefficient of sliding friction between
any two surfaces is 0.25. A is held at rest by a massless rigid
rod fixed to the wall while B and C are connected by a light A
flexible cord passing around a frictionless pulley. Find the force
B
F necessary to drag C along the horizontal surface to the left at
constant speed. Assume that the arrangement shown in the C
diagram, B on C and A on B, is maintained all through.
(g = 9.8 m/s2) (IIT 1978)

15. An aeroplane requires for take off a speed of 80 km/h, the run on the ground being 100 metres. The mass of
the plane is 10000 kg and the coefficient of friction between the plane and the ground is 0.2. Assume that the
plane accelerates uniformly during the take off. What is the minimum force required by the engine of the plane
for the take off? (g = 9.8 m/s2) (IIT 1977)

16. A circular disc with a groove along its diameter is placed horizontally on a rough
surface. A block of mass 1 kg is placed as shown. The co-efficient of friction
between the block and all surfaces of groove and horizontal surface in contact 25 m/s2
2
is . The disc has an acceleration of 25 m/s2 towards left. Find the
5
4 3
acceleration of the block with respect to disc. Given cos , sin . [2006]
5 5

Answer Key
Assertion-Reason
1. (A) 2. (D) 3. (C) 4. (E) 5. (A) 6. (D) 7. (B) 8. (E)

9. (E) 10. (A) 11. (E) 12. (A) 13. (D) 14. (D) 15. (B)

Level # 1

Q. 1 2 3 4 5 6 7 8 9
A ns. A C C A A C A D B
Q. 10 11 12 13 14 15 16 17 18
A ns. C A A A B C A D B
Q. 19 20 21 22 23 24 25 26 27
A ns. C D A B C D A A A
Q. 28 29 30 31 32
A ns. BD B C B D

www.physicsashok.in 47
DYNAMICS
Fill in the Blanks / True-False / Match Table
1. 5 2. 0.5 p
3. (a) Earth (b) 4N, Book (c) No (d) 4N, Earth, book, upward
(e) 4N, hand, book, downward, (f) Second law (g) Third law.
4. False 5. False 6. False 7. False
8. [(A P, Q, R); (B P, Q, R); (C Q, S); (D P, Q, R)]
9. [(A P, S); (B P, S); (C Q); (D R, S)]

Passage Type Question


Q. 1 2 3 4 5 6 7 8 9
Ans. B D C C C B C B C
Q. 10 11 12 13 14 15 16
Ans. C B B D B D C

Level # 2
Section-A
a
1. 2. (a) 7.35 m/s2, (b) 4.26 m/s (c) 1.54 m 3. 2M
7.47 a g
4 mg
4. (a) 37 N, (b) 55 N, (c) 36 m/s2, Upward 7.
17m M
8. 2.02 m/s2, P = 15 N 9. T = 174.1 N , f A = 1.27 m/s2 , downwards , f B = 1.08 m/s2 , f C = 0.9 m/s2

12. 1.272 m/s2 (down the plane) , 1.8 m/s2 (at 150 with horizontal ) , 149 N

Section-B
2(L )
1. (a) 24.5 N , (b) 109 N 2. 3. a = 6 m/s2 T 2 = 48 N, T 1 = 32 N
( 2 1 )g

2 5
4. Net force on the man = 65 1 = 65 N a0 = Mg = 0.2 9.8 = 1.96 m/s2 5. = 4.34 s
0.53
0 ( m M )g mMg (1 )
6. v = 7. (a) a , T
1 cos m M Mm
m(g b) Mg 2m1m3 g m(g b) M g mM[g (1 ) b]
(b) a , (c) a , T
mM (m2 m3)(m1 m2)m2m3 mM mM

mg cos sin mg cos


8. aB = 0, aC = 4 m/s2 , aD = 0.2 m/s2 9. m sin M , m sin M
sin sin

F ( M m )g F ( M m )g 7
10. < a< + 11. v
m m m m 4

www.physicsashok.in 48
DYNAMICS

12. The cart will be at the same place and will have a speed v = 7 m/s, directed to right & total covered distance
is 17.5 m.
m3 m 4 m1 m 2 g
13. 24.8 m/s 14. a1 = a2 = a3 = 0 a 4 15. (a) (i) a1 = a2 = 3.6 m/s2
m4


(ii) a1 = 6.08 m/s2 , a2 = 2.6 m/s2 (b) a1 = 5.08 m/s2 , a2 = 3.4 m/s2 16. t
gb cos
Level # 3
5 3 3

1. (a) 1 m/s (b) 1.48 sec 2. (a) a T 8 g (b) N mg
8

2
3. Velocity = 6.54 m/s, Time taken by m is 2.8 second, Time taken by 2 m is second.
3

m1 sin m2 sin m1 m 2 sin


4. m cos m cos g , T m cos m cos
1 2 1 2

5. (a) (i) 17400 N (ii) 14700 N (iii) 1200 N (b) 36 m (c) Average velocity = 3 m/s, Average acceleration = 0

6. 25 cm/s2 Eastward. 7. (a) ramains same (b) increase

8. (a) t = 2 sec (b) SB = 7 2 m (c) SS = 8 2 m

9. (b) 1 = 30 N, 2 = 15 N, F = 60 N Tension = 18 N
(Acceleration of M) = (Acceleration of m 1) = 0.6 m/s2 (m 1 is at rest w.r.t. M).
2 2 mg

10. (a) zero (b) 3 mg (c) 11. (a) M1 = 4.2 kg (b) 9.8 N
3 2

12. VC VF 2 g y x

13. (a) 1.31 m/s2 (b) 5.2 N 14. 78.4 N 15. 4.43 x 104 N 16. 10 m/s2

XXXX

www.physicsashok.in 49
GRAVITATION

DHANALAKSHMI NAGAR

NEAR ANNAMAIAH CIRCLE,

TIRUPATI.

PH NO. 9440025125
GRAVITATION

NEWTONS LAW OF GRAVITATION


Newton in 1687 proposed force law that we call Newtons law of gravitation. According to him, every
particle of matter in the universe attracts every other particle with a gravitational force whose magnitude is
directly proportional to the product of the masses of the particls and inversely proportional to the square of the
distance between them. Thus,
m1m 2
FG
r2
Here, m1 and m2 are the masses of the particles, r is the distance between them and G is the gravitational
constant, with a value that is now known to be
G = 6.67 1011 Nm2/kg2
G = 6.67 1011 m3/kgs2
The direction of the force F is along the line joining the two particles. Regarding gravitational force, following
points should be noted :
1. The gravitational force between two particles is independent of the presence of other bodies or the properties
of the intervening medium.
2. Gravitational force is conservative force, therefore work done in displacing a body from one place to another
is independent of path. It depends only on initial and final positions.
3. The gravitational force obeys Newtons third law i.e.,

F12 F21

C1 : Calculate the gravitational force of atraction between two spherical bodies, each of mass 1000 kg if the
distance between their centre is 10 metre. (G = 6.67 1011 Nm2 kg2)
m1m2 1000 1000
Sol. FG 2
6.67 1011
r (10)2
F = 6.67 107 newton.
Vector form of the law

Let us denote by F12 the force on m1 by m2. Similarly F21 denotes the force A
on m2 by m1. Then m1 A
F12 r12

F12 F 21 . r1 B
F21
That is, Newtons law of gravitation obeys. Newtons third law of motion B
m2
r2
(in strong form). The force on m1 is directed towards m2 and that on m2 is O
directed towards m1. It is a central force.

Let be r12 be position vector of m1 with respect to m2. From vector addition law, we get,

r12 r 2 r1

r12 r1 r 2

Now F12 can be written as the product of magnitude and direction (unit vector). The direction of F12 is

r12
opposite to r12 . Hence s the unit vector in the direction of F12 .
| r12 |

www.physicsashok.in 1
GRAVITATION


r12 r 2 r1
F12 | F12 | | F12 |
| r12 | | r 2 r1 |
Gm1m 2
Now | F21 | | F12 |
r2

the distance between m1 and m2 may be written as r | r1 r 2 | . This gives

Gm1m 2 r 2 r1
F12 2
| r1 r 2 | | r 2 r1 |
Gm m

F12 1 23 r 2 r1
| r 2 r1 |

Gm m

F12 1 2 3 r 2 r1 F21
Thus,

| r 2 r1 |
This is the vector form of Newtons law of gravitation.

Principle of Superposition
By using the principle of superposition, we can find the net (or result) gravitational force on any particle from
others in a group of particles. This is a general principle that says a net effect is the sum of the individual effects.
For n interacting particles, we can write the principle of superposition for gravitational force as

F1,net F12 F13 F14 F15 ...... F1, n ...(i)

Here, F1,net is the net force on particle 1.
We can express this equation more compactly as a vector sum.
n
F1,net F1i ...(ii)
i2
For a particle which is from a real extended object the sum of Eq. (ii) becomes an integral and we have

F1 d F
in which the integral is taken over the entire extended object and we drop the subscript net.
y
Example 1 : Fig. shows an arrangement of three particles, particle 1 having mass m2
m1 = 6.0 kg and particles 2 and 3 having mass m2 = m3 = 4.0 kg and with
a
distance a = 2.0 cm. What is the net gravitational force F1 that acts on particle x
m3 2a m1
1 due to the other particles ?

Sol. The magnitude of the force F12 on particle 1 from particle 2 is
Gm1m 2
F12
a2
(6.67 1011 )(6.0)(4.0)
F12
(0.020)2

www.physicsashok.in 2
GRAVITATION

F12 = 4.0 106 N



Similarly, the magnitude of force F13 on particle 1 from particle 3 is

Gm1m 2
F13
(2a)2

(6.67 1011 )(6.0)(4.0)


F13
(2 0.020) 2
F12 = 1.0 106 N

the force F12 is directed in the positive direction of y and has only the y-component F12. Similarly, F13 is
directed in the negative direction of x and has only the x-component F13.
y
Thus, the net force on particle 1 is
F12
F1, net (F12 ) 2 ( F13 )2
x
m1
6 2
F1, net (4.0 10 ) (1 10 ) 6 2 F13

F1, net = 4.1 106 N


C2 : Consider two particles of masses m and 4m separated by a distance of r. Find a point where the force exerted
by these two particles on a third particle will be zero.
Sol. Let the third particle of mass m be placed at a distance x from m on the line joining the particles. It must lie
G mm G 4 mm
between the particles so that force may get balance. Then
x2 (r x)2
1 4 1 2
2
2
x (r x) x rx
r x = 2x
r = 3x
x = r/3
The point is between the particles and at a distance of r/3 for m.
C3 : Consider a particle moving in a circle of radius r due to gravitational force exerted by m
another particle of mass M at the centre. Calculate the speed of particle assuming the r
M
central particle at rest.
Sol. Acceleration towards centre = v2/r
GM m
Force towards centre
r2
Using Newtons second law,
mv 2 Mm GM
G 2 , v
r r r
C4 : It is observed that two particles each of mass m are moving around a circle of radius R due to their mutual
gravitational fore of attraction; then calculate the velocity of each particle.
Sol. Here the force of attraction between them provides the necessary centripetal force.
[as the two particles always ramain diametrically oppositive] hence.

www.physicsashok.in 3
GRAVITATION
V
2 2
mV m
G F O F
R 4R 2 m m
Gm R
V
4R V

Example 2: Three particles are located at the corners of an equilatural triangle of side a. Mass of each particle is m.
They are all moving with the same speed in the same sense along the circumstance of the triangle. Find the
speed of each.
Sol. Figure shows forces acting on one particle, say A. Net force is vector sum of the two forces (superposition
principle is obeyed).
V m
Gm 2
Net force towards centre = 2 cos 30 2
a a a

3 Gm 2 r
V 30 V
=
a2 m a/2 m

a
From figure, r cos 30 =
2
r 3 a a
, r
2 2 3
v2 3 v2
Acceleration towards centre
r a
Using Newtons second law of motion
m 3 v2 3 Gm 2 GM
, v
a a2 a

m L m
Example 3 : Three particles each of mass m are placed on the three corners of a A D
square of side L metre. Calculate the gravitational force on the force on the fourth L
particle of mass m placed at the corner D. L
B C
Gm 2 m L m
Sol. Force due to the particle at C = 2 along DC.
L
Gm 2
Force due to the particle at A = 2 along DA.
L
Gm 2
Force due to the particle at B = along DB.
( 2 L)2
Resultant force on the particle m at D
Gm 2 Gm 2 Gm 2
2 cos 45 2 cos 45 , along DB
L L ( 2 L)2
Gm 2 1
2
2 , along DB.
L 2

www.physicsashok.in 4
GRAVITATION

Example 4 : A small mass and a thin uniform rod each of mass m are positioned
along the same straight line as shown. Find the force of gravitational 2L
attraction exerted by the rod on the small mass. L
m
m
Sol. The mass of considered element is
m
dm = dx
2L
dm
Gmdm Gm 2 m
dF = = dx F
(L + x)2 2L(L + x)2 x
2 2L 2 2L
Gm 1 Gm 1
F
2L (L x)
0
2
dx
2L (L x) 0
2
Gm 1 1
F
2L 3L L
Gm 2 2 Gm 2
F
2L2 3 3L2

Example 5 : Figure shows two sphere of mass M each (S1 and S2). A light rigid rod
has two lead balls each of mass m. The rigid rod is suspended from vertical wire.
S1 pulls A and S2 pulls B so that the rod turns by . The twist in suspension wire S2
is producing a restoring torque. d
B
A
If AS1 = BS2 = d, determine the torisonal constant of suspension fibre, S. Find its d
S1
order if G = 6.67 1011 Nm2 kg2, m M ~ 102 kg L ~ 1 m and d ~ 1 mm.
GM m
Sol. Torque due to gravitational forces on A and on B is L . This must be balanced by the restoring
d2
torque = c.
GM m
c L
d2
d
Now,
(L / 2)
Gmm
L
d2
GMmL2
c
2d 3
6.67 10 4 1011 1
c 3
~ 10 4119 ~ 103 Nm rad 1
2 (10)

GRAVITATIONAL FIELD
Can force be exerted by one body on another without contact? Some people think, yes ! A magnet can pull
iron nail from distance, earth pulls an apple from distance and so on. This view is known as action-at-a-
distance point of view.

www.physicsashok.in 5
GRAVITATION

The question that haunted Newton also was : how is it that a body acts on another body from a distance ? He
left the question unanswered but with the hope that there must be something that is mediating interaction; force
cannot be exerted from distance. Today we know that this something is field. Faraday used to think in term of
fields. Further investigations have given rise to certain particles mediating the interactions; we shall not go into
such details but put here only the preliminary developments in the theory of gravitational field.
Gravitational field is created by every mass around it and expends at speed of light. When this field reaches
another mass, it exerts a foce on that mass. Thus one mass exerts force on the field and that field exerts force
on this mass. The field plays intermediary role between two masses.

Gravitational field intensity


Let us put a test mass m0 at a point in a gravitational field created by some sources. The test mass experiences

a force F . This force is found to be proportional to the value of test mass m0.

F m0

We can write equality sign by introducing proportionality constant, say g . Then

F m0 g

Here g does not depend upon m0. Hence it is a vector property of the gravitational field. We call it gravitational
field intersity.
The strength of field at a point is described by force experienced by unit test mass placed at that point. The
force per unit test mass is known as gravitational field intensity.

The value of F given above usually depends upon the location of observation the point and the mass distribution
producting the field. Since the force is mutual, the test mass slightly disturbs the distribution of source masses

and g measured is not due to the given distribution but due to slightly different distribution of mases.
This disturbance is negligible if m0 0. With this is mind the above definition is recast as

F
g lim
m0 0 m
0

Unit and dimensions of g
S.I. unit N kg1
Dimensional formula : [LT2]
Gravitational Field intensity due to a point mass

Let us calculate as g at point due to a point mass M.

For this we imagine a test mass m0 put at P. Then we write force using F using Newtons law of gravitation.
Mm
Thus, F G 2 0 (r)
r
GM
The g 2 r .
r

The minus sign shows that is g is directed towards the source mass M.

www.physicsashok.in 6
GRAVITATION

Gravitational field intensity due to may point masses


We can obtain net gravitational field intensity as the vector sum of field intensities due to each point mass.

If i-th source mass produces field intensity g i , then net field due to i = 1 to N point masses is given by

g g1 g 2 ......... g N
N
g gi
i 1

If the mass distribution be continuous, we have



g dg
where the integration covers the whole body.
C5 : Three particles of equal mass are situated at the corners of an equilateral triangle of side a. How much is the
gravitational field intensity at its centroid ? (2) m

Sol. Let us put a test mass m0 at the centroid. Forces acting on it are F1 , F2 and F3 .
F2
These are of equal magnitude and act at 120 with each other. Hence, r
r
F1 F2 F3 0 F3
r
F1 m
g0 m
(1) (3)
C6 : In the previous example, find the gravitational field intensity at mid point of one side of the triangle.

Sol. The figure shows the situation. F1 is force on test mass m0 at P, due to M at A, etc. F2 and F3 are equal and
opposite.
Hence AM

F2 F3 0

The only force remains is F1 , whose
F1
GMm 0 m0
F1 M M
2
a 3 B P F3 C
F2
2

4 GM
g
3a 2
Thus the field intensity is 4 GM/3a2 in magnitude and directed towards the corner not jointed by the side
containg observation point.

GRAVITATIONAL POTENTIAL
Gravitational potentia at a point is defined as the work done by the field per unit test mass when the test mass
is taken from given point to infinity or standard point.
If W be the work done by the field when the test mass is taken to infinity the potential at the given point
W
V
m0 .
W is dependent upon the force exerted by the mass distribution and the mass distribution gets disturbed if m0
is not very small. Hence to get true gravitational potential m0 should be taken is very small. Thus

www.physicsashok.in 7
GRAVITATION

W
V lim
m0 0 m
0

S.I. Unit : J kg1 or, m2s2


Dimensional Formula [M0 L2 T2]

Gravitational potential due to a point mass


Let M be a point mass producing gravitational field. We want to find potential at point P.
A test mass m0 is taken from P towards infinite. At some position in this course, force on it is given by
GM m0 GM m0
Fx 2 . If the displacement be dx, the work done by Fx is given by dW = Fxdx = dx.
x x2
Now the total work done in moving it from p to W is given by integration. That is,

dx
W GM m 0
r
x2

W GM m0 x 2 dx
r

x 21 GM m 0
W GM m 0
1 r r
W GM
V
m0 r , It is a negative scalar.

Gravitational potential due to many particles


If there be many particles producing the field, the potential at a point is the scalar sum of potentials due to all the
N

point masses. that is, V Vi


i 1

If the mass distribution be continuous we resort to integration V dV


where integration covers the whole system.

Example 6 : Three particles, each of mass m, are located at the corners of an equilateral triangle of side a.
Calculate potential at the centroid.
a
Sol. The distance r of centroid from a corner of the triangle in given by r cos 30 = . m
2
a/2
ra/ 3
a 30
The potential at O due to mass m at the corner is equal to Gm/( a / 3 ).
O
Hence total potential at O due to three masses is given by
m m

GM Gm
V 3 3 3
a a .
3

www.physicsashok.in 8
GRAVITATION

GRAVITATIONAL POTENTIAL ENERGY


The gravitational potential energy of a mass m placed in the field of mass M will be calculated.
GM
Using the definition of potential at site of m, work per unit mass = potential = .
r
now the work done by field in taking m to infinite distance away is given by
GMm
W
r
This work is called potential energy U, of m in the field of M.
Thus U = potential m
(Potential energy = mass potential)
Work done is assembling a system : Work done by field in taking m to is the same as work done by agent
bringing m from to the given point without acceleration. Hence W = potential mass.
In bringing single mass m1, when no other mass is there, no work is done by agent
W1 = 0 ...(1)
The second mass m2 is brought is presence of m1.
Hence W2 = (potential due to m1) m2
Gm1
W2 = r m2 ...(2)
12

m1 m 3
Similarly, W3 = G r r m3 ...(3)
13 23

Summation gives total work done.


W = W1 + W2 + W3
mm mm mm
W G 0 1 2 2 3 3 1
r12 r23 r31
This called mutual potential energy of the assembly of masses.
If summation notation be used, we get
G N mi .m j
W
2 i 1
ri j ,
j i

1 m j .mi m i .m j
where factor of is to take into account the double terms like r and r .
2 ji ij

C7. A particle of mass 1 kg is kept on the surface of a uniform sphere of mass 20 kg and radius 1.0 m. Find the work
to be done against the gravitational force between them to take the particle away from the sphere.
Sol. Potential at the surface of sphere
GM (6.67 1011 )(20)
V J / kg
R 1
V = 1.334 109 J/kg

www.physicsashok.in 9
GRAVITATION

i.e., 1.334 109 J work is obtained to bring a mass of 1 kg from infinity to the surface of sphere. Hence, the
same amount of work will have to be done to take the particle away from the surface of sphere. Thus,
W = 1.334 109 J
Example 7 : Three particles each of mass m are placed at the corners of an equilateral triangle of side d. Calculate
(a) the potential energy of the system, (b) work done on this system if the side of the triangle is changed from
d to 2d.
Sol. (a) As in case of two-particle system potential energy is given by (Gm1m2/r), so
UA = U12 + U23 + U31
Gmm 3Gm 2
So, UA = 3 = C
d d m
(b) When d is changed to 2d, 60
2 d
3Gm
UB
2d
3Gm 2 Am B
So, work done = UB UA = m
2d

Example 8 : Four masses (each of m) are placed at the vertices of a regular pyramid
(triangular base) of side a. Find the work done by the system while m m
taking them appart so that they form the pyramid of side 2a.
a
Sol. Ui = Initial gravitation potential m
Gm 2
energy 6
a
Uf = Final gravitational potential
Gm 2
energy 6
2a
wext = U = Uf Ui
6Gm 2 6Gm 2

2a a
6Gm 2 6Gm 2 6Gm 2 3Gm 2
w ext
a 2a 2a a
Binding Energy
The minimum energy needed to dismantle a system into its constituents far apart is known as binding every of
the system.
Let E1 be energy of bound system and E2 the energy of dismantled system. Then E2 is greater than E1 and
(E2 E1) is the binding energy.
Example 9 : Write the BE of the binary star.
m

v d v

www.physicsashok.in 10
GRAVITATION

Sol. Let two identical stars move around common centre of mass at rest. They must have equal speeds and equal
distance from centre of mass.
m2
The force on one star is F = G 2 , where d is separation between the two stars. This star has the speed v in
d
the circle of radius d/2, Newtons second law of motion gives
mv 2 Gm2
2
(d / 2) d
2 Gm 2
mv ...(1)
2d
Using (1) we can write the kinetic energy K as
1 Gm 2
K mv 2 ...(2)
2 4d
The gravitational potential energy of the system is given by
Gm 2
U ...(3)
d
The mechanical energy is bound state is
Gm 2 Gm 2 3 Gm 2
E ...(4)
4d d 4 d
When the stars are far apart (dismantted), and at rest,
E=0 ...(5)
The binding energy is now written as
Eb = E E
3 Gm2
Eb 0
4 d
3 Gm 2
Eb .
4 d
The binding energy of any bound system is positive.

Example 10 : A particle is projected from the surface of earth with an initial speed of 4.0 km/s. Find the maximum
height attained by the particle. Radius of earth = 6400 km, and g = 9.8 m/s2.
Sol. The maximum height attained by the particle is,
v2
h
v2
2g
R
Substituting the values, we have

(4.0 103 )2
h
(4.0 103 )2
2 9.8
6.4 106
h = 9.35 105 m
or h 935 km

www.physicsashok.in 11
GRAVITATION

Example 11 : An object is projected vertically upward form the surface of the earth of mass M with a velocity
such that the maximum height reached is eight times the radius R of the earth. Calculate :
(i) the initial speed of projection
(ii) the speed at half the maximum height.
Sol. (a)At maximum height, velocity becomes zero.
Applying conservation principle of mechanical energy.
Ui + Ti = Uf + Tf
GmM 1 GmM
mv 20
R 2 9R
1 GmM GmM
or mv 20
2 R 9R
GmM 8 8GmM
=
R 9 9R
4 GM
v0 =
3 R
(b) Ui + Ti = Uf + Tf
GmM 1 GmM 1
or mv02 mv 2
R 2 5R 2
GmM 1 16 GM GmM 1
or m mv 2
R 2 9 R 5R 2
2 2GM
v=
3 5R

Example 12 : A projectile is thrown from the surface of the earth with a velocity v0. It loxses % of its mechanical
energy in doing work against air friction. What height above the surface does it attain if (a) = 0 (b) 0. the
radius of the earth is R and the value of gravitational field intensity is g at the surface.
1 GmM
Sol. The initial energy of the projectile is the sum of kinetic energy mv20 and the potential energy , i.e.,
2 R
mg R. Thus
1
Ei mv 20 mg R
2
GMm GMm mgR
At a height h, the gravitational potential energy will be
Rh h h
R 1 1
R R
The kinetic energy will be zero. Hence the mechanical energy at height h is v=0
v0 h
mgR
Ef
h
1 R
R
(i) (f)
According to question
Ef = Ei % of Ei
(a) For = 0, Ef = Ei .
mg R 1
mv 20 mgR
1 h / R 2

www.physicsashok.in 12
GRAVITATION

v02 1
1
2gR 1 h / R
2 gR h 2gR 2gR v02
2
1 , h R 2
2 gR v0 R 2gR v0
R
h ve
2Rg R
, h where n v
v 2 1 2
n 1 0
0

mgR 1
(b) 1 mv02 mgR
h 100 2
1
R
h mgR h 2gR
1 1
R R
1
100

mgR mv 0 / 2 ,
2
1 2
mgR v 0
100


2gR
h R 1
2gR v 2 1
0
100

Relationship between gravitational potential and field intensity



Let V be potential and g be gravitational field intensity at a point. If a test mass m0 be placed at this point, the
potential energy U and force F, will be
U = mass potential
U = m0V ...(1)

F m0 g ...(2)

If m0 be displaced by dr , work done by gravitational force is m0 g . dr . This must be equal to the fall in
potential energy, dU. That is,
dU = dWcons

d(m0 V) = m0 g . dr

dV = g . dr ...(3)
This is the relationship in differential form. In one dimension the above equation may be written as
dV = gx dx
dV
gx
dx
In two dimension, V is function of two variable. Then we use partial differentiations. We have
V y y
gx
x (x, y)
V V x
gy
y
x
www.physicsashok.in 13
GRAVITATION


Here means differentiate relative to x keeping other variables constant.
x
In three dimensions we have
V V V
gy ; gx and g z .
y x z

[In short g i j k V V ]
x y z
For a radial dependence V(r), we have gr = dV/dr

Here gr is the component of g along r .
Integral form : Integrating eqn. (3), we have
V2 r2

dV g . dr
V1 r1
r2

V1 V2 g . dr
r1

This is the integral for of the relationship.


k
Example 13 : Consider a gravitational potential given by V = where r x i y j z k . Write down x, y and
r

z components of gravitational feild intensity g .

Sol. Here | r | x 2 y2 z2

k
V(x, y, z) = x 2 y2 z 2

V k
gx
x x x 2 y 2 z 2

d(x 2 y 2 z 2 )1/ 2 2
gx k 2 2 2
(x y 2 z 2 )
d(x y z ) x
k 2
gx (x y 2 z 2 ) 3/ 2 . 2x
2
kx kx
gx 2 2 2 3/ 2
3
(x y z ) r
ky
Similarly, gy
r3
kz
gz 3
r

k
kr
g 3 (x i y j z k)
r r3

www.physicsashok.in 14
GRAVITATION

Gravitation potential and field intensity at a point on the axis of a uniform ring
The figure shows a uniform ring of mass M and radius R. On its axis is situated a point P, at a distance x from
the centre.
Let us imagine the ring to be made up of elements of masses like dM. Potential at P due to dM is given by
dM dM
dV = G 2 2 . x2 + R2
x R
R
Potential at P due to the whole ring is given by M
x P
dM
V G
x2 R 2
ring

G GM
V
2
x R 2 dM x2 R2
ring

Here x 2 R 2 is slant distance of P from the ring.


dV
Field intensity : As V = V(x), we use gx = .
dx
Putting the expression for V,
d GM
gx
dx x 2 R 2

d(x 2 R 2 )1/ 2 d(x 2 R 2 )


g x GM
d(x 2 R 2 ) dx

1
g x GM (x 2 R 2 )3 / 2 . 2x
2

GMx
gx
( x R 2 )3 / 2
2

GMx
Plots : The graph of V(x) is shown in the figure. Its slope is which is zero at x = 0.
(x R 2 )3/ 2
2

V(x)

x
GM slope = zero
R

The plot of gx against x is as shown. We see that

www.physicsashok.in 15
GRAVITATION

(i) x=0 gx = 0
GM gx
(ii) x << R gx = x
R3
GM
(iii) x >> R gx = x
x2
(iv) x gx = 0

Example 14 : A point P lies on the axis of a fixed ring of mass M and radius a, at a distance a from its centre C. A
small particle starts from P and reaches C under gravitational attraction only. Its speed at C will be_______.
Sol. Machenical energy consevation law
Ui + Ti = Uf + Tf
GmM GmM 1
0 mv 2
a2 a2 a 2

GmM 1 1 2
or 1 2 mv
a 2

2GM 1
or v2 1
a 2

2GM 1
v 1
a 2

Example 15 : Find the gravitational field strength and potential at the centre of arc of linear mass
density subtending an angle 2at the centre.
2
2G R
Sol. E sin
R 2
E
2G 2
E sin
R 2
2G
E sin
R
G m
V
R
m = (2R)
m = 2R
G
V 2 R v 2G
R
Example 16 : Consider a uniform semicircular rod of mass m and length L. An equally massive particle is placed
at the centre of curvature. Calculate the force acting on this particle.
Sol. The length of the rod is half of perimetre of a circle.
r = L
r = L/ ...(1)
The mass per unit length of the rod is m/L.

www.physicsashok.in 16
GRAVITATION

Let us divide the rod into elementary masses, each of mass dm contained in angle d at position (see figure).
Then dm = m/Lr d. = /2
The force exerted by this point mass on the mass m at centre

of currature is dF . It has two components dF and dF sin .
There is a symmetrical mass dm at whose effect is to
cancel out dF sin . The only contribution remains uncancelled
is dF cos . Hence net force Fnet on the mass is givien by dm
dF sin
d
dF
Fnet dF cos
=0 m
whole wire
dFcos
dF
dm
Fnet Gm 2 cos dF sin
r
m cos
Fnet Gm d
L r
/2
Gm 2
Fnet
rL cos d
/ 2 = /2

Gm 2 Gm 2
Fnet [sin ]/ 2/ 2 , Fnet 2
rL rL
Putting r = L/
2 Gm 2
We have Fnet
L2
This force is directed towards the mid point of the semicircular wire.

Gravitational potential and field intensity due to a uniform Spherical Shell


Let us consider a uniform spherical shell of mass M. Its mass per unit area will be M/4R2 = .
Let us consider a strip of width R d and radius R sin (see figure). The whole shell is made up of such strips.

M Rd
z
d
R sin
x P
Uniform
spherical R
shell

Potential at P due to the ring is given by


dM
dV = G
z
2 R 2 sin d
dV = G ...(1)
z
Now, z (x R cos ) 2 (R sin ) 2
z2 = x2 + R2 2 xR cos

www.physicsashok.in 17
GRAVITATION

Differentiating, we get
2 xR sin = 2z dz
z dz
sin d= ...(2)
xR
From (1) and (2)
GM z dz
dV = .
2z xR
GM
dV = dz ...(3)
2x R
Case I : P is outside the shell.
In this case minimum value of z is x R and maximum value is x + R. Therefore,
x R
GM
Potential V
2x R dz
xR

GM GM
V . 2R
2x R x
GM
V
x
The gravitational field intensity is obtained by using,
dV d GM
gx
dx dx x
GM
gx 2
x
The shell is acting as if it were a point mass located at the centre.
Case II : Point P lies inside the shell.
Here Zmin = R x x Z
x R P
GM
V
2x dz R
Rx

GM
V gx = 0
R
Potential at all the inside points is equal to that at the surface.

V(x)

x=R x

GM
R not differentiable

The plot V(x) is shown here. At x = R, the plot has a corner where it is not differentiable. The plot of g(x) has
a x finite discontinuity at x = R.

www.physicsashok.in 18
GRAVITATION

g(x)
gx = 0 discontinuity
x=R x

gx = GM
x2

C8. Two concentric spherical shells have masses m1 and m2 and radii r1 and r2(r2 > r1). What is the force exerted
by this system on a particle of mass m3 if it is placed at a distance r (r1 < r < r2) from the centre.
Sol. The outer shell will have no contribution in the gravitational field at point P. m2
Gm1 m1
EP
r2 O m3
r1 r
Thus, force on mass m3 placed at P is, P
F = m3EP r2

Gm1m3
or F
r2

The field EP and the force F both are towards centre O.

Gravitational potential and field intensity at a point due to homogenous solid sphere
The point where potential and field are to be evaluated may be (i) outside the sphere (ii) inside the sphere.
Case I : Outside point.
We assume the sphere to be made up of concentric spherical shells, one of which is shown. Its potential at P is
given by
dV = GdM/r
Integration gives dm
G
r
V= dm

GM
V
r
For observation at outside points, sphere like a point mass located at its centre.
dV d GM GM
gr 2
dr dr r r

GM
gr
r2
The solid homogenous sphere acts as a point mass at its centre.
Case II : Inside point : Potential at P due to the whole sphere may be treated as the sum of two potentials : The
potential due to
(i) Sphere of radius OP, V1 and
(ii) Hollow sphere of inner radius OP = r and outer radius R, V2.

www.physicsashok.in 19
GRAVITATION

(i) Potential (V1) due to sphere of radius OP.


Here P lies at the surface of this sphere. Hence
O r
4 / 3r 3 P
V 1 G
r R
M M
where = density =
V 4 / 3R 3
(ii) Potential (V2) due to hollow sphere : The hollow sphere is assumed to be made up of spherical shells. One
such shell, shown in the figure is producing potential at P given by
dM
dV2 G
x dM
4x 2dx P
dV2 G x
x dx
Due to the whole hollow sphere we have
R
V2 4G xdx
r

4G 2 2
V2 (R r ) ...(ii)
2
Thus, using (i) and (ii) potential at P is given by
V = V1 + V2
4
G r 3
3 4 2 2
V G (R r )
r 2
G 4r 2 G 4 2 2
V (R r )
3 2
r2 R 2 r2
V G 4
3 2

3R 2 r 2
V 4 G
6

GM 3R 2 r 2
V 2
4 / 3R 3 3
GM
V (3R 2 r 2 )
2R 3
This is as function of r. The gravitational field intensity along radial direction denoted by.
It is given by
dV GM
gr 3
(0 2r)
dr 2R
GM
gr 3 r
R
The graphs of V(r) and gr are shown below. Mark that | gr | is maximum at the surface of the sphere.

www.physicsashok.in 20
GRAVITATION

V(r)

r=R r

outside sphere
(hyperbola)
3GM
2R
inside sphere
(Parabola)

Table : Field and Potential


Source Potential (V) Field intensity (gr)
GM GM
Point mass
r r2
GM GM x
Ring
x R2 2 (x R 2 )3/ 2
2

GM
R , r R 0,

rR
Spherical Shell GM
GM , r R r 2 , r R
r
GM 2 2 GM
2R 3 (3R r ), r R R 3 r, r R
Solid Sphere
GM , r R GM , r R
r r 2

C9 : A tunnel is made in the earth along its chord at a distance of R/2 from the centre.
A mail is released from the mouth of the tunnel. there is no friction between the N
main and the tunnel wall. Calculate the normal force exerted by the tunnel on the mg cos
mg sin
mg
mail if its mass is m. The mass of the earth is M and radius R. R/2
Sol. Forces on the mail are (i) mg, towards the centre of the earth (ii) N, normal to tunnel
Here g = GMr
or, N = mg cos
GM
N = m 3 r cos
R
GM R R
N=m r cos
R3 2 2

GM m
N=
2R 2
Thus, N is independent of ; it depends on R.

www.physicsashok.in 21
GRAVITATION

C10 : A spherical mass of radius r = R/2 is taken out from a sphere of radius R and density . Calculate the force
which this sphere having a cavity will exert on a mass m placed at a distance of x from its centre (x > R).
Sol. For the whole sphere of radius R
Mm
F1 G
x2
4 / 3 R 3m R
F1 G P
x2 O r = R/2
and for small sphere of radius
r = R/2
M1m G4 / 3 (R / 2)3 m x
F2 G
(x R / 2) 2 (x R / 2)2
Resultant force = F1 F2
4 1 1
R 3 Gm 2 2
3 x 2(2x R)

KAPLERS LAWS OF PLANETARY MOTION


Before we actually put the kinematical laws of motion of planets given by Kepler, let us familiarise ourselves
with an ellipse.

F2

F1 ellipse

Activity for an Idea of an ellipse : Take a drawing broad. Fix on it a graph paper. Fix two pins (F1, F2) on the
paper. Take a thread whose length in larger than the distance between the two pins. Tie its ends to each of the
pins. Take a pencil and pull the thread by it as in the figure. Move the pencil keeping the thread tight.
You get a curve called an ellipse. The nails are two focii (plural world for focus) of the ellipse.
Draw a line through F1 and F2. It cuts the ellipse at A and B. The length AB is called A
major axis of the ellipse. Now draw a perpendicular bisector of the major axis. It is
CD. It is called minor axis of the ellipse (because CD < AB as shown in the figure). F2
2a
The ratio OF1/OB or OF2/OA is known as eccentricity (e) of the ellipse. It is a measure C D
of how much distorted the ellipse is releative to a circle (e = 0). Using the graph paper,
you may find the area of the ellipse and compare with the standard formula, to judge F1
how good your ellipse is :
B
Area = ab 2b

The Keplers laws : Kepler (15711640) used an ellipse to describe his three laws of planentary motion. The
three laws are :

www.physicsashok.in 22
GRAVITATION

(a) Law of orbits,


(b) Law of areal velocity and
(c) The law of periods.
We shall now state and discuss these.

The Law of orbits


The closed path along which a planet moves around the sun is called an orbit. the first law gives its geometry
and location of the sun relative to the orbit.
Every plate moves around the sun in an elliptical orbit with the sun at one of the focii.
P
In the figure sun is located at the focus F1, and F2 is vacant. The planet P is
moving along the ellipse. Another planet will move in another ellipse but one focus F2
of that ellipse will always be at the sun.
S F1

The Law of Areal velocity or sector velocity


If we join a planet (P) to the sun (S), the line (SP) turns as the planet moves. It sweeps some area in given time.
The area swept per unit time is known as areal velocity (or sector velocity) of the planet. The Keplers second
law tells us that this areal velocity remains uniform.
The line joining a planet to the sun sweeps equal and in equal time interval.
P P
Q
(Aphelion)
(Perihelion)
S F1 F2 slow
Fast
Q

Let a planet move, from P to P in a time t, covering area SPP. The planet goes from Q to Q in the same
time interval t. Then, according to Kepler,
Ar (SPP) = Ar (SQQ)
If A be the area and t the time taken then,
A
= constant.
t
When the Earth, for example, is closest to the sun (at perihelion), its speed in larger than when it is farthest from
the sun (apehelion).

The Law of Periods


A planet takes some time to move once around the sun. This time is known as time period of the planet. If
several planets are moving around the sun, they have their own periods. The time period of the earth for motion
around the sun, for example, is one year. The law of periods connects this time period to size of ellipse. The law
was formulated by Keplers in 1619.
The square of period of revolution of planets around the sun are directly proportional to the cube of
semimajor axes of respective orbits of the planets.
Let T1 be the period of planet-1 moving in an ellipse of semimajor axis a1. Similarly T2, a2 are period and
semimajor axis for planet-2, T3, a3 for planet-3, etc. Then

www.physicsashok.in 23
GRAVITATION

T12 T22 T32


= ....... = constant.
a13 a 32 a 33

In short, T2 a3.
Notice the Table giving the value of constant.

Table : Law of Periods


Planet Semi-major axis (a) Period (T) Ratio T2/a3
( 1010 m) (years) ( 1034 year2/metre3)
Mercury 5.79 0.24 2.95
Venus 10.8 0.615 3.00
Earth 15.0 1 2.96
Mars 22.8 1.88 2.98
Jupiter 77.8 11.9 3.01
Saturn 143 29.5 2.98
Uranus 287 94 2.98
Neptune 450 165 2.99

The Physical basis



The sun exerts a force of pull ( F ) on a planet that passes through the centre of mass of the planet. Newton
discovered the law that gives the force : It is central and attractive (always directed towards the fixed point, the
sun), and obeys inverse square law of distance (the force is inversely proportional to square of distance between
the sun and the planet).
The torque of this force about the sun is zero. Hence angular momentum of the planet about the sun is conserved.

If r be the position vector and v be the velocity of the planet, the angular momentum L of the orbiting planet
is given by
v
m
L r mv L d
r
Now the area of triangle formed by v dt and r can be represented by F planet
geometrical meaning of cross product of two vectors. Thus L
1 p
dA | r v dt | r
2
Using this we have

| L | | r v| m
dA
| L | 2 m
dt

The direction of L is constant and magnitude is also constant. Thus,

dA | L |
= constant
dt 2m

www.physicsashok.in 24
GRAVITATION

The law of areal velocity is, thus, consistent with conservation of angular momentum of the planet around the
sun. In turn, it bases on the central nature of force between the planet and the sun.
The other two laws are consistent with inverse square and attractive nature of gravitational force. Demonstrating
these by calculations is straight forward but a bit cumbersome and not necessary at this level.
Since a circle is also an ellipse with zero eccentricity, we have
T2 = kr3 (Law of periods).
2
2 2
Also F m r m r
T

4 2 m
F .r
kr 3
4 2 m
F
kr 2
1
Thus F
r2
Thus the periods law is consistent with inverse square attractive force.

The value of constant in Keplers law of periods for planetary motion


The square of the period of revolution of a planet around the sun is proportional to the cube of semimajor axis
of its orbit.
T2 a3
T2 = ka3 ...(1)
k is the constant of proportionality.
To find k which is independent of the value of a, we take circle as a special case of an ellipse, where major
axis equals minor axis
Here we have P
GmM s a
F= S
a2
mV 2 GmMs

a a2
GMs
v
a
2a
T
v
a2 4 2 3
T 2 4 2 a
GM s GM s ...(2)

a
From (1) and (2)
4 2
k
GM s

www.physicsashok.in 25
GRAVITATION

Example 17 : A saturn year is 29.5 times the earth year. Find the distance of saturn from the sun if the Earth is at
1.50 108 km from the sun.
Sol. Using Keplers law of periods,
2 3
TS rS
T r
E E

2 /3
T
rS S . rE
T E

2/3
29.5 y
rS . 150 108 km .
1 y
We need log table for exact evaluation. However we have only three significant calculation and can use binomial
method.
2/3
2/3 2/3 2/3 2.5
(29.5) = (27 + 2.5) = 27 . 1
27

2 2.5
(3 3 3)2/3 1 3 27 = 9.5 (Exact evaluation gives it as 9.5413)

Using this rS = 14.25 108 km


rS = 1.42 1012 m

To show that Keplers laws are consistent with Newtons laws of gravitation m
F
Let a planet be moving in an elliptical orbit around the sun. Since the mass of the sun is very R
MS
large than that of the planet, the sun will exert a gravitational force (attractive) on the planet.

G mM s
F ...(1)
R2
Since the eccentricities are small and sun is more massive than the planets, the path can be assumed as
approximately a circle. The centripetal force, exerted by the sun on the planet, is given by
2
mv 2 m 2R
Fc ...(2)
R R T
From (1) & (2)
GmM m 4 2 R 2

R2 R T 2

T 2 4 2

R 3 GM
Here, 42, G and M are constants, therefore,
T2
= constant.
R3

www.physicsashok.in 26
GRAVITATION

T2 = (const) R3
T2 R3
This is the law of periods.
Areal velocity
R 2 R 2
= constant.
T 4 2 R 3
GM

Thus the areal velocity in circular orbit is constant.

C11 : The speed of a planet at perihelion is v0 and the distance from the sun is l. The
ae
major axis of elliptical path is 2a. Determine the speed at aphelion. If e be the ae a
l
eccentricity of the ellipse l = a(1 e). Use it to expresses your answer. S
v0
Sol. At perihelion the angular momentum about the sun-axis normal to plane of the orbit, is mv0 l where m is the
mass of the planet. When it travels to aphelion let its speed be v. Then distance from the sun is (2a l). The
angular momentum about the above axis is now mv(2a l).
Since action line of the force exerted by the sun on the planet is passing through the sun, angular momentum
about the sun is conserved. Hence
mv0l = mv (2a l)
v0 l v
v
2a l 2a / l 1
Using l = a (1 e),
v0
v
2
1
1 e
1 e
v
1 e .
Example 18 : A body is launched from the earths surface at an angle = 30 to the horizontal at a speed
1.5 GM
v0 . Neglecting air resistance and earths rotation. Find the height to which the body will rise.
R
Here M is mass of earth and R the radius of earth.
Sol. Let velocity at highest point be v and R + h = r
Applying conservation of angular momentum between P and Q, we have Q
v
mvr = mv0R cos 30
3 v0 R =30 r
or v ...(1)
v0
2r R
R P
Applying conservation of mechanical energy between P and Q, we have :
1 GMm 1 GMm
mv02 mv 2
2 R 2 r
substituting the value of v from equation (1), we get

www.physicsashok.in 27
GRAVITATION

1 2 GMm 1 3v 20 R 2 GMm
mv0 m
2 R 2 4r 2 r

2 2GM 3v02 R 2 2GM


or v
0
R 4r 2 r
1.5GM 2GM 3 1.5GM R 2 2GM 1.5 GM
or v0 is given
R R 4 R r2 r R
1 9 R2 2
or .
2R 8R r 2 r
or 4r2 = 9R2 16Rr
or 4r2 16Rr + 9R2 = 0

16R 256R 2 144R 2


or r
8
16R 10.58R
r= = 3.323 R and 0.677 R
8
but r R
Hence r = 3.323 R or h = r R = 2.323 R

ACCELERATION DUE TO GRAVITY AND ITS VARIATION


The force on a particle near the earth can be written as

F = mass gravitational field intensity
Newtons second law of motion gives

F = mass a grav
where agrav is the acceleration due to gravity.
This gives gravitational field intensity mathematically equal to the acceleration due to gravity.

a grav g (gravitational field intensity)

Variation in acceleration due to gravity g with altitude and depth


If we assuming that the earth is a homogenous sphere (which she is not exactly) the gravitational field intensity
may be written as
GM
g= (outside)
r2
GM
g= r (inside)
R3
where r = distance of observation point from the centre of the earth.
The above expression shown that g decreases when we go away from the |g|=g
surface either inward (depth) outward (altitude).
For small height h above the surface r=R r

www.physicsashok.in 28
GRAVITATION

GM GM
g
(R h) 2 h
2
2

R 1
R
2
GM h
g 2
1
R R
GM
Now, g 0 (surface value
R2
2
h
g g0 1 [which h << R]
R
2h
g g 0 1
R
[Using binomial theorem (1 + x)n 1 + nx when | x | << 1.]
If we go inside the earth at depth h
GM
g (R h)
R3
h
g g 0 1
R
The above discussion shows that g varies at faster rate is going upward than in going inward from the surface
of the earth.
As we go upward initially g falls at faster rate but at later distance its rate of fall becomes smaller-smaller
whereas when we go inside the earth g goes on decreasing at constant rate becoming zero at the centre.

C12 : The weight of a man on the surface of the earth is 50 kg, what would be his weight at the centre of earth ?
Sol. Zero
[Since g at the centre of the earth is zero.]

C13 : In above question, at what depth below the surface of the earth the weight of the man will be half of that on
the surface of the earth ?
Sol. Acceleration due to gravity at a given depth.
h
g1 g 1
R
g1 h 1 h
1 or, 1
g R 2 R
h 1 1 R 6400
1 h= = = 3200 km.
R 2 2 2 2

C14 : In the above question, at what height the weight of the man will be half of that on the surface of the earth.
Sol. The acceleration due to gravity at a given height g.
2 2
R g1 R
g1 g
R h g R h

www.physicsashok.in 29
GRAVITATION

2
1 R Rh
or 2
2 R h R
h
2 1 R
h = 0.41 R = 0.41 6400 = 2624 km
C15 : Calculate the value of acceleration due to gravity at a height R from the surface of the earth, where R is the
radius of the earth.
2 2
g1 R 1 1
Sol.
g R h 1 h / R 4
g1 = g/4
C16 : Calculate the % change in acceleration due to gravity on the surface of the earth, if radius of the earth shirnks
by 2%. (assuming density to be same).
Sol. It will decrease by 2 %.
M 4 R3 4
gG 2
G 2 GR
R 3 R 3
g R; if is constant.
C17 : How high a man will be able to jump on a surface of a planet of radius 320 km, but having density same as
that of the earth, if he jumps 5 metre on the surface of the earth (radius of the earth = 6400 km).
GM 4
Sol. For the earth, g RG
R2 3
GM1 4
For the planet; g1 R 1G
R 12 3
g R 6400
20
g1 R 1 320
Let h and h1 be the distance upto which the man can jump on surface of the earth and planet, then
mgh = mg1h1
g
h1 = g h = 20 5 = 100 m.
1

C18 : How high a man can jump on the surface of the moon if he jumps 5 metre on the surface of the earth,
gm = 1/5 ge.
Sol. gehe = gmhm
ge
hm = g he = 25 meter..
m
C19 : Calculate the value of acceleration due to gravity on the surface of a planet whose mass is double of the
earth and radius half of the earth.
GM GM1
Sol. For the earth, g = ; and for the planet, g = R 12
R2 1

2 2
g1 M1 R 2M R
= 2 4 = 8.
g M R1 M R / 2
g1 = 8g = 78.4 m/sec2.

www.physicsashok.in 30
GRAVITATION

C20 : Calculate the value of acceleration due to gravity on a planet whose radius is half that of the earth but the
mean density is same as that of the earth.
GM 4
Sol. We have g RG
R2 3
for same density, g R
g1 R 1 R g
or, g g 4.9 m/sec2.
g R 1
2R 2
Example 19 : Calculate the distance from the surface of the earth at which above and below the surface acceleration
due to gravity is the same.
GM
Sol. Acceleration above the surface of the earth = 2
R+h ( )
d
Acceleration below the surface of earth = g 1
R
According to question
GM d
2
g 1
R h R
GM h
or 2
g 1 { d = h }
R h R
2 2
GM h h h h
or 2
1 g 1 or g 1 g 1
R R R R R
1 h
h
2
h 1
h
2
R
or 1 1
R
or
1
R R
h h
2
h 2 2h h
or 1 1 1 or 1 1 2 1
R R R R R
h 2 2h h h 3 2h 2
or 1 1
R2 R R R3 R2
h2 h h3 h h h2
0 2 3 1
R R R R R R 2 = 0
h2 h R 2 h2 h
or 1 or
R2 R R2 R
or R2 h2 = Rh
or h2 + Rh R2 = 0
R R 2 4R 2 R 5 R
h h
2 2
Physically possible value is

h
5 1 R
2

www.physicsashok.in 31
GRAVITATION

C21 : A mail is dropped along a smooth a tunnel along diameter of the Earth. assuming the Earth a homogenous
sphere determine the type of motion executed by the mail.
Sol. When the mail is at distance r from the centre, the force exerted on it is directed
r
GMm
towards the centre and is r.
R3

GMm
In vector form, F r
R3
This shows that the particle is pulled towards the centre, at the centre its velocity is maximum but force is zero.
Due to inertia of motion it is deflected beyond the centre where it is again pulled back and decelerates. It stops
at the mouth of the tunnel and is pulled back again. Thus the mail performs oscillations.

Shape of the earth : The core of the earth is more massives and is near to centre
responsible almost wholly for the value of g. If a point is nearer to oblate spheroid
Low
this core (like polar points) the attraction is largter or g is larger. density
On the other hand at the equator the point is further away from High (farther from
the core where attraction is smaller. Thus g is smallest at the density centre)
equator and largest at the pole.
Variation in crust density : The density of crust is not uniform g is
smaller if the density of curst below is smaller due to presence of low density material like oil and gas. Similarly
g is larger if high density material like some iron ore is lying beneath the crust.
Effect of rotation : If we use Earth frame, it is a rotating frame. An object at rest on it feels a centrifugal force
directed normally away from rotation axis. It, in effect, reduces the value of g. At equator g = g R.
At the poles the effect of rotation on is zero.

Weight
the force with which a body acts on the support that prevents it from free fall in a reference frame is called to
weight of the body in that reference frame.
In a inertial reference frame, weight equals the gravitational pull by all the bodies in the universe. On a planet,
it equals the pull of the planet. In interstellar space the pull becomes faint and it is called true weightlessness.

Apparent weightlessness is observed in a frame falling (rising) freely with acceleration g produced by all the
gravitating bodies.

Interial and Gravitational masses


The mass due to which particle take part in gravitation in known as gravitational mass. The mass due to which
it opposes any change in its natural state of motion or rest (i.e., shows its inertia) is called inertial mass.
Hence a body is likely to posses two kinds of masses inertial and gravitational. Let m be inertial mass and m
be gravitational mass. Then the gravitational force is decided by gravitational masses.
GMm
F
r2
If a be the acceleration produced in the body due to gravitational force, Newtons second law of motion tells
that it is equal to ma where m is inertial mass. Hence.

www.physicsashok.in 32
GRAVITATION

GMm
ma
r2
GM m
a
r2 m
Near the earth, all the objects fall with the same acceleration. This was first notated by Galileo and is true.
Hence m/m= constant
The difference between m and m were measured experimentally. Etvos measured equality of m and m upto
12th place after decimal. In the mean time Einstein floated theory of relativity.
If we consider a freely falling lift, bodies becomes weightless in it. There is no compression or elongation in the
spring balance

mg
g FS = 0
mg = mg
mg

Gravitational force can be cancelled in lift only if m = m.


Einsteins equivalence principle states that gravitational and inertial masses are the same.
The question of two kinds of masses is solved there is only one type of mass in a body playing both of the
roles.

ORBITAL SPEED OF A SATELLITE


A body moving in a closed orbit around a planet is called its satellite. The Keplers
laws are applicable to it. We shall consider approximately circular orbit for the
satellite. The centripetal force is produced by gravitational force of a planet. Using
M
dynamics of circular motion, we have r
m>>M
2 m
mv GMm
v
r r2

GM
v
r
This is called orbital velocity of a satellite. It is independent of mass of satellite only if m << M.
If the satellite moves around the earth near the surface
r R = 6400 km
GM GM
v R gR
R R2
g 10 ms2
1
v 64 106 ms 1 = 8 kms
www.physicsashok.in 33
GRAVITATION

Time period : The length 2r is covered with uniform speed v.

2r 2 r r 3/ 2
T 2
v GM GM
r
Square of T is proportional to cube of r (Keplers law of periods)

2R 3/ 2
the smallest time period = [ rmin R ]
GM

2R 3/ 2 2 3/ 21
R
GM 2 g
R
R2

R
2 . ( 84 min)
g
The period of natural satellite of the earth, the moon, is 27.3 days.

Example 20 : A satellite is revolving round the earth at a height of 6 105 m. Find :


(a) the speed of satellite and
(b) the time period of the satellite.
Radius of earth = 6.4 106 m and mass of the earth = 6 1024 kg.
Sol. The height of satellite from earths centre
= 6.4 106 + 6 105 m
= 7.0 106 m
(a) The speed of satellite is
GM
v
r
6.67 1011 6 10 24
v
7 106
v = 7.6 103 m/s
(b) The time period is
2r
T=
v
2 7 106
T=
7.6 103
T = 5.8 103 s

C22 : A satellite is rotating near the surface of the earth in a circular orbit; then calculate its speed.
mV 2 mM
Sol. We have, G 2
R R

www.physicsashok.in 34
GRAVITATION

GM
or V
R

but gR2 = GM V gR

9.8 6400 103 8 km / sec.


C23 : In the above Qus; calculate the time period of the satellite.
mV 2
Sol. 1. = mg or, V2 = Rg 2R2 = Rg
R
2
2 R
R g T 2
T g

VT 2 R
2. T = 2 or = 2 or, T
R V
C24 : Calculate the valocity of a satellite which is revolving round the earth in a circular orbit at distance of 6400
km above the surface of the earth.

GM V2 R1 6400
Sol. Since, V
R V1 R2 12800

V1 8
V2 4 2 km/sec.
2 2
C25 : In the above problem, calculate the time period of the satellite.
mV 2 GmM 2 GM
Sol. We have, 2 or, V
R R R
GM 2
2
GM 4 2 3
2R 2 or, R 2
2
or T R .
R T R GM
Hence T R 3/ 2
If T1 and T2 be the time period of the satellite near the surface of the earth and at a height 6400 km above the
surface of the earth, then
R 2 12800
2
R 1 6400
3/ 2
T2 R 2
23/ 2 2 2
T1 R1
T2 = 2 2 T1 = 2 2 84 = 238 minutes nearly..

GEOSTATIONARY SATELLITE
If a satellite remains stationary relative to a place on the Earth, it is called geo-stationary satellite. For the
satellite,

www.physicsashok.in 35
GRAVITATION

GMm
m 2e r e
r2
N
1/ 3
GM F
R 2 h

e S
Putting the values, we get
r 42400 km (from centre)
h = 36000 km (above the surface).
This satellite is used for communication. Hence it is also called communication satellite.
Parking orbit is the Orbit in which the satellite is placed. A geostationary satellite is parked only in an equational
orbit.
If such a satellite is attempted in non-equatorial orbit the gravitational force by planet will have a component
normal to the plane of the circle. This force will take the satellite in helical path. The orbit will be unstable.
Hence the satellite will not be geostationary.
A communication satellite will directly communicate with places with co-lititude not less than as in the figure.
R
We have sin N
r latitude
co-latitude
6400 km 1
sin
42400 km 7 r
1
sin 1 8
7 S Under direct communication

The lattitude will be
2
1
cos 1 82
7
C26 : Calculate the gain in the potential energy of an object of mass 2 kg. When it is raised from the surface of the
earth to a height equal to the radius R of the earth.
GMm
Sol. P.E. on the surface of the earth = .
R
GMm
P.E. at a ht. R above the surface of the earth =
2R
GMm GMm
gain in P.E. dU =
2R R
GMm mgR
= gR
2R 2
Example 21 : A satellite close to the earth is in orbit above the equator with a period of rotation of 1.5 hours. If it
is above a point P on the equator at some time, it will be above P again after time______.
Sol. v 0 = gR
R gR
g

R

www.physicsashok.in 36
GRAVITATION

2 g
or
Ts R
1 1 g 1
or { 2 g }
Ts 2 R 2 R

1 1

Ts 2 6400 103
1 1 1
= =
Ts 1600 10 5060
1
0.000198 ...(1)
Ts
1 1
...(2)
TE 24 60 60
For an observer at the surface of earth
2 2
T (when rotating from west to east)
s E 2 2
Ts TE
1 1 1
= 0.000189
T Ts TE
T = 5576 second = 1.6 hour
Similarly, for rotating from east to west.
1 1 1
= + ...(3)
T Ts TE
Putting the values, from eqn (1) and (2) in eqn (3)
24
T= hour
17

Polar Satellites
These pass through polar regions, are at attitudes of 500 ~ 800 km and scan the earth surface is each revolution.
Information from there are used in remote sensing, meteorology and studies of earths environmental science.

C27 : A polar satellite has a period of TP while the Earth turns around its axis at a period of TE. Neglecting orbital
motion, calculate the angular shift of satellite orbit per revolution relative to earth.
Sol. Relative to the earth the orbit is rotating at angular speed of the earth in opposite sense to that of the Earth. The
angle turned E

| E | TP
2
TP
TE
T
2 P
TE

www.physicsashok.in 37
GRAVITATION

Example 22. If a satellite is revolving around a planet of mass M in an elliptical orbit of semi-major axis a, show
that the orbital speed of the satellite when it is at a distance r from the focus will be given by :
2 1
v 2 GM
r a
GMm
Sol. As in case of elliptical orbit of a satellite mechanical energy E remains constant, at any position
2a
of satellite in the orbit,
GMm
KE + PE = ...(i)
2a
Now, if at position r, v is the orbital speed of satellite,
1 GMm
KE = mv2 and PE = ...(ii)
2 r
So from Eqs. (i) and (ii), we have
1 GMm GMm
mv 2
2 r 2a

2 1
i.e., v 2 GM
r a
Example 23 : Consider two satellites A and B of equal mass m, moving in the same circular orbit
of radius r around the earth E but in opposite sense of rotation and therefore on a A r B
collision course (see figure). Me
(a) In terms of G, Me, m and r find the total mechanical energy EA + EB of the two satellite
plus earth system before collision.
(b) If the collision is completely inelastic so that wreckage remains as one piece of tangled material
(mass = 2m), find the total mechanical energy immediately after collision.
(c) Describe the subsequent motion of the wreckage.
GmMe 1 2
Sol. (a) Ei mv 0 2
r 2

GmMe 1 GMe GmM e


Ei m 2, Ei
r 2 r r
(b) According to conservation principle of momentum,
Pi = Pf
mv0 mv0 = Pf
Pf = 0
It means just after collision, kinetic energy becomes zero.
Total mechanical energy after collision is
Ef = P.E. + K.E.
GmMe GmMe 2GmM e
Ef 0 Ef
r r r
(c) After collision, combined mass starts to move towards the centre of earth due to gravitational attraction of
earth.

www.physicsashok.in 38
GRAVITATION
M
Example 24 : Two planets of equal radius R are separated (centre-to-centre) by 8R. One
has a mass M and the other a mass 4 M.
A small body is projected from the planet of mass 4M towards the centre of the other 8R
planet. What should be its minimum speed seas to reach the other planet ? u

Sol. The projectile should be able to reach the neutral point. As it crosses it, the other
4M
planet will automatically pull it.
Neutral point : Let its distance from 4M be x. Then a particle at this point will feel equal attraction by two planets.
Thus,
G(4M) m GM m
2

x (8R x) 2
4(8R x)2 = x2
2(8R x) = x
16 R = 3x or x = 16 R
16
x R or x = 16 R
3
At x = 16R the magnitude is equal but direction is not opposite. It is not a neutral point. Hence x = 16/3R.
Now we consider energy conservation. Kinetic energyof the projectile at starting point is 1/2mv2.
Potential energy is written as
G(4M) m GM m
U
R 8R R
Thus, total mechanical energy of the projectile at the surface of launch planet is
1 GM m 1
Ei mv 2 4
2 R 7
1 29 GMm
Ei mv 2 .
2 7 R
When the projectile reaches neutral point, its velocity is negligible. Only then projection speed v will be a
minimum. Thus,

G (4M) m GM m
Ef 0
16 R
8R
16
R
3 3
GM m 3 3 3 GM m
Ef
R 4 8 4 R
As the gravitational force is conservative,
Ef = Ei
3G Mm 1 29 GMm
mv 2
4R 2 7 R
1 29 3 GMm
mv 2
2 7 4 R

95 GM
v .
14 R

www.physicsashok.in 39
GRAVITATION

Example 25 : A hypothetical planet of mass M has three moons each of equal mass m each m
revolving in the same circular orbit of radius R. The masses are equally spaced
and thus form an equailateral triangle. Find : R
(i) the total P.E. of the system m m
(ii) the orbital speed of each moon such that they maintain this configuration.
Gm 2
Sol. (i) U 3 a
m
3 Gm 2 3GmM
UT

30
a R F1 30 F1
3Gm m F2
UT
R 3 M
{ a = 3R } R
m
(ii) Net force towards centre of planet is m
mv 02
F 2F1 cos 30 F2
R
Gm 2
3 GmM mv02
or 2 2
a 2 R2 R
a
But R
3
Gm GM v02 G m
or 3 v0 M
3R 2 R2 R R 3
Example 26 : A satellite is moving in a circular orbit around the earth. The total energy of the satellite is E =
2105J. The amount of energy to be imparted to the satellite to transfer it to a circular orbit where its potential
energy is U = 2 105J is equal to ________.
GmM
Sol. Here U 2 10 5
(R h)
GmM
2 105 ...(1)
Rh
1
Also K.E. = Tf = mv 20
2
2
1 GM GM
Tf m v0
2 Rh R h
1 GmM
Tf
2 (R h)
1
Tf 2 105 (from eqn (1)
2
Tf = 105 Joule
Ef = Uf + Tf
= 2 105 + 105 = 105 Joule
Required energy = Ef Ei
= 105 (2105) Required energy 1 105 Joule

www.physicsashok.in 40
GRAVITATION

Example 27 : A satellite of mass m is orbiting the earth in a circular orbit of radius r. It starts losing energy due to
small air resistance at the rate of C J/s. Then the time taken for the satellite to reach the earth is _____.
Sol. As due to air drag some mechanical energy of sattelite with be converted into heat energy.
Dur to this, radius of orbit will decrease
Hence, the satellite will follow spiral path towards the earth.
The total mechanical energy at orbital radius x is
GmM 1
E mv02
x 2
GmM 1 GM
E m
x 2 x
GmM
E
2x
(The path of satellite in
dE GmM d 1
or
dt

2

dt
x the presence of air drag)

t R
GmM 2 dx
GmM dx
or C 1 x C dt
2 x2
2 dt 0 r
R R
Gm M x 2 1 GmM 1
or Ct Ct
2 2 1 r 2 x r
GmM 1 1 GmM 1 1
or Ct t

2 R r 2C R r
Example 28 : Two small dense stars rotate about their common centre of mass as a binary system with the period
1
1 year for each. One star is of double the mass of the other and the mass of the lighter one is of the mass of
3
the sun. Find the distance between the stars if distance between the earth & the sun is R.
Gm 2m 2r
Sol. 2
m 2
r 3
2 3Gm
3 ...(1)
r
GM s m C 2m
Orbital velocity of earth is 2r r
R 3 3
GM s
v0
R
GM s
or R ...(2)
R
Time period of earth about Sun is one year.
=
From eqn (1) and (2)
3Gm GMs 3Gm
2 3 or 3
r R3 r
Ms
But m=
3
G3m 3Gm
3 r R
R3 r

www.physicsashok.in 41
GRAVITATION

C28 : A body moving radially away from a planet of mass M, when at distance r from planet, explodes in such a
way that two of its many fragments move in mutually perpendicular circular orbits around the planet. What will
be
(a) then velocity in circular orbits
(b) maximum distance between the two fragments before collision and
(c) magnitude of their relative velocity just befroe they collide.
Sol. (a) Since, total momentum of system in perpendicular direction of radius is zero.
So, after breaking, the momentum in perpendicular direction should be zero.
This is possible only when these two fragments move with same speed in opposite direction.
GmM mv 20
ma
x
r r r
GM
v0 = r
r
(b) max = r 2 + r 2 = 2 r
(c) Just before collision, their velocity are perpendicular to each other.
v rel = v 20 + v02
Gm
v rel 2 v 0 2
r
2Gm
v rel
r

ESCAPE VELOCITY
The minimum speed needed for a body to get rid of gravitational attraction of a planet V=0
F=0
is known as escape speed of the body on that planet. It is also called first cosmic
velocity. ve
R
GMm 1
mv 2e 0 0
r 2
2 GM
ve
R
ve 11.2 km s1 for earth
ve = 2 orbital speed.

Black Holes
Black holes are astronomical bodies having very high gravitational field such that even light cant escape from
these bodies. Hence they are called black holes.
Escape velocity on the surface of a black hole is larger than speed of light.
Calculation of radius (Event horizon) : We use formula for escape speed
2GM
v
R

www.physicsashok.in 42
GRAVITATION

Assuming the mass M distributed in the radius of R,


4
2G R 3
3
v
R
c
R
4
G
3
Typical data are
= 1017 kgm3
G = 1010 Nm2 kg2
We get
108
R~ 7 ~ 105 m = 10 km.
10
Any signal of an event will come out only if it occurs outside the sphere of radius R.
c
No news of events in the entrior to sphere of radius R = 8 G / 3 can be had. Hence R is called the radius
of event horizon.

Table : Some Speeds


GM 4 G
Orbital speed gr r
r 3
2GM 8 G
Escape speed 2gr r
r 3
Surface of Black hole ve c
Interstellar speed v e n 2 1 , n v / ve

Trajectory of a body pojected from point A in the direction AB with different initial velocities : Let
a body be projected from point A with velocity v in the direction AB. For different values of v the paths are
different. Here are the possible cases.
(i) If v = 0, path is a straight line from A to O. A B
v
(ii) If 0 < v < v0, path is an ellipse with centre O of the earth as a focus.
(iii) If v = v0, path is a circule with O as the centre
O
(iv) If v0 < v < ve, path is again an ellipse with O as a focus. R
Earth
(v) If v = ve, body escapes from the gravitational pull of the earth and path is a parabola
A 0 < v < v0 A v0 < v < ve
O

www.physicsashok.in 43
GRAVITATION

GM
(vi) If v > ve, body again escapes but now the path is a hyperbola. Here, v0 = orbital r at A and

ve = escape velocity at A.
A v0
Note :
1. From case (i) to (iv) total energy of the body is negative. Hence, these are the h
closed orbits. for case (v) total energy is zero and for case (vi) total energy is R
positive. In these two cases orbits are open.
2. If v is not very large the elliptical orbit will intersect the earth and the body will fall
back to earth.
C29 : Calculate the escape velocity from the moon. The mass of the moon = 7.4 1022 kg and radius of the moon
= 1740 km.
Sol. The escape velocity is

2GM
v
R

2 6.67 10 11 7.4 1022


v
1740 103
v = 2.4 km/s
C30 : Calculate the escape velocity on the surface of a planet whose mass is 1025 kg. and radius is 6.67 109
metre.

2GM 2 6.67 1011 1025


Sol. Ve
R 6.67 109
Ve = 2 105 = 447.2 m/sec.
C31 : A body is thrown with a speed of 3ve where ve is escape speed from the surface of the Earth. Find its speed
in interstellar space. Assume no other gravitational field than the earth, and neglect air.
Sol. Conserving mechanical energy
GMm 1 2 1
m 3ve 0 mv 2
R 2 2
2 GM
Now, v2e
R
v 2e 1 2 1
3v e v 2
2 2 2
2 2 2
v = (3ve) ve

v 2 2 ve
Example 29 : A particle is fired vertically from the surface of the earth with a velocity ku e , where u e is the
escape velocity and k < 1. Neglecting air resistance and assuming earths radius as Re. Calculate the height to
which it will rise from the surface of the earth.

www.physicsashok.in 44
GRAVITATION

Sol. Applying conservation principle of mechanical energy,


Ui + Ti = Uf + Tf
here Tf = 0
GmM 1 GmM
mk 2 ve2
Re 2 Re h
GmM 1 GmM
or mk 2 2gR
Re 2 Re h
GmM 1 GM GmM GM
or mk 2 2 2 R e v e 2gR e and g 2
Re 2 Re Re h Re
By solving,
Re Re
Re h or h Re
1 k2 1 k2
R e R e R ek 2 Rek 2
h h

1 k2
1 k2
Example 30 : Two stars each of radius R are at rest at a distance where the force between them is negligible. The
mass of one star is M and that of the other is 2M. Find their speeds when their centres are a distance d > 2R
apart.
Sol. Let the stars of mass M be moving with velocity V1 and 2M with V2. Then
MV1 + 2 MV2 = (M + 2M)Vcm
Since their initial velocities are zero, Vcm = 0
V1 + 2V2 = 0
V1 = 2 V2
Negative sign indicates that they are moving in opposite directions. M M

When separated by distance d, the potential energy is given by V V/2


GM (2M) d
U
d2
The kinetic energy is
1 1 V1
KMV 2 (2M) (V / 2) 2 V2
2 2 2
3
K MV 2
4
The sum of two energies, K and U, must not change. Hence
Ki + Ui = Kf + Uf
3 2GM 2
00 MV 2
4 d
8 GM
V
3 d

8GM 2 GM
The speed of star of mass M is and that of mass 2 M is .
3d 3d

www.physicsashok.in 45
GRAVITATION

Example 31 : A spaceship of mass 1000 kg. is situated on the surface of a planet whose distance from the sun is
2.28 108 km.
(a) How much energy is the spaceship having ?
(b) How much energy must be expended on it to launch it to go to infinity ?
The mass of the sun is 2 1030 kg, planet is 6.4 1023 kg, radius of planet is 3395 km and G = 6.67 1011
Nm2 kg2. Neglect motion of the planet.
Sol. The spaceship at rest has only potential energy. It is given by
m
M M r
U Gm S R
R r S r
M
But r >> R hence we take r r. Thus

6.4 1023 2 1030


U = 6.67 10 11 3
10 3395 103 2.28 108 103

6.4 1017
8 2
U 6.67 10 1019 J
3.395 2.28

6.4 200
U 6.67 109 J
3.395 2.28

U = 6.67 89.5 109 J


U = 597.62 109
U = 6.0 1011 J
When the spaceship will go to infinity, its total energy will be zero.
Ef = 0

M M
Ei Gm S
R r
Energy supplied to spaceship = Ef Ei

M M
GM S
R r
= 6.0 1011 J [as calculated in part (a)]

www.physicsashok.in 46
GRAVITATION

THINKING PROBLEMS
1. If the force of gravity acts on all bodies in proportion to their masses, why does a heavy body not fall faster than
a rigid light body?
2. Does the concentration of the earths mass near its centre change the variation of g (acceleration due to gravity)
with height from its surface?
3. Why is it that we can learn more about the shape of the earth by studying the motion of an artificial satellite than
by studying the motion of the moon?
4. One clock is based on an oscillating spring, the other on the pendulum. Both are taken to Mars. Will they keep
the same time there as they keep on Earth? Will they agree with each other ? Explain.
5. The gravitational force exerted by the Sun on the Moon is almost twice as great as the gravitational force exerted
by the Earth on the Moon. Why then doesnt the Moon escape from the Earth?
6. Would you expect the total energy of the solar system to be constant? The total angular momentum? Explain.
7. Objects at rest on the earths surface move in circular paths with a period of 24 hours. Are they in orbit in the
sense that an earth satellite is in orbit? Explain.
8. The suns tide-raising power is only half as great as that of the moon. The direct pull of the sun on the earth,
however, is about 175 times that of the moon. Why is it then that the moon causes larger tides ?
9. Does a rocket really need the escape valocity from the very beginning to escape from the earth?
10.Can a satellite move in a stable orbit in a plane not passing through the earths centre? Explain.
11. The total energy of the earth sun system is negative. How do you interpret the negative energy of a system?
12.As measured by a n observer on earth, would there be any difference in the periods of two satellites, each in a
circular orbit near the earth in an equatorial plane, but one moving eastward and the other westward?
13.An artificial satellite in the presence of frictional forces will move into an orbit closer to the earth and may have
increased kinetic energy. Explain this.
14.If a planet of given density were made larger, its force of attraction for an object on its surface would increase
because of the planets greater mass but would decrease because of the greater distance from the object to the
centre of the planet. Which effect predominates ?
15.A stone is dropped along the centre of a deep vertical mine shaft. Assume no air resistance but consider the
earths rotation. Will the stone continue along the centre of the shaft? If not, decribe the motion.
16.Two air bubbles with radius r are present in water. Are these bubbles attracted or repelled?
17.Two setellites move along a circular orbit in the same direction at a small distance from each other. A container
has to be thrown from the first satellite on to the second one. When will the container reach the second satellite
earlier. If it is thrown in the direction of motion of the first satellite or in the opposite direction? The velocity of the
container is small in comparison to that of the satellite.
18.When a train moves from west to east at high speed, does its weight increase or decrease?
19.Why is there weightlessness in a satellite?
20.Is the velocity of escape of a particle at rest on the surface of the earth the same as that of a particle just orbiting
the earth? Explain.
21.Two metal blocks ar shaped such that A can fit into B. When A is placed on B, the spring S
is compressed. The system is now allowed to fall freely under gravity. It is found that A and B
separate during the fall. explain.

www.physicsashok.in 47
GRAVITATION

SOLUTION OF THINKING PROBLEMS

1. Though heavier bodies are acted upon by greater forces, the acceleration produced in them is the same as that
produced in light bodies. Let F be the force on any body of mass m and g be the acceleration. Then
Mm GM
F = G 2 = mg or g= 2
R R
an expression independent of mass.
Hence, all bodies fall with the same repidity, provided there is no air resistance.
2. No. The acceleration due to gravity, that is, gravitational intensity at points outside the surface may be calculated
by assuming the mass to be concentrated at the centre of the sphere. Hence, the distribution of mass over the
earth does not affect the variation of g with height.
3. We can learn more about the shape of the earth by studying the motion of an artificial satellite than by studying
the motion of the moon because the motion of the satellite is governed by the value of g at each of its positions.
Since the moon is very far away, the variation of g near the surface of the earth is averaged out at the position of
the moon.
4. The spring-operated clock will keep the same time but the pendulum clock will not keep the correct time as g on
Mars is different from g on earth. They will not agree.
5. Because of the rotational equalibrium of the moon around the earth and also that of the earth + moon system
around the sun. The attractive force on the moon due to the earth provides the necessary centripetal force for the
rotational motion of the moon around the earth and the attractive force on the moon due to the sun provides
necessary centripetal force for the rotational motion of the moon around the sun.
6. Yes. The gravitational forces are conservative. When a system is under the action of conservative forces, its total
energy (kinetic + potential) is a constant and hence, the energy of the solar system is a constant. The gravitational
forces on planets of the solar system are central forces directed towards the sun and hence, the moment of forces
about the sun is zero. Thus, no torque acts on the planets and the angular momentum is a constant.
7. No, they are not in orbit in the sense that an artificial satellite is, because the whole of the gravitational pull on
the artificial satellites provides the necessary centripetal force, but for objects at rest on the earth, the whole of
the gravitational pull (i.e., their weights) does not provide the necessary centripetal force; only a small friction
provides the necessary centripetal force. In short, bodies in orbit are weightless but bodies on the earth at rest
are not weightless.
8. The tide-raising power of a body (sun or moon) is determined by the difference in the gravitational attraction
between the water on the far and near sides, not by the total pull. The tide producting force varies inversely as the
cube of the distance. Thus, the relatively short distance separating the earth and the moon makes the moons
tide-raising force more than double that of the distance sun.
9. No. What is actually needed is the supply of total amount of energy required to take it beyond the gravitational
field of the earth.
10.No. For stability of rotational motion there should not be any torque produced by the force acting on it about the
centre of the orbit. When the orbit does not lie in a plane passing through the centre, the gravitational pull will
have a moment (torque) about centre of the orbit.
11. The meaning of the total energy being negative is that the system (sun + earth) is a closed one, earth always being
bound to the attracting solar centre and never escaping from it.
12.The westward satellite will appear to have a shorter period than the eastward satellite, owing to the rotation of
the earth itself from west to east.

www.physicsashok.in 48
GRAVITATION

1 GMm
13.E (total energy) mv 2 where m is the mass of the satellite, M = mass of the earth and r = radius
2 r
of the orbit. By the dynamics of circular motion.
GMm mv 2 GM
2
or v2 =
r r r
1 GM 1 2
or E or mv or K (kinetic energy)
2 r 2
1 GM 1 GM
Ei (initial energy) 2 r and Ef (final energy)
0 2 r
By the principle of conservation of energy
Ei = Ef + W (work done by frictional force)
1 GM 1 GM
W
2 r0 2 r
1 1 1
or GM W (a positive quantity)
2 r r0
1 1
> or r < r0
r r0
Also K0 = K + W
or K K0 = W (a positive quantity)
K > K0
1
14.Mass-effect dominates over the distance-effect [mass R3. fprce pf attraction 2 ].
R
15.Due to the rotation of the earth from west to east the stone will deviate slightly to the east.
16.The two bubbles attract each other. A body immersed in an extended denser medium behaves like a negative
mass so far as gravitational attraction is concerned. The two bubbles, therefore, behave like two negative masses,
each of absolute value equal to the mass of an equal volume of water. So by the law gravitation
mm
F G 1 2 2 r
r
m1 m 2 r G m1m 2 r
Here F G
r2 r2
The negative force of interaction means force of attraction.
17.If the container is thrown in the direction opposite to the motion of the satellite, it will
begin to move along an ellipse (say,1) lying inside the circular orbit of the satellites. The
period of revolution will be slightly less than that of the satellites. Therefore, they can
meet at the point of contract of the orbits (circular orbit of satellite and the elliptical
orbit of the container) only after a greater number of revolutions. On the other hand, if
the container is thrown in the direction of the satellite, the container begins to move
along a ellipse like 2 in the figure. The velocity u should be such that during one revolution of the container,
satellite B also makes one revolution and in addition covers the distance AB. So the container should be thrown
in the direction of the first satellite.
18.The weight decreases. See Example 2.
19.The feeling of weight arises from the reaction of the ground on a man. In an orbiting satellite, the astronaut and
the floor of the satellite both have the same acceleration (the centripetal acceleration towards the earth). Hence,
the floor of the satellite offers no reaction to the astronaut and he feels weightless.

www.physicsashok.in 49
GRAVITATION

For circular motion of man


GM m mv 2
N
r2 r
For a particle in circular motion
GMm ' m ' v 2 GM v 2

r2 r r2 r
2 2
mv mv
N N0
r r
20.No, the velocity of escape arises from the energy required to take it from the present condition to infinity. This is
equal to the difference between energy on the surface of the earth and infinity. For a particle at rest
1 GM
mv 2e = m (only potential energy)
2 R
2GM
ve
R
For a particile in circular motion
GM 1
E (energy) mv 2c
R 2
2
mvc GMm
But =
R R2
(from dynamics of circular motion)
1 GMm
E
2 R
1 1 GMm
mve2
2 2 R
GM
or ve =
R
21.Bodies falling freely under gravity are in a state of weightlessness. In this state, the compressed spring will
separate A and B.

www.physicsashok.in 50
GRAVITATION

ASSERTION-REASON TYPE
A statement of Statement-1 is given and a Corresponding statement of Statement-2 is given just below it of
the statements, mark the correct answer as
(A) If both Statement-1 and Statement-2 are true and Statement-2 is the correct explanation of Statement-1.
(B) If both Statement-1 and Statement-2 are true and Statement-2 is NOT correct explanation of Statement-1.
(C) If Statement-1 is true but Statement-2 is false.
(D) If both Statement-1 and Statement-2 are false.
(E) If Statement-1 is false but Statement-2 is true.

1. Statement-1 : Keplers laws for planetary motion are consequence of Newtons laws.
Statement-2 : Keplers laws can be derived by using Newtons laws.

2. Statement-1 : Three orbits are marked as 1, 2 and 3. These three orbits have same
2 3
semi-major axis although their shapes (eccentricities) are different. The three identical 1
satellites are orbiting in these three orbits, respectively. These three satellites have the
same binding energy.
Statement-2 : Total energy of a satellite depends on the semi-major axis of orbit according to the expression,
GMm
E= .
2a

3. Statement-1 : A particle of mass m is projected towards the heavy planet as v m


shown in figure. The angular momentum of the particle is conserved about M d
point O.
O
Statement-2 : Torque of gravitational force about point O is always zero.

MATCH THE COLUMN

1. An artificial satellite is in circular orbit around the earth. one of the rockets of the satellite is momentarily fired,
the direction of firing of rocket is mentioned in Column I and corresponding change(s) are given in Column-II.
Match the entries of ColumnI with the entries of Column II.
Column I Column II
(A) Towards the earths centre (P) Orbit changes and becomes elliptical
(B) Away from the earths centre (Q) Orbit plane changes
(C) At right angle to the plane of orbit (R) Semi-major axis of orbit increases
(D) In forward direction (S) Energy of earth-satellite system increases.

2. In Column-I, 4 artificial satellites of earth are shown, while in Column-II, some statements are given related to
motion or other facts of satellites. Match the entries of Column-I with the entries of Column-II.
Column I Column II
(A) Earth (P) May be a geostationary satellite
A

Earth
(B) (Q) Not a geostationary satellite
B

www.physicsashok.in 51
GRAVITATION

Earth
(C) (R) Not possible

Earth D
(D) (S) If a rocket of a satellite fired in the direction
to perpendicular to plane or orbit, then the
orbit of the satellite changes.

3. Four identical satellites are orbiting in four elliptical orbits having same semi- 3
major axis but different eccentricities. In Column-I some quantities associated 4
with four orbits are given and in Column-II the words which can give the 1 E 2
r1 r2
information about physical quantities mentioned in Column-I. Match the entries
of Column-I with the entries of Column-II.
Column I Column II r1 r2
(A) Total energy of all four orbits (P) Same
(B) Speed of satellite in all four orbits (Q) Different
(C) Velocity of satellite in all four orbits (R) Constant
(D) Angular momentum of satellites about (S) Varying
centre of earth in all four orbits.

4. A particle is taken to a distance r (> R) from centre of the earth. R is radius of the earth. It is given velocity V
which is perpendicular to r. With the given values of V in column-I you have to match the values of total energy
of particle in column-II and the resultant part of particle in column-III. Here G is the universal gravitational
constant and M is the mass of the earth.
Column-I (Velocity) Column-II (total energy) Column-III (Path)
(A) V Gm / r P. Negative T. Elliptical
(B) V 2Gm / r Q. Positive U. Parabolic
(C) V 2Gm / r R. Zero V. Hyperbolic
(D) Gm / r V 2Gm / r S. Infinite W. Circular

www.physicsashok.in 52
GRAVITATION

LEVEL 1

1. Keplers law of are for motion of planets around the sun


(A) the law of energy conservation (B) the law of momentum conservation
(C) the law of charge conservation (D) the law of angular momentum conservation about the sun

2. If the semi-major axis of orbit of a planet be doubled, the revolution period will become
(A) 8 times (B) 2 2 times (C) 3 times (D) none

3. The major axis of an ellipse is l1 and minor axis l2. A planet moving along it has a period T. The area swept by
the line joining the planet to the sun at one of the focii in a time t will be
l1l2 l1 l2 l12 l2 2
(A) t (B) t (C) t (D) t
T T T T

4. If the angular momentum of a planet of mass m around the sun is L, its areal velocity will be
L L L L2
(A) (B) (C) 2 (D)
2m m m 2m

5. Within a uniform spherical shell


(A) gravitational force is zero, but potential is GM/R at every point.
(B) any passenger will feel weightlessness relative to the spheres gravity.
(C) the space is equipotential (D) all of these

6. A particle is located in a uniform spherical shell at its centre. It experiences a force F. Now a small hole is made
by removing a mass m from the shell. Now the force on the particle is F. Then
(A) F = 0, F 0 (B) F 0, F = 0 (C) F = 0 = F (D) F 0 F

7. Consider a uniform sphere with centre at C but heving a spherical cavity passing
through C and surface of the sphere. There is a point A in the cavity. The direction of A
gravitational field intensity at A is along C C

(A) AC (B) CA (C) CC (D) CC

8. A homogenous body has large extension such that at any point within the body the B
gravitational field intensity is zero. Now a cavity is made centred at some point A. Its
A
radius is R. The gravitational field intensity at some point B outside the cavity is directed
along

(A) AB (B) BA (C) Zero (D) None

9. Consider a homogenous sphere of density . Pressure at some point A is P. Now


a cavity of radius r < OA is made in it. The pressure at A is P. Then r
R
(A) P = P (B) P > P O A
(C) P < P (D) nothing can be said

www.physicsashok.in 53
GRAVITATION

10. The satellite is moving in an elliptical orbit about the earth as shown in figure : P
The minimum and maximum distance of satellite from earth are 3 units and 5 units,
respectively. The distance of satellite from the earth when it is at P is equal to S Q
E
(A) 4 units (B) 3 units
(C) 3.75 units (D) None of these
m
11. A wreckage of mass m present in space, collides with a satellite of mass 10 m and orbital
radius R as shown. v

As a result of collision, the wreckage sticks to satellite and satellite is transferred to an R


orbit whose minimum distance from the planet is R/2. Take mass of earth as M. The R/2
velocity of the wreckage with which it collides to the satellite is

GM 58 GM 63 GM 59 GM
(A) (B) (C) (D)
R R R R

12. A tunnel is dug along diameter inside the surface of earth and a particle is projected from the centre of tunnel.
The minimum velocity of particle such that it escape out from the earths gravitation field is (Radius of
earth = Re)
3 5
(A) 2g R e (B) g Re (C) 3g R e (D) g Re
2 2

13. A frictionless tunnel is dug along a chord of the earth at a perpendicular distance R/2 from the centre of earth
(where R is radius of earth). An object is released from one end of the tunnel. The correct graph, showing the
variation of acceleration of particle with its distance r from centre of earth is
a a a a

(A) (B) (C) (D)


r r r r
R/2 R R/2 R R/2 R R/2 R

14. A ring of mass m1 and radius R is fixed in space at some location. An external
agent brings a point mass m2 from infinite to centre of the ring. Work done by the R m1
external agent will be m2

Gm1m 2 Gm1m 2 G m12 m 22 Gm1m 2


(A) (B) (C) (D) R m m
R R R 1 2

15. The work done to take a particle of mass m from surface of the earth to a height equal to 2R is (R is radius of
earth)
(A) 2 mgR (B) mgR/2 (C) 3 mgR (D) 2mgR/3

16. The radius of a planet is R1 and a satellite revolves around it in a circle of radius R2. The time period of
revolution of satellite is T. Acceleration due to the gravitation of the platnet at its surface will be
4 2 R 32 R 32 4 2 R 13 R13
(A) 2 2 (B) (C) 2 2 (D)
T R1 4 2 T 2 R12 T R2 4 2 T 2 R 22

www.physicsashok.in 54
GRAVITATION

17. A planet of small mass m moves around sun of mass M along an elliptical orbit such that its minimum and
maximum distance from sun are r and R respectively. Its period of revolution will be

(r R)3 (r R)3 (r R ) 3 (r R)3


(A) 2 (B) 2 (C) (D) 2
6GM 3GM 2GM GM

18. A tunnel is dug along a chord of the earth at a perpendicular distance R/2 from the earths centre. The wall of
the tunnel may be assumed to be frictionless. A particle is released from one end of the tunnel. The pressing
force by the particle on the wall, and the acceleration of the particle varies with x (distance of the particle from
the centre) according to :
Pressing Force Pressing Force acceleration acceleration

(A) (B) (C) (D)


x x x x
x=R/2 x=R x=R/2 x=R x=R/2 x=R x=R/2 x=R

19. Consider two satellites A and B of equal mass m, moving in same circular orbit about
earth, but in opposite sense as shown in figure. The orbital radius is r. The satellites r B
A
undergoe a collision which is perfectly inelastic. For this situation, mark out the correct
statement(s). [Take mass of earth as M]. Earth

(A) The total energy of the two satellites plus earth system just before collision is GMm/r.
(B) The total energy of the two satellites plus earth system just after collision is 2GMm/r
(C) The total energy of two satellites plus earth system just after collision is GMm/2r.
(D) The combined mass (two satellites) will fall towards the earth just after collision.
20. A solid sphere of uniform density and radius 4 unit is located with its centre at the y
origin O of co-ordinates. Two spheres of equal radii 1 unit, with their centres at
A(2, 0, 0) and B(2, 0, 0) respectively, are taken out of the solid leaving behind x
m
spherical cavities as shown in figure. Then A B
O
(A) the gravitational field due to this object at the origin is zero.
(B) the gravitational field at the point B(2, 0, 0) is zero z
(C) the gravitational potential is same at all points on the circle y2 + z2 = 36
(D) the gravitational potential is same at all points on the circle y2 + z2 = 4

21. Three planets of same density have radii R1, R2 and R3 such that R1 = 2R2 = 3 R3. The gravitational field at their
respective surfacs are g1, g2 and g3 and escape velocities from their surfaces are v1, v2 and v3. Then
(A) g1/g2 = 2 (B) g1/g3 = 3 (C) v1/v2 = 1/4 (D) v1/v2 = 3

22. Suppose the earth suddenly shrinks in size, still remaining spherical and mass unchanged (All gravitational forces
pass through the centre of the earth).
(A) The days will become shorter
(B) The kinetic energy of rotation about its own axis will increase
(C) The duration of the year will increase.
(D) The magnitude of angular momentum about its axis will increase.

23. Two artificial satellites of the same mass are moving around the earth in circular orbits of different radii. In
comparison to the satellite with lesser orbital radius, the other satellite with higher orbital radius will have :
(A) greater kinetic energy (B) greater potential energy (C) greater total energy
(D) greater magnitude of angular momentum, about the centre of the circular orbit

www.physicsashok.in 55
GRAVITATION

24. Two objects of mass m and 4m are at rest at an infinite separation. they move towards each other under mutual
attraction. If G is the universal gravitational constant. Then at separation r.
(A) The sum of energy of the two objects is negative
1/ 2
10Gm
(B) Their relative velocity of approach is in magnitude
r
(C) The sum of kinetic energy of the objects is 4Gm2/r.
(D) The sum of angular momenta of both the objects is zero about any point.

25. The earth is moving around the sun in an elliptical orbit. Point A is the closest and
point B is the farthest point in the orbit, as shown. In comparison to the situation Earth
A B
when the earth passes through point B : Sun
(A) total energy of the earth-sun system is greater when the earth passes through point A.
(B) gravitational potential energy of the earth-sun system is greater when the earth passes through point A.
(C) kinetic energy of the earth due to the motion around the sun is greater when it passes through the point A.
(D) magnitude of angular momentum of the earth about the sun is greater when the earth passes through point A.

26. A person abandoned on a small spherical asteroid of mass m1 and radius R, sees a satellite orbiting the
asteroid in a circular orbit of period T. The satellite appears stationary to a person on the asteroid. If the
asteroid rotates with a period Ta, the radius of satellite is
1/ 3 1/ 3 1/ 3 3
Gm1 Ta2 Gm1 Ta2 Gm1 T 2 Gm1 T 2
(A) 2 (B) 2 (C) 2 (D) 2
2 4 4 4

27. If the distance between the earth and the sun were half its present value, the number of days in a year would
have been [JEE, 06]
(A) 64.5 (B) 129 (C) 182.5 (D) 730

28. An artificial satellite moving in a circular orbit around the earth has a total (K.E. + P.E.) E0. Its potential energy
is [JEE, 97]
(A) E0 (B) 1.5 E0 (C) 2 E0 (D) E0

29. In a region of only gravitational field of mass M a particle is shifted from A to B via (3)
B C
three different paths in the figure. The work done in different paths are W1, W2, W3
respectively then [JEE-(Scr), 03] (2)
(1)
(A) W1 = W2 = W3 (B) W1 > W2 > W3
A
(C) W1 = W2 > W3 (D) W1 < W2 < W3

30. A system of binary stars of masses mA and mB are moving in circular orbits of radii rA and rB respectively. If TA
and TB are the time periods of masses mA and mB respectively, then [JEE, 06]
(A) TA > TB (if rA > rB) (B) TA > TB (if mA > mB)
2 3
TA rA
(C) (D) TA = TB
TB rB

www.physicsashok.in 56
GRAVITATION

PASSAGE TYPE

Comprehension # 1
Changing from a circular to An elliptical orbit
Let us identify the system as the spacecraft and the Earth but not the portion of the fuel in the spacecraft that we
use to change the orbit. In a given orbit, the mechanical energy of the spacecraft-Earth system is given by
GMm
E
2r
This energy includes the kinetic energy of the spacecraft and the potential
energy accosicated with the gravitational force between the spacecraft and
Elliptical
the Earth. If the rocket engines are fired, the thrut force moves the spacecraft Circular orbit
through a displacement. As a result, the mechanical energy of the spacecraft - orbit
Earth system increass.
The spacecraft has a new higher energy but is constrained to be in an orbit that
includes the original strarting point. It can not be in a higher energy circular Earth
orbit having a larger radius because this orbit would not contain the starting
Rocket engine
point. The only possibility is that the orbit is elliptical as shown in the figure. is fired here
GMm
E
2a
Above equation gives the energy of the spacecraft - Earth system for an elliptical orbit where a is semimajor
axis. Thus if we know the new energy of the orbit, we can find the semi-major axis of the elliptical orbit.
Conversely, if we know the semi-major axis of an elliptical orbit we would like to achieve, we can calculate
how much additional energy is required from the rocket engines.
A spacecraft is moving in a circular orbit around the Earth (Radius 6400 Km), at a height of 300 km from the
surface. To place the spacecraft in an elliptical orbit, the magnitude of the mechanical energy of the spacecraft-
Earth system is decreased by 10.0%.
1. If the spacecraft-earth system had initial energy (E0), then the total mechanical energy of the system after firing
the rocket will be :
(A) 1.1 E0 (B) 1.9 E0 (C) E0 (D) None of these

2. Semimajor axis of the new elliptical orbit is


(A) 7437 Km (B) 6700 Km (C) 7370 Km (D) None of these

3. maximum height of the spacecraft above the surface of the Earth will be :
(A) 970 km (B) 1474 km (C) 300 km (D) 1774 km

Comprehension # 2
A triple star system consists of two stars, each of mass m in the same circular V
orbit about central star with mass M = 2 1030 Kg. The two outer stars always
lies at opposite ends of a diameter of their common circular orbit. The radius of
the circular orbit is r = 1011 m and the orbital period of each star is 1.6 107 S. m M m

20
[Take = 2 = 10 and G = 10111 Nm2 kg2]
3 V
4. The mass m of the outer stars is
(A) 16/15 1030 Kg (B) 11/8 1030 Kg (C) 15/16 1030 Kg (D) 8/11 1030 Kg

www.physicsashok.in 57
GRAVITATION

5. The orbital velocity of each star is


5 5 5 5
(A) 10 103 ms1 (B) 10 105 ms1 (C) 10 102 ms1 (D) 10 104 ms1
4 4 4 4

6. The total mechanical energy of the system is


1375 1375 1375 1375
(A) 1035 J (B) 1038 J (C) 1034 J (D) 1037 J
64 64 64 64

LEVEL 2
1. Ravi can throw a ball at a speed on earth which can cross a river of width 10 m. Ravi reaches on an imaginary
planet whose mass density is twice of the earth. Find out the maximum possible radius of Planet so that if Ravi
throws the ball at same speed it may escape from planet. Given radius of earth = 6.4 106 m.

2. Two equal masses m and m are hung from a balance whose scale pans differ in vertical height by h. calculate
the error in weighing, if any, in terms of density of earth .

3. A sky lab of mass 2 103 kg is first launched from the surface of earth in a circular orbit of radius 2R (from the
centre of earth) and then it is shifted from this circular orbit to another circular orbit of radius 3R. Calculate the
minimum energy required (a) to place the lab in the first orbit, (b) to shift the lab from first orbit to the second
orbit. Given, R = 6400 km and g = 10 m/s2.
4. If a planet was suddenly stopped in its orbit supposed to be circular, show that it would fall onto the sun in a
time
2 / 8 times the period of the planets revolution.

5. A cord of length 64 m is used to connect a 100 kg astronaut to a space-ship whose mass is much larger than
that of the astronaut. Estimate the value of the tension in the cord. Assume that the space-ship is orbiting near
earth surface. Also assume that the space-ship is orbiting near earth surface. Also assume that the space-ship
and the astronaut fall on a straight line from the earths centre. The radius of the earth is 6400 km.

6. The weight of a body is 81N at the surface of the earth. How much is the weight at a height equal to helf the
radius of the earth ?

7. A body weights w at a point half way from surface to centre of the earth. How much will it weight on the
surface? Assume the Earth a homogenous sphere.

8. A rocket is launched vertically upward with a speed of 5-6 km s1. How high will it go without using any fuel?
Neglect air resistance. G = 6.67 1011 Nm2 kg2, mass of the Earth = 6.0 1024 kg and mean radius of the
Earth = 6.4 106 m.

9. Calculate escape speed of molecules on the surface of moon where gm = 1/6 of gE. The density of moon is the
same as that of the earth. The escape speed on the Earth is 11.2 km/s.

10. Consider a satellite of mass m is orbiting in a circle of radius r around a plant of mass M.
Calculate its (a) kinetic energy (b) potential energy and (c) mechanical energy; take reference m
M
level of potential energy as zero at infinite radius.

www.physicsashok.in 58
GRAVITATION

11. A light rod of length l connects two balls each of mass m. The system is free to rotate
about pivot O. Two small spheres of masses M and 4M are placed respectively at O
normal distance x and y from the balls. the balls stay in equilibrium. For x = l/2, x y

determine y. How much is the force exerted on the rod by pivot at O ?

12. A binary star is a system of two stars separated by a distance l and moving around l
their centre of mass at rest (See figure) Determine the mechanical energy of the binary m C M
star shown. How much is its binding energy ?

13. A smooth tunnel passes through a uniform solid sphere of radius R at a distance R/3 from the centre of the
sphere. Neglect effect of tunnelling on the gravitational field. Determine the force exerted on a mail of mass m
in the tunnel by tunnel wall. the mass of the sphere is M. Is this force gravitational or electromagnetic ?

44. A sensitive meter can measure changes in standard value of g in units of gals (1
gal = 1 cm s2). the value of g at the surface of sphere of density and radius R. h
A cavity is now made directly below the observation point. The change in the r
value of g is g. (See figure). Find an expresion for g. R

15. A star is found to be moving in a circle with speed V. The radius of the circle is R. Determine the mass of a
possible block hole at the centre of the circle.

16. What is the force exerted on a geostationary satellite of mass 200 kg ? Its distance from the centre is
4.24 107 m and radius of the earth is 6.4 106 m.

17. Three spherical shells, each of mass M and radius R, are in contact due to
gravitational pull. Find the magnitude of the contact force between any two spheres.

18. The gravitational potential at a point (x, y) is (4x + 6y) J kg1. Determine the gravitational field intensity vector
at this point.

19. A person is inside a statellite cabin moving round the earth. In the frame fixed to the cabin, draw forces acting
on the person and show that if he stands on the platform of a spring balance in the satellite, the reading of the
balance will be zero (w eightlessness).
20. An artificial satellite of mass m of a planet of mass M, revolves in a circular orbit whose radius is n times the
radius R of the planet. In the process of motion, the satellite experiences a slight resistance due to cosmic dust.
Assuming resistance force on satellite depends on velocity as F = av2 where a is constant. Calculate how long
the satellite will stay in orbit before it falls onto the planets surface.

21. A satellite of mass m is orbiting the earth in a circular orbit of radius r. It starts losing energy due to small air
resistance at the rate of C J/s. Find the time taken for the satellite to reach the earth.

22. A spaceship is sent to investigate a planet of mass M and radius R. While hanging
motionless in space at a distance 5R from the centre of the planet, the spaceship v0
m R
fires an instrument package with speed v0 as shown in the figure. The M
package has mass m, which is much smaller than the mass of the spaceship. For 5R
what angle will the package just graze the surface of the planet ?

www.physicsashok.in 59
GRAVITATION

LEVEL 3

1. Distance between the centres of two stars is 10 a. The masses of these stars are M and 16M and their radii a
and 2a respectively. A body of mass m is fired at night from the surface of the larger star towards the smaller
star. What should be its minimum initial speed to reach the surface of the smaller star ? Obtain the expression
in terms of G, M and a. [JEE, 96]

2. A cord of length 64 m is used to connect a 100 kg astronaut to spaceship whose mass is much larger than that
of the astronaut. Estimate the value of the tension in the cord. Assume that the spaceship is orbiting near earth
surface. Assume that the spaceship and the astronaut fall on a straight line from the earth centre. The radius of
the earth is 6400 km. [REE, 98]

3. A body is projected vertically upwards from the bottom of a crater of moon of depth R/100 where R is the
radius of moon with a velocity equal to the escape velocity of the surface of moon. Calculate maximum height
attained by the body from the surface of the moon. [JEE, 2003]

www.physicsashok.in 60
GRAVITATION

Answer Key

ASSERTION-REASON TYPE

Q. 1 2 3
Ans. D A A

MATCH THE COLUMN

1. [(APRS), (BPRS), (CPQRS), (DP)]

2. [(APS), (BPS), (CQS), (DRS)]

3. [(APR), (BQS), (CQS), (DQR)]

4. [(APW), (BRW), (CQV), (DPT)]

LEVEL 1

Q. 1 2 3 4 5 6 7 8 9 10
Ans. D B A A D A D A B A
Q. 11 12 13 14 15 16 17 18 19 20
Ans. B C D A D A C BC ABD ACD
Q. 21 22 23 24 25 26 27 28 29 30
Ans. ABD AB BCD BCD C B B C A D

PASSAGE

Q. 1 2 3 4 5 6
Ans. B A D B D B

LEVEL 2

2 GM e
1. R = 4 km 2. 8/3 Gmh 3. (a) 9.6 1010 J; (b) 1.1 1010 J 5. T m r 2
r
6. 36 N 7. 2 W 8. h = 2.13 106 m 9. 1.86 km/sec.
1 GMm GmM 1 GMm 6GMm
10. (a) ; (b) U ; (c) 11. l1
2 r r 2 r l2

www.physicsashok.in 61
GRAVITATION

GMm M 2m GMm(M 2m) GMm


12. ; 2l (M m) 13. ; electromagnetic.
2l Mm 3R 2
4Ger 3 Rv2 GM 2
14. 15. 16. 44.66 N 17.
3h G 4R 2

m R ( n 1)

18. 4 i 6 j N / kg 19. N 0 20. t
a Gm

GMm 1 1
1 1 8GM
21. T 22. sin 5 1 5v 2 R
2C R r 0

LEVEL 3

3 5GM
1. vi min 2. T = 3 102 N 3. h = 99 R
2 a

www.physicsashok.in 62
CENTRE OF MASS

DHANALAKSHMI NAGAR

NEAR ANNAMAIAH CIRCLE,

TIRUPATI.

PH NO. 9440025125
CENTRE OF MASS

CENTRE OF MASS
If we consider a system of many particles, each particle will have a complicated motion and it will be difficult to
follow it. However, there is one point whose motion represents the translation motion of the whole system in a
simple way. That point is located with respect to the system at a position decided by positions and masses of
all the particles in the system. It is defined in such a way that it becomes possible to apply formulae of kinematics
and dynamics in a form similar to those for single particle (See table).

Item Single Particle Many - Particle System



Velocity cm

Acceleration a a cm

Momentum m m cm
Newtons second

Law of motion F = ma Fext = M a cm

Impulse J = P J ext = P

Momentum Conservation F = 0 , P = constant Fext = 0, P = constant

p2 P2
N
pi2
Kinetic energy K=
2m
K
2M
;K=
i 1 2m i

NOTE : Concept of centre of mass loses its meaning at relativistically high speeds.

Centre of mass of two particles H


If we consider hydrochloric acid molecule is a system, it contains two Cl System
atoms H and Cl. If we regard H as a single particle and Cl also as a single
r1
particle, this molecule is a system of two particles. Suppose we know the r2
position vectors or hydrogen atom and chlorine atom, regarding them as
as points, and their masses also. Using these, we want to define centre
of mass of two particles. Since we want to write equations of motion of
system of many particles in the form similar to those for a single particle this is possible only if position of centre
of mass be defined in a particular way. And that way is

m r m Cl r2
rcm = H 1
mH mCl
We may compare this result with mean value of a quantity x given by


f1x1 f 2 x 2
x = f1 f 2
Here f are known as frequency of occurrence of variable x, and
x is usually called weighted mean of x1 and
x2.

If expression for rcm mass plays the role of frequency or (statistical) weight.

www.physicsashok.in 1
CENTRE OF MASS

Definition
For a two particle system, the centre of mass is the point whose position m1
vector is the weighted mean of position vectors of the two particles with masses
C
of particles playing the role of statistical frequency.

In the figure rcm is the position vector of point C which is the CM of m1 and m2
r1
m2. Then
r2
m1 r1 m 2 r2
rcm =
m1 m 2 O

C1 : Properties
r1 r2
1. The centre of mass of two particles lies on the line joining them.
m1 m2

To this, let r1' and r 2' be position vector of m1 and m2 relative to CM(C).
rcm
Then, m1 ( rcm + r1' ) + m2( rcm + r1' ) r1
r2

= (m1 + m2) rcm

m1 r1' + m2 r1'

Thus, r1' and r 2' are directed opposite to each other. This shows that r1' and r 2' are at 180 with each other
but their tails lie at a common point. Thus, C lies in the line joining the particles.
2. The centre of mass divides the distance between two particles in the inverse ratio of masses.
As shown in the previous section

m1 r1' = m2 r 2'

| r1'| m2
= [taking modulus]

| r2' | m1

1
Thus, the centre mass is dividing the distance in inverse ratio. Thus, r . Thus, the centre mass less closer
m
to more massive particle.

Note: Using m1r1 = m2( r1 ) m1 r1 C m2 > m1



m2
r1 = m m
1 2

3. The sum of first moment of masses of particles about centre of mass is zero.

The first moment of mass m1 about C is m1 r1' and that of m2 is m2 r 2' . The sum is m1 r1' + m2 r 2' . This sum is
zero.
4. When CM does not move,

rcm = 0

m1 r1 + m2 r2 = 0

www.physicsashok.in 2
CENTRE OF MASS

The relative displacement of 1 as seen by 2 is given by



r1 r2 = r12
Using these, we have

m1 r1 + m2( r1 r12 ) = 0

(m2 + m1) r1 m2 r12 = 0

m 2 r12
r1
m1 m2
Similarly,

m r
r2 = 1 21
m1 m 2

C2 : The masses of two particles are 300 g and 200 g. They are separated by 60 cm. Determine the distance of
centre of mass from 200 g particle.
Sol:
m2 200 g This dist (r1) 300 g
m1 m2
We have =
r1' m m r 1
1 2
m1 C m2 That mass
300
= 60 cm
200 300
3
= 60 cm
5
= 36 cm.

C3 : A system consisting of two masses connected by a massless rod lies along the x-axis. A 0.4 kg mass is at
x = 2m while a 0.6 kg mass is at x = 7m. Calculate the x co-ordinate of the centre of mass.
m1x1 m 2 x 2 (0.4 2) (0.6 7)
Sol. XCM = m1 m 2 = =5m
(0.4 0.6)

C4 : Four particles of masses 1 kg, 2 kg, 3 kg and 4 kg are placed at the corners A, B, C and D respectively of a
square ABCD of edge 1 m. If point A is taken as origin, edge AB is taken along x-axis and edge AD is taken
along y-axis, the co-ordinates of centre of mass in SI is
(a) (1, 1) (b) (5, 7) (c) (0.5, 0.7) (d) None of these
mA x A mBx B mC x C mD x D y
Sol. XCM = mA mB mC mD

1 0 2 1 3 1 4 0 D C(1, 1)
= (0, 1)
1 2 3 4
23 1
= = = 0.5 m
10 2
A x
(0, 0) B(1, 0)

www.physicsashok.in 3
CENTRE OF MASS

m A y A m B y B mC yC mD y D
Similarly, yCM = mA mB mC mD
1 0 2 0 3 1 4 1
=
1 2 3 4
7
= = 0.7 m
10

C5 : A man of mass M stands at one end of a plank of length L which lies at rest on a frcitionless surface. The man
walks to the other end of the plank. If the mass of plank is M/3, the distance that the mass moves relative to
the ground is
3L L 4L L
(a) (b) (c) (d)
4 4 5 3
Sol: Since, external force on system is zero so, no change takes place in the centre of mass.
m1x1 m 2 x 2
XCM = m1 m 2
m1x1 m 2 x 2
or XCM = m1 m 2
Here, xCM = 0
m1x1 + m2x2 = 0
M
or Mx1 + x2 = 0
3
M x
x1 = x2 = 2 .....(i)
3M 3
But xrel = x1 x2
or L = x1 x2
x 2 4
or L = x2 = x2 [From equation (i)]
3 3
3
x2 = L
4
x 2 3L L
x1 = = =
3 4 3 3
Negative sign indicates that both moves in opposite directions.

Many particle system



Let r1 , r2 ........... rN be position, vectors of masses m1, mi..........., mN in a system of N-particles. Then rcm is
given by

m1 r1 m 2 r2 ......m N rN
rcm =
m1 m 2 ......... m N

www.physicsashok.in 4
CENTRE OF MASS

In general,
N m1 m2
Rod
mi ri mi
i 1 mN System
rcm = N [discrete mass distribution]
System
m i
i 1 Discrete mass distribution Continuous mass distribution
If the system has continuous distribution of mass we have.

dm r
rcm = [continuous mass distribution]
dm
where integration covers the whole body.
C1
C
C2
System of systems Rcm
If there be several systems having masses M1, M2 and having CMs at R1 R2
C3
R 1, R 2 then CM of the system of systems is given by similar formula:
R3

M i R i
R cm = M i

CENTRE OF MASS OF SOME RIGID BODIES


We have several objects around us whose shapes and sizes remain the same. Such bodies are known as rigid
bodies. The motion of such bodies can be described using their centres of mass. We take up the takes of
locating centres of mass of a few rigid bodies like uniform rod, ring, disc, sphere, etc.

Locating centre of mass of uniform rod


Let a rod of length L be oriented along x-axis with ends at x = 0 and x = L.
Let an elementary mass dm be located at position x. If be mass per x=0 X
x=L
unit length, then x dm
dm = dx.
The x-coordinate of centre of mass may be written as
L

dm x ( dx)x
xcm = = 0
m L

[x 2 / 2]L0 L
= =
L 2
This denotes mid point of the rod. Thus, centre of mass of uniform rod lies at the centre of the rod.

Example 1. A non-uniform thin rod of length L is placed along X-axis as such it one of ends is at the origin. The
linear mass density of rod is = 0 x. The distance of centre of mass of rod from the origin is
L 2L L L
(a) (b) (c) (d)
2 3 4 5

www.physicsashok.in 5
CENTRE OF MASS

Sol: The mass of considered element is


dm = dx = 0 xdx
L L
dm
x dm x(
0
0 x dx) O
0 x
XCM = = L
dx
dm 0 x dx
0
L
x3
0 L3
3 0 0
3 2
= = 2 = L
2 L L 3
x
0 0
2
2
0

Locating centre of mass of circular ring


Let a circular ring be placed with centre at origin in XY plane as in the figure.
Let us choose an elementary mass dm at angular position . Then Y
x = R cos
y = R sin dm
dm = Rd [ = mass per unit length] d
The x-co-ordinate of centre of mass, xcm may be written as
=
2
X
=0
( Rd).R cos
0
( = 2)
xcm =
(2 R)
3
=
2
R 2
= [sin ]20 = 0
(2R)
The y-co-ordinate of centre of mass, ycm may be written as
2

( Rd).R sin
ycm = 0

(2 R)

R 2
= [ cos ]20 = 0
(2R)
Thus, the centre of mass of uniform ring lies at the centre of the ring. Y

Locating centre of mass of a uniform circular disc y=R

Let be the mass per unit area of the disc of radius R. Let it be dy
R y
composed of uniform rods of proper lengths. One such strip (rod)
X
2 2
is located at position y. Its width is dy and length is 2 R y .
Its whole mass is centered at position (0, y). Using the formula for
system of systems, we have
y = R

www.physicsashok.in 6
CENTRE OF MASS
R

ycm = {dy(2 R 2 y2 )}y / (R 2 )


R

R
2
= y R 2 y2 dy
R 2 R

For integration let us consider


d
2 2 3/2
2
3 2 2 3/2 1 d R y
2

(R y ) = (R y )
dy 2 dy
3
= ( R 2 y 2 )( 2y)
2
= 3 R 2 y 2 y

(R 2 y 2 )3/ 2
d = R 2 y 2 ydy
3

(R 2 y 2 )3/ 2

d
3
=
R 2 y 2 y dy

1
or, (R2 y2)3/2 = R 2 y 2 y dy
3
Thus,
2 1
ycm =
2
[(R 2 y 2 )3/ 2 ]R R
R 3
Thus, the centre of mass of a uniform disc lies at its centre.

Locating centre of mass of a uniform sphere Y 2 2


R y
Let us imagine that a solid homogeneous sphere y=R
is made up of elementary discs, one of which is dy
shown in the figure. Its thickness is dy and radius y

is R 2 y 2 . The mass in the volume of the disc


y=0
is (R2 y2)dy, where is density. The centre y=0
of mass of the disc is located at (0, y). Using
formula for CM of system of systems, we have
y = R
R 2 2
4 3
yCM = (R y ) ydy / 3 R
R

4 3
where R is the mass of the sphere.
3
Thus,
R
3 2
ycm = (R ydy y3dy)
4R 3 R

www.physicsashok.in 7
CENTRE OF MASS

R R
3 2 y 2 y 4
= 4R 3 R 2 4 = 0.
R R
Thus, the centre of mass of a homogeneous sphere lies at its centre.
m
Centre of mass of symmetric bodies r
If two equal masses be located in a line through C, at equal distances
C
from C, their centre of mass lies at C. If this is true for all pairs of r
practices forming a system, the centre of mass lies at the centre of
symmetry. m

CM-1

CM CM CM CM
CM CM

CM-2

NOTE : (i) There need not be a mass at the centre of mass.


(ii) The centre of mass may lie outside the body.

C5 : Which of the following has centre of mass not situated in the material of body ?
(a) A rod bent in the form of a circle (b) Football
(c) Handring (d) All of the above
Sol: (a) If a rod is bent in the form of a circle, its centre of mass is at the centre of curvature.
(b) Football is in teh shape of hollow sphere. So, its centre of mass is at the centre of sphere
(c) The reason is same as (a).
In all the above geometrical shape, no material is present at the centre of mass.

Example 2. Find the position of centre of mass of the uniform lamina shown in figure.
Y
Sol: Here, A1 = area of complete circle = a2
A2 = area of small circle

2
a a 2 a
= = O X
2 4
(x1, y1) = co-ordinates of centre of mass of large circle
= (0, 0)

www.physicsashok.in 8
CENTRE OF MASS

(x2, y2) = co-ordinates of centre of mass of small circle

a
= , 0
2

A1 x 1 A 2 x 2
Using XCM = A1 A 2

a 2 a 1
a
4 2 8
we get XCM = 2 = a=
a 3 6
a 2
4 4
and yCM = 0 as y1 and y2 both are zero.

a
Therefore, co-ordinates of CM of the lamina shown in figure are , 0 .
6

Example 3. In the figure shown a hole of radius 2 cm is made in a semicircular disc of


2cm
radius 6 at a distance 8cm from the centre C of the disc. The distance of the centre

8cm
of mass of this system from point C is : C
(A) 4 cm (B) 8 cm (C) 6 cm (D) 12 cm
Sol. The centre of mass of semicircular disc is
4r y
3
m2
The position of COM of this system from point C is
m y m2 y2 8cm
y cm 1 1 ...(i) m1
m1 m 2
m1 r12 (6 ) 2
m1 36 3
2
m 2 r22 2
m2 4
From equation (i)
4
36 3 6 4 8
y cm 3
36 3 4

36 3 8 32
y cm
36 3 4

72 2 8 701.89
y cm = = 8 cm
9 2 1 87.73

www.physicsashok.in 9
CENTRE OF MASS

IMPORTANT FEATURES
C6 : Centre of mass of some well known rigid bodies are given below:
(a) Centre of mass of a uniform rectangular, square or circular plate lies at its centre.

CM
CM
CM

(b) Centre of mass of a uniform semicircular ring lies at a distance of


2R
h= from its centre, on the axis of symmetry where R is the radius R CM
2R

of the ring.
O
(c) Centre of mass of a uniform semicircular disc of radius R lies at a
4R
distance of h = from the center on the axis of symmetry as R CM
3 4R

shown in figure.
O
(d) Centre of mass of a hemispherical shell of radius R lies at a distance
R
of h = from its centre on the axis of symmetry as shown in R CM
2 R

figure.
O
(e) Centre of mass of a solid hemisphere of radius R lies at a distance of
3R R CM
h= from its centre on the axis of symmetry.. 3R
8
O

MOTION OF SYSTEM OF PARTICLES


If the particles of a system are displaced, the CM may or may not be displaced. But the displacement of CM
is possible only lay displacing the particles.

Displacement of centre of mass



If ri be the displacement of ith particle (mass = mi), then the displacement of centre of mass, rcm may be
written as

mi ri
rcm =
m i
where summation runs over all the particle in the system.

C7 : Two particles of masses 10 g and 20 g are displaced such that centre of mass does not move. If 10 g particle
is displaced by 0.02 cm, what is the displacement of 20 g particle ?

www.physicsashok.in 10
CENTRE OF MASS

mi x i
Sol: Using xcm = mi , we have 10 0.02 + 20 xm = 0
x = 0.01 m
The negative sign shows that the direction of displacement of this particle is opposite to that of the given
displacement.

The velocity of centre of mass:



If the particles of a system have velocities 1 , 2 , ......... N , the velocity of centre of mass of the system is
defined by
Y
mi i
cm = 3ms1
mi System 2ms1

where summation includes all the particles within the system. 1.4 ms1
0.50 kg 60
7 kg
C8 : Consider the particles confined in the system. Write X-components 1.2 kg 37 1
1.5ms
of the velocity of centre of mass of the system. 5 kg
37 1.5 kg
mi i x 2ms1
1.0 ms1 37
Sol: We have cmx = 1.0 kg 2ms
1
mi
1.2 0 0.50 3 1.0 ( 1.5 cos 37 ) 1.0 2 cos 37 1.5 1 cos 37 1
= ms
1.2 0.50 1.0 1.0 1.5
0 1.5 1.2 1.6 0.3
= 5
5.2 3

2.2 1 37
= ms = 0.45 ms1
5.2 4
Note that 7 kg and 5 kg particles are outside the system.

C9 : Two blocks of masses m1 and m2 are lying on a table when m2 is given a velocity 0 to the right. Write the
velocity of the centre of mass.
m1 m2

m1 0 m 2 Rest 0
0
Sol: Here cm = m1 m 2
m2
= m m 0
1 2

Momentum and centre of mass motion



The momentum of a system of particles ( P ) is defined as the vector sum of momenta of all the particles
enclosed.

Thus, m i = P

mi i P
However, cm = =
mi M

www.physicsashok.in 11
CENTRE OF MASS

P = MVcm

Total momentum = Total mass Vcm .

C10 : The mass of a system is 200 kg. Its particles are moving in unknown ways but centre of mass is moving at 10
ms1. What is the magnitude of the momentum of the system ?

Sol: Using P = MVcm

| P | = 200 kg 10 ms1
= 2000 kg ms1

The Acceleration of centre of mass


The rate of change of velocity of centre of mass is its acceleration.

If various particles have acceleration a i , the acceleration a cm is given by

mi a i
a cm =
mi
Here, summation runs over all the particles in the system.

C11 : Two blocks in an Atwood machine have masses m and 2m. Calculate the acceleration of centre of mass
of the two blocks when they are released, by using the formula for individual acceleration.
m 2 m1
a = m m g.
2 1

Sol: In the figure m is accelerating upward while 2m downward.



a1 = acceleration of m
2m m
= g, up
2m m

g
=
3 m

a 2 = acceleration of 2m
2m

2m m g
= g, downward =
2m m 3


m1a1 m 2 a 2
Using, a cm = , we have
m1 m 2

m( g / 3) 2m(g / 3) g
a cm = =
m 2m 9

g
Thus, the acceleration of CM is .
9

www.physicsashok.in 12
CENTRE OF MASS
Dynamics of system of particles
Now we consider forces acting on various particles in a system.


Let F1 be the net force acting on a particle of mass m1 producing acceleration a1 in it. Then according to
Newtons second law of motion,


F1 = m1 a1

Now F1 may arise due to internal force F1int and force exerted to by particles outside the system, F1ext . That
is

F1 = F1ext + F1int
Similarly we can write the following:


m1 a1 = F1 = F1int + F1ext fji
fji
m2 a2 = F2 = F2int + F2ext




mi a1 = Fnet = Fint + Fext
Now the sum of internal forces is zero as

Fij + Fji = 0 [Newtons third law of motion]

M acm = Fext
This shows that
1. Net force on a system of particles is equal to net external force.
2. Sum of internal forces on particles of a system is zero.
3. CM cannot be accelerated by internal forces.
4. External forces equal mass multiplied by acceleration of CM.
5. Action point of external force is immaterial in producing acceleration in centre of mass.

C12 : If a projectile exploids in air in different parts, the path of the centre Explosion
of mass remains unchanged. This is because during explosion no U B
external force (except gravity) acts on the centre of mass. The situation
Path of COM
is as shown in figure.
Path of COM is ABC, even though the different parts travel in different A
C
directions after explosion,.
C13 : Suppose a system consists of more than one particle (or bodies). Net external force on the system in a
particular direction is zero. Initially the centre of mass of the system is at rest, then obviously the centre of mass
will not move along that particular direction even though some particles (or bodies) of the system may move
along that direction. The following example will illustrate the above theory.

C14 : Can a car accelerate on a smooth horizontal road ?


Sol: No, for accelerating the car (i.e. accelerating the centre of mass of the car), external horizontal force is
needed. This force can come only from friction. Here the road is smooth. Hence no horizontal external force
acts on the car. It cannot accelerate.
www.physicsashok.in 13
CENTRE OF MASS

C15 : A uniform rod of mass m and length L is tied to a vertical shaft. It


rotates in horizontal plane about the vertical axis at angular velocity
. How much horizontal force does the shaft exert on the rod ? L

L
Sol: Let T be the force applied horizontally on the rod. The acceleration of CM is 2 , as it is moving in a circle
2
L
of radius .
2
2
Using Fext = M acm acm = (L/2)
T
cm
L 2
T = M
2
1
Thus, the horizontal force exerted is M2 L.
2

C16 : A chain of irregular shape and total mass m is tied to a string whose
other and is connected to the shaft. The shaft make it rotate at angular
velocity as in the figure.
Calculate the distance of centre of mass of the chain from the centre of
circle in which the CM is moving. r m
Sol: Here, acm = 2r

Fext = T + mg

Using, Fext = M a cm
T
Fextc = mac
Fext= ma = 0
Here acm is horizontal vector..
2
T sin + 0 = m r .....(i) acm
T cos mg = 0 .....(ii) CM m
2 C
r
From these, tan =
g
g mg
r= tan.
2
This radius is the required distance.

C17 : A ball kept in a closed box moves in the box making collisions with the walls. The box is kept on a smooth
surface. The centre of mass
(a) of the box remains constant
(b) of the box pulls the ball system remains constant
(c) of the ball remains constant
(d) of the ball relative to the box remains constant
Sol: Since, no external force is present on teh system so, the centre of mass of system will not be changed.

www.physicsashok.in 14
CENTRE OF MASS
C18 : A loaded spring gun of mass M fires a shot of mass m with a velocity at an angle of elevation . The gun is
initially at rest on a horizontal frictionless surface. After firing, the centre of mass of the gun-shot system
m
(a) moves with a velocity
M
m
(b) moves with a velocity cos in the horizontal direction
M
(c) remains at rest
(M m)
(d) moves with a velocity in the horizontal direction
(M m)
Sol: If we consider a system consisting of gun and mass m, then no external force is present on the system. So, the
acceleration of centre of mass of the system will be zero. Since, initially the system is in rest so, centre of mass
will always remain at rest.

C19 : Two blocks A and B are connected by a massless string (shown in


figure). A force of 30 N is applied on block B. The distance travelled B
by centre of mass in 2 s starting from rest is A
10 kg 20 kg F = 30N
(a) 1 m (b) 2 m Smooth
(c) 3 m (d) non of these
Sol: The acceleration of centre of mass is
F
aCM = m m
A B

30
= = 1 m/s2
10 20
1 1
s= aCMt2 = 1 22 = 2m
2 2
C20 : Two bodies A and B of masses m1 and m2 respectively are connected by a massless spring of force constant
k. A constant force F starts acting on the body A at t = 0. Then
F k
(a) at every instant, the acceleration of centre of mass is m m m2 m1 F
1 2 A B
(b) at t = 0, acceleration of B is zero but that of A is maximum
(c) the acceleration of A decreases continuously
(d) all of the above
Sol: (a) Since, external force on the system is F
F
aCM = m m
1 2
(b) At t = 0, elongation in spring is zero. So, force on m2 is zero, but force on m1 is only in forward direction,
i.e., F.
F
a1 = m at t = 0
1

This is maximum acceleration of m1. Hence, (d) is correct.

www.physicsashok.in 15
CENTRE OF MASS
C21 : In the given figure, two bodies of masses m1 and m2 are connected by massless spring of force constant k and
are placed on a smooth surface (shown in figure), then:
k
(a) the acceleration of centre of mass must be zero at every instant F m1 m2 F
(b) the acceleration of centre of mass may be zero at every instant
(c) the system always remains in rest
(d) none of the above
Sol: The resultant force on the system is zero. So, the centre of mass of system has no acceleration.

Example 4. Two planks each of mass m and length L are connected by a frictionless,
massless hinge as shown in the figure. Initially the system is at rest on a level frictionless
surface. The vertical plank falls anticlockwise and finally comes to rest on the top of
the horizontal plank.
Find the displacement of the hinge till the two planks come in contact.
Sol. In horizontal direction centre of mass does not move.
xic = xf c

L L
0 m m 2m x 0
2 2 A
=
2m 2m L
2
B O
L L L
or = x0 2
4 2
C
L L 2L L L x0
x0 = = = L x0
2 4 4 4 2

nv
Example 5. Two masses, nm and m, start simultaneously from the intersection of two
m
straight lines with velocities v and nv respectively. It is observed that the path of their
centre of mass is a straight line bisecting the angle between the given straight lines. Find
the magnitude of the velocity of centre of inertia. cm
[here = angle between the lines] O v
nm

m1 v1 m1 v 2
Sol. v cm
m1 m 2

nm v i m nv cos i nmv sin j


v cm
m nm y
2 2
nmv mnv cos nmv sin
v cm
m 1 n
x
2 2
nmv 1 cos sin
v cm
m 1 n

nv 1 cos 2 2 cos sin 2



1 n
www.physicsashok.in 16
CENTRE OF MASS

nv
2 2 cos
n 1

nv
2 2 cos 2
n 1 2

nv
2 cos
n 1 2


2nv cos
v cm 2
n 1

Example 6. Two blocks of equal masses m are released from the top of a smooth
fixed wedge as shown in the figure. Find the magnitude of the acceleration of the m m
centre of mass of the two blocks. 30 60

mg sin 30 i mg sin 60 j
Sol. a cm 90
mm m m
g sin 30 i g sin 60 j
mgsin30
a cm mgcos30
2 30 60
g 3g x y
acm i j
4 4
acm = g/2

Example 7. An open water tight railway wagon of mass 5 103 kg coasts at an initial velocity 1.2 m/s without
friction on a railway track. Rain drops fall vertically downwards into the wagon. The velocity of the wagon after
it has collected 103 kg of water will be
(A) 0.5 m/s (B) 2 m/s (C) 1 m/s (D) 1.5 m/s
Sol. Applying conservation principle of momentum,
mivi = mfvf
5 103 1.2 = (5 103 + 103) vf
5 103 1.2
vf 1m/s
6 103

Example 8. A parallel beam of particles of mass m moving with velocity v impinges on a wall at an angle to its
normal. The number of particles per unit volume in the beam is n. If the collision of particles with the wall is
elastic, then the pressure exerted by this beam on the wall is :
(A) 2 mn v2 cos (B) 2 mn v2 cos2 (C) 2 mn v cos (D) 2 mn v cos2
Sol. The change in velocity is v = 2v cos
The no. of particles in volume dv = sdx is dN = ndv = nsdx
dm = mdN = mn sdx
dm dx
mns mnsv
dt dt
vsin
dp dm s
F v v v
dt dt
F = (2v cos )mnsv www.physicsashok.in 17
vcos
vcos
n
CENTRE OF MASS

F = 2mn v2s cos



F.s
Pressure, p 2
s
Fs cos F cos
p
s2 s
2 2
p = 2mn v cos

THE C-FRAME
In many cases when we examine only the relative motion of particles within a system, but not the motion of this
system as a whole, it is mot advisable to resort tot he reference frame in which the centre of inertia is at rest.
Then we can significantly simplify both the analysis of phenomena and the calculations.
The reference frame rigidly fixed to the centre of inertia of a given system of particles and translating with
respect to inertial frames is referred to as the frame of the centre of inertia, or, briefly, the C frame. The
distinctive feature of the C frame is that the total momentum of the system of particles is equal to zero; this
immediately follows from equation. In other words, any system of particles as a whole is at rest in its C frame.
The C frame of a closed system of particles is inertial, while that of a non-closed system is non-inertial in the
general case.

Example 9. In the diagram shown, no friction at any contact surface.


Initially, the spring has no deformation. F

What will be the maximum deformation in the spring ? Consider 2M M


all the strings to be sufficiency large. Consider the spring constant to be K.
(A) 4F / 3K (B) 8F / 3K (C) F / 3K (D) none
Sol. At the time of maximum separation, blocks come in rest in C-frame.
In C-frame,
T = 0 v=0 v=0
Wpeseudo =WS + W1F + W2F = T = 0 ...(i) 2M M
2F F
Workdone by pseudo frame in C-frame is zero.
W1F = Fx1C
W2F = 2Fx2C
1
0 kx 2max Fx1C 2Fx 2C 0 ...(ii)
2
1 2
kx max Fx1C 2Fx 2C
2
xmax = x1C + x2C
From the concept of centre of mass
Mx1C = 2Mx2C
x1C = 2x2C
x1C + 2x2C = xmax x
3x2C = xmax

www.physicsashok.in 18
CENTRE OF MASS
x max
x 2C
3
2x max
x1C
3
Putting the values in equation (2)
1 2 2x x
kx max F max 2F max
2 3 3
1 2F 2F
kx max
2 3 3
8F
x max Ans.
3K

MOMENTUM CONSERVATION AND CENTRE OF MASS MOTION


The momentum of a system of particles is the vector sum of the momenta of all its particles. This is related to the
velocity of centre of mass as

P = M Vcm
Hence momentum of a system is possible due to centre of mass motion.

The change in momentum P of a body occurs only if an external force be present

dP
= Fext
dt

What does happen if Fext be zero ?

The Law of Conservation of momentum


We know that

Fext = M a am

Fext = 0 acm = 0 Vcm = constant

M Vcm = constant

P = constant.
If the external force on a system be zero, its momentum remains constant.
This is the law of conservation of total momentum of a system of particles.

Conservation of component of momentum



If external force is not zero, then a component of P taken perpendicular to the Fext is conserved.

C22 : A block is placed on a smooth triangular block and released. Fext x = 0


W1 acm
(a) Is external force on the system zero ? X Px = const
(b) Which component of total momentum is conserved ? W2

www.physicsashok.in 19
CENTRE OF MASS

Sol: The external forces on the system are pull of the earth and
normal reaction of table - both are vertical. Since the block slides down, the centre of mass is coming down
from rest into motion. Hence external forces are not balanced. But the x-component of forces is zero. Hence
Px is conserved.
(a) No.
(b) Horizontal component (X- component in the figure).

Example 10. Two balls of equal masses are projected upward simultaneously, one from the ground with speed 50
m/s and other from a 40 m high tower with initial speed 30 m/s. Find the maximum height attained by their
centre of mass.
Sol. The initial position of centre of mass is
m 0 m 40 y
yic = = 20m
2m
30 m/s
m 50 m 30 B m
ucm = = 40 m/s
2m
acm = g h = 40m
2 2
vcm = u cm + 2acmH
50 m/s
0 2 = 402 2 10 H A m
1600
H = = 80 m
20
yfc yic = 80
yfc = 80 + 20 = 100 m

C23 : A body at rest break up into three parts. If two parts having equal masses fly off perpendicularly to each other
with a velocity of 18 m/sec, then calculate the velocity of the third part which has a mass 3 times the mass of
each part. mv1

Sol: 6 2 m/sec at an angle 135 from the either ball. mv1 2


3mv = mv1 2

2v1 2 18
v= = =6 2 mv1
3 3

3 mv
C24 : A body which is moving with a velocity of 10 m/sec towards east breaks up into three equal parts. If two of
them move in the north and south direction (i.e., moves up and down) respectively with a velocity of 10 m/sec,
then calculate the velocity of third part.
Sol: mv1

Momentum along X-axis before and after explosion is conserved. mv2


Mv = 0 + 0 + mv2 cos


www.physicsashok.in 20
Mv

mv1
CENTRE OF MASS
Mv = mv2 cos .....(i)
Momentum along Y-axis before and after explosion is conserved.
mv1 mv1 + mv2 sin = 0
mv2 sin = 0 or, = 0
Mv 3m 10
From (i) v2 = = = 30 m/sec.
m cos m 1

Example 11. A hemisphere of radius R and of mass 4m is free to slide with its base on a
smooth horizontal table. A particle of mass m is placed on the top of the hemisphere.
Find the angular velocity of the particle relative to hemisphere at an angular displacement
when velocity of hemisphere has become v.
Sol. Actual velocity of ball is v1 is horizontal direction. N
vrel cos
v1 (v) = vrel cos
v
v1 + v = vrel cos ...(i)
Applying conservation principle of momentum in horizontal direction. vrel sin
Pix = Pfx
0 = mv1 4mv
v1 = 4v ...(ii)
Applying mechanic energy conservation principle
Loss in P.E. = gain in K.E.
2
1
mg r r cos
2
m v12 v rel sin
2
1
4mv 2
2
...(iii)

After solving equation (i), (ii) and (iii)


5v
v rel
cos
v 5v
rel rel
R R cos

C25 : Disintegration of stationary nucleus U238


Initially U238 is in state of rest, so its momentum is zero. Therefore, after disintegration, momentum will be zero.
Hence,
U238
U238
92 90
Th234 + 2He4 92

-particle
or M234 v 234 + M v = 0
234 4
M 4 92 U 2 H

or v 234 = M v = v
234 234

K M 234
and = 234 =
K Th M 4
Disintegration of stationary kayon

Final momentum P P = 0 Before disintegration
K0 (at rest)

P P+
www.physicsashok.in After 21
disintegration
+
K +
0 +
CENTRE OF MASS


So, P P = 0

or P = P or | P | = | P |

K m 1
Thus, K = m = [ m+ = m]
1
C26 : Recoil force
In n balls reach of mass m are fired from a cannon time t second, then
nmv nmv
recoil force F = rate of change of momentum F= t
= reaction force
Cannon
= upthrust
nm
= N
t
C27 : A man of mass m1 is standing on platform of mass m2 kept on a smooth
horizontal surface. The man starts moving on the platform with a velocity r
relative to the platform. Find the recoil velocity of platform. vr v
Sol: Absolute velocity of man = r , where = recoil velocity of platform. v

Taking the platform and the man as a system, net external force on the system in horizontal direction is zero.
The linear momentum of the system remains constant. Initially both the man and the platform were at rest.
Hence 0 = m1(r ) m2
m1 r
= m m
1 2

Example12. A gun (mass = M) fires a bullet (mass = m) with speed 60 vsin60


r

r relative to barrel of the gun which is inclined at an angle of 60


with horizontal. The gun is placed over a smooth horizontal surface. v M vr cos60v
Find the recoil speed of gun.
Sol: Let the recoil speed of gun is . Taking gun + bullet as the system. Net external force on the system in
horizontal direction is zero. Initially the system was at rest. Therefore, applying the principle of conservation of
linear momentum in horizontal direction, we get
Mv m(r cos60 v) + 0
n r cos 60
=
Mm
n r
or =
2(M m)
C28 : A bullet of mass 20 gram is fired from a gun of mass 6 kg, if the speed of the bullet is 600 m/sec, then calculate
the speed of the recoil of the gun.
Sol: As the momentum of the bullet and the gun must be equal and opposite therefore,

www.physicsashok.in 22
CENTRE OF MASS

20
MV = mv or 6V = 600
1000
V = 2 m/sec.

C29 : A machine gun can fire bullets of 50 grams at a speed of 2000 m/sec; the man holding the gun can exert an
average force of 200 newtons against the gun. Calculate the maximum number of bullets which he can fire per
minute.
Sol: We have, Force = mnu
where m = mass of bullet, u = speed of bullet,
n = number of bullets per unit time.
50
F = mnu or, 200 = 2000 n
1000
Hence, number of bullets fired per minute = 60 2 = 120.

C30 : A plate of mass M is held at rest by firing bullets from below. Each M
bullet has a mass m, velocity u(up) just before hitting and stops after
hitting the plate for a moment and falls. Determine the number of
bullets striking the plate per unit time.


Sol: The change in momentum of a bullet due to hitting the plate is p = 0 mu . If N bullets be hitting the plate

per unit time, the rate of change of momentum of the bullets is N mu . This is also the force exerted by the

plate during collision. Using Newtons third law of motion, an opposite force, N mu , will act on the plate due
to the colliding bullets. (rest)
u
p
Considering the equilibrium of forces acting on the plate,
m

m
N mu + Mg = 0
The change in
Just after
Nmu = Mg the hitting
bullet's momentum
Going to hit
Mg
N=
mu Nm u

This is the number of bullets striking the plate per unit time.
Mg

C31 : A projectile is launched at an angle with velocity V0. At the highest


V0
point of its path, it explodes into two parts of masses in the ratio 1 : 3.
The lighter mass retraces its path. Find the velocity of the heavier V0
part just after the explosion. 0
Sol: The figure shows the mass m moving with horizontal velocity of V0 cos0 at the top just before explosion. The
lighter mass (m0) formed after explosion retraces its path, its velocity at the top is just reversed due to explosion.
Thus m0 moves leftward at V0 cos0. At this moment, 3 m0 is moving rightward with the requested velocity, V.
Conservation of momentum along horizontal X-axis, gives

www.physicsashok.in 23
CENTRE OF MASS
mV0cos0 = m0(V0 cos0) + 3m0V
V0cos0 V0cos 0
m m
= V0 cos0 + 3. V m V
4 4 m0 3m0
V0 3 X = m/4 =3m/4
Just before X
V0cos0 = cos0 + V Just before
4 4 explosion explosion
5
V = V cos0
3 0
This is the velocity of the heavier part just after the explosion.

C32 : Motion of a ball under gravity


(i) If the ball is the system, the force of gravity will be external and so the linear momentum o the ball will change.
This is not violation of law of conservation of linear momentum as linear momentum is conserved only if
Fext = 0.
(ii) If ball and the earth is the system, all the acting forces will become internal and so, v

p S = p B + p E = constant Ball m

Now as initially both ball and the earth are at rest



p B + pE = 0 Earth M
From this it is clear that

(a) p E = p B , i.e., if ball acquires some momentum, the earth also acquires
V
equal and opposite momentum.


(b) As p = mv + MV = 0, i.e., V = (m/M) v i.e., the velocity of earth is opposite to that of ball in
direction and much smaller in magnitude (as M > > m). So, if ball moves away from the earth, the earth
also moves away from the ball and if ball moves towards the earth, the earth also moves towards the ball
but the speed of the earth is much lesser than that of the ball (as its mass is much greater than that of ball).

p2
KE m
(c) Now as K = and | p E | = | p B |, K = i.e., KE of ball and earth will not be equal and as M >>m
2m B M
the, KE of earth will be much lesser than that of the ball.
(d) Initially KE of the system is zero (as both are at rest) and after some time KE of the system is not zero(as
both are in motion). So, KE of the system is not constant but changes. However, (KE + PE), i.e.,
mechanical energy of the system is conserved. When the ball and earth approach each other KE increases
and PE decreases and when they recede from each other KE decreases and PE increases.

Lecture 5
C33 : Motion of two masses connected by a spring v2 v1
Consider two blocks. resting on a frictionless surface and
connected by a massless spring as shown in figure. If the spring is m2
m1
stretched (or compressed) and then released from rest.
Then as Fext = 0

pS = p1 + p 2 = constant

www.physicsashok.in 24
CENTRE OF MASS

However, initially both the blocks were at rest so,



p1 + p 2 = 0
From this it is clear that:

(i) p 2 = p1 i.e., at any instant the two blocks will have momentum equal in magnitude but opposite in direction
(Though they have different values of momentum at different positions).

(ii) As p = mv , m1 v1 + m2 v 2 = 0, i.e., v 2 = (m1/m2) v1 i.e., the two blocks always move in opposite
directions with lighter block moving faster.
p2 K1 m2
(iii) Now as K = and | p1 | = | p2 | , K = m , i.e., the KE of two blocks will not be equal but in the inverse
2m 2 1
ratio of their masses and so lighter block will have greater KE.
(iv) Initially KE of the blocks is zero (as both are at rest) and after sometime KE of the blocks is not zero (as both
are in motion). So, KE is not constant but changes. here during motion of blocks KE is converted into elastic
potential energy of the spring and vice-versa such that
KE + PE = ME = constant
C34 : (a) Can kinetic energy of a system be changed without changing its momentum ?
(b) Can momentum of a system be changed without changing its kinetic energy ?
Sol. (a) Yes; In explosion of a bomb or inelastic collision between two bodies as force is internal, momentum is
conserved while kinetic energy changes.
(b) Yes; If a force acts perpendicular to motion, work done will be zero and so kinetic energy will remain
constant. However, the force will change the direction of motion and so, the momentum, e.g., in case of
uniform circular motion of a body KE is constant while momentum changes.
C35 : A meteorite burns in the atmosphere before it reaches earths surface. What happens to its momentum ?
Sol. The momentum of meteorite is transferred to air molecules by air drag and so, the momentum of meteorite
plus air molecules (system) always remains constant.
C36 : Can a sail boat be propelled by air-blow. at the sails from a fan attached to the boat ?
Sol. As sails and fan both are attached to the boat, force due to the air-blown on the sails by the fan is an internal
force. Now as by an internal force momentum of the system cannot be changed, so the system (boat + fan +
sails) cannot be propelled by blowing air at the sails from a fan attached to the boat.
C37 : A bird is held in a light container which is completely closed. Can we tell when the bird is resting or flying by
weighing the container ?
Sol. According to the law of conservation of linear momentum we know that the linear momentum of a system of
constant mass remains unchanged in the absence of any external force. Now treating the bird, container and
the air in the container as the system of constant mass we find that the force which the bird exerts in flying is an
internal force. But as by an internal force the momentum of a system cannot be changed, so the weight of the
system will not change, i.e., we cannot tell when the bird is flying or sitting by weighing the container.

Note : However, if in the above problem the container is of wire gauge, the momentum of
the system will not be conserved (why ?) and due to this the weight of the system will be lesser
when the bird is flying as compared to the weight of the same system when bird is resting,
i.e., we will be able to tell when the bird is resting or flying by weighing the container.

www.physicsashok.in 25
CENTRE OF MASS

Example 13. Two trolleys A and B are free to move on a level frictionless track, and are initially stationary. A man
on trolley A throws a bag of mass 10 kg with a horizontal velocity of 4 m/s with respect to himself on to trolley
B of mass 100 kg. The combined mass of trolley A (excluding bag) and the man is 140 km. Find the ratio of
velocities of trolleys A and B, just after the bag lands on trolley B.
Sol. Applying conservation principle of momentum for trolly A and bag
mv m1v1 = 0 v
A B
or mv = m1v1
m1 = 140 kg m2 = 100 kg
or 10v = 140 v1 v1 v2
or v = 14v1 .....(i)
also, v1 + v = 4
or v1 + 14v1 = 4
4 56
v1 = m/s , v = 14v1 = m/s
15 15
Applying conservation principle of momentum for bag and trolley B,
mv = (m + m2)v2
56
10
mv 15 56
v2 = = =
m m2 110 15 11

v1 4 11
v = 15 11 =
2 15 56 14

Example 14. A bob of mass m attached with a string of length l tied to a point on ceiling is released from a position
when its string is horizontal. At the bottom most point of its motion, an identical mass m gently stuck to if. Find
the angle from the vertical to which it rises.

Sol. The velocity of bob just before collision is u = 2g


Apply conservation principle of momentum mu = 2mv0
m
u g
v0 = = =1
2 2

Applying conservation principle of energy,


v0 2m
1
(2m)v 20 = 2mgH
2

v 20 g
H= = =
2g 2 2g 4

1
But = (1 cos) , or = 1 cos
4 4
3 3
or cos = , = cos1
4 4
www.physicsashok.in 26
CENTRE OF MASS

Example 15. A block of mass m starts from rest and slides down a frictionless semi-
circular track from a height h as shown. When it reaches the lowest point of the
track, it collides with a stationary piece of putty also having mass m. If the block
h
and the putty stick together and continue to slide, the maximum height that the
block-putty system could reach is :
(A) h/4 (B) h/2 (C) h (D) independent of h
Sol. v 0 2gh before collision.
Apply conservation principle of momentum,
mv0 = (m + m)u
v0
u
2
Applying conservation principle of momentum,
1
2m u 2 2mgh
2
u2
h
2g

v02
h =
4g
2gh
h =
4g
h
h =
2

Question No. 16 to 19
A small ball B of mass m is suspended with light inelastic string of length L from a A
block A of same mass m which can move on smooth horizontal surface as shown in the L
figure. The ball is displaced by angle from equilibrium position & then released. L

Example 16. The displacement of block when ball reaches the equilibrium position is u=0
B
L sin
(A) (B) L sin (C) L (D) None of these
2
Sol. If ball and blocks are taken as system, then in horizontal direction no external force is present. Hence, centre
of mass remains in rest in horizontal direction.
m(L sin x) mx = 0
or mL sin mx mx = 0
L
x sin
2
B
Example 17. Tension in string when it is vertical, is
(A) mg (B) mg(2 cos) (C) mg(3 2 cos ) (D) None of these
Sol. Loss in P.E. = gain in K.E.
1 1
mg(L L cos ) = mv12 mv 22
2 2
www.physicsashok.in 27
CENTRE OF MASS

Applying momentum conservation principle in horizontal direction.


Pix = Pfx
0 = mv1 mv2
v1 = v2 = v
mg(L L cos ) = mv2
v gl (1 cos )

m(2v 2 )
T mg
l
4mg l 1 cos
T mg
l
T = mg + 4mg (1 cos )
Hence (D) is correct.

Example 18. maximum velocity of block during subsequent motion of the system after release of ball is
(A) [gl (1 cos)]1/2 (B) [2gl (1 cos)]1/2
(C) [gl cos]1/2 (D) informations are insufficient to decide
Sol. v max gl 1 cos
(From sol. of previous problem.)

Example 19. The displacement of centre of mass of A + B system till the string becomes vertical is
L L
(A) zero (B) 1 cos (C) 1 sin (D) None of these
2 2
m1 y1 m 2 y 2
Sol. y cm
m1 m 2
m1y1 m 2 y 2
ycm
m1 m 2
0 mL 1 cos
y cm
mm
L
ycm 1 cos in downward direction.
2

IMPULSE AND IMPULSIVE FORCE


A great force acting for a very short interval of time on a body is called an impulsive force. There are numerous
examples of impulsive forces. When we kick a ball, the force on the ball is an impulsive force. The force
experienced by the coil of a moving coil galvanometer due to sudden flow of current is an impulsive force.
According to Newtons second law,

m(v u)
F = ma =
t

where F is the average force during the short interval t and a is the average acceleration produced. The
www.physicsashok.in 28
CENTRE OF MASS

impulse of an impulsive force is the great change in momentum produced by it.



Thus the impulse of an impulsive force = change in momentum = F t.
A
In fact an impulsive force is variable force. The plot of an impulsive
(a)
force is a curve which is zero in the beginning and the end of the interval
showing a peak value at some intermediate instant. F
A
The total impulse of the force during the interval for which it acts is given (b)

by the summation of the impulse from instant to instant. At the instant t the O O t
elementary impulse is F.dt.
t
total impulse = Fdt
0

= area OAO
= change in momentum.
When a cricket ball is hit by a bat, it exerts an impulsive force on the ball and the impulse of the force on the
balls is so great that it moves in the opposite direction with tremendous speed. If the same ball is to be stopped
in the same interval t by a fielder he must apply the same average impulsive force of figure (a). The force being
very large he (the fielder) will feel severe pain, particularly in the bare hands. If the lowers his hand so as to
lengthen the duration of the force, the same change of momentum will need a comparatively smaller force
(figure b) and so he will not feel the pinch of catching the ball.
For the same reason when a man jumps from a high wall on the side concrete road he hurts himself more
seriously than when he jumps from the same wall on a heap of sand.

C38 : A tennis ball of mass 200 gm is dropped on the floor from a height of 1.2 m. It rebounds to a height of 0.9m.
If the ball was in contact with the floor for 0.01s, what was the average impulsive force and what was its
impulse.
Sol. Let 0 be the velocity of the ball immediately before reaching the ground.
02 = 2 9.8 1.2
Let n be the velocity after bouncing n2 = 2 9.8 0.9
Taking upward as the positive direction

2 9.8 .9 = 2 9.8 1.2 + a 0.01 [From = 0 + at)
where a is the average acceleration (upward) during the interval.

4.2 4.85 9.05


or a = = = 905 ms2
.01 .01

200
F = ma = 905 = 181 newton.
1000

Impulse F .t = 181 .01 = 1.81 kms1.

C39 : A croquet ball (0.5 kg) is struck by a mallet, receiving the impulse shown in graph. What is the balls velocity

www.physicsashok.in 29
CENTRE OF MASS
just after the force has become zero ?

Sol: Each square represents 100 1 103 = .1 Ns 1500 A

Force in newton
There are in all 42 2 = 84 squares (approximately) within OAA.
Area of OAA = 84 0.1 = 8.4 newton second 1000
= impulse experienced
= change in momentum 500
= 0.5 v 0.5 0
= 0.5 v A
0 4 8
Time in milli second
8.4
= = 16.8 ms1
0.5
C40 : A truck of mass 2 103 kg travelling at 4 m/s is brought to rest in 2 s when it strikes a wall. What force
(assume constant) is exerted by the wall ?
Sol: Using impulse = change in linear momentum
We have F.t = mvf mvi = m(vf vi) F u
+ve
or F(2) = 2 103 [0 (4)]
or 2F = 8 103
or F = 4 103 N

^ ^ ^
C41 : A ball of mass m, travelling with velocity 2 i 3 j receives an impulse 3m i . What is the velocity of
the ball immediately afterwards ?

Sol: Using j = m( v f v i )
^ ^ ^
3m i = m[ v f ( 2 i 3 j )]
^ ^ ^
or vf = 3 i + ( 2 i 3 j )
^ ^
or vf = i 3 j

C42 : A bullet of mass 103 kg strikes an obstacle and moves at 60 to its original direction. If its speed also
changes from 20 m/s to 10 m/s. Find the magnitude of impulse acting on the bullet.
Sol: Mass of the bullet m = 103 kg
Consider components parallel to J1
J1 = 103[10 cos60 (20)] 20 m/s J1 10 cos60

or J1 = 15 103 N-s 120 60


10 m/s
Similarly, parallel to J2, we have J2
10 sin60
J2 = 103[10 sin60 0] = 5 3 103 N-s
The magnitude of resultant impulse is given by

www.physicsashok.in 30
CENTRE OF MASS

J = J12 J 22 = 103 (15) 2 (5 3)2

or J = 3 102 N-s

Example 20. A force exerts an impulse I on a particle changing its speed from u to 2u. the applied force and the
initial velocity are oppositely directed along the same line. The work done by the force is
3 1
(A) Iu (B) Iu (C) I u (D) 2 I u
2 2
Sol. I = P
I = 2mu (mu)
I = 3mu ...(i) I +
2u
According to work energy theorem m m
W = T u

W = Tf Ti
1 2 1 2
W m 2u m u
2 2
1 1
W
2

m 4u 2 mu 2
2
3mu 2
W ...(ii)
2
From equation (i) and (ii)
1
W Iu Ans.
2

Example 21. A man whose mass is m kg jumps vertically into air from a sitting position in which his centre of mass
is at a height h1 from the ground. When his feet are just about to leave the ground his centre of mass is h2 from
the ground and finally rises to h3 when he is at the top of the jump.
(a) What is the upward force exerted by the ground on him treating it as a constant ?
(b) Find work done by normal reaction from ground.
F
Sol. a = [Here F is force applied by ground on man]
m
F mg
V02 = 02 + 2 (h h )
m 2 1
After leaving the surface,
0 2 = V02 2g(h3 h2)

V02 = 2g(h3 h2)

F mg
or 2 (h h ) = 2g(h3 h2)
m 2 1
Here F is impulsive for a so mg may be neglected.

www.physicsashok.in 31
CENTRE OF MASS

2F(h 2 h1 )
= 2g(h3 h2)
m
mg(h 3 h 2 )
F= (h 2 h1 )
(b) Since, contact point does not move. Hence, workdone by normal reaction is zero.

Example 22. In the figure shown, each tiny ball has mass m, and the string has length
L. One of the ball is imparted a velocity u, in the position shown, in which the
initial distance between the balls is L / 3 . The motion of ball occurs on smooth 60
u
horizontal plane. Find the impulse of the tension in the string when it becomes taut.
L L
Sol. L
sin 60 3 sin 3
3 1 60
Tdt
sin = 30
2 3 2 v1
Tdt = mv1 and Tdt = mv1 mucos30 Tdt
or mv1 = mv1 mucos30
u cos30
or 2v1 = u cos 30 u cos30 30 u
v1
3u Tout position
v1
4

3 mu
Tdt mv1
4

Example 23. A platform of mass m and a counter weigth of mass (m + M) are connected by a light cord which
passes over a smooth pulley. A man of mass M is standing on the platform which is at rest. If the man leaps
vertically upwards with velocity u, Find the distance through which the platform will descend. Show that when
the man meets the platform again both are in their original positions.
Sol. FdT = Mu
Fdt Tdt = mv0
Tdt = (m + M)v0
Fdt (m + M)v0 = mv0
or Mu = (2m + M)v0
Mu
v0
2m M
The acceleration of platform is
Tdt
a
m M g mg Fdt v0
upward
m Mm m+M
m
Mg v0
a upward
2m M
v2 = v02 2aS

www.physicsashok.in 32
CENTRE OF MASS

2Mg
or 0 v02 S
2m M 0
2

S0
2m M v20 2m M Mu
2m M
2Mg 2Mg

Mu 2
S0
2g M 2m

Example 24. A mass 2m rests on a horizontal table. It is attached to a light


inextensible string which passes over a smooth pulley and carries a mass m at
the other end. If the mass m is raised vertically through a distance h and is
then dropped, what is the speed with which the mass 2m begins to rise ?

h
2gh 2gh
(a) (b)
5 3
2m
gh
(c) (d) 2gh
3
Sol: In the figure it is clear that

v = 2gh .....(i)
Using impulse = change in momentum
For mass 2m
J = 2mv1 .....(ii) J J
For mass m
v v1 v1
J = mn mv1 .....(iii)
Solving equations (i), (ii) and (iii), we get
Just when the string jerks Just after string jerks
2gh
v=
3

COLLISION
A cricket ball is struck by a bat an there is large change ein momentum of the ball and bat in a very short time
of contact. We say that collision between the ball and the bat has taken place. Similarly an -particle moving
towards gold nucleus in Rutherford like experiment undergoes large change in momentum in short time. We say
that collision has taken place. The feature we observe is short time and large change in momentum. The rate
of change of momentum will be very high. The force will be high but short lived. A force of high magnitude but
acting for short period of time is known as an impulsive force. Hence in collision, impulsive forces act on
colliding bodies.

Collision is the process in which bodies exchange energy and momentum in a very short period of time.

www.physicsashok.in 33
CENTRE OF MASS
If a collides with B, the force exerted on A is equal and opposite to that
exerted by A on B, i.e.,

FAB = FBA .
Since these forces are impulsive, external forces may be ignored as they F FBA
BA
produce very small change in momentum of a colliding body in the very short
period. We can ignore this change and may conserve momentum of colliding mg < < FAB
bodies: Total momentum just before the collision equals the total momentum mg (external force)
just after the collision.

Demonstration
Three identical steel balls are suspended by threads of equal length. A is moved to left and released. It goes
to collide with B. But A stops, and C is thrown out. Why ?

A B C A B C A B C

Steel is highly elastic. Hence collision is elastic. A and B collide horizontally. The velocity of A goes to B and
that of B (zero) goes to A. A is stopped, B collides with C. Velocity of C goes to B and that of B goes to C.
Thus B is stopped and C is thrown out. This happens quickly.

DIRECT CENTRAL IMPACT


As an introduction to impact, we consider the collinear motion of two spheres of masses m1 and m2, figure a,
travelling with velocities v1 and v2. If v1 is greater than v2, collision occurs with the contact forces directed along
the line of centres. This condition is called direct central impact. Following initial contact, a short period of
increasing deformation takes place until the contact area between the spheres ceases to increases. At this
instant, both spheres figure b, are moving with the same velocity v0. During the remainder of contact, a period
of restoration occurs during which the contact area decreases to zero. The final condition is shown in the c-part
of the figure where the spheres now have new velocities v1 and v2, where of necessity v1 is less than v2 . All
velocities are arbitrarily assumed positive to the right, so that this scalar notation a velocity to the left would
carry a negative sign. If the impact is not overly severe and if the spheres are highly elastic, they will regain their
original shape following the restoration. With a more severe impact and with less elastic bodies, a permanent
deformation may result.
v1 > v2 v0 v1 < v 2

m1 m2 m1 m2 m1 m2

(a) (b) (a)


Before impact Maximum deformation After impact
during impact

In as much as the contact forces are equal and opposite during impact, the linear momentum of the system
remains unchanged. Thus, we apply the law of conservation of linear momentum and write
m1v1 + m2v2 = m1v1 + m2v2
It is assumed that any forces other than the large internal forces of contact that may act on the spheres during
impact are relatively small and produce negligible impulses compared with the impulse associated with each
internal impact force. In addition, we assume that no appreciable position change occurs during the short
www.physicsashok.in 34
CENTRE OF MASS
duration of the impact.
For given masses and initial conditions, the momentum equation contains two unknowns, v1 and v2 . Clearly,
an additional relationship is required before the final velocities can be found. This relationship must reflect the
capacity of the contacting bodies to recover from the impact and can be expressed by the ratio e of the
magnitude of the restoration impulse to the magnitude of the deformation impulse. This ratio is called the
coefficient of restitution. If F1 and F2 represent the magnitudes of the contact forces during the restoration and
deformation periods, respectively, as shown in figure for particle 1 the definition of e together with the impulse
momentum equation give us

t v1 v2 v0 v0
F dt
t0
r
m1[ v1' ( v 0 )] v 0 v1' m1 Fd m2 m1 Fr m2
e = = =
t0 m 1[ v 0 ( v1 )] v1 v 0 v1 v2
Deformation period
F dt
0
d
Restoration period
Similarly, for particle 2 we have
t

F dt
t0
r
m 2 (v'2 v 0 ) v '2 v 0
e = t0 = =
m 2 (v0 v 2 ) v0 v2
F dt
0
d

We are careful in these equations to express the change of momentum (and hence v) in the same direction as
the impulse (and hence the force). The time for the deformation is taken as t0 and the total time of contact is t.
Eliminating v0 between the two expressions for e gives us

v '2 v1' | relative velocity of separation |


e = = | relative velocity of approach |
v1 v 2
In addition to the initial conditions, if e is known for the impact condition at hand, then equations give us two
equations in the two unknown final velocities v1 and v2 .
Impact phenomena are almost always accompanied by energy loss, Coefficient of
restitution , e
which may be calculated by subtracting the kinetic energy of the system Perfectly elastic
just after impact from that just before impact. Energy is lost through 1
the generation of heat during the localized inelastic deformation of the Glass on glass
material, through the generation and dissipation of elastic stress waves
within the bodies, and through the generation of sound energy.
Steel on steel
According to this classical theory of impact, the value e = 1 means Lead on lead
that the capacity of the two particles to recover equals their tendency Perfectly plastic
0
0 Relative impact velocity
to deform. This condition is one of elastic impact with no energy loss.
The value e = 0, on the other hand, describes inelastic or plastic impact where the particles cling together after
collision and the loss of energy is a maximum. All impact conditions lie some where between these two extremes.
Also it should be noted that a coefficient of restitution must be associated with a pair of contacting bodies.
The coefficient of restitution is frequently considered a constant for given geometries and a given combination
of contacting materials. Actually, it depends on the impact velocity and approaches unity as the impact velocity
approaches zero as shown schematically in figure. A handbook value for e is generally unreliable.

www.physicsashok.in 35
CENTRE OF MASS

HEAD ON ELASTIC COLLISION


A collision is said to be head on (or direct) if the directions of the velocity of colliding objects are along the line
of action of the impulses, acting at the instant of collision.
m1 m2 m1 m2
u1 u2 v1 v2

(a) Before collision (a) After collision

Let the two balls of mass m1 and m2 collide each other elastically with velocities u1 and u2 in the directions
shown in figure (a). Their velocities become 1 and 2 after the collision along the same line. Applying conservation
of linear momentum, we get
m1u1 + m2u2 = m11 + m22 .....(i)
m1(u1 1) = m2(2 u2) .....(ii)
Applying conservation of kinetic energy
1 1 1 1
m1u12 + m 2 u 22 = m112 + m 2 22 .....(iii)
2 2 2 2
m1 (u12 12 ) = m 2 (u 22 22 ) .....(iv)
Solving equations (ii) and (iv), we get
2 = 1 + u1 u2
Substituting for v2 in equation (i), we get

m1 m 2 2m 2
1 = m m u1 + m m u2 .....(v)
1 2 1 2

Similarly,

2m1 m 2 m1
2 = m m u1 + m m u2 .....(vi)
1 2 1 2

Special Cases
1. If the second ball is at rest, i.e., u2 = 0, then

m1 m 2 2m1
1 = m m u1 and 2 = m m u1
1 2 1 2

(a) If m1 = m2, then 1 = 0 and 2 = u1


i.e, if two colliding balls are of equal mass and the second ball is at rest then after collision the balls
exchange their velocities, i.e., first ball comes to rest and second starts moving with the velocity of first.
(b) If m1 > > m2, then
m1 m2 = m1 and m1 + m2 = m1
1 ~ u1 and 2 = 2u1
i.e., when a massive moving ball collides with a light ball at rest then after collision the massive ball
continues to move with the same velocity but light ball starts moving with twice the velocity of massive
ball.

www.physicsashok.in 36
CENTRE OF MASS

(c) If m1 < < m2, then


m1 m2 = m2
and m1 + m2 = m2
v1 = u1
and v2 = 0
i.e., when a light moving ball collides with a massive ball at rest then the light ball rebounds with the same
velocity and massive ball continues to be at rest.
(2) If m1 >> m2 equation (v) and (vi) become v1 = u1 and v2 = 2u1 u2
(3) If m2 << m2 equation (v) and (vi) become v1 = u1 + 2u2 and v2 = u2

C43 : Two particles of masses m and 2m moving in opposite directions collide elastically with velocities v and 2.
Find their velocities after collision.
Sol: Here, u1 = , u2 = 2, m1 = m and m2 = 2m.
Substituting these values in equations (v) and (vi), we get 2m 2v m +ve

m 2m 4m 8
1 = () + (2) or 1 = + = 3
m 2m m 2m 3 3

2m m 2m
and 2 = (2) + ()
m 2m m 2m

2 2
or 2 = v v=0
3 3
i.e., the second particle (of mass 2m) comes to a rest while the first (of mass m) moves with velocity 3 in
the direction shown in figure.

C44 : Two balls M and N each of mass 100 gm are moving in the same direction with velocities 60 m/sec and 40 m/
sec respectively. They collide and after collision they begin to move in the same direction. Calculate their
velocities after collision, assuming the collision to be perfectly elastic.
Sol: 40 m/sec, 60 m/sec

m1 m 2 2m 2
v1 = m m u1 + u
1 2 m1 m 2 2

m 2 m1 2m1
and v2 = m m u2 + u
1 2 m1 m 2 1

C45 : In the above question what will be their velocities after collision if the second ball (ball N) is stationary.
Sol: zero, 60 m/sec.
In this case u2 = 0
Hence, v1 = 0 and v2 = u1 = 60 m/sec.

www.physicsashok.in 37
CENTRE OF MASS
C46 : Two balls M and N, each of mass 10 kg are approaching each other with velocity 40 m/sec and 20 m/sec
respectively. Calculate the velocity after collision, assuming the collision to be perfectly elastic.
Sol: 20 m/sec, and 40 m/sec.
Here the initial direction of ball M is taken as positive.
Since collision is perfectly elastic, i.e., e = 1,
hence, V2 V1 = u1 u2 = 40 (20) = 60 .....(i)
Also, momentum before and after collision is conserved, hence
10 40 10 20 = 10V1 + 10V2
V1 + V2 = 20 .....(ii)
V2 = 40 m/sec and V1 = 20 m/sec.

C47 : Two balls M and N of masses 10 kg and 20 kg respectively are approaching each other with equal speeds of
30 m/sec. Calculate their velocities after collision, assuming the collision to be perfectly elastic.
Sol: 50 m/sec, 10 m/sec.
Since the collision is perfectly elastic, therefore, e = 1. Also taking the initial direction of M to be +ve.
V2 V1 = u1 u2 = 30 (30) = 60
from conservation of momentum,
10 30 20 30 = 10V1 + 20V2
or 300 = 10V1 + 20V2
or V1 + 2V2 = 30
3V2 = 30,
or V2 = 10 m/sec, and V1 = 50 m/sec

C48 : A body of mass 100 kg moving with a velocity of 50 m/sec strikes another body of mass 200 kg which is at
rest. If the two bodies after collision stick together, then calculate the velocity of the composite body.
Sol: Momentum before collision = m1u1 + m2 0 = m1u1
Momentum after collision = (m1 + m2)V
where V is the common velocity
(m1 + m2)V = m1u1
m1u1 100 50
V = (m m ) = = 16.6 m/sec
1 2 300

C49 : Two balls of mass 10 kg and 5 kg are moving with velocities 15 m/sec and 10 m/sec due east and west
respectively. They collide and stick together after collision. Calculate the final velocity of the bodies.
Sol: 6.66 m/sec towards east.
Considering the direction of velocity towards east to be positive, we have
m1u1 m2u2 = (m1 + m2)v
m1u1 m 2 u 2 10 15 5 10
v = m1 m 2 =
10 5
150 50 100
= = = 6.66 m/sec.
15 15
www.physicsashok.in 38
CENTRE OF MASS

C50 : A body of mass 100 kg moving with a velocity of 10 m/sec, strikes an identical body at rest. After collision
they stick together. Calculate the loss in K.E.
Sol: 2500 joule
1 1
K.E. before collision, E1 = m1u12 m 2 u 22
2 2

1
K.E. after collision, E2 = (m1 m 2 )v 2
2
Loss in K.E. = E1 E2

Example 25. A sphere of mass m1 in motion hits directly another sphere of mass m2 at rest and sticks to it, the total
kinetic energy after collision is 2/3 of their total K.E. before collision. Find the ratio of m1 : m2.
Sol. m1v1 = (m1 + m2)v0 ...(i)
1 2 1
2
m1 m 2 v02 m1v12
3 2
m1v12
or m1 m1 v 3
2
0 ...(ii)

After solving eqn. (i) and (ii)


m1
2
m2

Example 26. Three blocks are initially placed as shown in the figure. Block A has v
A B C
mass m and initial velocity v to the right. Block B with mass m and block C with
m m 4m
mass 4m are both initially at rest. Neglect friction. All collisions are elastic. The
final velocity of blocks A is
(A) 0.6v to the left (B) 1.4v to the left (C) v to the left (D) 0.4v to the right
Sol. Since, A and B have same mass. So, after elastic collision, they interchange their velocity.
vB = v
After collision between B and C,
m 4m 2 4m 0 3
vB v v
m 4m m 4m 5
Again, collision takes place between A and B. So, velocity will be inter changed.
3

vA vB
5
v in left ward direction.

Example 27. Two blocks A and B of masses m and 2m respectively A B C


are connected by a spring of force constant k. K v
m 2m m
The masses are moving to the right with uniform velocity v each,
the heavier mass leading the lighter one. The spring in between them is of natural length during the motion.
Block B collides with a third block C of mass m, at rest. The collision being completely inelastic. Calculate the
maximum compression of the spring.

www.physicsashok.in 39
CENTRE OF MASS

Sol. At the velocity of (B + C) just after collision is v0.


2mv = 3mv0
2v
v0
3
At the time of maximum compression, A and (B + C) moves with common velocity.
Applying conservation principle of moment.
2v
mv 3m 4mvC
3
3mv 3
vC v
4m 4
3
vC
v
4
Apply conservation principle of momentum,
2
1 1 2v 1 1
mv 2 3m 4m v 2C kx 20
2 2 3 2 2
After solving,
mv 2
vC
12k

OBLIQUE COLLISION
During collision between two objects a pair of equal and opposite impulses x
y
act at the moment of impact. If just before impact at least one of the
objects was moving in a direction different from the line of action of these
v
impulses the collision is said to be oblique.
In the figure, two balls collide obliquely. During collision impulses act in
the direction xx. Hence forth, we will call this direction as common normal
direction and a direction perpendicular to it (i.e., yy) as common tangent. y
x
Following four points are important regarding on oblique collision.
1. A pair of equal and opposite impulses act along common normal direction. Hence, linear momentum of
individual particles do change along common normal direction. If mass of the colliding particles remain constant
during collision, then we can say that linear velocity of the individual particles change during collision in this
direction.
2. No component of impulse act along common tangent direction. Hence, linear momentum or linear velocity of
individual particles (if mass is constant) remain unchanged along this direction.
3. Net impulse on both the particles is zero during collision. Hence, net momentum of both the particles remain
conserved before and after collision in any direction.

C51 : A ball of mass m is moving with a speed of 20 m/sec. It strikes an identical ball which is at rest. After collision
each ball moves making an angle of 45 with the original line of motion. Calculate their velocities after collision.

www.physicsashok.in 40
CENTRE OF MASS

Sol: 10 2 m/sec for each ball. mv1 sin


m
Momentum along Y-axis before collision = 0 V1
Momentum along Y-axis after collision = mv1 sin45 mv2 sin45
Since, momentum along Y-axis before & after collision is m m = 45 mv1 cos
conserved, hence u1 u2 = 0 = 45 mv2 cos
mv1sin45 mv2 sin45 = 0
v1 = v2
m V1
Again, momentum along X-axis before and after collision shall remain
mv2 sin
conserved, hence
mu1 + mu2 = mu1 cos45 + mv2 cos45
mu1 + 0 = 2mv1 cos45
mu1 u1
v1 = =
2m cos 2 cos

u1 20
v1 = v2 = = = 10 2 m/sec
2 cos 1
2
2
Example 28. A ball of mass m hits a floor with a speed 0 making an angle of incidence with the normal. The
coefficient of restitution is e. Find the speed of the reflected ball and the angle of reflection of the ball.
Sol: The component of velocity 0 along common tangent direction 0 sin will remain unchanged. Let be the
component along common normal direction after collision. Applying
v


v0 sin v0 sin

v0
v0 cos
Relative speed of separation = e(relative speed of approach), along common normal direction,
we get = e0 cos
Thus, after collision components of velocity are 0 sin and e0 cos ev0 cos v


= ( 0 sin )2 (e 2 cos )2
v0 sin
0 sin
and tan = e cos
0

tan
or tan =
e

NOTE : For elastic collision, e = 1


= 0 and =

www.physicsashok.in 41
CENTRE OF MASS

C52 : Important Features


1. During collision if mass of one body is very much greater than the mass of the other body then the velocity of
heavy body remains almost unchanged after collision, whether the collision is elastic or inelastic.
2. In the situation shown in figure e is the coefficient of restitution between the u=0
ball and the ground, then after nth collision with the floor the speed of ball will
h
remain en0 and it will go upto a height e2nh or, n = en0 = en 2gh and
hn = e2nh. v0 = 2gh

3. Coefficient of restitution is the mutual in strinsic property of two bodies. Its value varies from 0 to 1.

Example 29. A ball of mass m = 1 kg falling vertically with a velocity v0 = 2 m/s strikes a
wedge of mass M = 2kg kept on a smooth, horizontal surface as shown in figure. The m
M
coefficient of restitution between the ball and the wedge is e = 1/2. 30
Find the velocity of the wedge and the ball immediately after collision.
Sol. Fdt cos 60 = Mv1
y
= 2v1 vy Fdt sin60
Fdt = 4v1 ...(i) v0 Fdt
Fdt cos 60 = mvx vx
60
x
2v1 = vx ...(ii) v1
30
Fdt cos60
30
and Fdt sin 60 = + mvy (mv0) = vy + 2 60 30
3 Fdt cos60 60
or Fdt vy 2
2 Fdt

2 3 v1 2 v y ...(iii)

e
1


v x cos 60 v y cos 30 v1 cos 60
...(iv)
2 v0 cos 30 0
After solving equation (i), (ii), (iii) and (iv)
1
v1 m/s
3
2
and v 2 v 2x v 2y m/s
3

Example 30. In the figure shown, the two identical balls of mass M and radius R each, are
placed in contact with each other on the frictionless horizontal surface. The third ball of
mass M and radius R/2, is coming down vertically and has a velocity = v0 when it
simultaneously hits the two balls and itself comes to rest. Then, each of the two bigger
balls will move after collision with a speed equal to
(A) 4v0/ 5 (B) 2v0/ 5 (C) v0/ 5 (D) None
Sol. Fdt cos = Mv ...(i)
Fdt sin = 0 Mv0
Fdt sin = Mv0 ...(2)
From equation (i) and (ii)
www.physicsashok.in 42
CENTRE OF MASS

Fdt 2Fdt sin


v
tan 0 ...(iii) 90 Fdt
v
From AOC
C +
5 R/2 3R/2

90
tan 5
R /2 Fdt cos
v A R R B
5 0
v
Fdt
v0 Fdt
v
5

Example 31. In the above, suppose that the smaller ball does not stop after collision, but continues to move
downwards with a speed = v0/2, after the collision. Then, the speed of each bigger ball after collision is
(A) 4v0/ 5 (B) 2v0/ 5 (C) v0/ 5 (D) None
Sol. Fdt cos = Mv ...(i)
Mv0
Fdt sin = Mv0
2
Mv0
Fdt sin = ...(ii)
2
From equation (i) and (ii)
v0
tan
2v
tan 5
v0
5 Hence (C) option is correct.
2 5

VARIABLE MASS
In our discussion of the conservation of linear momentum, we have so far dealt with systems whose mass
remains constant. We now consider those systems whose mass is variable, i.e., those in which mass enters or
leaves the system. A typical case is that of the rocket from which hot gases keep on escaping, thereby continuously
decreasing its mass.

In such problems you have nothing to do but apply a thrust force ( Ft ) to the main mass in addition to the all
other forces acting on it. This thrust force is given by,
vr
dm
Ft = v rel v dm
v + dv
dt
m m dm
system

www.physicsashok.in 43
CENTRE OF MASS


Here, v rel is the velocity of the mass gained or mass ejected relative to the main mass. In case of rocket this is
dm
sometimes called the exhaust velocity of the gases. is the rate at which mass is increasing or decreasing.
dt
The expression for the thrust force can be derived from the conservation of linear momentum in the absence of
any external forces on a system as follows:

Suppose at some moment t = t mass of a body is m and its velocity is v . After some time at t = t + dt its mass


becomes (m dm) and velocity becomes v + dv . The mass dm is ejected with relative velocity v r . Absolute


velocity of mass dm is there fore ( v r + v + dv ). If no external forces are acting on the system, the linear
momentum of the system will remain conserved, or

Pi = Pf


or m v = m(m dm)( v + dv ) + dm( v r + v + dv )


or m v = m v + md v dm v (dm)( dv ) + dm v + v r dm + (dm)( dv )


m dv = v r dm

dv
dm
m dt = v r

or
dt

dv
m dt = thrust force ( F1 )

Here,

dm
and = rate at which mass is ejecting.
dt
Problems related to variable mass can be solved in following three steps.
1. Make a list of all the forces acting on the main mass and apply them on it.
dm
2. Apply an additional thrust force Ft on the mass, the magnitude of which v r and direction is given by
dt

the direction of v r in case the mass is increasing and otherwise the direction of v r if its is decreasing.
3. Find net force on the mass and apply

dv
Fnet =m [m = mass at that particular instant]
dt u v

At t = 0 At t = t
Rocket Propulsion v=u m=m
m = m0 v =v
Let m0 be the mass of the rocket at time t = 0. m its mass at
any time t and its velocity at that moment. Initially let us
suppose that the velocity of the rocket is u.
Exhaust velocity = vr

www.physicsashok.in 44
CENTRE OF MASS

dm
Further, let be the mass of the gas ejected per unit time and r the exhaust velocity of the gases.
dt
dm
Usually and vr are kept constant throughout the journey of the rocket. Now, let us write few equations
dt
which can be used in the problems of rocket propulsion. At time t = t,

dm
1. Thrust force on the rocket Ft = r [upwards]
dt
2 Weight of the rocket W = mg [downwards]
3. Net force on the rocket Fnet = Ft W [upwards]

dm
or Fnet = r mg
dt

F
4. Net acceleration of the rocket a=
m

d r dm
or = g
dt m dt

dm
or d = vr gdt
m
m t
dm
or d = vr m g dt
u m 0

m0
or u = r ln gt
m

m0
Thus, = u gt + r n .....(i)
m

dm dm
NOTE:1. F r = r is upwards, as vr is downwards and is negative.
dt dt

m0
2. If gravity is ignored and initial velocity of the rocket u = 0, equation (i) reduces to = r n .
m
Example 32. (a) A rocket set for vertical firing weighs 50 kg and contains 450 kg of fuel. It can have a maximum
exhaust velocity of 2 km/s. What should be its minimum rate of fuel consumption
(i) to just lift it off the launching pad ?
(ii) to give it an acceleration of 20 m/s2 ?
(b) What will be the speed of the rocket when the rate of consumption of fuel is 10 kg/s after whole of the
fuel is consumed ? (Take g = 9.8 m/s2)

www.physicsashok.in 45
CENTRE OF MASS

Sol: (a) (i) To just lift it off the launching pad


weight = thrust force

dm
or mg = vr
dt

dm mg
or = r
dt
Substituting the values, we get

dm (450 50)(9.8)
=
dt 2 103
= 2.45 kg/s
(ii) Net acceleration a = 20 m/s2
ma = Ft mg
Ft
or a = g
m

r dm
or a = g
m dt

dm m(g a)
This gives = r
dt
Substituting the values, we get

dm (450 50)(9.8 20)


=
dt 2 103
(b) The rate of fuel consumption is 10 kg/s. So, the time for the consumption of entire fuel is
450
t = = 45 s
10

m0
Using Equation (i), i.e., v = u gt + r n
m
Here, u = 0, r = 2 103 m/s, m0 = 500 kg and m = 50 kg
Substituting the values, we get

500
= 0 (9.8)(45) + (2 103)n
50
or = 441 + 4605.17
or = 4164.17 m/s
or = 4.164 km/s

www.physicsashok.in 46
CENTRE OF MASS
Example 33. A rocket of mass 4000 kg is set for vertical firing. How much gas must be ejected per second so that
the rocket may have initial upwards acceleration of magnitude 19.6 m/s2. [Exhaust speed of fuel = 980 m/s.]
(A) 240 kg s1 (B) 60 kg s1 (C) 120 kg s1 (D) None
Sol. FT mg = ma
dm
or v rel mg ma
dt FT

dm
or 980 4000 9.8 4000 19.6
dt
mg
dm 4000 9.8 3
120 kg / s
dt 980

THINKING PROBLEMS

1. If a body is not in translational equilibrium, will the torque about any point be zero if the torque about
some particular point is zero ?

2. Why does a porter bend forward while carrying a sack of rice on his back ?

3. Do the cm and the cg coincide for a building ? For a lake ? Under what conditions does the difference
between these two points become insignificant ?

4. Is it necessary for there to be any mass at the cm of a system ?

5. Does the cm of a solid necessarily lie within the body ? If not, give an example.

6. If only an external force can change the state of motion of the cm of a body, how does the internal force
of the brakes bring a car to rest ?

7. The bottom of a ship is made heavy. Wha ?

8. A rectangular block slides down a rough inclined plane with uniform velocity. Will the line of action of the
normal reaction of the plane pass through the cg of the block ?

9. Why do we take small steps while walking on a slippery road ? Explain ?

10. Two identical weights are connected by a spring. At the initial moment, the
spring is so compressed that the first is tightly pressed against a wall (see figure)
and the second weight is clamped by a stop S. How will the weights move if
the stop is removed ? Explain without calculations.

11. Some heavy boxes are to be loaded along with some empty boxes on a cart. Which boxes should be put
on the cart first and why ?

12. Standing is prohibited in a double decker bus. Explain.

www.physicsashok.in 47
CENTRE OF MASS
THINKING PROBLEMS SOLUTION
1. No. When a body is not in translational equilibrium, there is some resultant force on the body. The
moment of the forces about a point on the line of action of the resultant is zero but the moment about any
point off the line of action of the resultant cannot be zero.
2. To attain stability of equilibrium so that the vertical line through the cg of the system (porter + sack of
rice) passes through the base of the system, that is, the space between the feet of the porter.
3. Yes. No. When the size of the body is small in comparison to the radius of the earth.
4. No. For example, a ring. 5. No. An L-shaped rod.
6. In fact the internal force does not bring the car to rest. The internal force of the brakes on the wheel locks
the wheel. Now an enormous frictional force arises between the wheel and the ground. The force is
external to the system and this external force brings the car to rest.
7. The bottom is made heavy so that the cg of the ship is low and stability of equilibrium is ensured.
8. No. The three forces on the block are (a) its weight W, (b) the force of friction F, and (c) the normal
reaction N. Since W passes through the cg while F does not, N cannot pass through the cg. This is
because the torque due to W about the cg is zero. Hence, the torques due to F and N must balance. For
N to have a non-zero torque about the cg, it should not pass thorugh the cg.
9. When a man walks, the line joining his cg C to the toe O of his foot is inclined to
the vertical at a certain angle a. The value of this angle depends on the size of the
step the man takes. The smaller the step, the less the value of a. Let F be the
reaction of the ground on the toe of the man directed towards the centre.
The horizontal component of this force cannot be greater than the maximum value
of frictional force, otherwise he will slip. Therefore, for equilibrium.
F sin f friction and F cos mg
But ffriction f lim
Fsin F cos
or tan tan 1
When is small, a must also be small. This is why small steps are taken while walking on a slippery road.
10. When the stop is removed, the weight 2 will move to the right with acceleration due to the action of the
spring to the right and the weight 1 will remain pressed against the wall. When the spring regains its
normal length, the second weight will gain some velocity. The velocity of the cm at this instant is
m1 0 m 2 v 2 m2 v2
v cm
m1 m 2 m1 m 3
Till the spring regains its original length, the system is not free from external force along the horizontal as
the wall exerts force on 1. When the spring regains its original length, the system becomes free from
external force and so, the velocity of the cm of the system remains fixed at vcm, but 1 and 2 are respectively
accelerated and retarded. So they come closer with such velocities that the velocity of the cm remains
constant. Again they move away from each other till they are farthest from each other. Thus, the system as
a whole will move away from the wall with a constant velocity, but the weights will alternately converge
toward each other and diverge from each other.
11. The heavy boxes should be loaded first so that the cg of the loaded cart may remain at the lowest
position.
12. Standing passengers in the upper deck will raise the cg of the bus and that will make it less stable.

www.physicsashok.in 48
CENTRE OF MASS

Assertion-Reason Type
A statement of Statement-1 is given and a Corresponding statement of Statement-2 is given just below it of
the statements, mark the correct answer as
(A) If both Statement-1 and Statement-2 are true and Statement-2 is the correct explanation of Statement-1.
(B) If both Statement-1 and Statement-2 are true and Statement-2 is NOT correct explanation of Statement-1.
(C) If Statement-1 is true but Statement-2 is false.
(D) If both Statement-1 and Statement-2 are false.
(E) If Statement-1 is false but Statement-2 is true.

1. Statement 1 : Let us consider a case, in which we know the details of initial motion of system, then it is
possible to know description of final motion, without knowing the details of forces which are changing the
motion of system.

dp
Statement 2 : Fext =
dt

2. Statement 1 : Law of conservation of momentum holds good only in a particlular frame of reference.
Statement 2 : Momentum of a body depends on the frame of reference.

3. Statement 1 : Internal forces of the brake can bring the centre of mass of a moving car to rest.

Statement 2 : If Fext = 0, then centre of mass keeps on moving with constant velocity..

4. Statement 1 : A quick collision between two bodies is more violent than slow collision, even when initial
and final velocities are identical.
Statement 2 : The rate of change of momentum determines that force is small or large.

5. Statement 1 : In case of bullet fired from gun, the ratio of kinetic energy of gun and bullet is equal to ratio
of mass of bullet and gun.
Statement 2 : In firing, momentum is conserved.

6. Statement 1 : In an elastic collision between two bodies, the relative speed of the bodies after collision
is equal to the relative speed before the collision.
Statement 2 : In an elastic collision, the linear momentum of the system is conserved.

Match the Column


B
1. A particle of mass m is pulled along an arbitrary curved surface without
acceleration as shown in figure. coefficient of friction between surface
and particle is . Then:
Column I Column II
m h
(A) Work done by the frictional force (P) mgh
(B) Work done by the gravitational force (Q) mgl
(C) Work done by the external force (R) mgl mgh A
(D) Change in kinetic energy of the particle (S) Zero l

www.physicsashok.in 49
CENTRE OF MASS
2. A body of mass m 1 moving with certain velocity collides elastically with another body of mass m 2 at rest.
The percentage kinetic energy transferred by m 1 to m 2 in the following cases:
Column I Column II
(A) m2 m1 (P) 64 %
(B) m2 4m1 (Q) 36 %
(C) m2 3m1 (R) 100 %
(D) m2 9m1 (S) 75 %

3. The potential energy for a conservative system is given by U = ax 2 bx


Column I Column II
(A) The net force acting on the system (P) b/2a
(B) The equilibrium position (Q) b 2ax
(C) The potential energy at the equilibrium (R) b2/2a
(D) The equilibrium. (S) stable

4. Let h0 be the initial height of ball with respect to the earth. The coefficient of restitution is e.
Column I Column II
2n
(A) Total distance travelled by the ball (P) e h0
before coming to rest.
1 e2
(B) Height attained after n impacts h
(Q) 0 2
1 e
1 e
(C) Average force exerted by ball (R) P
1 e
(D) Total momentum transferred to the earth (S) mg

5. A body of mass m 1 elastically collide with another stationary body of mass m 2.


Column I Column II
(A) After the collision, velocity of second (P) > 1
body is maximum when m1 m2 is
(B) After the collision, the momentum of (Q) < 1
second body is maximum when m1 m2 is
(C) After the collision, K.E. of second body (R) = 1
is maximum when m1 m2 is
(D) For elastic collision, coefficient of restitution is (S) 0

1. A ball kept in a closed box moves in the box making collisions with the walls. the box is kept on a smooth
surface. The centre of mass:
(A) of the box remains constant (B) of the box plus the ball system remains constant
(C) of the ball remains constant (D) of the ball relative to the box remains constant

2. In the given figure, two bodies of masses m 1 and m 2 are connected by


massless spring of force constant k and are placed on a smooth
surface (shown in figure), then: k
(A) the acceleration of centre of mass must be zero at every instant F m1 m2 F
(B) the acceleration of centre of mass may be zero at every instant
(C) the system always remains in rest (D) None of the above

www.physicsashok.in 50
CENTRE OF MASS
m2
3. In the given figure the mass m 2 starts with velocity v 0 and moves with constant velocity
on the surface. During motion the normal reaction between the horizontal surface and m1
fixed triangular block m 1 is N. Then during motion:
(A) N = (m 1 + m 2) g (B) N = m 1 g (C) N < (m 1 + m 2) g (D) N > (m 1 + m 2) g A

4. Two blocks of mass m 1 and m 2 are connected by light inextensible string passing over a smooth fixed pulley
of negligible mass. The acceleration of the centre of mass of the system when blocks move under gravity is
2 2
m2 m1 m1 m2 m2 m1 m1 m2
(A) g (B) g (C) g (D) g
m1 m2 m2 m1 m1 m2 m2 m1

5. A block of mass M is tied to one end of a massless rope. The other end of the rope
is in the hands of a man of mass 2M as shown in the figure. the block and the man
are resting on a rough wedge of mass M as shown in the figure. The whole system
M
is resting on a smooth horizontal surface. The man pulls the rope. Pulley is massless
2m
and frictionless. What is the displacement of the wedge when the block meets the M
pulley. (Man does not leave his position during the pull)
(A) 0.5 m (B) 1 m (C) Zero (D) 2 3m

6. The figure shows the positions and velocities of two particles. If the particles v1= 5 m/s v2= 3 m/s
move under the mutual attraction of each other, then the position of centre of 1 kg 1 kg
mass at t = 1 s is
(A) x = 5 m (B) x = 6 m (C) x = 3 m (D) x = 2 m x=0 x = 10 m

7. A block of mass m moving with a velocity v 0 collides with a stationary v0


block of mass M at the back of which a spring f stiffness k is attached, k
as shown in the figure. Choose the correct alternative(s) Smooth m M
(A) The velocity of the centre of mass is v0 2
1 mM 2
(B) The initial kinetic energy of the system in the centre of mass frame is v0 .
4 M m
mM 1
(C) The maximum compression in the spring is v0
m M k .
(D) When the spring is in the state of maximum compression, the kinetic energy in the centre of mass frame
is zero.

8. A hemisphere and a solid cone have a common base. The centre of mass of the common structure coincides
with the centre of the common base. If R is the radius of hemisphere and h is height of the cone, then
h h 1 h h 1
(A) 3 (B) (C) 3 (D)
R R 3 R R 3

9. Masses of 1 g, 2 g, 3 g, ........ 100 g are suspended from the 1 cm, 2 cm, 3 cm ......... 100 cm marks of a light
metre scale. The system will be supported in equilibrium at
(A) 60 cm (B) 67 cm (C) 55 cm (D) 72 cm

10. A drum majors baton consists of two masses 2 M and M separated by a thin light rod of length L. The baton

is thrown into the air. Neglecting air drag, if R is the position vector of the centre of mass of the baton then
the equation of motion for the centre of mass is

d 2R 2g d 2R g d 2R d 2R 4g
(A) (B) (C) g (D)
dt 2 3 dt 2 3 dt 2 dt 2 3

www.physicsashok.in 51
CENTRE OF MASS
11. A rectangular block of length and breadth b is held with one corner resting on a frictionless table and is gently
released, thus falling in a complex tumbling motion. The trajectory of the centre of mass of the block is
b
(A) a circuit track of radius or depending on the manner it is released (or placed before releasing).
2 2
(B) a straight line. (C) an ellipse.
(D) unknown, as data provided is insufficient to arrive at any conclusion.

12. A cannon and a supply of cannon balls are inside a sealed rail road car at rest. The cannon fires to the right,
the car recoils to the left. The balls remain in the car after hitting the farther wall. Assume friction to be
absent. If L is the length of the rail road car, D is the distance traversed by the rail road car relative to the
ground, m is the total mass of cannon balls fired and M is the total mass of remaining system (cannon + car)
then
ML mL
(A) D > L (B) D < L (C) D (D) D
M m M m
13. Four point masses A, B, C and D are placed at points which are coplanar but non-collinear. If P is the centre
of mass of the system comprising A, B, C and D, then
(A) P must lie within the quadrilateral ABCD.
(B) P must lie on a line joining any two of the points A, B, C and D.
(C) P may or may not coincide with one of the point masses.
(D) P lies inside or on the edge of at least one of the triangles formed by taking any three of the points from
A, B, C and D.
L
14. The figure shows a small prism of mass M sliding on the bigger prism of mass
M
5M from the top to the bottom. Assuming all surfaces to be frictionless and the
system (big prism + small prism) to be at rest initially, the distance moved by the 5M
prism combination is

3 3 2 2 5L
(A) L to the left (B) L to right (C) L to right (D) L to left
2 2 3 3

15. A man of mass m, the light rope and the balloon of mass M are at rest as shown.
The man starts climbing the rope and reaches the top. Assume the crate in which
the man is standing to be light.
mL
(A) The balloon descends by a distance .
M m
mL
(B) The balloon ascends by a distance
M m
ML ML
(C) The balloon descends by a distance (D) The balloon ascends by a distance
M m M m
16. A large tray of mass M has in it a cubical block of ice of mass m and edge length L. The ice starts melting.
The shift in the height of centre of mass of system (ice + tray) is
1 mL 1 ML 1 mL 1 ML
(A) 2 m M ; upward (B) 2 m M ; upward (C) 2 m M ; downward (D) 2 m M ; downward

17. An ideal spring is connected between two blocks of mass M and m.
Blocks-spring system can move over a smooth horizontal table along a m M
straight line along the length of the spring as shown in figure. The blocks
are brought nearer to compress the spring and then released. In the
subsequent motion:
(A) Initially they move in opposite directions with velocities inversely proportional to their masses
(B) the ratio of their velocities remains constants.
(C) Linear momentum and energy of the system remains conserved.
(D) The two blocks will oscillate about their centre of mass, which remains stationary.

www.physicsashok.in 52
CENTRE OF MASS
18. All the particles of a body are situated at a distance R from the origin. The distance of the centre of mass of
the body from the origin is:
(A) = R (B) R (C) > R (D) R.

19. Select the correct alternative(s) :


(A) It is possible to have a collision in which whole the kinetic energy is lost.
(B) Only external forces can cause the centre of mass of a system to accelerate.
(C) Centre of mass of a solid body necessarily lie within the body.
(D) A ball is dropped form a height h to the ground. If the coefficient of restitution is e, the height to which ball
goes up after it rebounds second time is e2 h.

20. Which of the following are not correct about centre of mass?
(A) It depends on frame of reference. (B) In centre of mass frame, momentum of a system is always zero.
(C) Internal forces may affect the motion of centre of mass.
(D) Centre of mass and centre of gravity are synonymous.

21. A spring is compressed between two blocks of masses m 1


and m 2 placed on a horizontal frictionless surface as shown in
figure. When the blocks are released, they have initial velocity
of v 1 and v 2 as shown in figure. The blocks travel distances x 1 and
x 2 respectively before coming to rest. The ratio x 1 / x 2 is
m1 m2 m1 m2
(A) (B) (C) (D)
m2 m1 m2 m1

22. A spherical ball falls from a height h on a floor of coefficient of restitution e. Find the total distance covered
by the ball, before coming to rest.
(A) h (1 + e2)/(1e2) (B) h (1 e2)/(1+ e2) (C) h (1 + e)/(1e) (D) h (1 e)/(1+e)

23. A ball of mass m approaches a moving wall of infinite mass with speed v along the normal to the wall.The
speed of the wall is u towards the ball. The speed of the ball after an elastic collision with the wall is
(A) u + v away from the wall (B) 2u + v away from the wall
(C) u - v away from the wall (D) v- 2u away from the wall

24. Two particle of mass m 1 and m 2 in projectile motion have velocities V 1 and V 2 respectively at time t = 0.
They collide at time t 0 their velocities become V1 and V2 and time 2t 0 while still moving in air . The value of

| (m1v 1 m 2 v 2 ) (m1v 1 m 2 v 2 ) | is
1
(A) 0 (B) (m 1 + m 2) gt 0 (C) 2(m 1 + m 2) gt 0 (D) (m 1 + m 2) gt 0
2

25. A particle strikes a horizontal smooth floor with a velocity u making an angle with the floor and rebounds
with velocity making an angle with the floor. The coefficient of resitution between the particle and the
floor is e.
(A) The impulse delivered by the floor to the body is mu(1-e) sin (B) tan = tan ( /e)
(C) v = u 1 (1 e 2 ) sin 2 (D) none of these

26. A man of mass 50 kg. is standing on a platform of mass 100 kg. which is kept on a smooth ice surface. If
the man starts moving on the platform with a speed 15m/sec relative to the platform, then calculate with
what velocity relative to the ice the platform will recoil?
(A) 5 m/sec (B) 10 m/sec. (C) 15 m/sec (D) none of these

27. Two balls shown in figure are identicial, the first moving with a speed v towards
right and second staying at rest. The wall at the extreme right is fixed and smooth.
Assuming all collision to be elastic, which of the following statements are correct
(A) the speed of the first ball is reduced to zero finally after all collision
(B) only two collision are possible
(C) the speed of the balls remain unchanged after all collisions have taken place
(D) none of these
www.physicsashok.in 53
CENTRE OF MASS
28. A simple pendulum is suspended from a peg on a vertical wall pendulum is pulled L

away from the wall to a horizontal position and released. The ball hits the wall, the
coefficient of resitution being 2 . What is the aminimum number of collisions after
5
which the amplitude of ossicillation becomes less than 600 ?
(A) 2 (B) 3 (C) 4 (D) none of these

29. A shell of mass m is at rest initially. It explodes into three fragments having masses in the ratio 2 : 2 : 1. The
fragments having equal masses fly off along mutually perpendicular directions with a speed v. What will the
speed of the third (lighter) fragment ?
(A) v (B) 2 v (C) 2 2 v (D) 3 2 v

30. The bob A of a pendulum released from a height h hits head-on another bob B
of the same mass of an identical pendulum initially at rest. What is the result of
this collision? Assume the collision to be elastic
(A) Bob A comes to rest at B and bob B moves to the left attaining a maximum height h A

h h
(B) Bobs A and B both moves to the left, each attaining a maximum height
2 B
h
(C) Bob B moves to the left and bob A moves to the right , each attaining a maximum height .
2
(D) Both bobs come to rest

31. A ball of mass m moving with a veloctity v undergoes an oblique elastic collision with another ball of the
same mass m but at rest. After the collision, if the two balls move with the same speeds, the angle between
their directions of motion will be
(A) 300 (B) 600 (C) 900 (D) 1200

32. A boy throws a tennis ball vertically downwards. He wants the ball to rebound from the floor and just touch
the ceiling of the room which is at the height of 4 m. If the coefficient of restitution is 0.8, with what velocity
should the ball strike the floor ?
(A) 13.4 m/s (B) 11.2 m/s (C) 10.3 m/s (D) none of these

33. An empty box is put on the pan of a physical balance and the scale is adjusted to zero by counterpoising.
A stream of small identical beads each of mass m are then dropped into the box from a height h at a
constant rate of n beads per second. If the collision between the beads and the box is completely inelastic,
find the reading of the scale, t second after the beads begin to fill the box :
2h nmt 2 h
(A) nm t (B) (C) nmg t (D) nmght
g gh g

34. A pendulum consists of a wooden bob of mass m and length . A bullet of mass m 1 is fired towards the
v1
pendulum with a velocity v 1. The bullet comes out of the bob with speed and the bob just completes
3
motion along a vertical circle. The velocity v 1 is :
3 m m 3 m 3 m1
(A) 2 m 5g (B) 5g (C) g (D) 5g
1 m1 2 m1 2 m

35. A body of mass M at rest explodes into three pieces, two of which of mass (M/4) each are thrown off in
perpendicular directions with velocities of 3 m/s and 4m/s respectively. The third piece will be thrown off with
a velocity of :
(A) 1.5 m/s (B) 2 m/s (C) 2.5 m/s (D) 3 m/s

36. An isolated particle of mass m is moving in a horizontal plane (x y) along the x-axis at a certain height above
the ground. It suddenly explodes into two fragments of masses m/4 and 3m/4. An instant later the smaller
fragment is at y = + 15 cm. The larger fragment at this instant is at [JEE, 97]
(A) y = 5cm (B) y = + 20 cm (C) y = + 5 cm (D) y = 20 cm

www.physicsashok.in 54
CENTRE OF MASS
37. A set of n-identical cubical blocks lie at rest parallel to each other along a line on a smooth horizontal surface.
The separation between the near surfaces of any two adjacent blocks is L. The block at one end is given a
speed V towards the next one at time t = 0. All collisions are completely inelastic, then
(A) the last block starts moving at t = n(n 1)L/(2v)
(B) the last block starts moving at t = (n 1)L/v
(C) the centre of mass of the system will have a final speed v/n
(D) the centre of mass of the system will have a final speed v. [JEE, 95]


38. Two particles of masses m1 and m2 in projectile motion have velocities v1 and v 2 respectively at time t = 0.

They collide at time t0. Their velocities become v1 and v2 at time 2t0 while still moving in air. The value of

m v 1 m v 2 m v 1 m v 2

1 2 1 2
is [JEE(Scr), 01]

(A) zero (B) (m1 + m2)gt0 (C) 2(m1 + m2)gt0 (D) (m1 + m2)gt0

39. Two block of masses 10 kg and 4 kg are connected by a spring of negligible mass and placed on a frictionless
horizontal surface. An impulse gives a velocity of 14 m/s to the heavier block in the direction of the lighter block
The velocity of the centre of mass is : [JEE (Scr), 02]
(A) 30 m/s (B) 20 m/s (C) 10 m/s (D) 5 m/s

40. Look at the drawing given in the figure which has been drawn with ink of uniform line-
y
thickness. The mass of ink used to draw each of the two inner circles, and each of the
two line segments is m. the mass of the ink used to draw the outer circle is 6m. The
coordinates of the centres of the different parts are : outer circle (0, 0), left
x
inner circle (a, a), right inner circle (a, a), vertical line (0, 0) and horizontal line (a, a).
The y-coordinate of the centre of mass of the ink in this drawing is [JEE, 09]
a a a a
(A) (B) (C) (D)
10 8 12 3

41. A disk of radius a/4 having a uniformly distributed charge 6C is placed y


in the x y plane with its centre at (a/2, 0, 0). A rod of length a
carrying a uniformly distributed charge 8C is placed on the x-axis x
from x = a/4 to x = 5a/4. Two point charges 7C and 3C are
placed at (a/4, a/4, 0) and (3a/4, 3a/4, 0), respectively. Consider
a cubical surface formed by six surfaces x = a/2, y = a/2, z = a/2. The electric flux through this cubical
surface is [JEE, 09]
2C 2C 10C 12C
(A) (B) (C) (D)
0 0 0 0

www.physicsashok.in 55
CENTRE OF MASS

Passage Type Problems

PASSAGE 1
In a game of billiards ball A is given an initial velocity V0 of magnitude V0 = 3 m/s along line DA parallel to the
axis of the table. It hits ball B and then ball C, which are both at rest knowing that A and C hit the sides of the
table squarely at points A and C, respectively that B hits the side obliquely at B, and assuming frictionless
surfaces and perfectly elastic impacts. The motion of billiard balls are are assumed to be particle moving
freely in a horizontal plane rather than the rolling and sliding spheres they actually are.

1. Mark Correct option:


(A) The momentum of balls system remains conserved before hitting the wall of table.
(B) The angular momentum of balls system about point O remains conserved before striking with the walls of
table.
(C) Mechanical energy of system will remains constant.
(D) All the above.

2. VA , VB and VC represent the velocities of balls A and B with the balls hit the sides of the table. Then
(A) VA VB (B) VA VBy (C) 3 VB VC (D) VA 3 VC

3. Which of following is correct relation:


(A) VBx 3 VC (B) VA 3 VC (C) VA 3 VB (D) VA VB

4. Which of following equation is correct?


(A) 20VC2 78 VB 72 0 (B) VC2 VB2 72 0

(C) 20VC2 78 VC 72 0 (D) 20 VC 78 VB 72 VA 0

PASSAGE 2
Suppose a body of mass m 0 is placed on a smooth horizontal surface at rest. The mass of the body is
decreasing exponentially with disintegration constant .
Assuming that the mass is ejected backwards with a relative velocity 0 .
5. The mass of body at an instant t is

(A) m0 e t
(B) m0 1 e
t
(C) m0 1 t (D) None

6. The thrust force on the body is


(A) m0 0 (B) 0 (C) m0 0 e t (D) Zero

7. The acceleration of the body is


0 0
(A) 0 (B) (C) (D) Zero
t2

8. The velocity - time graph is

v v v v
(A) (B) (C) (D)
t t t t

www.physicsashok.in 56
CENTRE OF MASS

PASSAGE 3
A multistage rocket consists of two stages. The first stage has a total mass of 12000 kg, of which 8000 kg is
fuel. The total mass of second stage is 1000 kg, of which 600 kg is fuel. Assume that the relative speed V0 of
ejected material is constant and neglect any effect of gravity.
(The latter effect is small during the firing period if the rate of fuel consumption is large).
9. Suppose the entire fuel supply carried by the two stage rocket is utilized in a single stage rocket of the same
total mass of 13000 kg. In terms of V0, what is the speed of the rocket, starting from rest, when its fuel is
exhausted?
(A) 0.956 V0 (B) 1.87 V0 (C) 1.08 V0 (D) 3.74 V0

10. For the two stage rocket, what is the sped when the fuel of the first stage is exhausted if the first stage carries
the second stage with it to this point?
(A) 0.956 V0 (B) 9.56 V0 (C) 3.74 km/s (D) 1.08 V0

11. What is final speed of the second stage?


(A) 1.08 V0 (B) 1.87 V0 (C) 9.56 V0 (D) 3.74 km/s.

12. What value of V0 is required to give the second stage of the above rocket a speed of 7 km/s.
(A) 1080 km/s (B) 14 km/s (C) 374 km/s (D) 3.74 km/s

PASSAGE 4
A satellite travels through interstellar space. In do so, it picks up interstellar dust and its mass increases at
dm
a rate AV where A is satellite surface area, V its velocity and the (constant) dust density. At
dt
t = 0, the velocity is V0 and the mass is m 0. Assume that there are no external forces.

13. The acceleration of satellite as function of velocity is


A 2 A 2 AV
(A) V (B) V (C) g (D)
m m m

14. The velocity of satellite is


1
1 At m m
(A) (B) V0 (C) V0 (D) None.
V0 m At At

Level 2

1. Find the centre of mass of the system shown in figure.

2. A uniform wire is bent into the form of a rectangle with length L and
width W. If two of the sides coincide with the +x and +y axes, where
is its center of mass ?

www.physicsashok.in 57
CENTRE OF MASS

3. The uniform solid sphere shown in the figure has a spherical hole in it.
Find the position of its center of mass.

4. A rigid body consists of a 3-kg mass connected to a 2-kg mass by a massless rod. The 3-kg mass is
located at r1 = 2i + 5j m, and the 2-kg mass at r2 = 4i + 2j m. Find the length of the rod the coordinates of the
center of mass.

5. A straight rod of length L has one of its ends at the origin and the other at x = L. If the mass per unit length
of the rod is given by Ax where A is a constant, where is its center of mass ?

6. Consider the system composed of a 1-kg body and a 2-kg initially at rest at a center-to-center distance of
1m. All numerical values quoted in this exercise are to be considered exact, and the 2-kg body is to the right
of the 1-kg body. (A) How far is the systems center of mass from the center of the 1-kg body ? (B)
Beginning at t = 0s, a net rightward force of 2 N acts on the 2-kg body. What is its resultant acceleration ?
(C) How far does the 2-kg body move between t = 0s and t = 1s ? (D) How far is the center of mass from the
1-kg bosy at t = 1s ? (e) How far did the center of mass move between t = 0s and t = 1s ? (f) What is the
acceleration of the center of mass, beginning at t = 0s? (g) Suppose that all the mass in both objects were
concentrated at the center of mass, and the net rightward force of 2 N acted on this concentrated mass.
What would its acceleration be ? (h) State the general theorem which is illustrated by the results of parts (f)
and (g) . m
u m v
7. A particle of mass m moving with a speed u strikes a smooth horizontal surface

at an angle . The particle rebounds at an angle with a speed v. Determine an
expression for v and if the coefficient of restitution is e. Smooth Surface
m v0
8. In the figure shown, a ball of mass m collides perpendicularly on a smooth stationary
wedge of mass M. If the coefficient of restitution of collision is e (< 1) then determine the
velocity of the wedge after collision.

9. A projectile is fired from a gun at an angle of 450 with the horizontal and with a muzzle speed of 1500 ft/sec.
At the highest point in its flight the projectile explodes into two fragments of equal mass. One fragment,
whose initial speed is zero, falls vertically. How far from the gun does the other fragment land, assuming a
level terrain ?

10. A machine gun fires 50-gm bullets at a speed of 1000 meters/sec. The gunner, holding the machine gun in
his hands, can exert an average force of 180 nt against the gun. Determine the maximum number of bullets
he can fire per minute ?

11. A block of mass m 1 = 100 kg is at rest on a very long frictionless


table, one end of which is terminated in a wall. Another block of
mass m 2 is placed between the first block and the wall and set
in motion to the left with constant speed v 2. as in fig. Assuming
that all collisions are completely elastic, find the value of m 2
for which both blocks move with the same velocity after m 2 has
collided once with m 1 and once with the wall. The wall has infinite mass effectively.

12. A bullet of mass 10 gm strikes a ballistic pendulum of mass 2.0 kg. The center of mass of the pendulum
rises a vertical distance 12 cm. Assuming that bullet remains embedded in the pendulum, calculate the
initial speed.

13. An electron, mass m, collides head-on with an atom, mass M, initially at rest. As a result of the collision a
characterstics amount of energy E is stored internally in the atom. What is the minimum initial velocity v 0
that the electron must have ?

www.physicsashok.in 58
CENTRE OF MASS
14. Mass m 1 collides head-on with m 2, initially at rest, in a completely inelastic collision.
(A) What is the kinetic energy of the system before collision ?
(B) What is the kinetic energy of the system after collision ?
(C) What fraction of the original kinetic energy was converted into heat ?
(D) Let v cm be the velocity of the center of mass of the system.
View the collision from a primed reference frame moving with the center of mass so that v'1i = v 1i - v cm, v'2i = - v cm.
Repeat parts (A), (B) and verted to heat the same in each case ? Explain.
15. Two small blocks A and B of masses m 1= 0.5kg and m 2 = 1 kg
respectively, each carryin positive charge of q = 40mC are kept A B
stationary on a smooth horizontal floor. W hen the blocks are A + +
released, due to electrostatic repulsion, a moves towards left while 10 cm 10 cm 10 cm

B towards right. After moving 10 cm, A comes into contact with a


massless spring of force constant K = 6750 Nm-1 while after moving 20 cm, B collides nelastically with a
rigid wall as shown in fig. Calculate
(i) velocity of A when it comes in contact with the spring and
(ii) maximum compression of the spring.

16. A sphere of mass m falls with velocity v on a smooth inclined wedge of mass M and angle which rests on
2 mv cot
a smooth horizontal plane. Show that the velocity of the wedge immediately after collision is .
m M cos ec 2
The bodies are all perfectly elastic.
A B
17. Show that when two identical spheres A and B with a coefficient of
90
restitution e = 1 collide while moving with velocity v A and v B which
vA vB
are perpendicular to each other they will rebound with velocities vA
and vB which are also perpendicular to each other.
C
18. A 700 g ball B is hanging from an inextensible cord attached to a support C. A 350
g ball A strikes B with a velocity v 0 at an angle of 60 with the vertical. Assuming
60 v0
perfectly elastic impact (e = 1) and no friction, determine the velocity of each ball
immediately after impact. Check that no energy is lost in the impact. A
B

19. Three spheres, each of mass m, can slide freely on a frictionless, /2


horizontal surface. Spheres A and B are attached to an inextensible, C B
inelastic cord of length l and are at rest in the position shown when v0
sphere B is struck squarely by sphere C which is moving to the right
with a velocity v 0; knowing that the cord is slack when sphere B is

struck by sphere C and assuming perfectly elastic impact between B
and C, determine- A
(A) the velocity of each sphere immediately after the cord becomes taut.
(B) the fraction of the initial kinetic energy of the system which is dissipated when the cord becomes taut.

20. A 1 kg block B is moving with a velocity v 0 of magnitude v 0 = 2 m/s as it hits the


O
0.5 kg sphere A, which is at rest and hanging from a cord attached at O.
Knowing that k = 0.6 between the block and the horizontal surface and e=0.8
between the block and the sphere, determine after impact v0
h A
(A) the maximum height h reached by the sphere,
(B) the distance x traveled by the block. x

www.physicsashok.in 59
CENTRE OF MASS
21. A mass M = 1kg lies on a smooth horizontal base of a
rough inclined plane at an angle 370 with the horizontal as
shown in figure. A bullet of mass m=0.1 kg. is fired
horizontally with a velocity u = 110m/s and gets embeded
in it almost immediately. The impulse imparted carries the
combined mass up the incline and finally lands on the
horizontal level with the horizontal base. If the length of the incline is l = 1.8m and the incline offers a
coefficient of kinetic friction 0.5 to the sliding of the mass, find the horizontal distance from the base of the
incline to the point of landing of the combined mass. Assume the contact of the incline with the horizontal
plane is smooth and mass is not jerked when starts up on incline plane.
m

22. A small ball of mass m is connected by an inextensible massless string of length l


with an another ball of mass M = 4 m. They are released with zero tension in the
M
string from a height h as shown in figure. Find the time when the string becomes
taut for the first time after the mass M collides with the ground. Take all collisions
to be elastic.
F
23. The figure shows the force versus time graph for a particle.
(i) Find the change in momentum p of the particle. 100 N
(ii) Find the average force acting on the particle.

O 0.2 0.4 t(s)


24. While waiting in his car at a stoplight, an 80-kg man and his car are suddenly accelerated to a speed of 5
m/s at the result of a rear-end collision. Assuming the time taken to be 0.3 s, find (A) the impulse on the
man and (B) the average force exerted on him by the back of the seat of his car.

25. What force is exerted on a stationary flat plate held perpendicular to a jet of
water as shown in the fig. The horizontal speed of the water is 80 cm/s and
30 cm 3 of the water hits the plate each second. Assume that the water moves
parallel to the plate after striking it. One cubic centimeter of water has a mass
of one gram.

26. Three identical particles A, B and C lie on a smooth horizontal table,


Light inextensible strings which are just taut connect AB and BC and
ABC is 1350. An impulse J is applied to the particle C in the direction
BC. find the initial speed of each particle. The mass of each particle is m.

Level 3

1. A cylinderical solide of mass 10-2 kg and cross-sectional area 10-4 m 2 is moving a parallel to its axis (the
X-axis) with a uniform speed of 103 m/s in the positive direction. At t = 0, its front face passes the plane x
= 0. The region to the right of this plane is filled with stationary dust particles of uniform density 10-3 kg/m 3.
When a dust particle collides with the face of the cylinder, it sticks to its surface. Assuming that the
dimesions of the cylinder remain partically unchanged, and that the dust sticks only to the front face of
the cylinder, find the x-coordinate of the cylinder at t = 150 s. [JEE 1993]

2. A small sphere of radius R is held against the inner surface of larger sphere of
radius 6R (as shown in the fig.). The masses of large and small spheres are 4M
and M respectively. This arrangement is placed on a horizontal table. There is
no friction between any surfaces of contact. The small sphere is now released.
Find the coordinates of the centre of the large sphere, when the smaller sphere
reaches the other extreme position. [JEE 1996]

www.physicsashok.in 60
CENTRE OF MASS
3. A large open top container of negligible mass and uniform cross-sectional area A has a small hole of croos
-sectional are A/100 in its side wall near the bottom. The container is kept on a smooth horizontal floor and
contains a liquid of density and mass m 0. Assuming that the liquid starts flowing out horizontally through
the hole at t = 0, calculate
(i) the acceleration of the container, and
(ii) its velocity when 75% of the liquid has drained out. [JEE 1997]

4. A particle of mass m 1 is moving in a circular path of radius R with a constant


speed v 2 is located at point (2R,0) at time t = 0 and a man starts moving with a
velocity v 1 along the positive y-axis from origin at time t = 0. Calculate the
linear momentum of the particle w.r.t. the man as a function of time. [JEE 2003]

5. Two point masses m 1 and m 2 are connected by a spring of natural length l 0. The spring is compressed
such that the two point masses touch each other and then they are fastened by a string. Then the system
is moved with a velocity v 0 along positive x-axis. When the system reaches the origin the strings breaks (t
= 0). The position of the point mass m 1 is given by x1 = v 0t - A (1-cos t) where A and are constants. Find
the position of the second block as a function of time. Also find the relation between A and l 0.

6. A cart is moving along +x direction with a velocity of 4m/s. A person in the cart throws a stone with a velocity
of 6 m/s relative to himself. In the frame of reference of the cart the stone is thrown in y z plane making an
angle of 30 with the vertical z-axis. At the highest point of its trajectory, the stone hits an object of equal mass
hung vertically from branch of a tree by means of a string of length L. A completely inelastic collision occurs, in
which the stone gets embedded in the object. Determine
(a) the speed of the combined mass immediately after the collision with respect to an observer on the ground.
(b) the length L of the string such that the tension in the string becomes zero when the string becomes horizontal
during the subsequent motion of the combined mass. [JEE, 97]

7. Two blocks of mass 2kg and M are at rest on an inclined plane and are separated by a
m M
distance of 6.0 m as shown. The coefficient of friction between each of the blocks and 6.0
the inclined plane is 0.25. The 2kg block is given a velocity of 10.0 m/s 2kg

up the inclined plane. It collidies with M, comes back and has a velocity of 1.0 m/s when
it reaches its initial position. The other block M after the collision moves 0.5 m up and comes to rest. Calculate
the coefficient of restitution between the blocks and the mass of the block M.
[Take sin tan = 0.05 and g = 10 m/s2] [JEE, 99]

8. A car P is moving with a uniform speed of 5(31/2)m/s towards a carriage of


mass 9 kg at rest kept on the rails at a point B as shown in fig. The height AC
C
is 120 m. Cannon balls of 1 kg are fired from the car with an initial velocity P
100 m/s at an angle 30 with the horizontal. The first cannon ball hits
the stationary carriage after a time t0 and sticks to it. Determine t0. At t0, the
second cannon ball is fired. Assume that the resistive force between the rails A B
and the carriage is constant and ignore the vertical motion of the carriage throughout. If the second ball also hits
and sticks to the carriage. What will be the horizontal velocity of the carriage just after the second impact ?
[JEE, 01]
9. There is a rectangular plate of mass M kg of dimensions (a b). The plate is held b
in horizontal position by striking n small balls each of mass m per unit area per unit
time. These are striking in the shaded half region of the plate. The balls are colliding a
elastically with velocity v. What is v ? [JEE, 06]
It is given n = 100, M = 3 kg, m = 0.01 kg; b = 2m; a = 1m; g = 10 m/s2

www.physicsashok.in 61
CENTRE OF MASS

Answer Key
Assertion-Reason Type

Q. 1 2 3 4 5 6
Ans. B C D A A B

Match the Column

1. [(A Q), (B P), (C R), (D S)]


2. [(A R), (B P), (C S), (D Q)]
3. [(A Q), (B P), (C R), (D S)]
4. [(A Q), (B P), (C S), (D R)]
5. [(A P), (B Q), (C R), (D R)]

Level 1

Q. 1 2 3 4 5 6 7 8 9
A ns. B A C D A B CD A B
Q. 10 11 12 13 14 15 16 17 18
A ns. C B BD CD C A C A BCD B
Q. 19 20 21 22 23 24 25 26 27
A ns. AB A CD B A B C C A C
Q. 28 29 30 31 32 33 34 35 36
A ns. C C A A B C A C A
Q. 37 38 39 40 41
A ns. AC C C A A

Passage Type Problems

Q. 1 2 3 4 5 6 7 8 9
Ans. D B A C A C A A C
Q. 10 11 12 13 14
Ans. A B D B A

Level 2
L W a 3b 2
1. 0.167 L 2. , 3. 4. 3.71 m 5. L
2 2 R a3
3
3

6. (a) 2/3 m, (b) 1 m/s2 (righwards) (c) 0.5 m (d) 1 m (e) 1/3 m (f) 2/3 m/s2 rightwards (g) 2/3 m/s2 (right wards)
(h) 2/3 m/s2 (right wards

www.physicsashok.in 62
CENTRE OF MASS

1 e mv 0 sin

7. tan 1 e tan , v u cos 2 e 2 sin2 8.
M m sin2
9. 1.1 105 ft

1/ 2
(M m)
10. 216 bulltes/min. 11. m 1/3 12. 310 m/s 13. 2 E
Mm
2
1 1 m1v 1i m2 1 1
14. (a) m1v 12i (b) 2 2 2
(c) m m (d) (m1 v i v 1i v cm v cm ) m 2 v cm , zero, 10%, no
2 2 (m1 m2 ) 1 2 2 2

15. (i)12m/s (ii) 10cm. 17. 25m/s, 36.9, 43.3 m/s 53.1

18. vA = 2.73 m/s; vB = 3.78 m/s

19. (a) VA = V0 (1 e)/2 , VB = V0 (1 + e)/2 (b) VB = V0(1 e2 ) 4, VC = V0 (1 + e)2/4


(c) VC = V0 (1 + e)n 1/2n 1 (d) 0.881 V0


20. (a) 0.294 m (b) 0.0544 m 21. 8.787 m. 22. 2gh 23. (i) 20 N-S (ii) 50 N

2 10 3
24. 1330 N 25. 0.024 N 26. J, J, J
7m 7m 7m

Level 3
m0 g
1. 105m 2. (L + 2R, 0) 3. (i) g/50 (ii) 2A

v2 v2 m1 m1
4. m v 2 sin R t i v 2 cos R t v 1 j 5. x 2 = v 2t + m A(1 cos t), 0 m 1 A
2 2

6. 2.5 m, 0, 319 m 7. 0.84, 15.12 kg 8. 12 sec., 15.75 m/s 9. 10 m/s

XXXX

www.physicsashok.in 63
FLUID MECHANICS

DHANALAKSHMI NAGAR

NEAR ANNAMAIAH CIRCLE,

TIRUPATI.

PH NO. 9440025125
FLUID MECHANICS

DEFINITION OF A FLUID
The substances which flow are called fluids. Both liquids and gases come in this category. The science of
fluids at rest is called fluid statics, while that of moving fluids hydrodynamics. Fluid statics involves hydrostatics
pressure, floatation, Pascals law and Archimedes principle. Hydrodynamics involves continuity equation,
Bernoulliss principle and Torricellis theorem.

DENSITY AND PRESSURE


With fluids, we are more interested in the extended substance rather than small piece of the substance and
in properties that can vary from point to point in that substances. It is more useful to speak of density and
pressure.

Density
Density () of any substance is defined as the mass per unit volume or
Mass
=
Volume
or = m/V
Regarding density following points are worth noting :
C1: Density of a body means the ratio of mass of a body to the volume of the body while density of a substance
means the ratio of mass of substance to the volume occupied by the substance. Thus, for a solid body,
Density of body = Density of substance
While for a hollow body, density of body is lesser than that of substance because volume of hollow body is
greater than volume of substance.
C2: In case of a liquid, sometimes an another term relative density (RD) is defined.
Density of substance
RD =
Density of water at 4 C
RD is a pure ratio. So, it has no units. It is also some times referred as specific gravity.

Note : Density of water at 4C in CGS is 1 g/cm3. Therefore, numerically the RD and density
of substance (in CGS) are equal. In SI units the density of water at 4C is 1000 kg/m3.

C3: If two liquids of densities 1 and 2 having masses m1 and m2 are mixed together, then the density of mixture
will be
Total mass (m1 + m 2 )
= =
Total volume (V1 + V2 )

(m1 + m 2 )
=
m1 m 2

1 2

212
If m1 = m2, = = Harmonic mean
1 2
C4: If two liquids of densities 1 and 2 having volumes V1 and V2 are mixed, then the density of the mixture is,
Total mass (m1 + m 2 )
= =
Total volume (V1 + V2 )

www.physicsashok.in 1
FLUID MECHANICS

1V1 2 V2
= V1 V2
1 2
If V1 = V2, = = Arithmetic mean
2
C5: If the temperature of a liquid is increased, the mass remains the same while the volume is increased, then
1
density of the liquid decreases . Thus,
V
V

V
V V

V V V V
1
or
1
Here, = thermal coefficient of volume expansion and = rise in temperature

1
1
C6: With increase in pressure due to decrease in volume, density will increase, i.e.,
V V V

V V V V P / B V
1
or
1 P / B
Here, P = charge in preseure
and B = bulk modulus of elasticity of the liquid
P
1
Therefore,
1 P / B B

Example 1. A king orders for a gold crown having a mass of 4 kg. When it arrives from the goldsmith, the
volume of the crown is found to be 100 cm3. What is your opinion about the crown if the density of pure
gold is 8 103 kg/m3 ?
Sol. The density of crown
mass 4
crown =
volume 100 106
crown = 4 104 kg/m3
which is just half that of pure gold.
There are two possibilities about this deviation :
(a) The crown contains impurities of light metals decreasing its mass for a given volume and so the density.
(b) The crown contains cavities increasing its volume for a given mass and so decreasing the density.

www.physicsashok.in 2
FLUID MECHANICS

C7:. Relative density of an oil is 0.8. Find the obsolute density of oil in CGS and SI units.
Sol. Density of oil (in CGS) = (RD) g/cm3
= 0.8 g/cm3
= 800 kg/m3

Example 2. When equal volumes of two metals are mixed togethers, the specific gravity of alloy is 4. When
equal masses of the same two metals are mixed together, the specific gravity of the alloy is 3. Calculate the
specific gravity of each metal.
Sol. In case of two metals

1
m m2
V1 V2
So, when equal volumes are mixed, V1 = V2 = V and m1 = V1 and m2 = V2, then
V1 V2

VV
1 2
or 4 ...(1)
2
When equal masses are mixed, m1 = m2 = m and V1 = (m/1) and V2 = (m/2), then
mm

m / 1 m / 2
212
or 3 ...(2)
1 2
Solving Eqs. (1) and (2) for 1 and 2, we find specific gravities of metals are 2 and 6.

CONCEPT OF PRESSURE
The word pressure is linked with pressing. Two surfaces in contact exert on each other normal contact
force. With this force they press each other.
w w

A A

N w N
w .... .... .... ....
N A N A
In the figure an anvil of weight W is placed on a horizontal surface making contact in area A or A. The free
body diagram shows that contact force N is equal to W, because anvil is not accelerated along vertical.
N=W
In the two cases surface have different intensity of pressing. How much intense is the pressing ? This is
described by a term pressure.
In the figure same contact force N is distributed over large area A in one case and smaller area A is other.
We define pressure as the magnitude of normal contact force per unit area. Thus, pressure for area A is

|N|
P
A
and that for area A is

www.physicsashok.in 3
FLUID MECHANICS

|N|
P
A
We define presure at a point by taking a small area A around the point. The magnitude of normal contact

force over this area is | N | . Then pressure at the point is limiting value of | N | / A | , where A tends
to zero. That is,

| N|
P lim
A 0 A

dN
P
dA
SI. Unit : The unit of pressure is just unit of force divided by unit of area; Nm2 in SI. We cal 1 Nm2 = 1 Pa
(pascal).

Dimensional Formula :
[MLT2/L2] = [ML1 T2]

Example 3. A sample of metal gasket is placed between flat narrow ends of two conical diamond anvils and
compression of 50 kN is applied. The diameter of the end faces is 5.0 103 m., Calculate how much
pressure is existing on the sample.
(Metal Gasket)

50 kN 50 kN

Diamond Cone

Sol. As anvils are unaccelerated, the contact force N is equal to the applied force F.
Thus N = F = 50 kilo newton.
A
N
N F
A
acceleration = 0
This force is distributed over the contact area. The gasket is slightly protruding outside hence the whole end
area of anvils are in contact. Thus, area A of contact is given by
A = D2/4
3.141
A= (5.0 103)2 m2
4
Pressure existing at the face of sample is given by
N
P
A
4 50 103 N
P
(3.141) (5.0 10 3 ) 2 m 2
P = 2.5 1011 Pa.
www.physicsashok.in 4
FLUID MECHANICS

PRESSURE DUE TO FLUID COLUMN (AT REST)


One of the interesting things is the pressure at the surface of the earth exerted by air surrounding it. This is
known as atmospheric pressure at the surface.
How does it arise ? It is due to collision of gas z (up)
molecules (see kinetic theory of gases). We can,
however, find its value by considering statics of a fluid
column at rest.
We know that the magnitude of normal contact force (P+dP)A
per unit area of contact surface is equal to pressure. dz
Hence we calculate the contact force by assuming PA
z g
vertical acceleration of air column to be zero. This is Adz
known as hydrostatic condition in an inertial frame. z=0
acceleration = 0
The figure shows an air column of cross-sectional area
A, extending vertically along Z-axis.
At a height z we take an elementary height dz. The volume of this elements is A dz. multiply it by density of
air at this level, , and gravitational field intensity g. We get pull due to the Earth. Let P be pressure at z and
P + dP at z + dz. The free body diagram of the fluid element is shown. We have
PA = (P + dP) A + Adz g
0 = dP + dz g
dP
g
dz
This is the fundamental equation of lapse rate of pressure in fluid of density . There are several implications
of this equation. Z
C8: If we move along z = constant in a homogeneous fluid (horizontal level) we get
dP = 0 or P = constant. Thus pressure at points in the same horizontal level is
the same in a homogeneous fluid at rest in an inertial frame A B
C9: If we move along a path passing through heterogeneous fluid, pressure in the
same horizontal level is not the same. In the figure pressure at A is not equal to 1
g
pressure at B, because the path ACB is passing from A to C and C to B 2

through liquids of different densities (hetergeneous). C


C10: In the same liquid, the pressure decreases with height because dP/dZ is negative. In other words, pressure
increases with depth in a fluid.

Z-axis

P0
Z0
g

Z0 h

Z=0

To see the above fact, let P0 be pressure at the surface of a liquid of density . The pressure at depth h may
be found using the equation dP/dz = g.
We have, dP = g dz.

www.physicsashok.in 5
FLUID MECHANICS

Let us take z-coordinate at the top as z0. At z = z0 (top of liquid) the pressure is due to air. Let it be denoted
by P0. Then at depth h, the z-coordinate will be h less than z0, i.e., z0 h. Hence integrate L.H.S. from P to
P0 and R.H.S. from z0 h to z0. That is,
P0 z0

dP g dz
P z0 h

P0 P = .g. [z0 (z0 h)]


P = P0 + gh
This formula can be more easily derived as below.

P0 A (force due to pressure)

A Ah g(weight of column)

PA(force due to pressure)

Consider the vertical equilibrium of fluid column of height h. acting on if forces the area as shown in the
figure.
P0A + A h g = PA
P = P0 + hg
As we go in the depth, pressure increases.
Here P0 is atmospheric pressure. Using this formula we cannot know P without knowing P0. A barometer
does the job of measuring atmospheric pressure P0.

C11: A U-tube of uniform cross section (see Fig.) is partially filled with a liquid I. Another
liquid II which does not mix with liquid I is poured into one side. It is found that the liquid II I
levels of the two sides of the tube are the same, while the level of liquid I has risen by
2 cm. If the specific gravity of liquid I is 1.1, the specific gravity of liquid II must be
(A) 1.12 (B) 1.1 (C) 1.05 (D) 1.0
Sol. [B]
The level of liquid is same in both the limbs. Pressure in limbs I at B = Pressure of limb II at A

h P2 II h P1 I

A B

h1g = h2g
1 = 2

www.physicsashok.in 6
FLUID MECHANICS

Variation of pressure at two points when fluid is in horizontal acceleration :


When a fluid is subjected to a constant acceleration, different elements of the fluid orient themselves so as
to obtain a new equilibrium position with respect to the vessel. Since the fluid particles are relatively at rest
with respect to one another, therefore, the laws of fluid statics area applicable but in the modified form to
take into account the effect of acceleration. l
Consider a horizontally accelerating container with liquid in it. Take two
points A and B at a horizontal distance l (along the line of acceleration a). hA hB
Take a horizontal cylinder within the two areas S containing points A and
A B a
B. Let pressure at A is PA & at B is PB.
S
The forces along the line AB are PA S towards right and PB S towards
left. Under the action of these forces the element is accelerating.
thus, PA S PB S = ma
or (PA PB) S = (S)l a
or P1 P2 = la
i.e., two points in same horizontal level do not have equal pressure for a fluid accelerating horizontally. If
atmospheric pressure is P0 then, PA = P0 + hA g and PB = P0 + hB g, as there is no vertical acceleration.
hA and hB are the depths of points A & B from free surface.
h1 g h2 g = l a
h1 h 2 a
or
l g
a
or tan
g
where is the inclination of the free surface with the horizontal.

Example 4. A vessel containing water is given a constant acceleration a towards the right along a straight
horizontal path. Which of the following diagrams in Fig. represents the surface of the liquid ?

a a a
(A) (B) (C) (D) none of these

Sol. [C]

A (dx)
D

B C
Let us consider a small dotted segment of thickness dx for observation.
Since this segment is accelerated towards right, a net force is acting in this segment towards right from the
liquid towards the left of ABCD. According to Newtons third Law, the segment ABCD will also apply a
force on the previous section creating a pressure on it which makes the liquid rise.

www.physicsashok.in 7
FLUID MECHANICS

Variation of pressure at two points when fluid is in vertical acceleration:


Consider point A and B inside liquid with a vertical separation h where the
liquid is accelerating upward. Consider a vertical cylinder of length h with
boundaries at A & B. S is the surface area of flat boundaries. Let PA &
PB are the pressures at A & B respectively. Thus, forces acting on the
cylinder are PA S upward PB S downward and W = (S)hg downward. B a
Under the action of these forces liquid cylinder is accelerating upward with
h
acceleration a.
Thus, PA S PB S mg = ma A
or (PA PB) = (g + a) h
Special cases :
(i) If a is ()ve i.e. the vessel is accelerating downward then, (PA PB) = (g a) h
(ii) If a is zero then pressure is equal everywhere inside the liquid.
(iii) If a is greater than g then fluid occupies the upper part of the container.

Example 5. A bucket contains water filled upto a height = 15 cm. The bucket is tied to a rope which is passed
over a frictionless light pulley and the other end of the rope is tied to a weight of mass which is half of that of
the (bucket + water). The water pressure above atmosphere pressure at the bottom is
(A) 0.5 kPa (B) 1 kPa (C) 5 kPa (D) None of these
Sol. The acceleration of bucket is
2mg mg g
a downward
3m 3
P = P0 + (g a) h
10
P P0 1000 10 0.15
3
20
P P0 1000 0.15
3
P P0 = 103 N/m2 = 1 kPa Hence option (B) is correct.

Variation of pressure when fluid is under both horizontal and vertical acceleration :
When a fluid is accelerating in arbitrary direction then pressure varies both horizontally and vertical separately
for horizontal and vertical component of acceleration respectively.

Here, a a i a j and a = a cos & a = a sin
x y x y
Now, difference in pressure at two points in same horizontal level at a separation ay
a
l is given by
ax
P l a x l a cos
And similarly for two points in same vertical line with a separation h, the pressure
difference is
P (g a y )h (g a sin )h
Inclination of the free surface with respect to the horizontal is given by
ax a cos
tan ; tan
g ay g a sin

www.physicsashok.in 8
FLUID MECHANICS

Example 6. A trolley containing a liquid of density slides down a smooth inclined


plane of angle with the horizontal. Determine the angle of inclination of the
free surface with the horizontal.
Sol. The acceleration of the trolley along the inclined plane is
a = g sin
ax = a cos = g sin cos
ay = a sin = g sin2
The inclination of the free surface is =
ax
ax g sin cos
tan tan ay
a=gsin
g a y g(1 sin 2 )
or =
Note : That the free surface becomes parallel to the inclined plane. It is
because the net effective gravity inside the trolley is perpendicular to the
inclined plane.

Example 7. A fluid container is containing a liquid of density is accelerating a


upward with acceleration a long the inclined place of inclination as shown.
Then the angle of inclination of free surface is :

1 a 1 a g sin
(A) tan (B) tan
g cos g cos

1 a g sin 1 a g sin
(C) tan (D) tan
g(1 cos ) g(1 cos )
Sol. N cos = mg cos N cos N N sin
and N sin = ma + mg sin
a g sin
tan
g cos
ma+mg sin
a g sin mg cos
tan 1
g cos
Hence option (B) is correct.

Variation of pressure when a fluid is rotating :


When a liquid container is rotating with constant angular velocity, liquid
particles orient themselves to a new equilibrium position at which no
relative motion exists between different parts of the liquid. Consider
the container is rotated about its axis with constant angular speed . 2 rm

Consider a small fluid element which is under the action of two forces,
horizontal centrifugal reaction due to rotation and down-ward force mg
due to gravity. y
The slope of the free surface at the position of element is
O r
dy 2r 2r
tan or dy dr
dr g g

www.physicsashok.in 9
FLUID MECHANICS

2 2
Integrating we get, y = y0 + r
2g
where y0 is the integration constant which depends upon the initial values.
Let R is the radius of the cylinder and h0 is the height of the liquid before the start of rotation then
2R 2
at r = R, y = ymax = y0 +
2g
at r = 0, y = ymin = y0
Here original height of the liquid is the mean of maximum and minimum heights.
y max y min 2 R 2
So, h0 y0
2 4g

2R 2
or y0 h 0
4g
2R 2
y max h 0
4g
2R 2
y min h 0
4g
2R 2 2r 2
and y h0
4g 2g
Note : The above equation shows that the free surface of liquid is that of a paraboloid.

Example 8. A cylinder of radius R = 1 m and height H = 3 m, two-third filled


with water, is rotated about its vertical axis, as shown in figure. Determine H
the speed of rotation when h=(2/3)H
(i) The water just starts spilling over the brim,
(ii) The point at the centre of the base is just exposed.
R
Sol. (i) We know that
ymax = H = 3 ; H = 2/3 H = 2 m;
R = 1 m; g = 10 ms2.
4g(H h) 4(10)(3 2)
Thus 2

R 12
40 rad s 1 y=H
(ii) We know that
r 2
y y0 R
2g
Here, y = H = 3m; y0 = 0; r = R = 1m
g = 10 ms2
2gH 2(10)(3)
2
; 60 rad s 1
R 12
www.physicsashok.in 10
FLUID MECHANICS

Example 9. Figure shows a three arm tube in which a liquid is filled upto levels
A B C
of height l. It is now rotated at an angular frequency about an axis passing
through arm B. The angular frequency at which level of liquid in arm
B becomes zero. l l

2g g 3g 3g l l
(A) (B) (C) (D)
3l l l 2l

3l 3g l
Sol. P P0 g P0
2 2
l
P P0 S S l 2 2
3g l S S l 2 2

2 2
3 l 2
g
2 2
3g
2
l
3g
Hence option (C) is correct.
l

Example 10. Length of a horizontal arm of a U-tube is l = 21 cm and ends of


both of the vertical arms are open to surroundings of pressure 10500

a = 6 cm
Nm2. A liquid of density = 103 kg m3 is poured into the tube such that
liquid just fills horizontal part of the tube. Now, one of the open ends is
sealed and the tube is then rotated about a vertical axis passing through the
other vertical arm with angular velocity 0 = 10 radians/sec. l = 21cm
If length of each vertical arm be a = 6cm, calculate the length of air column in the sealed arm.
Sol. When tube is rotated, liquid starts to flow radially outward and air in sealed arm is compressed. Let the shift
of liquid be x as shown in figure.
Let cross-sectional area of tube be S. Initial volume of air,
V0 = Sa and initial pressure P0 = 10500 Nm2
Final volume, V = S (a x) ax
P0 V0 P .a
Final pressure, P 0
V (a x) x
A
P0 a (l x) B
x
or pressure at B, P2 = P + xg = + xg
(a x)
Centripetal force required for circular motion of vertical column of height x of liquid is provided by reaction
of the tube while that to horizontal length (l x) is provided by excess pressure at B.
Force exerted by pressure difference is

www.physicsashok.in 11
FLUID MECHANICS

P0 x
F1 = (PB PA) S = (P2 P0) S = xg S
(a x)
Mass of horizontal arm AB of liquid is,
m = S(l x)
l x l x
Radius of circular path traced by its centre of mass is r = x +
2 2
Centripetal force, F2 = m02r

l x 2 P0 x
But F2 = F1 or {S (l x)} 0 xg S
2 (a x)
or x = 0.01 m = 1 cm
Length of air column in sealed arm = (a x) = 5 cm.

BAROMETER
We can measure atmospheric pressure P0 using a liquid column in a tube. This arrangement is shown in the
figure and is called barometer. (There are several other modifications). Mercury is filled in a tube of length
about 1m. Its mouth is covered (by thumb, say) and inverted into a vessel containing mercury. It is held in
position by a stand and mouth is now uncovered.
Z
P1A
z0

h h
A gh

Z0 h
P0 A

It is observed that some mercury comes out in the vessel. A vacant space is created at the top (closed) end.
This space is historically called Torricellis vacuum, although same mercurry vapour is present in it.
If pressure at the top of mercurry column be P1, we have, for equilibrium of column,
P1A + A 0gh = P0 A.
Here P1 is mercurry vapour pressure which we generally ignore. Then
P0 = 0gh
The atmospheric pressure is, thus, measured if h be measured.
The normal density (0) of mercury is 13.6 g cm3 and g = 981 cms1. For a given place and temperature,
these data are fixed. Hence directly h can be used to show pressure. One atmospheric pressure is a unit of
pressure equal to 76 cm of Hg.

Example 11. A mercury barometer reads h0 as pressure of air in a closed lift at rest. If the lift accelerates up at
g/2, what will be :
(a) atmospheric pressure in the lift; and
(b) the reading of the barometer ?
Sol. (a) The air enclosed in the lift will have the same temperature and volume. Its pressure will be the same as
P0 = h0g. Here is the density of mercury in barometer.
www.physicsashok.in 12
FLUID MECHANICS

(We shall see in the kinetic theory of gases that pressure is arising due to collision of molecules.)
(b) When the lift accelerates, mercury column also accelerates. Let h be the length of mercury column now, and
a (= g/2) be the acceleration. Then, using Newtons second law of motion,
P0A h Ag = (hA)a
= hAg/2
h Ag
P0 = 3/2 hg a
Now we compare it with h0.
P0 = gh0
P0 A
h0 = 3/2h
h = 2/3 h0 (reading)
The reading of barometer falls, although pressure in the lift remains the same.

Gauge Pressure : The pressure in excess of atmospheric pressure is called gauge pressure. P P0 = gh is the
gauge pressure. It is measured by a manometer.

P0

h g
P

manometer
If the pressure be lower than atmospheric, we define vaccum pressure as the deficiency in pressure.

Variation of pressure in air with height


The pressure decreases as we go up in the atmosphere as the temperature and density of air at different
heights is not the same. A rough estimate shows that at constant temperature, pressure falls exponentially
with height.

Other pressure units :


1.013 105 Pa = 1 atm.
105 Pa = 1 bar
1.013 bar = 1 atm.
1 mm of Hg = 1 torr = 133 Pa.

Example 12. Find the absolute and gauge pressures of the gas Gas
above the surface of the oil in the tank shown in the figure. 1.5 cm 1.0 cm
3
Take the density of oil as 800 kg/m , density of mercury as Oil 2.0 cm
13.6 103 kg/m3, atmospheric pressure as 1.0 105 A B
2 2
N/m and g = 10 m/s .
Sol. As the points A and B are at the same horizontal level in
mercury, the pressure at these points is the same. Now, if P is the absolute pressure of the gas and P0 is the
atmospheric pressure, then pressure at A is

www.physicsashok.in 13
FLUID MECHANICS

PA = P + (1.5 + 2.0) 800 10


PA = P + 0.28 105 N/m2
Similarly pressure at B is
PB = P0 + (1.0 + 2.0) 13.6 103 10
PB = P0 + 4.08 105 N/m2
Using PA = PB
Gauge pressure
we get P P0 = 3.8 105 N/m2
and Absolute Pressure
P = (1.0 105 + 3.8 105) Pa
P = 4.8 105 Pa.

PASCALS LAW
(i) The pressure in a fluid at rest is the same at the same height (or depth relative to top surface).

h
A B

Let two points A and B at the same depth h from the top surface. We consider a narrow tube of fluid AB
between points A and B. Let be its cross sectional area. Force at end A is PA and that at end B is PB.
Since the tube is not accelerating horizontally,

PA PB

these forces must be balanced. Thus,


PA = PB
PA = PB.
This proves the Pascals observation.

(ii) The change in pressure at any point in a closed liquid is totally transmitted to all the points of the liquid.
We can show this by using P = P0 + gh.
Let P0 be changed by p. Then, the pressure at depth h is given by
P = (P0 + p) + gh.
P P = p.
This does not contain h. Hence the change in pressure at the top is transmitted to all the points totally.

(iii) Pressure at a point does not depend upon orientation of the surface (area) used to define it.
To show this we take a small prism of liquid around the observation point O. Let N1, N2 and N3 be normal
forces acting on faces of the prism having area A1, A2 and A3.
N1 N2 N3
Now P1 = A , P2 = A and P3 = A .
1 2 3

www.physicsashok.in 14
FLUID MECHANICS

Since the prism is at rest,


N1
N1 N 2 N3 0
Resolving forces, N2
A2 O
N2 = N1 sin ...(1) A1
A3
N3 = N1 cos
Also A1 cos = A3 N3
A1 sin = A2 ...(2)
N1
P1 = A
1
N2 N1 sin N1
P2 = A = A sin = A
2 1 1

N3 N1 cos N1
P3 = A = A cos = A
3 1 1

Thus, P1 = P2 = P3.
This also distinguishes pressure from normal stress. Although they are dimensionally the same, normal stress
changes by changing the orientation of surface whereas pressure remains the same. Pressure is a scalar
quantity.

APPLICATIONS OF PASCALS LAW


Since the change in pressure at any point in a liquid is transmitted to all points of the liquid, we can obtain
larger force by using larger contact area. This is the principle of hydraulic lift or hydraulic brakes.

Hydraulic Machines
Figure shows a horizontal cylinder with a piston and three vertical tubes. Liquid has the same level in all the
three tubes. This shows that pressure is the same at their bases. Now we push the piston and wait. We find
that the level is raised but by the same height in the all the tubes. Thus the change in pressure has been
transmitted to all the points without any decrease. (This is Pascals law of transmission of pressure).

This is the principle of hydraulic machines.

Example : Hydraulic lift


Figure shows two pistons of areas A1 and A2. They are connected by a liquid. If we apply a force F1 a
change in pressure F1/A1 is produced. This, according to Pascal, is transmitted to all the points of the liquid,
and hence also at the base of right piston (of area A2). Corresponding to the change in pressure, there is
produced a force F2, acting over area A2, such that
F2 = (change is pressure) A2
F1
F2 = A A2.
1

www.physicsashok.in 15
FLUID MECHANICS

F1

A1

A2

F2

As A2 > A1, the force F2 is larger than the applied force F1.

Mechanical advantage of a machine is defined as the ratio of load (F2) to effort (F1). There (A2/A1) is equal
to (F2/F1), the mechanical advantage of hydraulic lift.
the large force F2 may be used to press a drum of rotating wheel, acting as brake. This is the principle of a
hydraulic brake. The force F2 may be used to press big bundles into small for typing Brahma press is such
a device.

Example 13. The area of cross-section of the wider tube shown in figure is
12 kg
800 cm2. If a mass of 12 kg is placed on the massless piston, the
difference in heights h in the level of water in the two tubes is :
(A) 10 cm (B) 6 cm (C) 15 cm (D) 2 cm
Sol. Pressure in horizontal pipe at both sides are same.
mg
P0 + gh + gH P0 gH
A
mg
gh
A
mg m 12
h
gA A 1000 800 104
12 3
h m 100 15 cm Hence option (C) is correct.
80 20

FORCES ACTING ON SOLID BOUNDARIES


(a) Whenever a fluid comes in contact with solid boundaries it exerts a force on it.
(b) This force on a particular bounary may be obtained by integrating the pressure
over the entire area of the boundary.
(c) The variation of liquid pressure acting at base, vertical wall and inclined
wall are shown in the figure.

Force at the base :


As the pressure is uniform at the flat base, therefore, force acting at the base is
F = P (area of the base) P = gh
F = gh (lb)
F g (hlb)
where is the density of liquid, h is the height of the liquid column, l & b are the length and breadth of the
base.

www.physicsashok.in 16
FLUID MECHANICS

C12: A beaker of circular cross-section of radius 4 cm is filled with mercury upto a height of 10 cm. Find the
force exerted by the mercury on the bottom of the beaker. The atmmospheric pressure = 105 N/m2.
Density of mercury = 13600 kg/m3. Take g= 10 m/s2.
Sol. The pressure at the surface
= atmospheric pressure
= 105 N/m2.
The pressure at the bottom.
105 N/m2 + hg
kg m
= 105 N/m2 + (0.1 m) 13600 3 10 2
m s
5 2 2
= 10 N/m + 13600 N/m
= 1.136 105 N/m2.
The force exerted by the mercury on the bottom
= (1.136 105 N/m2) (3.14 0.04 m 0.04 m) = 571 N.

Example 14. The vessel shown in the adjoining fig. is filled with water of density = 10 cm2
w = 103 kgm3.
(i) Find the net force acting at the base of the vessel. 5 cm
(ii) What is the weight of water in the vessel ?
Sol. (i) Pressure at the base of the vessel is
1 cm
P = w gh = (103) (10) (6 102) = 600 Pa
A2 = 50 cm
F = A = (600) (50 104) = 3 N
(ii) Weight of water in the tube is
W = wg (A1h1 + A2h2)
or W = (103) (10)[(10 104) (5 102) + (50 104) (1 102)]
W=1N

Example 15. A Utube having horizontal arm of length 20 cm, has uniform cross-sectional area = 1 cm2. It is
filled with water of volume 60 cc. What volume of a liquid of density 4 g/cc should be poured from one side
into the Utube so that no water is left in the horizontal arm of the tube ?
(A) 60 cc (B) 45 cc (C) 50 cc (D) 35 cc
Sol. P0 + l gh0 = P0 + gh
4 h0 = 60
h0 h
60
h0 15 cm
4
Volume of liquid pushed = A (h0 + 20) = 1 (15 + 20) = 35 cm3.
Hence option (D) is correct.

Force on the vertical wall y


Pressure acting on the vertical wall is not uniform but increases Py = gy
linearly with depth. h
Let us take an element dy of the boundary at a depth y from the
Ph = gh
free surface.

www.physicsashok.in 17
FLUID MECHANICS

Pressure at depth y, Py = gy
Force acting on the element dy of breadth b is
dF = Py (bdy) = gb ydy
Total force on the wall,
h 1
F dF gbydy gbh 2
0 2
Thus, total force acting per unit width of the vertical wall
F 1
gh 2
b 2
The point of application (or centre of force) of the total force from the free surface is
h h h
2

he
ydF ygbydy y dy
0 0 0
h h h
dF gbydy ydy
0 0 0

1 3
h
2
he 3 h
1 2 3
h
2

2
he h
3
i.e., the total force acts at a depth of 2/3 h from the free surface.

Force on an inclined wall :


We know pressure always acts normal to a surface, and it depends upon
the depth y of the point from the free surface.
h
The horizontal force Fx acts on the vertical projection of the inclined wall h/sin

1
Fx = gbh2 P = gh
2
And the vertical force Fy acts due to the weight of the liquid supported by the wall,
1 1
Fy = gb (h) (h cot ) = gbh2 cot
2 2
1
F Fx2 Fy2 gbh 2 1 cot 2
2
1 1
or F gbh 2 .
2 sin

F 1 h2
or g
b 2 sin

www.physicsashok.in 18
FLUID MECHANICS

Example 16. Some liquid is filled in a cylindrical vessel of radius R. Let F1 be the force applied by the liquid on
the bottom of the cylinder. Now the same liquid is poured into a vessel of uniform square cross-section of
side R. Let F2 be the force applied by the liquid on the bottom of this new vessel. (Neglect atmosphere
pressure) Then :
(A) F1 = F2 (B) F1 = F2/ (C) F1 = F2 (D) F1 = F2
2 2
Sol. V = R h = R H
H = h
F1 = gh R2
F2 = gH R2 = g R2h = F1
Hence option (D) is correct.

Example 17. A liquid of mass 1 kg is filled in a flask as shown in figure. The force
exerted by the flask on the liquid is (g = 10 m/s2) [Neglect atmospheric
pressure] :
(A) 10 N (B) greater than 10 N
(C) less than 10 N (D) zero
Sol.
Fy

Fx
Force diagram
Fx mg of liquid
ghA

Fy + gh A = mg = 10 N Hence option (A) is correct.

Example 18. A heavy hollow cone of radius R and height h is placed on a horizontal table surface, with its flat
base on the table. The whole volume inside the cone is filled with water of density . the circular rim of the
cones base has a watertight seal with the tables surface and the top apex of the cone has a small hole.
Neglecting atmospheric pressure find the total upward force exerted by water on the cone is
(A) (2/3)R2hg (B) (1/3)R2hg (C) R2hg (D) None
Sol. By force diagram of liquid :
Fy + gh R2 = mg Fy
Fy = mg gh R2
Fx
1 Fx
Fy = R2h g gh R2
3 gh R2
2 mg
Fy = R2h g
3
2
Hence upward force exter on come by water is R2hg. Hence option (A) is correct.
3

BUOYANCY
(a) If a body is partially or wholly immersed in a fluid, it experiences an upward force due to fluid surrounding
it. The phenomenon of force exerted by fluid on the body is called buoyancy and the force is called buoyant
force.

www.physicsashok.in 19
FLUID MECHANICS

(b) When a solid body is immersed in a liquid, it displaces the liquid, partially or completely of its volume. While
doing so it pushes or applies a downward force on the liquid, hence, in accordance to newtons third law,
the liquid also exerts an upward force (equal to the weight of the displaced liquid)

Archimedes principle
It states that when a body is partically or fully dipped into a fluid at rest, the fluid exerts an upward force of
buoyancy equal to the weight of the displaced fluid.
If V be the volume of displaced fluid of density then, buoyant force, FB = Vg
NOTE :
1. Buoyant force arises because the pressure in the fluid is not uniform but increases with depth.
2. Buoyancy force acts through the centroid of the displaced fluid.
3. Buoyant force may be different from the weight of the displaced fluid in non-inertial frame of reference.

Flotation
(a) When the force of buoyancy is equal to the weight of the dipped (partially or completely) solid, the solid will
remain in equilibrium. This is called flotation.
(b) When the overall density of the solid is smaller than that of fluid, the solid floats with a part of it in the fluid.
The fraction dipped is such, that the weight of the displaced fluid equals the weight of the solid.
(c) APPARENT WEIGHT : A solid of volume Vs is dipped inside a liquid, where it displaces the liquid by a
volume Vl. If s and l be the density of solid and liquid respectively then, apparent weight of the solid is,
Wa = Vs s g Vl l g
or Wa (Vs s Vl l )g
For completely submerged solid, Vl = Vs and we have
Wa = Vs (s l)g
(a) Stability of floating body depends on the effective point of application of the buoyant force in relation to the
point of application of its weight.
(b) The weight of a body acts through its centre of gravity G and the buoyant force acts through the centre of
mass/buoyancy B of the displaced liquid.

(A) STABILITY OF COMPLETELY SUBMERGED BODY W


(a) Neutral equilibrium : When a homogeneous cylinder is
G B
completely immersed in a liquid G and B coincide with each other.
Then all positions are equilibrium position. FB

G W G
FB
(b) Unstable equilibirum : When an additional weight is placed on
B
top of a homogeneous cylinder so that G lies above B. Then by a B W
slight disturbance the cylinder topples. B

(c) Stable equilibrium : When an additional weight is placed at the FB FB


bottom of a homogeneous cylinder so that G lies below B. A slight B
disturbance in the cylinder produces restoring couple which tries B
G
to bring the cylinder in earlier position. G W
W

www.physicsashok.in 20
FLUID MECHANICS

(B) STABILITY OF PARTIALLY SUBMERGED BODY W


(a) For partically submerged body under certain condition may be G
stable even if G lies above B. B
(b) It is because a slight rotation in the body may cause a sufficient FB
shift in the position of centre of buoyancy (as shape of the displaced
liquid changes) which may provide a restoring couple. W M
(c) The body remains in stable equilibrium as long as B remains below
G FB
the meta centre M (point of enter section of shifted line to original
B
line of action of buoyant force)

C13: A body floats in a liquid contained in a beaker. The whole system as shown in
Figure falls freely under gravity. The upthrust on the body is
(A) zero
(B) equal to the weight of the liquid displaced
(C) equal to the weight of the body in air
(D) equal to the weight of the immersed portion of the body
Sol. [A]
The whole system falls freely under gravity
Upthrust = weight of fluid displaced
= (mass of fluid displaced) g
For a freely falling body, g = 0
Upthrust = 0.

C14: The spring balance A reads 2 kg with a block m suspended from it. A balance
A
B reads 5 kg when a beaker with liquid is put on the pan of the balance. The
two balances are now so arranged that the hanging mass is inside the liquid in
the beaker as shown in the figure. In this situation :
m
(A) the balance A will read more than 2kg
(B) the balance B will read more than 5 kg B
(C) the balance A will read less than 2 kg and B will read more than 5 kg
(D) the balance A and B will read 2 kg and 5 kg respectively
Sol. [BC]
When the block of mass m is arranged as shown in the figure, an upthrust A
FT will act on the mass which will decrease the reading on A. but according
to Newtons second law, to each and every action, there is equal and
opposite reaction. So FT will act on the liquid of the beaker which will
increase the reading in B. FT FT

Example 19. A vessel contains oil (density = 0.8 gm/cm3) over mercury (density = 13.6 gm cm3). A homogeneous
sphere floats with half its volume immersed in mercury and the other half in oil. The density of the material of
the sphere in gm/cm3 is
(A) 3.3 (B) 6.4 (C) 7.2 (D) 12.8

www.physicsashok.in 21
FLUID MECHANICS

Sol. [C]
Wt. of sphere = Uptrust due to Hg + Upthrust due to oil
Oil
V V
Vdg d Hg g d oil g
2 2
d Hg d oil 13.6 0.8
d 7.2 g / cm3
2 2 Mercury

Example 20. In English the phrase tip of the iceberg is used to mean a small visible fraction of something that
is mostly hidden. For areal iceberg what is this fraction if the density of sea water is 1.03 g/cc and that of ice
is 0.92 g/cc ?
Sol. In case of floatation, weight = upthrust, i.e.,
mg = Ving, i.e., V = Vin [as = m/V]

or Vin = V so Vout = V Vin = V 1

Vout 0.92 0.11
or fout 1 1
V 1.03 1.03
fout = 0.106 or 10.6%

Example 21. A cork of density 0.5 gcm3 floats on a calm swimming pool. The fraction of the corks volume
which is under water is
(A) 0% (B) 25 % (C) 10 % (D) 50 %
Sol. mg = Vg
V0g = Vg
0.5 V0g = 1 Vg
V0
V
2
Fraction = 50 % Hence option (D) is correct.

Example 22. A small wooden ball of density is immersed in water of density to depth h and then released.
The height H above the surface of water up to which the ball will jump out of water is
h
(A) (B) 1 h (C) h (D) zero

Sol. When ball is insides water :
The acceleration of ball inside water is
Vg mg
a0
m
Vg Vg
a0 1 g g
V
The velocity of ball at the surface is
v02 = 2a0 h

www.physicsashok.in 22
FLUID MECHANICS

v2 = v02 2 gH
or 02 = v02 2gH
v02 2a 0 h a 0 h
H
2g 2g g


H h 1 h Hence option (B) is correct.

Example 23. A sphere of radius R and made of material of relative density has a concentric cavity of radius
r. It just floats when placed in a tank full of water. The value of the ratio R/r will be
1/3 1/3 1/ 3 1/ 3
1 1 1
(A) (B) (C) (D)
1 1
Sol. Since, sphere just floats,
mg = Vg
4 4 4
or 0 R 3 r 3 g R 3g
3 3 3
0
But

(R3 r3) = R3
r3 r3 1
or 1 3 1 or 1
R R3
3
1 r3 1 r
or 1 3 or
R R3
1/ 3
R
or Hence option (A) is correct.
r 1

Example 24. A cubical block of wood 10 cm on a side and of density 0.5 g/cm3 floats in a jar of water. Oil of
density 0.8 g/cm3 is poured on the water until the top of the oil layer is 4 cm below the top of the block. (a)
How deep is the oil layer ? (b) What is the gauge pressure at the lower face of the block ?
Sol. (a) Let the oil layer be x cm deep. Then weight of block = weight of oil displaced + weight of water
displaced.
or (103)(0.5) = 102 (x) (0.8) g + 102 (6 x) 1 4 cm
or x = 5 cm. x cm
(b) Gauge pressure at the lower face of the block Oil (104x)cm

= 5 0.8 98 + 1 1 980
= 4900 dynes/cm2. water

www.physicsashok.in 23
FLUID MECHANICS

Example 25. A cone of radius R and height H, is hanging inside a liquid of density
by means of a string as shown in the figure. The force, due to the liquid
acting on the slant surface of the cone is (neglect atmosphere pressure)
(A) gHR2 (B) HR2 H
(C) 4/3 gHR2 (D) 2/3 gHR2
R
Sol. Fy = PR2 = Vg = Buoyancy force
1
or Fy + ghR2 = R2hg T
3
Fy
1
Fy = R2hg ghR2
3 Fx
2 Fx
Fy = R2hg
3
P R2
2 mg
Hence Fy = R2hg in downward direction.
3
Hence option (D) is correct.

Example 26. A dumbbell is placed in water of density . It is l


M,V 2M,V
observed that by attaching a mass m to the rod, the dumbbell
floats with the rod horizontal on the surface of water and each
sphere exactly half submerged as shon in the figure. The volume Water
of the mass m is negligible. The value of length l is d
d(V 3M) d(V 2M) d(V 2M) d(V 2M)
(A) (B) (C) (D)
2(V 2M) 2(V 3M) 2(V 3M) 2(V 3M)
Sol. For equilibrium,
V V
Mg + 2Mg + mg = g+ g ...(i)
2 2
Taking torque about mass m,
V V
g Mg d l g 2Mg l ...(ii)
2 2
From (i) and (ii)
d(V 2M)
l Hence option (B) is correct.
2(V 3M)
B
b F
Example 27. A rod of length 6 m has a mass of 12 kg. It is hinged at one end W
at a distance of 3 m below the water surface. C
3m
(i) What weight must be attached to the other end so that 5 m of the rod is O
submerged ? H O
A 12kgf
(ii) Find the magnitud and direction of the force exerted by the hinge on the N
rod. The specific gravity of the material of the rod is 0.5.
Sol. Mass per unit length of the rod is 2 kg. Therefore, mass of the submerged portion of the rod is 10 kg and its
10 3
volume = m
500
www.physicsashok.in 24
FLUID MECHANICS

mass
(using the simple formule, volume = and density = specific gravity 1000 kg m3). Therefore,
density
buoyant force,
10
Fb = 1000 = 20 kgf
500
Let N and H be the vertical downward and horizontal reactions of the hinge on the rod. Considering
horizontal and vertical translational equilibrium of the rod, N + 12 + W = 20 (where W is the weight to be
attached and H = 0).
N + W = 8 and H = 0
Considering the rotational equilibrium of the rod about A
20 g 2.5 cos + 12 g 3 cos + W 6 cos = 0
6W = 50 g 36 g = 14 g
7 7
W g kgf
3 3
7 17
N 8 kgf
3 3

Example 28. A wooden plank of length 1 m and uniform cross-section is hinged at


one end to the bottom of a tank as shown in figure. The tank is filled with water F
C
up to a height of 0.5 m. The specific gravity of the plank is 0.5. l B
Find the angle that the plank makes with the vertical in the equilibrium position. A mg
O
(Exclude the case = 0).
Sol. The forces acting on the plank are shown in the figure. The height of water level is l = 0.5 m. The length of
the plank is 1.0 m = 2l. The weight of the plank acts through the centre B of the plank. We have OB = l. The
buoyant force F acts through the point A which is the middle point of the dipped part OC of the plank.
OC l
We have OA
2 2 cos
Let the mass per unit length of the plank be .
Its weight mg = 2lg.
l
The mass of the part OC of the plank = .
cos
1 l 2l
The mass of water displaced = .
0.5 cos cos
2l g
The buoyant force F is, therefore, F = .
cos
Now, for equilibrium, the torque of mg about O should balance the torque of F about O.
So, mg (OB) sin = F (OA) sin
2l l
or (2 l ) l
cos 2 cos
or cos2 = 1/2
1
or cos = , or = 45
2

www.physicsashok.in 25
FLUID MECHANICS

Example 29. A solid rod of length L cross-sectional area A and density is suspended freely in a wide tank filled
with a liquid of density 0 to a height h = L/ (where > 1). The lower end of the rod touches the base of
the tank.
(i) Determine the angle of inclination of the rod with the horizontal in the equilibrium position.
(ii) Determine the normal reaction between the rod and the base of the tank.
Sol. In the figure shown, let us assume
L = length of the rod
l = immersed length of the rod
= angle of inclination of rod with the horizontal.
(i) If the rod is in equilibrium, then the net torque about the point A is zero, i.e.
L l B
A (ALg) cos (0 Al g) cos 0
2 2 FB
2 h=L/
L 0
or W 0
l2 A

N
h L 1 0
Now, sin
l l
(ii) Using Newtons second law
N = W FB = ALg 0Alg
l
or N ALg 1 0
L

or N ALg 1 0

FLUID DYNAMICS
Blowing air, water flowing in a river, liquid flowing in a tube, blood flowing in vein are examples of fluid
motion.
The study of fluid in motion is known as fluid dynamics.

Types of fluid flow


A fluid element is a small volume of fluid containing a large number of molecules. During motion, this fluid
element may be observed moving in various fastion as decided by the forces acting on it. These define
various types of fluid flow.
Compressible flow is defined as that is which the density of fluid changes during the flow.
Incompressible flow is that in which fluid density does not change during the flow. The flow of liquids is
generally incompressible and that of gases is compressible (at low speeds).
A fluid element may be rotating while flowing. This is known as rotational flow. Irrotational flow is that is
which a fluid element does not rotate while moving. If we look at various points of space in which fluid has
irrotational flow, velocity vectors of fluid elements at various points of space (at a given time) have their line
integral in a closed path equal to zero :

v . dl 0 (irrotational flow)

www.physicsashok.in 26
FLUID MECHANICS

During flow heat exchange or work exchange may take place between fluid and its surrounding. In such a
flow, we define isothermal flow as that in which temperature of fluid element remains the same during the
flow. Adiabatic flow is that in which no heat exchange between fluid element and surrounding is allowed.
Real fluid have fractional motion. The name of fluid friction is viscosity. If viscous forces are
considerable the flow is called viscous flow. If the, viscous force has negligible effect, the flow is idealised
as non-viscous flow.

Streamline flow

Turbulent flow uncertain type of motion

Figure shows water tap turned on slowly. Water flow in upper part looks smooth while in lower part (larger
speed) the flow is not smooth (it is zig-zag). The reason for such a look is that flow at low speed is
streamline while at large speed it is turbulent.

Steady or Streamline flow


The flow of fluid in which every fluid element crosses a given point of space with the same velocity as its
predecessor, is known as steady flow. Thus, at a very given point of space we have a vector of velocity. We
have a velocity field, just like electrostatic fleid or gravitational field.
The path followed by all the particles of fluid passing through V
a point is the same. This path is streamline. hence we
also call the steady flow a streamline flow. A streamline is A
V
analogous to electric lines of force. Velocity at a point is Streamlines
tangent to streamline passing through that point. B
The magnitude of velocity is proportional to the density of streamlines (per unit area normal to velocity
vector). In the figure same number of streamlines are spaced more closely at A than at B and hence velocity
at A is larger then that at B :
V > V
Two streamlines never cross each other because that would mean two directions of velocity at the crossing
point which is not possible in a steady flow. v2
The line integral of velocity in a closed path is zero is steady
flow. The velocity field here is an example of a conservative A2
field. v1
We can always imagine a pipe of flow constituted by
streamlines. Fluid in this pipe flows in a disciplined way, Flow pipe
never crossing its wall. A flow pipe or flow tube is a bundle A1
of streamlines.
The figure shows a flow tube. Its cross-sectional area is non-uniform. In a uniform velocity field, streamlines
are parallel and equispaced. We have in that case cross-sectional area as the same every where for a flow
tube and fluid elements have equal velocity at all sections at a time.

www.physicsashok.in 27
FLUID MECHANICS

In the case of non-uniform steady flow, larger area means rarefied streamlines and hence low speed of
particles. In a flow tube the same number (N) of streamlines are crossing different areas (A). Hence, by
definition of streamlines,
N
V
A
N
Vk
A
VA = kN = constant.
The product of flow tube area and fluid velocity at each cross-section of a flow pipe remains the same. This
is the constant of equation of continuity (describing continuous flow) :
A1V2 = A2V2 = ...........
We can show that equation of continuity is based a conservation of fluid mass during flow. The product Av

is, in fact, volume of fluid per unit time passing through surface normal to V . If we multiply it by density ,
we get the rate of mass flow. If mass is neither created nor destroyed, this rate must be the same. Hence
AV= constant.
For incompressible flow, is also constant. Then AV = constant.
Narrower the flow tube, faster the flow.

C15: From a tube of inner radius 4 cm, water is flowing at an average speed of 2m/sec. Calculate the rate of
flow.
Sol. Q = Rate of flow
Q = volume of liquid flowing per sec. = area of cross-section velocity
Q = A v = r2v
22 2
= (4) 2 100 c.c./sec. = 10053.09 c.c./sec.
7

C16: The rate of flow in a tube of inner radius 4 cm is 36 litre/hour. Calculate the speed of the water in the tube.
Sol. Q = A v = r2v
36 1000 22 2
or (4) v
60 60 7
v = 0.198 cm/sec.

C17: A horizontal pipe having a constriction is shown in the figure. The radius at M and N are respectively 8 cm
and 4 cm. Calculate the velocity at N if velocity of water at M is 16 cm/sec.
Sol. We have A1v1 = A2v2 M
r2 =8cm
N v
2 2
r1 v1 = r2 v2 v r1 =4cm 1
2
2
r
v1 2 v 2 64 cm / sec.
r1

BERNOULLIS THEOREM
Deniel Bernoulli (1700 1782) studied flow of fluids using energy method. If we apply conservation of
energy of steady flow, we get an equation known as Bernoullis equation (representing Bernoullis theorem).
It relates velocity of flow, elevation of flowing fluid and pressure.

www.physicsashok.in 28
FLUID MECHANICS

The sum of kinetic energy per unit volume, gravitational potential energy per unit volume and pressue,
remains constant at all the points of a stream line. This is Bernoullis theorem.

v Stream line
P
Y
Reference land
(y = 0

If a fluid of density be flowing with speed V along a streamline, and P be the pressure and y be height
relative to a reference line, then
kinetic energy per unit volume = 1/2 V2
gravitational potential energy per unit volume = gy.
The sum is gy + 1/2 V2 + p. According to Bernoullis theorem
p + gy + 1/2V2 = constant.
This is Bernoullis equation.
To prove it, we consider a flow pipe and fluid contained in the pipe between sections a and b (see fig.) The
pressure p1 causes a force p1 A1 at section-a and pressure p2 causes a force p2A2 at section-b. Due to
velocity the section-a moves by v1dt and section-b by v2dt. The fluid we considered as (i) in the figure,
acquires the position (f) in time dt.
Work done on the fluid is W = p1A1V1dt p2V2V2dt. p2 A2
This must be equal to change in mechanical energy, E. (i)

Now E = U + K.
b v dt
where U is the change in gravitational energy, and K is 2

a (f)
the charge in kinetic energy of the fluid lying, between
section-a and section-b. a
During motion section-a goes to a and b to b. the fluid p1 A1 a v1 dt
between a and b has the same kinetic energy and potential
energy in the states i and f. Hence
K = (Kab + Kbb ) (Kaa + Kab)
K = Kbb Kaa
1 1
K = (A2V2dt ) V22 (A1V1dt )V12
2 2
U = (Uab + Ubb ) (Uaa + Uab)
U = Ubb Uaa
U = (A2V2dt ) gy2 (A1V1dt)gy1
Therefore using E = W, we have
1 1
(A2V2dt ) V22 (A1V1dt )V12 + (A2V2dt ) gy2 (A1V1dt)gy1
2 2
= p1A1V1dt p2A2V2dt
Now, A1V1 = A2V2
1 1
V22 V12 + gy2 gy1 = p1 p2
2 2

www.physicsashok.in 29
FLUID MECHANICS

Arranging subscripts-1 on RHS and subscripts-2 on LHS,


1 1
V22 + gy2 + p2 = V12 + gy1 + p1
2 2
The sum of pressure (p), kinetic energy per unit volume (1/2 V2) and gravitational potential energy per unit
volume (gy) has the same value at section-1 as at section-2. This is true for any other section also. Area A1
or A2 of the flow tube are small enough so that unique velocity can be taken. Hence the result is true when
pipe becomes very narrow, i.e., a stream line.
The Bernoullis equation
1
p + gy + V2 = constant
2
must be used for ideal flow only. By this we mean incompressible, non-viscous, irrotational and streamline
flow.
In the special case of hydrostatics, (we take v = 0). This gives
p + gy = constant.
As y decreases, p increases, i.e. hydrostatic pressure increases with depth.

C18: The work done in pushing 10 cubic metre of water in a pipe of diameter 8 cm is 16000 joule. Calculate the
difference in the pressure at the two ends of the pipe.
Sol. We have, work done
W = (P1 P2)V
W 16000
P1 P2 1600 N / m 2
V 10

Example 30. Water flows in a horizontal tube as shown in figure. The


pressure of water changes by 600 N/m2 between A and B where
A B
the areas of cross-section are 30 cm2 and 15 cm2 respectively.
Find the rate of flow of water through the tube.
Sol. Let the velocity at A = vA and that at B = vB.
v B 30 cm2
By the equation of continuity, 2
v A 15 cm 2
By Bernoullis equation,
1 1
PA + vA2 = PB + vB2
2 2
1 1 3
or PA PB = (2vA)2 vA2 = vA2
2 2 2
N 3 kg
or 600 2
1000 3 v2A
m 2 m

or v A 0.4 m 2 / s 2 0.63 m / s
The rate of flow = (30 cm2) (0.63 m/s) = 1890 cm3/s

www.physicsashok.in 30
FLUID MECHANICS

Example 31. A liquid of density 0.83 g/cm3 is contained in a vessel under a total internal pressure of 3 atm.
Neglecting viscosity, calculate the speed with which the liquid will escape from a small hole made in the side
wall of the vessel.
1 1
Sol. P1 v12 P2 v 22
2 2
v2
Here P1 = 3 atm v1 = 0
P1 P2
P2 = 1 atm
v1 = 0
2(P1 P2 )
v2

2 2 1.013 105
v2 22.1 m / s
0.83 103

Example 32. A non-viscous liquid of constant density 1000 kg/m3 flows in a


streamline motion along a tube of variable cross-section. The tube is kept
inclined in the vertical plane as shown in the figure. The area of cross-
Q
section of the tube at the points P and Q at heights of 2 metre and 5 metre
P
are respectively 4 103 m2 and 8 103 m2. The velocity of the liquid at 5m
2m
point P is 1 m/s.
Find the work done per unit volume by the pressure and the gravity forces
as the fluid flows from point P to Q. Take g = 9.8 m/s2.
Sol. Given : A1 = 4 103 m2, A2 = 8 103 m2, h1 = 2 m, h2 = 5 m, v1 = 1 m/s and = 103 kg/m3
From continutiy equation, we have
A1v1 = A2v2
A
or v 2 1 v1 A2
A2 v2
A1
4 103 v
or v2 1 m / s 2
8 103 1 h2
h1
1
v2 m/s
2
Applying Bernoullis equation at section 1 and 2,
1 1
P1 v12 gh1 P2 v 22 gh 2
2 2
1
or P1 P2 g(h 2 h1 ) (v 22 v12 ) ...(1)
2
(i) Work done for unit volume by the pressure as the fluid flows from P to Q
W1 = P1 P2
W1 = g(h2 h1) + (v22 v12) [from eq. (1)]

www.physicsashok.in 31
FLUID MECHANICS

1 1
W1 (103 )(9.8)(5 2) (103 ) 1 J/m3 = {29400 375} J/m3
2 4
or W1 = 29025 J/m3 Ans.
(ii) Wor done per unit volume by the gravity as fluid flows from P to Q.
W2 = g(h2 h1) = (103)(9.8)(5 2) J/m3
or W2 = 29400 J/m3 Ans.

Example 33. A siphon tube is used to remove liquid from a container as C


shown in fig. In order to operate the siphon tube, it must initially be filled
H
with the liquid. Datum A
(i) Determine the speed of the liquid through the siphon.
h
(ii) Determine the pressure at the point C.
Sol. (i) Applying bernoullis equation at points A and D, we get B
y
1 1
pA + vA2 + gyA = pD + vD2 + gyD
2 2
Assuming datum for potential energy at the free surface, we have
yA = 0; yD = (h + y); pA = pD = patm vA2 0 ; vD = v
1 2
patm + 0 + 0 = patm + v = g[(h + y)]
2
or v 2g(h y)
(ii) Applying Bernoullis equation at A and C, we get
1 1
pA + vA2 + gyA = pC + vC2 + gyC
2 2
Here, yC = + H ; vC = v (according to the continuity equation)
1 2
patm + 0 + 0 = pC + v + gh
2
or patm = pC + g (h + y) + gh
or pC = patm g (h + H + y)

Example 34. The figure shows a siphon in action. The liquid flowing B C
through the siphon has a density of 1.5 g/cc. Calculate the pressure
difference between (a) points A and D, and (b) points B and C. 20 cm

Sol. (a) At A and D the pressure is atmospheric, therefore, A 200 cm


PA PD = 0
(b) PA PB = 1.5 980 20 dynes/cm2
PD PC = 1.5 980 (200 20) D
As PA = PD ; PA PC
PA = 1.5 980 (200 20)
PA = 26400 dynes/cm2.

www.physicsashok.in 32
FLUID MECHANICS

Example 35. A liquid flows out of a broad vessel through a narrow vertical pipe.
How are the pressure and the velocity of the liquid in the pipe distributed when 1
the height of the liquid level in the vessel is H from the lower end of the pipe
and the length of the pipe is h ?
x
Sol. Let us consider three points 1, 2, 3 in the flow of water. The positions of the 2 h
points are as shown in the figure.
Applying Bernoullis theorem to points 1, 2 and 3
p0 1 2 p2 1
v1 gH v 22 g(h x)
2 2
p0 1 2
v3 0
2
By continuity equation
v1 A1 = A2v2 = A2v3
Since A1 >> A2, v1 is negligible and v2 = v3 = v (say)
p0 p 1 p 1
gH 2 v 2 g(h x) 0 v 2
2 2
v 2gH ...(i)

p0 p
and gH 2 gH g(h x)

p0 = p2 + g (h x)
p2 = p0 g (h x) ...(ii)
Thus pressure varies with distance from the upper end of the pipe according to equation (ii) and velocity is
a constant and is given by (i).

Applications of Bernoullis theorm


A variety of phenomena involving ideal fluid flow, or approximately ideal flow, could be explained using
Bernoullis theorem.

1. Speed of efflux : Torricellis law :


If we puncture an open tank filled with water, (or any liquid) water (with liquid) will flow out of the pin hole.
This hole of very small radius is called on orifice. The speed of outflow is called speed of efflux.
Torricelli observed that speed of efflux at the orifice is equal to the speed of
freely falling stone released from the top of liquid surface. This is known as
Torricellis theorem. If h be the depth of the orifice from free surface, the y=h

speed of efflux will be 2gh . yaxis h

Let us consider a streamline joining orifice to the top of the liquid. Let the y=0
reference level for gravitational potential energy be at the level of orifice. Reference level
Pressure at the top of liquid is atmospheric pressure P0 and also at the
point where orifice is made. The speed at the top is negligible as area here
is very large. Hence
1 2
p0 + gh + 0 = P0 + v .
2
www.physicsashok.in 33
FLUID MECHANICS

Solving it we get
v 2gh .
In certain situations the tank is not open. The pressure above the liquid top is not equal to pressure at the
orifice. In that case
1 2
P1 + gh + 0 = P0 + 0 + v .
2
In rockets, the inside pressure is much larger and the gh - term is ignored.

C19: A tank contains water upto a height H. There is a very narrow hole at a depth h below the level of water
in the tank. At what distance the stream of water coming out from the orifice in the wall will strike the floor
?
Sol. Water coming out from the orifice has a horizontal velocity
v 2gh
Let in time t the water strikes the floor. Then
1
H h 0 gt2
2
2(H h)
t
g
Therefore horizontal distance travelled
2(H h)
x vt 2gh 2 (H h)h
g

C20: In the above problem at what depth the hole should be so that the stream of water coming out from it may
strike the floor at maximum distance from the tank.
Sol. We have, x 2 (H h)h
x2 = 4(H h)h
or x2 = H2 H2 + 4(H h)h
Clearly x will be maximum, if (H 2h)2 = 0
H
Hence, h
2

C21: In the above problem calculate the maximum distance from the tank where the stream of water will strike
the floor.
Sol. We have, x2 = H2 (H 2h)2
Clearly x is maximum,
when (H 2h)2 = 0
Hence maximum value of x = H.

C22: A tank contains water upto height H. What should be the position of two different holes in its wall so that
water stream coming out from them may strike the ground at the same point ?

www.physicsashok.in 34
FLUID MECHANICS

Sol. The height of second hole from the bottom must be equal to the depth of the 1st hole from the level of the
surface.
For same range we have x1 = x2 h
h 1

2 (H h)h 2 (H h1 )h1 H

or H h1 = h h
x1 = x2
C23: There are two narrow holes in the wall of a vessel filled with water at a distance of 50 cm. The area of
cross-section of each hole is 0.2 sq. cm; calculate the distance of the point of intersection of their streams if
140 cm3 of water is filled in the vessel in every second.
Sol. Let M and N be the two holes.
v1 2gh , v 2 2g(H h)
If x is the distance of the point of intersection then,
h M
x = v1t1 = v2t2 V1
It can be shown that H=50cm
V2
1 V2 2
2 2g x
x H 120 cm
2 2g 2 V 2

Example 36. An open tank is filled with two immiscible liquids of densities 1 and
2 as in the figure. Determine the speed of efflux.
Sol. As Torricellis theorem is applicable to open tank, single density system, we 1 h1

shall use Bernoullis theorem for lower liquid. Pressure at the junction of liquids
h2
is P0 + 1 gh1 and presure at the orifice is P0. Hence 2
v
1
(P0 + 1gh1) + 2gh2 + 0 = P0 + O + 2V2
2
2(1gh1 2 gh 2 )
V
2

V 2gh 2 2 1 gh1
2
NOTE : That it depends upon (1/2), whereas in single liquid system it
does not depend upon the type of liquid.

Example 37. A cylindrical tank having cross-sectional area A = 0.5 m2 is filled


with two liquids of density 1 = 900 kg m3 and 2 = 600 kg m3, to a
h ( 2)
height h = 60 cm each as shown in figure. A small hole having area a = 5
cm2 is made in right vertical wall at a height y = 20 cm from the bottom.
Calculate
h ( 1)
(i) velocity of efflux,
F y
(ii) horizontal force F to keep the cylinder in static equilibrium, if it is placed on
a smooth horizontal plane and
(iii) minimum and maximum values of F to keep the cylinder in static equilibrium, if coefficient of friction between
the cylinder and the plane is = 0.01. (g = 10 ms2)
www.physicsashok.in 35
FLUID MECHANICS

Sol. Since area of hole is very small in comparison to base area A of the cylinder, therefore, velocity of liquid
inside the cylinder is negligible.
Let velocity of efflux be v and atmospheric pressure P0.
Consider two points A (inside the cylinder) and B (just outside the hole) in the same horizontal line as shown
in figure.
Pressure at A, PA = P0 + h2g + (h y)1g
Pressure at B, PB = P0 2
According to Bernoullis theorem,
pressure energy at A
PA = pressure energy at B + kinetic energy at B 1
B
1 2
A
PA = PB + 1v
2
1
v = 4 ms Ans.
When cylinder is on smooth horizontal plane, force F required to keep cylinder stationary equals horizontal
thrust exerted by water jet. But that is equal to mass flowing per second x change in velocity of this mass.
F = (av ) (v 0) = av2 mg
or F = 7.2 N Ans.
Total mass of the liquid in the cylinder is
m = Ah1 + Ah2 = 450 kg Fmax
Limiting friction = mg = 45 N 7.2 N
F < Limiting friction, therefore, minimum
force requied is zero. Ans. N
Consider free body diagram for maximum value of force.
Considering vertical forces, N
N = mg
Now considering horizontal forces,
Fmax = F + N
or Fmax = 52.2 N Ans.

Position of Liquid level in tank


Let A be the area of the top of liquid and y be its position relative to the hole. Then in a time dt, the level will
fall by | dy |, expelling a volume A | dy | through the hole. If x be area of the hole andy V be the velocity of
efflux, volume expelled in time dt is Vdt. Hence
A | dy | = V dt ...(1)
using Torricelis theorem (for << A), we have |dy|
y
V 2 gy ...(2)

Thus, A | dy | = 2 gy dt
As y is decreasing,
| dy | = dy ...(3)
Thus, A dy = 2 gy dt
dy
2g dt
y A
Integrating LHS from y = H to y and t = 0 to t, we get

www.physicsashok.in 36
FLUID MECHANICS

y t
dy
y

A
2g dt
H 0


2 y H A
2g t

g
H y t
A 2

g
t
A 2
This gives position y of the equal level of time t.
Solving this
A 2
t
g
H y
Example 38. A tank of cross-sectional area A filled with a liquid to a height H has A
a small hole a its base of area a (a << A). Find the time during which the liquid
H
level falls to a height h = H/ (where > 1)
Sol. Let h be the instantaneous height of liquid in the tank. Then, the instantaneous
velocity of flow through the hole is a
v 2gh
Using the equation of continuity, we have
dh
A av a 2gh
dt
H/ t
dh a
or
h A
2g dt
h
H 0

H a v
or 2 H 2gt
A

A 2 H
or t H
a g

Example 39. A cylindrical tank of base area A has a small hole of area a at the bottom. At time t = 0, a tap
starts to supply water into the tank at a constant rate m3/s.
(a) what is the maximum level of water hmax in the tank ?
(b) find the time when level of water becomes h(< hmax).
Sol. (a) Level will be maximum when rate of inflow of water = rate of outflow of water
i.e. = av
or a 2gh max

www.physicsashok.in 37
FLUID MECHANICS

m3
2 Rate = s
h max
2ga 2
(b) Let at time t, the level of water be h. Then A

dh
A a 2gh
dt h

h dh t dt v = 2gh
or
0 a 2gh

0 A
a

Solving this, we get


A a 2gh
t ln 2gh Ans.
ag a

Example 40. Curved surface of a vessel has shape of a truncated cone having semi-vertex angle = 37. Top
and bottom radii of the vessel are r1 = 3 cm and r2 = 12 cm respectively and height is h = 12 cm. The vessel
is full of water (density = 1000 kg m3) and is placed on a smooth horizontal plane in vacuum. Calculate
(i) mass of the liquid in the vessel,
(ii) force on the bottom of the vessel.
(iii) resultant force on curved walls.
A hole having area S = 1.5 cm2 is made in curved wall near the bottom.
Calculate
(iv) velocity of efflux,
(v) horizontal range of water jet, and
(vi) horizontal force required to keep the vessel in static equilibrium. Neglect atmospheric pressure.
(g = 10 ms2)
Sol. The vessel is made from a cone having axial length (h + h
= 16 cm) and semi vertex angle 37 by cutting top 4 cm of
h = 4 cm
37

its length as shown in figure. Therefore, volume of vessel is


1 1
V= r22 (h + h) r12h
3 3 h=12cm r1 =3cm
3
V = 756 cm
or V = 756 106 m3
Mass of water in the vessel is M = V
r2 =12cm
M = 0.756 kg Ans. (i)
Force on bottom of the vessel is due to pressure exerted by water and this pressure is
P = hg = 1200 Nm2
Area of the bottom is A = r22 = 0.0144 m2
Force on bottom is F = P.A = 17.28 N Ans. (ii) mg
Let resulant force exerted by curved walls on water F
body be F (vertically down-wards)
Considering free body diagram of water body (figure).
F + mg = F
or F = F mg F
F = 9.72 N

www.physicsashok.in 38
FLUID MECHANICS

According to Newtons third law, water exerts an equal but opposite force on vessels curved walls.
Hence, this force is numerically equal to F but upwards.
Resultant force on curved walls
= 9.72 N (upwards) Ans. (iii)
Velocity of effulx is v0 = 2gh (Normal to curved walls)
v0 = 2.4 ms1 at angle 37 to horizontal Ans. (iv)
Now water molecules moves under gravity like a free projectile.
v 20 sin 2
Hence, horizontal range of water jet is R =
g
or R = 0.2304 m or 23.04 cm Ans. (v)
Vessel experiences a force at hole, due to rate of change of momentum of water. This force is equal to
(Mass flowing per second) (change in velocity of this mass)
Hence, this force is equal to (Sv0) (v0 0) = S . v02 = 0.36 N (normal to the wall)
Its vertical component is balanced by floor reaction, while horizontal component (0.36 cos ) is to be
balanced by applying a horizontally rightward force on vessel.
Horizontal force required to keep the vessel stationary is
0.36 cos = 0.288 N Ans. (vi)

2. Venturimeter :
The flow speed of incompressible fluid (or ideal fluid) can be measured using a hollow tube having broad
diameter (section) and a small constriction (section-2). These two regions are connected by a manometer.
The whole arrangement is shown in the adjoining figure.

A
a
V0
V

2 1
1
density =
h
density =
manometer

Let be the fluid density, V0 be the flow velocity. We insert veturimeter with horizontal axis. The flow
velocity near the broad ends is almost V0 but at constriction (a narrow opening) the speed is larger (V). The
pressures at section-1 and section-2 are P1 and P2. Then using Bernoullis theorem,
1 1
P1 + V02 = P2 + V2.
2 2
Equation of continuity gives
AV0 = aV ...(1)
A
V V0 ...(2)
a
The pressure difference P1 P2 is measured by manometer. It equals gh where is density of liquid in the
manometer. That is,
www.physicsashok.in 39
FLUID MECHANICS

P1 P2 = gh ...(3)
From equations (1), (2) and (3) V0 can be calculated.
1
we have, P1 P2 = (V2 V02)
2
2
1 A
P1 P2 V0 V02
2 a

1 A2
P1 P2 V02 2 1
2 a

1 2 A2
gh V0 2 1
2 a

2( / )gh
V0
A2
a 2 1

Thus, flow speed V0 could be determined.

Example 41. The figure shows a venturimeter tube. It is a device A1


A2
used to measure the velocity of a fluid through a pipe. The
cross-sectional area of the pipe at positions 1 and 2 are A1 1 0 2

and A2 respectively. An U-tube manometer with a fluid of


density is used to measure the pressure difference between
the points 1 and 2. If the density of the flowing fluid be 0, h
then determine the velocity of flow at position 1.
Sol. Using the equation of continuity at position 1 and 2, we get
A1
A1v1 = A2v2 or v2 = A v1 ...(1)
2
Applying Bernoullis equation at position 1 and 2, we get
1 1
p1 + 0v12 + 0gy1 = p2 + 0v22 + 0gy2
2 2
Assuming datum for potential energy at the axis of the pipe, we get
y1 = y2 = 0
2(p1 p 2 )
v22 v12 = 0
Substituting the value of v2, we get
A 2 2(p p )
2
v 1 1
1
1 2
...(2)
A 2 0

www.physicsashok.in 40
FLUID MECHANICS

The pressure difference (p1 p2) may be obtained by writing the manometric equation.
p1 + 0gh gh = p2
or p1 p2 = ( 0)gh
Substituting the value of (p1 p2) in equation (2), we get
2

2 A1
v1 1 2gh 1
A 2 0


2gh 1
0
v1
or A 2
1 1
A 2

Example 42. The area of cross-section of a large tank is 0.5 m2. It has an opening near the bottom having area
of cross-section 1 cm2. A load of 20 kg is applied on the water at the top. Find the velocity of the water
coming out of the opening at the time when the height of water level is 50 cm above the bottom. Take g = 10
m/s2.
Sol. As the area of cross-section of the tank is large compared to that of the opening, the speed of water in the
tank will be very small as compared to the speed at the opening. The pressure at the surface of water in the
tank is that due to the atmosphere plus due to the load.
(20kg)(10m / s2 )
PA P0 2
P0 400N / m 2
0.5m
At the opening, the pressure is that due to the atmosphere.
Using Bernoullis equation,
1 1 A
PA gh v 2A PB v 2B
2 2
or P0 + 400 N/m + (1000 kg/m3) (10 m/s2) (0.2m) + 0
2

B
1
= P0 + (1000 kg/m3) vB2
2
or 5400 N/m2 = (500 kg/m3) vB2
or vB 3.3 m/s.

Some applications of Venturimeter


Carburetor of an automobile uses a venturi channel (nozzle) for mixing air with gasoline (petrol) in correct
ratio for combustion. We know that higher the speed smaller the pressure along a horizontal streamline.
Hence by forcing air flow through nozzle, low pressure is generated. This cause petrol to be sucked upward
and mix with flowing air.
nozzle
motion air + petrol
air

petrol

www.physicsashok.in 41
FLUID MECHANICS

A spray gun, a filter pump, a Bunsen burner or an atomiser use the principle of venturimeter. Flowing fluid
generates low pressure which is used to such fluid and mix it with the flowing fluid.

The blood pressure (B.P.) :


The flow of blood and its the pressure are better understood by applying Bernoullis theorem.
Circulatory system in animals are tubes through which fluid called blood is circulated by a pumping system
called heart. In some animals like human or giraffe, the flow can take place against gravity while in some
others like snakes, rats, rabbits, the flow against gravity is not possible by venous system. Arteries are tubes
through which the blood flows.
Arteries may get constricted (narrow boring) due to deposition of plaque on inner walls. In order to force
the blood in correct amount through these narrowed tubes, velocity of flow has to be increased (product of
area and velocity is the volume rate of flow, which will be constant if narrower tubes have larger flow
speeds). Higher the flow speed, lower the fluid pressure (Bernoullis theorem). The viscous effect is masked
by the flow speed and arteries with constriction attain low pressure.
But arteries are surrounded by other materials that press it to balance the inside normal pressure. Now that
pressure inside has decreased while outside pressure is the same, a mechanical constriction is produced.
The heart has to exert more to open the artery and hence activity of heart suddenly increases. This may lead
tomalfunctioning of the heart, heart attack and even death. By chemical means we try to dissolve plaque or
take such a food and do exercises as to reduce the deposition of plaque.
Blood pressure in arteries is an indicator of how healthy the arteries are. The pressure at various points of
arteries in the body is not indentical; it depends on posture.
Figure shows average guage pressure (above atmospheric pressure) averaged over one cycle action of
heart.

9.3 kPa Brain

13.3 kPa Heart


= 100 torr
= 100 mm of Hg
y

26.8 kPa Feet

Bernoulli's theorem gives


1
P + V 2 + gy = constant
2
Here V constant. Hence higher the
point, lower the pressure.

13.2 kPa 13.3 kPa 13.1 kPa

www.physicsashok.in 42
FLUID MECHANICS

Blood pressure can be measured by an instrument called sphygmomanometer, usually available with a
medical practioner.

Dynamic Lift :
If the flow of fluid across a body is such that pressure across it become unequal, a net force is generated
from region of high pressure to low pressure. This force may lift a body against gravity. It is called dynamic
lift. Such a lift if observed, for example, in the following situation.
(a) A ball moving with spin feels a sideways force. This is called Magnus Effect after Gustav Magnus who
studied the effect in 1850s.
(b) Airplane wing or hydrofoil moving in fluid feels a force normal to wing.
(c) Rotating wing of helicopter gets lifteed.
To see how Magnus Effect arises, we use Bernoullis equation to fluid in the vicinity of a ball in motion with
spin.

FD
v

Ball in motion without spin Fluid motion relative to ball.


Let us first consider a ball moving without spin. The fluid moves past the ball with equal speed as seen from
balls frame. Only a dragging (FD) acts on the ball.
Now we consider spin also. The air in contact with the surface of the spinning ball (boundary layer) is
dragged along with the surface. As a result, the relative speed of air past the ball is reduced on one side and
increased on the other side.

v Low speed

v FD
FL
FL
High speed
Streamlines get denser on one side
Spin causes dragging of
fluid in contact

According to Bernoullis theorem, lower speed means higher pressure. Hence a force (FL) acts on the side

from low speed to high speed region. This force is along v .
High speed
In a cricket match or base ball match, one enjoys every much the
motion of spinned ball that poses trouble for batsman. Magnus effect FL
is central to curving of the ball in flight.
The force of dynamic lift acting on the wing of an aeroplane or
helicopter arises due to shape of wing. Low speed
The wings section is such that speed of wind past the wing is larger Dynamic lift on a wing or airfoil
on upper side than lower side. This causes an upward lift (FL).

www.physicsashok.in 43
FLUID MECHANICS

Example 43. Study the structure in the figure. Air is blown in the tube T air in
which outlets radially between discs D and D. As a result, D is T
being pulled upward. If the flow speed in the gap between disks is
v, what is the dynamic lift on D ? Area of D is and density of air
is .
Sol. Let P be the pressure in the gap. Then, using Bernoullis theorem, D
air out
1 V
P + v2 P0 + 0
2 D
1 2 Disc being lifted
P0 P = v ...(i)
2
The net force due to fluid on disk D is (P0 P . ) upward. That is, P
FL = (P0 P) ...(ii)
From (i) and (ii) D
1 2
FL = v
2
P0
Here we have ignored variation of v with radial distance.

FLOW OF VISCOUS FLUID


Uptill now we studied fluid in motion without frictional effects. This was an ideal case just like motion of a
block on a smooth table. Now we consider real fluid motion which involves an important property of fluid-
viscosity.

Viscosity :
Whenever one layer of a fluid moves past another layer of a fluid or solid, there arises a force parallel to
contact layers that opposes the relative motion. This force is known as viscous force and the property of
fluid is called viscosity.
If we put water in one beaker and honey in another, stir them and leave to themselves, honey comes to rest
first. This is because honey is more viscous than water.

Co-efficient of Viscosity : Let us consider a fluid layer moving with velocity v.


Z

F v

The layer in contact with it exerts a tangential force, the lower layer retarding and upper layer accelerating.
the net force F is of retardation.
In the figure, velocity v is a function of z where Z-axis is normal to flow. The quantity dv/dz is known as
velocity gradient along zaxis.
The force of viscosity is proportional to area of the layer and gradient of velocity in a direction normal to
flow.

www.physicsashok.in 44
FLUID MECHANICS

The force of viscosity is written as


Fvis S
dv

dz
dv
Fvis S
dz

It acts oppsoite to v . The proportionality constant (eta) is known as co-efficient of visocity or simply
viscosity. The above rule is valid for steady flow and was observed first by Newton. The fluid obeying it
is known as Newtonian fluid.
Dimensions of : Using the above result,
LT 1
MLT L
2 2

L
[] = [ML1 T1]
The dimensions of are 1 in mass, 1 in length and time and zero in the remaining base quantities.
Units : C.G.S. unit of is g cm1 s1 which is called poise (P).
1 P = 1 g cm1 s1.
The S.I. unit of is kg m1 s1 which is called (pascal second) or poiseuille (Pl)
1 Pl = 1 Pa s = 1 kg m1 s1 = 10 P.

C24: A horizontal plate of area 100 sq. cm rests on a layer of oil of thickness 4 mm; whose coefficient of
viscosity is 4 poise. What tangential force is required to move the plate with a constant speed of 10 cm/sec.
dv 10
Sol. F A 4 100 10 4 dyne
dx 0.4

STOKES LAW
It gives us viscous drag acting on a sphere moving through a fluid having infinite extensions (wall containing
the fluid is far away from the sphere).
As a sphere moves in a fluid, the boundary layer is carries with it. This sets up relative motion between
layers of fluid. Thus arises a velocity gradient near the sphere and viscous force acts on the sphere.
Sir George stokes (1819-1903), a British scientist, found that the viscous force on a sphere of radius r
moving with velocity v relative to fluid of viscosity co-efficient is given by
F = 6 rv
The direction of the force is opposite to velocity. Hence in vector from

F 6 rv
This is known as Stokes law.

C25: A sphere of radius r and mass m is thrown with velocity v0 in a viscous fluid of viscosity . How long does
the sphere move ? Consider only viscous force acting on the sphere.
Sol. Using Newtons second law of motion,
dv
m 6rv
dt

www.physicsashok.in 45
FLUID MECHANICS

dv 6 r
dt
v m
Now we integrate LHS from v0 to v and RHS from t = 0 to t.
v t
dv 6 r
v m dt
v0 0

v 6 r
ln t
v0 m
v = v0 ekt,
6r 6r 9
where k
m 4 / 3r 2 r 2
3

The velocity decreases exponentially with time; the sphere stops only after infinite time.

Terminal velocity of a ball : Stokes Formula


A ball of radius r and material density is dropped in a fluid of density under gravity. The forces acting on
the ball are :
(a) Gravitational pull, mg, acting downward;
(b) buoyant force, (m/)g, acting upward; and
(c) viscous force, 6rv, acting opposite to velocity.
Using Newton second law of motion,
ma x Fxi
i

dV m
m mg g 6rV
dt p

dV 9V 4r 3
g 1 m
dt 2r 2 3

6 rV
m g

V mg x

The acceleration decreases as speed v increases. When the speed grows to such a value that acceleration
becomes zero, the ball falls with constant velocity. That velocity is called terminal velocity. If VT be the
terminal velocity, then
9
0 g 1 VT
2 r 2

2( )r 2g
VT
9
This is known as Stokes formula.
It can be used as a laboratory method for determing viscosity of think liquids using steel balls of suitable

www.physicsashok.in 46
FLUID MECHANICS

radius. You can also measure terminal velocity of water drops falling through a long glass cylinder containing
mustard oil.

Some Speeds
2( )r 2 g
Terminal speed VT
9
Re
Steady flow speed V= (l = r or 2r)
l
(V VC) (Re = Reynolds Number)
r2
Speed profile (Poiseuille flow) V V0 1 2
R

Example 44. A flat plate of area A = 0.1 m2 is sliding on a oil-film of thickness 1 105 m with a speed of
1 mm s1. Find the force F maintaining the speed of co-efficient of viscosity of oil is 1.5 kg m1 s1.

A F

oil film

Sol. Here thickness is small so that dv/dz


1 103 0
(ms1/m) = 102 s1.
1 105
= 1.5 kg m1 s1 v
A = 0.1 m2
Fvis A
dv F
F = Fvis = A
dz dz
F = 0.1 1.5 102 = 15 N v=0

Example 45. A spherical ball of mass 2 gm and having radius 0.6 cm takes 5 seconds to fall steadily through a
height of 40 cm inside a long column of liquid of density 1.2. Calculate the cofficient of viscosity of the liquid.
Sol. Here density of the ball
m m 23
3
2.2 gm / cc.
V 4 / 3r 4 22 / 7 (0.6)3

2 r 2 ( )g 2 r 2 ( )gt
Now, . . [ s vt]
9 v 9 s
2 (0.6)3 (2.2 1.2)980 5
5.88 poise
9 40

www.physicsashok.in 47
FLUID MECHANICS

Example 46. Two equal drops of water each of radius r are falling through air with a steady velocity of 16 cm/
sec. The two drops combine to form a big drop. Calculate the terminal velocity of the big drop.
Sol. Since volume remains constant, hence,
4 4
R 3 n r 3
3 3
where n = No. of small drops, r = radius of small drop and R = radius of bid drop.
R = n1/3 r
If v1 and v2 are the terminal velocities of the small and big drops respectively, then
2 2
v2 R r
n1/ 3 n 2 /3
v1 r r
v2 = n2/3v1 = 22/3 16 cm/sec.

Example 47. Two drops of equal size have terminal speeds 10 cm s1 in air. What will be the terminal speed if
they coalesce ?
Sol. Using Stokes formulae,
2
VT ( ) r 2g /
9
2
VT r
Then
VT r
For two drops forming a single drop
4 3 4
r 2 r 3
3 3
1/3
r = 2 r
VT = 32/3 VT = 10 22/3 cms1
1/ 3
2/3 1/3 1/3 1/3 1
1
Now 2 =4 = (8 4) =8
2
1 5
2 1 2 1.67
6 6
1
VT = 16 cms

Example 48. In Millikans oil drop experiment, an uncharged drop of radius 2.0 105 m is falling with terminal
velocity in air. The density of air is negligible relative to oil density, 1.2 103 kg m3. The viscosity co-
efficient for air at the temperature of working day is 1.8 105 Pa s.
Determine : (a) the terminal speed of the drop and (b) the viscous force acting on the drop.
Sol. The terminal velocity as given by
2
(a) VT ( ) r 2 g /
9
As << , we have
2
VT r 2 /
9

www.physicsashok.in 48
FLUID MECHANICS

2 1.2 103 (2.0 105 ) 2 9.8


VT 5
5.8 cm s 1
9 1.8 10
(b) The viscous force is given by Stokes law.
Thus, F = 6 rv
F = 6 3.14 (1.8 105 Pa s) (2.0 105 m) (5.8 cm s1)
F = 3.9 1010 N
As another method we may have also used the equilibrium condition that weight is balanced by viscous
force. Hence
viscous force weight of drop = 4/3 r3g
= 4/3 3.14 (2.0 105)3 (1.2 103) 9.8 N
= 3.9 1010 N.

SURFACE TENSION AND SURFACE ENERGY


The surface of a liquid open to air, vapour or gaseous system is known as free surface. It is composed of
a few layers of molecules of the liquid. Every liquid has a tendency of minimising the area of its free surface.
This property is known as surface tention.
Different liquids have different extent of the tendency of minimising the free surface area. A measure of how
strong is this tendency is defined below.
(a) In terms of molecular force : Draw a line in the free surface.
There are liquid molecules lying in the free surface on either side Free surface
of this line. Let molecules on one side exert a force F on the
Line in the
molecules on the other side, along the surface. Divide this force free surface
by the length of the line, l. The quantity F/l is the quantitative
measure of strength of stretching force that tends to contact the
free surface area. This quantity also is called surface tension, S. Vessel Liquid
Thus, the force per unit length of a line in free surface, exerted wall
by free surface molecules on one side of the line on those on the other side is known as surface tensions, S.
F
S
l
It is a scalar quantity as F is the magnitude of the force.
Its SI unit is Nm1 and C.G.S. unit is dyne cm1. Its dimensional formula is [ML0T2].
movable wire
(b) In terms of surface energy : Let us consider a wiere frame and a
movable wire forming a loop. Let a liquid film fill up the frame. The film l
has two free surfaces. Each free surface exerts a force F on length l of Wireframe F
2F
wire. The two free surfaces exerts an equal force F and displaces the
wire quasi-statically by dx.
Liquid film dx
The work done by agent is stored in the free surface created. The area
of created free surface is l dx on one side. Hence total free surface area created is 2l dx. The work done by
agent is Fdx.
Fdx F
The free surface, per unit of its area, has stored an energy of .
2 l dx 2 l
But F is 2F. Hence the stored energy per unit area is F/l, which is S, the surface tension. Thus, surface
tension of a liquid is the stored energy per unit area of the free surface of the liquid.

www.physicsashok.in 49
FLUID MECHANICS

The above experiment is usually performed in thermal contact with environment. Surface energy increases
at the cost of kinetic energy of body molecules coming to surface. There will be cooling. Flow of energy to
liquid from surrounding maintains the temperature to normal value.

C25: A horizontal wire of length 20 cm is kept on the surface of water. It is observed that a force of 288 10
4
newton in addition to its weight is required to pull the wire up. Calculate the force of surface tension.
Sol. Here the total length = 20 + 20 = 40 cm.
F 288 10 4
Hence, T 72 103 N / m
L 0.4

C26: What should be the maximum diameter of a wire of length 1 metre and density 6 103 kg m3 which can
float horizontally on the surface of a liquid of surface tension 80 103 N/m (neglect the buoyancy effect)
2
d
Sol. Here Lg T 2L
2
8T 8 80 10
d 1.86 10 3 metre.
g 22
6 103 9.8
7

C27: Explain the following : A thin steel needle floats on water but when a little soap solution is carefully mixed
with the water the needle sinks.
T A T
Sol. When a needle is placed gently on the surface of water, as no part of it is
submerged in water, it is not buoyed up by Archemedes principle. There is
of course on way for steel to float by Archimedes principle. Since its density
is greater than that of water so it will sink if submerged. mg
It is actually kept afloat by the surface tension of water. When it is placed
on the surface of water, it depresses the surface of water slightly due to its weight. The stretched surface due
to surface tension exerts a restoring force, the vertical component of which can maintain equilibrium with the
weight of the needle.
However, when a detergent is added to water its surface tension will suddely decrease and the force due to
surface tension is no longer sufficient to support the weight of the needle and so it will sink.

Example 49. A ring is cut from a platinum tube of 8.5 cm internal and F
8.7 cm external diameter. It is supported horizontally from a pan of F
a balance so that it comes in contact with the water in a glass vessel.
What is the surface tension of water of an extra 3.97 g weight is T T
required to pull it away from water ? (g = 980 cm/s2) Cross
Section
Sol. The ring is in contact with water along its inner and outer
circumference; so when pulled out the total force on it due to surface
tension will be
F = T (2r1 + 2r2)
mg
So T
2 r1 r2 [as F = mg]
3.97 980
i.e., T= = 72.13 dyne/cm
3.14 8.5 8.7

www.physicsashok.in 50
FLUID MECHANICS

Surface Energy
Liquid surface in contact with vapour is usually called a free surface. It is in fact an interface of liquid and
vapour. The surface in contact with solid is also an interface of solid and liquid. The molecules in the surface
interact with both similar and dissimilar molecules. The forces between similar molecules use cohesive
forces and those between dissimilar molecules are adhesive forces. If one removes a molecule from a
interface to infinity, one has to spend an energy of vaporization, equal
to work done against the attractive forces of cohesion and adhesion.
What happens when a molecule in the body of liquid comes to the Fr
surface? To see it let us inquire into its surround in the two states.
The molecular force varies with distance and vanishes if distance
between molecule (r) is large (see figure). r B R
The molecule A is fixed at the origin and distance of molecule B from r
A r0 F
A is r. At r = r0 the milecules are at equilibrium distance having minimum
potential energy of molecular force.
For r < r0, repulsion and for r > r0 attraction results. We can draw a sphere around A of radius R such that
r > R means negligible force. This sphere is called sphere of influence. A molecule within the sphere of
influence interacts with A while that outside it does not.
A molecule in the body of liquid (well below the surface) is surrounded by almost equal density of similar
molecules. The resultant cohesive force is almost zero. This is the rate of that molecule also whose sphere of
influence is almost touching the free surface [Fig (a) and (b)].

(a) (b)
As the milecule rises towards the surface, the sphere of influences rises above the surface. The number of
molecules in the upper hemisphere becomes less than those is the lower hemisphere [Fig (c) and (d)]. The
molecule feels an inward force against which it has to rise.

(c) (d)
It is will lose kinetic energy and average kinetic energy per particle will fall; cooling will occur. Thus if we
increase the free surface area of a given volume of liquid, molecules in the body are brought to surface losing
the kinetic energy in the form of energy of potential form called surface energy. This increment in free surface
area is attended by fall in internal kinetic energy (and hence cooling) and increase in surface energy.
The surface energy is the result of intermolecular forces and depends upon the nature of molecules (host
and foreign) and their spacing (temperature).
It is associated with free surface of solids also. In the figure molecule 1


A of medium1 has a stored energy like the molecule B of medium2. r1



A


When considering phenomena on the interface between various media,
r2
we must keep in mind the surface energy of a liquid as well as solid- B
liquid and solid-gas interfaces. 2

www.physicsashok.in 51
FLUID MECHANICS

C28: PQRS represents a frame which is kept vertically. Over this frame a rod MN Q R
of length 20 cm can slide as shown. The whole system is dipped in a soap
solution and then withdrawn. What should be the mass of the bloock C which
has to be suspended from the wire in order to keep it in equilibrium ? M N
3
[S.T. of soap solution = 30 10 N/m; neglect friction]
P C S
Sol. Here the force exerted by the S.T. = 2TL
For equilibrium of the rod, Mg = 2TL
Where M is the mass of the block C.
2TL 2 30 103 0.2
M 1.22 103 kg
g 9.8

Example 50. A drop of radius 4 cm is broken into 125 equal small drops. Calculate the work done if surface
tension of water is 75 103 N/m.
Sol. Let us suppose that,
R = Radius of the big drop, and
r = radius of the small drop.
Since volume remains constant, V = nV1
4 4
R 3 n r 3
3 3
1
3
Rn r
Hence, increase in surface area
R2 1/3

= 4r n 4R = 4 2/ 3 n 4R2 = 4R2 n 1
2 2
n
Therefore work done

4R 2 n1/ 3 1 T

4 R 2 (5 ) 1 T
3 1/3

= 4R2[5 1]T = 16 R2T

C29: Calculate the work done in breaking a water drop of radius 4 cm into 64 small drops, surfae tension of
water is 75 103 N/m.
Sol.
W 4R 2 n1/ 3 1 T 4R 2 (43 )1/ 3 1 T
2
4
2
12R T 12 75 103 4.52 10 3 joule
100

C30: Calculate the work done in breaking a soap-bubble of radius 10 cm into 512 small bubbles; S.T. of soap
solution is 30 103 N/m.
Sol.
W 2 4R 2 n1/ 3 1 T
W 2 4R 2 83 1/3

1 T 56 R 2 T

www.physicsashok.in 52
FLUID MECHANICS

C31: Calculate the surface energy released if eight small drops of water each of radius 6 cm combines to form
a big drop.
Sol. Let r and R be the radius of small drop and big drop respectively then as the volume remains constant,
4 4
R 2 n r 3
3 3
R n1/3 r
Total surface area of n small drops = 4 r2n
and surface area of big drop = 4R2
Total release of surface energy
= [4r2n 4R2]T = 4[r2n n2/3r2]T
= 4r2[n n2/3]T = 4r2[8 (23)2/3]T
= 4r2[8 4]T = 16 r2T

Example 51. The surface tension of water is 0.073 Nm1. 8 drops of water, each of radius 1 mm, merge to form
a single drop. Determine the change in internal energy of the drops.
Sol. Surface energy lost due to merging goes into the form of change in internal energy.
Surface energy befoce the merging = 8 4 (1 mm)2 0.073 (Nm1).
The eight drops merge to form a single drop.
4 4
Hence, 8 (1 mm)3 = r3
3 3
r = 2 mm
New surface area = 4(2mm)2.
Surface energy after the merger
= 4(2mm)2 0.073 Nm1
Lost surface energy
= [8 4 (1 mm)2 0.073 (Nm1)] [4 (2 mm)2 0.073 Nm1]
= 4 0.073 [8 106 4 106] J
= 4 0.073 4 106 J = 3.67 106 J
The change in internal energy is, thus, 3.67 103 J

EXCESS PRESSURE IN A LIQUID DROP


liquid drop is spherical when effect of gravity is masked by surface tension effect, or when it is in gravity
free space.
Let P be the pressure inside the drop and P0 outside the drop.

P0 P0 ds

P P R2 P0 R2

Sdl
Liquid Drop

We consider equilibrium of half drop (hemisphere) under the forces listed below :
(a) The liquid on the lift exerts normal contact force on the right part, equal to P R2.
www.physicsashok.in 53
FLUID MECHANICS

(b) Air on the right exerts normal contact force P0 ds at each area element ds, whose resultant is P0 R2,
acting leftward.
(c) The free surface of left hemisphere pulls the free surface of right hemisphere at the contact circle with
leftward force S dl over length dl of the circle whose sum over the whole circle is S 2 R.
Thus, balancing these forces leads to
S 2R + P0 R2 = P R2
2S/R + P0 = P.
We see that pressure P is greater than outside pressure. The excess pressure P P0 is given by
P P0 = 2S/R
This is true also for an air drop enclosed in a liquid.

BUBBLE
A bubble has two free surfaces, external and internal. Thickness of liquid film is negligible relative to radius
of the bubble. Hence we take R as radius of both the outer and the inner surfaces.

P0 P R2 P0 ds

Sdl
P
outer Pds
free surface

inner free surface Liquid

The equilibrium of half bubble is under the forces


(a) Resultant of Pds forces, which equals PR2 (right)
(b) Resultant of P0ds forces, which equals P0 R2 (left)
(c) Resultant of S dl forces on two free surfaces, which equals 2 2rS.
Balancing these
P R2 = P0 R2 + 4R2S
P P0 = 4S/R
The exces pressure is twice that in a drop.

C32: Compare the state of a soap-bubble with that of a rubber balloon in following respects : (a) Has each
surface tension ? (b) Does the surface tension depend on area ? (c) is Hookes law applicable ?
Sol. (a) The soap bubble (having two free surface of a liquid) has surface tension while the balloon being made
of rubber has no surface tension but tension due to elasticity.
(b) Surface tension is independent of the area of the soap film while the tension in the balloon is proportional to
stretch, i.e., change in area.
(c) As in case of bubble surface tension is independent of stretch, Hookes law is not applicable.
However, in case of balloon as tension arises due to elasticity and is proportional to stretch, Hookes law is
applicable.
C33: Calculate excess pressure inside a soap bubble of radius 4 cm; surface tension of soap solution is
30 dyne/cm.
Sol. Excess pressure
4T 4 30
P = 30 dyne/cm2.
r 4

www.physicsashok.in 54
FLUID MECHANICS

Example 52. The lower end of a capillary tube of diameter 2.00 mm is dipped 8.00 cm below the surface of
water in a beaker. Find the pressure required in the tube in order to blow a hemispherical bubble at its end
in order to blow a hemispherical bubble at its end in water. Also calculate the excess pressure. Surface
tension of water is 7.30 102 N/m and atmospheric pressure is 1.01 105 Pa.
Sol. The excess pressure in a bubble of gas in a liquid is 2S/r, where S is the surface tension of the liquid-gas
interface and r is the radius of the bubble. The pressure outside the bubble equals the atmospheric pressure
plus the pressure due to 8.00 cm of water column :
pout = 1.01 105 + 0.08 1000 9.8
pout = 1.01784 105 Pa
The pressure inside the bubble is
pin = pout + 2S/r
Pin = 1.01784 105 + (2 7.3 102/107)
Pin = (1.01784 + 0.00146) 105
Pin = 1.02 105 Pa.
Here the radius of the bubble is taken to be equal to the radius of the capillary, since the bubble is
hemispherical.
Excess pressure inside the bubble
= 0.00146 105 Pa = 146 Pa.

Example 53. A mercury drop of radius 1 cm is sprayed into 106 drops of equal size. Calculate the energy
expended. Surface tension of mercury is 35 105 N/m.
Sol. Let R be the radius of the big drop and r be the radius of a small drop. Since the total volume remains
conserved on the formation of small drops.
4 3 4
r 106 R 3
3 3
2
or r = R 10 = 0.01 cm
Energy expended = Surface tension Increase in surface area
= T 4(R2 106 r2)
= 35 103 4 3.14 [106 (0.01 102)2 (102)2]
= 4.35 103 J

Example 54. A glass plate of length 10 cm, breath 1.54 cm and thickness 0.20 cm weights 8.2 g in air. It is held
vertically with the long side horizontal and the lower half under water. Find the apparent weight of the plate.
Surface tension of water is 7.3 102 N/m.
Sol. Apparent weight = True weight + Force due to surface tension Buoyant force
True weight = mg = 8.2 103 kgwt
Force due to surface tension
= 7.3 102 2 (10 + 0.2) 102
= 14.892 103 N
14.892 103
= = 1.5196 kgwt
9.8
= 1.52 kg wt
Buoyant force = Volume of plate immersed in water Density of water g

www.physicsashok.in 55
FLUID MECHANICS

1.54
= 10 0.2 106 103 9.8 N
2
= 1.54 103 kgwt
Apparent weight
= (8.2 + 1.5196 1.54) 103
= 8.2 kgwt.

Example 55. What is the excess pressure inside a bubble of radius 5.00 mm jormea of a soap solution of
surface tension 2.50 102 N/m. If an air bubble of the same dimension were formed at a depth of 40.0 cm
inside a container containing the soap solution (or relative density 1.20), what be the pressure inside the
bubble ? (1 atm = 1.01 105 Pa)
Sol. Excess pressure inside the soap bubble.
4S 2 2.5 102
P
r 5 103
Excess pressure inside the air bubble in soap solution
2S
p 10 Pa
r
Total pressure inside the bubble = Atmospheric pressure + Hydrostatic pressure due to soap solution of
height 40 cm + Excess pressure due to surface tension
= 1.01 105 + 0.4 1200 9.8 + 10
= 101000 + 4704 + 10
= 105714 Pa
= 1.06 105 Pa

Example 56. The limbs of a monometer consist of uniform capillary tubes of radii 1.4 103 m and
7.2 104 m. Find out the correct pressure difference if the level of the liquid (density 103 kg/m3, surface
tension 72 103 N/m) in narrower tube stands 0.2 m above that in the broader tube.
Sol. If p1 and p2 are the pressures in the broader and narrower tubes of radii r1 and r2 respectively, the pressure
just below the meniscus in the respective tubes will be
2T 2T p1 p2
p1 and p2
r1 r2
B
2T 2T h
So that p1 p2 hg
r1 r2
A
1 1
or p1 p2 = hg 2T r r
2 1
Assuming the angle of contact to be zero, i.e., radius of meniscus equal to that of capillary,
1 1
p1 p2 = 0.2 103 9.8 2 72 103 4
4
7.2 10 14 10

or p1 p2 = 1960 97 = 1863 Pa

www.physicsashok.in 56
FLUID MECHANICS

Example 57. Two separate air bubbles (radii 0.02 m and 0.004 m) formed of the same liquid (surface tension
0.07 N/m) come together to form a double bubble. Find the radius and the sense of curvature of the internal
film surface common to both the bubbles.
Sol. If r1 and r2 are the radii of smaller and larger bubbles and p0 is the atmospheric pressure, the pressure inside
them will be
4T 4T
p1 = p0 + r and p2 = p0 + r ...(1)
1 2

Now as the pressure inside the smaller bubble will be more than inside the larger bubble, so for interface,
p = p1 p2 ...(2)
Now as excess pressure acts from concave to convex side, the
r1
interface will be concave towards smaller bubble and convex
R
towards larger bubble (as shown in Fig.) and if R is the radius p
r2 p1
of interface,
p = (4T/R) ...(3)
p2
So substituting Eqns. (1) and (3) in (2), we get

4T 1 1 r1r2
4T , i.e., R
R r1 r2 r2 r1
0.002 0.004
So here R 0.004 m
0.004 0.002

Example 58. Under isothermal condition two soap bubbles of radii a and b coalesce to form a single bubble of
radius c. If the external pressure is p0 show that surface tension,
p0 (c3 a 3 b3 )
T
4(a 2 b 2 c2 )
Sol. As excess pressure for a soap bubble is (4T/r) and external pressure p0,
pi = p0 + (4T/r)
4T 4T
so pa p0 , pb p 0
a b

4T
and pc p0 ...(1)
c
4 3 4 3 4
and Va a , Vb b and Vc c3 ...(2)
3 3 3
Now as mass is conserved,
a + b = c
p a Va p b Vb pc Vc pV
i.e., as PV RT, i.e., RT
RTa RTb RTc
As temp. is constant, i.e., Ta = Tb = Tc, so the above expression reduces to
paVa + pbVb = pcVc

www.physicsashok.in 57
FLUID MECHANICS

Which in the light of Eqn. (1) and (2) becomes


4T 4 3 4T 4 3 4T 4 3
p0 a 3 a p 0 b 3 b p 0 c 3 c
i.e., 4T(a2 + b2 c2) = p0(c3 a3 b3)

T

p 0 c3 a 3 b3
i.e.,
4a 2
b 2
c
2

ANGLE OF CONTACT
The free surface of a liquid in contact with a solid wall meets at some angle. The angle between tangents
drawn in the free surface and solid, measured inside the liquid, is known as angle of contact.

90
> 90
< 90

glass-water silver-water glass-mercury

In the figure, is the angle of contact. The angle of contact may be acute, right angle or obtuse.

Tangent in the
free surface drawn
through contact Tangent along
the solid

How does angle of contact arise ?


In the figure three substances, namely, a liquid (i), a solid (2), and a gas (3), are in direct contact with one
another. They produce such an orientation of free surface at contact line as to have minimum of the total
energy. this is required for stability. This happens when the line of contact of the three substances are
oriented on the surface of the solid in such a way that the sum of the surface tension forces acting on its
length element equals zero.
S13 3
3 S13 1
1
S23
Solid 2 S12 S23 2 S12

Let S12, S13 and S23 be the surface tensions at the solid-liquid, liquid-gas and solid-gas interfaces.
Balancing forces along the solid surface gives
S23 = S13 cos + S12
S23 S12
cos
S13

www.physicsashok.in 58
FLUID MECHANICS

The angle is measured inside the liqiud between tangents to the surface of the solid and the surface of the
liquid and is called the contact angle.
Since | cos | 1, the contact angle is obtained if
| S23 S12 |
1
S13
If this condition is not satisfied, equilibrium cannot set in at any value of ; the contact line will accelerate.

Wetting and Non-wetting


Water wets glass but mercurry does not. At water-air-glass contour, the angle of contact is tending to zero
and wetting takes place. the contact angle is zero in complete wetting. The inward adhesive force by solid
is more than normal cohesive force.
For any value of close to , the liquid-solid interface tends to contract into a point, and the liquid
separates from the surface of the solid. Complete non-wetting takes place when the contact angle is . The
adhesive force is smaller than normal cohesive force.

Capillarity
The Latin capilles means hair. Literally, capillary is a tube as thin as a hair. Usual capillaries may have
larger bore than hair. If such a tube of glass be immersed in water, water rises in it. If this tube be immersed
in mercury, the level is depressed. Such actions are known as capillarity or meniscus effect. We may understand
capillary action in the following way.
The existence of the contact angle leads to curvature of the surface of a liquid near the walls of the vessel
containing it. In a narrow tube (capillary) or in a narrow gap between two walls, the entire surface is curved.
If the liquid wets the walls, the surface is concave, and if it does not wet them, the surface is convex. Such
curved surfaces of a liquid are called meniscuses. For a narrow tube it may be assumed spherical.
Meniscus
R
P0

P0 2S
R Meniscus
h P0

< 2 ;h>0

> 2 ;h<0
Using pressure concept, we have high pressure on concave side. To meet this requirement hydrostatically,
the meniscus rises (or falls) by suitable height (or depth).
A difference h sets in between the level of a liquid in a capillary and in the broad vessel such that the
hydrostatics pressure gh is compensating it by the capillary pressure :
2S
P0 gh P0
R
2S
gh
R
In this euation, S is the surface tension on the liquid-gas interface, and R is the radius of curvature of the
meniscus. The latter (R) can be expressed through the contact angle and the radius of the capillary r.
www.physicsashok.in 59
FLUID MECHANICS

Indeed, figure shows that


R = r/cos.
Using this value in we arrive at the equation
2Scos
h
gr
This is known as Jurins law. If we consider effect of liquid between lower level of meniscus and the free
surface, h is replaced by h + r/3 for spherical meniscus. In accordance with the fact that a wetting liquid rises
in a capillary, while a non-wetting liquid lowers in it, gives a positive h for < /2 (because cos > 0)and
negative h for > /2 (because cos < 0) the radius r must be measured at the position of the meniscus.
[Note : We assumed here that the meniscus has a spherical shape. The equation for h can also be obtained
on the basis of energy considerations, and there is no need to make a special assumption on the shape of the
meniscus. The equilibrium position of the meniscus will correspond to a minimum energy E of the liquid-
capillary system. This energy E is the sum of the surface energy of the liquid-wall, liquid-gas and wall-gas
interfaces, and also of the potential energy of the liquid in the field of the Earths gravitation.]

Surface Tension events


Even Informations
Complete wetting, capillary rise = 0

wetting, capillary rise <
2

non-wetting, no capilarity =
2

non-wetting, capillary depression >
2
Complete non-wetting, capillary depression =
Ssg SSL
equilibrium states cos ; | cos | 1
Sl g
non-equilibrium states cos > 1
2S cos r
Capillary rise h
rg 3
2S
Liquid Drop P P0 =
R
4S
Bubble P P0 =
R

Variation of surface tension : stronger the intermolecular force of a liquid larger is the surface tension. It is
dependent upon temperature (decreases on rising temperature) and contamination (decreases by mixing
detergent in water)

www.physicsashok.in 60
FLUID MECHANICS

Surface Tension events


Substance Surface Tension (Jm2)

Water 7.29 102


Mercury 4.6 101
Benzene 2.89 102
Ethanol 2.23 102
Glycerol 6.34 102
Oxygen (183C) 1.32 102
Helium (270C) 2.39 104

C34: Calculate the rise of water in a capillary tube of radius 2 mm, assuming the angle of contact to be 0; S.T.
of water is 72 103 N/m.
Sol. Here 2r T cos = r2hg
If = 0, cos = 1
2T 2 72 10 3
then, h 3 3
7.34 103 metre.
rg 2 10 10 9.8

C35: A capillary tube of radius 2 mm is dipped in mercury kept in a vessel. Calculate the depression of the
mercury in the capillary tube with respect to the level in the vessel [angle of contact of Hg with glass is 135,
and S.T. of Hg is 465 dyne/cm.]
2T cos 2T cos135 2T cos(90 45 )
Sol. Here, h
rg r g r g
2T cos 45 2 455 1/ 2
h 24.67 cm
rg 0.2 13.6 9.7

C36: In a vessel equal mass of alcohal (sp. gravity 0.8) and water are mixed together. A capillary tube of radius
1 mm is dipped vertically in it. If the mixture rises to a height 5 cm. in the capillary tube, then calculate the
S.T. of the mixture; (assuming angle of contact to be 0).
Sol. 2rTcos = r2hg
rhg
T=
2 cos
212 2 0.8 1 1.6 8
but gm / cm3
1 2 1 0.8 1.8 9
0.1 5 8 / 9 980
T= = 217.9 dyne/cm.
2

Example 59. A glass capillary sealed at the upper end is of length 0.11 m and internal diameter 2 105 m. The
tube is immersed vertically into a liquid of surface tension 5.06 102 N/m. To what length has the capillary
to be immersed so that the liquid level inside and outside the capillary becomes the same ? What will happen
to the water level inside the capillary if the seal is now broken ?

www.physicsashok.in 61
FLUID MECHANICS

Sol. If A is the cross-sectional area of the tube and L its length, the initial volume of air inside it will be V1 = AL
while pressure p1 = p0 = atmospheric pressure.
Now when the tube is immersed in water with its length x in water, the level of water inside and outside is
same; so the volume of air in the tube will be V2 = A(L x). Further if p2 is the pressure of gas in the tube,
2T 2T ........
p2 p0 , i.e., p2 p0 .......................
r r ......p............ p
Now if temperature is constant, (Lx) ................2........ 0
..................
P1V1 = P2V2 ....
.............
.
.....................
..
2T rp x
p0 AL p0 A(L x) or x 1 0 L
r 2T
2T
p2 r
1.012 105 1 105
i.e., x 1 0.11
2 5.06 102

0.11
or x 0.01 m
11
If the seal is broken the pressure inside the capillary will become atmospheric, i.e., p0 while capillarity will
take place and the rise will be
2T 2 5.06 102
h 1.03 m
rg 105 103 9.8
However, the length of the tube outside the water is 0.11 0.001 = 0.1 m; so the tube will be of insufficient
length and so the liquid will rise to the top of the tube and will stay there with radius of meniscus,
hR 1.03 105
r 1.03 104 m
L 0.1

Example 60. A conical glass capillary tube of length 0.1 m has diameters 103 and 5 104 m at the ends.
When it is just immersed in a liquid at 0 C with larger diameter in contact with it, the liquid rises to 8 10
2
m in the tube. If another cylindrical glass capillary tube B is immersed in the same liquid at 0 C, the liquid
rises to 6 102 m height. The rise of liquid in the tube B is only 5.5 102 m when the liquid is at 50C.
Find the rate at which the surface tension changes with temperature considering the change to be linear. The
density of the liquid is (1/14) 104 kg/m3 and angle of contact is zero. Effect of temperature on density of
liquid and glass is negligible.
Sol. If r is the radius of the meniscus in the conical tube, then as shown in Fig.
r1
r r1 r2 r1 A B
tan
Lh L r
L
r 2.5 104 (5 2.5) 10 4 h
i.e.,
0.1 0.08 0.1
i.e., r 104 2.5 = 0.2 2.5 r2
i.e., r = 3 104 m
Now as capillarity is independent of the shape
of tube so at same temp. = 0 C.

www.physicsashok.in 62
FLUID MECHANICS

hArA = hBrB = (2T0/g) = constant.


so rB = (0.08 3 104)/(6 102) = 4 104 m
Now as from h = (2T/rg) for cylindrical tube,
h 0gr 1 1
T0 6 10 2 104 9.8 4 104
2 2 14
T0 = 8.4 102 N/m
Now as for a given tube and liquid T h (as T = hgr/2)
T50 h 50

T0 h0

5.5 102
So, T50 8.4 102 = 7.7 102 N/m
6 102
So rate of change of surface tension with temperature assuming linearity,
T T50 T0 (7.7 8.4) 102

50 0 50
2
= 1.4 10 N/mC Ans.
Negative sign shows that with rise in temperature surface tension decreases.7

Example 61. A conical glass capillary tube A of length 0.1 m has diameters 103 m and 5 104 m at the ends.
When it is just immersed in a liquid at 0C with larger diameter in contact with it, the liquid rises by
8 102 m in the tube. In another cylindrical glass capillary tube B, when immersed in the same liquid at
0C, the liquid rises by 6 102 m. The rise of liquid in tube B is only 5.5 102 m when the liquid is at
50C. Find the rate at which the surface tension charges with temperature considering the change to be
linear. The density of the liquid is (1/14) 104 kg/m3 and the angle of contact is zero. Effect of temperature
on the density of liquid and glass is negligible.
Sol. Let r1 and r2 be the radii of the lower and the upper ends of the capillary. The radius r at height h at the
position of the meniscus is given by
r1 r r1 r2 A r2
( tan )
h l
r
r = r1 (r1 r2) (h / l) l
(0.25 0.5) h
r = 0.5 103 0.8 101
0.25 B r1
r = 0.3 103 m
Surface tension of the liquid at 0C
rhg 0.3 10 3 8 102 104 9.8
S0 ( = 0)
2cos 2 14 1
S0 = 0.084 N/m
If r is the radius of the cylindrical tube B, then at 0 C
rh 0 g
S0
2

www.physicsashok.in 63
FLUID MECHANICS

and at 50C
rh 50 g
S50
2
S50 h 50 5.5 102 11

S0 h0 6 102 12

11 11
or S50 S0 0.084 0.077 N / m
12 12
Rate of change of surface tension with temperature, assumed constant for S () as a linear function, will be
S50 S0 0.077 0.084
Nm 1 K 1
50 50
= 1.4 104 Nm1 C1
Thus, the surface tension decreases at the rate of 1.4 104 N/m per degree celsius rise in temperature.

www.physicsashok.in 64
FLUID MECHANICS

ELASTICITY

ELASTIC BEHAVIOUR
The length of a steel wire can be increased by applying forces at its ends. In the figure a steel wire of length
l is fitted with a vernier scale V by the side of main scale M. By putting a load W, the length l becomes
l + l as indicated by reading of the vernier scale.
l
l+ l
V

M
W

If load W is decreased, the value of l also decreases. When w becomes zero, l also goes to zero. The
wire has returned to its original shape and size.
There are several bodies around us, which behave like a steel wire we described just now. This behaviour
is known as elastic behaviour or elasticity. The property of certain materials to regain their shape and size
when the cause of deformation is removed, is known as elasticity. The material showing elasticity is called
elastic.
In deformation, the particles of the body are diplaced from their equilbrium positions to new positions.
These new positions give rise to new shape and size of the body. The forces between particles try to bring
them back to original positions. The tendency is decided by how the molecules act an each other during
deformation. These molecular forces are called internal forces that oppose the applied forces and want to
restore the old shape and ize (see figure).
When the cause of deformation ceases, there are back motions of displaced particles. If the particles return
back reversibly, the deformation is categorised as elastic.
A model of molecules under spring-like forces of other molecules is as shown here for simplicity sake. If
atoms (black balls) are displaced and released, the springs force them to come back. Such forces are at the
root of elastic behaviour. It shows that it is a molecular (atomic) level phenomenon, electromagnetic forces
governing the whole game. Condon and Morse proposed a force derivable from potential.

r0
r

repulsion attraction

A B
U
rm rn
(A, B, m, n are suitable constants)
This potential has both repulsive and attractive parts, giving spring-like action.
In certain materials, particles are pushed to new arrangement of equilibrium during deformation. Hence
upon removal of cause of deformation, the body does not return to previous shape and size fully. Such
bodies are called plastic. If you press a putty or mud, and remove your force, it stays in the pressed
position. Thus it is plastic. The deformation is irreversible.

www.physicsashok.in 65
FLUID MECHANICS

We can observe a mixture of elasticity and plasticity in the same body. It depends of upon the extent of
deformation. For small deformation elastic behaviour is observed while a mixture at more deformation and
purely plastic at large deformation. We must perform experiments to know the actual elastic behaviour of
bulk matter.

STRESS AND STRAIN


Elastic behaviour is described by answering two questions :
(a) How much is deformation produced.
(b) How much is opposition to the deformation.
There are two terms stress and strain is answer such questions. Strain is related to the first
question while stress to the second.

Stress : If a body is deformed, forces arise that want to restore the equilibrium configuration. These are known
as internal restoring forces or elastic forces. If we consider a surface in the body it divides the body into two
parts, one on one side of the surface and the other on the other side. The two parts exert on each other
elastic forces. An area A of the surface may have a force F distributed on it. The internal elastic force per unit
F
area of a surface is known as stress. We denote it by . Then .
A

A A F F A

Surface

The value of depends upon the orientation of surface. Hence it may happen that

(i) F is normal to tbe surface. In that case we call normal stress, denoted by n.

(ii) F is not normal to the surface. In that case we can resolve F into two components :
Fn, normal to surface and Ft along the surface,
Normal stress is defined as normal internal elastic force per unit area. That is,
Fn
n .
A
Fn
A
F
Ft
Normal stress may be arising due to tension in the body. In that case it is called tensile stress. On the other
hand, if it arises due to compression, it is called compressive stress. It produces a change in size.
Tangential Stress is defined as tangential internal elastic force per unit area. That is,
Ft
t .
A
It is also called shaearing stress. It tries to move the surface tangentially causing a change in shape.
In hydrostatics, the normal force per unit area of a surface is pressure. The value of pressure at a point does
not depend upon choice of the orientation of the surface. The normal stress however, is not pressure
because its value depends upon choice of orientation of the surface. It may in same cases coincide with
pressure. Thus, stress, although has the same unit and deminsions as the pressure, is not a scalar, nor it is a
vector. The complete specification of stress requires description of both normal and tangential stresses,
which depend upon orientation of surface and are known to behave as tensors.

www.physicsashok.in 66
FLUID MECHANICS

There are situations where interaction of one part of a body on the other part is not only a force but also by
a couple. Such couple moments are distributed in the body, for example in a polar dielectric in electric field
or in a magnet in a magnetic field. In those cases couple movement per unit area is also needed to describe
the stress. Stress is distributed in the volume the body. We must consider the particular point and force at
that point for finding out stress at that point of the body.

Example 62. A thick book like Ramcharit Manas is placed on a rough table. It is pushed slant by hand as in the
figure. We find that its base remains fixed but upper layers shift. Also the thickness is slightly reduced due to
pressing. Consider a hand area of A = 5 cm2 in contact and applying a force of F = 10 N at 60 with normal.
Calculate normal stress and tangential stress.
Ft Fn
60
A
Fn
Ft

rough

Fn
Sol. Normal stress n
A
Here Fn = F cos 60
Fn = 10 1/2 = 5 N
A = 5 cm2 = 5 104 m2
5N
n
5 10 4 m 2
n = 104 Nm2
n = 104 Pa (1 pascal = 1 Pa = 1 Nm2)
Ft
Shearing stress, t
A
Here Ft = F sin 60 = 5 3 N
t = 3 104 Pa = 1.7 104 Pa.
Example 63. A scissor fulcrum is a nail of radius r. It is squeezed with force F
as in the figure by putting a strip to cut. In equilibrium, calculate the shearing F
stress developed by blades on the nail. F
a
Sol. Considering moment of forces about O in the figure, b
Ft

O
F
b a
N
f Ft

Ft f

F (a + b) = Ft b

www.physicsashok.in 67
FLUID MECHANICS

a
Ft F 1
b

Ft F(a b)
Blades exert forces Ft on area r2. Hence shearing stress t 2
.
r r 2 b
Example 64. A wire of cross-sectional area A and length L is hanging from ceiling. The density
of its material is p. The gravity is pulling it downward while ceiling is preventing it from fall.
So it is in stressed state. Calculate stress at a cross section located at height x from lower
end. (The force is axial, the stress is tensile).
Sol. Here the elastic internal force is tension force. Let T be the tension in the wire at a
cross section located at height x above the lower end. The tension T is balancing
the weight of wire of length x. This weight is T
w = volume density g
w = xA. . g
T = xA g. x
T
The stress is given by
A weight
= x g.
C37: A beam is placed on two nails P and Q. A load hung is from the centre. There is no friction. Which part of
the beam is in tensile stress and which part isn compressive stress ?

P Q P Q

Sol. The part above middle horizontal layer (surface) is shortened and is under compression while the part
below the middle surface is elongated and undre tension. thus, upper half is in Compresve stress while
lower half is in tensile stress. (The middle surface is neutral).

Strain : When the particles of a body are displaced from their equilibrium positions under the intermolecular
forces, the body is said to be strained. You squeez a rubber ball, it is strained. You may pull apart the ends
of a wire, it is strained.
l

l+ l
We can deform a body in three ways :
(a) Changing its length.
(b) Chang it volume.
(c) Twisting it (changing its shape).
Longitudinal strain is defined as change in linear dimension per unit of its original value. The linear dimension
may be length of a wire, diameter of a cylinder, width of a rectangular bar, etc. It l be a measure of linear

www.physicsashok.in 68
FLUID MECHANICS

l
dimention and l be the change (positive or negative), the ratio is known as longitudinal strain. If the
l
l
length is decreased due to deformation, l is negative. Then we call compressive strain. If l is
l
l
positive, the length is increased and is tensile strain. If D be dimater of a cylinder and D be the
l
D
change in it due to deformation, is known as lateral strain. The ratio of lateral strain to longitudinal
D
strain is defined as Poisson ratio.
D

D

l
l
Its theoretical value can be 0.5 to 0.5 but practical value is found between zero and half.
The minus sign is put to define as positive, D and l are of opposite signs.
l l b

D+ D D
h+ h h
l+ l
b+ b l+ l

b l
(l > 0, D < 0);
b l
D l h l

D l h l
Volume Strain or, hydraulic strain, is defined as the change in volume per unit original volume, due to
deformation. If V be the volume before the deformation and V + V be the volume due to the deformation.
V
Then is the volume strain.
V

V+ V

An air bubble at some depth in water has volume V. It is rising up. The
pressure is less at less depth. Hence the bubble is relaxed a bit, its volume
V + V,
becomes V + V. There is a volume strain of V/V, which is positive.
V<0
We can squeez a rubber ball in which case V is negative and volume
strain is V/V is negative. V

www.physicsashok.in 69
FLUID MECHANICS

Example 65. A metal wire of length l is stretched by dl. Its Poisson ratio is .Determine its volume strain.
Sol. The initial volume of the wire is V = r2l.
Here dV = d (r2l)
dV = (2r dr . l + r2 dl)

dV 2r dr l r 2 d l

V r 2l r 2 l

dV dr d l
2
V r l
dr d l
Now the Poisson ratio relates to as
r l
dr dl dr dD D

r l r D D
dV dl dl
2
V l l
dV dl
1 2
V l
Volume strain of wire is (1 2) times the longitudinal strain. (It will be zero if = 1/2).

The Shearing Strain


Shear is the kind of deformation in which all the flat layers of a body, parallel to certain plane, undergo rigid
displacement parallel to one another. During this, the size of layers does not change.
In the figure shear of a cube ABCD is shown. The shear plane is AB. A layer DC is parallel to it. It is
displaced to CD. There is no change in its size (CD = CD).
Ft
D
D
D C
D C
A B

A B A B
Ft

We observe that the ratio of relative shift in a layer relative to some layer, to the separation between these
two layers is the same for any pair of parallel layers and is called shearing strain. In the figure.
DD = shift of layer relative to AB.
D D = shift of layer relative to AB.
DD DD
tan
AD AB
This ratio is known as shearing strain. Let us denote it by t.

C38: A wire of length l and radius r is clamped at one end. It is twisted at the l
other end by an angle about its axis. Write the shearing strain produced,
in a small area lying in the cross-section at the end at a distance x from
the axis.

www.physicsashok.in 70
FLUID MECHANICS

Sol. The figure shows an area A at position x from axis which is


parallel to area P at the clamped end. Due to twisting, A
= t
goes to A by suffering a displacement AA x relative to A
P A
P. The separation between the parallel area P and A is l. x
x
Hence shearing strain, t .
l

Relationship Between Stress and Strain


To see the relationship between stress and strain, we must perform experiment on the given body.

V M
A slotted weight

Let us take a metal wire, measure its length and diameter, and suspend from ceiling. Now e attach a vernier-
scale and main scale by a side wire. Suspend a hanger on which slotted weight could be put. Adjust the
vernier to read zero; this remove calculation for effect of weight of hanger.
Let a load w be put on the hanger. Then stress is
W

(D 2 / 4)
The strain is calculated by noting the much (l) the vernier descends and dividing it by the length of wire
l
above vernier. This is l . Increasing W in small steps, we get several pairs of and t .
l
l
A graph is plotted between and for various values of W and corresponding values of . This graph
l
shows elastic behaviour of the material. It is dependent upon material. We discuss typical graphs for steel
wire, rubber band and aorta.
The vernier-method described is good for small stretching. For more sensetive step wise stretching and
measuring tension a tensometer is used.

Steel wire : A common graph of stress against stream for a wire of metal has the features shown in the
figure.
(i) Proportionate region : For low stress, the stress-strain graph is a straight passing through origin.
B C D
W
= 2 E
( D ) A
4

F
O =
l
l
We have stress = (slope) strain. For the straight. Part the slope is constant Hence
stress strain.
This behaviour is called proportionality of stress to strain. The region OA is, therefore, called proportionate
region.

www.physicsashok.in 71
FLUID MECHANICS

(ii) Elastic region : If the cause of deformation be removed, the body returns reversibly (along to the same
states through which it went on deforming) to original shape and size. This behaviour occurs upto a stress
called elastic limit. In the figure OA and AB are such regions, B is called elastic limit and A is called proportional
limt. In the region between proportional limt and elastic limit, the graph is curved and hence stress is not
proportional to strain, although it grows with growing strain.
If the stress is further increased, the plastic and elastic behaviour mix up. As material yields to plasticity
(agrees to become slightly plastic also), the elastic limit is also known as yield point. The stress at yield point
is known as yield strength, y.
(iii) Plastic region : As the material is deformed beyond yield point (elastic limit), there is a few degree of
structural change in equilibrium positions of its particles causing plasticity. If the stress is lowered, the body
returns towards initial configuration but not totally. When stress is zero, a permanent deformation stays with
the body, called permanent set (OF).
In the graph, BD is the region of plasticity. The stress increases with strain upto an ultimate value known
as ultimate tensile strength, u (See Table ).

Table : Some Elasticity Data

Material Ultimate Strength Yield Strength


(GPa) (GPa)
Aluminium 110 95
Copper 400 200
Iron (Wrought) 330 170
Steel 400 250

(iv) Fluid region : Beyond the point where stress is equal to ultimate tensile stress, u, the material undergoes
deformation even for smaller value of stress than u. It acts as if it is flowing. The wire thickness goes on
reducing, called neck formation. At a certain point, it breaks. This point is called fracture point (E) The
corresponding stress is breaking stress, b.
The important of fluid region is that it classifies material into brittle and ductile.
If the point of ultimate tensile stress and fracture point are very class to each
other the material is brittle, like brass, cast iron.
If these points are for apart, the material is ductile. That is, it can be drawn into
wire form.
If you want to test the proportionate behaviour of a steel wire in laboratory, the allowed load is upto half of
that causing fracture.

Elastomers : If you stretch a steel wire, a large force will cause a small strain. However, a rubber wire can be
stretched by a small force to have a large strain. There are several such substances. The vessels carrying
blood from heart, called aorta, have tissues that show large reversible strain for small stresses, without well
defined plasticity. Such substances are called elastomers.

(x102 Pa)
0.5

0.5 1.0

www.physicsashok.in 72
FLUID MECHANICS

The stressstrain graphs of elastomers are not straight lines, yet they are elastic. The tissue of aorta has a
graph of stress against strain as in the figure (a); the figue (b) shows that for a rubber.

Governing the elastic behaviour : The elastic behaviour depends upon the following factors :
(a) Temperature (b) presence of notches
(c) time rate of loading (d) presence of embrittling agent like hydrogen
(e) loading and unloading (f) structure of material.

HOOKES LAW
Robert Hooke was the first person to notice the property of elasticity. Based
on experiments on springs and springy bodies, he announced, Ut tensio sic
vis. (The stretching of a spring body is as the applied force). This law has
been given more precise form now and is known as Hookes law : Within
elastic limit stress is directly proportional to strain.
If stress be and strain be then

or, = E .
Here the proportionality constant is E which depends upon the material and the type of stress. We call it
modulus of elasticity.
Hookes law need not be obeyed for a body to be elastic. We know that up to elastic limit a metal wire
remains elastic. While Hookes law is valid only within proportionate region. Also there are several materials
which are elastic but have non-linear stress strain graphs. The elastic materials obeying Hookes law are
known as Hookean and those not obeying it are known as Hencky. For Hencky materials we have
= a n
where a and n are material constants. Modulus of elasticity for such bodies are defined by slopes of
tangent and secant of strain-stress graph. We shall not consider these further.

Different Moduli of Elasticity : There are three types of stress longitudinal, bulk and shearing and there
are three moduli defined for these.
(a) Youngs modulus, Y, is the ratio of longitudinal stress to longitudinal strain. If elastic normal force Fn acts on
l
areas A and be strain, then
l
Fn / A
Y
l / l
(b) Bulk modulus, B, is the ratio of volume stress change in pressure to volume strain.
If a change in pressure dP causes a change in volume V by dV, the strain is (dV/V) and bulk modulus B
is given by
dP
B
dV / V
where minus sign in put to keep B a positive quantity.
Inverse of bulk modulus is known as compressibility, K.
That is,
dV
K
VdP

www.physicsashok.in 73
FLUID MECHANICS

(c) Rigidity modulus, G, is the ratio of shear stress to the shear strain. If elastic internal force Ft acts parallel
to area A, and be the angular shift in the layer, we get
Ft / A
G

Table shows elastic constants of some materials.
Units and dimensions of Elastic moduli :
Stress
As E = , and strain is unitless and dimensionless, E has the same unit and dimensions as stress, which
Strain
is the ratio of force and area. Thus, the SI unit of modulus of elasticity is Nm2 or Pa (passcal). The
dimensional formula is [ML1 T2]

Example 66. The following four wires are made of the same material. Which of these will have the largest
extension when the same tension is applied ?
(A) length = 50 cm, diameter = 0.5 mm (B) length = 100 cm, diameter = 1 mm
(C) length = 200 cm, diameter = 2 mm (D) length = 300 cm, diameter = 3 mm
Sol. [A]
T/A
Y
l / l
Tl T l
l
AY Y A
T l
Here is constant. Therefore l .
Y A
l
is largest in the first case.
A

MEANING OF A MORE ELASTIC BODY


In common mans language more elastic bodies are easily elongated to larger extent. However in physics,
elasticity is measure of opposition of to a deformation. Harder to deform a body, more elastic is the body.
Thus more elastic bodies will have higher value of modulus of elasticity. This is way we say that steel (Y =
200 GPa) is more elastic than rubber (Y = 2.4 103 GPa).

Relationship among elastic constants : Let Y, B and G be Youngs modulus, bulk modulus and rigidity
modulus of material of a solid body and be Poison ratio. The relationship among these may be summerised
as below. Derivation of these results is out of scope of the book.
(i) Y = 3B (1 2) = 2G (1 + )
3B 2G
(ii)
2G 6B
9 1 3
(iii)
Y B G

Elastic Fatigue
Due to repeated deformation and restoration, the elasticity falls and delayed recovery is observed. This is
known as elastic fatigue. If the body is allowed to take rest for time, the original elastic property is restored.

www.physicsashok.in 74
FLUID MECHANICS

Elastic after-effect
The delayed recovery of original configuration by some elastic materials when the cause of deformation is
removed is known as elastic after effect. This is a type of viscoelastic behaviour. In some materials the
recovery of original configuration is quicker (quartz, phosphor bronze, etc.) while in some it is considerably
large (glass fibre).

Strainhardening
As the deformation becomes plastic, material needs larger and larger stress (becomes harder to deform).
This is known as strainhardening.

Example 67. A copper bar has a compression force of 2kN distributed over its cross section of diameter 20
mm. Its length is 2 m and Youngs modulus is Y = 110 GN m2.
Calculate the compression l of the bar.
Sol. Using Hookes law,
F l
2
y
(D / 4) l
Fl
l
Y (D2 / 4)
(2 kN) (2m)
l
( / 4)(110 GN m 2 ) (20 mm)2

(2 103 ) 2
l
(3.14 / 4)(110 109 ) (20 103 )2
l = 1.116 104 m
l = 1.12 104 m.

Example 68. A load of 4.0 kg is suspended from a ceiling through a steel wire of length 2.0 m and radius 2.0
mm. It is found that the length of the wire increases by 0.031 mm as equilibrium is achieved. Find Youngs
modulus of steel. (Take g = 3.1 m/s2)
(4.0) (3.1 )
Sol. Longitudinal stress =
(2.0 103 )2
= 3.1 106 N/m2
0.031 103
Longitudinal strain =
2.0
= 0.0155 103
3.1 106
Thus, Y= = 2.0 1011 N/m2.
0.0155 103

Example 69. Determine the elongation of the steel bar 1 m long and 1.5 cm2 cross-sectional area when subjected
to a pull of 1.5 104 N. Take Y = 2.0 1011 N/m2.
F/A
Sol. Y
l /l

www.physicsashok.in 75
FLUID MECHANICS

Fl
l
AY
(1.5 104 )(1.0)
Substituting the values, l
(1.5 10 4 )(2 1011 )

l = 0.5 103 m = 0.5 mm


A
Example 70. A bar of mass m and length l is hanging from point A as shown in figure. Find the
increase in its length due to its own weight. The Youngs modulus of elasticity of the wire is
Y and area of cross-section of the wire is A.
Sol. Consider a small section dx of the bar at a distance x from B. The weight of the bar for a B
length x is, A
mg
W x
l
Elongation in section dx will be dx
x
W
dl dx B
AY
mg
dl x dx
l AY
Total elongation in the bar can be obtained by integrating this expression for x = 0 to x = l .
xl
l dl
x0

mg l
l AY 0
l x dx

mg l
or l
2AY

Example 71. Two wires of metals A and B have unloaded lengths lA and lB and equal
cross sectional area . They are loaded as in the figure with weights W1 and W2. A
If the Youngs moduli are YA for A and YB for B, calculate elongations in A and B W1
due to loading. B
Sol. The free body diagrams are shown for wire A, loads W1 and W2, and wire B. We see that W2
TB = W2 ...(i)
TA = W1 + TB ...(ii)
TA
The stress in wire A is
TA TB TA
A A

B TA W1 TB
W T
A 1 B
TB TB

W1 W2
A ...(iii) W2

www.physicsashok.in 76
FLUID MECHANICS

lA
Now A YA
lA
A
lA lA
YA
(W1 W2 ) lA
lA [using (iii)]
YA
The stress in wire B is
TB
B

W2
B [using (i)]


Hence lB B lB
YB

W2 lB
lB
YB

Example 72. A wire of length l = 1.0 m and cross-sectional area = 5.00


104 cm2 is stretched by load at mid point with a mass m = 100 g. Assuming
stretching within elastic limit, calculate the depression of wire at the mid point.
The Youngs modulus of elasticity is 1.9 1011 Pa.
m
x
Sol. Let the depression be x. Then tan .
(l / 2)
Now W = T cos + T cos
W W l/2
T sec x
2 cos 2
T T
Using Hookes law
T
T z W
Y
(l / 2)
where z is elongation in the half length (l /2).
W 2Y
sec z
2 l
2
l2 l 2x 2
Now z x
2 2 l
2
l 2
x
2 l
sec
x 2

www.physicsashok.in 77
FLUID MECHANICS

Wl 2x 2 2x

4 y l l
1/ 3
W l3
x
16 y
Putting the value
100 103 10 13
x 4 4 11
0.10 m
16 5 10 10 1.9 10

Example 73. A mass m is tied to an elatic band of length l and whirled in a vertical circle. The angular speed of
the mass at the lowest point of the path is . If Y be Youngs modulus, calculate elongation at this point.
Sol. Let x be the elongation. Using dynamics of circular motion m2 (l + x) = T mg
T = m 2 (l + x) + mg ...(i)
l+x
T x T
Using Hookes law, Y
l
V
Y mg
T x ...(ii)
l
From (i) and (ii),
Y
x m 2 (l u) mg
l
Y 2 2
l m x m( l g)

m( 2l g)
x
Y
m 2
l

Example 74. The breaking stress of a material is B = 106 Nm2 and density is = 5 103 kgm3. Determine
the maximum length that can be hung without breaking.
Sol. Let l be the length hung. The weight will be
w = l , where is cross-sectional.
The stress due to weight is maximum at the top end. Hence we expect fracture from the top. We avoid
break of a top by writing
w
B

lp
B


l B

B 106
The maximum length is, then, , i.e., and across-sectional area lmin 200 m
5 103

www.physicsashok.in 78
FLUID MECHANICS

Example 75. Two metal wires A and B have the same length l under no loading.
The Youngs moduli are Y A and YB, cross-sectional areas are and 2 for A
A and B respectively. The rod PQ is light. Now a load W is applied. x B
(a) Write extension in the wires A and B as a function of x. P Q
(b) Write the value of x for equal stress in the wires. L
(c) Write the value of x for equal strain in the wires. W
Sol. Let tension be TA in A and TB in B. The equilibrium of rod suggests that moment of w and TB about P must
be balanced. Hence
Wx = TB L TA TB
x
x
TB W ...(i)
L L
W
As TA + TB = W
x
TA W 1 ...(ii)
L
The stress in the wire A is given by
la
A YA
l
TA l
YA A
l
W x
lA 1 l
Y L
The stress in the wire B is given by
lB
B YB
l
TB l
YB B
2 l
W x
lB l
2 L
(b) Equal stresses in the two wires means
TA TB

x 2
TB
Ta
2
Putting the values of TA and TB,
Wx x
W 1
2L L

www.physicsashok.in 79
FLUID MECHANICS

x x
1
2L L
2
x L
3
(c) Equal strains in the two wires means
lA lB

l l
TA T
B [ stress = Y. strain]
YA 2YB
TA T
B
YA 2YB
Putting the values of TA and TB,
W x Wx
1
YA L 2L YB

x YA x
1
L YB 2L

x Y
1 A 1
L 2YB

L
x
(1 YA / 2YB )

Example 76. A solid cube of steel of volume 1 m3 is immersed in water at a depth of 1 km. Find the decrease
in its volume due to volume stress. The density of water is = 1000 kg m3 and the bulk modulus of steel
is B = 135 GPa.
V
Sol. Using Hookes law, P = B
V
Here volume stress is P = gh
P = 1000 9.81 1000 Pa
P = 9.81 106 Pa
V = V (P)
V = (1 m3) (9.81 106 Pa)/ (135 GPa)
V = 0.072 103 m3
V = 7.2 105 m3
The minus sign shows that the volume has decreased.

Example 77. A wire of length l and radius r is clamped at one end. Its rigidity modulus is G. Find the torque
about its axis needed to produce a twist by angle at the end.

www.physicsashok.in 80
FLUID MECHANICS

Sol. Let d be the torque acting along the axis due to forces of share along the ring surface of area 2 x dx.
x dF
The sheart strain is . The stress is . Using
l 2 x dx
dF x x d
Hookes law G .
2 x dx l


dF 2 G x 2 dx l
l
Torque about the cylinder axis is given by
d = x dF
2 3
d Gx dx
l
The total torque needed is given by
r
2 G 3
l 0
x dx

2 G r 4

l 4

r4
G
2 l

r 4G
Here / is constant. This constant is called torsional constant, c. c = .
2l

ELASTIC ENERGY
The energy stored in deformed body per unit volume is equal to half of the product of stress and strain. It is
called elastic energy density.
If u be the elastic energy per unit volume,
1
u= stress strain.
2
We consider the Longitudinal case for deriving the above result
(without loss of generality).
Let there be a wire of length l and cross sectional area A which is
extended by x using a force F. Then, using Hookes law, l

F x
Y x
A l
dx
Y F
F Ax
l
Let the wire be further extended by dx. The work done by F is dW. Here
dW = F dx

www.physicsashok.in 81
FLUID MECHANICS

YA
dW x dx
l
If the wire is stretched from x = 0 to x = l,
l
YA
w
l x dx
0

2
YA l
w
l 2
1 YA l
w l
2 l
1
w= Force extension.
2
This work is stored in the wire as elastic energy U. Thus
2
YA l 1 Yl l
U l Al
2 l 2 l l
This gives
1
U= stress strain volume
2
U
u
volume
1
u= stress strain.
2
This is true for all the three types of deformations.

Example 78. A metal wire of length 2m is supporting a 20 kg ball. Suddenly the ball is detached. Calculate the
thermal energy developed in the wire if Youngs modulus of its material is 200 GPa.
Sol. The elastic energy stored in the wire in presence of load is
1
U= stress strain volume
2
1 l l 1 l
U Y Al Y A l
2 l l 2 l
l
Now Y A mg
l
1 mg l 1 (mg)2 l
U mg .
2 YA 2 YA
1 (200)2 2
U 0.04 J
2 200 109 5 106
This energy is converted into thermal energy.

www.physicsashok.in 82
FLUID MECHANICS

Example 79. A wire of area of cross-section 3.0 mm2 and natural length 50 cm is fixed at one end and a mass
of 2.1 kg is hung from the other end. Find the elastic potential energy stored in the wire in steady state.
Youngs modulus of the material of the wire = 1.9 1011 N/m2. Take g = 10 m/s2.
Sol. Volume of the wire is
V = (3.0 106) (0.50) = 1.5 106 m3
Tension in the wire is
T = mg
T = (2.1) (10) = 21 N
Stress = T/A
21
Stress = = 7.0 106 N/m2
3.0 106
Strain = Stress/Y
7.0 106
Strain = = 3.7 105
1.9 1011
The elastic potential energy of the wire is
1
U = (stress) (strain) (volume)
2
1
U = (7.0 106) (3.7 105) (1.5 106)
2
U = 1.9 104 J

Determination of Youngs modulus of the material of a wire in laboratory


Two identical wires (say of steel) are hung from the ceiling. A is auxiliary wire and B is experimental wire
whose Y is to be determined. A main scale M is fitted in auxiliary wire A and vernier scale V in experimental
wire. Identical pans H (or hangers) are attached and loaded to remove kinks. Measure the diameter of
wires B by a screw gauge vernier is adjusted to zero. Now allowed load is calculated from table of constants
(half of breaking stress). We have to put a load not greater than this.
Now we start from a minimum load. Put the load on right pan and take the
reading on vernier scale. Calculate the value of (l / l) for given load. Plot a B
l A l
graph between stress W/r2 and strain (l / l). It turns out to be a straight line.
l
W l
Hookes law gives 2
Y .
r l M V
The value of Y can be calculated :
(W / r 2 ) H
Y H
( l / l )
It can also be graphically calculated from the slope of stress strain graph.

Stress
(W/ r 2)

x Strain ( l )
l

www.physicsashok.in 83
FLUID MECHANICS

y
Slope = tan == Youngs modulus.
x
Why do we take wire A along with B ? This is to compensate for any temperature effect on strain.

IMPORTANT FEATURES
1. Moduli of elasticity are three, viz, Y, B and while elastic constants are four, viz, Y, B, and . Poissons
ratio is not modulus of elasticity as it is the ratio of two strains and not of stress to strain, Elastic constants
are found to depend on each other through the relations :
Y = 3 B (1 2) and Y = 2(1 + )
Eliminating or Y between these, we get
9 B 3B 2
Y and
3B 6B 2

2. Table : Some Elastic Constans of Selected Materials of Engineering and Medical Interest

Material Youngs Modulus Shear Modulus Bulk Modulus


(Y) (1011 Pa) (S) (1011 Pa) (B) (1011 Pa)
Aluminium 0.70 0.30 0.70
Brass 0.91 0.36 0.61
Copper 1.1 0.42 1.4
Glass 0.55 0.23 0.37
Iron 1.9 0.70 1.0
Steel 2.0 0.84 1.6
Tungsten 3.6 1.5 2.0

3. Gases have two bulk-moduli, namely isothermal elasticity E and adiabatic elasticity E. It has been found
that at a given pressure P,
E = P and E = P
E CP
So that 1 , i.e., E > E
E CV
i.e., adiabatic elasticity is greater than isothermal elasticity.

4. In case of compression of a fulid


m V
density = = , so
V V V
But by definition of bulk modulus, i.e.,
V P V P
B or
V V B
P P
or [as = ]
B B

www.physicsashok.in 84
FLUID MECHANICS

P 1
or 1 [1 C P] as B C
B

5. In case of bending of a beam of length l, breadth b and thickness d, by a load Mg at the middle, depression
is given by
Mg l 3

4bd 3 Y
and for a beam of circular cross-section of radius r and length l.
Mg l 3

12 r 4 Y

6. In case of twisting of a cylinder (or wire) of length l, and radius r, elastic restoring couple per unit twist is
given by
l 4
C
2l
when is modulus of rigidity of the material of wire.

7. In case of a rod of length l and radius r fixed at one end, angle of shear is related to angle of twist by
the relation.
l = r .

www.physicsashok.in 85
FLUID MECHANICS

THINKING PROBLEMS

FLUID MECHANICS
1. A wooden cylinder floats in water in a vessel with its axis vertical. How will the level of water in the vessel
change if the cylinder floats with its axis horizontal?
2. A vessel filled with water is supported on a knife-edge. Will the equalibrium be disturbed if a small board
carrying a weight is placed on the surface of the water?
3. A boat floats in a swimming pool. Water from a well is pumbed into the pool. When will the pumps perform
move work: when a big boat floats in the pool, or a small toy-boat ?
4. A vessel of water is placed on the floor of an elevator. Will the pressure at the bottom of the vessel change if
the elevator goes up with uniform acceleration a ?
5. A wooden cylinder floats in water in a vessel placed on the floor of an elevator. The length of the cylinger
outside water is l. If the elevator goes down with uniform acceleration a, will l change?
6. Mercury is poured into a vertical U-tube, and water is poured in above it. The level of water is the same in
both arms. Will the level of the water and the mercury be the same if a piece of wood is dropped into one arm
and some water equal in weight to this piece is added to the other ?
7. An air mattress is filled with air to a pressure greater than the atmospheric pressure. When will the air
pressure in the mattress be greater: when a man stands on it or when he lies on it?
8. A man carries a tyre tube and decides to make it lighter by making use of the buoyancy of air. In order to do
this, he inflates the tube, thus increasing its volume. Will his aim be achieved?
9. A tank containing water is placed on a spring balance. A stone of weight w is hung and lowered into the water
without touching the sides and the bottom of the tank. Explain how the reading will change.
10.A ball floats on the surface of water in a countainer exposed to the atomsphere. Will the ball reemain immersed
at its former depth or will it sink or rise somewhat if (a) the container is covered and the air is removed, (b) the
container is covered and the air is compressed?
11. A solid cylinder is placed erect in a container in contact with the base. When liquid is poured into the
container, none of it goes beneath the solid, which remain closely in contact with the base. Is there a buoyant
force on the solid? Explain.
12.An open bucket of water is on a smooth inclined plane, forming an angle with the horizontal. How will the
level rest relative to the plane when the bucket is allowed to slide down the plane?
13.Explain why a uniform wooden stick which will float horizontally if it is not loaded, but will float vertically if
enough weight is added at one end.
14.It is found that a liquid will flow faster and more smoothly from a sealed can when two holes are punctured in
the can than when one hole is made. Explain.
15.Two row boats moving parallel to one another in the same direction are pulled towards one another. Explain.
16.Can the action of a parachute in retarding free fall be explained by Bernoullis theorem ?
17.Can you explain why an object falling from a great height reaches a steady terminal velocity?
18.While taking off would it be better for an airplane to move into the wind or with the wind? While landing?
19.Does the difference in pressure between the lower and upper surfaces of an airplance wing depend on the
altitude of the moving plane? Explain.
20.If p is the pressure of gas inside the exhaust chamber of a rocket and p0 is the pressure of the gas outside the
chamber, the forward thrust on the rocket is 2a (p p0) instead of a (p p0), where a is the area of the orifice.
Why does the factor 2 appear ?
21.The destructive effect of a tornado (twister) is greater near the centre of the disturbance than near the edge.
Explain.
22.Why will a light ping-pong ball dance over a stream of gas or water issuing at high speed from a tube?

www.physicsashok.in 86
FLUID MECHANICS

23.During storms, the strong winds tear off the roofs of thatched houses along the ridge C if the
roof is fastened more firmly at the edges A and B (see figure) than at the ridge. On the other
hand, if the roof is secured more firmly at the ridge C than at the edges, the wind will first lift
the roof up and then carry it away. Explain.
24.A device (see figure) consists of two discs A and B. The lower disc B hangs on three pins
fixed to A along which it can freely move up and down. The upper disc A is provided with
a pipe at its centre. If a compressed air stream is passed through the pipe, the lower disc
begins to knock the upper one. Explain.
25.A balloon filled with air is weighed so that it barely floats in water as shown in the figure.
Explain why it sinks to the bottom when it is submerged more by a short distance.
26.A block os wood is floating on water at 0C with a certain volume V above the water
level. The temperature of water is slowly raised from 0C to 20C. How will the volume
V change with the rise in temperature?
27.A body immersed in a liquid is balanced on a scale. Will the reading on the scale be d if the
liquid is heated together with the body?
28. A glass bulb is balanced by a brass weight in a sensitive beam balance. State what will
happen when the balance is covered by a bell jar which is the evacuated. Explain.
29.A block of ice is floating in a liquid of specific gravity 1.2 contained in a beaker. When the ice melts completely,
will the level in the beaker change ?
30.It is the practice of masons, while laying and levelling the foundations of buildings, to use a long, transparent,
plastic pipe. The theory is that water, seeking a common level, will be at the same level in both halves of the
pipe and thus help to obtain levelling. What happens if a bubble of air is left in the pipe?
31.When a steam boiler in which the pressure of steam is 10 to 15 atm blows up, considerable damage is
caused. Yet when a hydraulic press, in which the pressure is much higher, blows up, the damage caused is not
very great. Why ?
32.A hollow sphere and a solid sphere of the same radius and the same material fall through air from the same
height. Which one arrives on the ground first ? (Neglect viscous effect of the air.)
33.Do rain drops of different sizes reach the earth with the same speed? Explain.
34.A spherical bob made of cork floats half-submerged in a pot of tea at rest on the earth. Will the cork float or
sink aboard a spaceship coasting in free space? On the surface of Jupiter?
35.Why is it easier to curve the flight of a tennis ball than it is to curve that of a baseball?
36.Explain why it is dangerous to stand near a speeding train.
37.Very often a sinking ship turns over as it gets immersed in water. Why?
38.What is the pressure at a depth h below the free surface of a liquid falling freely?
39.A ship gets a large hole O in its underwater portion. In what direction will it begin to
move in the beginning and after a short time? (see figure)

40.The vessel shown in Figure is entirely filled with water. What will happen if the tap S is
opened?

41.A sailor found a small hole in the hole of his vessel, through which water was entering into it.
He tried to stop the flow with a plank but the stream of water pushed the plank away. He managed to bring
the plank close to the hole with the aid of another sailor, and then found that he could hold the plank alone.
Explain why the pressure on the plank is different in the two cases.

www.physicsashok.in 87
FLUID MECHANICS

42.A vessel with a side-cock is filled with water and placed on the platform of a spring
balance. Will the equilibrium change if the cock is opened? The outflowing water falls
on to the same pan on which the vessel is placed.
43.Explain why one has to blow over a piece of paper rather than under it, to keep it horizontal.
44.Why does a flag flutter in a strong breeze ?
45.Why do water jets taper when the tap is almost closed ?
SURFACE TENSION
1. Water can rise to a height h in a certain capillary tube. Suppose that this tube is immersed in water so that only
a height h/2 is above the surface. Will there be a fountain? Explain.
2. Why is moisture retained longer in the soil if it is harrowed?
3. Why are raindrops spherical in shape ?
4. Why does the end of a glass rod become round on being heated strongly?
5. Explain how an iron can be used to remove greasy spots from clothing.
6. Why do drops of water appear at the end of a piece of firewood when it is being dried in the sun while its
other end is in the shadow ?
7. A capillary tube is dipped in water vertically. It is long enough for the water to rise to the maximum height h in
the tube. The length of the portion immersed in water is l < h. The lower end of the tube is closed and then the
tube is taken out and opened again. Will all the water flow out of the tube? Explain.
8. Two capillary tubes A and B are immersed in water One is straight and the other is in the form of a
rectangular U-tube. The tube A is sufficiently long. The lower end of the bent tube is at a depth H. What form
will the meniscus take and will there be any flow of the water? Consider the following five cases :
(a) H > h (b) H = h (c) 0 < H < h (d) H = 0 (e) H < 0
9. Explain how detergents clean dirty clothes.
10.Two soap bubbles A and B of different diameters are blown at the two ends of a bent tube. By opening the
stop cock S, the two bubbles are put in communication. What will happen?
ELASTICITY
1. Stress and pressure are both forces per unit area. Then in what respect does stress differe from pressure ?
2. Is elastic limit a property of the material of the wire ?
3. Which is more elastic, steel or rubber ?
4. Which one has a greater force constant a steel wire or a rubber wire of the same length and radius ?
5. There are two springs of the same material and length but the area of cross-section of one is double that of the
other. Which one will have a greater force constant ?
6. Among solids, liquids and gases, which one can have all the moduli of elasticity ?
7. Among solids, liquids and gases, which possesses the greatest bulk modulus ?

THINKING PROBLEMS SOLUTIONS


FLUID MECHANICS
1. There will be no change in the level of water because in both the positions the cylinder displaces the same
volume of water.
2. No. The equilibrium will not be disturbed, since according to Pascals law, the pressure on the bottom will be
the same at every point.
3. In both the cases the pumps perform the same work, since the same amount of water pumped in rises to the
same level.
4. Considering the upward motion of a column of liquid of depth

www.physicsashok.in 88
FLUID MECHANICS

h, pA p0A (Ah)g = (Ah) a p = p0 + h (g + a).


When the elevator is at rest, pi = p0 + hg
p = p pi = ha. Thus the pressure at the bottom increases.
5. When there is no motion ALg = (L l) Ag, where L = total length of cylinder, A = area of cross-section,
= density of cylinder, = density of water, L = (L l)
When the elevator goes down with acceleration a, the effective acceleration due to gravity on both cylinder
and water is (g a).
AL (g a) = (L l) A (g a) L = (L l)
l = l. So the cylinder will neither sink nor rise.
6. Dropping the piece of wood is exactly the same as adding the amount of water displaced by it, which is equal
to the weight of the body. Hence, if the cross-sections of the arms are the same, the levels of water in both the
arms will stand at the same horizontal level.
7. If the man stands on the mattress, his weight will be distributed over a smaller area, that is, the area of his feet
than when he lies down. Hence, the air pressure in the mattress will be greater when the mass stands than
when he lies on it.
8. No, his aim will not be achieved. The increased buoyancy of air will be made up for by weight of air blown
in. Since the density of compressed air is generally greater than the density of atomspheric air, he will achieve
the opposite of his aim.
9. Make free-body diagrams of the bodies separately and consider their equilibrium. Like all other
forces, buoyancy is also exerted equally on the two bodies in contact. Hence if the water exerts
a buoyant force, say, B on the stone upward, the stone exerts the same force on teh water
downward. The forces acting one the water + container system are : W, weight of the system
downward, B, buoyant force of the stone downward; and the force of the stone downward; and
the force R of the spring in the upward direction. For equilibrium R = W + B.
Thus the reading of the spring scale will increase by an amount equal to the weight of the liquid displaced, that
is, by an amount equal to the buoyant force.

Note : The weight of the stone will not be included in the free-body diagram of the
water + container system because the weight is the action and reaction between the stone
and the earth and not between the stone and the water.
10.Let v be the volume of the ball above the level of water and V its total volume. W = weight of the ball. Then
remembering that, strictly speaking, air also exerts buoyant force
W = v0g + (V v) g
where 0 = density of atmopheric air, = density of water.
Let v be the volume above water level when density of air is changed. then W = vg + (V v) g where
= density of air.
v0g + (V v) g = vg + (V v) g
0 v ' w 0
or v' v w or
w v w
(a) When air is removed = 0
v ' w 0

v w
v < v. So the ball sinks.
(b) When air is compressed

www.physicsashok.in 89
FLUID MECHANICS

w 0
> 0 < 0 1
w
v'
1 or v > v. So the ball rises.
v
11. No. There is no buoyant force on the solid because the liquid is not in contact at the bottom and so it exerts
no upward thrust on it.
12.The level will remain parallel to the plane during the journey down the plane. The component of the force mg
down the plane, which is m sin produces acceleration down the plane. The surface is subjected to the force
mg cos perpendicular to the plane. A liquid surface keeps itself at right angles to the force to which it is
subjected. So the level rests parallel to the plane during the downward journey of the bucket.
13.A floating body float s in stable equilibrium when its centre of gravity is well below the centre of buoyancy,
that is, the centre of gravity of the displaced liquid. When loaded by enough weight the centre of gravity goes
well below the centre of buoyancy.
14.It is found that a liquid will flow faster and more smoothly from a sealed can when two holes are punctured in
the can then when one hole is made. Explain.
15.When they are close to each other, the velocity of the water between them increases, resulting in a fall of
pressure there according to Bernoullis theorem. The pressures from the sides push them together.
16.When the parachute opens out, the pressure of the air above drops and so an upward thrust is called into play
to balance the weight of the parachutist.
17.The viscous force on a body depends on its velocity. The greater the velocity the greater is the viscous force.
When a body falls from a sufficient height, it acquires enough velocity to produce a viscous force that balances
its weight. The resultant force on the body being zero, the body moves with uniform velocity, called terminal
velocity.
18.While taking off an airplane needs maximum dynamic lift, which is associated with an unsymmetrical set of
streamlines relatively close together on the upper side and relatively far apart below. The unsymmetric distribution
of streamlines can be obtained better by moving into the wind. While landing, it should move with the wind
when the streamlines will be uniformly distributed on the two sides of the airplane and so no lift will arise and
the plane will land under gravity.
1 2 p1 1 2 p 2
19.Yes, because v1 v 2
2 2
1

p p 2 p1 v12 v22
2

At high altitudes the density is considerably low and so the pressure difference will depend on the altitude of
the airplane.
20.The formula thrust = pressure difference area holds for fluids at rest. This does not hold for fluid in motion.
p 1 2 p0 1 2
By Bernoullis theorem (a theorem applicable to fluids in streamline motion) V v and AV =
2 2

av where A = area of the chamber, a = area of the orifice. Since A >> a, v >> V, hence V2 may be neglected
in comparison to v2.

v2 2
p p0


dM
Now thrust on the rocket v But the mass (dM) of gas flowing out in time dt is (a v dt).
dt
dM
av
dt
www.physicsashok.in 90
FLUID MECHANICS

2 p p0
thrust = av2 = a 2a p p0

21.Near the edge the velocity is zero and the pressure is the normal atmospheric pressure. At the centre, the
velocity is maximum and so there is maximum drop of pressure at the centre. Thus, the pressure difference
between the central region and the surrounding air being the maximum, the destructive power of a tornado is
maximum at the centre.
22.Since the stream has a high velocity, the pressure inside the stream is below the atmospheric pressure. The
ball is supported from the bottom by the thrust of the stream and kept in position by the lateral pressure
difference.
23.The pressure of the air streaming over the roof is less than that of the stationary air below the roof. This is
according to Bernouliis theorem. The surplus pressure of the stationary air of the room acts normally on the
two inclined halves of the roof.
When the roof is secured at the edges A and B, the thrusts on the two halves will have turning effects about
the edges and so the roof is likely to tear along the ridge. On the other hend if the roof is secured rigidly along
the ridge, the thrusts will then lift the roof against gravity and it will be carried away by the wind.
24.When air is blown through the pipe, the air between the discs is set in motion, resulting in a heavy drop of
pressure. The surplus pressure from below pushes the disc B and knocks it against the upper disc.
25.When it is submerged more by a short distance, the air inside the balloon is compressed resulting in a
reduction of the buoyant force on it. Thus, downward force (weight of the sinker attached + weight of air in
the balloon) being in excess of the buoyant force, it sinks to the bottom.
26.Let V be the total volume of the block of wood and W be the weight of the block. Then by the law of
flotation
(V V) tg = W where t = density of water at tC.
Since wood has a negligible coefficient of expansion V may be considered constant.
V = V W/tg
As the temperature is gradually increased from 0C, t increases and so V increases up to 4C, when the
density of water becomes maximum. Above 4C, t decreases and so V decreases continuously.
27.Reading on the scale
= weight of liquid and beaker + buoyant force
= W = Vtg, where W = weight of liquid + beaker
V = volume of the solid at tC and t = density of liquid at tC
0
Reading W V0 1 t g
1 ' t
where V0 = volume of the solid at 0C, = coefficient of expansion of the solid, = coefficient of expansion
of the liquid
Reading = W + V00 {1 ( )t}g
As t is increased, {1 ( )t} decreases and so the reading also decreases because W is a constant.
28.In air Wg Fg = Wb Fb
where Wg = weight of glass bulb, Fg = buoyant force on the bulb, Wb = weight of brass weights, Fb =
buoyant force on the brass weights
Since the volume of the bulb is greater than the brass weights
Fb < Fg or Wb Wg + Fg < Fg or Wb < Wg
Thus when air is removed the bulb will become heavier than the brass weights.
29.Let H be the height of the liquid in the beaker and H be the height of the liquid after all the ice melts.
Let A be the area of cross-section of the beaker.

www.physicsashok.in 91
FLUID MECHANICS

Let V be the volume of the ice block and V the volume of it inside the liquid. Then
V
Vice g = V 1.2 1000 g V ' ice
1200
Vice
Volume of liquid in the beaker = AH V = AH
1200
When all the ice melts, it is converted into water of the same mass but of volume, say, V.

V 1000 = Vice V '' Vice


1000
Therefore, total volume of liquid after ice melts
Vice Vice
AH AH '
1200 1000
1 1
A H ' H Vice ve
1000 1200
H > H. So the level of the liquid will rise.
30.Let h1 and h2 be the heights above the horizontal line.
Let p1 be the pressure of the entrapped air.
Then p1 p0 = gh2 and also and p1 p0 = gh1
gh1 = gh2 h1 = h2
So the entrapped air will not affect the levelling test.
31.Steam is highly compressible. When a steam boiler blows up, steam expands violently causing considerable
damage while water being incompressible expands nominally with release of pressure. So it does not cause
much damage.
32.Since they are of the same radius, the buoyant force on each of them is the same.
W
W Fb a
g
where W is the weight of the hollow sphere
W'
and W ' Fb a'
g
where W = weight of solid sphere
a a '
W 1 Fb W ' 1
g g
But W > W
a a'
1 1 a a'
g g
Hence the solid sphere arrives at the ground earlier.
33.No. Raindrops of different sizes reach the earth with different speeds. The terminal velocity acquired by a
drop is proportional to the square of its radius.
At the final stage, upward viscous force = downward driving force
4 3
6rv r g v r2
3
34.For flotation
Weight of the body = weight of the liquid displaced
Vg = Vg
where V = volume of the body, V = volume of the submerged portion of the body.
V/V = /

www.physicsashok.in 92
FLUID MECHANICS

Thus, the ratio of the submerged volume to the total volume is independent of the acceleration due to gravity.
Hence in free space and on the surface of Jupiter the cork will neither sink more nor rise more.
35.The pressure difference on the two sides of a tennis ball due to the spin imparted to it is greater than it is in the
case of a baseball because the former is roughter. Hence, it is easier to make the flight a tennis ball curve.
36.Due to the high speed of the train, the pressure of the air between the train and the man may fall considerably
and the pressure difference may be sufficiently high to push the man towards the speeding train.
37.To pass on to the stable equilibrium position.
38.p = p0 (atmospheric pressure). In general p = p0 + (g + a)h. Here a = g. Therefore p = p0.
39.Initially, the ship will move to the right because the in-flowing water (flowing to the left) will exert a force 2pA
to the right, where p is the pressure at a depth h of the hole, and A is its area. As soon as the stream of water
reaches the opposite wall, this wall will be acted upon by the leftward force F = Av2, where v is the velocity
of the stream with respect to the ship. As a result, the motion will begin to retard.
40.Siphonic action will start and water will flow to the reservoir.
41.By Bernoullis theorem the velocity of flow is v = 2gh where h is the height of water above the hole. The
force on the plank = v2S = 2ghS. Once the hole is covered, the velocity of flow is zero and so the force on
the plank = ghS which is just half as great.
42.In the beginning when the stream has not yet reached the pan, equilibrium will be disturbed. The pan will
swing upwards since the water flowing out of the vessel will not longer exert force on its bottom and hence on
the pan. That is, the reading of the balance will decrease.
The moment the stream reaches the pan, equilibrium will be restored. On each element gravity imparts a
2h
downward impulse mg m 2gh (impulse = force time) and on reaching the pan the same element
g
of the liquid experiences on upward impulse m 2gh (impulse = change in momentum). Thus, each element
experiences equal and opposite impulse. since the stream is continuous, the upward and downward impulses
of elements cancel out and o the equilibrium is restored.
The moment when the stream stops flowing, the pan will swing down, since the last elements of the liquid
falling on the pan act on it with a force that exceeds the weight of the elements.
43.When air is blown over the paper there is a drop in pressure and the atmospheric pressure from below
balances the weight of the paper.
44.Due to wind on the two sides of the flag, the difference of pressure produces folds in the flag in compliance
with Bernoullis theorem.
45.This is in compliance with the equation of continuity. V (rate of volume flow) a1v1 = a2v2 i.e., a 1/v. At the
lower end the velocity is greater and so the are of cross-section is less.
SURFACE TENSION
1. No, there will not be a fountain. The curvature of the free liquid surface at the top of the tube will change till
the upward force 2rT cos balances the gravitational pull of the column of liquid of height h/2.
2. When the soil is not harrowed, there are large capillaries in it. Water in the soil rises up the capillary holes to
the surface from where it evaporates continuously. Thus the soil continuously loses water. When harrowed, all
these capillaries are destroyed, and so capillary suction stops altogether. Thus, water is retained longer in the
soil.
3. Because of surface tension of water. Due to surface tension the surface of a small volume of water tends to
occupy the minimum area. The area of a given volume of water has the minimum value for a spherical shape.
4. The end of a glass rod becomes round on being heated because of the surface tension of molten glass.

www.physicsashok.in 93
FLUID MECHANICS

5. The grease melts, and capillary forces carry it to the surface of the cold fabric placed under the clothing,
where it is soaked.
6. The end in the shadow is colder than the end in the sun. The capillary forces drive the water in this direction.
7. No, the water will not flow out. When the tube is taken out, a convex meniscus is formed and
a force due to surface tension is called into play in the upward direction which keeps the water
in the tube. Thus the length of water column remaining in the tube will be l + h.
8. In a capillary tube the height of the liquid that can be retained is given by r2hg = 2rT cos
where is the angle at which the liquid meets the solid.
hgr = 2T cos
(a) Since H > h, surface tension cannot sustain the column of water and so water flows out.
(b) Since H = h, the water does not flow out. The meniscus is convex.
(c) The water does not flow out. The meniscus is convex and is less curved than in the
second case.
(d) The water does not flow out. The meniscus is flat.
(e) The water does not flow out. The meniscus is concave.
9. Water cannot remove grease stains. This is because water does not wet a greasy
spot. The molecules of a detergent are hair pin shaped, experiencing different amounts
of forces at the ends due to water molecules and grease molecules.
The molecules are as if pinned to small globes of greasy dirt and form an interface between water and greasy
dirt globes. The greasy dirt is thus dislodged from the clothes and is carried away with running water.
10.The excess pressure inside a soap bubble is inversely proportional to its radius.
Hence, the pressure inside A is greater than the pressure inside B. Air will flow from
A and B. Therefore, A will become smaller and B will become larger.

ELASTICITY
1. Pressure is the external force per unit area, while stress is the internal force called into play from within a
strained body acting transversely per unit area of the body.
2. No. It depends on the radius of the wire as well.
3. Steel. Elasticity is measured by the stress per unit strain. For the same amount of strain, much greater stress
is produced in steel than in rubber.
4. The steel wire has a greater force constant.
5. The one with greater area will have a smaller force constant.
6. Only solids. Liquids and gases have only bulk modulus.
7. Solids.

www.physicsashok.in 94
FLUID MECHANICS

ASSERTION-REASON

1. Statement-1 : Imagine holding two identical bricks under water. Brick A is completely submerged just
below the surface of water, while Brick B is at a greater depth. The magnitude of force exerted by the
person (on the brick) to hold brick B in place is the same as magnitude of force exerted by the person on the
brick) the hold brick A in place.
Statement-2 : The magnitude of buoyant force on a brick comletely submerged in water is equal to magnitude
of weight of water it displaces and does not depend on depth of brick in water.

2. Statement-1 : Upto elastic limit of a stress-strain curve the steel wire tends to regain its original shape when
stress is removed.
Statement-2 : Within elastic limit the wire follows Hooks law.

3. Statement-1 : When an ideal fluid flows through a horizontal tube of variable cross-section, the pressure
becomes different as different points.
Statement-2 : Raindrops falling from a great height reach the ground with a relatively small velocity. This
phenomena involves the viscosity of air.

4. Statement-1 : When a drop of ink falls on a newspaper, it spreads on it.


Statement-2 : Adhesive force between ink and paper is greater than cohesive force between ink molecules.

5. Statement-1 : In the diagram shown, a cube is floating in water in tilted position, in shown
situation, the cube is in stable position i.e., cube wont sink. C
Statement-2 : The point of application of gravitational force and buoyancy force are passing
through the same line in above described case.

MATCH THE COLUMN

1. Bernoullis equation can be written in the folliwing different forms (column-I). Column-II lists certain units
each of which pertains to one of the possible forms of the equation. Match the unit associated with each of
the equations :
Column-I Column-II
2
v p
(A) + z = constant (P) Total energy per unit mass
2g g
V 2
(B) + P + gz = constant (Q) Total energy per unit weight
2
V2 P
(C) + gz = constant (R) Total energy per unit volume
2

2. In Column-I, a uniform bar of uniform cross-section area under the application of forces is shown in the
figure and in Column-II, some effects/phenomena are given. Match the entries of Column-I with the entries
of Column-II.
Column-I Column-II
(A) F F (P) Uniform stresses developed in the rod

www.physicsashok.in 95
FLUID MECHANICS

(B) F F (Q) Non-uniform stresses developed in the rod


Smooth
(C) F (R) Compressive stresses developed
Smooth
(D) F (S) Tensile stresses developed

3. Match the following :


Column-I Column-II
(A) Splitting of bigger drop into small droplets (P) Temperature increases
(B) Formation of bigger drop from small droplets (Q) Temperature decreases
(C) Spraying of liquid (R) Surface energy increases
(S) Surface energy decreases

4. Column-II shows five systems in which two objects are labelled as X and Y. Also in each case a point P is
shown. Column-I gives some statements about X and/or Y. Match these statements to the appropriate
system(s) from Column-II. [JEE, 09]
Column-I Column-II
Y
(A) The force exerted by X on Y (P) Block Y of mass M left
has a magnitude Mg. on a fixed inclined plane X
X, slides on it with a P
constant velocity.
(B) The gravitational potential energy (Q) Two ring magnets Y and Z, P
of X is continuously increasing. each of mass M, are kept Z
in frictionless vertrical plastic
stand so that they repel each Y
other. Y rests on the base X
and Z hangs in air in equilibrium. X
P is the topmost point of the
stand on the common axis of the two rings. The whole
system is in a lift that is going up with a constant velocity.
(C) Mechanical energy of the system (R) A pulley Y of mass m0 is fixed to
X + Y is continuously decreasing. a table through a clamp X. A P
Y
block of mass M hangs from a
string that goes over the pulley X
and is fixed at point P of the table.
The whole system is kept in a lift
that is going down with a constant velocity.
(D) The torque of the weight of Y about (S) A sphere Y of mass M is put in a
point P is zero. nonviscous liquid X kept in a
container at rest. The sphere is Y
released and it moves down in X
P
the liquid.
(T) A sphere Y of mass M is falling
with its terminal velocity in a
Y
viscous liquid X kept in a container.
X
P

www.physicsashok.in 96
FLUID MECHANICS

LEVEL 1

1. The force due to pressure in a liquid at a given depth below the free surface
(A) is always exerted downward of orienting the surface
(B) is the same in all directions of orienting the surface
(C) equals the total weight of liquid above that depth
(D) depends upon the amount of liquid below that depth
S
2. In a U-tube experiment, a column PQ of water is balanced by a column P
RS of (some liquid see figure). The relative density of the liquid is h1 h2
R Q
(A) h2/h1 (B) (h1 h2)/h1
(C) 2 h1/h2 (D) 2h2/(h1 + h2)

3. The pressure at the bottom of a liquid tank is not proportional to the


(A) height of the liquid above the bottom (B) acceleration due to gravity
(C) density of the liquid (D) area of the liquid surface at the top

4. A sample of a metal weights 210 g in air, 180 g in water and 120 g in an unknown liquid. Then
(A) the density of the metal is four times the density of the unknown liquid
(B) the density of the metal is 3 g/cm3
(C) the density of the metal is 7 g/cm3 (D) the metal will float on water

5. A beaker containing water is put on the platform of a spring balance. A piece of iron suspended from a string
is immersed in the water without touching the sides or the bottom of the beaker. How will the reading on the
spring balance be affected, assuming that no water flows out of the beaker ?
(A) The balance will indicate less weight (B) The balance will indicate more weight
(C) The reading will not change
(D) The reading will be less or more depending the upon position of piece of iron in the liquid

6. Which is based on an application of Bernoullis equation for fluid flow ?


(A) Capillary rise (B) Dynamic lift of an aero plane
(C) Viscosity meter (D) Hydraulic press

7. A steel ball of mass m falls in a viscous liquid with a terminal velocity 4 cm s1. Another steel ball of mass
64 m will fall through the same liquid with a terminal velocity of
(A) 4 cm s1 (B) 16 cm s1 (C) 8 cm s1 (D) 64 cm s1
B
8. In the apparatus shown, initially the stop-cock A is closed and B is open. C A
Now B is connected to a vacuum pump so that pressure is reduced and
the liquid rises up to some point C above A. B is closed at this position of
liquid and A is opened. Then
(A) liquid in the tube will fall till its level is same as outside
(B) liquid will flow out of A
(C) liquid will rise further
(D) liquid will not flow out of A

www.physicsashok.in 97
FLUID MECHANICS

9. In a hydraulic press, f and F are the forces acting on the small piston and the large piston having diameters
d and D respectively. Then f/F is
(A) 2 d/D (B) 2 D2/d2 (C) d2/D2 (D) D/d

10. Two capillary tubes of the same radius and lengths l1 and l2 are fitted horizontally side by side to the bottom
of a vessel containing water. The length of a single tube of the some radius that can replace the two tubes
such that the rate of steady flow through this tube equals the combined rate of flow through the two tubes,
is
2l1 l2 l1 l2 l1 l2
(A) l l (B) l1 + l2 (C) l l (D)
1 2 1 2 2

11. Two capillary tubes of the same length and radii r1 and r2 are fitted horizontally side by side to the bottom
of a vessel containing water. The radius of a single tube that can replace the two tubes such that the rate of
steady flow through this tube equals the combined rate of flow through the two tubes, is
(A) r1r2 (B) (r12 + r22)1/2 (C) r1 + r2 (D) (r14 + r24)1/4

12. A capillary tube is dipped in a beaker containing a liquid. The angle of contact of the capillary with the liquid
is 90. The liquid in the tube will
(A) fall (B) rise
(C) neither rise nor fall (D) may rise or fall depending on the density f the liquid

13. An oil drop is placed on the surface of water. It will


(A) partly be as a spherical droplets and partly as thin film
(B) remain on the surface as a sphere
(C) spread as a thin layer
(D) remain on the surface as distorted drop

14. If the work done in blowing a bubble of volume V is W, then the work done in blowing a bubble of volume
2V will be
(A) (4)1/3 W (B) 2 W (C) 8 W (D) (2)1/3 W

15. Which one of the following curves shows correctly the variation of velocity v with time t for a small spherical
body falling vertically in a long column of viscous liquid ?
v v v v

(A) (B) (C) (D)


t t t
t

16. A cylindrical vessel filled with water is released on an inclined surface of angle
as shown in figure. The friction coefficient of surface with vessel is (<
tan ). Then the constant angle made by the surface of water with the incline Fixed
will be :
(A) tan1 (B) tan1 (C) + tan1 (D) cot1

www.physicsashok.in 98
FLUID MECHANICS

17. A cylindrical container of radius R and height h is completely filled with a 2R


liquid. Two horizontal L shaped pipes of small cross-section area a are
connected to the cylinder as shown in the figure. Now the two pipes are h/2 h/2
opened and fluid starts coming out of the pipes horizontally in opposite R R
directions. Then the torque due to ejected liquid on the system is :
(A) 4 aghR (B) 8 aghR
(C) 2 aghR (D) none of these
F2
18. In the figure shown water is filled in a symmetrical container. Four
pistons of equal area A are used at the four opening to keep the
water in equilibrium. Now an additional force F is applied at each
piston. The increase in the pressure at the centre of the container
F1
due to this addition is F3
(A) F/A (B) 2F/A
(C) 4F/A (D) zero
F4
19. A narrow tube completely filled with a liquid is lying on a series of
cylinders as shown in figure. Assuming no sliding between any surfaces, open to
the value of acceleration of the cylinders for which liquid will not come atmosphere
out of the tube from anywhere is given by
H
gH gH L
(A) (B)
2L L a
2gH gH
(C) (D)
L 2L

20. A square box of water has a small hole located in


one of the bottom corner. When the box is full and
sitting on a level surface, complete opening of the
hole results in a flow of water with a speed v0, as
shown in figure (1). When the box is still half empty, V0
it is tilted by 45 so that the hole is at the lowest
Fig. (1) Initial V0
point. Now the water will flow out with a speed of Later-on

(A) V0 (B) V0/2 (C) V0 / 2 (D) V0 / 4 2

21. A mosquito with 8 legs stands on water surface and each leg makes depression of radius a. If the surface
tension and angle of contact are T and zero respectively then the weight of mosquito is :
(A) 8T . a (B) 16 Ta (C) Ta/8 (D) Ta/16

22. In determination of young modulus of elasticity of wire, a force is applied 4 mm


and extension is recorded. Initial length of wire is 1m. the curve between
Extension
extension and stress is depicted then young modulus of wire will be : 2 mm

4000 KN/m2 8000 KN/m2


(A) 2 109 N/m2 (B) 1 109 N/m2 Stress
(C) 2 1010 N/m2 (D) 1 1010 N/m2 (KN/m2)

www.physicsashok.in 99
FLUID MECHANICS

23. A block of mass M area of cross-section A & length l is placed on smooth Area A
horizontal floor. A force F is applied on the block as shown. If y is young
modulus of material, then total extension in the block will be : F
(A) Fl /Ay (B) Fl / 2Ay
(C) Fl / 3Ay (D) cannot extend l

24. A uniform rod of mass m and length l is rotating with constant angular velocity about an axis which passes
through its one end and perpendicular to the length of rod. The area of cross section of the rod is A and its
youngs modulus is Y. Neglect gravity. The stream at the mid point of the rod is :
m 2 l 3m 2 l 3m 2 l m 2 l
(A) (B) (C) (D)
8AY 8AY 4AY 4AY

25. A ball of mass 10 kg and density 1 gm/cm3 is attached to the base of a container 2 m/s 2
having a liquid of density 1.1 gm/cm3, with the help of a spring as shown in the
figure. The container is going up with an acceleration 2 m/s2. If the spring
constant of the spring is 200 N/m, the elongation in the spring is
(A) 2 cm (B) 4 cm
(C) 6 cm (D) 8 cm

26. In the figure shown, the heavy cylinder (radius R) resting on a smooth
surface separates two liquids of densitieis 2 and 3. The height h for the 2
h R 3 R
equilibrium of cylinder must be

3R 3 3
(A) (B) R (C) R 2 (D) R
2 2 4

27. A conical flask of mass 10 kg and base area 103 cm2 is floating in liquid of
specific gravity 1.2 as shown in the figure. The force that liquid exerts on
curved surface of conical flask is (g = 10 m/s2) 10 cm
(A) 20 N in downward direction
(B) 40 N in downward direction
(C) 20 N in upward direction
(D) 40 N in upward direction

28. In the figure shown, forces of equal magnitude are applied to


the two ends of a uniform rod. Consider A as the cross-section
area of the rod. For this situation, mark out the incorrect F F
statement(s).
(A) The rod is in compressive stress.
(B) The numerical value of stress developed in the rod is equal to F/A.
(C) The stress is defined as internal force developed at any cross-section per unit area.
(D) None of the above.

www.physicsashok.in 100
FLUID MECHANICS

29. For the block shown in the figure, the Poissons ratio is . The
c
body is under state of compression.
F F
In this situation
(A) the volume of the stressed body decreases b
(B) the decrease in length is more, than compensating for increase in area a
(C) the volume of the stressed body increase (D) both (A) and (B) are correct.

30. In above question, if the fractional change in length of block due to application of force F is 0.001 and
corresponding change in volume is 0.0005, find the value of Poissons ratio for the material of the blocks?
(A) 1/2 (B) 1/10 (C) 1/4
(D) Not possible to determine the value of Poissons ratio from the given data

31. The space between two larger horizontal metal plates, 6 cm apart, is filled with a liquid of viscosity
0.8 N-s/m2. A thin plate of surface area 0.01 m2 is moved parallel to the length of the plate such that plate
is at a distance of 2 cm from one of the plates and 4 cm from the other. If the plate moves with a constant
speed of 1 m/s, then
(A) the layer of the fluid which is having maximum velocity is lying mid-way between the plates.
(B) the layers of the fluid which is in contract with the moving plate is having the maximum velocity.
(C) the layer of the fluid which is in contact with the moving plate and is on the side of farther plate is moving
with the maximum velocity.
(D) the layer of the fluid which is in contact with the moving plate and is on the side of nearer plate is moving
with the maximum velocity.

32. The coefficient of restitution for collision of two bodies


(A) depends on Youngs modulius of both the bodies
(B) does not depend on Youngs modulius of both the bodies
(C) depends on Youngs modulus of the lighter body
(D) may or may not depend upon Youngs modulus

33. A heavy block of mass 150 kg hangs with the help of three vertical
wires of equal length and equal cross-section area as shown in the I II III
figure.
Wire II is attached to the midpoint (centre of mass) of block. Take
x x
Y2 = 2Y1. For this arrangement mark out the correct statement(s)
(A) The wire I and III should have same Youngs modulus. Y1 Y2 Y3
(B) Tension in I and III would be always equal.
(C) Tension in I and III would be different. 150 kg
(D) Tension in Ii is 75 g.

34. A metal wire of length L, cross-section area A and Youngs modulus Y is stretched by a variable force F. F
is varying in such a way that F is always slightly greater than the elastic forces of resistance in the wire. When
the elongation in the wire is l, upto this instant
YAl 2 YAl 2
(A) the work done by F is (B) the work done by F is
2L L
YAl 2
(C) the elastic potential energy stored in wire is (D) no energy is lost during elongation
2L
www.physicsashok.in 101
FLUID MECHANICS

35. A solid floats in a liquid is in partially dipped position,


(A) the solid exerts a force equal to its weight on the liquid
(B) the solid doesnt exert any force on the liquid
(C) the solid exerts a force equal to buoyancy force on the liquid
(D) the liquid exerts a force of buoyancy on the solid which is equal to the weight of solid

36. All the four wires in the options below are made up of the same material. Which of these will have the largest
extension, when the same tension is applied ?
(A) Length 200 cm and diameter 2 mm (B) Length 100 cm and diameter 0.5 mm
(C) Length 300 cm and diameter 1 mm (D) Length 50 cm and diamter 0.5 mm

37. A uniform plank of length L and Youngs modulus Y is


pushed over a smooth horizontal surface by a constant
smooth
horizontal force F0. F0
The area of cross-section of the plank is A. The compressive
strain (L/L) of the plank in the direction of the force is :
3F0 F0 F0 2F0
(A) (B) (C) (D)
AY AY 2AY AY
38. Two wires of same material and length but cross-sectional area in the ratio 1 : 3 are used to suspend the
same loads. The extensions in them will be in the ratio :
(A) 1 : 3 (B) 3 : 1 (C) 4 : 1 (D) 1 : 4

39. A wire elongates by l mm when a load W newton is hang from it. If the wire goes over a pulley and two
weights W each are hung at the two ends, the elongation of the wire (in mm) will be :
(A) zero (B) l (C) l / 2 (D) 2l
40. A work of 2 102 J is done on a wire of length 50 cm and area of cross-section 0.5 mm2. If the Youngs
modulus of the material of the wire is 2 1010 N/m2, then the wire must be :
(A) elongated to 50.1414 cm (B) stretched by 0.707 mm
(C) contracted by 2.0 mm (D) none of these

41. The value of Poissons ratio for all practical bodies lies between :
(A) 0 to 0.5 (B) 1 to 1.5 (C) 0.5 to 1 (D) 1 to 1

42. Two bodies of masses 1 kg and 2 kg are connected by a metal wire F = 10N
shown in figure. A force of 10 N is applied on the body of mass 2 kg. 1 kg 2 kg

The breaking stress of metal wire is 2 109 N/m2. What should be Smooth surface
minimum radius of the wire used, if it is not to break ?
(A) 0.23 104 m (B) 4 104 m (C) 5 104 m (D) 5.2 104 m
43. Two wires, ne mode of coper and other of steal are joined
end to end. (as shown in figure). The area of cross-section
of copper wire is twice that of steel wire. Cooper Steel
F F
They are placed under compressive force of magnitudes F.
The ratio of their lengths such that change in lengths of both wires are same is :
(YS = 2 1011 N/m2 and YC = 1.1 1011 N/m2)
(A) 2.1 (B) 1.1 (C) 1.2 (D) 2

www.physicsashok.in 102
FLUID MECHANICS

44. The curve in figure represents potential energy (U) in between two atoms y
in a diatomic molecules as a function of distance x between atoms. The U
atoms are : A B C
(A) attracted when x lies between A and B and repelled when x lies x
between B and C
(B) attracted when x lies between B and C and repelled when x lies between A and B
(C) attracted when they reach B (D) repelled when they reach B a0 = 2m/s2

45. one end of a steel wire is fixed to ceiling of an elevator moving up with an
acceleration 2 m/s2 and a load of 10 kg hangs from other end. Area of cross-
section of the wire is 2 cm2. The longitudinal strain in the wire is :
(Take g = 10 m/s2 and Y = 2 1011 N/m2)
(A) 4 1011 (B) 3 106 (C) 8 106 (D) 2 106

46. Two bodies of masses 1 kg and 2 kg are connected by a steel wire of cross-
section 2 cm2 going over a smooth pulley (as shown in figure). The longitudinal
strain in the wire is : (Take g = 10 m/s2, Y = 2 1011 N/m2)
1 kg
(A) 3.3 107 (B) 3.3 106 (C) 2 106 (D) 4 106 2 kg
47. Water from a tap emerges vertically downwads with an initla speed of 1.0 ms1. The cross-sectional area
of the tap is 104 m2. Assume that the pressure is constant throughout the stream of water, and that the flow
is steady. The cross-sectional area of the stream 0.15 m below the tap is [JEE, 98]
4 2 5 2 5 2
(A) 5.0 10 m (B) 1.0 10 m (C) 5.0 10 m (D) 2.0 105 m2
48. A large open tank has two holes in the wall. One is a square hole of side L at a depth y from the top and the
other is a circular hole of radius R at a depth 4y from the top. When the tank is completely filled with water,
the quantities of water flowing out per section from both holes are the same. then, R is equal to :[JEE,2000]
(A) L / 2 (B) 2L (C) L (D) L/2
49. A hemispherical portion of radius R is removed from the bottom of a cylinder
of radius R. The volume of the remaining cylinder is V and its mass is M. It is h
suspended by a string in a liquid of density where it stays vertical. The upper
surface of the cylinder is at a depth h below the liquid surface. The force on
2r
the bottom of the cylinder by the liquid is [JEE, 01]
2
(A) Mg (B) Mg vg (C) Mg + R hg (D) g (V + R2h)

coin
50. A wooden block, with a coin placed on its top, floats in water as shown in
figure. The distances l and h are shown there. After some time the coin falls
into the water. Then [JEE, 02] l
(A) l decreases and h increases (B) l increases and h decreases h
(C) both l and h increase (D) both l and h decrease

51. Water is filled in a container upto height 3m. A small hole of area a is punched in the wall of the container
at a height 52.5 cm from the bottom. The cross sectional area of the container is A. If a/A = 0.1 then v2 is
(where v is the velocity of water coming out of the hole) [JEE, 05]
(A) 48 (B) 51 (C) 50 (D) 51.5

www.physicsashok.in 103
FLUID MECHANICS

PASSAGE

Comprehension 1
Two cylindrical tanks of radii r and 2r with their bases at the same
level contain a liquid of density to heights H and 3H, respectively r 2r
as shown in figure. The tanks are linked through a pipe of very
3H
small cross-sectional area A. Due to pressure difference liquid starts H
flowing from narrower vessel to broader vessel to equalize the
pressure.
Based on above information, answer the following questions : A
1. The final common level of liquid in both vessels is
(A) 2H (B) 7H/5 (C) 3H/2 (D) 5H/2

2. The time taken for the liquid levels to become equal from initial levels is

5H r 2 5r 2 H r 2 H 8r 2 H
(A) (B) (C) (D)
gA A g A g 5A g

3. The work done by gravity in equalising these levels is


41 8 15
(A) r2 gH2 (B) r2 gH2 (C) r2 gH2 (D) r2 gH2
10 5 2

Comprehension 2
A wooden cylinder of diameter 4r, height h and density /3 is kept on a h1 4r /3
hole of diameter 2r of a tank, filled with water of density as shown in the
h2 h
figure. The height of the base of cylinder from the base of tank is H.
4. If level of liquid starts decreasing slowly when the level of liquid is at a
H
height h1 above the cylinder, the block just starts moving up. Then, value of 2r
h1 is
(A) 2h/3 (B) 5h/4 (C) 5h/3 (D) 5h/2 [JEE, 06]

5. Let the cylinder is prevented from moving up, by applying a force and water level is further decreases.
Then, height of water level (h2 in figure) for which the cylinder remains in original position without application
of force is [JEE, 06]
(A) h/3 (B) 4h/9 (C) 2h/3 (D) h

6. If height h2 of water level is further decreased, then [JEE, 06]


(A) cylinder will not move up and remains at its original potition.
(B) for h2 = h/3, cylinder again starts moving up
(C) for h2 = h/4, cylinder again starts moving up
(D) for h2 = h/5 cylinder again starts moving up

www.physicsashok.in 104
FLUID MECHANICS

LEVEL 2

1. The solid ball in the figure hangs from a spring balance B1 and submerged in a liquid
contained in a beaker placed on a spring balance B2. The mass of the beaker is 1.0 B1

kg and that of the liquid is 2.5 kg. Balance B1 reads 3.5 and B2 reads 8.5 kg. The
volume of ball is 0.005 m3.
(a) Write the buoyant force.
(b) Find the density of the liquid.
(c) What will the balances B1 and B2 read if the ball is pulled up out of the liquid ? B2

2. A uniform rod of length L m, specific gravity 0.5 and mass m is hinged at one end M
at a distance of 1/2 L m below water surface.
(a) What mass M must be attached to the other end of the rod so that 5/6 L m of
the rod gets submerged?
(b) Find the magnitude and direction of the force exerted by the hinge on the rod.

3. A U-tube, placed in a vertical plane, is partially filled with liquid X. A liquid Y, immiscible with X, is next
poured into one side until X rises on the other side by 25 cm. If the density of Y relative to X is 0.8, by what
distance the Y-level will stand higher than the X-level ?

4. A vessel contains a liquid X (density 0.8 g/cm3) over another liquid Y (density X
13.6 g/cm3), not mixing together. A homogeneous sphere floats with half its
volume immersed in upper and the other half in lower liquid. Find the density Y
of the material of the sphere.

5. A liquid flows out of two small hles X and Y in the wall of a tank. The two Y
streams strike the ground at the same point. If the hole X is at a height H
X
h above the ground and the level of water stands at a height H above the
ground, find the height of Y. h

6. Two copper (density ) balls A and B are released in a liquid of viscosity . A r


They are connected by a narrow light rod. How does the tension in the rod
vary with time ? B 2r

7. Water rises to a height of 4 cm in a capillary tube. If the area of cross-section of the tube is reduced to 1/16
of the former value, find how high will the level rise.

8. A soap bubble A of radius r1 and another soap bubble B of radius r2 (> r1) are brought together so that the
combined bubble has a common interface. Find the radius of the interface.

9. Find the work that must be done to get n small equal size spherical drops from a bigger spherical drop
(radius = R) of a liquid having surface tension S. A mercury drop breaks into 8 equal drops. The surface
energy increases by a fact or n. Find n.

www.physicsashok.in 105
FLUID MECHANICS

10. Calculate the work done against surface tension in blowing a soap bubble from a radius of 10 cm to 20 cm
if the surface tension of soap solution is 2.5 102 Jm2.

11. A U-shaped wire is dipped in a soap solution and removed. The thin soap film
formed between the wire and a light slider supports a weight of W = 1.5 102 N
(which includes the small weight of the slider). The length of the slider is 30 cm.
What is the surface of the liquid forming the film ?

12. A tube T of radius r is connected to a bubble of a liquid having surface


tension S. The radius of the bubble is R at time t = 0. Show that it will l
shrink to zero in time t = 2lR4/Sr4, where is the co-efficient of T
R
viscosity of air.

13. Length of horizontal arm of a uniform cross-section U-tube is l = 21 cm and


ends of both of the vertical arms are open to surrounding of pressure 10500
N/m2. A liquid of density = 103 kg/m3 is poured into the tube such that liquid
just fills the horizontal part of the tube. Now one of the open ends is sealed 6 cm
and the tube is then rotated about a vertical axis passing through the other
vertical arm with angular velocity 0 = 10 rad/sec.
21 cm
If length of each vertical arm be a = 6 cm. Calculate the length of air column in
the sealed arm. [g = 10 m/sec2]

14. A simple accelerometer (ann instrument for measuring acceleration) can be


made in the form of a tube filled with a liquid and bent as shown. During
motion, the level of the liquid in the left arm will be at a height h1, and in the ring h1 h2
arm at a height h2. /4 /4
Determine the acceleration a of a carriage in which the accelerometer is installed,
assuming that the diameter of the tube is much smaller than h1 and h2.
.................................................
15. A hemispherical tank of diameter 4m is filled with water. A very small hole is .................................
.............................
punched at 1m above the bottom as shown in figure. Find x (distance at which ......................... .
................... .
. . ....... . . 1m
water strikes the surface). x
16. Two arms of a tube have unequal diameters d1 = 1.0 mm and d2 = 1.0 cm. If water (surface poured into the
tube held in the vertical position, find the difference of level in the tube. Assure the angle of contact to be
zero. T = 0.07 N/m.
17. A block mass 0.5 kg is attached to free end of 80 cm long aluminium wire of
diamter 0.7 mm and suspended vertically. It is now given circular motion into a 85
horizontal plane at a rate such that the wire makes an angle of 85 with the vertical.
Find the increase in length of the wire.
(sin 85 = 0.9962, cos 85 = 0.0872, Youngs modulus of aluminium = 7 1010 Nm2)
18. A metal wire of cross section 4 mm2 and length 5 m is suspended vertically from a rigid support. A bob of
mass 100 kg is now attached at the lower end. If the is bob gets snapped, calculate the thermal energy
generated in the wire. The Youngs modulus is 210 PGa and density is 7800 kg/m3.

www.physicsashok.in 106
FLUID MECHANICS
(GPa)
19. The graph of stress against strain () for a material is shown in the figure. 2.0
The arrow shows the case of increasing stress. How much is the elastic 1.5
energy density in the material when the strain is : 1.0
(a) 0.1 (b) 0.3 and (c) 0.4 ? 0.5

0.1 0.2 0.3 0.4


20. An elastic band of length l is attached to a block of mass M which is held by a person at
rest so that the band is unstretched. The person suddenly releases the block.
(a) Find the maximum elastic energy stored in the band as the block falls.
l
(b) If the block finally comes at rest, how much is thermal energy generated in the bank ? The
Youngs modulus of the band is Y, cross-sectional area is and mass is negligible relative to
that of the block.
21. A column of mercury of 10 cm length is contained in the middle of a narrow horizontal 1 m long tube which
is closed at both the ends. Both the halves of the tube contain air at a pressure of 76 cm of mercury. By what
distance will be column of mercury be displaced if the tube is held vertically ?

22. A sinker of weight w0 has an apparent weight w1 when weighted in a liquid at a temperature t1 and w2 when
weight in the same liquid at temperature t2. The coefficient of cubical expansion of the material of sinker is
. What is the coefficient of volume expansion of the liquid.

23. A point mass m is suspended at the end of a massless wire of length l and cross section A. If Y is the Youngs
modulus for the wire, obtain the frequecy of oscillation for the simple harmonic motion along the vertical line.
24. A cube of wood supporting 200 gm mass just floats in water. When the mass is removed, the cube ruses by
2 cm. What is the size of the cube ?

25. A beaker containing water is placed on the pan of balance which shows a reading of M gms. A lump of
sugar of mass m gms and volume Vcc. in now suspended by a thread in such a way that it is completely
immersed in water without touching the beaker and without any overflow of water. What will be the reading
of the balance just when the lump of sugar is immersed ? How will the reading change as the time passes on?
26. A boat floating in a water tank is carrying a number of large stones. If the stones are unloaded into water,
what will happen to the water level ?

27. Two idendical cylindrical vessels with their bases at the same level each contain a liquid of density (rho).
The height of the liquid in one vessel is h1 and in other is h2. The area of either base is A. What is the work
done by gravity in equalizing the levels when the two vessels are connected.

28. A wooden plank of length 1 m and uniform cross-section in hinged at one end to
the bottom of a tank as shown in fig. The tank is filled with water upto a height 0.5
m. The specific gravity of the plank is 0.5. Find the anlge that the plank makes
with the vertical in the equilibrium position. (Exclude the case = 0)

29. A thin tube of uniform cross-section is sealed at both ends. It lies horizontally, the middle 5 cm containing
mercury and the two equal and containing air at the same pressure P. When the tube is held at an angle of
60 with the vertical direction, the length of the air column above and below the mercury column are 46 cm
and 44.5 cm respectively. Calculate the pressure P in centimeters of mercury. (The temperature of the
system is kept at 30C).

www.physicsashok.in 107
FLUID MECHANICS

LEVEL 3

1. A container of large uniform cross-sectional area A resting on a horizontal surface, holds two immiscible,
non-viscous & incompressible liquids of densities d & 2d, each of height H/2 as shown in figure. The lower
density loquid is open to the atmosphere having pressure P0.
(a) A homogeneous solid cylinder of length L(L < H/2) cross-sectional
area A/5 is immersed such that it floats with its axis vertical at the
liquid-liquid interface with the length L/4 in the denser liquid.
Determine :
(i) The density D of the solid and
(ii) The total pressure at the bottom of the container. H/2 d
(b) The cylinder is removed and the original arrangement is restored.
A tiny hole of area s(s << A) is punched on the vertical side of the H/2 2d h
container at a height h(h < H/2). Determine :
(i) The initial speed of efflux of the liquid at the hole;
(ii) The horizontal distance x travelled by the liquid initially and x
(iii) The height hm at which the hold should be punched so that the liquid travels the maximum distance xm
initially. Also calculate xm. [Neglect the air resistance in these calculations]. [JEE, 95]
2. A cylindrical tank 1 m in radius rests on a platform 5m high. Initially the tank is filled with water to a height
of 5 m. A plug whose area is 104 m2 is removed from an orifice on the side of the tank at the bottom.
Calculate the following :
(i) initial speed with which the water flows from the orifice; [REE, 95]
(ii) initial speed with which the water strikes the ground and
(iii) time taken to empty the tank to half its original value.
3. A thin rod of length L & area of cross-section S is pivoted at its lowest point
P inside a stationary, homogeneous & non-viscous liquid (Figure). The rod is
free to rotate in a vertical plane about a horizontal axis passing through P. The d1
density d1 of the material of the rod is smaller than the dencity d2 of the liquid. d2
The rod is displaced by a small angle from its equilibrium position and then
released. P
Show that the motion of the rod is simple harmonic and determine its angular
frequency in terms of the given parameters.[JEE, 96]
4. A large open top container of negligible mass & uniform cross-sectional area A has a small hole of cross-
sectional area A/100 in its side wall near the bottom. The container is kept on a smooth horizontal floor and
contains a liquid of density and mass m0. Assuming that the liquid starts flowing out horizontally through
the hole at t = 0, calculate
(i) the acceleration of the container and; (ii) its velocity when 75% of the liquid has drained out. [JEE, 97]
5. A nonviscous liquid of constant density 1000 kg/m3 flows in a streamline
motion along a tube of variable cross section. The tube is kept inclined in
Q
the virtical plane as shown in the figure. The area of cross section of the
tube at two points P and Q at heights of 2 meters and 5 meters are P
5m
2m
respectively 4 103 m2 and 8 103 m3.
The velocity of the liquid at point P is 1 m/s. Find the work done per unit volume by the pressure and the
gravity forces as the fluid flows from point P to Q. [JEE, 97]

www.physicsashok.in 108
FLUID MECHANICS

6. A wooden stick of length l, and radius R and density has a small metal piece of mass m (of negligible
volume) attached to its one end. Find the minimum value for the mass m (in terms of given parameters) that
would make the stick float vertically in equilibrium in a liquid of density (> ). [JEE, 99]

7. A uniform solid cylinder of density 0.8 gm/cm3 floats in equilibrium in a


combination of two non mixing liquids A and B with its axis vertical. The densities air h
3 3
of the liquids A and B are 0.7 gm/cm and 1.2 g/cm , respectively. The height
A hA
of liquid A is hA = 1.2 cm. The length of the part of the cylinder immersed in
liquid B is hB = 0.8 cm. B hB
(a) Find the total force exerted by liquid A on the cylinder.
(b) Find h, the length of the part of the cylinder in air.
(c) The cylinder is depressed in such a way that its top surface is just below the upper surface of liquid A and
is then released. Find the acceleration of the cylinder immediately after it is released. [JEE, 02]

8. Consider a horizontally oriented syringe containing water located


at a height of 1.25 m above the ground. The diameter of the plunger D=8mm d=2mm
is 8 mm and the diameter of the nozzle is 2 mm. The plunger is
pushed with a constant speed of 0.25 m/s. Find the horizontal range 1.25 m
of water stream on the ground. Take g = 10 m/s2. [JEE, 04] Ground

9. A solid sphere of radius R is floating in a liquid of density with half of its volume submerged. If the sphere
is slightly pushed and released, it starts performing simple harmonic motion. Find the frequency of these
oscillations. [JEE, 04]

10. A horizontal pipe line carries water in a streamline flow. At a point along the pipe where the cross-sectional
are is 10 cm2, the water velocity is 1 ms1 & the pressure is 2000 Pa. The pressure of water at another point
where the cross sectional area is 5 cm2, is __________ pa.
[Density of water = 103 kg m3] [JEE, 04]

11. A U tube is rotated about one of its limbs with an angular velocity . Find the H
difference in height H of the liquid (density ) level, where diameter of the tube
d << L. [JEE, 05]
L
12. A cylindrical vessel of height 500 mm has an orifice (small hole) at its bottom. The orifice is initialy closed
and water is filled in it up to height H. Now the top is completely sealed with a cap and the orifice at the
bottom is opened. Some water comes out from the orifice and the water level in the vessel becomes steady
with height of water column being 200 mm. Find the fall in height (in mm) of water level due to opening of the
orifice.
[Take atmospheric pressure = 1.0 105 N/m2, density of water = 1000 kg/m3 and g = 10 m/s2. Neglect
any effect of surface tension.] [JEE, 09]

13. Two soap bubbles A and B are kept in a closed chamber where the air is maintained at pressure 8 N/m2.
the radii of bubbles A and B are 2 cm and 4 cm, respectively. Surface tension of the soap-water used to
make bubbles is 0.04 N/m. Find the ratio nB/nA, where nA and nB are the number of moles of air in bubbles
A and B, respectively. [Neglect the effect of gravity.] [JEE, 09]

www.physicsashok.in 109
FLUID MECHANICS

Answer Key

ASSERTION-REASON TYPE

Q. 1 2 3 4 5
Ans. A C B A C

MATCH THE COLUMN

1. [(AQ), (BR), (CP)]


2. [(APR), (BPS), (CQS), (DQR)]
3. [(AQR), (BPS), (CQR)]
4. [(APT), (BQST), (CPRT), (DQ)]

LEVEL1

Q. 1 2 3 4 5 6 7 8 9
A n s. B A D C B B D D C
Q. 10 11 12 13 14 15 16 17 18
A n s. C D C C A D A A A
Q. 19 20 21 22 23 24 25 26 27
A n s. A D B A B B C B A
Q. 28 29 30 31 32 33 34 35 36
A n s. D D C B A ABD ACD ACD B
Q. 37 38 39 40 41 42 43 44 45
A n s. C B B A A A B B B
Q. 46 47 48 49 50 51
A n s. A C A D D C

PASSAGE

Q. 1 2 3 4 5 6
Ans. B D C C B A

www.physicsashok.in 110
FLUID MECHANICS

LEVEL2

1. (a) 50 N, (b) 1.7 104 kg m3, (c) 8.5 kg (B1); 3.5 kg (B2) 2. (a) 7/8 m, (b) 5/4 mg
3. 12.50 cm 4. 7.2 g cm3 5. H h
6rg 4r 3g 3
6. 3 1 e

t

where
4r 7. 16 cm
r1 r2
8. r r 9. 4 SR2 (n1/3 1) ; 2 10. 6 mJ
2 1

g(h 2 h1 )
11. 2.5 102 Jm2 13. 5 cm 14. a h h
2 1

15. x = 2.81 m 16. 25.2 mm 17. 1.67 mm


18. 2.98 J 19. (a) 0.025 Jm , (b) 0.225 Jm , (c) 0.4 Jm3
3 3

2M 2 g 2 l M2g 2 l w 2 w1 (w 0 w1 )
20.(a) (b) 21. 3 cm 22. l (w w ) (t t ) (w w )
AY 2AY 0 2 2 1 0 2

1 YA
23. 24. 10 c 25. Reading will increasing.
2 mL
Ag 2
26. The level of water falls. 27. Wg
4
h1 h 2 28. = 45 29. 75.4 cm

LEVEL3

5 1 g 3
1. (a)(i)D= d, (ii) p = P0+ (6H+L)dg ; (b) (i) v = (3H 4h) , (ii) x = h(3H 4h) , (iii) xmax = H
4 4 2 4

3g d 2 d1 m0
2. (i) 10 m/s, (ii) 14.1 m/s, (iii) 2.5 hr 3. w = 4. (i) 0.2 m/s2, (ii) 2g
2L d1 A
5. +29625 J/m3, 30000 J/m3
6. mmin = r2l ( ) ; if tilted then its axis should become vertical. C.M. should be lower than centre of
bouyancy.
1 3g
7. (a) 0, (b) h = 0.25 cm, (c) a = g/6 (upward) 8. x = 2 m 9. f =
2 2R

L2 2
10.500 Pa 11. H = 12. 6 mm 13. 6
2g

*******

www.physicsashok.in 111
ROTATIONAL MECHANICS

DHANALAKSHMI NAGAR

NEAR ANNAMAIAH CIRCLE,

TIRUPATI.

PH NO. 9440025125
PHYSICS ROTATIONAL MOTION ROTATIONAL MECHANICS
SHEET

A RIGID BODY
There are several bodies around us whose shape and size remains the same because the distance between any
two points of the body remains the same. Such a body is called rigid. An iron rod, steel ball, stone etc. are
almost rigid bodies.
The rigid bodies are so only for small forces. A large force can be applied to a steel rod to elongate or bend it.
So a rigid body is an ideal case when small changes in the shape or size of the body can be just neglected.
We shall consider only such ideal bodies unless stated otherwise.

C1: A (rigid) rod PQ is sliding at the corner of walls as in the figure. The

\\\\\\\\\\\\\\\\\\\\\\\\\\\\
P
velocity of P is downward. Write the velocity of Q rightward when
v
PQ makes an angle with floor.
Sol: As the rod is rigid, the distance between P and Q will not change. For Q u
\\\\\\\\\\\\\\\\\\\\\\\\\\\\\\\\\\\\\\
this, the approach velocity of P towards Q must be zero.
Now approach velocity of P towards Q (see figure)
can be written as vPx vQx. Hence P
vcos(90 ) ucos = 0
vsin = ucos 9
v 0

v sin 90
u = u
cos Q x
= v tan
Another method: Setting up axes XY and writing the
co-ordinates of P and Q, we have
y
PQ = 2
x y 2
P (0, v)

l2 = x 2 y2 y v
Squaring both side, (x, 0)
u x
l2 = x2 + y2 O Q
Differentiating relative to time, x

d 2 d 2 d
(l ) = (x ) + (y2)
dt dt dt
d 2 dx d dy
= (x ) + (y2) [chain rule of differentiation]
dx dt dy dt
dx dy
0 = 2x + 2y
dt dt
Now, P is coming down so y is decreasing:
dy
= v
dt
Q is moving to the right, so x is increasing:
dx
= u
dt
0 = 2x(u) + 2y(v)
y
u = v
x
= (tan)v

www.physicsashok.in 1
PHYSICS ROTATIONAL MOTION ROTATIONAL MECHANICS
SHEET

A rigid body in motion


The motion of a rigid body is controlled by the condition that approach velocity of any of its point towards its
any other point must be zero; only then the disance between two points of rigid body will remain constant
during motion.
We consider now the very important type of rigid body motion- rotation. There will arise two aspects of this
motion-how does the body rotate and why does the body rotate. In the first case we say that we are studying
kinematics of rotation. We define angular position, angular displacement, angular velocity, angular acceleration,
and find their relationships. In discussing the second case, we turn to dynamics of rotation. We here define
torque, angular momentum, moment of inertia and recast Newtons laws of motion for rotating bodies.

Note: Although we like to present the things in the above order, you may opt your own order of studies.

Kinematics of rotation
It is better to locate a rigid body by fixing XYZ-axis in the body (hence called body-axes) and measuring
angles made by these body-axes with axes X0Y0Z0 fixed in laboratory.
Angles made by body-axes with respective axes of lab-frame Z0 Z
are sometimes known as Eulerian angles denoted by , and
:
X X0 : Y0 Y
Y Y0 : O
Z Z0 : X0 X
These specify the angular position of the body. During rotation the angular positions change with time.

Rotation:
If a body moves in such a way that either a point, or a line of the body remains x
stationary, the motion is called rotation.
In the figure point O is at rest. It is called centre of rotation. If a line like XX
be at rest the line is called axis of rotation.
If a rigid body rotates about of point, the points not lying on the centre of
rotation move along the surface of concentric spheres. On the other hand, for O
O axis of
a rigid body rotating about an axis, the points not lying on the axis move in centre of
x rotation
rotation
circles whose centres line in the axis. Kinematics of such points are those of
circular motion. Point P in the figure, for example, has a centripetal acceleration
aC and a tangential acceleration aT, where
aT
v2 P
aC = = 2r .....(i) a
r
aC
dv d
aT = = r .....(ii)
dt dt
a = a c2 a 2T .....(iii)

Angular displacement
If a rigid body turns, all its points suffer the same angular displacement but different linear displacements. The
number of such linear displacements is infinite. Therefore we prefer angular displacement for describing rotation.

We define infinitesinal angular displacement ( d ) as a vector whose direction

www.physicsashok.in 2
PHYSICS ROTATIONAL MOTION ROTATIONAL MECHANICS
SHEET
is along the thumb of right hand with fingers curled in the sense of rotation.
d
It is found that such small angular displacments obey commutative law of vector
addition and therefore qualify as vectors:

d1 + d 2 = d 2 + d1 d

d is a vector.

Since there is no physical effect in the direction of d , it is called a pseudo vector. It is defined along the axis
of rotation using right hand rule and is also called and axial vector. The cross product of real vectors is a pseudo

vector. Thus r p , r F , etc. are psuedo vectors. You need not go into more details at this level. You may
see other pseudo vectors while studying magnetic fields.

Angular velocity

The rate of angular displacement is known as angular velocity. We denote it by .

d
Thus, =
dt

Its direction is along the rotational axis at the given moment and is along d .
Unit rad s1 or s1
Other units
1 rps = 1 revolution (rotation) per second
= 2 rad s1
1 rpm = 1 revolution per minute
2
= rad s1 = rad s1.
60 30

Angular Acceleration
The rate of change of angular velocity is called angular acceleration.

d
=
dt
= d
90 dt


Note that is in the direction d (the change in ). We show the examples for (i) in the direction of ,

(ii) opposite to and (iii) perpendicular to in the figures.

In general and can have any angle between them just as acceleration a and velocity V in linear motion.

V
C2: A particle is moving in a circle. Its angular velocity is and linear velocity is
r
v . The radius vector from centre to particle is r . Determine (a) r

(b) v (c) r v .

Sol: The vector is pointing out of the place of the circle
www.physicsashok.in 3
PHYSICS ROTATIONAL MOTION ROTATIONAL MECHANICS
SHEET

(a)Curl the fingers of right hand from to r . The thumb is found to point

along v .
r
Now we know that | r | = r = v. Hence magnitude also equals the
90
magnitude of v . Two vectors are equal if they have the same direction and
the same magnitude. Hence r

(b) We put the tails of to v together and curl the fingers of right hand from

v . The thumb points towards the centre of the circle, i.e., along r .
The magnitude is v. Thus

r 90 r
v = v ( )
r
v
r
where ( ) is unit vector directed towards the centre.
r
Direction of r V

(c) We put the tails of r and v together and curl the fingers of right v

hand from r to v . The thumb points parallel to . 90

Now, | r v | = rv. Hence
v



r v = rv .




where is unit vector along .


C3: A particle is moving in a circle of radius r with speed v. Find an expression for its acceleration.

Sol: Let v be the velocity, r be the radius vector and be the angular velocity of the particle.

Then V = r
Differentiating both the sides relative to t,

dV d
= ( r )
dt dt

d dr
= r +
dt dt

d dr
Now, is angular acceleration, and is velocity v .
dt dt

dv
Hence = ( r ) + ( v ).
dt

= ( r ) + ( v ) Using rules of cross product, ( r ) is of magnitude r and is either

along + v or along v . This is tangential acceleration:
aT = r
www.physicsashok.in 4
PHYSICS ROTATIONAL MOTION ROTATIONAL MECHANICS
SHEET

( v ) is directed towards centre and is called centrepetal acceleration
v2
aC = v = = 2r..
r

Rotation about stationary axis


Let the axis of rotation be static in laboratory. The rigid body is Z Z
turning at angular velocity along this axis. Then taking Z-axis d =
dt
along the rotation axis, we can write component of angular velocity. axis

The angular acceleration will the also be along the defined
axis and we can write.
d z d 2 dz d
z = = 2 =
dt dt d dt
dz
=
d z

Uniform rotation: If the body turns by equal angles in each equal time interval, howsoever small, the rotation is
said to be uniform.
The angular velocity is constant. Hence
d
= z = constant.
dt
= z t
Thus, angle turned = angular velocity time taken.

C4: A table fan is rotating at uniform angular speed of 200 rpm. How much angle does it turn in 3s?
Sol: Here z = 200 rpm
= 200 revolution one minute

= 200 rads1. [ 1 rpm = rad s1]
30 30
t = 3s
= ?
Using = Zt, we have

= 200 3 radian
30
= 62.8 radian
Note that radian is S.I. unit of angle. You may use radian = 180 to find it in degrees which is not necessary
unles told to do so.
Rotation with uniform angular acceleration: If the angular velocity changes by equal value in each equal
time interval howsoever small, the rotation is uniformly accelerated. The angular accelertion in this case is
constant.
Z = constant
d z
= Z = constant
dt
This equation is similar to
dv x
= ax = constant
dt
Hence we may obtain kinematical equation using similar treatment.

www.physicsashok.in 5
PHYSICS ROTATIONAL MOTION ROTATIONAL MECHANICS
SHEET
C5: Angular displacement of all points of a rigid body is same. In case of non-rigid body greater than distance of the
point from axis of rotation greater will be its angular displacement.

P
2 P
2 2
2
P
1 P
1
1 1

O O
P1 P2 P1 P2
Rigit body ( 1 = 2) Non-rigit body ( 2 > 1)

Equation of Linear Motion and Rotational Motion


Linear Motion Rotational Motion
(a) If acceleration is 0, v = constant and s = vt. (a) If acceleration is 0, = constant and = t.
(b) If acceleration a = constant, (b) If acceleration a = constant then
(u v) (1 2 )
(i) s = t (i) = t
2 2
vu 2 1
(ii) a = (ii) =
t t
(iii) v = u + at (iii) 2 = 1 + t
1 2
(iv) s = ut + (1/2) at2 (iv) = 1t + t
2
(v) v2 = u2 + 2as (v) 22 = 12 + 2
1
(vi) Snth = u + a (2n 1) (vi) nth = 1 + (2n 1)
2 2
(c) If acceleration is not constant, the above (c) If acceleration is not constant, the above equation
equation will not be applicable. In this case will not be applicable. In this case
dx dv d 2 x d d d 2
(i) v = (ii) a = = 2 (i) = (ii) = = 2
dt dt dt dt dt dt
(iii) vdv = ads (iii) d = d

C6: A flywheel of radius 0.5 metre starts from rest with angular acceleration of 0.34 radian/sec2. What will be its
angular velocity after 100 sec ?
Sol. Here 2 = 1 + t
2 = 0 + 0.34 100 = 34 radian/sec.

C7: A circular disc of radius 2 m is revolving at 240 revolutions per minute a torque is applied which slows it at
constant rate of radian/sec2. In what time the disc will completely stop ?
Sol. Here 2 = 1 + t
or 0 = 2n t
2n 2 240
t 8 sec.
60

www.physicsashok.in 6
PHYSICS ROTATIONAL MOTION ROTATIONAL MECHANICS
SHEET
C8: In the above question, calculate how many revolutions the disc makes before it comes to rest.
Sol. If is the angular displacement then,
22 = 12 2
or 0 = 12 2
2
12 4 2 n 2 4 2 240
32
2 2 2 60
No of revolutions = /2 = 16

Example 1: Two gears G1 and G2 have radii and R > r. They are coupled and
angular velocity of G1 varies at uniform rate of . Determine the angle turned
by G2 in time t if the motion starts from rest. T
Sol: Since the contact point of gears is not sliding, the velocity is common. Hence G1 R
G2
v
1 = (magnitude)
r
2 = v
v V R
2 = (magnitude)
R
1 = v
r
1r R
2 = (magnitude) r
R
Differentiating this
r
2 = 1 (magnitudes)
R
Now we can use for gear 2
1
= Z 0t + t2
2 Z
1 r 2
Then = 0+ t
2 R
1 r 2
= t
2 R
1 2 r
[The Gear-1 turns by t while Gear-2 turns times this value.]
2 R

Example 2. A disk 8 cm in radius rotates about its central axis at a constant rate of 1200 rev/min. Determine
(a) its angular speed,
(b) the speed at point 3 cm from its centre,
(c) the radial acceleration of a point on the rim, and
(d) and total distance a point on the rim moves in 2.00 s.
1200 2
Sol. (a) 40 rad / s
60
= 40 3.14 rad/s
= 126 rad/s
(b) v = r = 0.03 126 = 3.78 m/s

www.physicsashok.in 7
PHYSICS ROTATIONAL MOTION ROTATIONAL MECHANICS
SHEET

(c) ar = R2 = 0.08 (126)2 = 1.26 km/s2


(d) = t = 126 2 = 252 radian
S

R
S = R = 0.08 252 = 10.1 m

Dynamics of rotation
The study of rotational motion with explanation of its cause is known as rotational dynamcis.
According to rotational dynamics, torque is responsible for changing angular velocity. Torque is also known
as moment of force.


MOMENT OF FORCE
If a body is free to rotate about a point O at rest in an inertial frame S, and a
force F is applied at A, if produces rotation about O (see figure). We find by O r F
experiment that rotational effect is directly proportional to r, R and sin. We F
define a quantity
= rF sin
We define torque as a vector by r

= r F.
Which is combined effect of force and its action point. We call it moment of force F about O or torque about
O.

The torque of a force F about a point is defined as the cross product of the position vector of action point of
the force and the force vector.

In the figure, torque about O is = r F . Note how the direction is obtained using right hand rule for cross

product and vector is shown at O.
S.I. unit of torque is Nm and [ML2T2] is its dimensional formula.

Moment arm : Moment arm of a torque is the perpendicular (r) dropped on r||
action line of force from the point about which torque is calculated.
r F
moment r
In the figure r is moment arm. We can write r as vector sum of r arm

and r || which is parallel to F . Then

= ( r + r ||) F

= ( r F ) + ( r || F )

= r F Y
F
Giving | | = r Fsin90 = r1F.
(2, 2 3)
C9: A force of 20 N acts at (2, 2 3 ) along the line y = 3 x. Calculate the torque 3x
y=
of this force about the origin.
Z X
(0, 0)

www.physicsashok.in 8
PHYSICS ROTATIONAL MOTION ROTATIONAL MECHANICS
SHEET
Sol: The line of action of F passes through origin. Hence

r = 0

= r F = 0.

Y
C10: In the previous example, calculate the torque of the force F
about ponit (1, 0)m. (20N)

3x
Sol: We have y= (2, 2 3)
r r
= r F 60
Z X
O (1, 0)

= r F

| | = r F
From figure

| r | = (1m) sin60

3
= m
2
F = 20 N
= 10 3 Nm (inward along - Z axis)
In vector from
^
= (10 3 Nm)( k ).


Example 3. A force of F 2 i 3 j N is applied to an object that is pivoted about a fixed axle aligned along the z.

coordinate axis. if the force is applied at the point r 4 i 5 j m, find
(a) the magnitude of the net torque about the z axis and
(b) the direction of the torque vector .

Sol. r F


4 i 5 j 2 i 3 j

12 k 10 k 2 k

Torque about a point due to several forces


If several forces act on a body at various points, their rotational effect about a point is described by net torque.
Net torque or resultant torque is the vector sum of torques due to all the forces about the same point. We

denote it by (gamma).

= 1 + 2 + .......... N


where 1 , 2 , .......... N are torques about the same point due to forces F1 , F2 , ........ FN . If these be acting

at r1 ,
r2 , ...... rN relative to the point about which torque is requested.

www.physicsashok.in 9
PHYSICS ROTATIONAL MOTION ROTATIONAL MECHANICS
SHEET



= r1 F1
1


2 = r2 F2
| | |


N = rN FN
In the special case, when the forces are concurrent, we can obtain net
torque as torque due to net force.

Torque about an axis



Let several forces F1 , F2 ...... be acting on a body. We want to find torque about Z-axis. Then we take any

point O on this axis. The action points have positions r1 ,
r2 ........... relative to O. Give the total of this torque

about O is :
Z




= ( r1 F1 ) + ( r2 F2 ) + ............... Ztorque about
Z-axis
Its Z-component,Z, is called torque about Z-axis.
Torque about an axis is defined as the component of torque vector along the
axis where the torque is calculated about any point on the axis. F2
F1
In the figure, z = cos
O
It may be positive, negative or zero. It is a scalar quantity.

Couple Moment
Two forces of the same magnitude, opposite directions and acting along parallel
lines constitute a couple. The torque due to couple of forces is called couple F
moment.
r2
r2
d
In the figure, F and F constitude a couple.
Their torque about O is given by F
r1



r1
0 = r2 F + r1 ( F )

= (r2 F ) + r1 ( F )

= d F [see figure]

| 0| = d F (one of the forces separation)
Couple moment is independent of the point about which it is calculated. It is a Free vector.
F
Anticlockwise and clockwise torque
The torque which tends to rotate the body clockwise is known as C
clockwise torque. F r grav
O
In the figure grav is clockwise about O. An anticlockwise torque is
mg
that which tends to rotate the body anticlockwise.

In the figure F is anticlockwise torque about O.

www.physicsashok.in 10
PHYSICS ROTATIONAL MOTION ROTATIONAL MECHANICS
SHEET
C11: The more is the value of r, the more will be torque and easier to rotate the body.
(i) The handle of screw driver is taken thick.
(ii) In villages the handle of flour-mill is placed near the circumference.
(iii) The handle of handpump is kept-long.
(iv) The rinch used for opening the tap, is kept-long.

2
C12: Work done by torque = 1
d
= Torque angular displacement.


Example 4. Given that, r 2 i 3 j and F 2i 6 k . The magnitude of torque will be
(A) 405 Nm (B) 410 Nm (C) 504 Nm (D) 510 Nm
Sol. We know that,

rF


2 i 3 j 2 i 6 k


12 j 6 k 18 i

12 j 6 k 18 i

Now | | (12) 2 ( 6) 2 (18) 2

| | 144 36 324 504 Hence (C) is correct.

Example 5. An automobile engine develops 100 kilo-watt, when rotating at a speed of 1800 rev./min. The torque
developed by it will be
(A) 60 Nm (B) 531 Nm (C) 5.31 Nm (D) 6.0 Nm
Sol. The power delivered by the torque exerted on rotating body is given by
P
P = or

Here P = 100 kW = 100,000 Watt
1800
2 60 rad / sec.
60

105
513 N.m Hence (B) is correct.
60 3.14

Equilibrium of rigid bodies


Several forces and torques acting on a rigid body may be balanced. Then the body is said to be in equilibrium
or meahanical equilibrium. It is classified into two parts: rotational equilibrium and translational equilibrium.

Translational Equilibrium:
A body is said to be in translational equilibrium if the vector sum of all the forces be zero. The forces are said

to be balanced and Newtons first law of motion is obeyed if Fi = 0.

www.physicsashok.in 11
PHYSICS ROTATIONAL MOTION ROTATIONAL MECHANICS
SHEET
Rotational Equilibrium:
If the vector sum of all the torques acting on a body about a point be zero, the body is said to be in rotational
equilibrium about that point.
If all the forces acting on the body lie in a plane, we can classify torque into clockwise and anticlockwise. For
rotational equilibrium, we get
Sum of clockwise torques = Sum of anticlockwise torques

Principle of moments
For a body in rotational equilibrium under coplanar forces clockwise torque is balanced by anticlockwise
torque.
In vector form, the vector sum of all the torques is zero.
i = 0

Rotational equilibrium is rotational version of the Newtons first law of motion in which forces acting on a
particle are balanced producing translational equilibrium.

Mechanical Equilibrium
Mechanical equilibrium refers to a system under rotational as well as translational equilibrium. In this case:

or, i = 0 [rotational equilibrium]

or, Fi = 0 [translational equilbrium]
Action point of resultant force.
The action point of resultant force is that point about which torque due to the forces is zero.
Centre of gravity (CG.) is the action point of resultant of gravitational forces
acting on all the particles forming the body.
In the figure, let C be the centre of gravity of the body. Let dm be an elementary
C.G. r

mass at position . Torque d about C is given by C


dc = r ( dm g ) dmg
If the total torque is evaluated, we have

c = r dm g
whole body
By definition, this torque is zero.

If g is uniform over the whole body, then

c = ( r dm g)


= dm r g = 0 [by definition of CG]


Now dm r is the first moment of mass about C.
If C happens to be centre of mass also, the first moment of mass of the whole body about it is zero.

www.physicsashok.in 12
PHYSICS ROTATIONAL MOTION ROTATIONAL MECHANICS
SHEET


dm r = 0
whole body
[C is centre of mass] CM
CG

Thus, for uniform g , centre of gravity coincides with centre of mass.

If g varies over points of the body CG and CM do not coincide as indicated
for a long rod shown in the figure.

C13: (i) When a rigid body is rotating about a fixed axis and a force is applied on it at some point then we are
concerned with the component of torque of this force about the axis of rotation not with the net torque.
(ii) The component of torque about axis of rotation is independent of the choice of the origin O, so long as it
is chosen on the axis of rotation, i.e., we may choose point O anywhere on the line AB.
(iii) Component of torque along axis of rotation AB is zero if

(a) F || AB (b) F intersects AB at some point

(iv) If F is perpendicular to AB, but does not intersect it, then component of torque about line AB = magnitude

of force F perpendicular distance of F from the line AB (called the lever arm or moment arm) of this torque.

(v) If there are more than one force F1 , F2 ........... acting on a body, the total torque will be


= r1 F1 + r2 F2 + ..........
But if the forces act on the same particle, one can add the forces and than take the torque of the resultant
force, or


= r ( F1 + F2 + ......)
(vi) In general, if many forces are acting on a body the net torque is different about different points but if the

body is in translational equilibrium (i.e. F1 + F2 + ...... = 0) the net torque about different points is equal. This
can be shown as follows:

Let the force F1 + F2 ,.......... are acting on a rigid body at position vectors r1 , r2 ,.......... etc. The
position vectors are about the origin O. Then


0 = r1 F1 + r2 F2 + ......... .....(i)

Now, suppose the position vector of an another point P is rp and now we want to find the torque about
this point P, then,


p = ( r1 rp ) F1 + ( r2 rp ) F2 + ........


= ( r1 F1 + r2 F2 + .....) rp ( F1 + F2 + ......) .....(ii)
From equations (i) and (ii) we can see that


0 = p if F1 + F2 + ..... = 0
or the net torque of all the forces about any point is same if the body is in translational equilibrium.

C14: It is correct that whenever the resultant force acting on a body is zero, the body is in static equilibrium.
Sol. No; zero resultant torque in also necessary.

www.physicsashok.in 13
PHYSICS ROTATIONAL MOTION ROTATIONAL MECHANICS
SHEET

C15: A meter rod is acted upon by two forces, 2N and 4N, normal to its
length as in the figure. Determine the distance of the point from CM at
which their resultant force is acting. 2N 2N
Sol: By definition, action point of resultant force is that about which the net 4N
4N
torque becomes zero. 10cm 10cm
CM CM

Case (a): In this case the two torques will cancel each other only if
(a) (b)
the point lies lower than the action point of 1N force. Let its distance
from CM be x cm.
Then
4 (10 x) = 2 (50 x) 2N
40 4x = 100 2x 50 x 4 x(10 x) 2 x(50 x)
4N Ncm
x = 30 cm x
PA 10 x P
CM x
Negative sign shows that the action point P is 30 cm below 2N
the centre of mass.
P
Case (b): In this case let the point P be at distance x-above CM. Equating moment about P.
x 4N
4 (x 10) = 2 (50 x)
10cm
4x 40 = 100 2x CM
6x = 140
x = cm = 36.7 cm
The point P lies 23.3 cm above the CM.

C16: A rectangular block having uniform density has centre of gravity


\
at C. It is placed on horizontal plane. Draw the normal contact \\\\
\\\\
\\\\\\\
C \\\\ \
force and its action point. \\\\
\\\\
C \\\\\\\
\
\\\\
The plane is now tilted by an angle (sec figure (b). Still the (a)
block is at rest under frictional force. Draw the normal force and
its action point.

Sol: N , mg and f and must be concurrent forces for they are balanced.

N Action point
of mg
N
\\\ \\\ Action point
Action point of mg Action point
\\ \\\\\ of friction
of N \\\\
\ \\ \\\\
Action point of N \\\ mg
\\ \\\\\
\\\
mg \\\\
(a) (b)

[The largest angle is that for which mg passes through edge. Above this angle, equilibriumis disturbed].

Example 6. A horizontal force F = mg/3 is applied on the upper surface of a uniform cube of mass m and side a
which is resting on a rough horizontal surface having S = 1/2. The distance between lines of action of mg and
normal reaction N is :
(A) a/2 (B) a/3 (C) a/4 (D) None

www.physicsashok.in 14
PHYSICS ROTATIONAL MOTION ROTATIONAL MECHANICS
SHEET
Sol. In this case, torque of mg is balanced by torque due to normal reaction and applied force F.
a a
N x Fa mg
2 2

a mga mga
or N x
2 3 2

a mga mga mga


or N x
2 2 3 6
But N = mg
a mga
mg mgx
2 6
a a 3a a a
or x Hence (B) is correct.
2 6 6 3

Example 7: A uniform rod of length l lies at rest. Draw forces acting on it.

\\\\\\\\\\\\\\\\\\\\\\\\\\\\\\\
Considering its rotational and translational equilibrium, determine the force of Smooth vertical wall
P
friction acting at Q as a function of angle . How does it varry by decreasing
angle ? What factor will decide minimum value of without slipping ? l
Rough
Sol: Figure shows how forces can be shown to be concurrent. Forces Q
on Q are N and f which adjust to produce the resultant Fc that \\\\\\\\\\\\\\\\\\\\\\\\\\\\\\\\\\\\\\\\\\\


passes through intersection of N and mg . N
P Fc
Considering horizontal and vertical forces, we have
f = N .....(i) Fc N
N = mg .....(ii) N

Considering torque about a horizontal axis passing through Q, mg


mg
l f Q
mg cos = N l sin
2
mg
N= cot .....(iii)
2
cot
Using these,
mg 2 s
f= cot No Slippapge
2

As is decreased, cot increases. Hence frictional force f increases. min 90
Using law of limiting friction
f SN .....(iv)
1
Thus, mgcots mg
2
cot 2S
1
tan 2
s

www.physicsashok.in 15
PHYSICS ROTATIONAL MOTION ROTATIONAL MECHANICS
SHEET
1
min tan 2s
1

1
The factor comes from the cubes centre of gravity and S from nature of
2
contact surfaces. These control min.
You may notice that a person walking on ice slips for larger steps (small , limiting
friction crossed). Small steps (large 0) ensure a friction less than limiting (no slipping).

Example 8. A uniform board of weight W and width 2L hangs from a light, horizontal
beam, hinged at the wall and supported by a cable (fig.) Determine
(a) the tension in the cable and
L
(b) the components of the reaction force exerted by the wall on the beam, in terms of
W, L and . 2L
Sol. Taking torque about O,
2L = (T sin ) 3L
2 L = 3TL sin T
2 = 3 T sin ...(1) Fy
Also, for translatory equilibrium, O
Fx
Fx = T cos ...(2) L
Fy + T sin = ...(3) 2L
From equation (1) w

2
T
3sin
From equation (1), (2) and (3)
2 cot
Fx
3

and Fy
3

MOMENT OF INERTIA
Moment of inertia takes same role in rotatory motion as mass in translatory motion. Smaller the moment of
inertia about an axis, easier to produce rotation about that axis; larger the moment of inertia, harder to produce
rotation. This means that moment of inertia about an axis is a measure of rotational inertia of the body about that
axis. This is its physical meaning.
I = miri2
Radius of gyration:
To gyrate means to rotate. The radius of gyration about an axis is defined as
that distance from the axis where total mass can be centred so as to give the
CM M
same moment of inertia as the actual mass distribution.

I
From figure, radius of gyration K is given by K = . It can be shown that
M
it has dimensions of length and depends on geometry of the system.

www.physicsashok.in 16
PHYSICS ROTATIONAL MOTION ROTATIONAL MECHANICS
SHEET
The radius of gyration of a homogenous sphere about its diameter, for ex, may be written as
2
MR 2 2
K= 5 = R
M 5

Theorems on Moment of Inertia


We shall discuss two important theorems on MI that help it. Theorem of perpendicular axes for laminear
bodies and Theorems of parallel axes.

Theorem of perpendicular axes


Let Z axis be normal to a lamina.
Let us draw XY axes intersecting Z-axis at (0, 0, 0). The points of lamina have z = 0. We shall denote a point
in lamina only by (x, y) a for simplicity without a loss.
Let mi be mass of i-th particle at (xi, yi, 0). Its distance ri from Z-axis is
given by
ri2 = x i2 + y i2 .
Its moment of inertia about Z-axis is mi ri2. For all the N-particles we have
IZ = mi ri2
= mi( x i2 yi2 )
= mi x i2 + mi y i2
= Iy + Ix

Statement: If thre axes x, y and z be drosen such that two of them lie in the plane of a lamina, the sum of moment
of inertia about these two axes is equal to the moment of inertia about the third axis.
This is known as theorem of perpendicular axes.
y
C17: A uniform square plate has moment of inertia Ix and Iz about X and Z axis
shown in the figure. Show that
Iy = Ix = Iz/2.
Also show that rotation of XY axes about t-axis by 45 or by does not CM
x
change the result. z

Sol: Using the theorem of perpendicular axes,


Iz = Ix + Iy .....(i)
By symmetry of mass distribution
Ix = Iy .....(ii)
Iz = 2Ix = 2Iy
Ix = Iy = Iz/2.
If the axes be rotated about Z-axis by , even then
Ix = Iy
Ix+ Iy Iz
Iz
I x I y
2

www.physicsashok.in 17
PHYSICS ROTATIONAL MOTION ROTATIONAL MECHANICS
SHEET
Parallel axes theorem (Steiners theorem) Parallel
axis through CM
Let a body have a moment of inertia Iz about Z-axis and Icm about an
axis passing through CM of the body and parallel to the Z-axis. Let d ri mi
be the separation between the two parallel axes and M be the mass of R i

d ri M
the whole body. i
CM
Then Iz = Icm + Md2.
This is known as parallel axes theorem. d
The moment of inertia of a body about an axis is equal to the sum of its
moment of inertia about an axis parallel to the given axis and passing
through CM, and the product of mass of the body with the square of
distance between the parallel axes.
Proof:
Let i-th particle have mass m. Drop perpendiculars on Z-axis and CM-axis from this particle. The MI about z-
axis is given by (see figure).
Iz = miri .....(i)
as from Z-axis the distance of mi is ri. The summation runs over all the particlar in the body.
Now, construct vectors by arrows as in the figure.

i r = d + Ri
We have

r . ri = ri2 = d2 + 2 d . R i + R i2
i .....(ii)
Using (ii) in (i)

Iz = mi(d2 + R i2 + 2 d . R i )

= (mi)d2 + mi R i2 + 2.mi R i

= Md2 + Icm + 2 d .mi R i .....(iii)

Let ri' be position vector of i-th particle from CM. Let i + R i = ri' .
By definition of CM, the sum of first moment of mass about CM is zero.

mi ri' = 0

mi i + mi R i = 0

Now mi i is a vector along CM-axis while mi R i is normal to this axis. But their sum is zero. We know that

a b & ab =0

a =0= b

mi R i = 0 .....(iv)
From (iii) & (iv), we have
I = Md2 + Icm
This proves the theorem.

www.physicsashok.in 18
PHYSICS ROTATIONAL MOTION ROTATIONAL MECHANICS
SHEET
Some Calculation of Moment of Inertia
1. Moment of inertia of a uniform rod about an axis passing through its
CM and normal to its length:
Let us choose an elementary mass dM at position x on x-axis. Its distance is dM
also x from axis. Its moment of inertia dIz about Z-axis is given by X
x dx x=L
x= L
2
2
M
dIz = dMx2 = dx x2
L
The total MI will be obtained by inetragion over the whole rod (from x = L/2 to x = L/2). Thus
L/ 2 3 1/ 2
M x dx 2M x
Iz =
L

L / 2
=
L 3 1/ 2

M L3 L3
= 3L 8 8

ML2
=
12
This is the MI of rod about CM-axis, normal to rod.

About one of its ends:


If Z-axis passes normally through one end, we may use parallel axes theorem. Z
2
ML CM
Here, Icm =
12 L
d= 2
L
d =
2
Using parallel axes theorem
Iz = Icm + Md2
2
L ML2 ML2
IL = M + =
2 12 3

2. Rectangular Plate:
Axis through CM, in the plane of the plate and parallel to an edge: Let M be the mass of the plate.
M
Then mass per unit area is . Let us consider the plate is made up of thin bars perpendicular to the axis one
lb
of which is shown in the figure and has mass dM. The moment of inertia of this bar about the given axis is given
by X
1 dM
dIx = dM l2 Y
12 CM b
Integrating over all the bars, the total moment of inertia is given by l
1 2
l . dM
12
I =

1
= Ml2
12
www.physicsashok.in 19
PHYSICS ROTATIONAL MOTION ROTATIONAL MECHANICS
SHEET
Similarly, the moment of inertia about an axis (Y) through CM in the plane of the plate normal to breadth b can
be written as
1
I = Mb2
12

Moment of inertia about an axis normal to the plate & passing through CM:
It can be obtained using perpendicular axis theorem. In the figure
1
Ix = Mb2
12
1
Iy = Ml2
12
1 1
Iz = Ix + Iy = Mb2 + Ml2
12 12
1
= M(l2 + b2)
12
If the plate is a square, its sides are equal then,
l=b=a
1
Iz = Ma2
6

3. Rectangular bar: A rectangular bar may be made up of rectangular plates of


mass dM. Then
Z
dM(l 2 b 2 ) dM
Iz = b
12 M

Integrating, we get h

M(l 2 b 2 )
Iz = [Effect of h contained in M]
12

4. Ring or Hoop: An element of mass dM lies at a distance R from Z-axis.


dIz = dMR2 Z
Integrating over the whole ring
R
Iz = R dM = MR2
2
dM

5. MI or Disc about its axis: Let M be mass of uniform disc of radius R. Then mass per unit area is M/R2. Let
the disc be made up of elementary rings of radius r and width dr. The mass of this ring is obtained:
Area = 2rdr
dM = (mass per unit area) area
M dr
= 2 2rdr 2r Z
R dr
r
Now we may write the MI of the ring about Z-axis as R
dIz = dM.r2

www.physicsashok.in 20
PHYSICS ROTATIONAL MOTION ROTATIONAL MECHANICS
SHEET

2M 3
= r dr
R2
Integrating over the whole disc we get
4
2M R
3
2M R 1
Iz =
R2 0 r dr = R 2 4 = 2 MR2 Y

Diametrical axis:
Let X and Y be mutually perpendicular diametrical axes. Then Z
Ix = Iy [by symmetry] X
Also, Iz = Ix + Iy [perpendicular axes theorem]
Now Iz = mR2
1
mR2 = 2Ix = 2Iy
2
1
Ix = Iy = mR2
4

6. Uniform solid cylinder: MI about its axis:


We may imagine the cylinder to be made up of large no of elementary discs, one of the them has mass dM. Its
MI about z axis is given by
1 Z
dIz = dMR2
2 R
Integrating over the whole cylinder,
M
1 2 1
Iz = R dM = MR2
2 2 dM
MI about perpendicular bisector axis: It is equal to
MR 2 Ml 2
Iz = +
4 12

7. Homogenous sphere, diametrical axis:


Let us consider a homogenous sphere of mass M and radius R. Its moment of inertia about diametrical axis
shown in the figure may be calculated as follows.
Let us consider the sphere to be made up of disc like slices one of which is located at position z. Its thickness
is dz and radius is R 2 z 2 .
The MI of this elementary disc is
Z
1
dIz = Mass (radius)2 Z=R
2 2 2
dz R z
1
= (R2 z2)dz (R2 z2) R
2 z=0

M
where = density = z = R
4
R 2
3

www.physicsashok.in 21
PHYSICS ROTATIONAL MOTION ROTATIONAL MECHANICS
SHEET

2 2
dIz = (R z )dz
2
Integrating over the whole sphere
R
4
Iz = (R 2R 2 z 2 z 4 ) dz
2 R

4R 2
R
2
R

= R
dz 2R z dz z 4dz
2 R R R
5 4 5 2 5
=
2 2R 3 R 3 R

30 20 6 5
= R
2 15
8 5
= R
15
Putting the value of , we get
2
Iz = mR2.
5

Moment of Inertia for simple geometrical objects


Some bodies of simple geometry are thin uniform rod, uniform rectangular plate, circular ring or hoop, uniform
circular disc, uniform solid cylinder, homogeneous sphere, hollow sphere, right circular cone etc. We shall give
you values of moment of inertia of these about frequently used axes.

Table MI of some regular bodies


Body Position of axis Moment of inertia
Z

Laminar body Y Iz = Ix + Iy (perpendicular axis theorem)


X
laminar body

CM
Any body IZ = ICM + Md2 (Parallel Axes theorem)
M
d

Body Position of axis Moment of inertia

www.physicsashok.in 22
PHYSICS ROTATIONAL MOTION ROTATIONAL MECHANICS
SHEET

90
CM
ml 2
Uniform rod Iz =
l 12
Total mass = m

Z
ml 2
Iz =
l 3

ml 2 2
Iz = sin
3


ml 2 2
Iz = sin
12
CM

CM b

l
[The axis is the plane
of plate]

X CM b ml 2
Uniform rectangular plate Iy =
l 12

Z X mb 2
Iz =
12
[Axis Z is normal to plate]

m 2
Iz = (l + b2)
12

Z
COM
R
Ring or Hoop Iz = mR2
[Z-axis : through CM and
normal to plane of hoop]

CM
1
R I= mR2
2
[Axis as diameter]

www.physicsashok.in 23
PHYSICS ROTATIONAL MOTION ROTATIONAL MECHANICS
SHEET

Z
COM
R 1
Uniform Circular Disc Iz = mR2
2
[Z-axis: through CM
and normal to disc]

1
I= mR2
4
[Axis as diameter]
l
R
CM 1
Right Circular Cylinder [Axis through CM along
I= mR2
2
the length]

l
R
CM mR 2 ml 2
I= +
4 12
[Axis through CM normal to
length]

CM 3
Right Circular Cone I= mR2
10

R CM
2
Spherical Shell I= mR2
3
[Diametrical Axis]

2
Solid homogenous sphere I= mR2
R 5
[Dimetrical axis]

R2 5 5
R2 2 R 2 R1
Party Hollow sphere I = m R3 R3
5 2 1
[Dimetrical axis]

www.physicsashok.in 24
PHYSICS ROTATIONAL MOTION ROTATIONAL MECHANICS
SHEET
C18: Theorem of parallel axes is applicable for any type of rigid body whether it is a two dimensional or three
dimensional, while the theorem of perpendicular axes is applicable for laminar type or two dimensional bodies
only.

C19: In theorem of perpendicular axs, the point of intersection of three axes (x, y and z) may be any point on the
plane of body (it may even lie outside the body). This point may or may not be the centre of mass of the body.

C20: Moment of inertia of a part of a rigid body (symmetrically cut from


the whole mass) is the same as that of the whole body. e.g., in figure R
O M
(a) moment of inertia of the section shown (a part of a circular disc)
about an axis perpendicular to its plane and passing
1
through point O is MR2 as the moment of inertia of the complete disc (a)
2 (b)

1
is also MR2. This can be shown as in figure. Suppose the given section
2
1
is th part of the disc, then mass of the disc will be nM.
n
1
Idisc = (nM)R2
2
1 1
Isection = Idisc = MR2
n 2

C21: Calculate the M.O.I. of a circular disc about an axis passing through its centre and perpendicular to its plane;
mass of the disc is 2 kg and its radius is equal to 50 cm.
1
Sol. Here I= MR2
2
2
1 50
I 2 0.25 kgm 2
2 100

Example 9. In the above question calculate its M.O.I. (1) about a tangent in its own plane; and (2) about a tangent
r to its plane.
Sol. (1) From the theorem of parallel axis, M.O.I. about tangent in its own plane.
I = M.O.I. about a parallel diameter + MR2
1
I MR 2 MR 2
4
2
5 5 50
I MR 2 2 0.625 kgm
2

4 4 100
(2) M.O.I. about tangent r to its plane.
= M.O.I. about r axis + MR2
1 3
MR 2 MR 2 MR 2
2 2
2
3 50
2 0.75 kgm 2
2 100

www.physicsashok.in 25
PHYSICS ROTATIONAL MOTION ROTATIONAL MECHANICS
SHEET
Y X
C22: The figure represents a thin circular disc of mass 4 kg and radius 0.60 metre. Calculate
its moment of inertia about an axis XX, passing through A and r to its plane.
O A
Sol. From theorem of parallel axis, M.O.I. about XX.
0.2m
= M.O.I. about YY + Mr2
1
= MR2 + Mr2 Y X
2
1
= 4 (0.6)2 + 4 (0.2)2 = 0.88 kgm2.
2

C23: Calculate the M.O.I. of a uniform cylinder of mass 6 kg, redius 0.4 metre and length 1 metre about an axis
passing through its centre and r to its length.

L2 R 2 2
Sol. I M 0.74 kgm
12 4

C24: There is a uniform circular disc of mass 10 kg and radius 2 metre. Calculate the radius of gyration if it is
rotating about an axis passing through its centre and r to its plane.
1
Sol. Here I= MR2
2
R 2
K= = = 1.414 m
2 2

C25: A solid sphere of mass 6 kg and radius 2 metre is rotating about its diameter; then calculate the radius of
gyration.
1
Sol. Here I= MR2
2
2 2
K R 2 1.26 metre
5 5

C26: For points masses each of mass m are attached to a massless string. Calculate the X
M.O.I. of the system about XX. m m m m
Sol. M.O.I about XX L L L
= mL2 + 0 + mL2 + m(2L)2 X
= 6 mL2.

C27: Three point masses each of mass m are kept as shown in the figure, the string is A m
massless. Calculate the M.O.I. of the system about an axis passing through (1) AB, 10m
8m
(2) BC and (3) about an axis passing rly through A. m
B
Sol. (1) M.O.I about AB = 0 + 0 + m(6)2 = 36 m m 6m C
(2) 64 m (3) 164 m

www.physicsashok.in 26
PHYSICS ROTATIONAL MOTION ROTATIONAL MECHANICS
SHEET
Example 10. Three bodies have equal masses m. Body A is solid cylinder of radius R, body B is a square lamina
of side R, and body C is a solid sphere of radius R. Which body has the smallest moment of inertia about an
axis passing through their centre of mass and perpendicular to the plane (in case of lamina)
(A) A (B) B (C) C (D) D
MR 2
Sol. For A IA
2
m 2 MR 2
For B IB
12

R R2 6
2
For C IC mR 2
5
where IA > IC > IB Hence, (B) is correct.
A B
Example 11. A point mass mA is connected to a point mass mB by a massless rod of length
l as shown in the figure. It is observed that the ratio of the moment of inertia of the
system about the two axies BB and AA, which is parallel to each other and perpendicular mA l mB
I BB
to the rod is I = 3. The distance of the centre of mass of the system from the mass
AA
A is A B
(A) (3/4)l (B) (2/3)l (C) (1/2)l (D) (1/4)l
2
Sol. IAA = mBl
IBB = mAl2
IBB
3
I AA
mA
3 mA = 3mB
mB
But mAxA = mB(l xA)
3mBxA = mB (l xA)
3xA = l xA
4xA = l
l
xA Hence (D) is correct.
4

Example 12. Three uniform rods each of mass m and length L metre are connected to form an equilateral triangle.
Calculate the M.O.I. of the system about an axis passing through one of the vertex and r to plane of the
triangle.
1
Sol. M.O.I. of the rod AB, and AC about the given axis are each equal to mL2. and M.O.I. of the rod BC about
3
the given axis (from the theorem of parallel axis) is
2
1 1 3
mL2 mx 2 mL2 m L
12 12 2

www.physicsashok.in 27
PHYSICS ROTATIONAL MOTION ROTATIONAL MECHANICS
SHEET

1 2 3mL2 10 2
mL mL
2 4 12 A
Hence, M.O.I. of the whole system about given axis
m, L L, m
1 1 10 18 3 x
I mL2 mL2 mL2 mL2 mL2
3 3 12 12 2 D
B
m, L C
C28: The figure below represents two spheres of mass M and radius R. They are kept in Y
contact as shown. Calculate M.O.I. about axis YY.
7 7
Sol. I mR 2 mR 2 Y
5 5
14
I mR 2
5
M
Example 13. Four spheres each of mass M and radius r are kept with their centres on the A L BM
corners of a squre of side L. Calculate M.O.I. of the system about an axis passing
L
through one side of the square. L
Sol. M.O.I. about AB
D L C
M M
2 2 2 2
Mr 2 Mr 2 Mr 2 ML2 Mr 2 ML2
5 5 5 5
8 2

Mr 2 2ML2 M 4r 2 5L2
5 5

C29: Two point masses are kept at distance of 9 metre on a thin uniform rod of negligible mass. The rod remains
horizontal if it is pivoted at A. Calculate M.O.I. about a vertical line XX through the pivot A.
Sol. Since the rod remains horizontal about A, hence
X
1 g(9 y) = 2g y 1kg 2kg
y = 3 metre A
I = 2(3)2 + 1(6)2 9y y
I = 54 kgm2 X

Example 14. The M.O.I. of a thin uniform rod of mass M and length L about an axis passing through its centre is
I1. This rod is bent in form of a ring, and if I2 is the M.O.I. of the ring formed about its centre then compare I1
and I2.
1
Sol. For thin rod, I1 ML2
12
For the ring, 2r = L
L
r
2
2
L
I2 Mr 2 M
2
I1 1 4 2 2
ML2
I 2 12 ML2 3

www.physicsashok.in 28
PHYSICS ROTATIONAL MOTION ROTATIONAL MECHANICS
SHEET
Example 15: Three rods each of mass m and length l are joined together to form A
an equilateral triangle as shown in figure. Find the moment of inertia of the
system about an axis passing through its centre of mass and perpendicular to
the plane of the triangle. CM
Sol: Moment of inertia of rod BC about an axis perpendicular to plane of triangle
ABC and passing through the mid-point of rod BC (i.e., D) is B C

ml 2 A
I1 =
12
From theorem of parallel axes, moment of inertia of this rod about the asked axis is CM
I2 = I1 + mr2
r
2 30
ml 2 l ml 2 B D C
m
12 2 3 6
Moment of inertia of all the three rods is
ml 2 ml 2
I = 3I2 = 3 6 =
2

Example 16. Three identical thin rods each of mass m & length l are placed along x, y & z-axis respectively they
are placed such that, one end of each rod is at origin O. Then moment of inertia of this system about z-axis is
ml2 2m l 2 2
ml2
(A) (B) (C) m l (D)
3 3 4
Sol. Since the one end of each rod is at origin O
ml2
Ix I y
3
According to perpendicular theorm.
Iz = Ix + Iy
m l 2 m l 2 2m l 2
Iz Hence (B) is correct. x
3 3 3

Example 17: Find the moment of inertia of a solid sphere of mass M and radius R
about an axis XX shown in figure.
Sol: From theorem of parallel axis,
IXX = ICM + Mr2 x
2
= MR2 + MR2 x
5
7
= MR2 CM
5

x r=R

www.physicsashok.in 29
PHYSICS ROTATIONAL MOTION ROTATIONAL MECHANICS
SHEET

Example 18:Consider a uniform rod of mass m and length 2l with two particles of
mass m each at its ends. Let AB be a line perpendicular to the length of the rod
A
and passing through its centre. Find the moment of inertia of the system about
AB. l l
m m
Sol: IAB = Irod + Iboth particles B
2
m(2l )
= + 2(ml2)
12
y
7 B
= ml2
3

Example 19: Find the moment of inertia of the rod AB about an axis YY as
shown in figure. Mass of the rod is m and length is l. A
m
Sol: Mass per unit length of the rod =
l
y
m
Mass of an element PQ of the rod, dm = dx
l
Perpendicular distance of this elemental mass about yy is r = x sin.
Moment of inertia of this small element of the rod (can be assumed as a point mass) about yy is,
m m y
dI = (dm)r2 = dx (x sin)2 = sin2 x2 dx B
l l
Moment of inertia of the complete rod,
r
x l 1 Q
m 2
I = dI = sin2 x dx x
P
x 0 l 0
A

ml 2
= sin2
3 y

ml 2
Note: (i) I = 0 if = 0 (ii) I = if = or 90
3 2

Example 20. Two rods of equal mass m and length l lie along the x-axis and y-axis with their centres origin. What
is the moment of inertia of both about the line x = y :
ml2 ml2 ml2 ml2
(A) (B) (C) (D)
3 4 12 6 y
2
ml
Sol. Ix I y sin 2
12
45
2 2 2
ml ml 2m l x
I , I
24 24 24

ml2
I Hence (C) is correct.
12
www.physicsashok.in 30
PHYSICS ROTATIONAL MOTION ROTATIONAL MECHANICS
SHEET
Example 21. A rigid body can be hinged about any point on the x-axis. When it is hinged such that the hinge is at
x, the moment of inertia is given by
I = 2x2 12x + 27
the x-coordinate of centre of mass is
(A) x = 2 (B) x = 0 (C) x = 1 (D) x = 3
2
Sol. I = 2x 12x + 27
dI
or 4x 12
dx
For minimum moment of inertia
dI
0
dx
4x 12 = 0
x=3
But M.I. about an axis passing through centre is minimum. Hence (D) is correct.

Example 22. The figure shows a uniform rod lying along the x-axis. The locus of all the y
points lying on the xy-plane, about which the moment of inertia of the rod is same as
that about O is
(A) an ellipse (B) a circle
O x
(C) a parabola (D) a straight line
ml 2
Sol. I0
3
2
l
Here r x y2
2 P (x, y)
r y
m l2
IP mr 2
12 l/2 x l/2

But I0 = IP
2
m l2 m l2 l
or m x y 2
3 12 2
2
m l2 m l2 l
or m x y 2
3 12 2
2
3m l 2 l
or m x y 2
12 2
l2 l2
or x xl y 2
2

4 4
or x2 + y2 xl = 0
l2 l2
or x 2 y 2 xl 0
4 4
2
l 2 l2
or x y
2 4
This is an equation of circle. Hence (B) is correct.

www.physicsashok.in 31
PHYSICS ROTATIONAL MOTION ROTATIONAL MECHANICS
SHEET
Question No. 23 to 26 Y
The figure shows an isosceles triangular plate of mass M and base L. The angle at the l
apex is 90. The apex lies at the origin and the base is parallel to X-axis. M
Example 23. The moment of inertia of the plate about the z-axis is
ML2 ML2
(A) (B)
12 24 X

ML2
(C) (D) None of these
6
h
Sol. tan 45
l/2
l/2 D l/2
l A B
h 45 45
2
In ACD, h
y l y dx
tan 45
x 2h
l
y x C
2l /2
y=x
dA = 2y dx = 2xdx
dm = 6 dA = 2 x dx
2

dI dm
2y
dm(x) 2

12
4x 2
dI 2xdx 2xdx x 2
12
2x 3 dx
dI 2x 3 dx
3
8
dI x 3 dx
3
l /2 8 3 8 x4
0 3
x dx
3 4
8 l4 l4

3 64 24
M
Here
area of plate
M 4M
2
1 l l
l
2 2
4M l 4 M l 2
I
l 2 24 6
ML2
I ( l = L ) Hence (C) is correct.
6
www.physicsashok.in 32
PHYSICS ROTATIONAL MOTION ROTATIONAL MECHANICS
SHEET
Example 24. The moment of inertia of the plate about the x-axis is
ML2 ML2 ML2 ML2
(A) (B) (C) (D)
8 32 24 6
Sol. dI = (dm) x2
dI = (2x dx) x2
dI = 2 x3 dx
l /2
I 2 x 3 dx
0

x4 l4
I 2 2
4 64
l 4 l 4 l 4 4M
I 2
64 32 32 l 2
Ml2
I Hence (A) is correct.
8

Example 25. The moment of inertia of the plate about its base parallel to the x-axis is
ML2 ML2 ML2
(A) (B) (C) (D) None of these
8 36 24
2
l
Sol. dI dm x
2
2
l/2 l
I dm x
x0 2

ML2
I Hence (C) is correct.
24

Example 26. The moment of inertia of the plate about the y-axis is
ML2 ML2 ML2
(A) (B) (C) (D) None of these
6 8 24
dm 2 4dmx 2
Sol. dI 2y
12 12
l /2 dmx 2
I
0 3
Ml 2
I Hence (B) is correct.
8

www.physicsashok.in 33
PHYSICS ROTATIONAL MOTION ROTATIONAL MECHANICS
SHEET

TORQUE AND NEWTONS SECOND LAW


In section 2.4, we have discussed about the turning effect of a force, torque. We know that for a body to be in
totational equilibrium, the sum of all the torque acting on it must be zero. Now what happens if net torque is not
zero. The case is similar if net forces acting on a body is not zero, the body wil accelerate according to
Newtons second law. In rotational motion also the law holds good, but require some modification as when net
torque on a body about a given axis of rotation is not zero, body will have angular acceleration. The magnitude
of angular acceleration can be obtained by Newtons Second law in rotational motion.
In translational motion we use F = ma ...(1)
In rotational motion we use =I ...(2)
Let hand side is the net torque acting on the body and on right hand side I is the moment of inertia of the body
about the given axis and is the angular acceleration of the body.
This corresponds to Newtons second law for translational motion, a F, where torque has taken the place of
force and correspondingly the angular acceleration takes the place of the linear acceleration a. In the linear
case, the acceleration is not only proportional to the net force but is also inversely proportional to the inertia of
the body, which we call its mass m, thus we can write a = F/m. In case of rotational motion moment of inertia
plays the role of mass.
As we have discussed that the rotational inertia of an object depends not only on its mass, but also on how that
mass is distributed. For example, a large diameter cylinder will have greater rotational inertia than one of equal
mass but smaller diameter (longer than previous). The former will be harder to start rotating, and harder to stop
as its moment of inertia is larger. When the mass is concentrated farther from the axis of rotation, the rotational
inertia is greater. This is the reason why in rotational motion the mass of a body can not be considered as
concentrated at its centre of mass.
F1
For understanding the application of Newtons second law in rotational motion,
consider the rod shown in figure, pivoted at an end about which it can rotate. If r1
r2
two forces F1 and F2 are applied on it as shown from opposite directions, tend to
rotate the rod. The respective torque of these forces are given as 90

Clockwise torque due to force F1 is 1 = F1 sin . r1

Anticlockwise torque due to force F2 is 2 = F2 . r2

Let 1 > 2, the rod rotates in clockwise direction with an angular acceleration , which can be shown from
equation (2), as net torque is in clockwise direction, we have

M l2
F1r1 sin F2 r2 ...(3)
3

M l2
As moment of inertia of the rod of mass M and pivoted at one of its end is given as . Above equation (3)
3
will given initial angular acceleration of the rod.

www.physicsashok.in 34
PHYSICS ROTATIONAL MOTION ROTATIONAL MECHANICS
SHEET
APPLICATION OF = I
For problems based on pure rotation :
Solution technique :
Step (i) Draw the free body diagram (if required)
Step (ii) Identify, the axis of rotation and angular acceleration w.r.t. axis of rotation.
Step (iii) Calculate net torque w.r.t. axis of rotation and then us = I with + ve/ ve sign convention.
F
Example 27. A rod of length L and mass M is free to rotate about a vertical axis passing
t=t
through its one end. A constant force F starts acting in horizontal plane. Initially the
L
direction of F is perpendicular to length of the rod. Calculate angular velocity of rod
as a function of angular displacement at any time t.
Sol. Let = angular displacement of rod at time t F
L cos
= angular velocity at this time.
= angular acceleration.
ML2
= FL cos =
3
3F cos
=
ML
d 3F cos

d ML
3F
d ML cos d
2 3Fsin
C
2 ML
When = 0 ; = 0 ; C=0
6Fsin

ML

KINETIC ENERGY OF ROTATING BODY


A rigid body is made up of large number of particles. The kinetic energy of a rotating body is the sum of kinetic
energy of its particles.
Let be the angular velocity. Then kinetic energy of i-th particle at distance ri from rotation-axis is

1
K mi vi2
2
where ri = vi. The sum of such terms over the whole body gives
1
K = m (r )2
2 i i
1
= m 2 r 2 mi
2 i i ri
1
= (mi ri2 )2
2

www.physicsashok.in 35
PHYSICS ROTATIONAL MOTION ROTATIONAL MECHANICS
SHEET
The quantity mi ri2 is the moment of inertia of the body about rotation axis, denoted by I. Then
1 2
K = I
2
Multiplying and dividing by I,
1 I2 2
K =
2 I
L2
K =
2I
Here L is angular momentum about the axis.

C30: Whether a particle is in translational motion, rotational motion or in both it merely depends on the reference
point with respect to which the motion of the particel is desribed.
For example: Suppose a particle P of mass m is moving in a straight line as shown in figure (a), (b) and (c).
v cos

P v
P v P
90 v
A r r
B v sin
C
(a) (b) (c)

Refer figure (a): With respect to point A, the particle is in pure translation motion. Hence, Kinetic energy of the
particle can be written as
1
K.E. = mv2
2
Refer figure (b): With respect to point B, the particle is in pure rotational motion. Hence, the kinetic energy of the
particle can be written as
2
1 1 v
K.E. = l2 = (mr2)
2 2 r
1
= mv2
2
Refer figure (c): With respect to point C, the particle can be assumed to be in rotational as well as translational
motion. Hence, the kinetic energy of the particle can be written as
1 1
K.E. = m(v cos)2 + I2
2 2
2
1 1 v sin
= m(v cos)2 + (mr2)
2 2 r
1 2
= mv
2
Thus, in all the three cases, the kinetic energy of the particle comes out to be the same.

C31: If a particle is moving in a circle it is in pure rotational motion about the centre of the circle, while for a moment
it may be in pure translational motion about some other point.
www.physicsashok.in 36
PHYSICS ROTATIONAL MOTION ROTATIONAL MECHANICS
SHEET
C32: If a particle P is moving in a circle, its angular velocity about centre of the circle (c) is two times the angular
velocity about any point on the circumference of the circle (0) P
O
or c = 20
This is because PCP = 2POP (by property of a circle) P
C

PCP POP
c = t pp , 0 = t pp
From these relations we can see that c = 20.
C33: If a rigid body is rotating about a fixed axis with angular speed , all the particles in
rigid body rotate same angle in same interval of time, i.e., their angular speed is same
(). They rotate in different circles of different radii. The planes of these circles are r
different. Linear speed of a particle situated at a distance r from the rotational axis is
v = r
or vr
d
C34: Angular velocity of a rigid body () is . Here is the angle between the
dt B
line joining any two points (say A and B) on the rigid body and any reference A
line (dotted) as shown in figure.
For example AB is a rod of length 4 m. End A is resting against a vertical wall Y
OY and B is moving towards right with constant speed vB = 10 m/s. To find
the angular speed of rod at = 30, we can proceed as under. A
OB = x = AB cos
x = 4 cos
vB = 10 m/s
dx d
or = 4 sin O X
dt dt B

d (dx / dt) dx
= [ = vB = 10 m/s]
dt
4 sin dt
10
or = = 5 rad/s
4 sin 30
d
Here negative sign implies that decreases as t increases 0 .
dt

C35: A circular disc of mass 2 kg and radius 1 metre is to be rotated about its centre in a horizontal plane with
angular acceleration of 4 rad/sec2. Calculate the torque required.
Sol. I
1
MR 2
2
1
a = 2 (1)2 4 = 4 kgm2/sec2.
2

C36: A constant torque of 40 Nm is applied to a wheel pivoted on a fixed axis. At what rate power is being
furnished to the wheel when it is rotating at 120 revolution per minute.
120
Sol. P = . = t . (2n) = 40 2 = 502.6 Watt.
60

www.physicsashok.in 37
PHYSICS ROTATIONAL MOTION ROTATIONAL MECHANICS
SHEET
C37: A fly wheel of moment of inertia 7.5 kgm2 is rotating at 240 revolution per minute; calculate its K.E.
240
Sol. = 2n = 2 =8
60
1 2 1
K.E. = I = [7.5] [8]2
2 2
1
= 7.5 64 p2 = 2366 joule.
2

Example 28. A uniform rod of mass m and length l hinged at its end is released from rest when it is in the horizontal
position. The normal reaction at the hinge when the rod becomes vertical is :
Mg 3Mg 5Mg
(A) (B) (C) (D) 2 Mg
2 2 2
Sol. Loss in P.E. = gain in K.E.
l 1
or mg I 2
2 2
m l2 2
or mg l F
3
3g
or 2
l mg

l
F mg m 2
2
l
or F mg m 2
2
l 3g 5 mg
F mg m Hence (C) is correct.
2 l 2

Problems with pure rotation and pure translational of two or more connected objects.
Solution techinque :
Step (i) Draw the free body diagram.
Step (ii) Assume linear acceleration of translatory object & angular acceleration of rotatory object.

Step (iii) Use F m a for translation motion and I for rotational motion.
Step (iv) Relate linear and angular acceleration and solve the obtained equation and get the answer.

Example 29. Consider a pulley fixed at its centre of mass by a clamp. A light rope is
wound over it and the free end is tied to a block of mass m. Find the linear acceleration
of block.
Sol. For rotational motion of pulley :
MR 2 .
TR = ...(1)
2

www.physicsashok.in 38
PHYSICS ROTATIONAL MOTION ROTATIONAL MECHANICS
SHEET

For translational motion of object :


Mg T = ma ...(2)
and a = Ra ...(3)
MR 2 a MaR
TR . ...(4)
2 R 2 T
Solving (2) and (4)
Ma a
mg ma
2
Mg
mg
a
m M / 2

Example 30. a uniform rod of length l, hinged at the lower end is free to rotate in the vertical plane. If the rod is held
vertically in the beginning and then released, the angular acceleration of the rod when it makes an angle of 45
with the horizontal (I = ml2/3)
3g 6g 2g 2g
(A) (B) (C) (D)
2 2l 2l l l
Sol. Loss in P.E. = gain in K.E.
l 1
or mg 1 cos I 2
2 2
Differenting both sides,
l d d
mg sin I
2 dt dt
l
or mg sin I
2
l
mg sin 45
2
m l2
3
3g
Hence (A) is correct.
2 2l

Example 31. A person pulls along a rope wound up around a pulley with a constant
force F for a time enterval of t seconds. If a and b are the radii of the inner and the
outer circumference (a < b), then find the ratio of work done by the person in the
two cases shown in the figure is W1/W2.
F F
Sol. W1 = F a Case I Case II
Fa = I 1
Fa
1
I
1
1 t 2
2

www.physicsashok.in 39
PHYSICS ROTATIONAL MOTION ROTATIONAL MECHANICS
SHEET

1 Fa 2
W1 Fa t
2 I
F2 a 2 2
W1 t
2I
F2a 2 2
Similarly, W2 t
2I
W1 a 2

W2 b2

Example 32. A uniform cylinder of mass m can rotate freely about its own axis which is horizontal. A particle of
mass m0 hengs from the end of a light string would round the cylinder which does not slip over it. When the
system is allowed to move, the acceleration of the descending mass will be
2m 0g m0g 2m 0 g m 0g
(A) m 2m (B) m m (C) m m (D) 2m m
0 0 0 0
Sol. m0g T = m0a
Tr = I
mr 2
or Tr
2
mr ma
or T
2 2
m0
ma
or m0 g m0 a
2
m
or m0 g m0 a
2
m0 g 2m0 g
a
m m 2m0 Hence (A) is correct.
m0
2
l/4
Example 33. For the pivoted slender rod of length l as shown in figure, the angular velocity l
as the bar reaches the vertical position after being released in the horizontal position is l/4

g 24g 24g 4g
(A) (B) (C) (D)
l 19l 7l l
Sol. Loss in P.E. = gain in K.E.
l 1
or mg I 2
4 2
2
l 1 m l 2 l 2
or mg m
4 2 12 4

2 2
l 1 ml ml 2
or mg
4 2 12 16

24 g
Hence (C) is correct.
7l

www.physicsashok.in 40
PHYSICS ROTATIONAL MOTION ROTATIONAL MECHANICS
SHEET

Problem based on situation in which same object is performing translation and rotational motion.
This type of motion of an object can be considered as superposition of two types of motion taking place
simultaneously.
(A) translatory motion of centre of mass
(B) rotational motion w.r.t. axis passing through centre of mass

Solution technique :
Step (i) Draw the free body diagram of object.
T T

mg mg
Step (ii) Assume linear acceleration of centre of mass and angular acceleration w.r.t. an axis passing through
the centre of mass.
Step (iii) Use F = ma for translating motion of centre of mass and = I for rotational motion w.r.t an axis
passing through centre of mass.
Step (iv) Relate a and and solve the equations.

Example 34. A cylinder of mass M is suspended through a two strings wrapped around it
an shown in figure. find the linear acceleration of centre of mass of the cylinder.

T T
Mg

a a
Sol.

mg mg mg
For translatory motion of centre of mass
Mg T = Ma ...(1)
For Rotational motion of cylinder w.r.t. an axis passing through the centre of mass
MR 2
TR ...(2)
2
Relation between a &
a = R ...(3)
solving (1), (2) and (3) we get
MR a
Mg . Ma
2 R
Mg = 3 Ma/2
a = 2g/3

Example 35. Two discs A and B touch each other as in figure. A rope lightly wound
on A is pulled down at 2 m/s2. Find the friction force between A and B if slipping R 2R
is absent.
1 kg 2 kg
2 m/s2

www.physicsashok.in 41
PHYSICS ROTATIONAL MOTION ROTATIONAL MECHANICS
SHEET

mR 2
Sol. TR fR
2
mR a
T f f
2 R
ma 1 2
T f 1
2 2
Tf=1 ...(1) f
2
m 2R 2 4 R2 a
Also, f 2R
2 2 2R
F=a=2N

LECTURE 7

ROLLING MOTION
(a) Rolling motion of an object can be consider as superposition of two motions taking place simultaneously.
(i) Translatory motion of the centre of mass
(ii) Rotational motion of body w.r.t. an axis passing through the centre of mass.
(b) In pure translational motion, all points move with the same linear velocity Vcm. In rotational motion about
centre of mass all points move with the same angular velocity about the central axis.
v+ R v
R
v v v v

v2 + 2 2
R v R
v R R

Perfect Rolling :
(a) If an object is in perfect rolling on a surface than point of contact of body with surface
must be relatively at rest w.r.t. surface. v
for perfect rolling
A
vAB = 0 B
net velocity of A = net velocity of B. surface
(b) Perfect rolling on ground :
(i) The relative instantaneous speed of the point of contact during rolling is zero.
(ii) For perfect rolling motion, work done against friction is zero.
(iii) In case of perfect rolling the rotating object remains relatively at rest w.r.t. surface on which it is rolling
therefore the force of friction will be static in nature.
(iv) Due to static nature of friction in perfect rolling its value lives in between O and N. Where is coefficient of
friction between rolling object and surface.
(v) For perfectly rolling v R = 0 where v is velocity of centre of mass and is angular velocity.

v v

v R
VG = 0

www.physicsashok.in 42
PHYSICS ROTATIONAL MOTION ROTATIONAL MECHANICS
SHEET

C38: If sphere is in perfect rolling on platform or on plank v = R = v1 sphere

Solution technique : v
(i) Draw the free body diagram of different components of system.
(ii) Assume linear and angular acceleration of different components of system. v1

(iii) User F m a and I for translational and rotational motion respectively..
(iv) Relate linear and angular acceleration.
(v) Finally solve the equations.
Note : Friction is not necessary for perfect rolling.

Example 36. The spool shown in figure is placed on rough horizontal surface and has inner F
R
radius r and outer radius R. The angle between the applied force and the horizontal
can be varied. The critical angle () for which the spool does not roll and remains r
stationary is given by

1 r 1 2r 1 r 1 r
(A) cos (B) cos (C) cos (D) sin
R R R R
Sol. For equilibrium
F cos = f ...(1)
and (F)r = fR
f r
or ...(2)
F R
From equation (1) and (2)
f r
cos
F R
r
cos 1 Hence (A) is correct.
R

Example 37. A round object of mass M and radius R rolls without slipping down an inclined plane set on angle
to horizontal. Find
(a) Linear acceleration of centre of mass.
(b) Minimum coefficient of friction required for perfect rolling. assuming M.I. w.r.t. an axis passing through centre
of mass is MK2.
Sol. For translatory motion
mg sin fr = ma ...(1) R
a
For rotatory motion fr
m
g
fr R = MK2 () ...(2) sin
and a R = 0 ...(3) mg cos
solve equation (1), (2) and (3) mg

g sin
a
1 K2 / R 2
mg sin
fr
R2
and
1 K 2

www.physicsashok.in 43
PHYSICS ROTATIONAL MOTION ROTATIONAL MECHANICS
SHEET
fr mg cos (for perfect rolling)
R2 5
Where K 2 2

2
tan
7

Imperfect Rolling
(i) When point of contact of Rolling object is not relatively at rest. w.r.t. surface on which it in rolling then it is
called imperfect Rolling.
(ii) Nature of friction in this case is kinetic & it should opposing the motion of point of contact (strictly).
(iii) Direction of linear acceleration and angular acceleration should be taken according to direction of friction
(strictly).
(iv) Nature of friction is kinetic therefore its value is KN. Where K is kinetic friction coefficient between surface
and object.

Transformation of Imperfect Rolling into perfect Rolling


N
v0 > 0R

v0 v= R
0
N
imperfect Rolling perfect Rolling

N
v0 < 0R

v= R
v0 v
0
N
imperfect Rolling perfect Rolling

v If becomes zero before v.

N
v0 < 0R
v If v becomes zero before .
0 Z
v0
N
v=0& =0
imperfect Rolling if v and both becomes
zero simultaneously.
Perfect rolling

Solution technique :
Step (i) Draw the free body diagram.
Step (ii) Indicate the direction of friction in correct sense.
www.physicsashok.in 44
PHYSICS ROTATIONAL MOTION ROTATIONAL MECHANICS
SHEET
Step (iii) Assume linear and angular acceleration.
Step (iv) Use F = ma and T = I. Calculate a and .
Step (v) then use the concepts of kinematics to solve the problems.

Kinetic energy of a rolling body:



Let the centre of mass be moving with velocity VC . Relative to the centre of mass, a particle at position ri will

have a velocity ri . Relative to ground the particle be moving with velocity

Vi = VC + ri mi

The speed square is given by ri


VC


2
V =
i Vi . Vi

= ( VC + ri ).( VC + ri )


= VC2 + | ri |2 + 2 VC .( ri )
The kinetic energy of the whole body is given by
1
K mi vi2
2
1 1
=mi VC2 + m |
ri |2 + 2mi VC .( ri ) .....(i)
2 2 i

Now, mi = total mass = M


1 1
mi Vi2 = MVC2 .....(ii)
2 2

Let ri be resolved along and normal to . Let ri be normal to and ri|| be along such that

r = ri + ri||
i

Then ri|| = 0.

| ri | = ri + ri

= | ri |
= r i
1 1 2 1
mi | ri |2 = mi 2 ri = Icm2 .....(iii)
2 2 2
The third term can be written as

2mi VC .( ri ) = 2 VC .( mi ri ) = 0 .....(iv)
In equation (iv) the summation is the first moment of mass about centre of mass, which is zero by definition of
CM. Hence the whole term becomes zero.
Using these,
1 1
K= MVC2 + Icm2
2 2

www.physicsashok.in 45
PHYSICS ROTATIONAL MOTION ROTATIONAL MECHANICS
SHEET

1 1
The quantity MVC2 is called translational kinetic energy and Icm2 is called rotational kinetic energy of the
2 2
rolling body.

Example 38. A uniform disc of mass M and radius R rolls on a smooth horizontal surface without slipping with a
linear velocity v. Calculate its K.E.
Sol. K.E. of a rolling body (without slipping)
1 1 1 1 v2
Mv2 I 2 Mv 2 MK 2 2
2 2 2 2 R

1 K2
Mv2 1 2
2 R
R
for disc, since K=
2
3
Hence K.E. of a disc = Mv2.
4

C39: A solid sphere of mass M and radius R is rotating as well as moving with a linear velocity v; then calculate its
K.E.
K2
1 1 2
Sol. K.E. = Mv2 R 2 ; for sphere, K = R
2 5
K.E. for a sphere which is rolling (without slipping)
7
Mv2.
10

Example 39. A solid cylinder of mass M and radius R has a length L. If it is rolling as well as moving with linear
velocity v, then calculate its K.e. (assuming no slipping).
K2
1 1
Sol. We have, K.E. = Mv2 R 2
2
R
for a solid cylinder, K =
2
3
K.E. = Mv2 (same as that for a circular disc of mass M and moving with same v).
4

Example 40: A disc of mass m rolls without sliding with speed v0. (a) write its kinetic energy. (b) Write the ratio of
translational to rotational kinetic energies.
Sol: (a) The kinetic energyof a rolling body is given by
1 2 1
K = MVCM + Icm2
2 2
VCM
Here, Vcm = V0, = (no slipping)
r
www.physicsashok.in 46
PHYSICS ROTATIONAL MOTION ROTATIONAL MECHANICS
SHEET
V0
=
r
1 2
Icm = mr (for disc)
2
V0

1 1 1 2 2
K = mv 20 + mr
2 2 2
1 1
= mv 20 + mv 20
2 4
3
= mv 20
4
1
(b) Translation KE = mv 20
2
1 1 2 2 1 2
Rotational KE = mr = mv 0
2 2 4
Translation KE
Required ratio = Rotational KE

1
mv02
= 2 =2
1 2
mv0
4
The ratio is, thus, 2 : 1.

Example 41. A solid disc is rolling without slipping as well as moving with linear velocity v. What will be the ratio of
translatory K.E. and total K.E.
Translatory K.E. 1/ 2 Mv 2
Sol.
Total K.E. [Trans. K.E. Rot. K.E.] 1/ 2 Mv 2 [1 K 2 / R 2
R2 R2 2
2 2
2 2

R K R R /2 3

C40: A circular disc of mass 2 kg and radius 0.2 metre rolls down an inclined plane from rest. What time it will take
to cover a distance of 1 metre along the plane, if angle of inclination of the plane is 30.
Sol. Acceleration,
1
g sin g g
a 2
2 2 2 2
1 K / R 1 R 2R 3
1 2
s at
2
1g 2
1 t
23
6
t 0.78 s
9.8

www.physicsashok.in 47
PHYSICS ROTATIONAL MOTION ROTATIONAL MECHANICS
SHEET
Mechanical Energy
The sum of kinetic energy (K) and potential energy (U) is mechanical energy. If a rigid body has its CM at
height hcm above gravitational zero level of potential energy,
U = mg hcm,
The mechanical energy (E) of rolling body will be
1 2 1
E= MVCM + Icm2 + Mghcm
2 2
In other fields like spring force, other terms may appera in the expression for U.
If there be no sliding, the work done by friction force is zero. The mechanical energy remains conserved in that
situation.

Example 42. A thin rod of length L is placed at angle to vertical on a frictionless horizontal floor and released. If
the center of mass has acceleration = A, and the rod an angular acceleration = at initial moment, then
(A) A = (L). sin (B) A/2 = (L).sin (C) 2A = (L).sin (D) A = L
L L/2 sin
Sol. a cm A sin
2 N

L
A sin L/2 90
2
2A = L sin
mg
Hence (C) is correct. acm

C
Example 43. A disc of radius R is rolling purely on a flat horizontal surface, with a constant P
angular velocity. The angle between the velocity and acceleration vectors of point P is
(A) zero (B) 45 (C) 135 (D) tan1(1/2)
Sol. ar = R2 (radial acceleration is directed towards centre at point P.
R 2 + (R )2 = 2 v
vP= vcm
v vcm 1
Here cos cm
vP 2 v cm 2
= 45 ar vcm vcm
Hence (B) is correct.
R vcm

Example 44. A wheel, of radius 1 m, is rolling purelly on a flat, horizontal surface. Its centre is moving with a
constant horizontal acceleration = 3 m/s2. At a moment when the centre of the wheel has a velocity 3 m/s, then
find the acceleration of a point 1/3 vertically above the centre of the wheel.
Sol. acm = r
v = r
v

r
at +acm
2
a a t a cm a 2
r
ar

1
Here at
3

www.physicsashok.in 48
PHYSICS ROTATIONAL MOTION ROTATIONAL MECHANICS
SHEET
1
a r 2
3
acm = 3 m/s2
v = 3 m/s
Putting the value, a = 5 m/s2.

Example 45. A solid sphere is spin of about its on axis with angular velocity in clockwise direction and is gently
placed on a rough horizontal surface. Coefficient of friction between sphere and the surface is . Calculate.
(a) Time after which sphere starts perfect rolling.
(b) Final linear and angular velocity.
(c) Energy loss against friction. N N
Sol. For translatory motion
N = ma and N = ma
mg = ma a = g ...(1) a m
0
R
For rotational motion
N
2
NR mR 2 mg
5
5g
...(2)
2R
Let after time t sphere starts perfect rolling.
For translatory motion
a = g
v = 0 + gt ...(3) v
For rotational motion u=0

5g t=0 t=t
0 .t ...(4)
2R
at t = t for perfect Rolling v R = 0
v = R ...(5)
From (3), (4) and (5)
g
gt 0 R 5 t
2
(a) Therefore time after which sphere starts perfect rolling 0

0 R t=0 t=t
t
5g
g
2

2 R
t 0
7g
(b) Linear velocity at the time of perfect rolling
2 0 R 2
v .g 0 R
7g 7
(c) Work done by friction or energy loss
1 2 1 1
I 0 mv 2 I 2
2 2 2

www.physicsashok.in 49
PHYSICS ROTATIONAL MOTION ROTATIONAL MECHANICS
SHEET

Example 46. A solid cylinder of mass m is moving with velocity v0 in horizontal direction.
It is gently placed on a rough horizontal surface. If coefficient of friction between v0
R
sphere and surface is . Calculate time interval after which it starts perfect rolling.
Sol. For translational motion :
mg = ma
a = g ...(1) N
For rotational motion
NR = I
a v0
mR 2
mg R
2
2g N
...(2) mg
R
Let v is the linear speed and is angular speed at the time of perfect rolling.
v = v0 at = v0 gt
2g
and 0 t 0 .t ...(3)
R
v R = 0 (for perfect rolling)
v
...(4)
R
v 2g
From (3) and (4) .t
R R
v 0 gt 2g
.t
R R
3g v v
.t 0 , t 0
R R 3g

Example 47. A spherical shell of mass m and radius r is moving with linear velocity v0 in forward direction and
rotating with 0 in anti clockwise direction. Now it is gently placed on a rough horizontal surface. If coefficient
of friction between shell and surface is . Find the relation between 0 and v0 for to be zero before v0.
Sol. For translatory motion mg = ma
a = g ...(1)
2 2
For rotational motion mgr mr .
3
= 3 g / 2r ...(2)
at time t : v = v0 gt ...(3) N

3g
and 0 t ...(4) a v0
2r
when = 0
3g N
0 t0 mg
2r
2 0 r 2 r
t and v v0 g 0 0
3g 3g
2
v0 0 r
3

www.physicsashok.in 50
PHYSICS ROTATIONAL MOTION ROTATIONAL MECHANICS
SHEET
Example 48. A cylinder of mass M and radius R is resting on a horizontal platform (which is parallel to the xy
plane) with its axis fixed along the y axis and free to rotate about its axis. The plateform is given a motion in the
x-direction given by x = A cos (t). There is no slipping between the cylinder and plateform. Find the maximum
torque acting on the cylinder during its motion.
dx
Sol. vx A sin t
dt
ax = A2 cos t
amax = 2A
Since, no sliping takes place.
acm = R
2A

R
MR 2
I
2
MR 2 2 A 1
mRA 2
2 R 2

Example 49. A bit of mud stuck to a bicycles front wheel of radius r detaches and is flung horizontally forward
when it is at the top of the wheel. The bicycle is moving forward at a speed v and it is rolling without slipping.
Find the horizontal distance travelled by the mud after detaching from the wheel.
Sol. x = 2vt
1 2
2r gt
2 2v

4r
t
g
x
4r
x 2v
g

r 16rv 2
x 4v
g g

0
Example 50. Find the relationship between v0 and 0 so that sphere of radius R and mass
m rolls up the inclined plane before it starts falling down the plane finally in given fig.
Sol. For translational motion v0
v
N mg sin = ma
mg cos mg sin = ma
a = ( cos sin )g ...(1)
2
for rotational motion mg cos R = mR2
5
5g cos
...(2)
2R

www.physicsashok.in 51
PHYSICS ROTATIONAL MOTION ROTATIONAL MECHANICS
SHEET
assume t is the time when v = 0
0 = v0 ( cos sin ) gt
v0 a N
t
cos sin g
N 0 mg
at this time > 0 co
s
5g cos mg cos
0 .t 0
2R
5g cos v0
0
2R cos sin g
5v 0 cos
0
2R cos sin

Example 51. Two spheres are rolling with same velocity (for their C.M.) their ratio of kinetic energy is 2 : 1 &
radius ratio is 2 : 1, their mass ratio will be :
(A) 2 : 1 (B) 4 : 1 (C) 8 : 1 (D) 2 2 : 1
1 1
Sol. T mv 2 I 2
2 2
1 12
T mv 2 mR 2 2
2 25
1 1 2
T mv 2 mv 2
2 2 5
1 7
T mv 2
2 5
T1 m1
Hence (A) is correct.
T2 m 2

Example 52. Two thin circular discs of mass 2 kg and radius 10 cm each are joined by a rigid massless rod of
length 20 cm. The axis of the rod is along the perpendicular to the planes of the disc through their centres. This
object is kept on a truck in such a way that the axis of the object is horizontal and perpendicular to the direction
of motion of the truck. Its friction with the floor of the truck is large enough so that
the object can roll on the truck without slipping. Take X-axis as the direction of
motion of the truck and Z-axis as the vertically upwards direction. If the truck has
an acceleration 9 m/s2, Calculate :
(i) the force of friction on each disc.
(ii) The magnitude and direction of the firctional torque acting on each disc about the centre of mass O of the
object. Express the torque in the vector form in terms of unit vectors i , j and k in X, Y and Z directions.
Sol. Given mass of disc, m = 2 kg and radius, R = 0.1 m
(i) FBD of any one disc is shown in figure :

Truck a = 9m/s2

Y X

www.physicsashok.in 52
PHYSICS ROTATIONAL MOTION ROTATIONAL MECHANICS
SHEET
Frictional force on the disc should be in forward direction.
Let a0 be the linear acceleration of COM of disc and the angular acceleration about its COM. Then :
f f a0
a0 ...(i)
m 2 f
P
f .R 2f 2f Q
10 f ...(2) f
2
I 1/ 2mR mR 2 0.1
Since there is no slipping between disc and truck, therefore,
a = 9m/s2
a0 + R = a
f
or (0.1) 10 f a
2
3 2a 2 9.0
or f a f N
2 3 3
f=6N
Since this force is acting in positive x-direction,


f 6 i N
20cm = 0.2m
(ii) rf
z
Here
f 6 i N (for both the discs)
O
2

y
r P r1 0.1 j 0.1 k x
Q
and r Q r 2 0.1 j 0.1 k P
f f
Therefore, frictional forque on disc 1 about point O (centre of mass) :


1 r1 f 0.1j 0.1k 6 i N m 0.6 k 0.6 j

or
1 0.6 k j N m

and | 1 | (0.6) 2 (0.6) 2 0.85 N m

Similarly,
2 r 2 f 0.1j 0.1k 6 i N m


2 0.6 j k N m

and | 2 | | 1 | 0.85 N m

INSTANTANEOUS AXIS OF ROTATION


The combined effects of translation of the centre of mass and
rotation an axis through the centre of mass are equivalent to a v
pure rotation witht he same angular speed about an axis passing
through a point of zero velocity. Such an axis is called the
instantaneous axis of rotation.
(IAOR). This axis is always perpendicular to the plane used to represent the motion and the intersection of the

www.physicsashok.in 53
PHYSICS ROTATIONAL MOTION ROTATIONAL MECHANICS
SHEET
axis with this plane defines the location of instantansus centre of zero velocity (IC).
For example consider a wheel which rolls without slipping. In V
Vp
this case the point of contact with the ground has zero velocity. r r
Hence, this point represents the IC for the wheel. If it is v Vp = r

imagined that the wheel is momentarily pinned at his point, the
velocity of any point on the wheel can be found using v = r.
IC
Here r is the distance of the point from IC. Similarly, the kinetic
energy of the body can be assumed to be pure rotational about
IAOR or,
Rotation + Translation Pure rotation about IAOR passing through IC]
1 1 1 2
K .E. = mv 2cm + Icm2 K.E. = I IAOR
2 2 2

C41: (i) Although the IC may be conveniently used to determine the velocity of any point in a body, it generally
does have zero acceleration and therefore, it should not be used for finding the acceleration of any point in the
body.
(ii) When a body is subjected to general plane motion, the point determined as the instantaneous centre of
zero velocity for the body can only be used for an instant of time. Since the body changes its position from one
instant to the next, then for each position of the body a unique instantaneous centre must be determined. The
locus of points which defines the IC during the bodys motion is called a centrode. Thus, each point on the
centrode acts as the IC for the body only for an instant of time.

Locaion of the IC
If the location of the IC is unknown, it may be determined by using the fact that the relative position vector
extending from the IC to a point is always perpendicular to the velocity of the point. Following three pssibilities
exist.

Give the velocity of a point (normally the centre of mass) on the body and the angular velocity of the
body.
If v and are known, the IC is located along the line drawn perpendicular to
v
v at P, such that the distance from P to IC is, r =
. Note that IC lie on that


side of P which causes rotation about the IC, which is consistent with the P v

direction of motion caused by and v . r

IC

Example 53: A rotating disc moves in the positive direction of the x-axis. Find the equation y(x) describing the
position of the instantaneous axis of rotation if at the initial moment of the centre c of the disc was located at the
point O after which it moved with constant velocity v while the disc started rotating counterclockwise with a
constant angular acceleration . The initial angular velocity is equal to zero.
y y
x IC
Sol: t= y
v
O c x O c x
v v
x
and = t =
v x
The position of IAOR will be at distance

www.physicsashok.in 54
PHYSICS ROTATIONAL MOTION ROTATIONAL MECHANICS
SHEET

v
y =

v
or y = x
v
v2
or y =
x
v2
or xy = = constant.

This is the desired x-y equation. This equation represents a rectangular hyperbola.

C42: The equation text = l does not hold good in a non-inertial frame. However, there exists a very special case
when text = l does hold even if the angular acceleration a is measured from a non-inertial frame. That special
case is, when the axis of rotation passes through the centre of mass and otherwise the pseudo forces produce
a pseudo torque about the axis. This is the reason, in above examples we calculated the angular acceleration a
about an axis passing through the centre of mass and perpendicular to the plane of motion of the particles.
ext
Because about that axis we can apply, = .
l
C43: Work done by friction in pure rolling on a stationary ground in zero as the point of application of the force
is at rest. Therefore, mechancal energy can be conserved if all other dissipative forces are ignored.
C44: In accelerated pure rolling the velocity of the bottommost point is zero but despite the relatin a = R, the
accelertion of the bottommost point is not zero. Because acceleration of any ponit P can be given as

aP = aC + a PC
Here,
a PC has two components:
Tangential acceleration at = r (which is perpendicular to CP) and radial or normal acceleration an = r2
(which is along PC)

Thus aP = a C + (
a PC )t + ( a PC )n


For the botommost ponit, a C + (
a PC )t = 0 C
r P v, a

R
as aC = a [in forward direction]
and (
a PC )t = R [in backward direction]

and since, a = R therefore a C + (
a PC )t = 0.
But (
a PC )n 0. It is R2 towards centre. Thus, acceleration of bottommost P
v, a
ponit is R2 towards centre.
Similarly in the problems like shown in figure, it is wrong to say that acceleration
of point P is equal to acceleration of block A. Although we can write,
A

aA = a + r
In case of pure rolling, problems can also be solved by using the energy conservation principle (provided no
other dissipative forces are present). So, in this case we will write the energy equation as,
Decrease in gravitational potential energy of A = increase in kinetic energy of A(only translational) + increase in

www.physicsashok.in 55
PHYSICS ROTATIONAL MOTION ROTATIONAL MECHANICS
SHEET
kinetic energy of spoot (both rotational as well as translational).
C45: In cases where pulley is having some mass and friction is sufficient enough to prevent slipping, the tension on
two sides of the pulley will be different and rotatinoal motion of the pulley is also to be considered.
C46: At a given instant the value of for a rigid body will be same for every point.
C47: The torque equation ( = l) can be applied only about two points. These are,
(i) centre of mass
(ii) point about which body is in pure rotation.

Example 54. There is rod of length l . The velocities of its two ends are v1 and v2 in opposite directions normal to
the rod. The distance of the instantaneous axis of rotation from v1 is
v2 v1 l
(A) zero (B) v v l (C) v v (D) l / 2
1 2 1 2
v1
v1 v
Sol. 2 x
x lx l
or v1(l x) = v2x O
or v1 l v1 x = v2 x lx
v1 l
x Hence, (C) is correct. v2
v1 v 2

Example 55. A ladder of length L is slipping with its ends against a vertical wall and a horizontal floor. At a certain
moment, the speed of the end in contact with the horizontal floor is v and the ladder makes and angle = 30
with the horizontal. Then the speed of the ladders center must be
(A) 2v/ 3 (B) v/2 (C) v (D) None
v
Sol.
l /2
2v intantaneous axis
O
of rotation
l 30
v2 60 r
2 l /4
3 l 2 l /2 l /2
4 l 4
3 l/4
Also, vc = r = C 30
l /2 60
30
1 l v
= l 31
4 2
l 2v
v Hence (C) is correct.
2 l

Example 56. A rectangular rigid fixed block has a long horizontal edge. A solid
homogeneous cylinder of radius R is placed horizontally at rest with its length
parallel to the edge such that the axis of the cylinder and the edge of the block are
in the same vertical plane as shown in figure. there is sufficient friction present at R
the edge so that a very small displacement causes the cylinder to roll of the edge
without slipping. Determine :
(a) the angle c through which the cylinder rotates before it leaves contact with the edge.

www.physicsashok.in 56
PHYSICS ROTATIONAL MOTION ROTATIONAL MECHANICS
SHEET
(b) the speed of the centre of mass of the cylinder before leaving contact with the edge, and
(c) the ratio of the translational to rotational kinetic energies of the cylinder when its centre of mass is in horizontal
line with the edge.
Sol. (a) The cylinder rotates about the point of contact. Hence the mechanical energy of the cylinder will be conserved
i.e.,

v
R R Rcos

v
(1) (2) (3)

(P.E. + K.E.)1 = (P.E. + K.E.)2


1 2 1 2
mgR + 0 = mgR cos + I mv
2 2
But = v/R (No slipping at point of contact)
1
and I= mR 2
2
11 2 1
Therefore, mgR = mgR cos +
2
2
mR v / R mv
2 2 2
2

3 2
or v = gR (1 cos )
4
v2 4
or g (1 cos ) ...(1)
R 3
At the time of leaving contact, normal reaction N = 0 and = c. Hence s
v
co
mg

sin
mv
mg cos = mg
R N=0
mg
v2
or = g cos ...(2)
R
From (1) and (2)
4
g(1 cos c) = g cos c
3
7
or cos c = 1 or cos c = 4/7
4
or c = cos1(4/7)
4
(b) v gR 1 cos (From equation 1)
3
At the time it leaves the contact
cos = cos c = 4/7
4 4
v gR 1 4 / 7 or v gR
3 7

www.physicsashok.in 57
PHYSICS ROTATIONAL MOTION ROTATIONAL MECHANICS
SHEET

4
Therefore, speed of COM of cylinder just before it leaves the contact is gR .
7
(c) At the moment, when cylinder leaves the contact
4
v gR
7
1 2
Therefore, rotational kinetic energy, KR = I
2
11 1 1 4
or
22
4

K R mR 2 v 2 / R 2 mv 2 m gR
4 7
mgR
or KR = ...(3)
7
Now once the cylinder loses its contact, N = 0, i.e. the frictional force, which is responsible for its rotation, also
vanishes. Hence its rotational kinetic energy now becomes constant, while its translational kinetic energy increases.
Applying conservation of energy at (1) and (3) :
Decrease in gravitational P.E. = Gain in rotational K.E. + translational K.E.
Translational K.E. (KT) = Decrease in gravitational P.E. KR
mgR 6
or KT = (mgR) = mgR ...(4)
7 7
From (3) and (4) we have
K T 6 / 7mgR KT
or 6
KR mgR / 7 KR

ANGULAR MOMENTUM
v
Let a particle of mass m have a velocity v in a frame S. Let O be a point at m
rest in S. The particle has a position vetor r relative to O. r


Then the cross prodcut r mv is called angular momentum about O. O
The angular momentum of a particle about O is equal to the cross product of its position vector relative to O
and the momentum vector.

lO = r p .
Its S.I. unit is kgm2s1 and dimensional formula [ML2T1].

Moment arm:

In the figure r is the moment arm for l . We have
r = moment arm

p

lO = ( r + r|| ) p rs
r
r

= r p [ r|| p = 0] O
www.physicsashok.in 58
PHYSICS ROTATIONAL MOTION ROTATIONAL MECHANICS
SHEET
For a particle to have angular momentum, it must posses a linear momentum. However, a particle having a
linear momentum may not have an angular momentum.
The concept of angular momentum is introduced for motion of particles in space here. However, you will find
in your studies of modern physics or atomic physics another angular momentum which is not related to motion
in space- that is spin angular momentum of particles like electron. It is instrinsic to a particle, say, electron.
There are several particles which have spin angular momenta. We shall not discuss then here. We shall consider
motion-related angular momenta only.

C48: A particle is moving with uniform velocity. Does its angular momentum about any point change as its moves ?
Sol: The angular momentum about O (see previous figure) may be written as

l = r p = rp inward, which does not change.

Example 57. A particle of mass 0.5 kg is rotating in a circular path of radius 2m and centrepetal force on it is 9
Newtons. Its angular momentum (in J.sec) is :
(A) 1.5 (B) 3 (C) 6 (D) 18
Sol. L = mvr
mv2
But F
r

Fr
or v
m
Fr
Lm r
m
92
L 0.5 2
0.5
L 36 6 J s Hence (C) is correct.

Example 58. A particle of mass m is rotating in a plane is a circular path of radius r, its angular momentum is L. The
centripital force acting on the particle is :
L2 L2 m L2 L2
(A) (B) (C) (D)
mr r mr 2 mr 3
Sol. L = mvr
L
v
mr
2
L
2 m
mv mr
F
r r
2
L
F 3 Hence (D) is correct.
mr


Example 59. A particle of mass 2 kg located at the position i j m has a velocity 2 i j k m/s. Its angular
momentum about zaxis in kg-m2/s is :
(A) zero (B) 8 (C) 12 (D) 8
www.physicsashok.in 59
PHYSICS ROTATIONAL MOTION ROTATIONAL MECHANICS
SHEET
Sol. The angular momentum about origin

L r mv


L i j 4 i j k


L 4 k j k i

L 4 i 4 j 8 k
Lz = 8 kg m2/s Hence (D) is correct.


Example 60. The position vector of a particle of mass 2.00 kg is given as a function of the by r 6 i 5t j m.
Determine the angular momentum of the particle about the origin, a function of time.

dr
Sol. v 5 j
dt

L r mv


L 6 i 5t j 10 j

L 60 k

Example 61. A particle of mass m is projected with a velocity u at an angle of with horizontal. Find the intial
angular momentum of the particle about the highest point of its trajectory.

Sol. u u cos i u sin j
y
| L | | r mu |
x
2 2 u O
u sin
Here h h
2g
R/2
u 2 sin 2
R
g
R
r h j i
2
mu 3 sin 2 cos
L
2g

Angular momentum of many particles



If a system has several particles, having angular momenta l1 , l2 , ........ ln about a point O at rest in frame S,
the total angular momentum of the system is defined as their vector sum:

L = l1 + l2 + ........ + ln

Angular momentum about centre of mass


For calculating angular momentum about centre of mass of a system, we must
take a frame S in which CM is at rest. From this frame we observe momenta

www.physicsashok.in 60
pi Z
PHYSICS ROTATIONAL MOTION ROTATIONAL MECHANICS
SHEET
Vi

p , p'2 , ........ p 'N of the particles located at r , r2' ,.... r
'
1 1
' '
N relative to centre
Y
of mass. Then the particle has angular momentum ri' p'i about centre of
mass. Total angular momentum is the vector sum of such angular momenta.
Frame fixed to CM
We denote it by Lcm : X but not to the body

L cm = ri' pi' .

Relationship to angular momentum about a Laboratory point:


Let O be a point in laboratory and O be the centre of mass of the system. Let i-th particle, having mass mi be

moving with velocity v i' in the frame of laboratory. Then angular momentum of this particle about O will be ri

mi v i' . The whole body will have an angular momentum

L O = ri mi v i' .
Relative to the frame in which O is at rest (CM- frame) the i-th particle has a different velocity
v i , it is relative

velocity
v i VCM, where VCM is the velocity of centre of mass. Simmilarlly its position vector is

r = ri rcm
i
'

The figure shows vi


VCM

Vi = Vi' + VCM vi
ri VCM

r = ri' + rcm O
i

Using these we have L ri


rcm
L = r mi v i i

= ( rcm + ri' ) mi
vi

= rcm mi
v i + ri' mi v i

= rcm mi
v i + ri' mi v i

= rcm M VCM + ri' mi ( v i' + VCM) [ M VCM = mi
vi]

= rcm M VCM + ri' mi v i' + mi VCM

= rcm M VCM + L cm + mi ri' VCM

= ( rcm M VCM) + L cm [ mi ri' = 0]

The first term gives angular momentum associated with motion of centre of mass, while the second with motion
about CM.

Example 62: A rod of length 3m and negligible mass connects two balls of
masses 1 kg and 3 kg. The system is thrown in air negligible (friction)
as in the figure. At a certain moment, the centre of mass is located at

www.physicsashok.in 61
1
PHYSICS ROTATIONAL MOTION ROTATIONAL = 2 radsMECHANICS
10 m/s SHEET
1kg
(2m, 4m) and moving with 10ms1 at 60 with horizontal. The rod is Y CM 2kg
1
turning at = 2 rad s about the centre of mass. ^j 4m
^k
(a) Determine the angular momentum of system about centre of mass X
O
^ ^ ^ ^
j
and express it using i , j and k .
^ ^i
(b) Determine the angular momentum of the system about origin. 1
6 ms
k

Sol: (a) See figure for quantities needed. 1m


1kg 2kg

2m
We have L cm = r m v .
i
'
i i 1
2 ms
^ ^ ^ ^
= {2m ( i ) 1 kg (6 j ms1)} + {(1m) i (2 kg) 2m1( j )}
^
= 16 k (kgm2s1)


(b) L O = ( rcm MVcm ) + L cm
^ ^ ^
Here, rcm = (4 i + 4 j )m, = 16 k (kg m2s1)
^ ^
VCM = 10 ms1(cos 60 i + sin60 j )
M = 3 kg
Using there
^
LO = 27.92 k (kg m2s1)

Angular momentum about an axis


The component of angular momentum about a point on the given axis is known as angular momentum along

that axis. If angular momentum about a point O on Z-axis is L . It is making an angle with Z-axis, then the
angular momentum about Z-axis (LZ) is given by
LZ = L cos
It is a scalar quantity.

Angular momentum of a body rotating about an axis at rest:


Let Z-axis be oriented along the stationary axis of rotating of the body. An i-th

particle will have angular momentum li about a point O in the Z-axis. Then Z

total angular momentum L is given by z
Its Z-component is called angular momentum about Z-axis, LZ. Now
LZ = liZ , Vi

ri
where liz is the Z-component of angular momentum li . To get liZ, we figure,
mi
li Ri
Here li = Rimivi
liZ = Rimivi cos O
= RimiZri cos
Now, Ricos = ri
liZ = m i ri2 z
LZ = (mi ri2 ) z
LZ = IZZ
where IZ = m i ri2 is called moment of inertia about Z-axis.

Torque and angular momentum


Now we shall consider fundamental law of rotational dynamics-also called Newtons second law of rotational

www.physicsashok.in 62
PHYSICS ROTATIONAL MOTION ROTATIONAL MECHANICS
SHEET
dynamics. It relates the rate of charge of angular momentum to torque.

Single Particle System



The figure shows a particle having momentum p .

There is a point O fixed in XYZ-frame. Let angular momentum about O be denoted by lo .

Then, L0 = r p .
Differentiating we get

dl dr dp
= p + r
dt dt dt

dp
= v p + r F [ = F]
dt

= 0 +

=
Thus

dl
=
dt
The rate of change of angular momentum about a point is equal to unbalanced torque about the same point.

The case of many particle system



Let there be many particles having momenta pi .

dpi
Then = Fi [Newtons second law of motion] .....(i)
dt

where Fi is unbalanced force. Now it arises due to two types of sources-particles within the system boundary
and the particles outside the boundary of the system. Let be the force exerted by internal particles. We call it

internal force. Let Fiext be the force exerted by external particle; we call it external force. Then

Fi = Fi int + Fi ext .....(ii)
Now we write angular momentum L in the form:

L = li = ri p i .....(iii)
Differentially relative to time t,

dr dpi


dL i
= pi ri
dt dt dt



= v i p i + ri ( Fi int + Fi ext) [Using (i) and (ii)]

= 0 + ri Fi int + ri Fi ext

= in t + ext

www.physicsashok.in 63
PHYSICS ROTATIONAL MOTION ROTATIONAL MECHANICS SHEET
Itis experimentally found that internal torque cant change angular momentum of the system.

Fint = 0 [postulate of rotational dynamics]

dL
Thus, = ext .....(iv)
dt
The rate of change of angular momentum of a system of particles about a point in an intertial reference frame is
equal to net external torque about the same point.
Taking Z-component of the above relationship (equation -iv) we get
dL z
ext z =
dt
Now, LZ is angular momentum about Z-axis
LZ = IZZ
where IZ is moment of inertia about Z-axis.
d
Hence, Z = (I )
dt Z Z
dI z d z
= wz + Iz
dt dt
dI z
z = +I [non-rigid system]
dt z z z
If moment of inertia. IZ constant then,
ext Z = IZZ [rigid bodies]
The external torque acting on a rigid body about an axis is equal to the product of moment of inertia and angular
acceleration about the same axis.

Example 63. The angular momentum of a flywheel having a moment of inertia of 0.4 kg m2 decreases from 30 to
20 kg m2/s in a period of 2 second. The average torque acting on the flywheel during this period is :
(A) 10 N . m (B) 2.5 N . m (C) 5 N . m (D) 1.5 N . m
L
Sol.
t
10
5 Nm
2
Hence (C) is correct.

Angular Impulse Theorem



As ext is equal to rate of change of angular momentum L , we have

ext
dt dL

= Lf Li
The LHS is called angular impulse which equals the change in angular momentum.

Some dynamical Relationships

www.physicsashok.in 64
PHYSICS ROTATIONAL MOTION ROTATIONAL MECHANICS
SHEET
Quantities Single Particle System of Particles
Angular momentum

about a ponit l = r p L = ri p i

L = Lcm + rcm M Vcm
about z-axis lZ = lcos LZ = L cos = Izz
Torque


about a point = r F = ri Fi ; = ext

about z-axis Z = cos Z = cos

Equation of Rotational Dynamics



dl dL
- about a point = =
dt dt

dl dL z
- about z-axis 2 = z 2 =
dt dt
z = Izz
dI z
z = Izz +
dt z
Angular Impulse

about a point dt = lf li ext dt L f Li

- about z-axis dt
z = lzf lzi extzdt = Lzf Lzi
Conservation of angular momentum

about a point = 0 l = constant ext = 0 L = constant
about z-axis 2 = 0 lZ = constant ext z = 0 LZ = constant

A
Example 64. The moment of inertia of semicircular plate of radius R and mass M about
axis AA in its plane passing through its centre is
MR 2 MR 2 MR 2 MR 2 A
(A) (B) cos2 (C) sin 2 (D)
2 4 2 4
mR 2
Sol. Here Ix
4
A
mR 2
and Iy
4 y
x
x = cos
ysin A
x
L I x x i I y y j

www.physicsashok.in 65
PHYSICS ROTATIONAL MOTION ROTATIONAL MECHANICS
SHEET
or e = unit vector along the axis.
e e cos i e sin j
e cos i sin j
mR 2 mR 2
Laxis I L . e cos2 sin 2
4 4
mR 2 mR 2
I cos2 sin 2
4 4
mR 2
I Hence (D) is correct option.
4

Question No. 65 & 66


A uniform rod is fixed to a rotating turntable so that its lower end is on the axis of the 20
turntable and it makes an angle of 20 to the vertical. (The rod is thus rotating with
uniform angular velocity about a vertical axis passing through one end). If the turntable
is rotating clockwise as seen from above.
Example 65. What is the direction of the rods angular momentum vector (calculated about its lower end) ?
(A) vertically downwards (B) down at 20 to the horizontal
(C) up at 20 to the horizontal (D) vertically upwards.

Sol. cos i sin j
Ix = 0 x
x
m l2
Iy
3 y
x
Lx = Ix x =0
m l2
and L y I y y sin
3
m l2
L sin i
3
ml2
L sin 20 j Hence (B) is correct.
3

Example 66. Is there a torque acting on it, and if so in what direction ?


(A) yes, vertically (B) yes, horizontally
(C) yes at 20 to the horizontal (D) no

mg sin

Sol. O
mg
mg cos
Torque of mg about O is

www.physicsashok.in 66
PHYSICS ROTATIONAL MOTION ROTATIONAL MECHANICS
SHEET
l
(mg sin)
2
Hence (B) is correct.

CONSERVATION OF ANGULAR MOMENTUM


The angular momentum of a system about a ponit can be changed only by an external torque about that point.
This is suggested by

dL
= ext .
dt
If the external torque about a point be zero (either the vector sum is zero, or the torque is absent) then

dL
=0
dt
The angular momentum will not change with time. This is known as law of conservation of angular momentum.
If the net torque or external torque about a point be zero, the total angular momentum about that point is
conserved.
If z = 0, Lz = constant.
If torque about an axis be zero, the angular momentum about the axis conserved:
Izz = constant.

C49: Is the angular momentum of the Earth conserved ? Explain.


Ans: Yes. The line of action of gravitational force due to the Sun on the earth
passes through the Sun. VCM

r
ext = r F = 0 L
S F

L s = constant.
Thus, torque on the Earth about the Sun is zero and hence angular momentum abou the Sun is conserved.

C50: Classical Angular momentum of the electron about nucleus in H-atom is conserved. Explain why.
Ans: Nucleus exerts electrostatic pull on the electron. Its line of action is passing through the nucleus. Hence its
torque is zero. F

ext = r F = 0 r

When external torque is zero, angular momentum remains constant. This is


why classical angular momentum of the electron about the nucles is conserved.

C51: A person is rotating with turn table without considerable friction. His hands are stretched. Now he folds his
hands. His angular speed increases. Explain.
Ans: Since friction is neglected, there is no torque about vertical axis of the system. Hence
I1 = I22
As the man folds hands, his moment of inertia decreases. The angular speed increases.

Example 67. A particle is moving in a circular orbit of radius r1 with an angular velocity 1. It jumps to another

www.physicsashok.in 67
PHYSICS ROTATIONAL MOTION ROTATIONAL MECHANICS
SHEET

circular orbit of radius r2 and attains an angular velocity 2. If r2 = 0.5 r1 and assuming that no external torque
is applied to the system, then the angular velocity 2, is given by :
(A) 2 = 41 (B) 2 = 31 (C) 2 = 21 (D) 2 = 1
Sol. Applying conservation principle of angular momentum,
mr121 = mr22 2
or r12 1 = r22 2
or r12 1 = (0.5r1)2 2
or 1 = 0.25 2
or 2 = 4 1 Hence (A) is correct.

Example 68. A man, sitting firmly over a rotating stool has his arms streched. If he folds his arms, the work done by
the man is
(A) zero (B) positive
(C) negative (D) may be positive or negative.
Sol. The kinetic energy is
1 2
T I
2
But L = I
L

I
2
1 L L2
T I
2 I 2I
But in this case, If < Ii
Tf > Ti (Because angular momentum remains conserved)
But w = T = Tf Ti > 0
Hence, workdone is positive. Hence (B) is correct.

Example 69. A small object is attached to a light string which passes through a hollow tube. The tube is held by one
hand and the string by the other. The object is stet into rotation in a circle of radius r1. The string is then pulled
down, shortening the radius of the circle to r2. The ratio of the new kinetic energy to original kinetic energy is
2 2
r1 r1 r2
(A) r (B) 1 (C) (D)
2 r
2 r
1

Sol. By conservation principle of angular momentum,


mv1r1 = mv2r2
v1r1 = v2r2 ...(1)
1
mv 22 v 2 r 2
2 2 1
But ratio = 1 Hence (C) is correct.
mv12 v1 r2
2

Example 70. A small bead of mass m moving with velocity v gets threaded on a stationary

www.physicsashok.in 68
PHYSICS ROTATIONAL MOTION ROTATIONAL MECHANICS
SHEET
semicircular ring of mass m and radius R kept on a horizontal table. The ring can freely
R
rotate about its centre. The bead comes to rest relative to the ring. What will be the final O v
angular velocity of the system ? m
(A) v/R (B) 2v/R (C) v/2R (D) 3v/R
Sol. Applying conservation principle of angular momentum,
mvR = (mR2 + mR2)
vR v
2
Hence, (C) is correct.
2R 2R

Example 71. A child with mass m is standing at the edge of a disc with moment of inertial I, radius R, and initial
angular velocity . See figure given below. The child jumps off the edge of the disc with v
tangential velocity v with respect to the ground. Then new angular velocity of the disc is

I 2 mv2 (I mR 2 ) 2 mv 2
(A) (B)
I I

(C)
I mvR
(D)
I mR mvR
2

I I
Sol. Applying conservation principle of angular momentum,
(I + mR2) = I + mv1R ...(1)
But v1 R = v (By relative concept) ...(2)
From equation (1) and (2)
(I mR 2 ) mvR
Hence (D) is correct.
I

Example 72. Two men, each of mass 75 kg, stand on the rim of a horizontal large disc, diametrically opposite to
each other. The disc has a mass 450 kg and is free to rotate about its axis. Each man simultaneously start along
the rim clockwise with the same speed and reaches their original starting points on the disc. Find the angle
turned through by the disc with respect to the ground.
Sol. vrel = v1 + R
ad vrel = v2 + R
v1 = vrel R
and v2 = vrel R
Li = Lf v1 vrel
vrel
MR 2
O = mv1R + mv2R v2
2
MR 2
or = mR (vrel R + vrel R)
2
= 2mR (vrel R) ...(1)
2 = (v1 + R) t
2 2 2

v1 R v rel R R v rel

www.physicsashok.in 69
PHYSICS ROTATIONAL MOTION ROTATIONAL MECHANICS
SHEET

2
t ...(2)
v rel
From equation (1)
MR
2mR 2mR v rel
2
M
or 2m 2m v rel
2
2m v rel

M / 2 2m
2 2 2m v rel

v rel v rel M / 2 2m

4m 75 4 75
4
M 225 150 375
2m
2
4
radian
5

Example 73. A uniform disc of mass m and radius R rotates about a fixed vertical axis passing through its centre
with angular velocity . A particle of same mass m and having velocity 2R towards centre of the disc collides
with the disc moving horizontally and sticks to its rim. Find
(a) the angular velocity of the disc.
(b) the impulse on the particle due to disc.
(c) the impulse on the disc due to hinge.
Sol. (a) Applying conservation principle of angular momentum about centre of disc.
mR 2 mR 2
mR 2
2 2 O

mR 2

3 3
2 mR 2
2
(b) Fydt = Jy = 2 mR
mR
Fx dt J x mR
3
1
J J 2x J 2y mR 22
9

37
mR
3
37
= mR
3

www.physicsashok.in 70
PHYSICS ROTATIONAL MOTION ROTATIONAL MECHANICS
SHEET
Example 74. A block of mass m is attached to a pulley disc of equal mass m, radius r by
means of a slack string as shown. The pulley is hinged about its centre on a horizontal
table and the block is projected with an initial velocity of 5 m/s. Its velocity when the
string becomes taut will be
(A) 3 m/s (B) 2.5 m/s (C) 5/3 m/s (D) 10/3 m/s
Sol. For block,
Tdt = mv mv0
For disc, rTdt = I
I Iv
or Tdt 2
r r
mv
or mv0 mv
2
v 3
or v0 v v
2 2
2 2 10
v v0 5 m/s Hence (D) is correct.
3 3 3

Example 75. A uniform rod of length l is given an impulse at right angles to its length as cm
shown. Find the distance of instantaneous centre of rotation from the centre of the rod. x

Sol. Let instantaneous axis of rotation is passing through point O. impulse


vcm y = 0
vcm = y
vcm
v O C
\ y cm
y x Fdt
Here Fdt = mvcm
ml2
xFdt = I=
12
m l 2
or Fdt
12 x

m l 2
or m v cm
12 x

vcm l2
y
12 x

Example 76. A solid sphere of mass m and radius R is placed on a smooth horizontal surface. A sudden below is
given horizontally to the sphere at a height h = 4R/5 above the centre line. If I is the impulse of the blow then
find
(a) the minimum time after which the highest point B will touch the ground.
(b) the displacement of the centre of mass during this internal.

www.physicsashok.in 71
PHYSICS ROTATIONAL MOTION ROTATIONAL MECHANICS
SHEET
Sol. (a) Here Fdt = mvcm
and hFdt = Icm Fdt
2
or hI mR 2
5
4R 2
or I mR 2
4 5
2I

mR
= t
or = t
mR
t
2I
(b) Fdt = mvcm
or I = mvcm
I
v cm
m
I mR R
S v cm t
m 2I 2

Angular momentum conservation in collision


If we consider a ball colliding with a bat, and neglected the force exerted by hand during collision, there is no
external torque and bodies are freely colliding.
If we pivot a body and then consider another body colliding with it, the pivot exerts an impulsive external force,
in general. The external force acting on the system during collision is not negligible.

Case (a): Free collision of free extended bodies: We use two conservation laws in this case.
(i) Total momentum is conserved.
(ii) Angular momentum about any point is cosnerved.

Case (b) Pivoted bodies: In this case, external force ( Fext ) on the system O

arises from pivot (O) and is considerable. r

(i) Total momentum is not conserved. N


N
(ii) Angular momentum is conserved about the pivot.

ROTATIONAL COLLISIONAND ANGUALR MOMENTUM


We have discussed head on and oblique collisions. In this section we will discuss the different cases of collision
of two bodies in which during or after collision rotational motion of the body is also taken into account.

www.physicsashok.in 72
PHYSICS ROTATIONAL MOTION ROTATIONAL MECHANICS
SHEET

A A

L L

w
m
u v B
B

We first discuss teh simples case of a collsion which is shown in figure. A rod AB of mass M and length L is
hinged at the point A, is hanging vertically. A small ball of mass m moving with a speed u horizontally strikes the
end B of the rod elastically. Let us consider that after collision the rod will start rotating with angular speed
and the ball continues to move forward with a less speed v. Sot the ball may rebound but we need not to
consider this case, as if b all will rebound, the result will give the velocity v negative. The values of v and w in this
problem can be obtained in two ways using energy and angular momentum conservation or using impulse
equations.

Note about hinge at point A: Students should note that when ball will strike the rod, an external impulse will
be developed at the hinge which will prevent and rod to move forward, as rod as only rotate, can not translate.
Due to this external linear momentum of the system can not be conserved but angular momentum can be
conserved about the hinge.
As stated in above paragraph, here we cannot use linear momentum conservation but as so external torque is
present we use angular momentum conservation about the hinge as
ML2
muL = mvL + .....(i)
3
Here mul is the angular momentum of the ball before striking the rod and it is the only angular momentum before
collision as rod is at rest. After collision aas ball will move with a speed v in same direction, its angular momentum
is mul and that of rod is I as it starts rotation with initial angular velocity .
As collision is elastic we use kinetic energy conservation before and after collison is
2
1 1 1 ML
2 2
mu = mv + 3 .....(ii)
2 2 2
Now using the above two equations, we get the values of v and .
The other way of solving this problems is by breaking equation (i). It can be
broken in two parts if required in some problems using impulse equations.
Again consider the initial case when ball strikes the rod, a rnormal force is
developed between the surface of ball and that of rod due to the push of ball
A
against rod, the situtation is shown in figure. This interaction for F acts for a
short duration (say dt) when the ball and rod are in contact. It retards the ball
and its torque accelerates the rod in anticlockwise direction. We can use the L
impuse equatons for ball (linear) and rod (angular).
For motion of ball, we have the linear impulse equation is m Fdt
u
mu = Fdt = mv .....(iii)
Fdt B
For motion of rod, we have the angular impulse equation as
0 + Fdt = I
ML2
or FLdt = 3 .....(iv)

www.physicsashok.in 73
PHYSICS ROTATIONAL MOTION ROTATIONAL MECHANICS
SHEET
If we use the above two equations along with equation (ii), we can solve the problem. Here if eqauation (iii) and
(iv) are merged, it results equation (i). This type of working might be of more utility in solving the problem
instead of directly using energy conservations.
The previous case might also be of inelastic or partical elastic collision. If in previous problem with same initial
conditions the collision is partical elastic and the coefficient of restitution is given as e, the angular momentum
conservation equation (i) remains same as no external torque is acting but now we cannot use enrgy conservation
as collision is not perfectly elastic. Here we use the definition of coefficient of restitution that it is the ratio of
velocity of separtion after collision on the velocity of approach before collision. In this case it is used as
L v
e=
u
or L = v eu .....(v)
Solving the equations (i) and (v), we get the results v and . If this collision were perfectly inelastic, we use
e = 0, which comes from equation (v), v = L, as no separation occurs in perfectly inelastic collision. Here it is
important to be noted that the ball and rod will be separate even if inelastic collision takes place because ball is
in translational motion and rod is in rotational motion. Figure explains the situation.

A A A A

L L L


m m B
u v B u
B B
e=0 v = L

Now we consider one more when ball sticks to the end of the rod and will rotate along with the rod. It is a
specific type of inelastic collision as ball sticks to rod. Here again we can neither use energy conservation nor
the coefficient of restitution equation. Only angular momentum conservation equation is sufficient for solving the
solving the problem as one of previous varialbes reduces, the linear velocity of the ball. Consider figure. Ball
and rod will start together in rotation with an initial angular velocity w. Thus we write the angular momentum
conservation equation as
muL = I [H = I = M.I. of rod plus ball]
ML2 2
muL = 3 mL .....(vi)

Heremoment of inertia of the system taken combined that of rod pulsball as both are in rotational motion with
angular velocity w after collision. Example (vi) will give us the angular velocity of the system.
A
Now we discuss another case when rod is not hinged at end end. A
Consider the collision shown in figure. Here rod AB is placed on a L
L
smooth surface which is free to move on it and ball moving shown in v 2
figure.
m d
As no external force is acting on the system, here we can also conserve u
v 1
linear momentum along with angular momentum. When the ball strikes B
B
the rod, the instantaneous axis of rotation is taken at the centre of the
rod hence the equation of angular momentum conservation is written
abut the centre of the rod. If after collision, ball moves with speed v1
and centre of rod moves with some angular velocity, let it be , then
according to angular momentum consevation we have.

www.physicsashok.in 74
PHYSICS ROTATIONAL MOTION ROTATIONAL MECHANICS
SHEET
mud = mv1d l
ML2
or mud = mv1d + 12 ....(vii)

From linear momentum conservation, we have
mu = mv1 + Mv2 .....(viii)
Equations (vi), (vii) and (viii) gives the unknown parameters after collisions v1, v2 and if this collision is not
elastic, we use coefficient of restitution instead of energy convervation equation (iv), as
(v 2 d) v1
e= .....(ix)
u
Here v2 + d is the linear speed of point P after collision as rod moves translationally with velocity v2 and
rotates with angular velocity and v1 is the final velocity of the ball after collision. For inelastic collisions, e can
be taken as zero.

Example 77. A rod of length R and mass M is free to rotation about ta horizontal axis
passing through hinge P as in figure. First it is taken aside such that it becomes
M P R
horizontal and then released. At the lowest point the rod hits the block B of mass m
and stops. Find the ratio of masses such that the block B completes the circle. m
Neglect any friction. B
Sol. Minimum velocity required by block m to complete the motion in 5gR conserving mechanical energy
1 2 R
I Mg
2 2
MgR

I
Cons. angular momentum wrt P before & after collision.
I m.R 5gR

MgR P
I. mR 5gR
I R.M m
5gR

MgRI = m2R2 5 gR
ML2 MR 2
puting I (since L = R)
3 3
M
15
m

Example 78: A rod of mass M is lying on a smooth horizontal table. A small disc sliding at speed V0 hits the rod at
one end normal to its length. The disc comes to a stop just after the collision.
(a) Write the velocity of the centre of mass of the rod just after the collision.
(b) Write the angular velocity of the rod about its CM just after collision.
(c) How much distance does the rod move in one rotation about CM ?
Sol: Since there is no external force on the rod + disc system, momentum is conserved. Let V be the velocity of CM
of the rod.

www.physicsashok.in 75
PHYSICS ROTATIONAL MOTION ROTATIONAL MECHANICS
SHEET

(a) Conserving momentum,


mv0 + M 0 = m 0 + MV
mv 0 M
V= .....(i)
M v
m
(b) Angular momentum conservation about a point of space initially coinciding with
the centre of the rod:
L
mv0 = I
2
ML2

12
6mv 0
= .....(ii)
ML
(c) As there is no friction, the rod slides with constant velocity after the collision. Let it move by s in time t.
In one rotation angle turned is 2. Hence
2 = t .....(iii)
v2
s =

mv 0 2 L
= =
M 6mv0 3

ML

Example 79. A rod AC of length L and mass m is kept on a horizontal smooth plane. It is free to rotate and move.
A particle of same mass m moving with velocity v strikes rod at point B which is at a distance L/4 from mid
point making angle 37 with the rod. The collision is elastic. After collision find
(a) the angular velocity of the rod. A B C
37
(b) the distance which centre of the rod will travel in the time in which it makes l/4
half rotation.
(c) the impulse of the impact force.
Sol. (a) The ball has v, component of its velocity perpendicular to the length of rod immediately after the collision
u is velocity of COM of the rod and is angular velocity of the rod, just after collision. The ball strikes the rod
with speed v cos 53 in perpendicular direction and its component along the length of the rod after the collision
is unchanged. u
Using for the point of collision.
Velocity of separation = Velocity of approach
3v l
u v ...(1) V
5 4
D
Conserving linear momentum (of rod + particle), in the direction to the rod.
3
mv . mu mv ...(2)
5
Conserving angular moment about point D as shown in the figure

www.physicsashok.in 76
PHYSICS ROTATIONAL MOTION ROTATIONAL MECHANICS
SHEET

l ml 2
0 0 mu
4 12
l
u ...(3)
3
24 v 72 v
By solving u , W
55 55 l

(b) Time taken to rotate by angle t = Ndt
l/4

l
In the same time, distance travelled = u2 . t =
3
Using angular impulse-angular momentum equation.
l m l 2 72 v
N . dt . 4
.
4 55 l
24mv
N . dt
55
or using impulse momentum equation on Rod
24m
Ndt mu 55
m
Example 80: In the previous example for what ratio is the collision elastic ?
M
Sol: For elastic collision initial kinetic energy should be equal to the KE just after the collision. That is,
K i = Kf
1 1 1
mv 20 = 0 + MV2 + I2
2 2 2
1 2
1 ML2 2
= MV +
2 2 12
2 2
1 mv 0 1 ML2 6mv 0
= M. +
2 M
2 12 ML

m2 m2
m = +3
M M
m
1 =4
M
m 1
=
M 4

www.physicsashok.in 77
PHYSICS ROTATIONAL MOTION ROTATIONAL MECHANICS
SHEET

Example 81: A small disc and a thin uniform rod of length L, whose mass is times greater than the mas of disc,
lie on a smooth horizontal plane. The disc is set in motion, in horizontal direction and perpendicular to the rod,
with velocity v, after which it elastically collides with the end of the rod. Find the velocity of the disc angular
velocity of the rod after the collision. At what value of h will the velocity of the disc after the collision be equal
to zero ? Reverse the direction ?
Sol: The situation is shown in figure. If mass of disc is m, then mass of rod hm. if v1 and v2 be the velocities of the
disc and rod after collision, using linear momentum conservation, we have
mv = mv1 + hmv2
or v = v1 + v2 ....(x)
If after collision, rod starts rotating with angular speed w, using consrvation of angular momentum we have

L mL2 L
mv = + mv1
2 12 2
1
or v = L + v1
6
6(v v1 )
or = .....(xi)
L
As collision is elastic, using kinetic energy conservation, we get
1 2 1 1 1
mv = mv12 + v 22 + I2
2 2 2 2

2 2
3
2
(v v1 )2
or v = v + (v - v1) +
1 2
Solving for v1, we get
4
v1 = v
4
Here we can see that v1 is zero if h = 4 and v1 will be negative when > 4.
v0 m
Example 82. On a smooth table two particles of mass m each, travelling with a
velocity v0 in opposite directions, strike the ends of a rigid massless rod of length l
l, kept perpendicular to their velocity. The particles stick to the rod after the
collision. Find the tension in rod during subsequent motion. v0 m
l l
Sol. mv 0 mv 0 I
2 2
ml 2
mv0 l
2
ml 2
or mv0 l
2
2v
0
l
2
l 2 l 2v0
T m m
2 2 l
2mv02
T
l

www.physicsashok.in 78
PHYSICS ROTATIONAL MOTION ROTATIONAL MECHANICS
SHEET

Example 83. A solid uniform sphere of radius R and mass M rolls without slipping
0
with angular velocity w0 when it encounters a step of height 0.4 R. Find the
angular velocity immediately after inelastic impact with the rough step. h
Sol. Applying conservatio principle of angular momentum about point of impact,
mvcm (R h) + Icm 0 = I
2 7
mR 0 R h mR 2 0 mR 2
5 5
7 7
or mR 2 0 mRh 0 mR 2
5 5
7 7
or R 0 h 0 R
5 5
5 7 5h
0
7R 5
5 7R 5h
0
7R 5
0
7R 2R
7R
5 0

7

CONCEPT OF TOPPLING
Rolling is generally a difficult concept to grasp. The primary question that rises in
the students mind is how can a point in the body have zero velocity and yet the F
Fig. : Force acting on a flat body
body moves forward. Let us consider a simple example as shown in figure.
Assume that a force F acts on a flat body as shown above.

How will the body move ?


Obviously the body will move forward translationally. The situation is shown v
v
in figure. Fig. : Motion of the body

All points in the body will move with the same velocity and acceleration. Therefore, the body will move
translationally. Now assume that the body becomes less wide.
F
What will the situation be ?
This is shown in figure. Fig. : Force acting on a
body with less width
In this case as well, the body is likely to move translationally. However as the body
becomes less and less wide, something else happens. Let us now consider the situation
in figure.
In this case as the force F increases, the frictional force also increases. At some point

www.physicsashok.in 79
PHYSICS ROTATIONAL MOTION ROTATIONAL MECHANICS
F SHEET

the body starts toppling. At this instance the body topples about point A. the motion in
Fig. : Force on a very
this situation is shown in figure. thin body
F
As the force increases, the body starts toppling about point A. The body now has
fallen down. Let us step back and consider the situation in greater depth.
Obviously this kind of motion is not translational. The body has moved forward
as a whole. We can say that though because the centre of mass of the body has A
Fig. : Body topples
moved, yet the motion has not been translational. What can we say about the
motion ? We can observe the following points :
Different points in the body move with different velocities and accelerations. Whereas the topmost point has
the greatest velocity, the point A has zero velocity.
The point of contact A is at rest, even while the body moves.
The body has moved forward even when one point has zero velocity.
We can therefore conclude that the body can move forward even if a single point in the body has zero velocity.
each one of these points is shown below : v

Different points in the body move with different velocities and accelerations. Whereas
v1
the topmost point has the greatest velocity, point A has zero velocity. This is shown in
v2
figure.
Fig. : Different points have
different velocities
The point of contact A is at rest, even while the body moves. This is shown in figure.

The body has moved forward even when one point has zero velocity and zero
acceleration. This is shown in figure. 0
A
F Fig. : Point A has zero velocity.
Velocity profile of other points

B
F
C

Fig. : Point has moved


The centre of mass was initially at B. It has then moved forward to point C. Therefore the body has moved
forward.
We call this kind of motion toppling. Now let us look at when exactly a body will topple :
A body will topple when the base is smaller.
A body will topple when the force exerted is higher from the ground level.
A body resting on a surface, which has friction, is more likely to topple.
Look at the situation below.
Which body is more likely to topple ?
F F

B C

Fig. : Which body will topple ?


Which body will topple ?
www.physicsashok.in 80
PHYSICS ROTATIONAL MOTION ROTATIONAL MECHANICS
SHEET
Obviously body B, because body C is wider then body B. This illustrates the first point that for a body to
topple, the base has to be smaller.
Consider the next situation now :
F

B F C

Fig. : Which body will topple ?


In this case, the width is the same.

Which body will topple ?


In the second case, the height at which the force is applied is lower. Therefore the body as shown first is likely
to topple.
If the surface has no friction, the body will be more likely to move by pure translation than by toppling.
Look at the situation below.
F
C F

Fig. : Which body will topple ?

Which body will topple ?


Obviously the the square, because it has a less wide base. Why? Explanations follow. F
Similarly, we can see that a pentagon is more likely to topple than a square. Similarly
an octagon will be more likely to move by toppling than a pentagon. Therefore, we
have seen that as the body becomes less and less wide, it tends to move more and
more by toppling. Fig. : An octagon is more likely
to move by toppling

The question, however, still remains unanswered. F

How and when will a body move by translation and when will it move B b
by pure translation ?
Let us consider the situation shown in figure
a A
Assume that the width is a and height is b. Let us draw the force diagram, just as Fig. : When will the
the body is about to topple. Assume that () is the coefficient of friction. We know body topple ?

from the earlier discussions that the body will topple when the normal force shifts to the rightmost point.
For the body to topple about A, taking moments about A, (why do we take point A ?
Because maximum number of force i.e. friction and normal force pass through A. It is F
also correct to take torque about centre of mass, though this will be lengthier). B
b
mga mg N
Total torque = Fb
2 a A
Fig. : Force diagram on the body
If torque > 0 then the body topples.

www.physicsashok.in 81
PHYSICS ROTATIONAL MOTION ROTATIONAL MECHANICS
SHEET
mga
There F>
2b
What this means is that
mga
If F < then body will move in pure translation and will not topple.
2b
mga
If F > , the body will topple.
2b
Let us study the concept to toppling in a little more mathematical detail. While we outline every scenari, try to
visualize physically the motion of the body.
mga
We have already established that for force F > , the body will start moving by topping.
2b
Let us now bring into this picture the friction, width and height of the force and study toppling.
Consider first a square body of width a. Assume that the force is applied at the top of
the square. Assume also that the coefficient of friction is . The situation is shown in F
figure. N
mg
Using the equations described above, the body will topple for force
f
mga Coefficient of friction =
F> (since a = b). Fig. : Condition for
2b toppling a square

mg
The body will topple when F > or F > 0.5 mg
2

If the coefficient of friction = 0.2 what will happen to the square ? Will it topple or translate ?
For the body to just topple or translate, the force equations are same. The force equation for just toppling or
just translation is
Ff=0 F=f ...(1)
N mg = 0 N = mg; f mg
If = 0.2, Maximum frictional force = 0.2 mg
Therefore using equation (1) the force required to move the body by translation is F = 0.2 mg. The Force
required to topple the body is F = 0.5 mg.
Therefore transalation happens first. The body will translate and not topple.

What happens if the coefficient of friction is 0.6 ?


In such a case, the force required to topple is F = 0.5 mg. This remains unchanged. However the force
required to move the body by translation is 0.6 mg. In this case toppling will take place first.
We can now summarize the scenario below :
For > 0.5, the body will move by toppling.
For < 0.5, the body will move by translation.

www.physicsashok.in 82
PHYSICS ROTATIONAL MOTION ROTATIONAL MECHANICS
SHEET

This off course assumes that the force is exerted at the top.

What happens if the force is exerted at any other point ?


In such a case, the coefficient of friction will have to be higher to move the body by toppling. Visualize this. It
is very difficult for a square to topple. it requires a very rough surface ( > 0.5).

What is the value of frictional forces when < 0.5 and > 0.5 ?
The figure explains it all.
F = mg F < mg

N
mg N mg
A A
f = mg f < mg
Coefficient of friction = < 0.5 Coefficient of friction = > 0.5
Moves by translation Moves by translation
Friction = mg Friction < mg

Note the interesting fact in the second case. The body topples and moves. but during this time f < mg.
We all know that when a body translates frictional force = mg.
The interesting thing to note is that a body a shown above can move in other methods not necessarily by just
translation. It can topple.

And what happens when the body topples ?


The body moves, but yet frictional force < mg. This is a very important point. Students need to visualize this
scenario very clearly in their minds. When the body topples, frictional force is less than mg. Also it requires
less effort to move a body. This is a classic property of toppling.
Let us summarize the characteristics of toppling that we have already discussed earlier :
All points have different velocities and accelerations.
The lowermost point will have zero velocity and acceleration when the body just starts toppling.
Frictional force is less than the maximum value of friction.
It requires less effort or force to move the body by toppling.

If this is the case, why doesnt a square body not topple so easily ?
The reason is very simple. The coefficient of frictions needs to be higher than 0.5 for a square body to topple
(assuming that the force acts at the topmost point). This does not happen in reality because if = 0.5, the
surface must really be very rough. Most surfaces are smoother than this. Hence a square body prefers to move
by translation and not by toppling. F

But now let us consider a hexagon. a hexagon. Again assume that the coefficient of
mg N
friction is and that the force is being exerted at the top. Assume the width of the sides
A
to be a.
Fig. : Force required to
topple a hexagon

Again we can take torque about A. The side of the hexagon is a. Therefore the height
will be a tan 60. You can easily see that this is the case, by looking at figure.
a tan 60

60
a
Fig. : Height is a tan 60
www.physicsashok.in 83
PHYSICS ROTATIONAL MOTION ROTATIONAL MECHANICS
SHEET

mga
The torque about A is, F a tan 60 =0
2
mga
Therefore the body will topple for F >
2 tan 60
Now tan 60 ~ 1.732. Therefore the body will topple for F > 0.21 mg
You can now see the huge difference. for < 0.21, the body will translate and for > 0.21, the body will
topple.
The situation is again reproduced below
F F

mg N mg N
A A
f = mg f = mg
Coefficient of friction = < 0.21 Coefficient of friction = > 0.21
Moves by translation Friction = mg Moves by translation Friction < mg

Fig. : Motion for toppling a hexagon body


Again the same conditions that were discussed before, regarding toppling hold true. When a body just topples,
the coefficient of friction is less than mg. The point of constact is a rest (i.e. point A). Different points have
different accelerations.
These are typical characteristics of toppling and you should now be clear about these.
Note that we can make an interesting observation. The value of required to make a square topple is 0.5,
whereas the value of required to make a hexagon topple is 0.21. this obviously bears out the fact that it is
easier to topple a hexagon than a square. Similarly you will find that the minimum value of required to make
a body topple will decrease if the body is an octagon. It will reduce still further for a decagon. This means that
a decagon will move by toppling more easily than by pure translation. The force required to move it by toppling
is less.

What can we say about an infinite-sided polygon ?


Extending the same logic, you will find that for values of < 0, the body will topple.

What does this mean ?


The body always topples. Even if a small force is applied, the body will topple. Therefore note the interesting
observation. Circular bodies move predominantly by toppling. Again let us reproduce the properties of toppling
here.
All points have different velocities and accelerations.
The lowermost point will have zero velocity and acceleration when the body just starts toppling.
Frictional force is less than the maximum value of friction.
It requires very little effort to move a body by toppling.
These are precisely the properties of toppling a circular body as well. The only difference is that since the side
of the circle is 0, virtually no force is required to topple the body. The body moves by toppling and F < N.
Now let us introduce a very interesting property. Toppling is Rolling. They are one and the same. Visualize
toppling motion and you can visualize rolling motion. If you look at the properties of toppling described above,
these are exactly the properties of rolling.

www.physicsashok.in 84
PHYSICS ROTATIONAL MOTION ROTATIONAL MECHANICS
SHEET
Toppling is therefore the same as rolling. To visualize rolling motion always think of toppling motion. It is easier
to visualize toppling.

How does a body move when it is toppled ?


This is shown below
C
F F F

B A
C

A B C
Fig. : Visualization of toppling

To start with, A is at the edge. During the first round of toppling, A is at rest and B falls down. Now the body
topples about point B. Point C touches the gound. In the next round, the body topples about point C.
An interesting point to observe here is that the body topples about different points. First it is point A, which is
at rest, and the body topples about A. Then the body topples about point B and B is at rest.
Therefore different points have zero velocities and accelerations.
This is exactly the situation with rolling as well. In rolling the point of contact has zero motion. This point
however keeps on changing. It is very important for the students to be able to visualize toppling and understand
rolling in this contaxt.
To summarize
Toppling is the same as rolling.
During toppling or rolling F < mg.
It requires negligible force to topple or roll a disc.
Point of contact has zero motion with respect to the surface.
Different points have zero motion at different points of time.
F > 0.21 mg
Another question that the student may ask is what happens in the case of a hexagon
if the value of the force F > 0.21 mg. In such a case, the body will difinitely topple,
but the point of contact will also move. This will not have zero acceleration. This
A
is shown in figure. Acceleration

f = mg
The point of contact will move. The body will topple, but in a different way. the Coefficient of friction = 0.21
Fig. : Value of force is higher than
point of contact moves. We call this kind of motion sliding or skidding. Notice the force required to topple.
that when a body slides or skids,
Frictional force mg and not less than mg.

F
What is the minimum force required for the body to topple ? Mass=1 kg
The body will topple when the normal force passes through the edge. The force
diagram at this stage is shown in figure. mg N
f
A
taking torque about point A we have Fig. : Force diagram when
body is just going to topple.
mg (1/2)= 4F
F = (1/8)Mg = 0.125 mg
The force required for the body to move by translation is 0.2 mg, but the body topples when the force is 0.125
www.physicsashok.in 85
PHYSICS ROTATIONAL MOTION ROTATIONAL MECHANICS
SHEET
mg. Therefore, it is obvious that the body will topple before it translates.

Example 84. A uniform cube of side b and mass M rest on a rough horizontal table. A
horizontal force F is applied normal to one of the face at a point, at a height 3b/4 above F
the base. What should be the coefficient of friction () between cube and table so that b 3b/4
is will tip about an edge before it starts slipping ?
(A) > 2/3 (B) > 1/3 (C) > 3/2 (D) None
b 3b
Sol. mg F ...(1)
2 4
F = f < mg ...(2)
From equation (1) and (2)
F < mg
2mg
g
3
2
or Hence (A) is correct.
3

Example 85. A uniform cylinder rests on a cart as shown. The coefficient of static friction
between the cylinder and the cart is 0.5. If the cylinder is 4 cm in diameter and 10 cm
in height, which of the following is the minimum acceleration of the cart needed to
cause the cylinder to tip over ?
(A) 2 m/s2 (B) 4 m/s2
(C) 5 m/s2 (D) the cylinder would slide before it begins to tip over.
Sol. Taking torque about O,
ma0 5 > mg 2 ma0
2g O a0
or a0
4
mg
2 10
a 0 min 4 m / s2 Hence (B) is correct.
5

www.physicsashok.in 86
ROTATIONAL MECHANICS
THINKING PROBLEMS

1. Can the mass of a body be taken to be concentrated at its centre of mass for the purpose of calculating its
rotational inertia?
2. If two circular discs of the same weight and thickness are made from metals of different densities, which disc will
have the larger moment of inertia about its central axis?
3. About what axis would a uniform cube have its minimum rotational inertia?
4. Consider four bodies : a ring, a cube, a disc and a sphere. All the bodies have the same mass. The ring, disc and
sphere have the same diameter, equal to the length of the cube on each edge. All rotate about their axes through
their respective centres of mass. Which one has the largest moment of inertia?
Which is the smallest?
5. A person can distinguish between a raw egg and a hard boiled one by spinning each one on a table. Explain.
6. A wooden sphere rolls down two different inclined planes of the same height but different inclines. Will it reach
the bottom with the same speed in each case ? Will it take longer to roll down one incline than the other? If so,
which one and why ?
7. A man, with two equal masses held at arms length, stands on a rotating table. If he moves the masses and puts
them over his shoulders, does this change his speed of rotation? Explain.
8. A student stands on a platform that can rotate only about a vertical axis. In his hand he holds the axle of a rim-
loaded bicycle wheel with its axis vertical. The wheel is spinning about the vertical axis with an angular speed 0,
but the student and the platform are at rest. The student turns the wheel through 180, that is, the axle of the
wheel is held downward by the student. What happens ?
9. A diver can turn several somersaults before striking water. Explain.
10.Show that in the course of diving, the rotational kinetic energy of a diver, turning somersaults before striking
water, increases. What is the source of this increased energy?
11. If the polar ice caps melt and spread uniformly, how will the length of the day be affected ?
12.A circular turntable rotates at constant angular velocity about a vertical axis. There is no friction and no driving
torque. An icepan containing ice also rotates along with it. The ice melts but none of the water escapes from the
pan. Is the velocity now greater, the same as, or less than the original velocity? Give reasons for your answer.
13.In order to get a billiard ball to roll without sliding from the start, the cue must hit the ball not at the centre but
exactly at a height of 2/5 R above the centre, where R is the radius of the sphere. Explain.
14.A student stands on a turntable with a rim-loaded bicycle wheel. he holds the shaft of the wheel vertical and in the
beginning, there is no motion of the turntable. Now the student is asked to rotate the wheel in a clockwise
direction relative to him. What happens to the motion of turntable + student?
15.The melting of the polar ice caps is supposed to be a possible cause of the variation of the earths time of
rotation. Explain.

www.physicsashok.in 87
ROTATIONAL MECHANICS

SOLUTION OF THINKING PROBLEMS

1. No.
1 2 1
2. I1 = mr1 and I 2 = mr22
2 2
I1 r12
=
I 2 r22
Now m r12 t 1 r22t 2
r12 2

r22 1
I1 2 1
or I
I 2 1
Thus, the one with greater density will have less rotational inertial.
3. About a diagonal, because the mass is more concentrated about a diagonal.
4. I (ring) = (1/4)ma 2 where a is the length of each edge of the cube,
I (cube) = (1/6)ma 2, I (disc) = (1/8)ma 2, I (sphere) = (1/10) ma 2.
I1 : I2 : I3 : I4 = 1/4 : 1/6 : 1/8 : 1/10 = 30 : 20 : 15 : 15.
Thus, the ring has the greatest moment of inertia and the sphere the smallest.
5. The hard boiled egg will continue to sping if they are momentarily stopped and then let go.
6. The acceleration down an inclined plane is given by a = g sin /(1 + k2/r 2) where k is the radius of gyration.
h (height of the plane) = s sin where s is the length of the plane.
1 2 1
Now s= at = g sin a t 2 / (1 + k 2 / r 2 )
2 2
t 2 sin 2 a = constant
t 1/ sin
Thus, the greater the time of descent the less the inclination to the horizontal.
g sin h
Now v 2 2as 2 2 2 v = a constant
1 k / r sin
Thus, the sphere will reach the bottom with the same speed, whatever be the inclination.
7. The only external force acting on the system (man + table) is gravity and reaction of the ground, and those
exert no torque about the axis of rotation. Hence, the angular momentum of the system is conserved about
this axis. When the man puts the masses on his shoulders, the moment of inertia of the system about the
axis decreases and so the angular speed of the system increases.
8. The platfor starts rotating with almost double the initial angular momentum of the wheel. To justify this, let
us consider wheel + student + platform asour system. The initial angular momentum of the system is I00,
where I0 is the moment of inertia of the wheel about the vertical axis. Since there is no external torque on
the system about the vertical axis, the angular momentum about the vertical axis must be conserved about
this axis. Let I be the moment of inertia of student + platform about the vertical axis. The angular momentum
of the wheel about the vertical axis is now I00.
Let be the angular velocity of the platform. Then by the principle of conservation of angular momentum
I00 = I00 + I or I = 2I00 2 0
on account of negligible m.i. of the platform.

www.physicsashok.in 88
ROTATIONAL MECHANICS
9. Let us consider the diver as a system and his limbs as its components. When he leaves the diving board, he
has a certain angular speed 0 about the horizontal axis through his centre of mass. Now there are no
external forces acting on him except gravity, and gravity exerts no torque about his centre of mass. His
angular momentum remains constant, and I00 = I. When he pulls in his limbs, his moment of inertia
decreases and consequently, his angular velocity increases.
10. Since the diver is free from external torque about the horizontal axis through his centre of mass, his angular
momentum remains constant. Therefore.
I00 = I or I 02w 02 = I 2w 2 ....(i)
When he pulls in his limbs, his moment of inertia about the horizontal axis decreases.
1 1 1 1
I < I0 or > or I 2 2 . I 02 02
I I0 I I0
1 2 1
or I I 0 02
2 2
The source of this increased kinetic energy is the diver himself, who does work when he pulls the parts of
his body together. His body energy is converted into his kinetic energy.
11. If the polar ice caps were to melt, the water formed would spread on the surface of the earth and so its
moment of inertia would increase. By the law of conservation of angular momentum, the speed of rotation
would be reduced, that is, the length of the day would increase.
12. Due to the accumulation of mass near the edge, the moment of inertia of the system increases and so the
angular velocity of the system decreases.

13. Let the ball be struck by a horizontal force F at a height d above the centre. Then the torque exerted by it
2
on the ball about its centre of F.d. By the formula = I, we have F.d = MR2
5
Let a be the acceleration of the centre of mass produced by this force. Then F = Ma. For rolling without
sliding, v = R and a = .R; F = M R
2 2
MR d MR 2 d= R
5 5

14. The system (turntable + student) will turn in the opposite direction. Since the entire system is free from
external torques, the angular momentum of the system (wheel + turntable + student) remain constant at
zero. If I = moment of inertia of turntable + student and I0 = moment of inertia about the vertical axis of
rotation of the table and 0 = angular velocity of rotation (relative to the earth), then
I00 + I= 0
I 0 0
or
I

15. A liquid of given mass has less inertia for rotational motion than the same mass of solid because a liquid
cannot sustain shearing force. It always tends to move translation-wise. Hence, the melting of the ice caps
would result in the reduction of the rotational inertia of the earth and hence, the angular speed of rotation of
the earth would increase or time of rotation decrease. If water were to spread uniformly, the result would
be as in Example 11.

www.physicsashok.in 89
ROTATIONAL MECHANICS

ASSERTION & REASON

A statement of Statement-1 is given and a Corresponding statement of Statement-2 is given just below it of
the statements, mark the correct answer as
(A) If both Statement-1 and Statement-2 are true and Statement-2 is the correct explanation of Statement-1.
(B) If both Statement-1 and Statement-2 are true and Statement-2 is NOT correct explanation of Statement-1.
(C) If Statement-1 is true but Statement-2 is false.
(D) If both Statement-1 and Statement-2 are false.
(E) If Statement-1 is false but Statement-2 is true.

1. Statement-1 : In pure rolling motion, net work done by friction is zero.


Statement-2 : Sum of translational work done by friction and rotational work done by friction is zero.
2. Statement-1 : A uniform solid cylinder rolling with angular velocity along a plane surface strikes a
vertical rigid wall. Angular velocity of cylinder when it begins to roll up a wall is less
than the initial Angular velocity ().
Statement-2 : After striking the vertical wall angular velocity
3. Statement-1 : If bodies slide down an inclined plane without rolling then all bodies reach the bottom
simultaneously.
Statement-2 : Acceleration of all bodies are equal and independent of the shape.
4. Statement-1 : The force of friction in the case of a disc rolling without slipping down an inclined plane
is zero.
Statement-2 : When the disc rolls without slipping, friction is required because for rolling condition
velocity of point of contact is zero.
5. Statement-1 : As the radius of earth reduces by 50% without any change in mass, length of a day
reduces.
Statement-2 : Angular momentum conservation provides drop in time as decreases to 25% of the
original.

6. Statement-1 : If there is no external torque on a body about its centre of mass, then the velocity of the
centre of mass remains constant.
Statement-2 : The linear momentum of an isolated system remains constant. [JEE, 07]

MATCH THE COLUMN

1. A circular body of mass M and radius R, initially spinning about its centre of mass with 0 is gently placed
on a rough horizontal surface. The moment of inertia of body about its C.M. is CM MK 2 . If the coefficient
of friction between the body and the surface is then:
Column I Column II
(A) Translational work done by the friction (P) ve
(B) Rotational work done by the friction (Q) +ve
(C) Larger the moment of inertia of body, the time (R) smaller
required for rolling motion
(D) Larger the moment of inertia of the body, (S) longer
work done by the friction
(T) greater

www.physicsashok.in 90
ROTATIONAL MECHANICS
2. In the following column - I mass of each object is m and circular of radius R. Column - II represents moment
of inertia.
Column I Column II
(A) Full ring (P) mR 2
mR 2
(B) Half ring (Q)
2
mR 2
(C) Quarter ring (R)
4

(D) Arc making an angle at the centre (S) mR 2
2
3. Suppose a force Of is applied at the top most point of a rigid body of radius R and mass M.
Column I Column II
(A) Force of friction will be zero for (P) Solid sphere
(B) Force of friction will be forward for (Q) Zero
(C) Force of friction will be backward for (R) Ring
(D) If a R then force of friction (S) No body

4. A rigid body of mass M and radius R rolling without slipping on the inclined plane, then the magnitude of force
of friction.
Column I Column II
(A) For ring (P) Mg Sin 2.5
(B) For solid sphere (Q) Mg Sin 3
(C) For solid cylinder (R) Mg Sin 3.5
(D) For hollow sphere (S) Mg Sin 2

5. A rigid body is rolling without slipping on the horizontal surface


Column I Column II
(A) Velocity at point A i.e. VA (P) V 2
(B) Velocity at point B i.e. VB (Q) Zero

(C) Velocity at point C i.e. VC (R) V

(D) Velocity at point D is i.e. VD (S) 2 V

LEVEL 1
1. A ring of mass 0.3 kg and radius 0.1 m and a solid cylinder of mass 0.4 kg and of the same radius are given,
kinetic energy and released simultaneously on a flat horizontal surface such that they begin to roll with the
same KE as soon as released towards a wall which is at the same distance from the ring and the cylinder.
Then:
(A) The cylinder will reach the wall first. (B) The ring will reach the wall first.
(C) Both will reach the wall simultaneously. (D) None of the above.

2. A solid homogeneous sphere is moving on a rough horizontal surface, partially rolling and partly sliding.
During this kind of motion of the sphere:
(A) total KE is conserved.
(B) angular momentum of the sphere about the point of contact is conserved.
(C) only the rotational KE about centre of mass is conserved.
(D) angular momentum about the centre of mass is conserved.

www.physicsashok.in 91
ROTATIONAL MECHANICS
3. The angular momentum of a projectile (in X-Y plane) about origin (point of projection) at time t has:
(A) only Z component. (B) only X and Y components.
(C) All the three X, Y and Z components. (D) only Y-component.

4. A solid sphere of radius R and mass M is pulled by a force F acting at F


the top of the sphere as shown in figure. Friction coefficient is sufficient
enough to provide rolling without slipping. Work done by force F when
the centre of mass moves a distance S is :
3FS
(A) FS (B) 2FS (C) Zero (D)
2 Rough

5. A body of mass M slides down an inclined plane and reaches the bottom with velocity v. If a ring of same mass
rolls down the same inclined plane, what will be its velocity on reaching the bottom?
(A) v (B) v 2 (C) 2v (D) v 2

6. A sphere cannot roll on:


(A) a smooth horizontal surface. (B) a smooth inclined surface.
(C) a rough horizontal surface. (D) a rough inclined surface.

7. A thin spherical shell of radius R lying on a rough horizontal surface is hit sharply and horizontally by a cue.
At what height from the ground should it be hit so that the shell does not slip on the surface?
(A) 2 3.R (B) 5 4.R (C) 5 3.R (D) 3 2.R

8. What is the moment of inertia of a triangular plate ABC of A


mass M and side BC = a about an axis passing through A,
shown in figure and perpendicular to the plane of the plate?

Ma 2 3Ma 2 Ma 2 Ma 2 45 a 45
(A) (B) (C) (D) B C
6 4 24 12

9. A thin circular ring of mass M and radius r is rotating about its axis with a constant angular velocity w, Two
objects, each of mass m, are attached gently to the opposite ends of a diameter of the ring. The wheel now
rotates with an angular velocity. (1983)
wM w M 2m wM w M 2m
(A) M m (B) (C) (D)
M 2m M 2m M

10. Two point masses of 0.3 kg and 0.7 kg are fixed at the ends of a rod of length 1.4 m and of negligible mass.
the rod is set rotating about an axis perpendicular to its length with a uniform angular speed. The point on the
rod through which the axis should pass in order that the work required for rotation of the rod is minimum, is
located at a distance of (1995)
(A) 0.42 m from mass 0.3 kg (B) 0.70 m from mass 0.7 kg
(C) 0.98 m from mass 0.3 kg (D) 0.98 m from mass 0.7 kg

11. A cubical block of side a is moving with velocity V on a horizontal a


smooth plane as shown in Figure. It hits a ridge at point O. The
angular speed of the block after it hits O is(1999) M V
O
(A) 3V 4a (B) 3V 2a (C) 3V 2a (D) Zero

12. An equilateral triangle ABC formed from a uniform wire has two small
identical beads initially located at A. The triangle is set rotating
about the vertical axis AO. Then the beads are released from rest
simultaneously and allowed to slide down, one along AB and the
other along AC as shown. Neglecting frictional effects, the quantities

www.physicsashok.in 92
ROTATIONAL MECHANICS
that are conserved as the beads slide down, are (2000)
(A) angular velocity and total energy (kinetic and potential)
(B) Total angular momentum and total energy
(C) angular velocity and moment of inertia about the axis of rotation.
(D) total angular momentum and moment of inertia about the axis of rotation.

13. One quarter sector is cut from a uniform circular disc of radius
R. This sector has mass M. It is made to rotate about a line
perpendicular to its plane and passing through the center of the
original disc. Its moment of inertia about the axis of rotation is (2001)
1 1 1
(A) MR 2 (B) MR 2 (C) MR 2 (D) 2MR 2
2 4 8

14. A cylinder rolls up an inclined plane, reaches some height, and then rolls down (without slipping throughout
these motions). The direction of the frictional force acting on the cylinder are (2002)
(A) up the incline while ascending and down the incline descending.
(B) up the incline while ascending as well as descending.
(C) down the incline while ascending and up the incline while descending
(D) down the incline while ascending as well as descending.

15. A circular platform is free to rotate in a horizontal plane about a vertical axis passing through its centre. A
tortoise is sitting at the edge of the platform. Now, the platform is given an angular velocity 0 . When the
tortoise move along a chord of the platform with a constant velocity (with respect to the platform), the angular
velocity of the platform t will vary with time t as (2002)

(t) (t) (t) (t)

0 0 0 0
(A) (B) (C) (D)

16. Consider a body, shown in figure, consisting of two identical L


balls, each of mass M connected by a light rigid rod. If an impulse M M
J = MV is imparted to the body at one of its ends, what would be
its angular velocity? (2003)
J = MV
(A) V L (B) 2V L (C) V 3L (D) V 4L

17. A particle undergoes uniform circular motion. About which point on the plane of the circle, will the angular
momentum of the particle remain conserved? (2003)
(A) centre of the circle (B) on the circumference of the circle.
(C) inside the circle (D) outside the circle.

18. A horizontal circular plate is rotating about a vertical axis passing through its centre with an angular velocity
0 . A man sitting at the centre having two blocks in his hands stretches out his hands so that the moment
of inertia of the system doubles. If the kinetic energy of the system is K initially, its final kinetic energy will be
(2004)
(A) 2 K (B) K 2 (C) K (D) K 4

19. A disc is rolling without slipping with angular velocity . P


and Q are two points equidistant from the centre C. The
order of magnitude of velocity is (2004)
C P
(A) VQ VC VP (B) VP VC VQ Q

(C) VP VC , VQ VC 2 (D) VP VC VQ

www.physicsashok.in 93
ROTATIONAL MECHANICS
20. A block of mass m is at rest under the action of
force F against a wall as shown in figure. Which
of the following statement is incorrect? (2005)
(A) = mg [where is the friction force]
(B) F = N [where N is the normal force]
(C) F will not produce torque (D) N will not produce torque

21. A particle is confined to rotate in a circular path decreasing linear speed, then which of the following is
correct? (2005)

(A) L (angular momentum) is conserved about the centre

(B) only direction of angular momentum L is conserved
(C) It spirals towards the centre
(D) Its acceleration is towards the centre.

22. Select the correct statement(s):


(A) A particle performs uniform circular motion with an angular momentum L. If the frequency of particles
motion is doubled and its KE is halved, the angular momentum becomes one-fourth.
(B) If a particle moves in X-Y plane, the resultant angular momentum about origin has only Z-component.
(C) A particle is moving along a straight line parallel to X-axis with constant velocity. Its angular momentum
about the origin decreases with time.
(D) Action of angular impulse is to change the angular momentum.

23. A ring, a cube, a cylinder, a prism and a sphere all having equal masses, equal heights and equal maximum width:
(A) The cube has the largest moment of inertia about an axis perpendicular to cross-section and passing
through the centre of mass.
(B) The ring has the largest moment of inertia about the axis mentioned above.
(C) The cylinder has the smallest moment of inertia.
(D) The sphere has the smallest moment of inertia.

24. Select the correct alternative(s):


(A) The mass of a body can be taken to be concentrated at its centre of mass for the purpose of calculating
its rotational inertia.
(B) Two circular discs of the same mass and thickness are made from metals of different densities, the one
with greater density will have less rotational inertia about its central axis.
(C) About an axis passing through diagonally opposite ends of a uniform cube, have its minimum rotational
inertia.
(D) None of these.

25. Select the correct statement(s):


(A) A very small particle rests on the top of a hemisphere of radius 20 cm. The smallest horizontal velocity
to be given to it, if it is to leave the hemisphere without sliding down its surface, is 3.2 m/s.
(B) A solid cylinder at rest at the top of an inclined plane of height 2.7m rolls down without slipping. If the
same cylinder has to slide down a frictionless inclined plane and acquires the same velocity as that
acquired by the centre of mass of the rolling cylinder at the bottom of the inclined plane, the height of
the inclined plane should be 1.8m.
(C) Three thin rods each of mass M and length L are welded so as to form an equilateral triangle. The
moment of inertia of the triangle about an axis perpendicular to the plane of triangle and passing through
3
of its vertices is ML2 .
2
(D) None of these.

26. Two boys of equal masses are sliding freely on smooth horizontal surface with the same speed v on parallel
straight paths in opposite directions. The paths are separated by a perpendicular distance d. One of the boys
carries a light pole of length d, held firmly at one end. The other boy grasps the other end of the pole just when
they are crossing each other. Which of the following statements is/are true for the subsequent motion?

www.physicsashok.in 94
ROTATIONAL MECHANICS

(A) Each boy moves in a circular path of radius d 2 with a constant speed v..
(B) The tension in the pole remains constant.
(C) They come to rest when the pole has rotated through 90 to lie along the direction of original motion.
(D) The tension in the pole varies with time.

27. A uniform rod AB of length is free to rotate about a horizontal axis A


passing through A as shown in figure. The rod is released from rest from
horizontal position. If the rod gets broken at mid-point C when it becomes
C
vertical, just after breaking of the rod:
(A) Angular velocity of upper part starts to decrease while that of
lower part remains constant. B
(B) Angular velocity of upper part starts to decrease while that of lower part starts to increase.
(C) Angular velocity of both the parts is identical.
(D) Angular velocity of lower part becomes equal to zero.
(E) lower part falls vertically.

28. If 1 is the moment of inertia of a thin rod about an axis perpendicular to its length and passing through its

centre of mass and 2 is the moment of inertia of the ring formed by bending the rod, then:
(A) 2 1 4 2 (B) 2 1 2 (C) 2 1 0.3 (D) 1 2 2 3

29. If the polar ice caps melt and spread uniformly:


(A) The moment of inertial of the earth about centroidal axis would increase.
(B) The moment of inertia of the earth about centroidal axis would decrease.
(C) Length of the day would increase.
(D) The speed of rotation would be reduced.

30. A sphere is rolling down a plane of inclination to the horizontal. The acceleration of its centre, down the
plane is:
(A) g sin (B) less than g sin (C) greater than g sin (D) greater than g sin 2

2m
2
31. The moment of inertia of the pulley system shown in figure is 4 kg m . 1m
The radii of bigger and smaller pulleys are 2m and 1m, respectively. The
angular acceleration of the pulley system is:
(A) 2.1 rad/s2 (B) 4.2 rad/s2
(C) 1.2 rad/s 2
(D) 0.6 rad/s2. 4 kg
5 kg

32. Two masses 1 kg and 2 kg are connected by an inextensible light thread passing over a pulley of mass 2 kg
and radius 20 cm. There is no slipping anywhere:
(A) Tensions at the two sides of the pulley will be different. (B) Accelerations of the two blocks will be g 4.
(C) Ratio of tensions on the two sides will be 56 (D) Ratio of tensions on the two sides will be 1 2.

33. A tube of length L is filled completely with an incompressible liquid of mass M and closed at both the ends.
The tube is then rotated in a horizontal plane about one of tis ends with a uniform angular velocity . The force
exerted by the liquid at the other end is :
(A) M 2 L 2 (B) M 2 L (C) M 2 L 4 (D) M 2 L2 2

34. Let be the moment of inertia of a uniform square plate about an axis AB that passes through its centre and
is parallel to two of its sides. CD is a line in the plane of the plate that passes through the centre of the plate
and makes an angle with AB. The moment of inertia of the plate about the axis CD is then equal to:
2
(A) (B) sin 2 (C) cos 2 (D) cos 2

www.physicsashok.in 95
ROTATIONAL MECHANICS
35. A rod of mass M kg and length L metre is bent in the form of an
equilateral triangle as shown in the figure. The moment of inertia of
triangle about a vertical axis to perpendicular to the plane of
triangle and passing through the centre (in units of kg m 2) is
O
ML2 ML2 ML2 ML2
(A) (B) (C) (D)
12 54 162 108
36. A cord is wound round the circumference of a solid cylinder of radius R
and mass M. The axis of the cylinder is horizontal. A weight mg is attached
to the end of the cord and falls from rest. After falling through a distance h,
the angular velocity of the cylinder will be

2mg 2gh 4mgh


(A) (B) (C) (D)
M 2m R M 2m R 2
37. A disc of moment of inertia is rotating about its axis, which is initially along the vertical direction, making n
revolution per minute. The axis gradually tilts becomes horizontal and the disc rotates about the horizontal
axis with the same revolution per minute. The total time, taken in doing so is t seconds then the torque acting
on the body is

2 n 2 2 n
(A) Zero (B) (C) (D) None of the above.
60 t 60 t

38. From the circular disc of radius R and mass M, a concentric circular disc of small radius r is cut and removed,
the mass of which is m. the moment of inertia of the angular disc remaining, about its axis perpendicular to
the plane and passing through the centre of mass will be
1 1
(A) M R2 r 2 (B) M m R2 r 2
2 2
1 1
(C) M m R2 r 2 (D) MR 2 mr 2
2 2

39. Three discs each of mass M and radius R are placed in contact with each
other as shown in the figure here. Then the M of the system about an XX is:

MR 2 11MR 2 2
(A) (B) (C) MR 2 (D) 7MR 2
4 4 3

40. In which case, it is easier to rotate?

(A) Case I (B) Case II (C) same (D) data is insufficient

41. A small mass m is attached to the rim of a circular disc of mass M and radius R. The disc rotates with angular
velocity about an axis passing through the centre O of the disc and perpendicular to its plane. Then the
angular momentum of the system will be

1 M 2m 2
(A) MR 2 (B) MR 2 (C) R (D) M m R
2 2

www.physicsashok.in 96
ROTATIONAL MECHANICS
42. When a force F acts on a side of the hexagonal body will
(A) topple (B) translate
(C) both (D) None

43. A particle of mass m is moving along a straight line y = x + 4 with constant velocity v, then angular momentum
of the particle about the origin is
(A) Zero (B) constant (C) decreasing continuously (D) increasing continuously

44. A cylinder can maintain its rolling motion on a


(A) rough horizontal surface (B) smooth horizontal surface
(C) rough inclined surface (D) smooth inclined surface

45. Choose the correct statement(s) for a particle moving along a circular path of constant radius
(A) centripetal acceleration is always perpendicular to velocity vector.
(B) centripetal acceleration is always perpendicular to angular velocity vector.
(C) angular acceleration vector is always perpendicular to angular velocity vector.
(D) angular velocity vector is always perpendicular to the linear velocity vector.

46. A solid sphere is executing rolling with slipping motion on a rough horizontal surface
(A) The frictional force will always perform a negative work on the sphere.
(B) The work done by friction cannot be zero.
(C) The angular momentum of the sphere is conserved about its centre of mass.
(D) The angular momentum of the sphere is conserved about any point on the horizontal surface.

47. A solid sphere of mass M and radius R is pulled horizontally on a rough surface as shown in the figure.
Choose the incorrect alternatives
(A) The acceleration of the centre of mass is F M.
F
2F CM
(B) The acceleration of the centre of mass is .
3M
(C) The frictional force on the sphere acts forward. Rough Surface
(D) The magnitude of the frictional force is F 3 .
F
48. A constant force F is applied at the top of ring as shown in figure.
Mass and the radius of the ring are M and R respectively. Angular R
M
momentum about the point of contact at time t
(A) is constant (B) increases linearly with time
(C) is 2 FRt (D) 2FRt

49. A disc of radius R rolls on a horizontal surface with linear velocity V


and angular velocity . There is a point P on circumference of disc
at angle , which has a vertical velocity. Here is equal to
V V
(A) sin 1 (B) sin 1
R 2 R
1 V 1 V
(C) cos (D) cos
R R
50. Which of the following statement(s) is/are correct for a spherical body rolling without slipping on a rough
horizontal surface at rest
(A) the acceleration of the point of contact with the ground is zero.
(B) the speed of some of the point(s) is/are zero.
(C) frictional force may or may not be zero
(D) work done by friction may or may not be zero.

www.physicsashok.in 97
ROTATIONAL MECHANICS
51. A 2kg mass attached to a string of length 1m moves in a horizontal circle as a conical pendulum. The string
makes an angle = 30 with the vertical. Select the correct alternative(s) (g = 10 m/s2).
(A) The horizontal component of angular momentum of mass about the point of support P is approximately
2.9 kg m 2/s.
(B) The vertical component of angular momentum of mass about the point of support P is approximately 1.7
kg m 2/s.
dL
(C) The magnitude of ( angular momentum of mass about point of support P) is approximately
dt L
kgm 2
10 .
s2
dL
(D) will not hold good in this case.
dt

52. A particle of mass m is travelling with a constant velocity v V0 along the line y = b, z = 0. Let dA be the area
swept out by the position vector of (from origin) particle in time dt and L is the magnitude of angular momentum
of particle about origin at any time t. Then
dA 2 L dA L
(A) L = constant (B) L constant (C) (D)
dt m dt 2m

53. A uniform rod kept on the ground falls from its vertical position. Its foot does not slip on the ground.
(A) No part of the rod can have acceleration greater than g in any position
(B) At any one position of rod, different points on it have different acceleration.
(C) Any one particular point on the rod has different acceleration at different positions of the rod.
(D) The maximum acceleration of any point on the rod, at any position is 1.5 g.

54. A thin uniform rod of mass m and length is free to rotate about its upper end. when it is at rest, it receives
an impulse J at its lowest point, normal to its length. Immediately, after impact
3J
(A) the angular momentum of rod is jl . (B) the angular velocity of rod is .
ml
3J 2 3J
(C) the kinetic energy of rod is . (D) the linear velocity of mid point of rod is .
2m 2m

55. The torque on a body about a given point is found to be equal to A L where A is a constant vector and

L is the angular momentum of the body about this point. It follows that:
dL
(A) is perpendicular to L at all instant of time.
dt

(B) the component of L in the direction of A does not change with time.

(C) the magnitude of L does not change with time.

(D) L does not change with time.

56. Two particles A and B initially at rest, move towards each other under mutual force of attraction. At the instant
when the speed of A is V and the speed of B is 2V, the speed of the centre of mass of the system is
(A) 3 V (B) V (C) 1.5 V (D) Zero (JEE, 1982)

57. When a bicycle is in motion, the force of friction exerted by the ground on the two wheels is such that it acts
(A) in the backward direction on the front wheel and in the forward direction on the rear wheel.
(B) in the forward direction on the front wheel and in the backward direction on the rear wheel.
(C) in the backward direction on both the front and the rear wheels.
(D) in the forward direction on both the front the rear wheels. (JEE, 1990)

www.physicsashok.in 98
ROTATIONAL MECHANICS
58. Two particles, each of mass m and charge q, are attached to the two ends of a light rigid rod of length 2R. The
rod is rotated at constant angular speed about a perpendicular axis passing through its centre. the ratio of the
magnitudes of the magnetic moment of the system and its angular momentum about the centre of the rod is
(JEE, 1998)
(A) q/2m (B) q/m (C) 2q/m (D) q/m

59. Let be the moment of inertia of a uniform square plate about an axis AB that passes through its centre and
is parallel to two of its sides. CD is a line in the plane of the plate that passes through the centre of the plate
and makes an angle with AB. The moment of inertia of the plate about the axis CD is then equal to
2
(A) (B) sin 2 (C) cos 2 (D) cos 2 (JEE, 1998)

60. Let I be the moment of inertia of a uniform square plate about an axis AB that passes through its centre and is
parallel to two of its sides. CD is a line in the plane of the plate that passes through the centre of the plate and
makes as angle with AB. The moment of inertia of the plate about the axis CD is then equal to [JEE, 98]
(A) I (B) I sin2 (C) I cos2 (D) I cos2 (/2)


61. The torque on a body about a given point is found to be equal to A L where A is a constant vector and L
is the angular momentum of the body about that point. From this it follows that [JEE, 98]

(A) dL / dt is perpendicular to L at all instants of time

(B) the components of L in the direction of A does not change with time

(C) the magnitude of L does not change with time (D) L does not change with time

62. A smooth sphere A is moving on a frictionless horizontal plane with angular speed and centre of mass velocity
v. It collides elastically and head on with an identical sphere B at rest. Neglect friction everywhere. After the
collision, their angularspeeds are A and B, respectively. Then [JEE, 99]
(A) A < B (B) A = B (C) A = (D) B =

63. A disc of mass M and radius R is rolling with angular speed on a horizontal as y
shown. The magnitude of angular momentum of the disc about the origin O is :
[JEE, 99]
M
(A) (1/2)MR2 (B) MR2 (C) (3/2)MR2 (D) 2MR2 O
x

64. A cubical block of side L rest on a rough horizontal surface with coefficient of friction F
. A horizontal force F is applied on the block as shown. If the coefficient of friction is
sufficiently high so that the block does not slide before toppling, the minimum force L
required to topple the block is : [JEE, (Scr) 2000]
(A) infinitesimal (B) mg/4 (C) mg/2 (D) mg(1 )

65. A thin wire of length L and uniform linear mass density is bent into a circular loop with B
x x
centre at O as shown. The moment of inertia of the loop about the axis XX is : 60
[JEE, (Scr) 2000] O
(A) L3/82 (B) L3/162 (C) 5L3/162 (D) 3L3/82

66. Two particles each of mass M are connected by a massless rod of length l. The rod is
lying on the smooth surface. If one of the particle is given an impulse MV as shown in
the figure then angular velocity of the rod would be : [JEE, (Scr) 03]
Mv M
(A) v/l (B) 2v/l (C) v/2l (D) None

www.physicsashok.in 99
ROTATIONAL MECHANICS
67. A disc has mass 9m. A hole of radius R/3 is cut from it as shown in the figure. The
moment of inertia of remaining part about an axis passing through the centre O of the R/3
2R/3

disc and perpendicular to the plane of the disc is : [JEE, (Scr) 05] Q
R
2 2
40 37
(A) 8 mR (B) 4 mR (C) mR2 (D) mR2
9 9

68. A particle moves in circular path with decreasing speed. Which of the following is correct

(A) L is constant (B) only direction of L is constant [JEE, (Scr) 05]

(C) acceleration a is towards the centre (D) it will move in a spiral and finally reach the centre

69. A solid sphere of mass M, radius R and having moment of inertia about an axis passing through the centre of
mass as I, is recast into a disc of thickkness t, whose moment of inertia about an axis passing through its edge
and perpendicular to its plane remains I. Then, radius of the disc will be [JEE, 06]
(A) 2R / 15 (B) R 2 /15 (C) 4R / 15 (D) R/4

70. A solid cylinder of mass m and radius r is rolling on a rough inclined plane of inclination . The coefficient of
friction between the cylinder and incline is . Then [JEE, 06]
(A) frictional force is always mg cos (B) friction is a dissipative force
(C) by decreasing , frictional force decreases (D) friction opposes translation and supports rotation

71. A ball moves over a fixed track as shown in the figure. From A to B the ball rolls
C
without slipping. Surface BC is frictionless. KA, KB and KC are kinetic energies A
of the ball at A, B and C, respectively. Then hA B hC

(A) hA > hC ; KB > KC (B) hA > hC ; KC > KA


(C) hA = hC ; KB = KC (D) hA < hC ; KB > KC [JEE, 06]

72. A small object of uniform density rolls up a curved surface with an initial velocity v. It
reaches up to a maximum height of 3v2/(4g) with respect to the initial position. The
object is [JEE, 07] v
(A) ring (B) solid sphere (C) hollow sphere (D) disc

FILL IN THE BLANKS


1. A uniform cube of side a and mass m rests on a rough horizontal table. A horizontal force F is applied normal
to one of the faces at a point that is directly above the centre of the face, at a height 3a 4 above the base.
The minimum value of F for which the cube begins to tip about the edge is ........... (Assume that the cube
does not slide). (1984)

2. According to Keplers second law, the radius vector to a planet from the sun sweeps out equal area in equal
intervals of time. This law is a consequence of the conservation of ................ (1985)

3. A smooth uniform rod of length L and mass M has two identical beads of negligible size, each of mass m,
which can slide freely along the rod. Initially the two beads are at the centre of the rod and the system is
rotating with an angular velocity 0 about an axis perpendicular to the rod and passing through the midpoint
of the rod (see figure). There are no external forces.

www.physicsashok.in 100
ROTATIONAL MECHANICS

When the beads reach the ends of the rod, the angular velocity of the system is .............. (1988)

4. A cylinder of mass M and radius R is resting on a horizontal platform (which is parallel to the x-y plane) with
its axis fixed along the y-axis and free to rotate about its axis. The platform is given a motion in the x-direction
given by x = A cos ( t). There is no slipping between the cylinder and platform. The maximum torque acting
on the cylinder during its motion is ................ (1988)

5. A stone of mass m, tied to the end of a string, is whirled around in a horizontal circle. (Neglect the force due
to gravity). The length of the string is reduced gradually keeping the angular momentum of the stone about the
centre of the circle constant. Then, the tension in the string is given by T = Arn where A is a constant, r is the
instantaneous radius of the circle and n = ....................... (1993)

6. The ratio of Earths orbital angular momentum (about the Sun) to its mass is 4.4 x 1015 m 2/s. The area
enclosed by Earths orbit approximately ........... m 2. (1997)

7. A uniform disc of mass m and radius R is rolling up a rough inclined plane which makes an angle of 30 with
the horizontal. If the coefficients of static and kinetic friction are each equal to and the only forces acting
are gravitational and frictional, then the magnitude of the frictional force acting on the disc is ........... and its
direction is .............. (wire up or down) the inclined plane. (1997)

8. A rod of weight w is supported by two parallel knife edges A and B and is in equilibrium in a horizontal
position. the knives are at a distance d from each other. The centre of mass of the rod is at distance x from
A. The normal reaction on A is ........... and on B is ................. (1997)
A B

9. A symmetric lamina of mass M consists of a square shape with a


semicircular section over of the edge of the square as shown in
Figure. The side of the square is 2a. The moment of inertia of the 2a
lamina about an axis through its centre of mass and perpendicular
to the plane is 1.6 Ma2. The moment of inertia of the lamina about O
the tangent AB in the plane of the lamina is .................... (1997)

TRUE & FALSE

10. A triangular plate of uniform thickness and density is made to rotate


about an axis perpendicular to the plane of the paper and (a) passing
through A, (b) passing B, by the application of the same force, F, at C
(midpoint of AB) as shown in the figure. The angular acceleration in A C B
both the cases will be the same. (1985) F

11. A thin uniform circular disc of mass M and radius R is rotating in a horizontal plane about an axis passing
through its centre and perpendicular to its plane with an angular velocity . Another disc of the same dimensions
but of mass M 4 is placed gently on the first disc coaxially. The angular velocity of the system now is
2 5 . (1986)

www.physicsashok.in 101
ROTATIONAL MECHANICS
12. A ring of mass 0.3 kg and radius 0.1 m and a solid cylinder of mass 0.4 kg and of the same radius are given
the same kinetic energy and released simultaneously on a flat horizontal surface such that they begin to roll
as soon as released towards a wall which is at the same distance from the ring and the cylinder. The rolling
friction in both cases is negligible. The cylinder will reach the wall first. (1989)

PASSAGE
PASSAGE 1
Moment of inertia is a physical term which oppose the change in rotational motion. Moment of inertia
depends on distribution of mass, shape of the body as well as distance from the rotational axis. Moment of
linear momentum is called angular momentum. If no external torque act on the system then angular momentum
of the system remains conserved. Geometrical meaning of angular momentum relates to the areal velocity.

1. Mass M is distributed over the rod of length L. If linear mass


density ( ) linearly increases with length as = Kx. The M. .
of the rod about one end perpendicular to rod i.e. (YY)

ML2 ML2 2 KL4


(A) (B) (C) ML2 (D)
3 12 3 4
Y
2. Four holes of radius R are cut from a thin square plate of side 4R and
mass M. The moment of inertia of the remaining portion about z-axis.

4 X
(A) MR 2 (B) MR 2
12 3 4
4 8 10
(C) MR 2 (D) MR
2

3 6 3 6

3. A particle of mass m is moving along the line y = 3x + 5 with speed V. The magnitude of angular momentum
about origin is

5 5 1 1
(A) mV (B) mV (C) mV (D) mV
12 2 2 3

4. A hollow sphere is rolling without slipping total energy during rolling is 60 J, then
(A) translational K.E. = 36 J (B) rotational K.E. = 36 J
(C) translational K.E. = 24 J (D) rotational K.E. = 24 J

5. Acceleration of block of mass m 1 is (given moment of inertia


of pulley is and string does not slip over the pulley).

m1 m2 g
m1 m2
(A) g (B) m m
m1 m2 1

2 2
R


m1 m2 2 g
m1 m2 2 g R
(C) R (D)
m1 m2 m1 m2 2
R

www.physicsashok.in 102
ROTATIONAL MECHANICS
PASSAGE 2
Angular velocity is defined as the rate of change of angular displacement.
The angular velocity is a measure of the degree of rotation of body. For a rigid body is constant. All angular
variable (such as angular displacement, angular velocity and angular acceleration) are directed along the axis
of rotation and perpendicular to plane. Rotating rigid object has two acceleration, one centripetal and other
tangential both accelerations being normal to each other.

6. A disc of radius R rolls on a horizontal ground with linear acceleration


a and angular acceleration as shown in figure.
The magnitude of acceleration of point P at an instant, when its linear
velocity is V and angular velocity is , will be:
2 2 ar
(A) a ra r 2 (B) (C) r 2 a 2 r 2 4 (D) r
R
7. A disc is rotating clockwise at 20 radian/sec. Its centre has velocity 30 m/s in the forward direction. It has
radius of 20 m. Then
(A) velocity of topmost point is 430 m/s in forward direction.
(B) velocity of lowermost point is 370 m/s in backward direction.
(C) velocity of topmost point is 400 m/s (forward).
(D) velocity of lower most point is 50 m/s (forward).

8. The topmost and bottom most points have velocities V1 and V2 in the same direction. The radius of sphere is
R. Then the correct option are
V1 V2
(A) Angular velocity of sphere about cm is clockwise
2R
3V1 V2
(B) The Linear velocity at point P is VP .
4
3 3
(C) The Linear velocity at point Q is V1 V2 1
2 2 .
V1 V2
(D) Velocity of centre of mass is .
2
9. Three identical rods, each of mass m and length are joined to form
a rigid equilateral triangle. Its radius of gyration about an axis passing
through a corner and perpendicular to the plane of the triangle is

l 3 l 60
(A) (B) l (C) (D) l 3
2 2 2

10. A Disc of mass M and radius R is rolling with angular speed on Y


a horizontal plane as shown in figure,
The magnitude of angular momentum of the disc about O is
M V
MR 2 3 2
(A) (B) MR 2 (C) MR (D) 2MR 2 O X
2 2

PASSAGE 3
In figure 1, the winch is mounted n an axle, and the 6-sided nut is welded ot the winch. By turning the nut
with a wrench, a person can rotate the winch. For instance, turning the nut clockwise lifts the block off the
ground, because more and more rope gets wrapped around the winch.
Three students agree that using a longer wrench makes it easier to turn the winch. But they disagree
about why. All three students are talking about the case where the winch is used, over a 10-second time
interval, to lift the block one meter off the ground.

www.physicsashok.in 103
ROTATIONAL MECHANICS
Student : 1
By using a longer wrench, the person decreases the average force he must exert on the wrench, in order to
lift the block one meter in ten seconds.
nut

winch person grips


wrench here
block

Figure : 1 Figure : 2
Student : 2
Using a longer wrench reduces the work done by the person as he uses the winch to lift the block one
meter in ten second.
Student : 3
Using a longer wrench reduces the power that the person must exert to lift the block one meter in ten
seconds.
11. Student 1 is:
(A) Correct, because the torque that the wrench must exert to lift the block doesnt depend on the wrenchs
length
(B) Correct, because using a longer wrench decreases the torque it must exert on the winch.
(C) Incorrect, because the torque that the wrench must exert to lift the block doesnt depend on the wrenchs
length.
(D) Incorrect, because using a longer wrench decreases the torque it must exert on the wrench.

12. Which of the following is true about student 2 and 3?


(A) Students 2 and 3 are both correct.
(B) Student 2 is correct, but student 3 is incorrect.
(C) Student 3 is correct, but student 2 is incorrect.
(D) Students 2 and 3 are both incorrect.

13. If several wrenches all apply the same torque to a nut, which graph best expresses the relationship between
the force the person must apply to the wrench, and the length of the wrench?
force

force

force

force

(A) (B) (C) (D&&)

length length length length

PASSAGE 4
When a force Of is applied on a block of mass m resting on a horizontal surface then there are two
possibilities, either the block moves by translation or it moves by toppling. If the surface is smooth then the
block always translates but on a rough surface it topples only when the torque of the applied force Of is
greater than the torque of mg about a point in contact with the ground.
When the force Of is applied the body may topple about A or it may translate.

F
m h

a A

Answer the following question.

www.physicsashok.in 104
ROTATIONAL MECHANICS
14. When the block topples about A, the normal force
(A) passes through centre of mass (B) is / zero
(C) shifts to the right and passes through right most edge containing A.
(D) is zero if the surface is smooth.

15. The block will move in pure translation if


mga mga mga
(A) F (B) F (C) F (D) None
h 2h 2h

16. The block will topple about A if


mga mga mga
(A) F (B) F (C) F (D) None
h 2h 2h

17. If the block be a cube of edge a and = 0.2 then


(A) The body will translate (B) The body will topple
(C) The body may translate or topple (D) None

18. If the block is a cube of edge a and = 0.6 then


(A) The body will translate (B) The body will topple
(C) The body first translates and then topples (D) None

PASSAGE 5
Two discs A and B are mounted coaxially on a vertical axle. The discs have moments of inertia I and 2I
respectively about the common axis. Disc A is imparated an initial angular velocity 2 using the entire potential
energy of a spring compressed by a distance x1. Disc B is imparted an angular velocity by a spring having the
same spring constant and compressed by a distance x2. Both the discs rotate in the clockwise direction.
19. The ratio x1/x2 is [JEE, 07]
(A) 2 (B) 1/2 (C) 2 (D) 1/ 2

20. When disc B is brought in contact with disc A, they acquire a common angular velocity in time t. The average
frictional torque on one disc by the other during this period is [JEE, 07]
(A) 2I/(3t) (B) 9I/(2t) (C) 9I/(4t) (D) 3I/(2t)

21. The loss of kinetic energy during the above process is [JEE, 07]
(A) I2/2 (B) I2/3 (C) I2/4 (D) I2/6

LEVEL 2
1. A uniform solid sphere of mass 1 kg and radius 10 cm is kept stationary on a rough inclined plane by fixing
a highly dense particle at B. Inclination of plane is 37 with horizontal and AB is the diameter of the sphere
which is parallel to the plane, as shown in figure. Calculate:

www.physicsashok.in 105
ROTATIONAL MECHANICS
(i) mass of the particle fixed at B, and
(ii) minimum required coefficient of friction between sphere and plane to keep sphere in equilibrium.

2. A ball of radius R = 20 cm has mass m = 0.75 kg and moment of inertia (about its diameter) 0.0125 kg m 2 .
The ball rolls without sliding over a rough horizontal floor with velocity v 0 = 10 ms1 towards a smooth vertical
wall. If coefficient of restitution between the wall and the ball is e = 0.7, calculate velocity v of the ball long
after the collision. (g = 10 ms2).

3. AB is a horizontal diameter of a ball of mass m = 0.4 kg and radius R = 0.10 m.


At time t = 0, a sharp impulse is applied at B at angle of 45 with the horizontal,
as shown in Figure so that the ball immediately starts to moves with velocity
v 0 = 10 ms1.
(i) Calculate the impulse
If coefficient of kinetic friction between the floor and the ball is = 0.1, calculate
(ii) velocity of ball when it stops sliding.
(iii) time t at that instant,
(iv) horizontal distance ravelled by the ball upto that instant.
(v) angular displacement of the ball about horizontal diameter perpendicular to AB, upto that instant, and
(vi) energy lost due to friction.

4. A solid ball of diameter d = 11 cm is rotating about its one of the horizontal diameters with angular velocity
0 120 rad sec . It is released from a height so that it falls h = 1.8 m freely and then collides with the
horizontal floor. Co-efficient restitution is e 5 6 and co-efficient of friction between the ball and the ground
is 0.2 . Calculate fraction of energy lost during collision and the distance between the points where the
ball strikes the floor for the first and second time.

5. A heavy plank of mass 102.5 kg is placed over two cylindrical rollers of radii R = 10 cm and r = 5 cm. Mass
of rollers is 40 kg and 20 kg respectively. Plank is pulled towards right by applying a horizontal force Of = 25
N as shown in Figure. During first second of motion the plank gets displaced by 10 cm.

If plank remains horizontal and slipping doesnot take place, calculate magnitude and direction of force of
friction acting between
(i) plank and bigger roller. (ii) plank and smaller roller.
(iii) bigger roller and floor, and (iv) smaller roller and floor.

6. A wheel of radius R = 10 cm and moment of inertia I = 0.05 kg-m 2 is rotating about a fixed horizontal axis O
with angular velocity 0 = 10 rad/sec. A uniform rigid rod of mass m = 3 kg and length l = 50 cm is hinged
at one end A such that it can rotate about end A in a vertical plane. End B of the rod is tied with a thread as
shown in figure such that the rod is horizontal and is just in contact with the surface of rotating wheel.
Horizontal distance between axis of rotation O if cylinder and A is equal to a = 30 cm.

If the wheel stops rotating after one second after the thread has burnt, calculate co-efficient of friction
between the rod and the surface of the wheel. (g = 10 ms2)

www.physicsashok.in 106
ROTATIONAL MECHANICS
7. Two heavy and light cylindrical rollers of diameters D and d respectively rest on a P
horizontal plane as shown in figure. D
The larger roller has a string wound round it to which a horizontal force P can be d
applied as shown. Assuming that the coefficient of friction ahs the same value
for all surfaces of contact, determine the limits of so that the larger roller can be pulled over the smaller
one.

8. A block of mass m height 2h and width 2b rests on a flat car which moves horizontally with constant
acceleration a as shown in figure.

Determine:
(a) the value of the acceleration at which slipping of the block on the car starts, if the coefficient of friction
is .
(b) the value of the acceleration at which block topples about A, assuming sufficient friction to prevent
slipping and
(c) the shortest distance in which it can be stopped from a speed of 72 km/hr with constant deceleration so
that the block is not disturbed.
The following data are given: b = 0.6 m, h = 0.9 m; = 0.5 and g = 9.8 m/s2.

9. A block of and M = 4 kg of height h and breath b is placed on a rough plank of same mass M. A light
inextensible string is connected to the upper end of the block and passed through a light smooth pulley as
shown in the figure.

A mass m = 1 kg is hung to the other end of the string.


(a) What should be the minimum value of coefficient of friction between the block and the plank so hat,
there is no slipping between the block and the wedge?
(b) Find the minimum value of b/h so that the block does not topple over the plank, friction is absent
between the plank and the ground.

10. A homogeneous cylinder and a homogeneous sphere of equal mass m = 20 kg and equal radii R are connected
together by a light frame and are free to roll without slipping down the plane inclined at 30 with the horizontal.
Determine the force in the frame.

Assume that the bearings are frictionless. Take g = 10 m/s2)

www.physicsashok.in 107
ROTATIONAL MECHANICS
11. A uniform ring of mass m, radius a and centre C lies at rest on a smooth horizontal table. The plane of the ring
is horizontal. A point P on the circumference is struck horizontally and it begins to move in a direction at 60
to PC. If the magnitude of impulse is mv 7 , find the initial speed of point P..

12. A solid cube of wood of side 2a and mass M is resting on a horizontal surface.
The cube is constrained to rotate about an axis passing through D and perpen-
dicular to face ABCD. A bullet of mass m and speed v is shot at a height of
4a
as shown in figure.
3
The bullet becomes embedded in the cube. Find the minimum value of v r
equired to topple the cube. Assume m << M.

13. A semicircular ring of mass m and radius r is released from rest in the position shown with its lower edge
resting on a horizontal surface.

Find the minimum coefficient of static friction s . Which is necessary to prevent any initial slipping of the
ring.
2r
[Hint: centre of mass of a semicircular ring lies at a distance of from centre]

0
14. A spherical ball of radius r and mass collides with a plank of mass M kept on
a smooth horizontal surface. Before impact, the centre of the ball has a velocity
v 0 and angular velocity 0 as shown. the normal velocity is reversed with same v0
magnitude and the ball stops rotating after the impact.
Find the distance on the plank between first two impacts of the ball. The
coefficient of friction between the ball and the plank is . Assume that plank is large enough.
m
15. A disc of mass M is lying on a frictionless horizontal surface. A small particle of
mass m strikes the edge of the disc with a velocity v 0 in a tangential direction of
disc and gets embedded in it, as shown in the figure. v0
M
Find the angular velocity of the disc just after the collision?

16. A solid hemisphere of mass M = 8 kg and radius R = 5m is supported at the m


ends of the diameter of the circular cross-section. the supports are frictionless.
A particle of mass m = 1 kg is dropped freely from a height h and strikes the end
M
of the diameter of the circular section perpendicular to the diameter passing
thought he supports and sticks, as shown in the figure.
The hemisphere just reaches a position where the flat face of it is vertical. Find h. Also calculate the reactive
impulse J of the support during collision.

www.physicsashok.in 108
ROTATIONAL MECHANICS

LEVEL 3
1. A block of mass M with a semicircular of radius R, rests on a horizontal
frictionless surface. A uniform cylinder of radius r and mass m is released
from rest at the top point A (see Figure). the cylinder slips on the
semicircular frictionless track. How far has the block moved when the
cylinder reaches the bottom (point B) of the track?
How fast is the block moving when the cylinder reaches the bottom of the track? (JEE, 1983)

2. A homogeneous rod AB of length L = 1.8 m and mass M is pivoted at the centre O in such a way that it can
rotate freely in the vertical plane (Figure). The rod is initially in the horizontal position. An insect S of the
same mass M falls vertically with speed V on the point C, midway between the points O and B. Immediately
after falling, the insect moves towards the end B such that the rod rotates with a constant angular velocity .

(A) Determine the angular velocity in terms of V and L. (JEE, 1992)


(B) If the insect reaches the end B when the rod has turned through an angle of 90, determine v.

3. A uniform thin rod of mass M and length L is standing vertically along the y-axis on a smooth horizontal
surface, with its lower end at the origin (0, 0). A slight disturbance at t = 0 causes the lower end to slip on the
smooth surface along the positive x-axis, and the rod starts falling. (JEE, 1993)
(A) What is the path followed by the centre of mass of the rod during its fall?
(B) Find the equation to the trajectory of a point on the rod located at a distance r from the lower end. What
is the shape of the path of this point?

4. A block X of mass 0.5 kg is held by a long massless string on a frictionless inclined


plane of inclination 30 to the horizontal. The string is wound on a uniform solid
cylindrical drum Y of mass 2 kg and of radius 0.2 m as shown in Figure. The drum is
given an initial angular velocity such that the block X starts moving up the plane.
(A) Find the tension in the string during the motion. (JEE, 1994)
(B) At a certain instant of time the magnitude of the angular velocity of Y is 10 rad s1
calculate the distance travelled by X from that instant of time until it comes to rest.
P
5. Two uniform thin rods A and B of length 0.6 m each and of masses 0.01 kg
and 0.02 kg respectively are rigidly joined end to end. The combination is A
pivoted at the lighter end, P as shown in figure. Such that it can freely rotate
about point P in a vertical plane. A small object of mass 0.05 kg, moving
horizontally, hits the lower end of the combination and sticks to it. (JEE, 1994) B
What should be the velocity of the object so that the system could
just be raised to the horizontal position.

6. A wedge of mass m and triangular cross-section (AB = BC = CA = 2R) is moving with a constant velocity vi
towards a sphere of radius R fixed on a smooth horizontal table as shown in Figure. The wedge makes an
elastic collision with the fixed sphere and returns along the same path without any rotation. Neglect all
friction and suppose that the wedge remains in contact with the sphere for a very short time. t , during
which the sphere exerts a constant force F on the wedge. (JEE, 1998)

www.physicsashok.in 109
ROTATIONAL MECHANICS

(A) Find the force F and also the normal force N exerted by the table on the wedge during the time t .
(B) Let h denote the perpendicular distance between the centre of mass of the wedge and the line of action
of F. Find the magnitude of the torque due to the normal force N about the centre of the wedge, during
the interval t .

7. A uniform circular disc has radius R and mass m. A particle also of mass
m, is fixed at a point A on the edge of the disc as shown in Figure. The
disc can rotate freely about a fixed horizontal chord PQ that is at a
distance R 4 from the centre C of the disc. The line AC is perpendicular
to PQ. Initially, the disc is held vertical with the point A at its highest
position. It is then allowed to fall so that it starts rotating about PQ.
Find the linear speed of the particle as it reaches its lowest position.
(JEE, 1998)

8. A man pushes a cylinder of mass m 1 with the help of a plank of F m2


mass m 2 as shown in Figure. There in no slipping at any contact.
The horizontal component of the force applied by the man is F.
Find M1
(A) the accelerations of the plank and the center of mass
of the cylinder, and
(B) the magnitudes and directions of frictional forces at contact points. (JEE, 1999)

9. A small ball of mass 2 x 103 kg having a charge of 1 C is suspended by a string of length 0.8 m. Another
identical ball having the same charge is kept at the point of suspension. Determine the minimum horizontal
velocity which should be imparted to the lower ball so that it can make complete revolution. (JEE, 2001)

10. Three particles A, B and C, each of mass m, are connected to each other by
three massless rigid rods to form a rigid, equilateral triangular body of side
. This body is placed on a horizontal frictionless table (x-y plane) and is
hinged to it at the point A so that it can move without friction about the
vertical axis through A (see figure).
The body is set into rotational motion on the table about A with a constant
angular velocity . (JEE, 2002)
(A) Find the magnitude of the horizontal force exerted by the hinge on the body.
(B) At time T, when the side BC is parallel to the x-axis, a force F is applied on B along BC (as shown).
Obtain the x-component and the y-component of the force exerted by the hinge on the body, immediately
after time T.

11. A wooden log of mass M and length L is hinged by a frictionless


nail at O. A bullet of mass m strikes with velocity v and sticks
to it. Find angular velocity of the system immediately after the
collision about O. (JEE, 2005)

12. A cylinder of mass m and radius R rolls down an inclined plane of inclination . Calculate the linear acceleration
of the axis of cylinder. (JEE, 2005)

www.physicsashok.in 110
ROTATIONAL MECHANICS

13. A uniform circular disc has radius R and mass m. A particle also of mass m is fixed at A
a point A on the wedge of the disc as in fig. The disc can rotate freely about a fixed
horizontal chord PQ that is at a distance R/4 from the centre C of the disc. The line AC
C
is perpendicular to PQ. Initially the disc is held vertical with the point A at its highest R/4
position. It is then allowed to fall so that it starts rotating about PQ. Find the linear P Q
speed of the particle at it reaches its lowest position. [JEE, 98]

14. A man pushes a cylinder of mass m1 with the help of a plank of mass m2 as shown. F
There is no slipping at any contact. The horizontal component of the force applied m2
m1
by the man is F. Find [JEE, 99]
(a) the accelerations of the plank and the centre of mass of the cylinder, and
(b) the magnitudes and directions of frictional forces at contact points.

15. A rod AB of mass M and length L is lying on a horizontal frictionless surface. A particle of mass m travelling
along the surface hits the end A of the rod with a velocity v0 in the direction perpendicular to AB. The collision
is completely elastic. After the collision the particle comes to rest.
(a) Find the ratio m/M.
(b) A point P on the rod is at rest immediately after the collision. Find the distance AP.
(c) Find the linear speed of the point P at a time L/(3v0) after the collision. [JEE, 2000]

16. Two heavy metallic plates are joined together at 90 to each other. A laminar sheet of mass 30 Kg is hinged at
the line AB joining the two heavy metallic plates. The hinges are frictionless. The moment of inertia of the
laminar sheet about an axis parallel to AB and passing through its centre of mas is 1.2 Kgm2. Two rubber
obstacles P and Q are fixed, one on each metallic plate at a distance 0.5 m from the line AB. This distance is
chosen so that the reaction due to the hinges on the laminar velocity 1 rad/s and turns back. If the impulse on the
sheet due to each obstacle is 6 Ns. [JEE, 01] A
(a) Find the location of the centre of mass of the laminar sheet from AB.
Q
(b) At what angular velocity does the liminar sheet come back after the first
impact ? P
B
(c) After how many impacts, does the laminar sheet come to rest ?
P
17. Two identical ladders, each of mass M and length L are resting on the rough horizontal
surface as shown in the figure. A block of mass m hangs from P. If the system is in v m
equilibrium, find the magnitude and the direction of frictional force at A and B.
[JEE, 2005] A B

18. There is a rectangular plate of mass M kg of dimensions (a b). The plate is held
b
in horizontal position by striking n small balls each of mass m per unit area per unit
time. These are striking in the shaded half region of the velocity v. What is v ? a
[JEE, 06]
It is given n = 100, M = 3 kg, m = 0.01 kg; b = 2 m; a = 1m; g = 10 m/s2.

www.physicsashok.in 111
ROTATIONAL MECHANICS

ASSERTION-REASON
Q. 1 2 3 4 5 6
Ans. A C A D B D

MATCH THE COLUMN

13. A Q, B P, C S, D T
14. A P, B P, C P,, D P
15. A R, B P, C S, D Q
16. A S, B R, C Q, D P
17. A Q, B P, C S, D R

LEVEL 1
Q. 1 2 3 4 5 6 7 8 9 10
Ans. C B A B B B C A C C
Q. 11 12 13 14 15 16 17 18 19 20
Ans. A B A B B A A B C D
Q. 21 22 23 24 25 26 27 28 29 30
Ans. B ABD BD BC BC AB AC D ACD BD
Q. 31 32 33 34 35 36 37 38 39 40
Ans. A ABC A A B C C D B B
Q. 41 42 43 44 45 46 47 48 49 50
Ans. C C B ABC ABD BD ABCD BD CD BC
Q. 51 52 53 54 55 56 57 58 59 60
Ans. ABC AD BCD ABCD ABC D C A A A
Q. 61 62 63 64 65 66 67 68 69 70
Ans. ABC C C C D A B B A CD
Q. 71 72
Ans. AB D

FILL IN THE BLANKS AND TRUE-FALSE


2 M 0 5
1. mg 2. Angular momentum 3. 4. MRA 2
3 M 6m 3
mg dx xW
5. 3 6. 6.94 x 1022 7. , direction upward 8. W ,
6 d d

9. 4.8 Ma2 10. F 11. F 12. F

www.physicsashok.in 112
ROTATIONAL MECHANICS

PASSAGE

Q. 1 2 3 4 5 6 7 8 9
Ans. D D A AD B A AB ABCD C
Q. 10 11 12 13 14 15 16 17 18
Ans. C A D D C B C A B
Q. 19 20 21
Ans. C A B

LEVEL 2
1. (i) 3 kg (ii) 0.75 2. 2 ms 1

3. (i) 4 2kg ms 1 (ii) Zero (iii) 10 second (iv) 50 (Leftward) (v) 1250 radians (clockwise) (vi) 70 joule

d
4. 0.432, 2.2 m 5. (i) 3N (ii) 1.50 N (iii) 1.00 N (iv) 0.50 N 6. 0.2 7.
D
1 4
8. (a) 4.9 m/s2 (b) 6.53 m/s2 (c) 40.82 m 9. (a) (b) 10. 3.45 N 11. 4 v
9 9
M 12 4 M m v0 r 0 2mv0
12.
m
2ag 2 1
13. 0.398 14.
5 M g
15.
M 3m
16. h = 42 m, J = 30 kg m/s.

LEVEL 3
mR r 2g R r 12V L x y
2 2

1. ,m 2. (a) (b) 3.5 ms1 3. (i) Circular path of radius (ii) 1


M m M m M 7L 2 LR r

2mv 2mv 4mv


4. (i) 1.63 N, (ii) 1.22 m 5. 6.3 m/s 6. (a)
3t

3i k ,
3t
mg k , (b)
3t
h

3FM 1 FM 1
7. 5gR 8. (a) R , 2 R (b) , 9. 5.86 m/s
3M 1 8m2 3M 1 8m2
F 3mv
10. (a) 3m 2 (b) Fnet x
4
, Fnet y 3m
2
11.
3m M L
2g sin
12. a axies 13. v 5gR
3
4F 8F 3m1F m1F
14. a c 3m 8m , a p 3m 8m , f1 3m 8m , f 2 3m 8m
1 2 1 2 1 2 1 2

m 1 2L v0
15. (a) ; (b) x ; (c)
M 4 3 2 2
16. (a) l = 0.1 m; (b) = 1 rad/s ; (c) laminar sheet will never come to rest
cot
17. f M m g 18. 10 m/s
2
X X X

www.physicsashok.in 113
S.H.M

DHANALAKSHMI NAGAR

NEAR ANNAMAIAH CIRCLE,

TIRUPATI.

PH NO. 9440025125
SIMPLE HARMONIC MOTION

PERIODIC MOTION
If a particle moves along X-axis, its position depends upon time t. We express this fact mathematically by
writing
x = f(t) or x(t)
There are certain motions that are repeated at equal interval of time. By this we mean that particle is found at
the same position moving in the same direction with the same velocity and acceleration, after each period of
time. Let T be the interval of time in which motion is repeated. Then
x(t) = x(t + T)
The T is the minimum change it t. The function that repeats itself is known as a periodic function. During the
period, its values may remain finite. Such functions are bound functions. Periodic motion of a particle is also
bound functions. Periodic motion of a particle is also bound because it must not go to infinity and retun back
in one finite period. (That will mean infinite average speed).
Periodic motions may be oscillatory or non oscillatory. Uniform circular motion, the motion of a planet
around the sun, etc are periodic but not oscillatory. Also, an oscillatory motion may not repeat its position with
the old velocity due to friction and will be non-periodic.

C1 : The position x of a particle is plotted with time t as in the figure. Is the motion periodic ?
x

t
O A B C

Sol. Yes. The portion OA is repeated as AB, then as BC and so on. Hence the motion is periodic.

C2 : Is the following motion periodic ? Is it oscillatory ?


x

t
O
A B

Sol. Yes. The portion OA is repeated as AB, and so on. The motion is oscillatory also because the velocity changes
its sign (direction) periodically.

Period :
The minimum time interval is which a particle repeats its motion is called the time period of the motion.
The expression of period depends upon the type of motion.

C3 : A particle moves uniformly in a circle. Determine the time period of the motion.
Sol. Let V be the speed and r be the radius of the circle. Then time taken to complete
one revolution is the time period, T. Hence
r
2r
T V
V
If be the angular speed of the particle, its radial line turns by 2 radian in one time period.
2
Hence T .

www.physicsashok.in 1
SIMPLE HARMONIC MOTION

C4 : A ball is dropped from a height h and falls under uniform acceleration g. It hits the ground elastically. Its velocity
is reversed due to collision; the ball reaches the same starting position. Calculate the periodic time.
Sol. Here time t of descent is
h
2h
h = 1/2gt2 or t=
g
This is also the time of ascend. Hence total time, which equals the periodic time T of this motion, is 2t. Thus

2h
T 2t 2
g
C5 : The position x of a particle depends upon as x = 4 (mm) sin4(s1)t. Determine its time period.
Sol. We know that
sin = sin ( + 2)
Hence
sin 4t = sin (4t + 2)
We can express RHS in the same form as LHS as
2
sin 4t = sin4 t
4
Comparing this with
x(t) = x(t + T)
2
We see that T
4
2
Thus the period of the given motion is s.
4
2
Note : You may learn that period of sint or cos t is .

C6 : Determine the time period of x(t) motion.
x(m)

t (s)
O 2 4 6 8 10
Sol. Here the repetition occurs at every 4s of interval. Hence T = 4 s.

Frequency :
Any periodic motion that repeats itself at regular time interval and continues for long time is assigned a frequency
of repetition of motion. We say that one repetition means occurrence of motion once. The number of times the
motion is repeated per unit time is known as the frequency of the motion.
If T be the periodic time of a motion, the number of repetitions in unit time is 1/T. Hence frequency = 1/T.
Frequency is denoted by n, f or v. Then
v = 1/T
The S.I. unit of frequency is Hertz, denoted by Hz.
1Hz = 1 cycle per second (1 cps)

www.physicsashok.in 2
SIMPLE HARMONIC MOTION

Frequency is a feature of any periodic motion, not only of oscillations. Frequency is also called linear frequency.
Angular frequency is defined as 2 time the linear frequency. It is denoted by (unit : s1) :
= 2 v
(Note that here is not angular velocity)

C7 : A ball is dropped from height h on an elastic floor. Determine the frequency of the periodic motion executed by
the ball.
Sol. We have seen that periodic time of this motion is
2h
T=2
g
Hence frequency is given by
1 1 g
v= = .
T 2 2h

Displacement as a function of time :


Let the initial position of a particle be x0 and position at time t be x. Then its displacement relative to x0 is
x x0. This depends in some manner on time t. we say that displacement varies with time, or is a function of
time t. We denote the function by f(t). Thus
x x0 = f(t)
Here the expression for function depends upon type of motion.
If the motion is with uniform velocity, we have
x x0 = Vt (linear function)
Here we say that displacement x x0 is proportional to t. Position x = x0 + Vt is said to be a linear function of
t, as power of t is 1. The graph of x(t) is a straight line. If the motion is along x-axis with uniform acceleration
the displacement is given by
x x0 = ut + 1/2at2
Here we have quadratic function of t, as the highest power of t is 2. The graph of x(t) is a parabola.
If acceleration is not uniform, we have infinite number of ways in which acceleration can change. Of these, one
special case is very important-arising from a periodic function of time.

Periodic Function :
A function that repeats itself at equal interval is called a periodic function.
There are several periodic functions. All of them can be symbolically expressed as x(t) = x(t + T)
where the minimum value of T is known as t-period of the function x.

C8 : What is the t-period of


x = A sin (t + )
Sol. It is known from trigonometry that
sin = sin( + 2)

www.physicsashok.in 3
SIMPLE HARMONIC MOTION

Hence
2
x = A sin(t + + 2) A sin t

2
x = A sin (t + ), where t = t + .

This shows that the function at t coincides with the function at t. The interval t t is the period in t.
2
This period is .

C9 : What is the time period of
x = A cos (t + ) ?
Sol. From trigonometry, it is known that
cos = cos ( + 2)
Here x = A cos (t + + 2)
2
A cos t = A cos (t + ),

where t = t + 2/.
Thus, the function at t coincides with that at t. The period is, thus, 2/.
C10 : Determine the period T of the periodic function x = A sin2t.
Sol. Using cos2= 1 2 sin2 ,
1 cos 2
sin2 =
2
The period of cos 2is 2/2 i.e. .
Hence sin2t has a similar treatment.
T = /

NOTE :
1. Period is not changed by multiplying or dividing by (or by adding) a constant :
x(t) x 0
If x(t) x(t T) has the same period T..
m
2. Period is reduced times of t be multiplied by :
T
If x(t) = x(t + T) then x(t) will have a period .

3. According to Fourier, the sum of periodic functions is also periodic.

OSCILLATION
If you displace a particle from its equilibrium position in such a way that its potential energy is increased, a force
arises which tries to bring the particle to the position of lowest potential energy. If you release it, this force will
bring it to the equilibrium position. But it will not stop ! Why? It has gained velocity and due to inertia of motion,
it will go on the other side. The restoring force will again be bringing to back to the equilibrium position. Inertia
of motion will again be bringing it back to the equilibrium position. Inertia of motion will again shift it to the other
side. This to and fro motion will continue until the energy is lost in friction.
www.physicsashok.in 4
SIMPLE HARMONIC MOTION

To and fro motion is called oscillation. It may be periodic or non-periodic. If an angle is used to describe
oscillation, the oscillation is called angular. If linear quantities are used to describe oscillation, it is called linear
oscillation.

C11 : A block suspended from a spring performs oscillation.

C12 : A block moving to and fro along smooth surface between two strings.

x
C13 : A function f(t) is said to be periodic of time period T if
f(t + T) = f(t)
All sine or cosine functions of time are periodic. Thus,
2
Y = A sin t or A cos t is periodic, of time period T = .

C14. Find the period of the function,
y = sin t + sin 2t + sin 3t
Sol. The given function can be written as,
y = y1 + y2 + y3
Here y1 = sin t, T1 = 2/
2
y2 = sin 2t, T2 =
2
and y3 = sin 3t, T3 = 2/3
T1 = 2T2 and T1 = 3T3
So, the time period of the given function is T1 or 2/
Because in time T = 2/, first function completes one oscillation, the second function two oscillations and the
third, three.
C15: What is the period of motion shown by position-time graph ?

x
1 3 5
t
O
2 4 6

Sol. The motion is periodic, with period of 2s.


C16: (a) Is the motion x = sin t cos t periodic? simple harmonic ?
(b) What type of motion is represented by x = sin3t periodic, non-periodic or simple harmonic ?

www.physicsashok.in 5
SIMPLE HARMONIC MOTION

Sol. (a) We can always express


x = a sin t + b cos t
in the form
x = c sin (t + )
where a = c cos , b = c sin .
Hence the motion is simple harmonic and periodic.
(b) sin 3 = sin(2 + )
= sin2 cos + cos 2 sin
= 2sin cos2+ (cos2 sin2)sin
= 2sin(1 sin2) + (1 sin2) sin
= 3 sin 4 sin3
1 3
sin 3 sin 3 sin
4 4
1 3
x sin 3t sin t
4 4
This is a periodic function.
dx 1 3
v cos 3t cos t
dt 4 4
dv 9 2 3
a sin 3t 2 sin t
dt 4 4
2x Thus, the motion is not SHM.

SIMPLE HARMONIC MOTION :


If the position of a particle changes, we say that it is moving. then x = f(t). if the position changes periodically
are call the motion harmonic or periodic. In studying vibration of harmonium blades (musical instrument),
sine and cosine functions were used. We call sine and cosine functions a harmonic function. The most simple
function is linear, like sin , cos or asin + bcos .
If the displacement of a particle from a fixed point be expressed using simple form of harmonic functions, we
have
x x0 = a sin t + b cos t
By using trigonometry we can write this either as x x0 = Asin(t + ) or as Acos (t + ).
Hence the motion represented using simplest harmonic functions may be expressed as
x x0 = Asin(t + )
Here x0 is a fixed co-ordinate and may be set equal to zero. If a particle moves in such a way that its displacement
relative to a fixed point is given by
x = A sin(t + )
the motion is called simple harmonic
The velocity of the particle is given by
dx
v= = A cos(t + )
dt

www.physicsashok.in 6
SIMPLE HARMONIC MOTION

The rate of change of velocity is acceleration a.


dV
Thus a= = 2 A sin (t + ) = 2x
dt
The negative sign shows that acceleration is directed opposite to
displacement. If a particle moves in such a way that its acceleration is always
Fixed point Particle
directed towards a fixed point and is directly proportional to the magnitude a
of displacement relative to the fixed point, the motion is called simple X
O P
harmonic.
OP
In the figure a particle P is moving along X-axis. There is a fixed point O from which the displacement is

measured. OP is the displacement. If a be the acceleration of the particle then, for SHM,

(i) | a | OP

(ii) | a | OP

a = (positive const.) OP

Equation of SHM :
Let x = 0 lies at O and P be at x. Then

OP (x 0) i x i

a a x i
a x i 2 x i
where, 2 = positive constant for real .
ax = 2x (component form)
If O lies at x0, then
ax = 2(x x0)
This is the kinematical form of equation of SHM.
The above formulation is valid in all frames of reference. However, its dynamical form using force depends on
reference frame.

Dynamical Form : Multiplying ax = 2x by the mass m of the particle,


max = m2x
If the frame be inertial, Newtons second law of motion is valid. Then max = Fx
Then Fx = m2x = kx
where, k = m2
Thus, SHM in an inertial frame is the motion in which the force acting on the particle is directed opposite to the
displacement from a fixed point and is proportional in magnitude to the displacement from the fixed point.
If the frame be non-inertial, then Fx will be the x-component of both real and pseudo forces.

www.physicsashok.in 7
SIMPLE HARMONIC MOTION

Solving the differential equation of SHM :


d2x
We know that ax = , hence
dt 2
d2x
= 2x (Diff. eqn. of SHM)
dt 2
This is a differential equation in x.
dx
Let = Vx, Then
dt
d 2 x dVx dx dV
2
Vx x (Chain rule)
dt dx dt dx
dVx
Vx 2 x
dx
VxdVx = 2x dx (variables separated)
During oscillation, the particle returns at extreme points x = A (At x = A, the velocity becomes zero for a
moment.)
Vx x
2
Then V dV
x x xdx
0 A

Vx x
Vx2 2
2 x
2 2
0 A
Vx2 = 2[x2 A2]
Vx2 = 2(A2 x2)
Vx A 2 x 2 ...(1)
Equation (1) shows that
(i) Particle has opposite velocities for the same position. This is possible at different moments of time.
(ii) Particle has equal velocities at two positions at equal distance from mean position. The maximum speed occurs
at x = 0, and is A.
(iii) Equation (1) can be rearranged in the form

Vx2 x2
1
2A 2 A2
This shows that velocity-position graph is an ellipse. This is also true for momentum-position graph. The
momentum-position graph is known as phase-space graph of the particle.

Let Vx A 2 x 2 at time t. Then Vx


A
dx
A2 x 2 A A x
dt A
Separating the variables and integrating, we have

www.physicsashok.in 8
SIMPLE HARMONIC MOTION

dx
A2 x 2
dt

x
sin 1 t
A
where, = constant of integration.
x
= sin(t + )
A
x = A sin(t + ) ...(2)
is decided by the position x at a given time t.

PHASE
The general equation of SHM along X-axis may be written as
x = Asin(t + )
Here (t + ) contains all the informations of dynamical state of oscillator. It indicates what portion of motion
is complete by the time t and is called phase of motion. Characteristics of SHM are the following :
(i) Amplitude : the maximum distance of the particle from its mean position is known as amplitude.
From the equation | x |max = A
Thus, A = amplitude of SHM.
(ii) Phase angle or phase : The portion of motion that is completed by a given time is represented by phase or
phase angle.
(t + ) is phase at time t in the expression

x
= sin(t + )
A
(iii) Phase constant or epoch or initial phase : The phase at t = 0 is called initial phase, phase constant, or
epoch. In the expression above,
= phase constant = epoch
(iv) Angular frequency : 2 times frequency is called angular frequency. In the above expression, is angular
frequency :
2
= = 2v
T
S.I. Unit of is s1.
Special case
(i) If the particle is at x = 0 at t = 0
Then 0 = 0 +
=0
x = A sin t
(ii) If at t = 0, the particle is at rest (extreme), x = A.
0 = 0 +

www.physicsashok.in 9
SIMPLE HARMONIC MOTION

=0
x = A sin t

For x = A For x = A
A A
= sin( 0 + ) = sin( 0 + )
A A
1 = sin 1 = sin
= sin1 1 = sin1(1)
= /2 = /2
x = Acos t x = A cos t

Example 1: If x = A/2 at t = 0, find phase constant in x = A sin(t + ). At t = 0, a particle executing SHM is


going along x-axis.
A
Sol. Here = Asin( 0 + )
2
A 1
= sin or sin =
2A 2
= 30 or 5/6
Now we use the other condition : Vx < 0 at t = 0
We have Vx = A cos(t + )
Putting t = 0 and = /6,
Vx = A cos( 0 + /6)
3
Vx A , which is positive.
2
Now = 5/6
5 3
Vx = A cos = A
6 2

5
which is negative and satisfies the given condition. Hence the required =
6

Example 2: A small mass executes linear SHM about O with amplitude a and period T. Its displacement from O
at time T/8 after passing through O is :
(A) a/8 (B) a / 2 2 (C) a/2 (D) a / 2
Sol: x = A sin t
2 T
x A sin
T 8
A
or x A sin Hence option (D) is correct.
4 2

www.physicsashok.in 10
SIMPLE HARMONIC MOTION

Example 3: The figure shows the displacement-time graph of a particle executing x


SHM. If the time period of oscillation is 2s, then the equation of motion is given 10
(mm)
by x = _______. 5
Sol. In the case of SHM, 0
t(s)
x = A sin (t + )
or 5 = 10 sin ( 0 + ) (Here A = 10 mm = 0.01 m)
1
sin
2


6
2 2
Here
T 2

x = 0.01 sin t where x is in metre.
6

Example 4: Time period of a particle executing SHM is 8 sec. At t = 0 it is at the mean position. The ratio of the
distance covered by the particle in the 1st second to the 2nd second is :
1 1
(A) (B) 2 (C) (D) 2 1
2 1 2
Sol: Here x = A sin t
2
x1 = A sin 1
8
A
x1 = A sin
4 2
2
and x 2 A sin 2A
8
The distance traveled in 2nd second is
A
x2 x 2 x1 A
2

( 2 1)A

2

x1 A/ 2 1
Ratio
x2 ( 2 1)A ( 2 1)
2

2 1 ( 2 1)
=
( 2 1)( 2 1) 2 1
= ( 2 1) Hence Option (D) is correct.

www.physicsashok.in 11
SIMPLE HARMONIC MOTION

C17: A particle executing SHM has an amplitude of 1 metre and time period 2 sec. What are the velocity and the
acceleration when the displacement is 0.5 metre ?
2
Sol. 3.14 rad / sec.
T
Acceleration, f = 2y = (3.14)2 0.5 = 4.93 m/sec2.
Velocity, V a 2 y 2 3.14 (1)2 (0.5)2 2.72 m / sec.
C18: In the above question, calculate the maximum velocity and maximum acceleration.
Sol. Vmax = a = 3.14 1 = 3.14 m/sec
fmax = 2a = (3.14)2 1 = 9.86 m/sec2.
C19: A particle executing SHM of amplitude 10 cm has speed 3.14 cm/sec. at the mean position; calculate its
time period.
Sol. At the mean position,
2 2a
V a a T 20 sec.
T V
C20: A particle is executing SHM of time period 10 sec., and amplitude 9 cm. What would be the velocity of the
particle 2.5 sec. after it passes through the mean position ?
Sol. V = a cos t., here t is measured from the mean position.
2 2
Va cos t
T T

2 2
V 9 cos 2.5
10 10

2
V 9 cos 0
10 2
C21: A body executing SHM has its velocity 10 cm/sec. and 7 cm/sec. when its displacement from the mean
position are 3 cm and 4 cm respectively. Calculate the length of the path.
Sol. We have V a 2 y2

V2 a 2 y 22

V1 a 2 y12

49 a 2 16
or
100 a 2 9
a = 4.76 cm.
Length of the path = 2a = 9.5 cm

Example 5: A particle executes SHM on a straight line path. The amplitude of oscillation is 2 cm. When the
displacement of the particle from the mean position is 1 cm, the numerical value of magnitude of acceleration is
equal to the numerical value of magnitude of velocity. The frequency of SHM (in second1) is :
2 3 1
(A) 2 3 (B) (C) (D)
3 2 2 3

www.physicsashok.in 12
SIMPLE HARMONIC MOTION

Sol: A2 x 2 2 x
2 A2 x2 4x 2
or 2 A2 2 x 2 4 x 2
or A2 x2 = 2x2
or 22 12 = 2 12
or 3 = 2
3
2
or 3
T
2 1 3
T f Hence option (C) is correct.
3 T 2

Example 6: A stone is swinging in a horizontal circle 0.8 m in diameter at 30 rev/min. A distance horizontal light
beam causes a shadow of the stone to be formed on a nearly vertical wall. The amplitude and period of the
simple harmonic motion for the shadow of the stone are
(A) 0.4 m, 4 s (B) 0.2 m, 2 s (C) 0.4 m, 2 s (D) 0.8 m, 2 s
d 0.8
Sol: A 0.4m
2 2
30 2

60
2
or T = 2 sec. Hence option (C) is correct.
T

Example 7: A body undergoing SHM about the origin has its equation is given by x = 0.2 cos 5t. Find its average
speed from t = 0 to t = 0.7 sec.
Total distance
Sol. v=
Total time
Here x = 0.2 cos t
Here = 5
2 2
or 5 , T 0.4 sec.
T 5
nT
Here t t0
4
t = n 0.1 + t0 or 0.7 = n 0.1 + t0
From this, n = 7 and t0 = 0
But distance travelled in T/4 is A = 0.2 unit
Total distance = 0.2 7 = 1.4 unit.
1.4
v 2 unit
0.7

www.physicsashok.in 13
SIMPLE HARMONIC MOTION

Example 8: A particle perfroms SHM with a period T and amplitude a. The mean velocity of the particle over the
time interval during which it travels a distance a/2 from the extreme position is
(A) a/T (B) 2a/T (C) 3a/T (D) a/2T
Sol: x = a cos t
a/2 = a cos t0

or cos cos t 0
3

or t 0 t0
3 3
/ 3

dx v
0
vdt
v a sin t / 3
dt dt
0

2 3a
But v Hence option (C) is correct.
T T

d2 y
Example 9: The angular frequency of motion whose equation is 4 2 + 9y = 0 is (y = displacement and t = time)
dt
9 4 3 2
(A) (B) (C) (D)
4 9 2 3
d2 y
Sol: 4 2 9y 0
dt
d2y 9
or y
dt 2 4
Comparing with SHM equation
d2 y 2 9 3
2 y Hence option (C) is correct.
dt 2 4 2 a

Example 10: The acceleration-displacement (a x) graph of a particle executing


simple harmonic motion is shown in the figure. Find the frequency of oscillation. O x
Sol. In SHM,
= 2x
= 2()
= 2 ...(i) (from graph)

and

1
or 2f or f=
2

www.physicsashok.in 14
SIMPLE HARMONIC MOTION

Example 11: A graph of the square of the velocity against the square of the acceleration of a given simple harmonic
motion is :

v2 v2 v2 v2

(A) (B) (C) (D)


a2 a2 a2 a2
Sol: x = A sin t
dx
v A cos t A 2 x 2
dt
dv
a A 2 sin t
dt
dv
a 2 x
dt
a
But x 2

a2 2 2 2 a2
v A 4 2
or v A 4 Hence option (D) is correct.

C22: A particle executing SHM has its acceleration represented by the equation f = 42y. Calculate its time
period.
Sol. f = 42y = 2y where, 2 = 42
2 2
T 1 sec
2

Example 12: A block is kept on a horizontal table. The table is undergoing simple harmonic motion of frequency
3Hz in a horizontal plane. The coefficient of static friction between block and the table surface is 0.72. Find the
maximum amplitude of the table at which the block does not slip on the surface.
Sol. amax = 2A
fmax = Smg
or mamax = Smg
or m2A = Smg
Sg 0.72 10
A
2 (2f ) 2
0.72 g
A [ 2 = g]
4 2 32
A = 0.02 m = 2 cm

Example 13: A plank with a small block on top of its is under going vertical SHM. Its period is 2 sec. The minimum
amplitude at which the block will separate from plank is :
10 2 20
(A) (B) (C) (D)
2 10 2 10

www.physicsashok.in 15
SIMPLE HARMONIC MOTION

Sol: Here g = A2

g g gT 2
A
2 2 2 4 2

T

10 4 10
A 2 m Hence (A) option is correct.
4 2

Example 14: A force f = 10x + 2 acts on a particle of mass 0.1 kg, where k is in m and F in newton. If it is
released from rest at x = 2m, find :
(a) amplitude; (b) time period ; (c) equation of motion.
Sol. f = 10x + 2
df dx
10
dt dt
f = 10 x
ma = 10 x
10
a x
0.1
or 2x = 100 x
or = 10 rad/s
2 2
(b) T sec.
10 5
Let at x = x0, mean position is situated.
f = 10x + 2
0 = 10x0 + 2
2
x0 0.2 m
10
f = 10(x + 0.2) + 2
or f = 10x 2 + 2
ma = 10 x
10
or a x
0.1
or a = 100 x
x = A sin (t + )
or x = A sin(10t + )
At t = 0, x = 2.2 metre
2.2 = A sin
Also, v A 2 x 2
or 0 10 A 2 (2.2) 2
A = 2.2 m
2.2 = 2.2 sin

www.physicsashok.in 16
SIMPLE HARMONIC MOTION



2

x 2.2sin 10t
2
x = 2.2 cos 10 t
The displacement from original is x = 2.2 cos 10 t + x0
x = 0.2 2.2 cos 10 t
11
x = 0.2 cos 10 t
5

Example 15: A particle of mass 4 kg moves between two points A and B on a smooth horizontal surface under the

action of two forces such that when it is at a point P, the forces are 2PA N and 2PB N . If the particle is
released from rest at A, find the time it takes to travel a quarter of the way from A to B.
Sol. F = 2(l x) 2x
F = 2l 2x 2x = 2l 4x 2x P
2l 4x l 2(l x) B
a x A
or x
4 2
l
v v l
or 0 v dv 0 2 x dx
x
v2 l x2
or x
2 2 2 0

v2 l x2
or x
2 2 2
2 2
or v = lx x
or v lx x 2
dx
or lx x 2
dt
l dx t0
or 0
lx x 2
dt
0


t0 sec ond
3

How to solve problems (Force method) :


Step 1 : Write equilibrium equation.
Step 2 : Displace the particle along x-axis from equilibrium position and write the forces in terms of x.
Step 3 : Take x-component of all the forces and use step 1. Make approximation (like small x, etc.) Compare
with standard equation Fx = m2x to get .

www.physicsashok.in 17
SIMPLE HARMONIC MOTION

Oscillation of a spring :
A block of mass m is hanging vertically from the lower end of an ideal spring whose upper end is fixed to the
sealing. The stiffness of the spring is k. The block is now pulled downward and released. We can see the
motion of the block is simple harmonic and find the angular frequency of its oscillation.
In equilibrium, the extension in the spring is x0. The spring force is kx0. This balances mg. Hence
mg = kx0 ...(1)
Now the block is pulled by x. Then spring force is k(x0 + x) and is more than mg. taking x-component,
Fx = mg k(x + x0)
Fx = mg kx kx0 (2)
Using (1)
Fx = kx ...(3)
This satisfies the equation of SHM. Hence the motion is SHM.
The restoring force is kx where k is spring constant and x is extension or compression relative to equilibrium.
Comparing Eq. (3) with standard equation of SHM,

k
m2x = kx; =
m
This is the angular frequency of the oscillations.

C23: A mass of 200 gm is connected to a light spring of force constant 5 N/m and is free to oscillate on a
horizontal, frictionless surface. If the mass is displaced 5 cm from the equilibrium position and released from
rest, then calculate the period of its motion.
Sol. We have K / m 5 / 200 103 5 rad / sec.
T = 2 / = 2/5 = 1.26 sec.
C24: A heavy brass sphere is hung from a light spring and is set in vertical small oscillation with a period T. The
sphere is now immersed in a non0-viscous liquid with a density 1/10th the density of the sphere. If the system
is now set in vertical SHM, its period will be
(A) (9/10)T (B) (9/10)2T (C) (10/9)T (D) T
m
Sol: T 2
k
Hence option (D) is correct.
Example 16: Two springs of spring constant K1 and K2 are suspended as shown in the figure. A body of mass M
is suspended as shown.
(a) Calculate the effective spring constant of the system, and
(b) The time period.
Sol. (a) In this case same force will act on both the springs, but their displacement will not be the same.
Let y1 and y2 be their displacement, and let F be the force, then
y = y1 + y2
and F = K1y1 y1 = F/K1
Also F = K2y2 y2 = F/K2
If K is the effective spring constant of the system, then

www.physicsashok.in 18
SIMPLE HARMONIC MOTION

F = Ky y = F/K
K1
F F F
Since, y = y1 + y2
K K1 K 2

1 1 1 K1 K 2 K2
or K
K K1 K 2 K1 K 2
M
K
If K1 = K2 = K, then K =
2

2M M(K1 K 2 )
(b) T 2 2
K K1K 2
If the two spring have same spring constant, i.e. if K1 = K2 = K, then K = K/2
2M
T 2
K
Example 17: Two spring of spring constant K1 and K2 are arranged as shown in the figure. A body of mass M is
attached as shown.
(a) Calculate the effective spring constant of the system, and
(b) The time period.
Sol. (a) In this case the displacement (y) in the two springs will be same, but different restoring forces will act on
them.
Let F1 and F2 be the restoring forces acting on the two springs, then
F = F1 + F2
Where F is the total force.
K1 K2
Again, F1 = K1y and, F2 = K2y
If K is the effective force constant of the system, then
F = Ky M
Since, F = F1 + F2 Ky = K1y K2y
K K1 K 2

If K1 = K2 = K, then K 2K
(b) Time period,
M M
T 2 2
K K1 K 2

M
If K1 = K2 = K, then T 2
2K

Example 18: Two spring of spring constant K1 and K2 are arranged as shown. A body of mass M is attached as
shown in the figure.
(a) Calculate the effective force constant of the system, and
(b) The time period.
Sol. (a) In this case same displacement y will take place in the two springs, but different force will act on them.
If the mass M is displaced in downward direction, then the other spring will be compersed.

www.physicsashok.in 19
SIMPLE HARMONIC MOTION

Since th forces are different, therefore,


F = F1 + F2
K1
and, Let y be the common extension or compression, then
F1 = K1y, and F2 = K2y M
If K is the effective force constant of the system, then
F = Ky K2
Since, F = F1 + F2 Ky = K1y K2y
K K1 K 2

If K1 = K2 = K, then K 2K
(b) Time period

M M
T 2 or T 2
K K1 K 2

M
If K1 = K2 = K then T 2
2K

Example 19: A spring of length L, force constant K has time period T. it is cut in two equal parts; calculate the
time period of each part, if same mass is suspended from each part. Also calculate the spring constant of each
part.
Sol. Now, for one part
1
y1 y and K1 = 2K
2

M M 1
Therefore, T1 2 2 T
K1 2K 2

original Time period


Time perid of each
No. of parts

Example 20: A body is in SHM with period T when oscillated from a freely suspended spring. If this spring is cut
in two parts of length ratio 1 : 3 & again oscillated from the two parts separately, then the periods are T1 & T2
then find T1/T2.
Sol. As we know, spring constant is inversely propertional to its natural length.
C
k
l
C
k1
l1
C
and k2
l2
k1 l 2
3
k 2 l1

www.physicsashok.in 20
SIMPLE HARMONIC MOTION

k1 = 3 k2
1 1 1
But
k k1 k 2
1 1 1 1 3
or
k 3k 2 k 2 3k 2
3k 2
k
4
4k
k2
3
k1 = 3k2 = 4k
m
But T 2
k

m
T1 2
k1

m
and T2 2
k2

T1 k2 4k

T2 k1 3 4k

T1 1
1 : 3
T2 3

C25: In the above question the two parts are connected in series, if same mass is connected to one of them, then
(a) calculate the spring constant of the system, and (b) the time period.
K 1K 2
Sol. (a) K
K1 K 2 but K1 = K2 = 2K K = K

M M
(b) T 2 2 T
K K

C26: In the above questions. If the two parts are connected


as shown in the figure, then (1) calculate the force constant K1 = 2K K1 = 2K
K1T
of the system (2) time period of this system.
Sol. Clearly K = K1 + K2 = 2K1 = 4 K and time period. M M
M M T
T1 2 2
K 4K 2

Example 21: Find the ratio of time periods of two identical springs if they are first joined in series & then in parallel
& a mass m is suspended from them :
(A) 4 (B) 2 (C) 1 (D) 3

www.physicsashok.in 21
SIMPLE HARMONIC MOTION

m
Sol: T1 2 (in series)
k eq

1 1 1 2

k eq k k k

k
k eq
2

2m
T1 2
k

m
T2 2 (in parallel)
k
But k = k + k = 2k
m
T2 2
2k

2m
2
T1 k 2

T2 m
2
2k

Example 22: Two bodies P & Q of equal mass are suspended from two separate massless springs of force
constants k1 & k2 respectively. If the maximum velocity of them are equal during their motion, the ratio of
amplitude of P to Q is :
k1 k2 k2 k1
(A) k (B) (C) k (D) k
2 k1 1 2

k1
Sol: 1
m

k2
and 2
m
v1 max = v2 max
or A11 = A22
A1 2 k2
or Hence option (B) is correct.
A 2 1 k1

C27: A spring mass system is hanging from the ceiling of an elevator in equilibrium. The
k
elevator suddenly starts accelerating upwards with acceleration a, find

(a)the frequency; (b) the amplitude of the resulting SHM. m

www.physicsashok.in 22
SIMPLE HARMONIC MOTION

Sol. Frequency will remain unchanged only mean position will change.
1 k
f
2 m kx k(x + x0)
Amplitude : figure (a) : Free body diagram of block w.r.t. ground
is shown in figure. For equilibrium of block. ma = pseudo force
kx = mg ...(i)
Refer figure (b) : Free body diagram of block w.r.t. elevator is mg mg + ma
shown in figur. For equilibrium of block, (a) (b)
k(x + x0) = mg + ma ...(ii)
From equations Eqs. (i) and (ii),
ma
x0 = amplitude =
k
Example 23. A point mass m is suspended at the end of a massless wire of length l and cross-section A. If Y is
Youngs modulus of elasticity for wire, obtain the frequency of oscillation for simple harmonic motion along the
vertical line.
Sol. The original length of wire = l
The downward force acting on the block = mg
If T is the tension in the wire, then
T = mg because the is in equilibrium
According to Hookes law
Long stress T / A
y=
Long strain l / l
where l is the increase in length of the wire.
l
T = YA
l
Putting T = mg, mg = YA (l / l) ...(1)
Let the block be displaced down a little through a distance x during oscillators. The tension in the wire acting
upwards will be
YA mg
( l x) ( l x)
l l
The downward force = mg
Resultant downward force acting on the mass will be
mg
F= (l x) mg
l
mg
F = x
l
or, F = kx ...(3)
Thus the net force is directly proportional to the displacement, but oppositely directed. Hence the motion of the
block is SHM.
Comparing with standard euation of SHM is
F = m2x, ..(5)
We get (k / m)

www.physicsashok.in 23
SIMPLE HARMONIC MOTION

T = 2/ = 2 m / k = 2 ml / AY
1 1 AY
and frequency of vibration = n =
T 2 ml
ENERGY IN SHM
A particle performing SHM has motion and hence, a kinetic energy the restoring force being a conservative
force. It also has a potential energy.
KINETIC ENERGY
Let m be the mass of the simple harmonic oscillator moving at a velocity Vx. It has got a kinetic energy due to
motion. It is given by
1
K mv 2x
2

1
K
2

m 2 A 2 x 2
The plot of K against x is a parabola.
k(x)

A A x
If we have x = A sin(t + )
We get V = A cos(t + )
This gives
1
K= m2A2 cos2(t + )
2
Its plot K(t) is given below :
k(t)

O t

Note that its period its , the half of period of oscillation.

C28 : How much is the frequency of variation of KE of a particle moving simple harmonically with a time period of
2s?
Sol. T = 1/2 2s = 1 s
Potential energy of oscillator :

If
. dr 0 , then F is said to be conservative.
F

The force acting on a simple harmonic oscillator is Fx = m2x.


xf

F . dr Fx dx m xdx 0
2
Here,
xi

www.physicsashok.in 24
SIMPLE HARMONIC MOTION

Thus, Fx(= m2x) is conservative.


Since the force responsible for simple harmonic motion is conservative, a potential energy must exist
corresponding to this force. If U be the potential energy then the decrement in U is equal to the work done by
the conservative Fx.
i.e., dU = Fxdx = m2x dx
dU = m2x dx
It can be integrated to get U as a function of x. On integrating, we get
1
U= m2x2 + constant
2
Umax =1/2m 2A2
The value of constant depends upon our choice.
By chosing U = 0 at x = 0, the equation becomes
1
U= m2x2
2 A A x
The plot U(x) is a parabola as in the figure. The potential is maximum at extremes.

Mechanical energy of the particle in SHM :


It is the sum of kinetic and potential energies. We have
1 1
K= m2 (A2 x2) or U= m2 x2
2 2
Hence eh mechanical energy E is given by
E=K+U
1
E = m2 A2
2
The mechanical energy of the oscillator remains constant. This is so because no non-conservative force is
doing work.
The figure shows plot of K(x), U(x) and E(x), where U(0) = 0 is the reference value of U.
E
1/2m 2A2
E
K

U
A A x
If U(0) 0, we shift U(x) and E(x) by U0. Then the graphs. E(x) and U(x) is shifted by U0 but K remains
unshifted.
E(x)
We note that energy taking part in oscillation is given by Etot = 1/2m 2A2
2 2
EOSC = 1/2m A

UOSC = 1/2m2 x2 U(x)


Eosc = 1/2m 2A2

KOSC = 1/2m2 (A2 x2). K(x) U0


x = A x=A

www.physicsashok.in 25
SIMPLE HARMONIC MOTION

C29: In SHM, F = kx or a = 2x, i.e., Fx graph or ax graph is a straight line passing through origin with
negative slope. The corresponding graphs are shown below.
F a
F = kx F= 2x

x x

2
(a) Slope = k (b) Slope =
C30: Any function of t, say y = y(t) oscillates simple harmonically if

d2 y
y
dt 2
or we can say if above condition is satisfied, y will oscillate simple harmonically.
C31: All sine and cosine functions of t are simple harmonic in nature. i.e., for the function
y = A sin (t ) or y = A cos (t )

d2 y
is directly proportional to y. Hence, they are simple harmonic in nature.
dt 2
C32: How the different physical quantities (e.g., displacement, velocity, acceleration, kinetic energy, etc.) vary
with time or displacement are listed below in tabular form.

TABLE
S.No. Name of the equation Expression of the equation Remarks
1. Displacementtime x = A cos(t + ) x varies between +A and A
2. Velocity-time (v = dx/dt) v = A sin(t + ) v varies between + A
and A
3. Acceleration-time (a = dv/dt) a = A2 cos(t + ) a varies between + A2
and A2
4. Kinetic energy-time (K = 1/2mv2) K = 1/2mA22 sin2(t + ) K varies between 0
and 1/2mA22
5. Potential energy-time (U = 1/2m2x2) K = 1/2m2 A2 cos2(t + ) U varies between 1/2mA22
and 0
6. Total energy-time (E = K + U) E = 1/2m2 A2 E is constant
7. Velocitydisplacement v A2 x 2 v = 0 at x = A and at x = 0
v = A
8. Accelerationdisplacement a = 2 x a = 0 at x = 0
a = 2A at x = A
9. Kinetic energydisplacement K = 1/2m2 (A2 x2) K = 0 at x = A
K = 1/2m2A2 at x = 0
10. Potential energy-displacement U = 1/2m2 x2 U = 0 at x = 0, U = 1/2m2A2
at x = A
11. Total energydisplacement E = 1/2m2 A2 E is constant

www.physicsashok.in 26
SIMPLE HARMONIC MOTION

C33: From the above table we see that x, v and a are sine or cosine functions of time. So, they all oscillate simple
harmonically with same angular frequency . Phase difference between x and a is and between any other
two is /2.
C34: Kinetic energy versus time equation can also be written as
K = 1/4mA22[1 cos 2(t + )]
This function is also periodic with angular frequency 2. Thus, kinetic energy is SHM is also periodic with
double the frequency then that of x, v and a. But these oscillations are not simple harmonic in nature, because
d2K
is not proportional to K. But,
dt 2

1 1
K mA22 = mA22 cos 2(t + ) = K0 (say)
4 4
Here, K0 is simply a cosine function of time. So, K0 will oscillate simple harmonically with angular frequency 2.
Same is the case with potential energy function. U also oscillate with angular frequency 2 but the oscillations
are not simple harmonic in nature. Total energy does not oscillate. It is constant.
Thus,
x oscillate simple harmonically with angular frequency
v oscillate simple harmonically with angular frequency
a oscillate simple harmonically with angular frequency
K oscillate with angular frequency 2but not simple harmoncally
U oscillate with angular frequency 2but not simple harmoncally
E does not oscillate.
C35: In the above discussion we have read that potential energy is zero at mean position and maximum at
extreme positions and kinetic energy is maximum at mean position and zero at extreme positions. But the
correct statement is like this,
At mean position K is maximum and U is minimum (it may be zero also, but it is not necessarily zero).
at extreme positions K is zero and U is maximum.
U(J) U(J) U(J)

x(m) x(m) x(m)


2 2
(a) (b) (b)
Thus, in figure (a), oscillations will take place about the mean position x = 0 and minimum potential energy at
mean position is zero.
In figure (b) mean position is at x = 2 m and the minimum potential energy in this position is 5 J.
In figure (c) mean position is at x = 2m and the minimum potential energy in this position is again zero.

THE CAUSES OF OSCILLATION


Consider a particle free to move on x-axis, is being acted upon by a force given by,
F = kxn
Here, k is a positive constant.
Now, following cases are possible depending on the value of n.

www.physicsashok.in 27
SIMPLE HARMONIC MOTION

(i) If n is an even integer (0, 2, 4, ..... etc), force is always along negative x-axis, whether x is positive or negative.
Hence, the motion of the particle is not oscillatory. If the particle is released from any position on the x-axis
(except at x = 0) a force in negative direction of x-axis acts on it and it moves rectilinearly along negative x-axis.
(ii) If n is an odd integer (1, 3, 5, ..... etc), force is along negative x-axis for x > 0, along positive x-axis for x < 0
and zero for x = 0. Thus, the particle will oscillate about stable equilibrium position, x = 0. The force in this case
is called the restoring force. Of these, if n = 1, i.e., F = kx the motion is said to be SHM.

Example 24. Describe the motion of a particle acted upon by a force


(i) F = 2(x 2)3 (ii) F = 2(x 2)2 (iii) F = 2(x 2)
Sol. (i) F = 2(x 2)3
F=0 at x=2
Force is along negative x-direction for x > 2
and it is along positive x-direction for x < 2. Thus, the motion of the particle is oscillatory (but not simple
harmonic) about x = 2.
(ii) F = 0 for x = 2, but force is always along negative x-direction for any value of x except at x = 2. Thus, the
motion of the particle is rectilinear along negative x-direction.
(iii) Let, us take x 2 = X, then the given force can be written as,
F=2X
This is the equation of SHM. Hence, the particle oscillates simple harmonically about X = 0 or x = 2.

Example 25: A body of mass 1kg is executing simple harmonic motion which is given by y = 6.0 cos(100t + 4)
cm. What is the (i) amplitude of displacement, (ii) Angular frequency, (iii) initial phase, (iv) velocity, (v) acceleration,
(vi) maximum kinetic energy ?
Sol. The given equation of SHM is
y = 6.0 cos (100t + /4) cm.
Comparing it with the standard equation of SHM, y = A sin (t + ), we have
(i) Amplitude A = 0.6 cm
(ii) Angular frequency = 100 sec1.
(iii) Initial phase = /4

(iv) Velocity v = (A 2 y 2 ) = 100 (36 y 2 ) cm/sec


(v) Acceleration = 2y = (100)2y = 104y
1 1
(vi) Kinetic energy = mv2 = m2(A2 y2)
2 2
The kinetic energy of a particle in SHM is maximum, when it passes its mean position i.e. at y = 0
1 2 1
(K.E.)max = mv max = mA22
2 2
1
= 1 104 (0.06)2 = 18 joules
2

www.physicsashok.in 28
SIMPLE HARMONIC MOTION

Example 26: A particle of mass 0.8 kg is executing simple harmonic motion with an amplitude of 1.0 metre and
periodic time 11/7 sec. Calculate the velocity and the kinetic energy of the particle at the moment when its
displacement is 0.6 metre.
Sol. We know that, at a displacement y from mean position particles velocity is given as

v = (A 2 y 2 )

2
or v (A 2 y 2 )
T
2 3.14
v [(1.0) 2 (0.6)2 ] 3.2 m/sec.
(11/ 7)
Kinetic energy at this displacement is given by
1
K= mv2
2
1
K= 0.8 (3.2)2 = 4.1 joule
2
Example 27: A person normally weighing 60 kg stands on a platform which oscillates up and down harmonically at
a frequency 2.0 sec1 and an amplitude 5.0 cm. If a machine on the platform gives the persons weight against
time, deduce the maximum and minimum reading it will show, take g = 10 m/sec2.
Sol. As shown in figure, platform is executing SHM with amplitude and angular frequency given us
A = 5.0 cm
and = 2n = 4 rad/sec [as n = 2 sec1]

+A
weighing
machine

A
Here weighing machine will show weight more then that of man when it is below its equailibrium position when
the acceleration of platform is in upward direction. In this situation the free body diagram of man relative to
platform is shown in figure. Here ma is a pseudo force on man in downward direction N
relative to platform (or weighing machine). As weighing machine will read the normal
reaction on it thus for equilibrium of an relative to platform,
we have N = mg + ma
mg + ma
or N = mg + m(2y) [as | a | = 2y]
Where y is the displacement of platform from its mean position. We wish to find the maximum weight shown by
the weighing machine, which is possible when platform is at its lowest extreme position as shown in figure, thus
maximum reading of weighing machine will be
N = mg + m2A
N = 60 10 + 60 (4)2 0.05

www.physicsashok.in 29
SIMPLE HARMONIC MOTION

N = 600 + 480
N = 1080 Nt = 108 kg t
Similarly the machine will show minimum reading when it is at its upper extreme position when pseudo force on
man will be in upward direction, thus minimum reading of weighing machine will be
N = mg m2A
N = 600 480 = 120 Nt = 12 kg t

Example 28: A 20 g particle is oscillating simple harmonically with a period of 2 second and maximum kinetic
energy 2 J. The total mechanical energy of the particle is zero. Write the following
(a) Amplitude of oscillation
(b) Potential energy as a function of displacement x relative to mean position.
Sol. (a) m = 20 g = 20 103 kg
T = 2s
Kmax = 2 J
1
Using Kmax = m2 A2
2
2
1 2
we have 2 20 103 . A 2
2 2
2
3
A2
5 10 4 2
10
A m

(b) Etot = EOSC + U0
0 = 2 + U0
U0 = 2 J
1
U(x) = m2 x2 2
2
2
1 20 2 2
U(x) x 2
2 1000 2

2 2
u(x) x 2 (S.I. units)
100
Example 29: A body is executing SHM under the action of force whose maximum magnitude is 50N. Find the
magnitude of force acting on the particle at the time when its energy is half kinetic and half potential.
Sol. Total energy
E = 1/2 mA22
Up = Uk
1 1
mA 2 2 sin 2 t mA 2 2 cos 2 t
2 2

www.physicsashok.in 30
SIMPLE HARMONIC MOTION

or tan2t = 1

t
4
a = 2A sin t

F = ma = 2Asin (in the sence of magnitude)
4
50
F 25 2 N
2
Example 30: A point particle of mass 0.1 kg is executing SHM with amplitude of 0.1 m. When the particle passes
through the mean position, its K.E. is 8 103 J. Obtain the equation of motion of this particle if the initial phase
of oscillation is 45.
1
Sol. mv 2max 8 10 3
2
1
or mA 2 2 8 103
2
1
or 0.1 (0.1) 2 2 8 10 3
2
or 2 = 16
= 4 rad/s

x = A sin t
4


x = 0.1 sin 4t
4
Example 31: In case of simple harmonic motion (a) what fraction of total energy is kinetic and what fraction is
potential when displacement is one half of the amplitude, (b) at what displacement the kinetic and potential
energies are equal?
Sol. In case of simple harmonic motion
1 1 1
2

K m 2 A 2 y 2 , U m 2 y 2 and
2
E m 2 A 2
2
2
K y 1 3 A
(a) So f K 1 2 1 as y given
E A 4 4 2
2
U y 2 (A / 2) 1
fP 2
E A A 4
(b) According to given problem
1 1
K = U, i.e., m2(A2 y2) = = m2y2
2 2
A
i.e. 2y2 = A2 or y 0.7 A
2

www.physicsashok.in 31
SIMPLE HARMONIC MOTION

Example 32: A particle starts oscillating simple harmonically from its equilibrium position then, the ratio of kinetic
energy and potential energy of the particle at the time T/12 is : (T = time period)
(A) 2 : 1 (B) 3 : 1 (C) 4 : 1 (D) 1 : 4
Sol: x = A sin t
dx
v = A cos t
dt
1 1
Uk mv 2 mA 2 2 cos 2 t
2 2
1
and UP mA 2 2 sin 2 t
2
U k cos 2 t
2
cot 2 t
U P sin t
Uk 2 T
cot 2
UP T 12
Uk
cot 2 cot 2 30
UP 6
Uk
3 :1 Hence option (B) is correct.
UP

Example 33: A body performs simple harmonic oscillations along the straight line ABCDE with C as the midpoint
of AE. Its kinetic energies at B and D are each one fourth of its maximum value. If AE = 2R, the distance
between B and D is
R A B C D E
3R
(A) (B) (C) 3R (D) 2R
2 2
1
Sol: Uk m 2 (A 2 x 2 )
2
1 1 1
or mA 2 2 m 2 (A 2 x 2 )
4 2 2
A2
or
4

A2 x 2
A 2 3A 2
or x 2 A2
4 4
3A 3
x R Hence option (A) is correct.
1 2

Example 34: The potential energy of a simple harmonic oscillator of mass 2 kg in its mean position is 5 J. If its total
energy is 9J and its amplitude is 0.01 m, its time period would be
(A) /10 sec (B) /20 sec (C) /50 sec (D) /100 sec

www.physicsashok.in 32
SIMPLE HARMONIC MOTION

1
Sol: mv 2max 9 5 4
2
or mv 2max 8
or 2 v 2max 8
or v max = 2 m/s = A
v max 2
200
A 0.01
2
or 200
T
2
T Hence (D) option is correct.
200 100

Example 35: A spring mass system preforms SHM. If the mass is doubled keeping amplitude same, then the total
energy of SHM will become :
(A) double (B) half (C) unchanged (D) 4 times
1
Sol: E mA 2 2
2
1 k
E mA 2
2 m
1 2
E A k (Independent of mass) Hence option (C) is correct.
2
C36: A mass at the end of a spring executes harmonic motion about an equilibrium position with an amplitude A. Its
speed as it passes through the equilibrium position is V. If extended 2A and released, the speed of the mass
passing through the equilibrium position will be
(A) 2V (B) 4V (C) V/2 (D) V/4
Sol: v = A
v = 2A
v = 2v Hence option (A) is correct.
C37: A linear harmonic oscillator has a total mechanical energy of 200 J. Potential energy of it at mean position is
50 J. Find,
(i) the maximum kinetic energy
(ii) the minimum potential energy
(iii) the potential energy at extreme positions.
Sol. At mean position, potential energy is minimum and kinetic energy is maximum. Hence,
Umin = 50 J (at mean position)
and Kmax = E Umin = 200 50
Kmax = 150 J (at mean position)
At extreme positions, kinetic energy is zero and potential energy is maximum
Umax = E
Umax = 200 J (at extreme position)

www.physicsashok.in 33
SIMPLE HARMONIC MOTION

C38: The potential energy of a particle oscillating on x-axis is given as


U = 20 + (x 2) 2
Here U is in joules and x in metres. Total mechanical energy of the particle is 36 J.
(a) State whether the motion of the particle is simple harmonic or not.
(b) Find the mean position.
(c) Find the maximum kinetic energy of the particle.
dU
Sol. (a) F= = 2(x 2)
dx
By assuming x 2 = X, we have F = 2X
Since, FX
The motion of the particle is simple harmonic
(b) The mean position of the particle is X = 0 or x 2 = 0, which gives x = 2 m
(c) Maximum kinetic energy of the particle is,
Kmax = E Umin = 36 20 = 16 J.
Note : Umin is 20 J at mean position or at x = 2 m.

Example 36. If a particle moves in a potential energy field U = U0 ax + bx2, where a and b are positive constant,
obtain an expression for the force acting on it as a function of position. At what point does the force vanish? Is
this a point of stable equilibrium ?
Calculate the force constant and frequency of the particle.
dU
Sol. F a 2bx
dx
F = 0, at x = a/2b
d2U
2b 0
dx 2
a
i.e., x is a point of minimum potential energy. Hence, the equilibrium is stable.
2b
k = 2b
1 k 1 2b
and f
2 m 2 m

Example 37: A particle of mass m moves in a one-dimensional potential energy U(x) = ax2 + bx4, where a and b
are positive constants. The angular frequency of small oscillations about the minima of the potential energy is
equal to
a a 2a a
(A) (B) 2 (C) (D)
2b m m 2m
Sol: U = ax2 + bx4
dU
F 2ax 4bx 3
dx

www.physicsashok.in 34
SIMPLE HARMONIC MOTION

At mean position F = 0
2ax = 4bx3
2a a
or x2
4b 2b
a
x0
2b
F = 2a(x0 + x) + 4b(x0 + x)3
3
x
F = 2ax0 2ax + 4bx03 1
x
0

3x
F = 2ax0 2ax + 4bx03 1 x
0

12bx 30 x
3
F = 2ax0 2ax + 4bx0 +
x0
F = 2ax0 2ax + 4bx03 + 12bx02x
a a a a
F 2a 2ax 4b 12b x
2b 2b 2b 2b

a a
F 2a 2ax 2a 6ax
2b 2b
ma = + 4ax
4a
a x
m
a x
a = 2x
4a a
2
m m
U(x)
Example 38: A particle of mass m moves in the potential energy
U shown above. The period of the motion when the particle
has total energy E is U= 1 2
2 kx , x > 0
U=mgx. x > 0

x
(A) 2 m / k 4 2E / mg 2 (B) 2 m / k

(C) m / k 2 2E / mg 2 (D) 2 2E / mg 2
Sol: For x<0
dU
F kx
dx
ma = kx
k
or a x
m

www.physicsashok.in 35
SIMPLE HARMONIC MOTION

k
12 x x
m
k
1
m

m
T1 2
k
For x > 0
U = mgx
dv
F mg
dx
1
But E mv 20
2

2E
v0 , it is speed at lowest point.
m
2v 0 2 2E
T2
g g m
T1 m 2 2E
T T2
2 k g m
Hence option (C) is correct.

PASSAGE (Q. 39 to 41)


The graphs is figure show that a quantity y varies with displacement d in a system undergoing simple harmonic
motion.
y y y y

(I) (II) (III) (IV)


O d O d O d O d
Example 39: The total energy of the system
(A) I (B) II (C) III (D) IV
Sol: In SHM, total energy remains constant.

Example 40: The time


(A) I (B) II (C) III (D) IV
Sol: x = A sin t
Hence option (D) is correct.

Example 41: The unbalanced force acting on the sytem


(A) I (B) II (C) III (D) None
2
Sol: a = x
F = ma = m2x Hence option (D) is correct.

www.physicsashok.in 36
SIMPLE HARMONIC MOTION

Energy method for SHM problems :


1. Write the equilibrium equation.
2. Take displaced position and write the potential plus kinetic energies : E = K + U
dE
3. Set = 0 and obtain dynamical equation.
dt

Example 42: The friction coefficient between the two blocks shown in figure is and
the horizontal plane is smooth. If the system is slightly displaced and released,
m
(a) find the time period k
M
(b) find the magnitude of the frictional force between the blocks when the displacement
from the mean position is x,
(c) find the maximum amplitude if the upper block does not slip relative to the lower block during oscillations.
Sol. (a) For small amplitude, the two blocks oscillate together.
The angular frequency is of a single block spring system :
[k /(M m)]
and so the time period is :
T 2 [(M m) / k]
(b) When the blocks are at distance x from the mean position, their acceleration is :
kx
a = 2x =
Mm
The resultant force on the upper block is, therefore
mkx
mf =
Mm
The force is prompted by the frictional of the lower block. Hence the magnitude of the frictional force is
mk | x |
.
Mm
(c) the maximum frictional force required for simple harmonic motion of the upper block is mKA/(M + m) at the
extreme position. But the maximum frictional force can only be mg.
mkA (M m)g
Therefore = mg or A=
Mm k

Example 43: A 2 kg mass is attached to a spring of force constant 600 N/m and rests on a smooth horizontal
surface. A second mass of 1 kg slides along the surface toward the first at 6 m/s.
(a) Find the amplitude of oscillation if the masses make a perfectly inelastic collision and remain together on the
spring. What is the period of oscillation?
(b) Find the amplitude and period of oscillation if the collision is perfectly elastic.
(c) For each case, write down the position x as a function of time t for the mass attached to the spring, assuming
that the collision occurs at time t = 0.
What is the impulse given to the 2 kg mass in each case ?
Sol. (a) From conservation of linear momentum,
1 6 = (1 + 2)v
v = 2 m/s

www.physicsashok.in 37
SIMPLE HARMONIC MOTION

Now from conservation of mechanical energy,


1 1 6m/s
1kg 2kg
mv 2 kA 2
2 2
m 3
A v 2 or A = 0.141 m = 14.1 cm
k 600

m 3
T 2 2 0.44 s
k 600
(b) For perfectly elastic collision,
m m1 2m1 2 1
v2 2 v2 v1 or v2 0 6 4 m/s

m1 m 2 m1 m 2 1 2

m 2 1/ 2
A v 2 600 4 or A = 0.23 m = 23 cm
k

m 2
T 2 2 0.36 s
k 600
2 2
(c) In the first case, 14.28 rad / s
T 0.44
and amplitude A = 14.1 cm
x = A sin t = (14.1 cm) sin (14.28 t)
In the second case,
2 2
17.45 rad / s
T 0.36
and amplitude A = 23 cm
x = 23 sin (17.45 t) cm
Impulse, J = P = 2 2 = 4 N-s in the first case
and 4 2 = 8 N-s in the second case.
Example 44: A body A of mass m1 = 1.00 kg and a body B of mass m2 = 4.10 kg are A
interconnected by a spring as shown in figure. The body A performs free vertical
harmonic oscillations with amplitude a = 1.6 cm and frequency = 25 rad/s. Neglecting
the mass of the spring, find the maximum and minimum values of force that this system
B
exerts on the bearing surface.
Sol. Nmax : k = 2m1 = (25)2 (1.0) = 6.25 N/m
Acceleration of centre of mass in extreme position, a= 2
A
m1
2
m1a m A
a CM (upwards) 1
m1 m 2 m1 m 2

m1 2 A
Now N max (m1 m 2 )g (m1 m 2 ) m2
(m1 m 2 )
Nmax = (m1 + m2)g + m12A
Nmax = (1.0 + 4.10)9.8 + (1.0)(25)2(1.6 102) = 59.98 newton

www.physicsashok.in 38
SIMPLE HARMONIC MOTION

Example 45: In the figure shown, the block A collides with the block B and after u
collision they stick together. Calculate the amplitude of resultant vibration. M M
B A
u
Sol. Common velocity after collision =
2
2
1 u 1 2
Hence, K.E. = (2M) Mu
2 2 4
It is also the total energy of vibration because the string is unstretched at this moment, hence if A is the amplitude,
then
1 1 M
KA 2 Mu 2 or A u.
2 4 2K
Example 46: A body of mass m falls from a height h on to the pan of a spring balance. The mass of the pan is M
and spring is massless. The force constant of the spring is k. The body gets struck to the pan and starts
performing harmonic oscillations in the vertical direction. Calculate the period, amplitude and energy of these
oscillations.
Sol. Let v be the velocity immediately after impact.
Then m 2gh = (M + m)v
m
or v 2gh ...(1)
Mm
The period of oscillation of a spring depends on the mass attached and its force constant and it is given by
m
T 2
k

Mm k
here T 2 or
k Mm

x0 m
1 x
0
l0 x0
M 2
l 3

The body vibrates about the equilibrium position.


Let the spring be compressed by x0 under the weight of the pan. Then Mg = kx0.
Let the spring be compressed by x0 from the previous position under the weight (M + m)g. Then
(M + m)g = k(x0 + x0)
This positive (position 2) is the equilibrium position of the system (pan + body). Let x0 be the maximum
compression of the spring relative to position 1. This is the lower extreme position (position 3) of the system.
Applying Conservation of energy between positions 1 and 3
1 1
(M m)gl k x20 (M m)v 2
2 2
1
(M m)g(l x 0 ) k(x0 x 0 ) 2 0
2

www.physicsashok.in 39
SIMPLE HARMONIC MOTION

m 2 gh 1
(M m)g x 0 k x0 x 0 k x 02
Mm 2
Mg 1
Mg x 0 mg x 0 k x 0 k x 20
k 2

2 2m 2 gh
kx 2mg x 0
0 0
Mm

mg mg 2kh
x0 1
k k Mm
The negative value is not acceptable
mg mg 2kh
x0 1
k k (M m)g
Since the amplitude of SHM is the distance between the mean position and the extreme position, here a,
amplitude
mg 2kh
x 0 x0 1
k (M m)g
1
and E (M m)a 2 2
2
1 m 2g 2 2kh k
E (M m) 2 1
2 k (M m)g M m

1 m2g 2 2kh
E 1
2 k (M m)g

Example 47: The spring shown in figure is kept in a stretched position x0.
Assuming the horizontal surface to be frictionless, calculate the frequency k
M m
of oscillations after the system is released.
Sol. As there is no external force acting on the system of two blocks plus the spring, the centre of mass of the
system will remain at rest. The mean positions of two simple harmonic motion or two blocks occur when the
spring becomes unstretched. If the mass m moves towards left through a distance X area the mass M moves
towards left through a distance x before acquires natural length,
x + X = x0 ...(1)
where x and X will represent the amplitudes of two blocks m and M respectively. Because the centre of mass
of the system must not move during the motion, we can wtire
mx = MX ...(2)
From (1) and (2)
Mx 0 mx 0
x= and X= ...(3)
Mm Mm
x and X also give the amplitudes of m and M during oscillations. At the mean positions the two blocks have
1 1 1
kinetic energies m2x2 and M2x2. Their sum must be kx02.
2 2 2

www.physicsashok.in 40
SIMPLE HARMONIC MOTION

Mm
Now, from (3) X0 = X
M
1 1
kx02 = 2(mx2 + Mx2)
2 2
Using (3), angular frequency
K(M m)

Mm
and the frequency
K(M m)
n
2 Mm

Example 48: Two identical balls A and B each of mass 0.1 kg are attached to two
identical massless springs. The spring-mass system is constrained to move inside
a rigid smooth pipe bent in the form of a circle as shown in fig. The pipe is fixed in A B
y1 y2
a horizontal plane. The centres of the balls can move in a circle of radius 0.06 m.
/6 /6
Each spring has a natural length of 0.06 m and force constant 0.1 N/m. Initially, P Q
O
both the balls are displaced by an angle = /6 radian with respect to diameter
PQ of the circle and released from rest.
(a) Calculate the frequency of oscillation of the ball B.
(b) What is the total energy of the system ?
(c) Find the speed of the ball A when A and B are at the two ends of the diameter PQ.
Sol. (a) As here two masses A and B are connected by two springs, this problem is equivalent to the oscillation of
a reduced mass m by a spring of effective force constant keff given by
m1m 2 0.1 0.1
m 0.05 kg
m1 m 2 0.1 0.1
and keff = k1 + k2 = 0.1 + 0.1 = 0.2 N/m

1 k eq 1 0.20 1
So f Hz
2 m 2 0.05
(b) As here one spring is compressed while the other is stretched by same amount (say y) and balls are at rest at
A and B, so
1 1
E k1y 2 k 2 y 2 ky 2 [as k1 = k2]
2 2
But from above figure
y = y1 + y2 = R1 + R2 = 2R
y = 2 0.06 (/6) = 0.02 m
So E = (0.1)(0.02)2 = 42 105 J
(c) As at P and Q springs are unstretched so the whole energy becomes kinetic of the balls A and B, i.e.
1 1
m1v12 m 2 v 22 E 4 2 105 J
2 2
Here m1 = m2 = 0.1 kg and v1 = v2 = v
2 2 5
So 0.1 v = 4 10 , i.e., v = 2 102 m/s

www.physicsashok.in 41
SIMPLE HARMONIC MOTION

Example 49: Figure shows a block P of mass M resting on a horizontal


smooth floor at a distance l from a rigid wall. Block is pushed
toward right by a distance 3l/2 and released, when block passes P
from its mean position another block of mass m1 is placed on it k
m
which sticks to it due to friction. Find the value of m1 so that the
combined block just collides with the left wall. l
Sol. When block P is released from rest from a distance 3l/2 toward right from mean position, this will be the
amplitude of oscillation, so velocity of block when passing from its mean position is given as
3l k k
v A [as = ]
2 m m
If mass m1 is added to it and just after if velocity of combined block becomes v1, from momentum conservation
we have
mv = (m + m1)v1

m 3l k
or v1
m m1 2 m
If this is the velocity of combined block at mean position, it must be given as
k
v1 = A1 [now 1 = m m1 ]
Where A1 and 1 are the new amplitude and angular frequency of SHM of the block. It is given that combined
block just reaches the left wall thus the new amplitude of oscillation must be l so we have
m 3l k k
. l1
(m m1 ) 2 m m m1

3 m 5
or 1 or 9m = 4m + 4m1 or m1 m
2 m m1 4

Example 50: For the arrangement shown in figure, the spring is initially
compressed by 3cm. When the spring is released the block collides with 4
k = 10 N/m m = 1 kg
the wall and rebounds to compress the spring again.
(a) If the coefficient of restitution is 0.7, find the maximum compression in the
spring after collision. 4 cm
(b) If the time starts at the instant when spring is released, find the minimum time after which the block becomes
stationary.
Sol. (a) Velocity of the block just before collision,
1 1 1
mv 20 kx 2 kx 02
2 2 2
k 2
or v0
m 0

x x2
here, x0 = 0.03 m, x = 0.01 m, k = 104 N/m, m = 1 kg
v0 2 2 m / s

www.physicsashok.in 42
SIMPLE HARMONIC MOTION

After collision, v = ev0 = (0.7) 2 2 = 2 m/s


Maximum compression in the spring is
1 2 1 2 1
kx m kx mv 2
2 2 2

2 m 2 2 1(2) 2
or x m x v (0.01) 2.23 cm
k 104
(b) In the cse of spring-mass system, since the time period is independent
of the amplitude of oscillation, therefore sin 1(1/3)
+3
T0 1 x
T 2 sin 1
2 3 O
1
/2 /2
v
m 1 1 3
T 2 sin 3
k sin 1(1/3)

Example 51: The system shown in the figure can move on a smooth surface. the spring is initially compressed by
6 cm and then released. Find
k=800N/m
(a) time period 6 kg
3kg
(b) amplitude of 3 kg block
(c) maximum momentum of 6 kg block.


Sol. (a) T 2
k
m1m 2 3 6
Here 2 kg
m1 m 2 9
2 1
T 2 2
800 20

T sec ond
10
(b) Here x0 = 6 cm
But x10 + x20 = x0
and m1x10 = m2x20
or 3x10 = 6x20
or x10 = 2x20
x10 + x20 = x0
x10
or x10 x0
2
3
or x10 x 0
2
2 2
x10 x 0 6 4 cm
3 3

www.physicsashok.in 43
SIMPLE HARMONIC MOTION

(c) Applying conservation principle of momentum


3 v1 6 v2 = 0
3v1 = 6v2
v1 = 2v2
Applying mechanical energy conservation
1 1 1
3 v12 6 v 22 kx 20
2 2 2
3 2 1
or v1 3v 22 800 (0.06) 2
2 2
3
or (2v 2 ) 2 3v 22 400 36 104
2
3
or 4v 22 3v 22 1.44
2
or 9 v22 = 1.44
1.2
or v2 0.4 m / s
3
P2max = 6 0.4 = 2.4 kg m/s

Example 52: A 2 Kg block moving with 10 m/s strings a spring of constant


10 m/s
2 N/m attached to 2 Kg block at rest kept on a smooth floor. The
2 kg 2 kg
time for which rear moving block remain in contact with spring will be
1 1
(A) 2 sec (B) sec (C) 1 sec (D) sec
2 2


Sol: T 2
k
22
Here reduced mass = 1
2 2
1
T 2 2 s.
2
Initially, system is at mean position.
Again blocks separate when spring comes in natural length (i.e. mean position).
Hence, t = T/2 = 1 second.

Example 53: Two blocks A(2kg) and B(3kg) rest up on a smooth horizontal surface are connected by a spring of
stiffness 120 N/m. Initially the spring is undeformed. A is imparted a velocity
of 2m/s along the line of the spring away from B. Find the displacement of 3kg 2kg
2m/s
A t seconds later. B A

22 4
Sol. v cm 0.8 m / s
23 5
In C-frame, the motion is SHM.

www.physicsashok.in 44
SIMPLE HARMONIC MOTION

But m1x1C = m2x2C


But x0 = x0OC + x0OC
m2 x 0 m1x 0
x1OC and x 2OC
m1 m 2 m1 m 2
Applying conservation principle of mechanical energy in C-frame,
1 2 1 2
rrel kx 0
2 2
1 23 1
or 4 120 x 02
2 5 2
64
or x 20
5 120
2
x0
10
2
3
m2 x 0 10 6
x1OC
m1 m 2 5 50

12
x1OC 0.12 m
100

k
Also,

m1m 2 3 2 6
here
m1 m 2 5 5

120
10 rad / s
1.2
x1C = x1OC sin t
or x1 xC = 0.12 sin 10t
or x1 = xC + 0.12 sin 10t
x1 = 0.8t + 0.12 sin 10t
Example 54. Two blocks of masses m1 and m2 are connected by
m1 m1 m2
a spring of stiffness k. Another block of mass m1, sliding at V0 k
without friction, hits the set up elastically as shown in the figure. V0

Plot elastic energy of the spring as a function of time t after the


collision.
Sol. Due to one dimensional elastic collision of identical bodies, velocity is transferred totally to m1.
m1 m2
V0

Let x1 and x2 be positions of m1 and m2 at a time t. Then extension in the spring (with natural length = l)
x = x2 x1 l ...(1)

www.physicsashok.in 45
SIMPLE HARMONIC MOTION

The equation of motion of m1 is writen as


d 2 x1 d 2 x1
m1 2 kx or m1 2 kx
dt dt
From these
d2 x d 2 x 2 d 2 x1
(x x l ) 2
dt 2 2 1
dt 2 dt
k k k k
x x x
m1 m2 m1 m 2
d2x
2
2 x ...(2)
dt
1 1
where k ...(3)
m 2 m1
Solving these,
x = A sin (t + )
At t = 0, x = 0, sin (t + )
sin = 0
= 0, ...(1)
dx 2 dx 2
At t = 0, 0, V0
dt dt
dx
0 V0 or A cos = V0
dt
= ...(2)
From (i) and (ii)
2 2
V02
sin + cos = 0 + 2 2
A
V0
A

V0 V
x sin(t ) 0 sin t

1
Now we can write U(x) kx 2 U(t)
2 kv02
kV 2 2 2 2
0 Umax
The plot of U(t) sin t
2CO 2

t
Circular Representation of SHM
y
We have discussed that an oscillation can be regarded B
as ShM if it satisfies the basic requirements to be y 2 2
SHM. Every SHM can be best represented as a 1 1
projection of a particle in circular motion on its P P
t=0 x x
diameter. In fact the motion of projection of a uniform x C
x O t=0 A
circular motion on its diameter satisfies both the
conditions required to be SHM. Let us analyze it from y
the figure shown. D y

www.physicsashok.in 46
SIMPLE HARMONIC MOTION

A particle P is executing a uniform circular motion on the circle with a uniform angular velocity as shown. The
particle is shown at its initial position A at t = 0. It starts in anticlockwise direction with uniform angular velocity
. If we follow the motion of the projection of particle on its vertical diameter in figure then we can see, along
withP, its projection P starts from mid point (centre of circle) in upward direction and the respective position
of P for position of P at point 1 and 2 are 1 and 2 as shown in figure.
When P reaches the topmost position, P will also be at this point and when P starts tracing the second
quadrant of circle, P starts coming down towards point O. Thus we can see, as point P traces its circular path
ABCD, its projection on diameter YY follows oscillatory motion along OBODO .... and so on. This oscillatory
motion of P can be taken as SHM as it follows the conditions to be a SHM. Well prove it in next section.
Here motion of particle is in a straight line with amplitude equal to the radius of circle. As shown in figure, the
displacement of P from point O as a function of time can be given as y
y = R sin t ...(1) P t=t
As P is going up, its velocity can be given as y R
x t x
dy A t=0
v= =Rcos t ...(2)
dt
Its acceleration is
y
dv
a= = R2 sin t
dt
From equation (1), we have
a = 2y ...(3)
Here is a constant thus the acceleration of P is directly proportional to the displacement from its mean
position O and negative sign in equation (3) shows that direction of acceleration is opposite to y that is towards
mean position O. Hence the motion of projection P can be regarded as SHM.

Equation of SHM
Equation of an oscillation is the mathematical expression giving the displacement of oscillating particle from its
mean position as a function of time.
B
If a particle is executing SHM with amplitude A, it can be regarded as the P t=t
projection of a circular motion of redius A as shown in figure, if circular motion P
R
of point P is at a constant angular velocity then this is termed as angular C
y
t P
frequency of the point P which is in SHM. P A t=0

Let us consider at t = 0, P was at point A and starts in anticlockwise


direction as shown in figure. Now in time t, point P traversed by an angle t (as shown) and PDreaches to a
displacement y as shown, can be given as
y = A sin t ...(4)
This equation-(4) in this case is called as equation of SHM of point P which is in SHM. Here t which is the
angular displacement of point P (in circular motion) is called phase angle of point P in SHM.
In previous article weve discussed the SHM of point P which was at its mean position at t = 0. But it is not
necessary that particle starts its SHM from its mean position. It can start from any point on its path, thus
equation-(4) can not be accepted as a general equation of SHM, this being the equation of those all SHMs
where particle starts (at t = 0) their SHM from their mean position.

www.physicsashok.in 47
SIMPLE HARMONIC MOTION

Now consider figure where a particle P starts its SHM with a point Y
having initial phase angle anticlockwise on the circle of point P of B t=t
P
which it is the projection. Let as t = 0, point P was at an angular P
y P P
displacement from its reference point (point A). Thus the point from t=0
t
which P will start its SHM is shown in figure. At this position (projec- X X
C A
tion of P at t = 0 on YY) is called as initial phase of point P in
SHM, as is the initial angular displacement of particle P which is in
circular motion. Now after time t, the angular displacement of P is (
D Y
+t) which is called the instantaneous phase of point P at time t = t
and at this instant the displacement of point P from mean position is
y = A sin (t + ) ...(5)
This equation-(5) is called as general equation of SHM of a particle which starts its SHM with initial phase .
Here initial phase of SHM implies it is the initial angular displacement of the particle which is in circular motion
of which the projection is executing SHM.
As equation-(4) gives the equation of SHM of those particles which starts their SHM from mean position.
Similarly we can define an equation of SHM of those particles which start their SHM from their extreme
position by substituting = /2 in equation (5). As if a particle starts from its extreme position, we can take its
initial phase /2 thus its equation of SHM from equation (5) can be given as
y = A cos t ...(6)
In all type of problems in which a body or a particle execute SHM, we assume that this is the projection of an
another perticle who is in circular motion and its projection is executing SHM (that body or particle which is in
SHM).

Valocity and acceleration of a particle in SHM


Equation (5) gives the general expression for displacement from mean position of a particle executing SHM as
a function of time. Thus velocity of this particle as a function of time can be given as
dy
v= = A cos (t + ) ...(7)
dt
To convert it in displacement function, we can write.

v = A 1 sin 2 (t )

y2
v = A 1 2 [as y = A sin t]
A

v = A 2 y 2 ...(8)
Equation (8) gives the velocity of a particle in SHM with amplitude A, and angular frequency as a function of
its displacement from mean position. From equation-(8) we can state that in SHM, particles velocity is maximum
when y = 0 i.e. at its mean position and is given as
at y = 0, vmax = A
At extreme positions of particle when y = A, its velocity is zero where it returns towards its mean position.
From equations-(5) and (8) we can plot the graphs of displacement and velocity as a function of time as shown
in figure.

www.physicsashok.in 48
SIMPLE HARMONIC MOTION
y

A sin

t
O T 2T

T= 2
y = A sin ( t + )
(a)

+A y

T/2 3T/4 7T/4


t T= 2
O T/4 T 5T/4 2T

A y = A sin t
(b)

T/4 T/2
t
O T 2T
3T/4

y = A cos t
(c)

Similarly acceleration of particle in SHM can be given as


dv
a= = A2 sin(t + ) ...(9)
dt
or a = 2y [as y = A sin (t + )] ...(10)
From equation-(10) we can see that at mean position (y = 0) when velocity of particle is maximum, its acceleration
is zero and at extremities where y = A, accelration of particle is maximum and its magnitude is given as
amax = 2A (towards mean position)
It also shows that as particle moves away from mean position, its acceleration continuously increases till it
reaches its extreme position (at amplitude) when its velocity becomes zero and it returns.
Equation-(10) can be rewritten in differential from as

d2 y
+ 2y = 0 ...(11)
dt 2
This equation-(11) is called Basic Differential Equation of motion of a particle in SHM. Every expression of
displacement y as a function of time which satisfies this equation can also be regarded as an equation of SHM.
C39: Prove that yAeit is an equation of SHM.
Sol. According to given equation in problem, differentiating with respect to time we get,

www.physicsashok.in 49
SIMPLE HARMONIC MOTION

dy
= i A eit
dt
Differentiating again with respect to time, we get

d2 y
= 2Aeit = 2y [as y = Aeit]
dt 2

d2 y
Thus we have + 2y = 0
dt 2
This is the basic differential equation of SHM hence y = Aeit is an equation of SHM.

Example 55: The displacement of a body executing SHM is given by x = A sin (2t + /3). The first time from
t = 0 when the velocity is maximum is
(A) 1/12 sec. (B) 0.16 sec (C) 0.25 sec (D) 0.5 sec
dx
Sol: v 2A cos 2t
dt 3

or 2A = 2A cos (2t + )
3 t = t0

or cos 2t 1 t=0
3 /3

Hence
2 3
3 2 Phasor diagram

6
= t0
1
or 2 t 0 t0 sec. Hence (A) option is correct
6 12 12
Phase analysis of a particle in SHM
Weve already discussed about phase angle of a particle in SHM. It is actually the angular displacement of that
particle who is in circular motion and whose projection is in SHM. At a general time t, the instantaneous phase
of a particle in SHM can be written as (t + ) if is its initial phase (already discussed).

Phase difference in two SHM


Case I : When two SHMs are of same agnular frequency
Figure shows two particles P and Q in SHM with same angular frequency . P and Q are the corresponding
particles in circular motion for SHM of P and Q.
Let P and Q both starts their circular motion at the same time at t = 0 then at the same instant P and Q starts
their SHM in upward direction as shown. As frequency of both are equal, both will reach their extreme position
(topmost point) at the same time and will again reach their mean position simultaneously at time t = T/2. [T =
Time period of SHM = 2/] and move in downward direction together or we can state that the oscillations of
P and Q are exactly parallel and at every instant the phase of both P and Q are equal, thus phase difference
in these two SHMs is zero. These SHMs are called same phase SHMs.

www.physicsashok.in 50
SIMPLE HARMONIC MOTION

P t=t Q t=t
P Q
t P t Q
t=0 t=0
A A
ref. point ref. point

Now consider figure, where we assume if P starts its SHM at t = 0 but Q will start at time t1. In this duriation
from t = 0 to t = t1, P will move ahead in phase by t1 radians while Q was at rest. Now Q starts at time t1
and move with same angular frequency . It can never catch P as both are oscillating at same angule frequency.
Thus here Q will always lag in phase by t1 then P or we say P is leading in phase by t1 then Q ans as
of both are constant their phase difference will also remains constant so keep it in mind that in two SHMs of
same angular frequency, if they have some phase difference, it always remains constant.

t = t1
P
t1 P Q Q
A t=0 t = t1

Now we consider a spacial case when time lag between the starting of two SHMs is T/2 i.e. half of oscillation
period. Consider figure, here if we assume, particle P starts at t = 0 and Q at t = T/2 when P completes its
half oscillation. Here we can see that the phase difference in the two SHM is by which Q is lagging. Here
when Q starts its oscillation in upward direction P moves in downward direction. As angular velocity of P and
Q are same, both complete their quarter revolution in same time. Thus when P reaches its bottom extreme
position, Q will reach its upper extreme position and then after Q starts moving downward, P starts moving
upward and both of these will reach their mean position simultaneously but in opposite directions, P has
completed its one oscillation where as Q is at half of its oscillation due to a phase lag of .

t = T/2 P Q Q
P t=0 t = T/2
P

Thus if we observe both oscillations simultaneously we can see that oscillations of the two particls P and Q
are exactly antiparallel i.e. when P goes up, Q comes down and at all instants of time their displacements from
mean position are equal but in opposite direction if there amplitudes are equal. Such SHMs are called opposite
phase SHMs.
Case II : When two SHMs are of different angular frequency
Weve discussed that when two SHMs are of same angular frequency, their phase difference does not change
with time. Consider two particles in SHM as shown in figure. Their corresponding particles for circular motion
are A and B respectively as shown. If both A and B starts their SHM from mean position at t = 0 with angular
frequencies 1 and 2, then we say at t = 0 their phase difference is zero but after time t, their respective phase

www.physicsashok.in 51
SIMPLE HARMONIC MOTION

are 1t and 2t. Thus after time t, the phase difference in the two SHMs is = (1 2)t thus equation
above shows that if angular frequencies of the two SHMs are different their phase difference continuously
changes with time.

t=t 2
A 1
B t=t
A 1t A B 2t B
t=0 t=0

Same Phase SHMs


As discussed above two particles execute SHM in such a way that their oscillations are exactly parallel to each
other or their phase difference during oscillation is zero, they are said to be in same phase weve seen that this
happens when both SHMs are started at same time with same angular frequency. This can happen also when
the time lag between starting of the two SHM is T or an integral multiple of time period of SHM. Because if one
starts at t = 0 and other starts at t = T, in this duration first particle will complete its first oscillation and is going
to start its second oscillations and the second particle will start in synchronization with the first. Hence the two
oscillation will still be parallel or in same phase. There phase diference in the two SHMs will be 2. Not only
this even if the time lag in starting of the two SHMs is 2T, 3T ..... nT or the phase difference in the two SHMs
is 4, 6, 8 ...... 2n, then also these SHMs can be treated in same phase.
Thus phase difference in two SHMs of same phase is
= 2, 4, 6 .......... 2n
Opposite Phase SHMs
As discussed in previous article, two SHMs are said to be in opposite phase when their oscillations are
antiparallel this happens when two SHMs of same angular frequency start with a time lag of T/2 and phase
difference among the two SHMs is . By analyzing the situation it can also be stated that the same thing also
happens when the time lag in starting of the two SHMs is 3T/2, 5T/2 ........ (2n + 1)T/2 or the phase difference
between the two is 3, 5 ..........(2n + 1).
Thus phase difference in two SHMs of opposite phase is
= , 3, 5 ........., (2n + 1)
To understand the concept of phase and phase difference in SHM, we take few examples.

Example 56: Two particles are in SHM in a straight line about same equilibrium position. Amplitude A and time
period T of both the particles are equal. At time t = 0, one particle is at displacement y1 = +A and the other at
y2 = A/2, and they are approaching towards each other. After what time they cross each other ?
(A) T/3 (B) T/4 (C) 5T/6 (D) T/6
Sol: From phasor diagram, t = t0
t0 t = t0
t 0 t 0 t0
6 2 t0
6
C 6 t0
3 4 2 A
or 2t 0 2 3
6 2 6 6 3
2 2 T
t0 Hence option (D) is correct.
6 6 2 / T 6

www.physicsashok.in 52
SIMPLE HARMONIC MOTION

Example 57. Two particles execute SHM with same amplitude A and same angular frequency on same straight
line with same mean position. Given that during oscillation they cross each other in opposite direction when at
a distance A/2 from mean position. Find phase difference in the two SHMs.
Sol. Figure shows that two respective particles P and Q in SHM along with their corresponding particles in
circular motion. Let P moves in upward direction when crossing Q at A/2 as shown in figure(a), at this instant
phase of P is
1
1 = sin1 = ...(1)
2 6

P Q
P Q
N2 N2
1 2
A

Similarly as shown in figure(b) we can take particle Q is moving in downward direction (opposite P) at A/2,
this implies its circular motion particle is in second quadrant thus its phase angle is
1 5
2 = sin1 = = ...(2)
2 6 6
As both are oscillating at same angular frequency their phase diff. remains constant which can be given from
equation-(1) and (2), as
5 2
= 2 1 = =
6 6 3
Example 58: A particle executes SHM with amplitude A and angular frequency . At an instant when particle is at
a distance A/5 from mean position and moving away from it. Find the time after which it will come back to this
position agin and also find the time after which it will pass through mean position.
Sol. Figure shows the circular motion representation for the particle P given in prolbem. B
The initial situation of particle is shown in figure. As P moves, its projection P will
go up and then come back to its initial position when P reaches to the corresponding P
P A/5
position in second quadrant as shown. In this process P traversed an angular C A
displacement with angular veliocity , thus time taken in the process is

1 1 2cos 1 (1/ 5) D
t [2 cos (1/ 5)]

Now when P reaches point C, P will reach its mean position. Thus time taken by P from its initial position to
point C is
( ) sin 1 (1/ 5)
t

The same time P will take from A/5 position to mean position through its extreme position.

Example 59: Two particles executing SHM with same angular frequency and amplitudes A and 2A on same
straight line with same mean position cross each other in opposite direction at a distance A/3 from mean
position. Find the phase difference in the two SHMs.

www.physicsashok.in 53
SIMPLE HARMONIC MOTION

Sol. Figure shows the two corresponding particles of circular motion for the
two mentioned particles in SHM. Let particle P is going up and particle Q
is going down. From the figure shown, the respective phase differences
of particles P and Q are Q Q P
N3
2
2A 1
1
1
1 = sin 3 [Phase angle of P]

1
and 2 = sin1 [Phase angle of Q]
6
Thus phase difference in the two SHMs is

1 1
= 2 1 = sin1 sin1 3
6
C40: A particle stars its SHM from mean position at t = 0. If its time period is T and amplitude A. Find the distance
travelled by the particle in the time from t = 0 to t = 5T/4.
Sol. We know in one complete oscillation i.e. in period T, a particle covers a distance 4A and in first one quarter of
its period it goes from its mean position to its extreme position as its starts from mean position thus the distance
travelled by the particle in time 5T/4 is 5A.

Example 60: A spring block pendulum is shown in figure. The system is hanging in
equlibrium. A bullet of mass m/2 moving at a speed u hits the block from downward
direction and gets embedded in it. Find the amplitude of oscillation of the block Nature
now. Also find the time taken by the block to reach its upper extreme position length of
after hit by bullet. spring h
m
Sol. If block is in equilibrium then spring must be at some stretch if it is h, we have
mg = kh
If a bullet of mass m/2 gets embedded in the block, due to this inelastic impact its new mass becomes 3m/2 and
now the new mean position of the block will be say at a dept. h1 from old mean position then, we must have
3
mg = k(h + h1)
2
3m
g = k(h + h1)
2
v
mg m
h1 (as mg = kh)
2k
Just after impact due to inelastic collision if the velocity of block becomes v, we have u m/2
according to momentum conservation
m 3m
u v
2 2
u
or v
3
mg
Now the block executes SHM and at t = 0 block is at a distance h1 above its mean position and having
2k

www.physicsashok.in 54
SIMPLE HARMONIC MOTION

a velocity u/3. If amplitude of oscillation is A, we have


2
u mg
A2
3 2k
2
u 2 2k 2 mg
or A
9 3m 2k
k
[As for this spring block system = ]
(3m / 2)
2
mu 2 mg
A
6k 2k
Now time taken by particle to reach the topmost point can be obtained by circular
motion representation as shown in figure. This figure shows the position of block
P and its corresponding circular motion particle P0 at the t = 0. Block P will reach P P1
h1 A
its upper extreme position when particle P0 will travers the angle and reach the
topmost point. As P0 moves at constant angular velocity , it will take a time
given as
cos 1 (h1 / A) 3m 3mg
t cos 1
k 2k 2kA
3m / 2

Example 61: Figure(a) shows a spring block system, hanging in


equilibrium. The block of system is pulled down by a distance
x and imparted a velocity v in downward direction as shown k
in figure(b). Find the time it will take to reach its mean position.
Also find the maximum distance to which it will move before equilibrium
m x
position
returning back towards mean position.
m
Sol. As shown in figure (b), when the block is pulled down by a distance x
and thrown downward, it will start executing SHM. It will go further (a) (b)
to a distance A (amplitude) from mean position before returning back
which can be found by using the velocity of block v at a displacement
x from its mean position as
v A2 x 2
k 2 k
or v2
m

A x2 [as for spring block system =
m
]

mv 2
or A x2
k
Now time of motion of bob can be obtained by circular motion representation of the respective SHM.
Corresponding circular motion representation for this SHM is shown in figure at t = 0. At t = 0, block P is at a
distance x from its mean position in downward direction and it is moving downward so we consider its
corresponding circular motion particle in III quadrant as shown in reference angular velocity we consider P will
reach its mean position when particle P0 reaches position A by traversing an angle . Shown in figure. Thus it
will take a time given as

www.physicsashok.in 55
SIMPLE HARMONIC MOTION

B
sin x / A
1
t
k
C A
m A x
P0
P
v

m x
D
t sin 1
or k mv 2
x2
k
The maximum distance to which block will move from its initial position is A x as it gas upto its lower extreme
at a distance equal to its amplitude A from to mean position.

Example 62: Figure shows a block of mass m resting on a smooth horizontal


m u
ground attached to one end of a spring of force constant k in natural
length. If another block of same mass and moving with a velocity u toward
right is placed on the block which stick to it due to friction. Find the time k
it will take to reach its extreme position. Also find the amplitude of m
oscillations of the combined mass 2m.
smooth
Sol. When second mass sticks to the lower mass, due to such an inelastic collision, the velocity of combined block
is reduced by half that is u/2 to conserve momentum. Now at mean position the velocity of block can be written
as
u
A (if A is the amplitude of oscillation)
2

u k k
or A [As here for combined block new angular frequency of SHM is =
2 2m 2m ]

u 2m m
or A u
2 k 2k
As oscillation starts from mean position, in reaching its extreme position, particle has to cover a phase of /2
radians, thus time taken by particle to reach its extreme position is
/2 k
t
2 2m
Example 63: In previous example if block is pulled toward riht by a distance x0 and released, when the block
passes through a point at a displacement x0/2 from mean position, another block of same mass is gently placed
on it which sticks to it due to friction. Find the new amplitude of oscillation and find the time now it will take in
reaching its mean position and extreme position on left side.
Sol. When block was released at x0 from mean position, this will be the amplitude of oscillation and when it is
passing through the position of half amplitude x0/2, its velocity can be given as
2
x
2 k x 02
v A 0 or v x 02
2 m 4

3 k
v x0
2 m

www.physicsashok.in 56
SIMPLE HARMONIC MOTION

When another block of same mass is added to it, due to momentum conservation its velocity becomes half and
k k
of oscillation will, also change from to . Again using the formula for velocity of SHM at a
m 2m
distance x0/2 from mean position, we get
2
v x
v A2 0
2 2
2
3 k k x
or x0 A2 0 [if A is the new amplitude of oscillation]
2 m 2m 2

2 3 2 x 02 5 2 5
A x 0 x0 or A x0
8 4 8 2 2
To find time of motion in SHM we use circular motion representation of the respective SHM. The figure shows
the corresponding circular motion. If at t = 0, the second mass is added to the oscillating block, it was at a
position x0/2 from mean position and moving towards it, and after adding the mass the new amplitude of
k k
oscillation changes to A and changes from to . Figure shows the corresponding position of
m 2m
particle in circular motion at t = 0 in II quadrant. When this particle P0 will reach the position C after traversing
the angle , particle P in SHM will reach its mean position and similarly when P0 will reach position D, P will
reach the extreme position on other side. thus the time taken by P to reach mean position from a position of
x0/2 from mean position is given as
x B
sin 1 0 P0
2A t=0 P
t1 A0 v
k A x 0 /2 A
2m C

2
sin 1
5 2m 2 D
t1 sin 1
or k k 5
2m
Similarly time taken by P to reach the left end extreme position is

/2 2m 1
2
t2 sin
k 2 5

Example 64: Figure shows a block P of mass m resting on a smooth


horizontal surface, attached to a spring of force constant k which is P
k
rigidly fixed on the wall on left side, shown in figure. At a distance l to
m
the right of block there is a rigid wall. If block is pushed toward lift so
that spring is compressed by a distance 5l/3 and released, it will start l
its oscillations. If collision of block with the wall is considered to be
perfectly elastic. Find the time period of oscillations of the block.
Sol. As shown in figure, as the block is released from rest at a distance 5l/3 from its mean position, this will be the
amplitude of oscillation. But on other side of mean position block can move only upto a distance l from mean

www.physicsashok.in 57
SIMPLE HARMONIC MOTION

position and then it will return from this point with equal velocity due to elastic collision. Consider figure. If no
right wall is present during oscillation block P will executing complte SHM on right side of mean position also
up to its amplitude 5l/3. Thus we can observe the block P at a point X at a distance l from mean position
(where in our case wall is present), if block passes this position at speed v (which is the speed with which block
hits the wall in our cse), after reaching its extreme position Y, it will return and during return path it will cross the
position X with the same speed v (as displacement from mean position is same).
v X
P v

m l (a)
Y
5l/3 5l/3

Thus we can state that in our case of given problem, block P is executing SHM but it skips a part XYX on right
half of its motion due to electric collision of the block with the wall. If T0 is the time period of this oscillation we
look at figure (b), which shows the corresponding circular motion representation. Here during oscillation of
point P, particle P0 covers its circular motion along ECDAF, and from F it instantly jumps to E (due to elastic
collision of P with wall at X) and again carry out ECDAF and so on, thus we can find the time of this total
motion as
2 2sin 1 (3/ 5) B
t Y
k P0 P X
m E
l
F
5l/3
C A
m 1 3 5l/3
t 2 sin 5 ...(1)
k
D
We can also find the time period of this motion by subtracting the time of
FYE from the total time period as (b)

3
2 2 cos 1 m 1 3
5 or t 2 2 cos 5 ...(2)
t k

these equation-(1) and (2) will result same numerical value.
Example 65: Figure shows a spring block system hanging in equilibrium. If a velocity v0
is imparted to the block in downward direction. find the amplitude of SHM of the Natural length k
block and the time after which it will reach a point at half of the amplitude of block.
Sol. Initially in equilibrium if block is at a depth h below the natural length of spring then h
we have m P
mg = kh Mean position

If at mean position block is imparted a velocity v0, this would be the maximum velocity of block during its
oscillation. If its amplitude of oscillation is A, then it is given as
k
v0 = A [where = ]
m
v0 m
or A v0
k
Now to find the time taken by block to reach its half of amplitude point we

www.physicsashok.in 58
SIMPLE HARMONIC MOTION

consider the corresponding circular motion of the SHM as shown in figure. B

Here block P will reach to half of its amplitude when particle P0 will reach point E E
v0 A A
shown in figure at an angular displacement from mean position relative to point C A/2
A, thus time taken by it is P P0
1
sin 1 2 m
t D
km 6 k
Angular Harmonic Motion :
If the angular acceleration of a system be directly proportional in magnitude to the Z
angular displacement and be directed opposite to angular displacement relative to
fixed position, its motion is said to be angular simple harmonic. Z
In the figure rotation of the body about z-axis is such that angular acceleration z is

d 2
given by z = 2 where 2 is a positive constant. The motion is then angular SHM. As z ,
dt 2

d 2
the differential equation may be written as 2
2
dt
Multiplying both the sides by the moment of inertia of the body about the z-axis (I), we get z = I2

where z is the z-component of net torque acting on the body..

Table : SHM Features


Quantity Linear SHM Angular SHM

d2x d 2
Equation m + kx = 0 I + C = 0
dt 2 dt 2
Angular frequency k/m C/I
Displacement x = A sin(t + ) = 0sin(t + )
Phase t + t +
Phase constant
Amplitude A 0
Total energy of oscillation 1/2mA2 = 1/2kA2 1/2I202 = 1/2C02
Potential energy 1/2m2x2 1/2I22
Kinetic Energy 1/2I2(A2 x2) 1/2I2(02 2)

Simple Pendulum (mathematical pendiulum)


A heavy point mass suspended from a rigid support using inextensible, elastic and massless thread and free to
oscillate without friction is called a simple pendulum.
All these conditions are ideal and cannot be realised completely in particle. Hence such a pendulum is also
called mathematical pendulum. In laboratory a thread, a bob, a split cork and stand are arranged to get an
approximate simple pendulum.

www.physicsashok.in 59
SIMPLE HARMONIC MOTION

Splitcork rigid

l L= l + h + r friction Zero
h massless, inextensible,
r elastic cord
heavy point mass
Motion of simple pendulum
Let the thread be deflected by an angle about z-axis as in the figure. The torque Z-axis l sin
abotu z-axis is given by l
z = mgl sin
For small angle,
sin (see table) =0 mg

Table : Some values of sin


(degree) (radian) sin
1 0.01745 0.01745
2 0.03490 0.03490
3 0.05235 0.05234
4 0.06981 0.06976
5 0.08727 0.08716
6 0.10472 0.10453
7 0.12217 0.12187
8 0.13963 0.13917
9 0.15708 0.15643
10 0.17453 0.17365
11 0.19199 0.19081
12 0.20944 0.20791
13 0.22689 0.22495
14 0.24435 0.24192
15 0.26180 0.25882
(Values reounded at 5th place)
Then,
z = mgl (angular SHM)
Now,
d 2 2
d 2
z = I = ml
dt 2 dt 2
d 2
= ml2 2 = mgl
dt
2
d g
2

dt l
Comparing this with standard equation, we get
g
2
l

www.physicsashok.in 60
SIMPLE HARMONIC MOTION

Thus, the angular frequency of the simple pendulum is given by


g

l
The time period of the pendulum is given by
2 l
T= T = 2
g
This expression contains four informations known as laws of simple pendulum.
Laws of simple pendulum :
(i) T l (law of length)

1
(ii) T g (law of acceleration due to gravity)

(iii) T doesnt depend upon mass (law of mass)


(iv) T doesnt depend upon amplitude (law of amplitude)
Seconds Pendulum : The simple pendulum having time period of 2s is called seconds pendulum. A pendulum
clock generally uses this pendulum.

Simple pendulum in accelerated frame :


Case I : Let the simple pendulum be located in a lift. The acceleration D the lift is a0.

a 0 upward : In this case ma0 and mg are downward giving effective-g


geff = (g + a0)

a0

ma0 mg m(g+a0)

l l
T 2 2
g eff g a0
Time period has decreased. A pendulum clock in such a lift will be running fast.
Case II :

a directed downward : In this case pseudoforce ma0 acts upward. The net downward force is mg ma0. This
gives effective-g equal to g a0. Then

l
T 2
g a0

ma0 a0< g

mg m(g+a0)

www.physicsashok.in 61
SIMPLE HARMONIC MOTION

Case III :

a 0 is horizontal : In this case ma 0 and mg are at right angles. They give as effective - g
geff g2 a20
Hence the period is
ma0 a0
l
T 2
g a 02
2

mgeff = m g2 + a02 mg

Simple pendulum in a fluid (non-visicous) :



In a non-visicous fluid, the vertical buoyant force (m/)g acts opposite to mg . The net downward force is
mg (m/)g. m = Volume displaced
Fb = m g
The effective-g is 1 g .
m = mass of bob
= density of bob
= fluid density <
g eff g 1 mg mg mg = mg 1

l
T 2

g 1

C41: The length of a simple pendulum is increased by 4%; calculate the percentage increase in its time period.
4L1 104L1
Sol. Here, L 2 L1
100 100

T2 L2 104L1 104 4
1
Therefore,
T1 L1 100L1 100 100

1/ 2
T2 4 2 102
1 1 1.02
T1 100 100 100

T2 1.02 T1
Increase [or change] in time period = T2 T1
= T1(1.02 1) = 0.02 T1
0.02 T1
% Increase (or change) in time period = 100% 2%
T1
C42: Calculate the length of a second pendulum on the surface of the moon (given gm = 1/6ge).
L L
Sol. T 2 or 2 2
8m 8m
8g m 9.8
L 2
16.5 cm
6 (3.14) 2

www.physicsashok.in 62
SIMPLE HARMONIC MOTION

C43: The length of a second pendulum is increased by 4%, then what will be its new time period ?
1/ 2
T2 L2 4
Sol. Here 1 1.02
T1 L1 100
T2 = 1.02 T1 = 2.04 sec.
C44: Calculate how much time a second pendulum will lose in a day, if its length is increased by 4% ?
Sol. If the length is increased 4% then the time period of the seconds pendulum will become 2.04 sec; hence it will
lose 0.04 sec in every 2.04 sec. Therefore it will lose.
0.04
86, 400 sec in a day = 1694 sec in a day (nearly).
2.04
C45: Calculate the frequency of a seconds pendulum in an elevator, which is accelerated upward with an acceleration
of 49.0 m/sec2.
2
n2 g 2 49 9.8 58.8
Sol. n g 9.8 9.8 6
1 1

1
n 2 n1 6 6 1.224 per sec.
2
C46: A simple pendulum 50 cm long is suspended from the roof of a cart

cm
accelerating in the horizontal direction with a = 7 m/s2 (figure). Find

50
the period of small oscillations of the pendulum about its equilibrium m
angle. a = 7 m/s2

Sol. g eff a 2 g 2 (7) 2 (9.8) 2 12.0 m / s 2


l 0.5
T 2 2 1.28 s
g eff 12

Example 66. The length of a simple pendulum is 1 m. The bob of the pendulum (mass = 0.10 kg) is released when
the string is horizontal. Calculate in the lowest point of its path :
(i) its kinetic energy, (ii) the tension in the string.
Sol. The amplitude 0 = /2 is large. We cannot use SHM equations
(i) When the bob is released from horizontal, the loss in its potential energy at the lowest point = mgl, where l is
the length of the pendulum.
Let the velocity of bob at the lowest point is v. Then gain in K.E. = 1/2 mv2
From conservation of mechanical energy
mgl = 1/2 mv2
Hence, K.E. = 1/2 mv2 = mgl
= 1/100 9.8 1 = 0.098 J
(ii) Tension in the string T mg = mv2/l
Using (i)
v 2gl 2 9.8 1 19.6 = 4.42 m/s
1 1 19.6
T= 9.8 = 0.294 N.
100 100 1

www.physicsashok.in 63
SIMPLE HARMONIC MOTION

Example 67. The figure shows a simple pendulum whose bob is an elastic ball. It collides
elastically with the slant wall. Calculate the period of the pendulum oscillations for l

>
small and as shown in the figure.
Sol. Let the string make and angle with the downward vertical.
Let t = 0 at = . Then
= cos t
g
where
l
=
At the wall =
= cos t =
1
t cos 1

The period is double of this value. Hence
l
T2 cos 1
g

Example 68: Figure shows two identical simple pendulums of length l.


One is tilled at an angle and imparted an initial velocity v1 toward
mean position and at the same time other one is projected away from l l
mean position at a velocity v2 at an initial angular displacement B. Find
the phase difference in oscillations of these two pendulums. A v2 B
v1
Sol. It is given that first pendulum bob is given a velocity v1 at a displacement l from mean position, using the
formula for velocity we can find the amplitude of its oscillations as
V1 A12 (l )2 [If A1 is the amplitude of SHM of this bob]

g
As for simple pendulum = we have
l
g
V12 [(A12 ) (l ) 2 ]
l

2v12l
2
A1 l ...(1)
g
Similarly if A2 is the amplitude of SHM of second pendulum bob we have

V2 A 22 (l) 2

v 22l
2 2
or A2 l ...(2)
g
Now we represent the two SHMs by circular motion representation as shown in figure.

www.physicsashok.in 64
SIMPLE HARMONIC MOTION

A0
B t=0 t=0 A
A2
B0 l
l 2 A1 1

I SHM II SHM
(a) (b)
In first pendulum at t = 0 the bob is thrown from a displacement l from mean position with a velocity v1
toward mean position. As it is moving toward mean position, in figure(a), we consider the corresponding
circular motion particle A0 of the bob A is second quadrant, as the reference direction of , we consider
anticlockwise. As shown in figure initial phase of bob A is given as
l
1 sin 1 ...(3)
A1
Similarly for second pendulum bob B, its corresponding circular motion particle B0 at t = 0 is considered as
shown in figure (b) its initial base is given as
l
2 sin 1 ...(4)
A2
As both the pendulums are identical, their angular frequency for SHM must be same, so their phase difference
will not change with time, hence their phase difference can be given as
l l
2 1 sin 1 sin 1
A2 A1
Example 69: In previous question if second pendulum bob is thrown at
l
velocity v2 at an angle from mean position but on other side of mean l
position. Find the phase difference in the two SHMs now as shown in v2
figure. A B
v1
Sol. In this case still amplitudes of the two SHMs will remain same and are given by equation (1) and (2) but when
we represent the two SHMs on their corresponding circular motions, the position of the particle in circular
motion in second pendulum is now different as shown in figure.
As shown in figure (a) and (b) the initial phases of the
A0
two pendulum bobs are t=0 A
l A1 l 2
1 = sin1 A A
1 1 2 l
B
B0
l
and 2 = sin1 II SHM II SHM
A2
(a) (b)
As for both SHM are same, their phase difference remains constant so it is given as
l l
1 1
= 2 1 = sin + sin A
A2 1

www.physicsashok.in 65
SIMPLE HARMONIC MOTION

Example 70: Two pendulums have time periods T and 5T/4. They starts SHM at the same time from the mean
position. After how many oscillations of the smaller pendulum they will be again in the same phase :
(A) 5 (B) 4 (C) 11 (D) 9
Sol: n1T1 = n2T2
5T
or n1T n 2
4
n1 5
or
n2 4
n1 = 5 Hence, option (A) is correct.

C47: A simple pendulum having a bob of mass m swings with an angular amplitude of 40. Show that the tension
in the string is greater than mg cos 20 when its angular displacement is 20.
Sol. The tension is the string at an angle 20 is
mV 2
T = mg cos 20 +
L
mV 2
or T mg cos 20 = 0
L
T > mg cos 20.

COMPOUND PENDULUM A
This is made whenever a rigid body is hanging freely from a horizontally pivoted
O
axis, as shown in figure. A body of mass m is pivoted at point O through a horizontal
A
axis AA. The body is hanging freely under gravity and in equilibrium position its
l
centre of mass C is vertically below the suspension point O, at a distance l from l
O. c
c
If the body is slightly tilted from its equilibrium position by an angle , mg will
exert a restoring torque on it in opposite direction to restore the equilirbium mg
position. Thus restoring torque on body in dotted position after tilting is
R = mg. l sin [ve sign for restoring nature]
R = mgl [for small , sin ]
If its angular acceleration is , we have
I = mgl [Here I is the momentum of inertia of body about axis AA]
mgl
or = ...(1)
I
Comparing equation (1) with standard differential equation of angular SHM we get
mgl

I
Thus its time period of oscillation is
2 I
T 2
mgl

www.physicsashok.in 66
SIMPLE HARMONIC MOTION

EQUIVALENT LENGTH OF A SIMPLE PENDULUM


We know that the time period of oscillations of a simple pendulum is given by

l
2 ...(3)
g
Similarly weve also discussed that the time period of a compound pendulum is given as

I
2 ...(4)
Mglc

Where I is the moment of inertia of the rigid body about the axis of rotation and lc is the distance of centre of
gravity of body from the suspension point (Axis of rotation). if we consider a simple pendulum of length leq
which has a bob of same mass M as that of rigid body of compound pendulum and has the time period same
as that of the compound pendulum then this length of simple pendulum leq is called Equivalent length of
simple pendulum for the given compound pendulum.
For the time periods of the two pendulums to be equal, we have

l I
2 2 ...(5)
g mglc
If k be the radius of gyration of the rigid body about an axis passing through its centre of mass then the moment
of inertia of the rigid body about point of suspension is given as
I = Mk2 + Mlc2 ...(6)
Now from equation-(5), we have
leq Mk 2 Mlc2

g mglc

k2
or leq lc ...(7)
lc
Equation-(7) gives the equivalent length of simple pendulum for the given compound pendulum. One important
point to be noted here is if in equation-(7) we replace lc by k2/lc, we get
k2 k2
leq
lc k 2 / lc
k2
leq lc ...(8)
lc
Which is same as that of equation-(7). Thus we can say that if the same rigid body, which is suspended from a
point, situated at a distance lc from centre of gravity of body, we suspend if from a point at a distance k2/lc from
centre of gravity of the body and oscillate like a compound pendulum, its equivalent length of simple pendulum
remains same or the time period of oscillation of body remains same. Consider figure, a rigid body is suspended
at a point O through a horizontal axis AA.
Here C is the centre of gravity of the body. If it oscillates then the time period of small oscillations can be given
as

www.physicsashok.in 67
SIMPLE HARMONIC MOTION

k2
lc
lc ...(9)
T 2
g
As discussed in last section we can say that if the same body is suspended either from any point on circular arcs
PQ or RS [as shown in figure(b)] of radius lc with centre at C (circle M1) or any point on the circle of radius
k2/lc with centre at C (circle M2), the time period of small oscillations of the body will remain same.
B
A
O1 Q
P
A 2 lc M2
lc
c M1
k2 /l c
C 3
R S
4
B
(a) (b)
Here we can develop one property of the body when it is used as a compound pendulum. This is, when a
straight line is drawn passing through the centre of gravity of body as in figure (b) a line BB is drawn. There
axis four points on this line (as here points 1, 2, 3 and 4) about which if body is suspended, the time period of
small oscillation of body remains same. This we can also prove graphically as if we plot the time period of
oscillation, the curve looks like as shown in figure.
As shown in firure, if the body is suspended from C, from T
equation-(9), we can see that if lc = 0, time period becomes
, and at lc = k i.e. if the body is suspended from a point at a
distance equal to radius of gyration of body from C, the time T1 1 2 3 4
period of oscillation is minimum and at all other suspension
points the time period is higher and if we draw a horizontal
line in graph at time period T1 which is more than minimum Tmin
period, it cuts the graph at four points as shown, which verifies x
k k
the statement weve discussed earlier. C
lc = k lc = 0 lc = k
From equation-(9) we can also find the value of lc at which this time period has a minimum value by equating
dT
dlc = 0, as

dT 2 1 k2
. 1 2 0
dlc g k 2 lc
2 lc
lc

or lc = k
Which also verifies the experimental result obtained by graph shown in figure.

www.physicsashok.in 68
SIMPLE HARMONIC MOTION

Example 71: A ring of diameter 2m oscillates as a compound pendulum about a horizontal axis passing through a
point at its rim. It oscillates such that its centre move in a plane which is perpendicular to the plane of the ring.
The equivalent length of the simple pendulum is
(A) 2m (B) 4m (C) 1.5 m (D) 3m

I 3mr 2
Sol: T 2
mgl 2mgr

3r
or T 2
2m

l 3r
or 2 2
g 2g

3 3
l r 1 1.5 m Hence option (C) is correct.
2 2
C48: Calculate the time period of a metre scale if it is free to rotate about a horizontal axis passing through its one
end.
Sol. Total length of the metre scale = 1 metre
1
Here, K = (radius of gyration) =
12
l = distance of the C.G. from one end = 0.5 m

K2 l 2
T = 2 = 1.6 sec (nearly).
lg
C49: Calculate the time period of small oscillation of a uniform metre stick, if it is suspended through 10 cm mark.
L 1
Sol. For a metre stick, K2 = L = 1 metre
12 12
K2 1
Hence, 0.208
l 12 0.4

K2 / l l 0.208 0.4
T 2 2 1.56 sec (nearly)
g 9.8
C50: A uniform circular disc of mass m and radius R is suspended from a small hole in the disc. Calculate the
minimum possible time period from small oscillations.

k2 / l l
Sol. T 2 ;
g
T will be minimum when (k2/l + l) is minimum, when k / l l 0 ; or l k R 2
C51: Calculate the length of a simple pendulum whose time period is equal to that of a particular physical pendulum.
L I
Sol. We have T 2 2
g mgl
I
or L
ml

www.physicsashok.in 69
SIMPLE HARMONIC MOTION

Example 72: Consider a pulley of moment of inertia I supporting a block of mass m, tied
to an ideal spring of stiffness k.
I
The block is slightly pulled downward and released. Determine its period of oscillation. R

Sol. In equilibrium
m
mg = T . T = kx0 k
Also, mg = kx0 ...(i)
Let the block be displaced downward by x and moving with velocity dx/dt. Then
1
U k(x x 0 ) mgx
2
2 2 2
1 dx 1 1 dx 1 I dx
K m I 2 or K m
2 dx 2 2 dt 2 R 2 dt
2
1 1 dx I
E k(x x 0 )2 mgx m 2 .
2 2 dt R
Differentiating it and setting to zero,
dE
0
dt
dx dx dx I d2x
k(x + x0) mg m =0
dt dt dt R 2 dt 2

d2x I
k(x + x0) mg 2 m 2 0 ...(ii)
dt R
Using (i) and (ii), we get
d2 x k
2
x
dt m I / R2
Comparing it with standard equation of SHM,
d2x
2
2 x ,
dt
k m I / R2
2 T 2
m I / R2 K
A

Example 73: An L-shaped bar of mass M is pivoted at one of its end so that it can
freely rotate in a vertical plane, as shown in the figure. L
0
(a) Find the value of 0 at equilibrium.
(b) If it is slightly displaced from its equilibrium position, find the frequency of oscillation. B
Sol. (a) Taking B as the origin, the co-ordinates of its c.m. are
L C
ML
A
L
xC 2 2
M M 4 M/2 L c.m.

2 2 yC L
ML C
L B xC M/2
and yC 2 2
M 4

www.physicsashok.in 70
SIMPLE HARMONIC MOTION

L/4 1
tan 0
3L / 4 3
1
0 tan 1
3
(b) The frequency of oscillation for a compound pendulum is
1 mgd
f
2 1 A
where d = distance of the COM from the point of suspension.
I = moment of inertia about the point of suspension. 3L/4

2 2 0
3L L L L/4
L/4
d 10 B
4 4 4
2 C
M L M L M ML2
2 2
L
I L2
2 3 2 12 2 2 3

L
1 Mg 4 10 1 g
f f (2.37)
2 ML2 or
2 L
3

Example 74: A system of two identical rods (Lshaped) of mass m and length l are
l P l
resting on a peg P as shown in the figure. If the system is displaced in its plane by
a small angle , find the period of oscillations :

2l 2 2l 2l l
(A) 2 (B) 2 (C) 2 (D) 3
3g 3g 3g 3g

I
Sol: T 2
mgL
l/2 l/2
ml 2 ml 2 2mgl 2 45 L
Here I 45
3 3 3
G
L
From figure : sin 45
l/2
l
L
2 2

2ml 2 2 2l
T 2 2
l 3g Hence option (C) is correct.
3 mg
2 2

www.physicsashok.in 71
SIMPLE HARMONIC MOTION

Example 75: A solid cylinder is attached to a horizontal massless spring so that it can roll without slipping along a
horizontal surface. The spring constant k is 3 N/m. If the system is released from rest at a point in which the
spring is stretched by 0.25 m, find
(a) the translational kinetic energy and
(b) rotational kinetic energy of the cylinder as it passes the equilibrium position.
(c) Show that under these conditions the centre of mass of the cylinder executes SHM with time period
T 2 (3M / 2k)
Sol. If at any position the stretch in the spring is x and the velocity of centre of mass of the cylinder is V, then as
1 2 1
U kx , KT MV 2
2 2
1 2 1 1
and KR I MV 2 [as L = Mr2 and = V/r]
2 4 2
So total mechanical energy of the system
E = KT + KR + U
1 1 1
i.e. E = MV2 + MV2 + kx2
2 4 2
3 1
i.e. E = MV2 + kx2 ...(1)
4 2
According to given problem V = 0 if x = 0.25
2
1 1 3
so E (3) J
2 4 32
Now at equilibrium position U = 0 [as x = 0].
3 3 1
so MV 2 J, i.e. MV 2 J
4 32 8
So at equilibrium position :
(a) translation KE = (1/2) MV2 = (1/16) J
(b) Rotational KE = (1/4)MV2 = (1/32)J
(c) As in SHM energy is conserved, i.e., dE/dt = 0. From eqn. (1), we have
3 dV 1 dx
M . 2V k . 2x 0
4 dt 2 dt
3 d2x 1 dx dV d 2 x
or M 2 kx as V and 2
4 dt 2 dt dt dt
d2x 2k
or 2
2 x with 2
dt 3M
This is the standard equation of SHM with time period T = (2/).
3M
So here T 2
2k

Example 76: A cylinder of radius r and mass m rests on a curved path of radius R as shown in Fig. Show that the
cylinder can oscillate about the bottom position when displaced and left to itself. Find the period of oscillation.
Assume that the cylinder rolls without slipping.

www.physicsashok.in 72
SIMPLE HARMONIC MOTION

Sol. Restoring torque acting on cylinder after a small displacement , about an axis through point of constant O with
the curved path,
= mg sin r mgr (as for small , sin = )
Angular acceleration of cylinder,
d2

dt 2
r = (R r) R
(R r)
A
r m
r
d 2 (R r) d 2 A
Consequently 2
dt r dt 2
Moment of inertia about contact point,
3mr 2
I
2
3 2 (R r) d 2
mgr mr
2 r dt 2
d 2 g
2
0
or dt 3
(R r)
2
g 3
(R r) 6(R r)
Hence 3 or T 2 2
(R r)
2 g g

Example 77. Two identical cylinders C1 and C2 are placed with their axes 2L
horizontal and in the same horizontal plane. They are rotated uniformly A B
about their once axes in clockwise and anticlockwise direction respectively.
A uniform plank AB of mass M is kept resting of these rotating cylinders. C1 C2
mg
If the plank is displaced slightly from its equilibrium position the demostrate
that it performs simple harmonic oscillations. Calculate the time period of these oscillations. The axes of the
cylinders are separated by a distance 2L and the co-efficient of friction between the plank and the wheels is .
Sol. Let the plank be displaced to the right (x-axis) through a small distance x and released. The normal forces
executed by the two cylinders not equal.
(L+x) (Lx)

A G G B
x
C1 X C2
mg
Let them be N1 and N2. The frictional forces are N1 and N2 to the right and left respectively. Considering the
vertical forces,
N1 + N2 = mg ...(1)
Also, considering the rotational equilibrium of the plank about C.
N1(L + x) = N2 (L x)
www.physicsashok.in 73
SIMPLE HARMONIC MOTION

(N1 N2) L = (N1 + N2)x ...(2)


Hence resultant force acting on the plank along x-axis :
Fx = N1 N2 = (N1 N2)
Fx = Mgx/L [using (1) and (2)]
This is the restoring force, acting towards the centre.
Comparing with standard equation of SHM
Fx = M2x
Here time-period of SHM is

2 L
T 2 .
mg
TORISIONAL PENDULUM
In a Torisional Pendulum an object is suspended from a wire with rigidity coefficient C. If such a wire is twisted
by an angle , due to its elasticity it exerts a restoring torque = C on the twisted object attached to it.
A general torisional pendulum is shown in figure. Here a disc D of radius R and mass M is attached to a stiff
wire whose other end is suspended from ceiling as shown.
From the equilibrium position of this disc if it is twisted by an angle as shown, the wire applies a restoring
torque on it, which is given as
R = C [ve sign, for restoring nature]

If during restoring motion the angular acceleration of disc is , we can write


= I where I is the moment of inertia of disc about its central axis, thus we have
M
R
C D
I = C or = ...(1)
I
Equation-(1) resembles with the basic differential equation of SHM in angular form thus we can state the
angular frequency of this SHM is

C/I ...(2)

2 I
Thus the period of SHM is T 2
C
In the above cases of some pendulums weve discussed, how to find the angular frequency and time period of
a body in SHM. Now we take some similar examples of physical situations in which an object is in SHM.

C52: A circular disc of mass 1.5 kg and radius 15 cm is suspended with the help of a wire, whose one end is fixed
to the centre of the disc. When a torque of 15 102 Nm/rad is applied, then the disc begins to oscillate.
Calculate the time period for small displacement.
1
Sol. M.O.I. of the dis, I Mr 2
2
I Mr 2 (1.5)(0.15) 2
and T 2 2 2 2.107 sec.
C 2C 2 15 102

www.physicsashok.in 74
SIMPLE HARMONIC MOTION

Example 78: Figure shows a torisional pendulum consists of a uniform disc D of mass M
and radius R attached to a this rod of torisional constant C. Find the amplitude and the
energy of small torisional oscillations of the disc, if initially the disc was imparted angular
speed 0.
D
Sol. We know for a torisional pendulum angular frequency of small oscillations is given as
C/I R
Where I = 1/2MR2, thus
2C

MR 2
At it is given that from mean position the disc is imparted as\an angular speed 0, if the angular amplitude of
oscillations of disc is , we have
0 =
2C
or
MR 2
Thus angular amplitude is given as

MR 2
0
2C
For angular SHM the total oscillation energy is given as
1 2 2
ET I [If Umin = 0]
2
Thus here oscillation energy of the disc is given as
2
11 2C MR 2
ET MR 2
2 2 MR 2 2C 0
1
or ET MR 2 02
4

SUPERPOSITION OF PERPENDICULAR SHMs :


Let us consider two SHMs acting on the same particle along perpendicular directions. The resulting motion of
the particle will be obtained by eliminating t from the expressions for x and y. y
Let x = A1 sin t ...(1)
y = A2 sin(t + ) ...(2) P(x, y)

From (2)
y/A2= sin t cos + cos t sin x

y x x2
= cos + 1 sin [using (1)]
A 2 A1 A12

2
y x x2 2
A cos A 2 sin
1
2 A1 1

www.physicsashok.in 75
SIMPLE HARMONIC MOTION

y2 x 2 2 2xy x2
2
2 cos cos sin 2 sin 2
2

A 2 A1 A1A 2 A1

y2 x 2 2xy
2
2 cos sin 2
A 2 A1 A1A 2
This is an equation to an ellipse.
Special cases :
Case (i) = 0. In this case we have

y2 x 2 2xy
2
2 0
A 2 A1 A1A 2

2
y x A2
y x A2
A A 0 A1
2 1 A1
A1
The path is a straight line segment (see figure)
A2
A
tan 2
A1
Y
This motion is SHM with amplitude A12 A22
A2 tan = A2
A1
Case (ii) = . In this case, we have
A1 A1 X
y2 x 2 2xy
2
2 0 A2
A 2 A1 A1A 2
A2
y x
A1
The path is a straight line segment inclined at and where the amplitude

of oscillation is A12 A22 . Y


A2
Case (iii) = /2. In this case, we have (x, y)
A1
x2 x 2
1 A1 X
A 22 A12
A2
This is an ellipse.
Case (iv) = /2, A1 = A2. In this case, we have y

x2 + y2 = A2
A
This is a circle. x

www.physicsashok.in 76
SIMPLE HARMONIC MOTION

Table : Superposition of SHMs


Superposition Superposition features
Same line x = x1 + x2 = A sin (t + );

A= A12 A 22 2A1A 2 cos


A 2 sin
tan = A A cos
1 2

y2 x 2 2xy
Perpendicular lines 2
2 cos sin 2
A 2 A1 A1A 2

Lissajous figure : This figure is the locus of particle in motion under action of mutually perpendicular SHMs
having different frequencies. It is a closed path known as Lissajous figure.

Example 79: Find the amplitude and initial phase of a particle in SHM, whose motion equation is given as
y = A sin t + B cos t
Sol. Here in the given equation we can write
A = R cos ...(12)
and B = R sin ...(13)
Thus the given equation transforms to
y = R sin(t + ) ...(14)
Equation-(14) is a general equation of SHM and here R is the amplitude of given SHM and is the initial
phase of the oscillating particle at t = 0.
Here R is given by squaring and additing equation-(12) & (13)
R = A 2 B2
Initial phse can be given by dividing - (13) & (12)
tan =B/A
= tan1 (B/A)
Note : Equations y = Aei(t + ) and y = A sin t + B cos t, we can also represent the general equation of
SHM.
Example 80. Two simple harmonic motions are represented by the following equations
y1 = 10 sin(/4) (12t + 1)
y2 = 5 (sin 3t + 3 cos 3t)
Here t is in seconds.
Find out the ratio of their amplituds. What are the time periods of the two motions ?
Sol. Given equations are
y1 = 10 sin(/4) (12t + 1) ...(1)
y2 = 5(sin 3t + 3 cos 3t) ...(2)
We recast these in the form of standard equation of SHM which is
y = Asin (t + ) ...(3)
Equation (1) may be written as
y1 = 10sin[(12t/4) + (/4)]

www.physicsashok.in 77
SIMPLE HARMONIC MOTION

y1 = 10sin[(3t + /4)] ...(4)


Comparing eqn. (4) with eqn. (3) we have
Amplitude of first SHM = A1 = 10 cm s1 and 1 = 3
Time period of first motion
T1 = 2/1 = 2/3 = (2/3) s
Eq. (2) may also be written as
y2 = 5sin 3t + 5 3 cos 3t
Let us put
5 = A2 cos ...(5)
and 5 3 = A2 sin ...(6)
Then y2 = A2cos sin 3t + A2sin cos3t
y2 = A2 sin(3t + ) ...(7)
Squaring (5) and (6) and adding, we have

A2 [(5)2 (5 3)2 ] 10 cm
i.e. amplitude of second SHM = 10 cm
and time period of second SHM = T2
2/2 = 2/3 = 2/3 s
Thus, the ratio of amplitudes
A1 : A2 = 1 : 1 and periodic times are
T1 = T2 = (2/3) s

Example 81: The resulting amplitude A and the phase of the vibrations
A A A 3
S = A cos(t) + cos t + cos(t + ) + cos t = A cos (t + ) are _______ and
2 2 4 8 2
________ respectively.
A 3A
Sol. ax A
4 4 y

A A 4A A 3A A
a2 = 2
and ay
2 8 8 8
x
9A 2 9A 2 A a1= A
A a 2x a 2y a3 = 4
16 64
A
2 2
a4 = 8
36A 9A
A a 2x a 2y
64

45A 3 5 A
A=
8 8
ay 1
and tan
ax 2
1
tan 1
2

www.physicsashok.in 78
SIMPLE HARMONIC MOTION

Example 82: A particle is subjected to two mutually perpendicular simple harmonic motions such that its x and y
coordinates are given by

x = 2 sin t ; y = 2 sin t
4
The parth of the particle will be :
(A) an ellipse (B) a straight line (C) a parabola (D) a circle
x
Sol: Hence sin t
2

4 x2
cos t
2

y 2sin t cos 2 cos t sin
4 4
2 2
or y sin t cos t
2 2

x 4 x2
y 2 2
2 2

x 4 x2
or y
2 2
or 2 y x 4 x2
or 2y2 + x2 2 2 xy = 4 x2
or 2y2 + 2x2 2 2 xy = 4
or x2 + y2 2 xy = 2
This is an equation of ellipse. Hence option (A) is correct.

Example 83: The amplitude of the vibrating particle due to superposition of two SHMs,

y1 = sin t and y2 = sin t is :
3
(A) 1 (B) 2 (C) 3 (D) 2
3
Sol: a x 1cos 1 y a1 = 1
3 2
3 3
a y a1 sin 1 /3 x
3 2 2
a2 = 1
2 2
3 3
A a 2x ay2
2 2

9 3
3 Hence option (C) is correct.
4 4

www.physicsashok.in 79
SIMPLE HARMONIC MOTION

Example 84: Two simple harmonic motions y1 = A sin t and y2 = A cos t are supre imposed on a particle of
mass m. The total mechanical energy of the particle is :
1 1
(A) m2A2 (B) m2A2 (C) m2A2 (D) zero
2 4
Sol: y1 = A sin t
y2 = A sin (t + /2)
ax = A
ay = A
y a2 = A
2 2
a = Resultant amplitude = a a
x y

A2 A 2 2 A 90 x
2 2
E = 1/2ma a1 = A

E = 1/2 m2a22 or E mA 2 2

Example 85: Vertical displacement of a plank with a body of mass m on it is varying according to law y = sin t +
3 cos t. The minimum value of for which the mass just breaks off the plank and the moment it occurs first
after t = 0 are given by : (y is positive vertically upwards)
g 2 g 2 g 2 2
(A) , (B) , (C) , (D) 2g ,
2 6 g 2 3 g 2 3 g 3g
Sol: amax = 2A
or g = 2A
or g = 2 A12 A22
2
or g = 2 1 3
or g = 22
g
and if occurs at extreme at upper extreme position.
2
A = sin t + 3 cos t
or 2 = sin t0 + 3 cos t0
or 2 = 2sin(t0 + )
3
But tan 3 60
1 3

or 2 2 sin t 0 or t 0
3 3 2

or t 0
2 3 6
2
or t0 Hence option (A) is correct.
6 6 g
www.physicsashok.in 80
SIMPLE HARMONIC MOTION

THINKING PROBLEMS

1. Which of the following functions are (a) aperiodic (b) periodic but not simple harmonic (c) simple harmonic : (i)
sin 2t (ii) 1 + cos 2t (iii) a sin t + b cos t (iv) sin t + sin 2t + cos 2t (v) sin3 t (v) log (1 + t) (vii)
exp (t) ?
2. Can every oscillatory motion be treated as simple harmonic motion in the limit of small amplitude ?
3. What would happen to the motion of an oscillating system if the sign of the force term in equation F = kx is
changed ?
4. (a) Can a motion be periodic but not oscillatory ?
(b) Can a motion be oscillatory but not simple harmonic ?
If your answer is yes, give an example and if not, explain why ?
5. Can a body have acceleration without having velocity ?
6. Determine whether or not the following quantitie scan be in the same direction for a simple harmonic motion :
(a) displacement and velocity, (b) velocity and acceleration, (c) displacement and acceleration.
7. (a) Can we ever construct a simple pendulum strictly according to its definition ?
(b) Is the motion of a simple pendulum linear SHM or angular SHM ?
8. A girl is swinging in a sitting position. How will the period of swing be affected if :
(a)The girl stands up while swinging ?
(b) Another girl of same mass comes and sits next to her ?
9. A hollow metal sphere is filled with water and a small hole is made at its bottom. It is hanging by a long thread
and is made to oscillate. How will the period of oscillation change if water is allowed to flow through the hole
till the sphere is empty ?
10. The resultant of two simple harmonic motions at right angles and of the same frequency is always a circular
motion. True or false? Explain.

SOLUTION OF THINKING PROBLEMS


1. (a) Functions (vi) log (1 + t) and (vii) exp (t) increase (or decrease) continuously with time and can never
repeat themself so are aperiodic.
(b) Function (ii) (1 + cos 2t), (iv) sin t + sin 2t + cos 2t and (v) sin3 t are periodic [i.e., f(t + T) = f(t)] with
periodicity (/), (2/) and (2/) respectively but not simple harmonic as for these functions (d2y/dt2) is
not y.
(c) Functions (i) sin 2t and (iii) a sin t + b cos t, i.e., (a2 + b2)1/2 sin [t + tan1 (b/a)] are simple harmonic
[with time period (/) and (2/) respectively] as for these (d2y/dt2) y.
2. No, not at all. In the limit of small amplitudes only those oscillatory motions can be treated as simple harmonic
for which the restoring force (or torque) becomes linear. For example, the oscillatory motion of a simple or
spring pendulum or motion of atoms in a molecule becomes simple harmonic in the limit of small amplitude as
restoring force (or torque) becomes linear while in case of oscillatory motion of a ball between two inclined
planes or two perfectly elastic walls, the motion does not become simple harmonic even for vanishingy small
amplitude.
3. If the sign is changed in the force equation, acceleration will not be opposite to displacement and hence the
particle will not oscillate, but will accelerate in the direction of displaement. So the motion will become accelerated
translatory. However, equations of motion cannot be applied to analyse the motion as acceleration (= 2y) is
not constant.
Mathematical analysis shows that in this situation both velocity and displacement will increase exponentially
with time.

www.physicsashok.in 81
SIMPLE HARMONIC MOTION

4. Yes; Uniform circular motion, (b) Yes; When a ball is thrown from a height on a perfectly elastic plane surface
the motion is oscillatory but not simple harmonic as the restoring force F = mg = constt. and not F x]
5. Yes; In SHM at extreme position velocity is zero but acceleration is maximum = 2A.
6. (a) Yes; when the particle is moving from equilibrium position to extreme position, (b) Yes; when the particle is
moving from extreme to equilibrium position, (c) No; as in SHM displacement is always opposite to acceleration.]
7. (a) No, (b) Angular SHM.
8. (a) decreases, (b) unchanged
9. Time period will first increase, reaches a maximum and then will decrease.
10. False. The general motion is elliptical. A circle is a particular case of ellipse when its major axis is equal to its
minor axis. In general, the motion is described by
d 2 y 2 2xy
2
2 cos sin 2
a b ab
When a = b and = /2,
x2 + y2 = a2

ASSERTION-REASON TYPE
A statement of Statement-1 is given and a Corresponding statement of Statement-2 is given just below it of
the statements, mark the correct answer as
(A) If both Statement-1 and Statement-2 are true and Statement-2 is the correct explanation of Statement-1.
(B) If both Statement-1 and Statement-2 are true and Statement-2 is NOT correct explanation of Statement-1.
(C) If Statement-1 is true but Statement-2 is false.
(D) If both Statement-1 and Statement-2 are false.
(E) If Statement-1 is false but Statement-2 is true.

1. Statement-1 : The force acting on a particle moving along xaxis is F = kx + v0t, where k is a constant.
Statement-2 : To an observer moving along x-axis with constant velocity v0, it represents SHM.

2. Statement-1 : A particle executing simple harmonic motion comes to rest at the extreme positions.
Statement-2 : The resultant force on the particle is zero at these positions.

3. Statement-1 : Soldiers are asked to break steps while crossing the bridges.
Statement-2 : The frequency of marching may be equal to the natural frequency of bridge and lead to resonance
which can break the bridge.

4. Statement-1 : The function Y = cos2 t + sin t does not represent a simple harmonic motion.
Statement-2 : Sum of two harmonic function may not be harmonic motion.

5. Statement-1 : A block of mass m is attached to a spring inside a trolley of 2m


mass 2m as shown in figure, all surfaces are smooth. Spring is stretched and m
released. Both trolley and block oscillate simple harmonically with same time
period and same amplitude.
Statement-2 : In absence of external force, centre of mass of system of particles
does not accelerate.

www.physicsashok.in 82
SIMPLE HARMONIC MOTION

MATCH THE COLUMN


V(m/s)
1. A simple harmonic oscillator consists of a block attached to a spring
with k = 200 N/m. The block slides on a frictionless horizontal surface, 2
with equilibrium point x = 0. A graph of the blocks velocity v as a 0.10 0.20
0
function of time t is shown. Correctly match the required information t(s)
in the column I with the values given in the column II. (use 2 = 10) 2
Column-I Column-II
(A) The blocks mass in kg (P) 0.20
(B) The blocks displacement at t = 0 in metres (Q) 200
2
(C) The blocks acceleration at t = 0.10 s in m/s (R) 0.20
(D) The blocks maximum kinetic energy in Joule (S) 4.0

2. Match the following


Column-I Column-II
R
(A) Linear combination of two SHMs (P) T = 2 (R is radius of the earth)
g
(B) y = A sin 1t + A sin(2t + ) (Q) SHM for equal frequencies and amplitude
(C) Time period of a pendulum of infinite length. (R) Superposition may not be a SHM always.
(D) Maximum value of time period of an oscillating (S) amplitude will be 2 A for 1 = 2 and
pendulum. phase difference of /2.

3. Match the following


Column-I Column-II
(A) A constant force acting along the line of (P) The time period
SHM affects
(B) A constant torque acting along the arc of (Q) The frequency
angular SHM affects.
(C) A particle falling on the block executing SHM (R) the mean position
when the later crosses the mean position affects
(D) A particle executing SHM when kept on a (S) The amplitude
uniformly accelerated car affects.

www.physicsashok.in 83
SIMPLE HARMONIC MOTION

LEVEL 1

1. Two particles P and Q describe SHM of same amplitude a, same frequency f along the same straight line. The
maximum distance between the two particles is a 2 . The phase difference between the particle is :
(A) zero (B) /2 (C) /6 (D) /3

2. A rod of length l is in motion such that its ends A and B are moving along x-axis and
y
d B
y-axis respectively. it is given that 2 rad/s always. P is a fixed point on the rod. P
dt l
Let M be the projection of P on x-axis. For the time interval in which changes from
A
0 to /2, choose the correct statement, x
M
(A) The acceleration of M is always directed towards right
(B) M executes SHM
(C) M moves with constant speed (D) M moves with constant acceleration

3. The coefficient of friction between block of mass m and 2m is m = 2 tan . There is no k


friction between block of mass 2m and inclined plane. The maximum amplitude of two m
block system for which there is no relative motion between both the blocks. 2m

k mg sin 3mg sin


(A) g sin (B) (C) (D) None of these
m k k

4. Graph shows the x(t) curves for three experiments involving a particular spring-block system oscillating in
SHM. The kinetic energy of the system is maximum at t = 4 sec. for the situation :
x
1
2

0 4se
c. t(in sec)
3

(A) 1 (B) 2 (C) 3 (D) Same in all


y
A(2, 2)
5. A particle of mass m = 2 kg executes SHM in xy-plane between points A and B

underaction of force F F i F j . Minimum time taken by particle to move
x y x
from A to B is 1 sec. At t = 0 the particle is at x = 2 and y = 2. Then Fx as function
of time t is B(2, 2)
(A) 42 sin t (B) 42 cos t (C) 42 cos t (D) None of these

6. The speed v of a particle moving along a straight line, when it is at a distance (x) from a fixed point of the line
is given by
v2 = 108 9x2
(all quantities are in cgs units) :
(A) the motion is uniformly accelerated along the straight line
(B) the magnitude of the acceleration at a distance 3cm from the point is 27 cm/sec2
(C) the motion is simple harmonic about the given fixed point.
(D) the maximum displacement from the fixed point is 4 cm.

www.physicsashok.in 84
SIMPLE HARMONIC MOTION

7. The time period of an ideal simple pendulum is given by : T = 2 l / g


The time period of actual simple pendulum (T) is slightly different due to small damping (friction). Then
(A) T > T (B) T < T (C) T = T (D) None

8. A small bob of a simple pendulum contains water. Its periodic time is T. During oscillation, the temperature of
surrounding is lowered so that water gets freezed. The new time period is T. Then :
(A) T > T (B) T < T (C) T = T (D) None

9. Two simple pendula A and B are shown in the figure. The phase difference between
A and B is : l l
(A) (B) /2
(C) 0 (D) /4 A B

10. A body executes simple harmonic motion under the action of a force F1 with a time period (4/5) second. If the
force is changed to F2 it executes SHM with a time period (3/5) second. If both the forces F1 and F2 act
simultaneously in the same direction on the body, its time period in second is :
(A) 12/25 (B) 24/25 (C) 35/24 (D) 25/12

11. What should be the displacement of a simple pendulum whose amplitude is A, at which potential energy is
1/4th of the total energy ?
(A) A / 2 (B) A/2 (C) A/4 (D) A / 2 2

12. The potential energy U of a particle is given by U = {20 + (x 4)2} J. Total mechanical energy of the particle
is 36 J. Select the correct alternative(s)
(A) The particle oscillates about point x = 4 m
(B) The amplitude of the particle is 4 m
(C) The kinetic energy of the particle at x = 2 m is 12 J
(D) The motion of the particle is periodic but not simple harmonic
k
L, M
13. A rod of mass M and length L is hinged at its centre of mass so that it can rotate
in a vertical plane. Two springs each of stiffness k are connected at its ends, as Hinge
k
shown in the figure. The time period of SHM is
M M ML M
(A) 2 (B) 2 (C) 2 (D)
6k 3k k 6k

14. A particle moves along the x-axis according to the equation x = A sin2 t
(A) The particle oscillates about the origin (B) The particle oscillates about the point x = A
(C) The particle oscillates with a period T = / (D) The particle oscillates with amplitude A/2

15. In the arrangement shown in figure the pulleys are smooth and massless. The
spring k1 and k2 are massless. The time period of oscillation of the mass m is
m 2m k1
(A) 2 k k (B) 2 k k k2
1 2 1 2

m(k1 k 2 ) m(k1 k 2 ) m
(C) 2 (D) 2
2k1k 2 k1k 2

www.physicsashok.in 85
SIMPLE HARMONIC MOTION


16. The equation of SHM of a particle oscillating along the x-axis is given by x = 3 sin t cm. The acceleration
6
of the particle at t = 1 s is
(A) 1.5 2 cms2 (B) 2.6 2 cms2 (C) 2.6 2 cms2 (D) 1.5 2 cms2

17. For the system shown in the figure, initially the spring is compressed by a
distance a from its natural length and when released, it moves to a distance k
b from its equilibrium position. The dicrease in amplitude for one half - m
cycle (a to +b) is a b
mg 2mg mg
(A) (B) (C) (D) none of these
k k 2k

18. The motion of a particle is given x = A sin t + B cos t. The motion of the particle is
(A) not simple harmonic (B) simple harmonic with amplitude A + B
(C) simple harmonic with amplitude (A + B)/2 (D) simple harmonic with amplitude A 2 B2

19. Two masses m1 and m2 are suspended together by a massless spring of force constant K. When the masses
are in equilibrium is removed without disturbing the system. The amplitude of oscillation is
(A) m2g/K (B) m1g/K (C) (m1+m2)g/K (D) (m1m2)g/K

20. Two bodies M and N of equal masses are suspended from two seperate massless springs of spring constants
k1 and k2 respectively. If the two bodies oscillate vertically such that their maximum velocities are equal, the
ratio of the amplitude of vibration of M to that on N is :
(A) k1/k2 (B) k1 / k 2 (C) k2/k1 (D) k 2 / k1

21. A particle executing SHM while moving from one extremity is found at distance x1, x2, x3 from the centre at the
ends of three successive seconds, the period is
(A) (B) /2 (C) 2/ (D) 1/
1
where = cos (x1 + x3)/2x2

22. One end of a spring of force constant k is fixed to a vertical wall and the other to a body of mass m resting on
a smooth horizontal surface. There is another wall at a distance x0 from the
body. The spring is then compressed by 2x0 and released. The time taken x0
m
to strike the wall is
1 k m 2 m k
(A) (B) (C) (D)
6 m k 3 k 3 m

23. In the above problem the velocity with which the body strikes the other wall is
m 3k k m
(A) x 0 (B) x (C) x (D) 2x 0
k m 0 m 0 k


24. A particle moves in the XY plane according to the equation : r 2 i 4 j sin t. The motion of the particle is:
(A) parabolic (B) circular (C) straight line (D) None of these

www.physicsashok.in 86
SIMPLE HARMONIC MOTION
25. The speed v of a particle moving along a straight line, when it is at a distance x from a fixed point on the line is
given by V = (108 9x2)1/2. All quantities are in SI units.
(A) The motion is uniformly accelerated along the straight line.
(B) The magnitude of acceleration at a distance 3 m from the fixed point is 27 m/s2.
(C) The magnitude of acceleration at a distance 3 m from the fixed point is 9 m/s2.
(D) The maximum displacement of the particle from the fixed point is 4 m.

26. The potential energy U of a particle is given by U = (2.5 X2 + 100) Joule. If the mass of the particle is 0.2 kg,
then :
(A) the motion of the particle is SHM. (B) the mean position is X = 0.
(C) angular frequency of the oscillation is 5 rad/s (D) the time period of oscillation is 1.26 sec.

27. In SHM :
(A) displacement and velocity may be in the same direction.
(B) displacement and velocity can never be in the same direction.
(C) velocity and acceleration may be in the same direction.
(D) displacement and acceleration can never be in the same direction.


28. A particle moves in the XY plane according to the equation r 3 i j cos 5 t . The motion of the particle is:
(A) Along a straight line (B) along an ellipse (C) periodic (D) along a parabola

29. The potential energy of a particle of mass 2 kg, moving along the xaxis is given by U(x) = 16(x2 2x) J, where
x is in metres. Its speed at x = 1 m is 2 ms1 :
(A) The motion of the particle is uniformly accelerated
(B) The motion of the particle is oscillatory from x = 0.5 m to x = 1.5 m.
(C) The motion of the particle is simple harmonic
(D) The period of oscillation of the particle is /2 s.

30. If a SHM is given by y = (sin t + cos t) m, which of the following statements is/are true ?
(A) The amplitude is 1 m (B) The amplitude is 2 m.
(C) Particle starts its motion from y = 1 m. (D) Particle starts its motion from y = 0 m.

31. Three simple harmonic motions in the same direction having the same amplitude a and same period are
superposed. If each differs in phase from the next by 45, then :
(A) the resultant amplitude is (1 + 2 ) a.
(B) the phase of the resultant motion relative to the first is 90.
(C) the energy associated with the resulting motion is (3 + 2 2 ) times the energy associated with any single
motion.
(D) the resulting motion is not simple harmonic.

32. A linear harmonic oscillator of force constant 2 103 N/m and amplitude 0.01 m has a total mechanical energy
of 160 J. Its :
(A) maximum potential energy is 100 J. (B) maximum kinetic energy is 100 J.
(C) maximum potential energy is 160 J. (D) minimum potential energy is zero.

www.physicsashok.in 87
SIMPLE HARMONIC MOTION

33. A uniform cylinder of length L and mass M having cross-sectional area A is suspended, with its length vertical,
from a fixed point by a massless spring, such that it is half-sub merged in a liquid of density at equilibrium
position. When the cylinder is given a small downward push and released it starts oscillating vertically with
small amplitude. If the force constant of the spring is k, the frequency of oscillation of the cylinder is :
1/ 2 1/ 2 1/ 2 1/ 2
1 k Ag 1 k Ag 1 k gL2 1 k Ag
(A) (B) (C) 2 M (D)
2 M 2 M 2 Ag

34. A particle of mass m is executing oscillations about the origin on the x-axis. Its potential energy is
V(x) = k|x|3 where k is a positive constant. If the amplitude of oscillations is a, then its time period T is
(A) proportional to 1/ a (B) independent of a [JEE, 98]
(C) proportional to a (D) proportional to a3/2

35. A particle free to move along the x-axis has potential energy given by
U(x) = k[1 exp(x2)] for < x < +, where k is a positive constant of appropriate dimensions. The
(A) at point away from the origin, the particle is in unstable equilibrium.
(B) for any finite nonzero value of x, there is a force directed away from the origin.
(C) if its total mechanical energy is k/2, it has its minimum kinetic energy at the origin.
(D) for small displacements from x = 0, the motion is simple harmonic. [JEE, 99]

36. Three simple harmonic motions in the same direction having the same amplitude a and same period are
superposed. If each differs in phase from the next by 45, then [JEE, 99]
(A) the resultant amplitude is (1 2) a
(B) the phase of the resultant motion relative to the first is 90
(C) the energy associated with the resulting motion is (3 2 2) times the energy associated with any single motion.
(D) the resulting motion is not simple harmonic.

37. The period of oscillation of simple pendulum of length L suspended from the roof of a vehicle which moves
without friction down an inclined plane of inclination is given by [JEE, 2000]
L L L L
(A) 2 (B) 2 (C) 2 (D) 2
g cos g sin g g tan

38. A particle executes simple harmonic motion between x = A and x = +A. The time taken for it to go from 0 to
A/2 is T1 and to go from A/2 to A is T2. Then [JEE(Scr), 01]
(A) T1 < T2 (B) T1 > T2 (C) T1 = T2 (D) T1 = 2T2

39. A particle is executing SHM according to y = a cos t. Then which of the graphs represents variations of
potential energy : [JEE(Scr), 03]
I II
P.E. P.E.
III IV

i x
(A) (I) & (III) (B) (II) & (IV) (C) (I) & (IV) (D) (II) & (III)

www.physicsashok.in 88
SIMPLE HARMONIC MOTION

40. A block P of mass m is placed on a frictionless horizontal surface. Another block Q k S


of same mass is kept on P and connected to the wall with the help of a spring of Q
spring constant k as shown in the figure. S is the coefficient of friction P
between P and Q. The blocks move together performing SHM of amplitude A.
The maximum value of the friction force between P and Q is [JEE, 04]
(A) kA (B) kA/2 (C) zero (D) Smg

41. A simple pendulum has time period T1. When the period of suspension moves vertically up according to the
equation y = kt2 where k = 1 m/s2 and t is time then the time period of the pendulum is T2 then (T1/T2)2 is
[JEE(Scr), 05]
(A) 5/6 (B) 11/10 (C) 6/5 (D) 5/4

42. Function x = A sin2t + B cos2t + C sin t cos t represents SHM [JEE, 06]
(A) for any value of A, B and C (expect C = 0) (B) if A = B; C = 2B, amplitude = | B 2 |
(C) if A = B; C = 0 (D) if A = B; C = 2B, amplitude = | B |

43. A student performs an experiment for determination of g = (42l/T2)l = 1 m and he commits an error of l. For
the takes the time on n oscillations with the stop watch of least count T and he commits a human error of 0.1
sec. For which of the following data, the measurement of g will be most accurate ? [JEE, 06]
l T n Amplitude of oscillation
(A) 5 mm 0.2 sec 10 5 mm
(B) 5 mm 0.2 sec 20 5 mm
(C) 5 mm 0.1 sec 20 1 mm
(D) 1 mm 0.1 sec 50 1 mm
1
x(cm)

44. The xt graph of a particle undergoing simple harmonic motion is shown t(s)
0
below. the acceleration of the particle at t = 4/3 s is 4 8 12
2 2
(A) 3 / 32 cm/s (B) 2/32 cm/s2 1

(C) 2/32 cm/s2 (D) 3 / 32 2cm/s2 [JEE, 09]

45. A uniform rod of length L and mass M is pivoted at the centre. Its two ends are
attached to two springs of equal spring constants k. The springs are fixed to rigid
supports as shown in the figure, and the rod is free to oscillate in the horizontal
plane. The rod is gently pushed through a small angle in one direction and
released. the frequency of oscillation is [JEE, 09]

1 2k 1 k 1 6k 1 24k
(A) (B) (C) (D)
2 M 2 M 2 M 2 M

46. The mass M shown in the figure oscillates in simple harmonic motion with amplitude A. The amplitude of the
point P is [JEE, 09]
k 1A k2A
(A) k (B) k k1 k2
2 1 M
P
k1A k 2A
(C) k k (D) k k
1 2 1 2

www.physicsashok.in 89
SIMPLE HARMONIC MOTION

PASSAGE
Passage 1
A block of mass m is suspended from one end of light spring as shown. The origin O
is considered at distance equal to natural length of spring from ceiling and vertical
k l
downward direction as +ve Y axis. When the sytem is in equilibrium a bullet of mass
O
m/3 moving in vertical upward direction with velocity v0 strikes the block and embeds
into it. As a result the block (with bullet embedded into it) moves up and starts oscillating.
m Y
Based on above information, answer the following equations :
1. Mark the correct statement(s). m/3 v0
(A) The block-bullet system performs SHM about y = mg/k.
(B) The block-bullet system performs oscillatory motion but not SHM about y = mg/k.
(C) The block-bullet system performs SHM about y = 4mg/3k.
(D) The block-bullet system perform oscillatory motion but not SHM about y = 4mg/3k.
2. The amplitude of oscillation would be
2 2 2 2
4mg mv 20 mv20 mg mv20 mg mv02 4mg
(A) (B) (C) (D)
3k 12k 12k 3k 6k k 6k 3k
3. The time taken by block-bullet system to move from y = mg/k (initial equilibrium position) to y = 0 (natural
length of spring) is [A represents the amplitude of motion]
4m 1 mg 1 4mg 3k 1 mg 1 4mg
(A)
3k cos 3kA cos 3kA (B)
4m cos 3kA cos 3kA

4m 1 4mg 1 mg
(C)
6k sin 3kA sin 3kA (D) None of the above

Passage 2
A platform is executing SHM in a vertical direction, with an amplitude of 5 cm and a frequency of 10/
vibrations/sec. A block is placed on the platform at the lowest point of its path. [Take g = 10 m/s2]
Answer the following questions based on above information :
4. At what point will the block leave the platform ?
(A) 2.5 cm from mean position when acceleration is acting down and velocity is in upward direction.
(B) 2.5 cm from mean position when platform is moving up.
(C) 2.5 cm above mean position when platform is moving down.
(D) 2.5 cm below the mean position.
5. Mark the correct statement(s).
(A) Normal contact force between the platform and block is constant.
(B) As platform approaches mean position from bottom, the normal contact force between the block and
platform increases.
(C) As platform moves up away from mean position, the normal contact force between the block and platform
decreases.
(D) Both (B) and (C) are correct.
6. At what point, the block returns to the platform ?
(A) 1.3 cm above equilibrium position (B) 1.3 cm below equilibrium position
(C) 4.3 cm above equilibrium position (D) 4.3 cm below equilibrium position

www.physicsashok.in 90
SIMPLE HARMONIC MOTION

LEVEL 2

1. A ring of mass m can freely slide on a smooth vertical rod. The ring is symmetrically
attached with two springs, as shown in the fig., each of stiffness k. Each spring
k k
makes an angle with the horizontal. If the ring is slightly displaced vertically, m
determine its time period. rod

2. A block B of mass m = 0.5 kg is attached with upper end of a vertical spring of force A
constant K = 1000 Nm1 as shown in fig. Another identical block A falls from a height h
h = 49.5 cm on the block B and gets stuck with it. The combined body starts to perform B
vertical oscillations.
Calculate amplitude of these oscillations. (g = 10 ms2)

3. One end of an ideal spring is fixed to a wall at origin O and axis of spring is parallel to x-axis. A block of mass
m = 1 kg is attached to free end of the spring and it is performing SHM. Equation of position of the block in
co-ordinate system shown in Figure is x = 10 + 3 sin(10t), where t is in second and x in cm.
Another block of mass M = 3 kg, moving towards the origin with velocity
3 kg
30 cm/sec collides with the block performing SHM at t = 0 and gets 1 kg
stuck to it. Calculate x
(i) new amplitude of oscillations, O
(ii) new equation for position of the combined body and (iii) loss of energy during collision. Neglect friction.

4. Two identical blocks A and B of mass m = 3 kg are attached with ends of an ideal spring of force constant
K = 2000 Nm1 and rest over a smooth horizontal floor. Another identical block C moving with velocity
v0 = 0.6 ms1 as shown in fig. 100 strikes the block A and gets stuck to it. Calculate for subsequent motion
(i) velocity of centre of mass of the system, m m K m
V0
(ii) frequency of oscillations of the system,
(iii) oscillation energy of the system, and (iv) maximum compression of the spring.
K2 K4
5. In the arrangement shown in fig, pulleys are small and light and springs are ideal.
K1, K2, K3 and K4 are force constants of the springs. m
Calculate period of small vertical oscillations of block of mass m.
K1 K3

6. Fig., shows a particle of mass m = 100 gm, attached with four identical springs, B
each of length l = 10 cm. Initial tension in each spring is F0 = 25 newton. Neglecting
gravity, calculate period of small oscillations of the particle along a line perpendicular
to the plane of the figure. p
A C
m

7. In the arrangement shown in Fig., body B is a solid cylinder of radius R = 10 cm


with mass M = 4 kg. It can rotate without friction about a fixed horizontal axis O
A block A of mass m = 2 kg suspended by an inextensible thread is wrapped D
around the cylinder. A horizontal light spring of force constant K = 100 Nm1 R
fixed at one end keeps the system in static equilibrium. Calculate O
(i) initial elongation in the spring and B
(ii) period of small vertical oscillations of the block. (g = 10 ms2)
A

www.physicsashok.in 91
SIMPLE HARMONIC MOTION
L/4
8. A rigid rod of mass m with a ball of mass M attached to the free end is restrained 3/4L
to oscillate in a vertical plane as shown in the figure. Find the natural frequency of M
m
oscillation.
k

9. Find the period of small oscillations in a vertical plane performed by a ball of mass m fixed at the midle of a
horizontally stretched string of length l. The tension of the string is assumed to be constant and equal to
F >> mg.

10. A particle moves along the x-axis according to the law x = a cos t. Find the distance that the particle covers
during the time interval t = 0 to t.

11. Find the amplitude, frequency and epoch of the simple harmonic motion represented by x = 3 sin t + 4 cos t.

12. A helical spring elongates 10 cm when subjected to a tension of 5 N. Find the mass which should be attached
to the bottom of the spring so that when pulled down and released the mass will vibrate twice per second. Find
also its maximum velocity when the amplitude of vibration is 1 cm.

13. Find the frequency of small oscillations of the arrangement illustrated in figure. The
radius of the pulley is R, its moment of inertia relative to the axis of rotation is I, the R
mass of the body is m, and the spring stiffness is k. The mass of the thread and the
spring are negligible, the thread does not slip over the pulley and there is no friction in m
the axis of the pulley.

14. A plank with a body of mass m on it executes simple harmonic motion of cyclic 2
frequency = 11 rad s1 between the levels 1 and 2 separated by a distance 2a a m
as shown in fig. Find : O
(a) the force that the body exerts on the plank as it moves from level 1 to 2 when
a = 4 cm; a
(b) the minimum value of a when the body starts falling behind the plank; 1
(c) the amplitude of oscillation at which the body jumps up to a height h = 50 cm relative to level 1.

15. (a) A block of mass m is tied to one end of a string which passes over a smooth fixed pulley A and under a light
smooth movable pulley B. The other end of the string is attached to the lower end of a spring of spring constant
k2. Find the period of small oscillations of mass m about its equilibrium position.
k2
A B
k2
A
B m
k1 k1 m

Figure (a) Figure (b)


(b) A block of mass m is attached to one end of a light inextensible string passing over a smooth light pulley B
and under another smooth light pulley A as shown in the figure. The other end of a string is fixed to a ceiling. A
and B are held by springs of spring constants k1 and k2. Find angular frequency of small oscillation of the
system.

www.physicsashok.in 92
SIMPLE HARMONIC MOTION

D m
16. Find the angular frequency of motion of block m for small motion of rod BD when we k2
b
neglect the inertial effects of rod BD. Spring constant are k1 and k2. Neglect friction k1
forces. C
B
17. Find the angular frequency for small oscillation of block of mass m in the arrangement shown in the figure.
Neglect the mass of the rod.

k1
m
k2
l/2 l/2
k3

18. A rod AB of mass M is attached as shown below to a spring of constant k. A


small block of mass m is placed on the rod at its free end A. k
m B
(i) if end A is moved down through a small distance d and released, determine the
period of vibration. A b
l
(ii) determine the largest allowable value of d if the block m is to remain at all
times in contact with the rod.
b
C
19. A rod of mass m and length l is hinged at its upper end and carries a block of
k1
mass M at its lower end. Two springs having the spring constants k1 and k2 l
respectively are attached to the rod at the distances b and c from the hinge as k2 m
shown in the figure. Find the angular frequency of vibration for small oscillation of
the system. M
l
20. A uniform broad of length L and weight W is balanced on a fixed semi-circular
cylinder of radius R as shown in the figure. If the plank is tilted slightly from its
equilibrium position, determine the period of its oscillations. r

21. A plank with a body of mass m placed on it start moving straight up according to the law y = a(1 cos t),
where y is the displacement from the initial position, = 11 rad/s. Find :
(a) The time dependence of the force that the body exerts on the plank.
(b) The minimum amplitude of oscillation of the plank at which the body starts falling behind the plank.

22. A particle of mass m free to move in the xy plane is subjected to a force whose components are Fx = kx and
Fy = ky, where k is a constant. The particle is released when t = 0 at the point (2, 3). Prove that the
subsequent motion is simple harmonic along the straight line 2y 3x = 0.

23. In the shown arrangement, both the springs are in their natural lengths. The
coefficient of friction between m2 and m1 is . There is no friction between m1 k2
m2
and the surface. If the blocks are displaced slightly, they together perform k1
m1
simple harmonic motion. Obtaion.
(a) Frequency of such oscillations.
(b) The condition if the frictional force on block m2 is to act in the direction of its displacement from mean position.
(c) If the condition obtained in (b) is met, what can be maximum amplitude of their oscillations ?
www.physicsashok.in 93
SIMPLE HARMONIC MOTION

LEVEL 3

1. State whether true or false


Two simple harmonic motions are represented by the equations
x1 = 5sin [2t + /4] and x2 = 5 2 (sin 2t + cos2t) their amplitudes are in the ratio 1 : 2 [REE,96]

2. A block is kept on a horizontal table. the table is undergoing simple harmonic motion of frequency 3 Hz in a
horizontal plane. The coefficient of static friction between block and the table surface is 0.72. Find the maximum
amplitude of the table at which the block does not slip on the surface. [REE,96]

3. A bob of mass M is attached to the lower end of a vertical string of length L and cross sectional area A. The
Youngs modulus of the material of the string is Y. If the bob executes SHM in the vertical direction, find the
frequency of these oscillations. [REE, 2000]

4. A diatomic molecule has atoms of masses m1 and m2. The potential energy of the molecule for the interatomic
separation r is given by V(r) = A + B(r r0)2, where r0 is the equilibrium separation, and A and B are positive
constants. The atoms are compressed towards each other from their equilibrium positions and released. What
is the vibrational frequency of the molecule ? [REE, 01]

5. Two masses m1 and m2 connected by a light spring of natural length l0 is compressed completely and tied by a
string. This system while moving with a velocity v0 along +ve xaxis pass through the origin at t = 0. At this
position the string snaps. Position of mass m1 at time t is given by the equation.
x1(t) = v0t A(1 cos t)
Calculate : [JEE, 03]
(a) Position of the particle m2 as a function of time.
(b) l0 in terms of A.

6. A small body attached to one end of a vertically hanging spring is performing SHM about its
mean position with angular frequency and amplitude a. If at a height y* from the mean
position the body gets detached from the spring, calculate the value of y* so that the height y0
H attained by the mass is maximum. The body does not interact with the spring during its
m
subsequent motion after detachment. (a2 > g). [JEE, 05]

www.physicsashok.in 94
SIMPLE HARMONIC MOTION

Answer Key

ASSERTION-REASON TYPE
Q. 1 2 3 4 5
Ans. A C A B D

MATCH THE COLUMN

1. [(AR), (BP), (CQ), (DS)]


2. [(AQR), (BQRS), (CP), (DP)]
3. [(ARS), (BRS), (CPQS), (DRS)]

LEVEL 1

Q. 1 2 3 4 5 6 7 8 9 10
Ans. B B C A B BC A B A A
Q. 11 12 13 14 15 16 17 18 19 20
Ans. C A BC A CD D D B D B D
Q. 21 22 23 24 25 26 27 28 29 30
Ans. C C B C B ABC A CD AC BCD B
Q. 31 32 33 34 35 36 37 38 39 40
Ans. AC BC B A D AC A A A B
Q. 41 42 43 44 45 46
Ans. C ABD D D C D

PASSAGE
Q. 1 2 3 4 5 6
Ans. C B A A C D

LEVEL 2
m
1. T 2 2. 5 cm 3. (i) 3 cm; (ii) x = 10 + 3sin(5t + )cm or x = 10 3sin(5t) cm; (iii) 0.135 Joule
2k

5 10 1 1 1 1
4. (i) 0.2 m/sec.; (ii) Hz ; (iii) 0.09 Joule; (iv) 3 10 mm. 5. T 4 m k k k k
1 2 3 4

1 3k ml
6. 0.02 sec. 7. (i) 20 cm ; (ii) 0.4 sec. 8. f 9. T
2 27M 7m F

www.physicsashok.in 95
SIMPLE HARMONIC MOTION

n n
10. S = a[n + 1 cos (t )] when n is even; S = [n + cos (t )] when n is odd.
2 2
1 k
11. A = 5, f = /2 and epoch = 53 12. m = 310 g, vmax = 12.5 cm/sec. 13. f
2 m 1/ R 2

a 2 g
14. (a) N mg 1 g cos t ; (b) a min 2 ; (c) a = 0.2 m.

m(k1 k 2 ) 1 k1k 2 k 1k 2 C 2
15. (a) T 2 ; (b) 16.
k1k 2 2 m(k1 k 2 )
m k1C 2 k 2 (a b) 2
1 4k 2 k 3 l 2 (m M / 3) gl 2 (m M / 3)
17. k1 18. (i) T 2 ; (ii) d max
m k 2 4k 3 kb 2 kb 2

(k1b 2 k 2 c2 ) (M / m / 2)gl 3
19. 20. T 2L gr 21. (a) N = m(gta2 cost); (b) 8 km.
(M m / 3)l 2 4

1 k1 k 2 k1 m1 (m1 m 2 )m 2g
23. (a) 2 m m ; (b) k m ; (c) m k m k
1 2 2 2 1 2 2 1

LEVEL 3

1 YA 1 m1m 2
1. True 2. 2cm 3. 4. f 2 2B(m m )
2 ML 1 2

m1 m1 mg g
5. (a) x 2 v0 t m A(1 cos t) ; (b) l0 m 1 A 6. y 2 a
2 2 k

www.physicsashok.in 96
OPTICS

DHANALAKSHMI NAGAR

NEAR ANNAMAIAH CIRCLE,

TIRUPATI.

PH NO. 9440025125
OPTICS
Review of Concepts
(a) Due to reflection, none of frequency, wavelength and speed of light change.
(B) Law of reflection :
(i) Incident ray, reflected ray and normal on inident point are coplanar.
(ii) The angle of incidence is equal to angle of reflection.

Some important points : In case of plane mirror :


(i) For real object, image is virtual.
(ii) For virtual object, image is real.
(iii) Image size = Object size.
(iv) The converging point of incident beam behaves as object.
(v) If incident beam on optical instrument (mirror, lens etc) is converging in nature, object is
virtual.
(vi) If incident beam on the optical instrument is diverging in nature, the object is real.
(vii) The converging point of reflected or refracted beam from an optical instrument behaves a
image.
(viii) If reflected beam or refracted beam from an optical instrument is converging in nature,
image is real.

P P

n
Real Virtual
Image Object


n
P
(ix) If reflected beam or refracted beam from P
an optical instrument is diverging in nature,
image is virtual. n
Real Virtual
Object Object

(x) For solving the problem, the reference frame is chosen in which optical instrument (mirror,
lens, etc.) is in rest.
(xi) The formation of image and size of image is independent of size of mirror.

(xii) Visual region and intensity of image


depend on size of mirror.

www.physicsashok.in 1
OPTICS
(xiii) If the plane mirror is rotated through an angle , the reflected ray and image is rotated
through an angle 2 in the same sense.
(xiv) If mirror is cut into a number of pieces, then the focal length does not change.
(xv) The minimum height of mirror required to see the full image of a man of height h is h/2.

v sin
Rest
Rest
v sin

v
Object Image
Object v cos v cos Image

(xvi)
(xvii)

Rest

vm vm
2 vm - v 2 vm
v Image
Object Image Object
In rest
(xviii) (xix)

Object vm
v 2 vm + v
Image

(xx)

(C) Number of images formed by combination of two plane mirrors : The images formed by
combination of two plane mirrors are lying on a circle whose centre is at the meeting points of mirrors.
Also, object is lying on that circle.
360
Here, n

where = angle between mirrors.
360
(i) If is even number, the number of image is n1.

360
(ii) If is odd number and object is placed on bisector of angle between mirrors, then

number of images is n1.
360
(iii) If is odd and object is not situated on bisector of angle between mirrors, then the

number of images is euqal to n.
n
(D) Law of reflecteion in vector form :
Let e1 = unit vecotr along incident ray.. n
Let e 2 = unit vector along reflected ray 1 2
n = unit vector along normal on point of incidence
Then, e 2 e1 2 e1 . n n
(e) Spherical mirrors :
(i) It is easy to solve the problems in geometrical optics by the help of co-ordinate sign convention.

www.physicsashok.in 2
OPTICS

y y y y y

x x x x x x x x x x

y y y y y

1 1 1
(ii) The mirror formula is
u
Also, R = 2
These formulae are only aplicable for paraxial rays.
(iii) All distances are measured from optical centre. It means optical centre is taken as origin.
(iv) The sign convention are only applicable in given values.
(v) The transverse magnification is
image size

object size u Sun
1. If object and image both are real, is negative.
2. If object and image both are virtual, is negative D
3. If object is real but image is virtual; is positive. F
4. If object is virtual but image is real, is positive.
d
5. Image of star; moon or distant object is formed
at focus of mirror.
If y = the ddistance of sun or moon from earth.
D = diameter of moon or suns disc.
= focal length of the mirror
d = diameter of the image
= the angle subtended by sun or moons disc
D d
Then tan = =
y
Here, is in radian.

Laws of Refraction
1. (a)The incident ray, the refracted ray and normal on incidence point are coplanar.

1
1

(B) 1 sin 1 2 sin 2 cons tan t . 2


2

1 1
(C) Snells law in vector form:
2
Let, e1 = unit vector along incident ray 2
e 2 = unit vecotr along refracted.

www.physicsashok.in 3
OPTICS
n = unit vector along normal on incidence point.
Then 1 e1 n 2 e 2 n .

Some important points :


(i) The value of absolute retractive index is always greater or equal to one.
(ii) The value of refractive index depends upon material of medium, colour of light and temperature
of medium.
(iii) When temperature increases, refractive index decreases.
(iv) Optical path is defined as product of geometrical path and refractive index.
i.e., optical path = x
(v) For a given time, optical path remains constant.
i.e., 1 x1 2 x 2 cons tan t
dx 1 dx 2
1 2
dt dt
1 c1 2 c 2
2 c1

1 c 2
1
i.e.,
c
(vi) The frequency of light does not depend upon medium.
c1 1 , c2 2
1 c 2 2 1

2 c1 1 Air
2. (a) When observer is in rarer medium and object is in denser medium: Observer
real depth
Then
apparent depth
(B) When object is in air and observer is in
denser medium: Apparent
Real
apparent position depth P Denser medium
depth
real position ()
P
Object

t
1
(C) The shift of object due to slab is x t 1

(i) This formula is ony applicable when observer is in Q
rarer medium. P P
(ii) The object shiftness does not depend upon the
position of object. Object
(iii) Object shiftness takes place in the direction of shiftness
incidence ray. =x

(D) The equivalent rerfractive index of a combination of a number of slabs for normal incidence
ti

t
is i
i
1 t1
Here, t i = t 1 + t 2 + ....... 2 t2

ti t t
1 2
i 1 2

www.physicsashok.in 4
OPTICS
i
ti
(e) The apparent depth due to a number of media is i
.

(f) The lateral shifting due to a slab is d = t sec r sin (i r). r
t
d

Rarer
3. (a) Cricital angle : When a ray passes from denser medium ( 2 ) 1
to rarer medium ( 1 ), then for 90 angle of refraction, the 90
corresponding angle of incidence is critical angle.
Denser
c 2
Mathematically, sin C 1
2

Rarer medium
(B) (i) When angle of incidence is lesser than (1)
critical angle, refraction takes place. the
corresponding deviation is
r

sin 1 2 sin i i for i = C

1
i c i i
(ii) When angle of incidence is greater than
critical angle, total internal reflection takes
place. the corresponding deviation is Denser medium
i<C i=C
2i when i > C (2)

4. The i graph is
(i) Critical angle depends upon colour of light, material of medium,

and temperature of medium.
(ii) Critical angle does not depend upon angle of incidenct. c /2
i
PRISM
A
(a) Deviation produced by prism is i i A .
(B) r + r = A n
(C) For grazing incidence, i = 90 n
(D) For grazing emergence, i = 90
i i
A r r
(e) For not transmitting the ray from prism, > cosec
2
(f) For limiting angle of prism, i = i = 90, the limiting
angle of prism = 2C where C is critical angle. If angle
of prism exceeds the limiting values, then the rays B C
are totally reflected.
(g) i graph for prism:
(h) For minimum deviation,

A
sin m
2 m
(i) i = i and r = r (ii) A
sin i
2

www.physicsashok.in 5
OPTICS
In the case of minimum deviation, ray is passing through prism symmetrically.
(i) For maximum deviation max .
i = 90 or i = 90
(j) For thin prism, 1 A
(k) Angular dispersion, D r A

(l) Angular deviation, y y 1 A
r

(m) dispersive power = 1
y

r
(n) y
2 B 2
1
(o) For dispersion without deviation, y 0
(p) For deviation without dispersion, D 0 x A O C x
Refractive surface formula,
2 1 2 1

u r
Here, = image distance, u = object distance, r = radius of curvature of spherical surface
(a) For plane surface, r =
(B) Transverse magnification,
Im age size 1
m
object size 2u
(C) Refractive surface formula is only applicable for paraxial ray.

LENS
1 1 1
1. Lens formula :
u
(a) Lens formula is only applicable for thin lens.
(B) r = 2 formula is not applicable for lens.
image size
(C) m
object size u
(D) Magnification formula is only applicable when object is perpendicualr to optical axis.
(e) Lens formula and the magnification formula is only applicable when medium on both sides of
lenses are same.

f(+ve)
(f) f(+ve) f(-ve) f(-ve) f(+ve)
f(-ve)
(i) (ii) (iii) (iv) (v) (vi)

(g) This lens formula is applicable for converging as well diverging lens. 1 1
Thin lens makers formula : 2

1 2 1 1 1

1 r1 r2

www.physicsashok.in 6
OPTICS
2. (a) Thin lens formula is only applicable for paraxial ray.
(B) This formula is only applicable when medium on both sides of lens are same.
(C) Intensity is proportional to square of aperture.
(D) When lens is placed in a medium whose refractive index is greater than that of lens. i.e.,
1 2 . Then converging lens behaves as diverging lens and vice versa.
(e) When medium on both sides of lens are not same. Then both focal lengths are not same to
each other.
(f) If a lens is cut along the diameter, focal length does not change.

(g) If lens is cut by a vertical, it converts into two lenses of different


focal lengths.
+
1 1 1
i.e.,
1 2
f1 f2

f
1
2
3
(h) If a lens is made of a number of layers of different refractive
index (shown in figure). + + +
Then number of images of an object formed by the lens is + + + 4
equal to number of different media. 5
6
(i) The minimum distance between real object and image in is 4 .
f1 f2
(j) The equivalent focal elngth of co-axial combination of two d<f1 d<f2
lenses is given by
1 1 1 d
o1 o2
F 1 2 1 2
(k) If a number of lenses are in contact, then
1 1 1 d

F 1 2

1
(l) (i) Power of thin lens, P
F
1
(ii) Power of mirror is P
F
(m) If a lens is silvered at one surface, then the system behaves as an equivalent mirror, whose
power P = 2PL + Pm
2 1 1 1
Here, PL = Power of lens =
r r
1 1 2

1
Pm = Power of silvered surface F
m

r2
Here, Fm , where r2 = radius of silvered surface.
2
1
P
F
Here, F = focal length of equivalent mirror.

www.physicsashok.in 7
OPTICS

ASSERTION & REASION


THE NEXT QUESTIONS REFER TO THE FOLLOWING INSTRUCTIONS
A statement of assertion (A) is given and a Corresponding statement of reason (R) is given just
below it of the statements, mark the correct answer as
(A) If both A and R are true and R is the correct explanation of A.
(B) If both A and R are true but R is not the correct explanation of A.
(C) If A is true but R is false.
(D) If both A and R are false.
(E) If A is false but R is true.

1. Assertion (A) : A single ray cant be isolated from a source however small it may be.
Reason (R) : The concept of single ray is hypothetical.
(A) (B) (C) (D) (E)

2. Assertion (A) : Virtual images can be photographed.


Reason (R) : Rays from virtual images are diverging.
(A) (B) (C) (D) (E)

3. Assertion (A) : Virtual object cant be seen by human eye.


Reason (R) : Virtual object is formed by converging rays.
(A) (B) (C) (D) (E)

4. Assertion (A) : A Convex mirror is used as rear view mirror.


Reason (R) : The Convex mirror always forms virtual, erect and diminished image.
(A) (B) (C) (D) (E)

5. Assertion (A) : The behavior of any lens depends on surrounding medium.


Reason (R) : A lens can be looked upon as a collection of small prism with varying prism angle.
(A) (B) (C) (D) (E)

6. Assertion (A) : Human eye can see virtual object.


Reason (R) : Virtual object is formed by apparent intersection of incident rays.
(A) (B) (C) (D) (E)

7. Assertion (A) : Real image is formed by real intersection of reflected or refracted rays.
Reason (R) : Real image cant be obtained on screen.
(A) (B) (C) (D) (E)

8. Assertion (A) : If a portion of lens or mirror is blocked or removed, then intensity of image reduces.
Reason (R) : As every portion of lens or mirror forms image, hence blocking or removing a portion
will result in intensity reduction.
(A) (B) (C) (D) (E)

9. Assertion (A) : A rectangular glass slab produces no deviation and no dispersion.


Reason (R) : Dispersive power of glass is zero.
(A) (B) (C) (D) (E)
10. Assertion (A) : A double convex lens 1.5 has focal length 10 cm. When immersed in water

4
, its focal length becomes 40 cm.
3
1 l m 1 1
Reason (R) :
f m R1 R2
(A) (B) (C) (D) (E)

www.physicsashok.in 8
OPTICS
11. Assertion (A) : A convex lens of glass 1.5 behave as diverging lens when immersed in carbon
disulphide of higher refractive index 1.65 .
Reason (R) : A diverging lens is thinner in the middle and thicker at the edges.
(A) (B) (C) (D) (E)

12. Assertion (A) : A biconvex lens of focal length 10 cm is split into two equal parts by a plane parallel to
its principal axis. The focal length of each part will be 20 cm.
Reason (R) : The focal length depends on how many parts the convex lens has been split.
(A) (B) (C) (D) (E)

13. Assertion (A) : Radius of curvature of a convex mirror is 20 cm. If a real object is placed at 10 cm
from pole of the mirror, image is formed at infinity.
Reason (R) : When object is placed at focus, its image is formed at infinity.
(A) (B) (C) (D) (E)

14. Assertion (A) : For a prism of refracting angle 60 and refractive index 2 , minimum deviation is
30.
A
Reason (R) : At minimum deviation, r1 r2 30
2
(A) (B) (C) (D) (E)

15. Assertion (A) : Image formed by concave lens is not always virtual.
Reason (R) : Image formed by a lens if the image is formed in the direction of ray of light with
(A) (B) (C) (D) (E)

16. Assertion (A) : Minimum deviation for a given prism does not depend on the refractive index of
the prism.
Reason (R) : Deviation by a prism is given by i1 i2 A and does not have the term .
(A) (B) (C) (D) (E)

Level # 1.
Objective Type Question
Multiple Choice Question with ONE correct answer :
1. Two plane mirrors M1 and M2 are inclined to each other at 70. A ray incident on the mirror M1 at an angle
falls on M2 and is then reflected parallel to M1 for
(A) = 45 (B) = 50 (C) = 55 (D) = 60

2. A light ray is incident on a horizontal plane mirror at an angle of 45.


At what angle should a second plane mirror be placed in order that the
reflected ray finally be reflected horizontally from the second mirror,
as shown in figure.
(A) = 30 (B) = 24
(C) = 22.5 (D) = 67.5

3. A plane mirror is placed in y-z plane facing towards negative x-axis. The mirror is moving parallel to y-
axis with a speed of 5 cm/s. A point object P is moving infront of the mirror with a velocity (3 cm/s) i + (4

cm/s) j + (5 cm/s) k . Find the velocity of image with respect to mirror

(A) (3 cm/s) i + (4 cm/s) j + (5 cm/s) k (B) (3 cm/s) i + (4 cm/s) j + (5 cm/s) k

(C) (3 cm/s) i (4 cm/s) j (5 cm/s) k (D) none of the above.

www.physicsashok.in 9
OPTICS
4. The size of the face of a dancer is 24 cm x 16 cm. Find the minimum size of a plane mirror required to see
the face of dancer completely by
(i) one eyed dancer. (ii) two eyed dancer.
(Distance between the eyes is 4 cm.)
(A) (i) 12 x 8 cm 2 (ii) 12 x 6 cm 2
2
(B) (i) 8 x 10 cm (ii) 12 x 2 cm 2
2
(C) (i) 10 x 12 cm (ii) 9 x 8 cm 2
2
(D) (i) 12 x 2 cm (ii) 6 x 13 cm 2

5. A bullet of mass m 2 is fired from a gun of mass m 1 with horizontal velocity v. A plane mirror is fixed at gun
facing towards bullet. The velocity of the image of bullet formed by the plane mirror with respect to bullet
is
m2 m1 m 2 2 m1 m 2
(A) 1 (B) (C) m1
(D) none of these
m1 m 1

6. In the given figure, the angle of reflection is


(A) 30 (B) 60
(C) 45 (D) none of these.

B
7. Two plane mirrors A and B are aligned parallel to each other, as shown in
figure. A light ray is incident at an angle of 30 at a point just inside one 30
0.2 m
end of A. The plane of incidence coincides with the plane of figure. The
maximum number of times the ray undergoes reflections (excluding the
first one) before it emerges out is A
(A) 28 (B) 30 (C) 32 (D) 34 2 3m

8. A point source of light B is placed at a distance L in front of the centre of a B


d
mirror of width d hung vertically on a wall as shown. A man walks in front of
the mirror along a line parallel to the mirror at a distance 2L from it as shown.
The greatest distance over which he can see the image of the light source
in the mirror is
(A) d (B) d (C) 2d (D) 3d L
2L
9. A plane mirror having a mass m is tied to the free end of a massless
spring of spring constant k. The other end of the spring is attached to
a wall. The spring with the mirror held vertically to the floor can slide
along it smoothly. When the spring is at its natural length, the mirror
is found to be moving at a speed of v cm/s. The separation between
k
the images of a man standing before the mirror, when the mirror is Wall
in its extreme positions
m v m m m
(A) v (B) (C) 2 v (D) 4 v
k 2 k k k
10. Two spherical mirrors M1 and M2, one convex and other concave having same radius
of curvature R are arranged coaxially at a distance 2R (consider their pole separation
to be 2R). A bead of radius a is placed at the pole of the convex mirror a shown. The
ratio of the size of the first three images of the bead is
1 1 1 1 1
(A) 1 : 2 : 3 (B) 1 : : (C) : : (D) 3 : 11 : 41
2 3 3 11 41 M M2
1

11. An object is placed in front of a convex mirror at a distance of 50 cm. A plane mirror is introduced
covering the lower half of the convex mirror. If the distance between the object and the plane mirror is 30
cm, there is no parallax between the images formed by the two mirrors. The radius of curvature of the
convex mirror (in cm) is
(A) 60 (B) 50 (C) 30 (D) 25

www.physicsashok.in 10
OPTICS

12. A rectangular glass slab ABCD of refractive index n1, is immersed in water of refractive index n2 (n1 > n2).
A ray of light is incident at the surface AB of the slab as shown. The maximum value of the angle of
incidence max , such that the ray comes out only from the other surface CD is given by


1 n1
1 n 2
1
1 1
(A) sin n cos sin n (B) sin n1 cos sin n

2 1 2

1 n1 1 n 2
(C) sin (D) sin
n2 n1
13. Two thin slabs of refractive indices 1 and 2 are placed parallel to y
each other in the x-z plane. If the direction of propagation of a ray
in the two media are along the vectors r a i b j and r c i d j 2
1 2
then we have
1
1a 2c
(A) 1a 2b (B) x
a2 b2 c 2 d2
(C) 1 (a2 + b2) = 2 (c2 + d2) (D) none of these

14. A man stands on a glass slab of height and inside an elevator accelerated upwards with a. The
bottom of the slab appears to have shifted with respect to the man by a distance (if the R. I. of the glass
is g )
(A) less then g (B) greater than g (C) equal to g (D) cant be said.

15. A ray of light travels from a medium of refractive index into air. If the angle of incidence at the plane
surface of separation is and the corresponding angle of deviation is D, the variation D with is shown
correctly by the figure.

D D D
D
D1 D2 D1 D1
D2 D2

(0, 0) (0, 0) C C
/2 (0, 0) /2 (0, 0) C /2

(A) (B) (C) (D)

16. An observer can see through a pin hole at the top end of a thin rod of height
h placed as shown in the figure. Beaker height is 3h and its radius is h. Eye
When the beaker is filled with a liquid up to a height 2h, he can see the
lower end of the rod. Then refractive index of the liquid is 3h

3 5 h
3 5
(A) (B) (C) (D) .
2 2 2 2 2h
(Assume that the distance between rod and the wall is negligible).

17. A glass sphere of radius 5 x 102 m has a small bubble 2 x 102 m from its centre. Bubble is viewed along
the diameter of the sphere, from the side on which it lies. If refractive index of glass is 1.5 then how far
from the surface will the bubble appear?
(A) 2.1 cm (B) 2.5 cm (C) 1.5 cm (D) 2.0 cm

www.physicsashok.in 11
OPTICS
18. A ray of light travelling in a transparent medium falls on a surface separating the medium from air at an
angle of incident 45. The ray undergoes total internal reflection. If is the refractive index of the medium
with respect to air, select the possible value(s) of from the following :
(A) 1.3 (B) 1.4 (C) 1.5 (D) 1.7
P
19. A tank contains a transparent liquid of refractive index n the bottom of which
is made of a mirror as shown. An object O lies at a height d above the mirror.
A person P vertically above the object sees O and its image in the mirror and
finds the apparent separation to be O
d
2d 2d d
(A) 2nd (B) (C) (D) 1 n
n 1 n n

20. A fish looks up at the surface of a perfectly smooth lake. The surface appears dark except a circular
area directly above it. The plane angle that this illuminated region subtends is
(A) 48.6 (B) 24.3 (C) 97.2 (D) 12.15

21. A ray of light enters an anisotropic medium from vacuum at grazing incidence. If is the angle made by
the reflected ray inside the medium with the interface and n( ) is the refractive index of the medium
then,
n() n()
(A) n( ) sin = 1 (B) n( ) cos = 1 (C) 1 (D) 1
sin cos

22. The slab of a material of refractive index 2 shown in figure has


a curved surface. APB of radius of curvature 10 cm and a plane
surface CD. On the left of APB is air and on the right of CD is
water with refractive indices as given in figure. An object O is
placed at a distance of 15 cm from pole P as shown. The
distance of the final image of O from P, as viewed from the left is
(A) 20 cm (B) 30 cm (C) 40 cm (D) 50 cm

23. An object is placed at a distance of 12 cm from a convex lens on its principal axis and a virtual image of
certain size is formed. On moving the object 8 cm away from the lens, a real image of the same size as
that of virtual image is formed. The focal length of the lens in cm is
(A) 15 (B) 16 (C) 17 (D) 18

24. A spherical surface of radius of curvature R separates air (refractive index 1.0) from glass (refractive
index 1.5). The centre of curvature is in the glass. A point object P placed in air is found to have a real
image Q in the glass. The line PQ cuts the surface at the point O and PO = OQ. The distance PO is equal
to
(A) 5 R (B) 3 R (C) 2 R (D) 1.5 R

25. A lens of focal length is placed in between an object and screen fixed at a distance D. The lens forms
two real images of object on the screen for two of its different positions, a distance x apart. The two real
images have magnifications m 1 and m 2 respectively (m 1 > m 2).

x D2 x 2
(A) (B) m 1 m 2 = 1 (C) (D) all the above
m1 m 2 4D

www.physicsashok.in 12
OPTICS
26. A liquid of refractive index 1.33 is placed between two identical
plano-convex lenses, with refractive index 1.50. Two possible
arrangement P and Q are shown. The system is
(A) divergent in P, convergent in Q.
(B) convergent in P, divergent in Q.
(C) convergent in both
(D) divergent in both. P Q

27. A lens of refractive index is put in a liquid of refractive index . If the focal length of the lens in air is
, its focal length in liquid will be

1 1
(A) (B) 1 (C) (D)

28. A convergent lens is placed inside a cell filled with a liquid. The lens has a focal length +20 cm when in
air and its material has a refractive index 1.50. If the liquid has a refractive index 1.60, the focal length of
the system
(A) 160 cm (B) 24 cm (D) 80 cm (D) + 80 cm

29. A double convex lens, made of glass of refractive index 1.5, has focal length 6 cm. The radius of curvature
of one surface is double than that of other surface. The small radius of curvature has value
(A) 4.5 cm (B) 6 cm (C) 4 cm (D) 9 cm

30. If the distance between a projector and screen is increased by 1%, then illumination on the screen
decreases by
(A) 1 % (B) 2 % (C) 3 % (D) 4 %

31. A lens forms a sharp image of a real object on a screen. On inserting a parallel slide between the lens
and the screen with its thickness along the principal axis of the lens it is found necessary to shift the
screen parallel to itself d away from the lens for getting image sharply focused on it. If the refractive
index of the glass relative to air is , the thickness of slab is

d d d
(A) (B) d (C) (D) 1
1

32. A thin convex lens in used to form a real image of a bright point object. The
apeture of the lens is small. A graph, shown is obtained by plotting a suitable Y
parameter Y against another suitable parameter x.
If = the focal length of the lens O
u = object distance
X
v = image distance
and Real Positive Convention is used then
-1
(A) (uV) x; (u + V) y (B) (u + V) x; (uV) y
u 1 1
(C) u x; y (D) x; y
v u v

www.physicsashok.in 13
OPTICS
33. Which of the following best represents object distance u vs image distance v graph for a convex lens.
y y y y

(A) (B) (C) (D)

34. Three thin prisms are combined as shown in figure. The refractive indices of the crown glass for red,
yellow and violet rays are r, y and v respectively and those for the flint glass are r, y and v
respectively. The ratio A/A for which there is no net angular dispersion.


2 y 1 y y
y 1 y 2 y . y
(A) y 1 (B) 2 (C) 1
(D)
y y y y y y

35. A point object is placed at distance of 0.3 m from a convex lens of focal length
0.2 m cut into two equal halves, each of which is displaced by 0.0005 m, as shown
in figure. If C1 and C2 be their optical centres then, C1
(A) an image is formed at a distance of 0.6 m from C1 or C2 along principal axis.
(B) two images are formed, one at a distance of 0.6 m and other at a distance O C2
of 1.2 m from C1 or C2 along principal axis.
(C) an image is formed at a distance of 0.12 m from C1 or C2 along principal axis.
(D) two images are formed at a distance of 0.6 m from C1 or C2 along principal axis
at a separation of 0.003 m.

36. A glass prism of refractive index 1.5 is immersed in water (refractive index 4/3). A light beam incident
normally on the face AB is totally reflected to reach on the face BC if (1983)
A B

8 2 8 2
(A) sin (B) sin (C) sin
9 3 9 3

37. A ray of light from a denser medium strike a rarer medium at an angle of incidence i (see Figure). The
reflected and refracted rays make an angle of 90 with each other. The angles of reflection and refraction
are r and r The critical angle is

i r

r'

1 1 1 1
(A) sin tan r (B) sin tan i (C) sin tan r (D) tan sin i

www.physicsashok.in 14
OPTICS
38. Two coherent monochromatic light beams of intensities and 4 are superposed. The maximum and
minimum possible intensities in the resulting beam are
(A) 5 and (B) 5 and 3 (C) 9 and (D) 9 and 3

39. An isosceles prism of angle 120 has a refractive index 1.44. The parallel monochromatic rays enter
the prism parallel to each other in air as shown. The rays emerge from the opposite faces

120

(A) are parallel to each other (B) are diverging


(C) make an angle 2 [sin1 (0.72) 30] with each other
(D) make an angle 2 sin1 (0.72) with each other

40. A diminished image of an object is to be obtained on a screen 1.0 m from it. This can be achieved by
appropriately placing
(A) a concave mirror of suitable focal length (B) a convex mirror of suitable focal length
(C) a convex lens of focal length less than 0.25 m (D) a concave lens of suitable focal length

41. A concave lens of glass, refractive index 1.5 has both surfaces of same radius of curvature R. On
immersion in a medium of refractive index 1.75, it will behave as a
(A) convergent lens of focal length 3.5 R (B) convergent lens of focal length 3.0 R
(C) divergent lens of focal length 3.5 R (D) divergent lens of focal length 3.0 R

42. A hollow double concave lens is made of very thin transparent material. It can be filled with air or either
of two liquids L1 and L2 having refractive indices 1 and 2 respectively 2 1 1 . the lens will
diverge a parallel beam of light if it is filled with
(A) air and placed in air (B) air and immersed in L1
(C) L1 and immersed in L2 (D) L2 and immersed in L1

43. A diverging beam of light from a point source Is having divergence angle , falls symmetrically on a
glass slab as shown. The angles of incidence of the two extreme rays are equal. If the thickness of the
glass slab is t and the refractive index n, then the divergence angle of the emergent beam is

1 1 1 1
(A) zero (B) (C) sin (D) 2sin
n n

44. A ray of light passes through four transparent media with refractive indices 1 , 2 , 3 and 4 as
shown in the figure. the surfaces of all media are parallel. If the emergent ray CD is parallel to the
incident ray AB, we must have

(A) 1 2 (B) 2 3 (C) 3 4 (D) 4 1

www.physicsashok.in 15
OPTICS
45. A given ray of light suffers minimum deviation in an equilateral prism p, Additional prism Q and R of
identical shape and of the same material as P are now added as shown in the figure. The ray will now
suffer

P Q R

(A) greater deviation (B) no deviation


(C) same deviation as before (D) total internal reflection

46. Which one of the following spherical lenses does not exhibit dispersion? The radii of curvature of the
surfaces of the lenses are as given in the diagrams.

(A) R1 R2 (B) R (C) R R (D) R

47. Two plane mirrors A and B are aligned parallel to each other, as shown in the figure A light ray is
incident at an angle 30 at a point just inside one end of A. The plane of incidence coincides with the
plane of the figure. The maximum number of times the ray undergoes reflections (including the first
one) before it emerges out is

(A) 28 (B) 30 (C) 32 (D) 34

48. The size of the image of an object, which is at infinity, as formed by a convex lens of focal length 30 cm
is 2 cm. If a concave lens of focal length 2 0 cm is placed between the convex lens and the image at a
distance of 26 cm from the convex lens, calculate the new size of the image.
(A) 1.25 cm (B) 2.5 cm (C) 1.05 cm (D) 2 cm

49. A ray of light is incident at the glass-water interface at an angle i, it emerges finally parallel to the
surface of water, then the value of g would be

(A) 4 3 sin i (B) 1 sin i (C) 43 (D) 1

50. A beam of white light is incident on glass air interface from glass to air such that green light just suffers
total internal reflection. The colors of the light which will come out to air are
(A) Violet, Indigo, Blue (B) All colors except green
(C) Yellow, Orange, Red (D) White light

www.physicsashok.in 16
OPTICS
51. An equilateral prism is placed on a horizontal surface. A ray pQ is incident onto it. For minimum deviation

R
Q S
P

(A) PQ is horizontal (B) QR is horizontal


(C) RS is horizontal (D) Any one will be horizontal

52. A source emits sound of frequency 600 Hz inside water. The frequency heard in air will be equal to
(velocity of sound in water = 1500 m/s, velocity of sound in air = 300 m/s)
(A) 3000 Hz (B) 120 Hz (C) 600 Hz (D) 6000 Hz

53. A point object is placed at the centre of a glass sphere of radius 6 cm and refractive index 1.5. The
distance of virtual image from the surface is
(A) 6 cm (B) 4 cm (C) 12 cm (D) 9 cm

54. A convex lens is in contact with concave lens. the magnitude of the ratio of their focal length is 2 3.
Their equivalent focal length is 30 cm. What are their individual focal lengths?
(A) 15, 10 (B) 10, 15 (C) 75, 50 (D) 75, 50

55. A container is filled with water 1.33 upto a height


of 33.25 cm. A concave mirror is placed 15 cm above
the water level and the image of an object placed at
the bottom is formed 25 cm below the water level.
Focal length of the mirror is
(A) 15 cm
(B) 20 cm
(C) 18, 31 cm
(D) 10 cm

Multiple Choice Question with ONE or MORE THAN ONE correct answer:

56. A convex lens of focal length 40 cm is in contact with a concave lens of focal length 25 cm. The power
of the combination is
(A) 1.5 dioptres (B) 6.5 dioptres (C) +6.5 diopres (D) +6.67 dioptres

57. A converging lens is used to form an image on a screen. When the upper half of the lens is covered by
an opaque screen
(A) half the image will disappear
(B) complete image will be formed
(C) intensity of the image will increase
(D) intensity of the image will decrease.

58. A short linear object of length b lies along the axis of a concave mirror of focal length f at a distance u
from the pole of the mirror. The size of the image is approximately equal to
12 12 2
u f f u f f
(A) b (B) b (C) b (D) b
f u f f u f

59. A beam of light consisting of red, green and blue colours is incident on a right angled prism, figure. The
refractive indices of the material of the prism for the above red, green and blue wavelengths are 1.39,
1.44 and 1.47 respectively. The prism will

www.physicsashok.in 17
OPTICS

45

(A) separate part of the red colour from the green and blue colours
(B) separate part of the blue colour from the red and green colours
(C) separate all the three colours from one another.
(D) not separate even partially any colour from the other two colours.

60. A thin prism P1 with angle 4 and made from glass of refractive index 1.54 is combined with another
thin prism P2 made from glass of refractive index 1.72 to produce dispersion without deviation. The
angle of the prism P2 is
(A) 5.33 (B) 4 (C) 3 (D) 2.6

61. Two thin convex lenses of focal lengths f 1 and f 2 are separated by a horizontal distance d (where
d f1 d f 2 ) and their centres are displaced by a vertical separation as shown in Figure.

Taking the origin of coordinates O, at the center of the first lens the x and y coordinates of the focal
point of this lens system, for a parallel beam of rays coming from the left, are given by:

f1 f 2 f1 f 2 d
(A) x ,y (B) x ,y
f1 f 2 f1 f 2 d f1 f 2

f1 f 2 d f1 d f1 d f1 f 2 d f1 d
(C) x ,y (D) x ,y0
f1 f 2 d f1 f 2 d f1 f 2 d

62. Which of the following form(s) a virtual and erect image for all positions of the object?
(A) Convex lens (B) Concave lens (C) Convex mirror (D) Concave mirror.

63. A ray of light travelling in a transparent medium falls on a surface separating the medium from air at an
angle of incidence of 45. The ray just undergoes total internal reflection. If n is the refractive index of
the medium with respect to air, select the possible value(s) of n from the following:
(A) 1.3 (B) 1.4 (C) 1.5 (D) 1.6

64. A concave mirror is placed on a horizontal table, with its axis directed vertically upwards. Let O be the
pole of the mirror and C its centre of curvature. A point object is placed at C. It has a real image, also
located at C. If the mirror is now filled with water, the image will be.
(A) real, and will remain at C
(B) real, and located at a point between C and
(C) virtual, and located at a point between C and O
(D) real, and located at a point between C and O

www.physicsashok.in 18
OPTICS

Fill in the blanks:


1. A light wave of frequency 5 x 1014 Hz enters a medium of refractive index 1.5, In the medium the
velocity of the light wave is .................. and its wavelength is ................ (2 Marks)

2. A convex lens A of focal length 20 cm and a concave lens B of focal length 5 cm are kept along the
same axis with a distance d between them. If a parallel beam of light falling on A leaves B as a parallel
beam, then d is equal to .......... cm.

3. A monochromatic beam of light of wavelength 6000 in vacuum enters a medium of refractive index
1.5. In the medium its wavelength is ..., its frequency is ............. (1985)

4. In youngs double-slit experiment, the two slits act as coherent sources of equal amplitude A and of
wavelength . In another experiment with the same set-up the two slits are sources of equal amplitude
A and wavelength , but are incoherent. The ratio of the intensity of light at the midpoint of the
screen in the first case to that in the second case is ................. (1986)

5. A thin lens of refractive index 1.5 has 7a focal length of 15 cm in air. when the lens is placed in a
4
medium of refractive index , its focal length will become ........... cm. (1987)
3

6. A point source emits sound equally in all directions in a non-absorbing medium. Two points P and Q
are at a distance of 9 meters and 25 meters respectively from the source. The ratio of amplitudes of
the waves at P and Q is .................... (1989)

7. A slab of a material of refractive index 2 shown in Figure, has a curved surface APB of radius of
curvature 10 cm and a plane surface CD. On the left of APB is air and on the right of CD is water with
refractive indices as given in the figure.

A C
n1=1.0
n2=2.0 4
n3 =
3
P C O
15 cm

B D
20 cm

An object O is placed at a distance of 15 cm from the pole P as shown. The distance of the final image
of O from P, as viewed from the left is ............... (1991)

f
8. A thin rod of length is placed along the optic axis of a concave mirror of focal length f such that itss
3
image which is real elongated, just touches the rod. The magnification is ............. (1991)

9. A ray of light undergoes deviation of 30 when incident on an equilateral prism of refractive index 2.
The angle made by the ray inside the prism with the base of the prism is ............. (1992)

10. A light of wavelength 6000 in air, enters a medium with refractive index 1.5. Inside the medium its
frequency is ........ Hz and its wavelength is ............... . (1997)

www.physicsashok.in 19
OPTICS

11. Two thin lenses, when in contact, produce a combination of power +10 diopters. When they are 0.25 m
apart, the power reduces to +6 diopters. The focal length of the lenses are ...... m and .... m.
(1997)

12. A ray of light is incident normally on one of the faces of a prism of apex angle 30 and refractive index
2 . The angle of deviation of the ray is .......... degrees. (1997)

True / False :
13. The intensity of light at a distance r from the axis of a long cylindrical source is inversely proportional
to r. (1981)

14. A convex lens of focal length 1 meter and a concave lens of focal length 0.25 meter are kept 0.75
meter apart. A parallel beam of light first passes through the convex lens, then through the concave
lens and moves to a focus 0.5 m away from the concave lens. (1983)

15. A beam of white light passing through a hollow prism give no spectrum. (1983)

16. A parallel beam of white light fall on a combination of a concave and a convex lens, both of the same
material. Their focal lengths are 15 cm and 30 cm respectively for the mean wavelength in white light.
On the other side of the lens system, one sees coloured patterns with violet colour at the outer edge.
(1988)

Table Match
17. Match List I and List II and select the correct answer using the codes given below the lists:
The arrangement shows different lenses made of substance of refractive index 1.5 and kept in air. R1
= 30 cm, R2 = 60 cm. Match the focal lengths
Table I Table II

R1 R2
I. A. 120 cm

R1 R2
II. B. +40 cm

R1 R2
III. C. 40 cm

IV. R1 R2 D. +120 cm

(A) I-A, II-B, III-D, IV-C (B) I-C, II-A, III-B, IV-D
(C) I-D, II-C, III-A, IV-B (D) I-B, II-D, III-C, IV-A

www.physicsashok.in 20
OPTICS

18.
Table I Table II
I. An object is placed at focus before A. Magnification is
a convex mirror
II. An object is placed at the centre of B. Magnification is +0.5
curvature before a concave mirror
III. An object is placed at focus before C. Magnification is +1
a concave mirror.
IV. An object is placed at centre of curvature D. Magnification is 1
before a convex mirror.
(A) I-B, II-D, III-A, IV-E (B) I-A, II-D, III-C, IV-B
(C) I-C, II-B, III-A, IV-E (D) I-B, II-E, III-D, IV-C

19. Match the followings:


Table I Table II
A. Magnification m = +1 (i) Convex mirror
2
B. Magnification m (ii) Plane mirror
3
3
C. Magnification m (iii) Concave mirror
2
(A) A (ii) B (iii) C (i) (B) A (i) B (ii) C (iii)
(C) A (ii) B (i) C (iii) (D) A (iii) B (ii) C (i)

20. For a concave mirror of focal length 20 cm, match the followings:
Table I Table II
Objective distance Nature of image
A. 10 cm (i) Magnified, inverted and real
B. 30 cm (ii) Equal size, inverted and real
C. 40 cm (iii) Smaller, inverted and real
D. 50 cm (iv) Magnified, erect and virtual
(A) A II, B I, C III, D IV (B) A IV, B I, C II, D III
(C) A I, B IV, C III, D II (D) A IV, B I, C III, D II.

PASSAGE TYPE QUESTIONS


THE NEXT QUESTIONS REFER TO THE FOLLOWING PASSAGE
The ciliary muscles of eye control the curvature of the lens in the eye and hence can alter the effective
focal length of the system. When the muscles are fully relaxed, the focal length is maximum. When the
muscles are strained the curvature of lens increases (that means radius of curvature decreases) and
focal length decreases. For a clear vision, the image must be on retina. The image distance is therefore
fixed for clear vision and it equals the distance of retina from eye-lens. It is about 2.5 cm for a grown-
up person.

www.physicsashok.in 21
OPTICS
A person can theoretically have clear vision of objects situated at any large distance from the eye. The
smallest distance at which a person can ciliary muscles are most strained in this position. For an
average grown-up person, minimum distance of object should be around 25 cm.
A person suffering for eye defects uses spectacles (eye glass). The function of lens of spectacles is
to form the image of the objects within the range in which person can see clearly. The image of the
spectacle lens becomes object for eye-lens and whose image is formed on retina. The number of
spectacle lens used for the remedy of eye defect is decided by the power of the lens required and the
number of spectacle-lens equal to the numerical value of the power of lens with sign. For example,
100
power of lens required is +3 D (converging lens of focal length cm), then number of lens will be
3
+3.
For all the calculations required you can use the lens formula and lens makers formula. Assume that
the eye lens is equiconvex lens. Neglect the distance between eye lens and the spectacle lens.

1. Minimum focal length of eye-lens of a normal person is


25 25
(A) 25 cm (B) 2.5 cm (C) cm (D) cm
9 11

2. Maximum focal-length of eye lens of normal person is


25 25
(A) 25 cm (B) 2.5 cm (C) cm (D) cm
9 11

3. A near-sighted man can clearly see object only upto a distance of 100 cm and not beyond this. The
number of the spectacles lens necessary for the remedy of this defect will be
(A) + 1 (B) 1 (C) + 3 (D) 3

4. A far-sighted man cannot see object only upto a distance of 100 cm from his eyes. The number of the
spectacles lens that will make his range of clear vision equal to an average grown up person is
(A) + 1 (B) 1 (C) + 3 (D) 3

5. A person who can see objects clearly from distance 10 cm to , then we can say that the person is
(A) normal sighted person (B) near-sighted person
(C) far-sighted person (D) a person with exceptional eyes having no eye defect.

THE NEXT QUESTIONS REFER TO THE FOLLOWING PASSAGE


Spherical aberration in spherical mirrors is a defect which is due to dependence of focal length ' f '
on angle of incidence ' ' as shown in figure. is given by
R
f R sec
2


Principal axis Pole (P)
C F

where R is radius of curvature of mirror and is the angle of incidence. The rays which are closed to
principal axis are called paraxial rays and the rays far away from principal axis are called marginal
rays. As a result of above dependence different rays are brought to focus at different points and the
image of a point object is not a point.

www.physicsashok.in 22
OPTICS

6. If f p and f m represent the focal length of paraxial and marginal rays respectively, then correct relationship
is:
(A) f p f m (B) f p f m (C) f p f m (D) None

7. If angle of incidence is 60, then focal length of this rays is:


R
(A) R (B) (C) 2R (D) 0
2
8. The total deviation suffered by the ray falling on mirror at an angle of incidence equal to 60 is:
(A) 180 (B) 90 (C) Cant be determined (D) None

9. For paraxial rays, focal length approximately is:


R
(A) R (B) (C) 2R (D) None
2

10. Which of the following statements are correct regarding spherical aberration:
(A) It can be completely eliminated
(B) it cant be completely eliminated but is cant be minimised by allowing either paraxial or marginal
rays to hit the mirror
(C) It is reduced by taking large aperture mirrors
(D) None

THE NEXT QUESTIONS REFER TO THE FOLLOWING PASSAGE


Rainbow is formed during rainy season due to refraction and total internal reflection of rays falling on
suspended water droplets. When rays of the sun fall on rain drops, the rain drops disperse the light and
deviate the different colours by refraction and total internal reflection to the eye of the observer. A
person observing the drops will see different colours of the spectrum at different angles. The rainbow
which results from single total internal reflection is called primary rainbow and secondary rainbow is
formed due to two total internal reflections suffered by rays falling on water drops.

Rays from A
sun
V Secondary
B rainbow
R
le t
v io

C
d
Re

R
Primary
D rainbow
d

V
Re

l et
vio

Figure shows formation of rainbow due to four drops A, B, C and D. The light surffers only one total
linternal reflection in drops C and D forming primary rainbow. Secondary rainbow is formed by drops A
and B where light suffers two total linternal reflections.
11. Rainbow is an arc of:
(A) Circle (B) Ellipse
(C) Parabola (D) Cant be determined

12. The visibility of the rainbow is due to:


(A) All rays (B) Rays undergoing maximum deviation
(C) Rays undergoing minimum deviation (D) None

www.physicsashok.in 23
OPTICS
13. In primary rainbow, the colour of outer edge is:
(A) Blue (B) Violet (C) Red (D) None

14. In secondary rainbow, the colour of inner edge is:


(A) Red (B) Violet (C) Indigo (D) None

15. The necessary condition for the observer to see rainbow is:
(A) Sun, observers eye and the centre of the rainbow arc lie on the same line
(B) Sun, observers eye and the centre of the rainbow arc lie on the different line
(C) From any position provided sun is at the back of the observer (D) None

THE NEXT QUESTIONS REFER TO THE FOLLOWING PASSAGE


The laws governing the behavior of the rays namely rectilinear propagation, laws of reflection and
refraction can be summarised in one fundamental also known as Fermats principle. According to this
principle a ray of light travels from one point to another such that the time taken is at a stationary value
(maximum or minimum). If c is the velocity of light in a vacuum, the velocity in a medium of refractive
c nl
index n is , hence time taken to travel a distance l is . If the light passes through a number of
n c
1 1
media, the total time taken is nl and n dl . If refractive index varies continuously. Now,, nl
c c
is the total optical path, so that Fermats principle states that the path of a ray is such that the optical
path in at a stationary value. This principle is obviously in agreement with the fact that the ray are
straight lines in a homogenous isotropic medium. It is found that it also agrees with the classical laws of
reflection and refraction.

16. If refractive index of a slab varies as 1 x 2 where x is measured from one end, then optical path
length of a slab of thickness 1 m is:
4 3
(A) m (B) m (C) 1 m (D) None
3 4

17. The optical path length followed by ray from point A to B given that laws of reflection are obeyed as
shown in figure is:

A B

P
(A) Maximum (B) Minimum (C) Constant (D) None

18. The optical path length followed by ray from point A to B given that laws of reflection are obeyed as
shown in figure is

A B

(A) Maximum (B) Minimum (C) Constant (D) None

www.physicsashok.in 24
OPTICS
19. The optical path length followed by ray from point A to B given that laws of refraction are obeyed as
shown in figure is

(A) Maximum (B) Minimum (C) Constant (D) None

20. The optical path length followed by ray from point A to B given that laws of refraction are obeyed as
shown in figure is

A B
A and B are focii
of ellipse

(A) Maximum (B) Minimum (C) Constant (D) None

THE NEXT QUESTIONS REFER TO THE FOLLOWING PASSAGE


One hard and stormy night you find yourself lost in the forest when you come upon a small hut.
Entering it you see a crooked old woman in the corner hunched over a crystal ball. You are about to
make a hasty exit when you hear the howl of wolves outside. Taking another look at the gypsy you
decide to take your chances with the wolves, but the door is jammed shut. Resigned to bad situation
your approach her slowly, wondering just what is the focal length of that nifty crystal ball.

21. If the crystal ball is 20 cm in diameter with R.I. = 1.5, the gypsy lady is 1.2 m from the central of ball,
where is the image of the gypsy in focus as you walk towards her?
(A) 6.9 cm from the crystal ball (B) 7.9 cm from the crystal ball
(C) 8.9 cm from the crystal ball (D) None

22. The image of old lady is:


(A) real, inverted an enlarged (B) erect, virtual and small
(C) erect, virtual and magnified (D) real, inverted and diminished

23. The old lady moves the crystal ball closer to her wrinkled old face. At some point you can no longer get
an image of her. At what object distance will there be no change of the gypsy formed?
(A) 10 cm (B) 5 cm (C) 15 cm (D) None

THE NEXT QUESTIONS REFER TO THE FOLLOWING PASSAGE


The table below contains some physical properties of common optical materials. The refractive index
of a material is a measure of the amount by which light is bent upon entering the material. The
transmittance range is the range of wavelengths over which the material is transparent.

www.physicsashok.in 25
OPTICS

Physical Properties of Optical Materials


Material Refractive Transmittance Useful range Chemical
index for light range (m) for prisms resistance
of 0.589 m (m)
Lithium 1.39 0.12-6 2.7-5.5 Poor
fluoride
Calcium 1.43 0.12-12 5-9.4 Good
fluoride
Sodium 1.54 0.3-17 816 Poor
chloride
Quartz 1.54 0.20-3.3 0.20-2.7 Excellent
Potassium 1.56 0.3-29 1528 Poor
bromide
Flint glass* 1.66 0.352.2 0.35-2 Excellent
Cesium iodide 1.79 0.37.0 15-55 Poor
*Flint glass is lead oxide doped quartz.

24. According to the table, which material(s) will transmit light at 25 m


(A) Potassium bromide only (B) Potassium bromide and cesium iodide
(C) Lithium fluoride and cesium iodide (D) Lithium fluoride and flint glass

25. A scientist hypothesizes that any material with poor chemical resistance would have a transmittance
range wider than 10 m . The properties of which of the following materials contradicts this hypothesis
(A) Lithium fluoride (B) Flint glass
(C) Cesium iodide (D) Quartz

26. When light travels from one medium to another, total internal reflection can occur if the first medium
has a higher refractive index than the second. Total internal reflection could occur if light were travelling
from
(A) Lithium fluoride of flint glass
(B) potassium bromide to cesium iodide
(C) quartz to potassium bromide
(D) flint glass to calcium fluoride

27. Based on the information in the table, how is the transmittance range related to the useful prism
range
(A) The transmittance range is always narrower than the useful prism range
(B) The transmittance range is narrower than or equal tot he useful prism range
(C) The tranmittance range increases as the useful prism range decreases
(D) The tranmittance range is wider than and includes within it the useful prism range

28. The addition of lead oxide to pure quartz has the effect of
(A) decreasing the transmittance range and the refractive index
(B) decreasing the transmittance range and increasing the refractive index
(C) increasing the transmittance range and the useful prism range
(D) increasing the transmittance range and decreasing the useful prism range.

THE NEXT QUESTIONS REFER TO THE FOLLOWING PASSAGE


A periscope viewing system is to be used to observe the behavior of primates in a large environmentally
controlled room on the upper floor of a large research facility. The periscope, like those used on

www.physicsashok.in 26
OPTICS
submarines, is essentially a large, folded-path, low power telescope (using prisms to fold the light path).
A sketch of the preliminary design appears below. Like all Newtonian telescopes, it uses a relatively long
focal length objective lens to form a real image in front of the eyepiece lens (of shorter focal length). The
observer looks through the eyepiece lens to see the final image, in the same manner that one would use
a magnifying glass.

The distance between the lenses is approximately equal to the sum of their focal lengths. The eyepiece,
in this design, can be moved forward or back in order to focus on the primates as they move closer to
or further away from the objective lens.

29. The total tube length of the three sections is to be 4 m. The objective lens available has a focal length
of 3 m. What should the focal length of the eyepiece lens be?
(A) 0.75 m (B) 1 m (C) 1.33 m (D) 7 m

30. A visitor seeing the sketch points out an important flaw that will require a design change. what is the
flaw?
(A) The focal length of the eyepiece lens is too short.
(B) The images of the primates will be inverted
(C) The objective lens should be a diverging lens.
(D) The prisms cannot be used in this way.

31. A visitor seeing the sketch points out an important flaw that will require a design change. What is the
flaw?
(A) The focal length of the eyepiece lens is too short.
(B) The images of the primates will be inverted
(C) The objective lens should be a diverging lens.
(D) The prisms cannot be used in this way.

32. What will be the approximate magnification of this periscope?


(A) 0.67x (B) 1x (C) 3x (D) 300x

33. The prisms (454590 prisms) turn the light path through 90 by total internal reflection from the
inside hypotenuse faces of the prisms when the incident angle is 45 as in the sketch.
Can one use crown glass with an index of refraction of 1.52 for the prism?
(A) yes, because the critical angle for crown glass is 47
(B) yes, because the critical angle for crown glass is 41.
(C) No, because the critical angle for crown glass is exactly 47
(D) No, because the critical angle for crown glass is exactly 41.

34. Describe the properties of the image that one sees with this preliminary design
(A) real, inverted, magnified
(B) real, upright, magnified
(C) virtual, upright, same size as object.
(D) virtual, inverted, magnified

35. The telescope is focused on a primate rather far away on the farside of the large habitat. As the
primate moves rather closer to the telescope, what must the observer do to see the primate clearly?
(A) No change, the image remains clear.
(B) Move the eyepiece away from the objective.
(C) Move the eyepiece closer to the objective.
(D) Use an inverting eyepiece because the image flips.

www.physicsashok.in 27
OPTICS
THE NEXT QUESTIONS REFER TO THE FOLLOWING PASSAGE
In the normal human eye, light from an object is refracted by the cornea-lens system at the front of the
eye and produces a real image on the retina at the rear of the eye. For a given eye, its lens-to-retina
distance is fixed at about 2.5 cm. Most of the focusing of an image is done by the cornea, which has a
fixed curvature that is convex with respect to incoming light. The importance of the lens is that its
radius of curvature ccan be changed, allowing the lens to fine-tune the focus.
The lens is surrounded by the ciliary muscle. Contraction of the muscle decreases tension on the
lens. This allows the natural elasticity of the lens to produce an increase in the radius of curvature.
when the muscle relaxes, the lens flattens out, decreasing tis radius of curvature. Unfortunately, the
lens losses elasticity with age and the ability to alter curvature decreases.
The range over which clear vision is possible is bounded by the far point and the near point. In normal
vision the far point is infinity and the near point depends on the radius of curvature of the lens. For
normal eyes the average near point for reading is 25 cm.

AGE, years NEAR POINT, cm


10 7
20 10
30 14
40 22
50 40
60 200
In the myopic (nearsighted) eye, the lens-to-retina length is too long and/or the radius of curvature of
the cornea is too great. This causes rays from an object at infinity to focus at a point in front of the
retina. The far point is closer than normal .A corrective lens will put a virtual image of a distant object
at the position of the actual far point of the eye.
In the hyperopic (farsighted) eye, the lens-to-retina length is too short and/or the radius of curvature
of the cornea is not great enough. This causes rays from an object at infinity to focus at a point behind
the retina. The near point is farther away than normal. A corrective lens will put a virtual image of the
close object at the position of the actual near point.
The relation among the object (o) and image (i) distances from the eye and the focal length (f) of the
lens is given by the lens-distance rule : 1 o 1 i 1 f .
When using this equation, all distances are given in centimeters.
The power of corrective lenses is usually given in units called diopters. Power, in diopters, is the
reciprocal of the focal length in meters : Pdiopter 1 f meter .
By convection
I. Converging lenses have positive focal lengths, and diverging lenses have negative focal lengths.
II. Real images have positive distances from the lens, and virtual images have negative distances from
the lens.

36. The lens system of the myopic eye is best described as


(A) producing too much convergence.
(B) producing too little convergence.
(C) producing too much divergence.
(D) producing too little divergence.

37. An optometrist examined Johns eyes. The farthest object he can clearly focus on with his right eye is
50 cm away. What is the power of the contact lens required to correct the vision in his right eye
(A) 0.50 diopters
(B) 2.0 diopters
(C) +2.0 diopters
(D) +5.0 diopters

www.physicsashok.in 28
OPTICS

38. In a mildly hyperopic eye, the focal length of the eyes natural lens can be corrected by
(A) contracting the muscle and increasing the radius of curvature.
(B) contracting the ciliary muscle and decreasing the radius of curvature
(C) relaxing the ciliary muscle and increasing the radius of curvature.
(D) relaxing the ciliary muscles and decreasing the radius of curvature.

39. Jane must wear a contact lens with a power of +3.00 diopters in one eye to be able to clearly focus on
an object 2.5 cm in front of the eye. Based on the vision in this eye, which of the following is the most
likely age range for Jane
(A) Less than 40 years old
(B) From 40 to 49 years old
(C) From 50 to 59 years old
(D) 60 years or older

40. George wears eyeglasses that sit 2.0 cm in front of his eyes. His incorrect far point is 50 cm. What is
the focal lengths of his eyeglasses
(A) 50 cm (B) +50 cm (C) 48 cm (D) +48 cm

41. In a surgical procedure called radial keratotomy, (RK), a laser is used to flatten the cornea by placing
as series of hairline cuts around the perimeter of the cornea. Which statement is most accurate
(A) RK corrects myopia by decreasing the focal length of the eye.
(B) RK corrects myopia by increasing the focal length of the eye.
(C) RK corrects hyperopia by decreasing the focal length of the eye.
(D) RK corrects hyperopia by increasing the focal length of the eye.

THE NEXT QUESTIONS REFER TO THE FOLLOWING PASSAGE


Student are given a variety of lenses and optics equipment, such as lens holders, lighted object
sources. Optical benches, meter sticks and tapes, image screens, and several examples of commercial
optical equipment, such as microscopes and telescopes. They are to work in an open-ended optics lab
in order to learn the general principles of lenses and the optical devices that can be constructed using
lenses.

42. A student is given a short focal length converging lens and long focal length converging lens. One lens
is placed in a holder. A lighted object is placed 18 cm in front of the lens and it is found that a clear
image can be focused on a screen placed 36 cm behind the lens. what is the focal length of this lens?
(A) 8 cm (B) 12 cm (C) 27 cm (D) 46 cm

43. What magnification is produced by the above lens when the object is 18 cm in front of the lens and the
image is 36 cm behind the lens?
(A) 2x (B) 3x (C) 4x (D) 6x

44. A lighted object is placed 6 cm in front of the second lens, which has a focal length of +24 cm. Where
is the image and which kind of image is it?
(A) 8 cm in front of the lens: a virtual image. (B) 8 cm behind the lens: a real image
(C) 16 cm in front of the lens: a real image (D) 16 cm behind the lens; a virtual image.

45. The 24 cm focal length lens is used as the objective of a simple refracting telescope and a third
converging lens of focal length +8 cm is used as the eyepiece. What is the magnification of this simple
refractor?
(A) 0.6x (B) 3x (C) 4x (D) 6x

46. A commercial microscope is examined by the student. The objective is marked 20x and the eyepiece
is marked 10x. what power objective should replace the above objective so that the microscopes
magnification will be 400x
(A) 5x (B) 10x (C) 40x (D) 100x

www.physicsashok.in 29
OPTICS

47. A lighted object is placed 24 cm in front of a +12 cm focal lengths lens. The image formed by this lens
is the object for a second lens of +24 cm focal length. The second lens is placed 72 cm behind the first
lens. where is the final image with respect to the second lens?
(A) 24 cm in front of # 2 (B) 24 cm behind # 2
(C) 36 cm in front of # 2 (D) 48 cm behind # 2

48. A lens of focal length +24 cm is used to view an object placed 12 cm in front of the lens. The object is
5 cm tall. How tall is the image?
(A) 2.5 cm (B) 3.3 cm (C) 7.5 cm (D) 10 cm

49. A diverging lens of focal length 24 cm is now used with the object 12 cm in front of the lens. How tall
is the image if the object is 5 cm tall?
(A) 2.5 cm (B) 3.3 cm (C) 8 cm (D) 10 cm

50. A near sighted student cannot see objects clearly unless they are as close as 80 cm (his far-point).
The image that he sees through his new contact lens is a virtual image because he looks through the
lens to see the image. what focal length lenses does he need in order to see very distance objects,
such as the starts?
(A) 20 cm (B) 30 cm (C) 4 cm (D) 25 cm

THE NEXT QUESTIONS REFER TO THE FOLLOWING PASSAGE


The phenomenon of refraction has long intrigued scientists and was actually used to corroborate
one of the major mysteries of early science: the determination of the speed of light.
The refractive index of a transparent material irrelated to a number of the physical properties of light.
In terms of velocity, the refractive index represents the ratio of the velocity of light in a vacuum to its
velocity in the material. From this ratio, it can be seen that light is retarded when it passes through
most types of matter. It is worth noting that prisms break up white light into the seven colors of the
rainbow because each color has a slightly different velocity in the medium.
Snells law allows one to follow the behavior of light in terms of its path when moving from a material
of one refractive index to another with the same, or different refractive index. It is given by:
n1 sin 1 n2 sin 2 , where I refers to the first medium through which the ray passes, 2 refers to the
second medium, and the angles refer to the angle of incidence in the first medium 1 and the angle
of refraction in the second 2 .

A ship went out on a search for a sunken treasure chest. In order to locate the chest, they shone a
beam of light down into the water using a high intensity white light source as shown in Figure. The
refractive index for sea water is 1.33 while that for air is 1.00.

51. From the information in the passage, how would you expect the speed of light in air to compare with
the speed of light in a vacuum (which is given by c)?
(A) It would be the same (=c) (B) It would be greater than c.
(C) It would be less than c. (D) This cannot be determined from the information given.

www.physicsashok.in 30
OPTICS
52. Using the information in the passage, what must the approximate value of 2 be such that it hit the
chest as shown in Figure?
(A) 15.2 (B) 30.4 (C) 45.6 (D) 63.4

53. How does the refractive index in water light compare with that of red light given that violet light travels
more slowly in water than red light?
(A) nviolet nred (B) nviolet nred

(C) nviolet nred (D) This depends on the relative speeds of the different colors in a vacuum.

54. Total internal reflection first occurs when a beam of light travels from one medium to another medium
which has a smaller refractive index at such an angle of incidence that the angle of refraction is 90.
This angle of incidence is called the critical angle. What is the value of the sine of this angle when the
ray moves from water towards air?
(A) 2 (B) 0.75 (C) 0.50 (D) 0

55. What would happen to the critical angle, in the previous question, if the beam of light was travelling
from water to a substance with a greater refractive index than air, but a lower refractive index than
water?
(A) It would increase (B) It would decrease
(C) It would remain the same (D) Total internal reflection would not be possible.

56. Which of the following would you expect to remain constant when light travels from one medium to
another and the media differ in their refractive indices?
(A) Velocity (B) Frequency (C) Wavelength (D) Intensity.

THE NEXT QUESTIONS REFER TO THE FOLLOWING PASSAGE


The invention of the compound microscope by Jansen in the late 1500s truly revolutionized the world
of science, particularly the field of cellular and molecular biology. The discovery of the cell as the
fundamental unit of living organisms and the insight into the bacterial world are two of the contributions
of this instrument to science.
It is unseemly that such a relatively simplistic apparantus took generations to be developed. Its main
component are two convex lenses: one acts as the main magnifying lens and is referred to as the
objective, and another lens called the eyepiece. The two lenses act independently of each other when
bending light rays. The actual lens set-up depicted in Figure.

Light from the object (O) first passes thought he objective and an enlarged, inverted first image is
formed. The eyepiece then magnifies this image. Usually the magnification of the eyepiece is fixed
(either x 10 or x 10) and three rotating objective lenses are used : x 10, x 40 and x 60. The most recent
development in microscope technology is the electron microscope which uses a beam of electrons
instead of light. Photographic film must be used otherwise no image would be formed on the retina.
This microscope has a resolution about a hundred times that of the light microscope.

57. Based on the passage, what type of image would have to be produced by the objective magnification?
(A) Either virtual or real (B) Virtual
(C) Real (D) It depends on the focal length of the lens.

www.physicsashok.in 31
OPTICS

58. Where would the first image have to be produced by the objective relative to the eyepiece such that a
second, enlarged image would be generated on the same side of the eyepiece as the first image (first
image distance = d1)?
(A) di Fe (B) di Fe (C) 2 Fe d i Fe (D) di 2 Fe

59. Two compound microscopes A and B were compared. Both had objectives and eyepieces with the
same magnification but A gave an overall magnification that was greater than that of B. Which of the
following is a plausible explanation?
(A) The distance between objective and eyepiece in A is greater than the corresponding distance in B.
(B) The distance between objective and eyepiece in A is less than the corresponding distance is B.
(C) The eyepiece and objective positions were reversed in A.
(D) The eyepiece and objective positions were reversed in B.

60. A student attempted to make a compound microscope. However, when she tried to view an object
through the apparatus, no image was seen. Which of the following could explain the mishap?
I. The object distance = focal length of objective.
II. The object distance for eyepiece lens as her eyepiece.
III. The student used a diverging lens as her eyepiece.
IV. The student used a converging lens as her objective
(A) I, II, III and IV (B) I, II, III
(C) I, II, IV (D) II, III, IV

61. The magnification of the eyepiece of a compound microscope is x15. The image height is 25 mm and
the magnification of the objective is x40. What is the object height?
(A) 1.67 mm (B) 0.60 mm (C) 0.38 mm (D) 0.04 mm

62. What is the refractive power of an objective lens with a focal length of 0.50 cm?
(A) 0.2 diopters (B) 2.0 diopters (C) 20 diopters (D) 200

THE NEXT QUESTIONS REFER TO THE FOLLOWING PASSAGE


Magnification by a lens of an object at distance 10 cm from it is 2. Now a second lens is placed
exactly at the same position where first was kept, without changing the distance object and lens. The
magnification by this second lens is 3.

63. Now both the lenses are kept in contact at the same place. What will be the new magnification.
13 12 6 5
(A) (B) (C) (D)
5 7 11 7

64. What is the focal length of the combination when both lenses are in contact.
60 5 12 13
(A) cm (B) cm (C) cm (D) cm
17 17 7 9

THE NEXT QUESTIONS REFER TO THE FOLLOWING PASSAGE


In the case of convex lens, when object is moved from f to 2f, its image is real, inverted and magnified.
It moves from f to infinity on other side.

65. Focal length of a convex lens is 10 cm. When the object is moved from 15 cm to 25 cm, the magnitude
of linear magnifications.
(A) will increase (B) will decrease
(C) will first increase then decrease (D) will first decrease than increase.

www.physicsashok.in 32
OPTICS
66. Image of object AB shown in figure will be like:
B

2F A F

A' A'
(A) F 2F (B)
F 2F
B' B'

A' A'
(C) F 2F
(D) F 2F
B' B'

THE NEXT QUESTIONS REFER TO THE FOLLOWING PASSAGE


Figure shows a simplified model of the eye that is based on the assumption that all of the refraction
of entering light occurs at the cornea. The cornea is a converging lens located at the outer surface of
the eye with fixed focal length approximately equal to 2 cm. Parallel light rays coming from a very
distant object are refracted by the cornea to produce a focused image on the retina. The retina then
transmits electrical impulse along the optic nerve to the brain.
cornea
retina

Two common defects of vision are myopia and hyperopia. Myopia, sometimes referred to as
nearsightedness, occurs when the cornea focuses the image of a distance object in front of the retina.
Hyperopia, sometimes referred to as farsightedness, occurs when the cornea focuses the image of a
nearby object behind the retina. Both of these problems can be corrected by introducing another lens
in front of the eye so that the two lens system produces a focused image on the retina. If an object is
so far away from the lens system that its distance may be taken as infinite, then the following relationship
1 1 1
holds: , where f is the focal length of the cornea, f , is the focal length of the correcting
f c fl x v c l

lens, x is the distance from the correcting lens to the cornea, and v is the image distance measured
from the cornea. (Note : The index of refraction is 1.0 for air and 1.5 for glass).

67. How far away should the retina be from the cornea for normal vision?
(A) 0.5 cm (B) 1.0 cm (C) 2.0 cm (D) 4.0 cm

68. For a distant object, the image produced by the cornea is:
(A) real and inverted (B) real and upright
(C) virtual and inverted (D) virtual and upright.

69. What kind of lens would be suitable to correct myopia and hyperopia respectively? (Note : Assume that
the correcting lens is at the focal point of the cornea so that x f c .)
(A) Converging, converging (B) Converging, diverging
(C) Diverging, diverging (D) Diverging, converging

70. The focal length of a womans cornea is 1.8 cm, and she wears a correcting lens with a focal length of
16.5 cm at a distance x = 1.5 cm from her cornea. What is the image distance v measured from the
cornea for a distant object?
(A) 1.0 cm (B) 1.5 cm (C) 2.0 cm (D) 2.5 cm

www.physicsashok.in 33
OPTICS

71. In the case of contact lens, the cornea and the correcting lens are actually touching and act together
as a single lens. If the focal length of both the cornea and the contact lens are doubled, then the image
distance v for a distant object would:
(A) be 1/4 the old value (B) be 1/2 the old value.
(C) be the same as the old value (D) be twice the old value.

Level # 2
A
1. In what direction should a beam of light be sent from point A (Figure) contained in B

a mirror box for it to fall onto point B after being reflected once from all four walls?
Point A and B are in one plane perpendicular to the walls of the box (i.e., in the
plane of the drawing).

2. A concave mirror has the form of a hemisphere with a radius R = 55 cm. A thin layer of an unknown
transparent liquid is poured into this mirror, and it was found that the given optical system produces, with
the source in a certain position two real image, one of which (formed by direct reflection) coincide with
source and the other is at a distance of = 30 cm from it. Find the refractive index of the liquid.

3. A point source of light S is placed on the major optical axis of concave mirror at a distance of 60 cm. At
what distance from the concave mirror should a flat mirror be placed for the rays to converge again at
the point S having been reflected from the concave mirror and then from the flat one? Will the position of
the point where the rays meet change if they are first reflected from the flat mirror? The radius of the
concave mirror is 80 cm.

4. A pile 4 m high driven into the bottom of a lake is 1 m above the water.
Determine the length of the shadow of the pile on the bottom of the lake if the sun rays make an angle of
45 with the water surface. The refractive index of water is 4/3.

5. In figure, a fish water watches a fish through a 3.0 cm thick glass wall of a fish tank. The watcher is in
level with the fish; the index of refraction of the glass is 8/5 and that of the water is 4/3.

8.0 cm 3.0 cm 6.8 cm

Observer

Wall Water

(a) To the fish, how far away does the watcher appear to be?
(B) To the watcher, how far away does the fish appear to be?

6. A hollow sphere of glass of refractive index has a small mark on its interior surface which is observed
from a point outside the sphere on the side opposite the center. The inner cavity is concentric with
external surface and the thickness of the glass is every where equal where equal to the radius of the
1 R
inner surface. Prove that the mark will appear nearer than it really is, by a distance 3 1 , where R is

the radius of the inner surface.

www.physicsashok.in 34
OPTICS
7. A long rectangular slab of transparent medium of thickness d Y
is placed on a table with length parallel to the x-axis and width
parallel to the y-axis.
A ray of light is travelling along y-axis at origin. the refractive A
0
index of the medium varies as , where 0 and d
1 x r
r (> 1) are constants. The refractive index of air is 1. medium
(a) Determine the x-coordinate of the point A, where the ray X
O
intersects the upper surface of the slab-air boundary.
(B) Write down the refractive index of the medium at A.
(C) Indicate the subsequent path of the ray in air.

8. A man of height 2.0 m is standing on level road where because of temperature variation the refractive
index of air is varying as 1 ay , where y is height from road. If a = 2.0 x 106 m 1. Then find distant
point that he can see on the road.

9. A portion of straight glass rod of diameter 4 cm and refractive index 1.5 is


bent into an arc of circle of radius R. A parallel beam of light is incident on
it as shown in the figure. Find the smallest value of R which permits all the
light to pass around the arc. R

Observer
10. A glass sphere has a radius of 5.0 cm and a refractive index of
1.6. A paperweight is constructed by slicing through the sphere
on a plane that is 2.0 cm from the centre of the sphere and
perpendicular to a radius of the sphere that passes through the 8.0 cm
center of the circle formed by the intersection of the plane and
the sphere. The paperweight is placed on a table and viewed 3.0 cm
directly above by an observer who is 8.0 cm from the table
top as shown in figure. when viewed through the paperweight, 5.
0
how far away does the tabletop appear to the observer? cm

11. A ray of light is incident on a composite slab at a angle of


incidence i as shown in the figure. Find the lateral shift x
of the ray when it comes out from the otherside.

12. A prism of apex angle A is made up of a material of refractive i


index . The refractive indices of the mediums on the left and
right sides are 1 and 2 respectively. A ray of light is incident
from the side of medium of refractive index 1 at an angle i and
comes out from the other side as shown in the figure. Find the
1 2
angle of deviation.

www.physicsashok.in 35
OPTICS

13. A hemisphere of radius a/2 and made up of a material of variable refractive


index is placed with its base centre O at the origin as shown in the figure.
a
The refractive index of the material of the hemisphere varies as .
ax
A ray of light is incident at the point O at an angle with the normal in
the xy plane and comes out through a point P on its curved surface.
Find the coordinate of the point P if 0 .

14. A ray of light is incident on the sphere of radius R and refractive index as
shown in the figure. The incident ray is parallel to a horizontal diameter and b
the distance between the incident ray and the horizontal diameter is b. Find R
the angle of deviation suffered by the ray.. u

15. An intense beam parallel to the principal axis is incident on


a convex lens. Multiple extra images F 1, F 2, ...... are formed Principal
due to feeble internal reflections, called flare spots as shown axis
in the figure. The radius of curvature of the lens is 30 cm and F1 F2 F0
60 cm and the refractive index is 1.5.
Find the position of the first flare spot.

16. The image of the object shown in the figure is formed at the bottom of the tray filled with water. From the
details given in the figure, calculate the value of h.

= 30 cm

/4

36 cm 1m 85 cm

17. In the given figure there are two thin lenses of same focal
length arranged with their principal axes inclined at an
angle . The separation between the optical centers of the

lenses is 2 . A point object lies on the principal axis of the X
O
convex lens at a large distance to the left of convex lens.
(a) Find the coordinates of the final image formed by the
system of lenses taking O as the origin of coordinate axes, and
(B) Draw the ray diagram.
2

www.physicsashok.in 36
OPTICS

18. (a) A prism has refracting angle equal to /2. It is given that is the angle of minimum deviation and
is the deviation of the ray entering at grazing incidence. Prove that sin = sin2 and cos = cos
(b) A ray of light passes through a prism in a principal plane the deviation being equal to angle of
2 1
incidence which is equal to 2 . It is given that is the angle of prism. Show that cos

8 2 1
where is the refractive index of the material of prism.

19. A thin flat glass plate is placed in front of a convex mirror.


At what distance b from the plate should a point source of
light S be placed so that its image produced by the rays
reflected from the front surface of the plate coincides with
the image formed by the rays reflected from the mirror? S
The focal length of the mirror is = 20 cm and the
distance from the plate to the mirror a = 5 cm. How can
the coincidence of the images be established by direct
observation?
a b

20. A concave mirror forms the real image of a point source lying on the
optical axis at a distance of 50 cm from the mirror. The focal length
of the mirror is 25 cm. The mirror is cut in two and its halves are drawn 1 cm
a distance of 1 cm apart in a direction perpendicular to the optical axis.
How will the images formed by the halves of the mirror be arranged?

21. A glass hemisphere of radius 10 cm and = 1.5 is silvered over tis curved surface. There is an air
bubble in the glass 5 cms from the plane surface along the axis. Find the position of the images of this
bubble seen by observer looking along the axis into the flat surface of the hemisphere.

22. The height of a candle flame is 5 cm. A lens produces an image of this flame 15 cm high on a screen.
Without touching the lens, the candle is moved over a distance of = 1.5 cm away from the lens, and a
sharp image of the flame 10 cm high is obtained again after shifting the screen. Determine the main
focal length of the lens.

23. A thin converging lens of focal length is moved between a candle and a screen. The distance between
the candle and the screen is d (> 4 ). Show that for two different positions of the lens, two different
images can be obtained on the screen. If the ratio of dimensions of the image is , find the value of (
+ 1/ ).

24. Three convergent thin lenses of focal lengths 4a, a and 4a respectively are placed in order along the
axis so that the distance between consecutive lenses is 4a. Prove that this combination simply inverts
every small object on the axis without change of magnitude or position.

25. A converging bundle of light rays in the shape in the shape of a cone with the vertex angle of 40 falls on
a circular diaphragm of 20 cm diameter. A lens with a focal power of 5 diopters is fixed in the diaphragm.
What will the new cone angle be?

26. A ray of light is incident on the spherical surface of radius of curvature R


as shown in the figure. Therefractive index on the right side of spherical
surface is . The medium ont he left side of the spherical surface is air..
The distance of the incident ray from the axis of the spherical surface is
b. After refraction the ray intersects the axis at a point. F. Find the
distance of the point F from the pole O.

www.physicsashok.in 37
OPTICS
10 cm 10 cm
27. Consider an arrangement of two equibi convex lenses of focal length
in air 10 cm. The refractive index of the glass of which the lenses are
g g
between the made is g = 3/2 and the refractive index of water filling
the space two lenses is w = 4/3. A small object O is placed on the axis O AIR AIR
at a distance of 10 cm from the first lens in air as shown in the figure. The
distance of separation between the two lenses is 10 cm. Find the
position and magnification of the final image.
A
28. A thin convex lens of focal length 1m is cut into three parts A, B and
C along the diameter. The thickness of the middle layer C is 1 cm. The C 1 cm
middle layer is now removed and the two parts A nad B are put together
B
to form a composite lens. Then the part C is also placed infront of this
composite lens symmetrically as shown in the figure. A paraxial beam
of light is incident along tyhe axis of the part C. Find the distance A
between the two images formed. C
B

29. An equi biconvex lens of focal length 10 cm in AIR and made up


of material of refractive index 3/2 is polished on one side. Another 10 cm 10 cm
identical lens (not polished) is placed infront of the polished lens
at a distance of 10 cm as shown in the figure. The space between
the two lenses is filled with a liquid of refractive index 4/3. An object
O is placed infront of the unpolished lens at a distance of 10 cm.
Find the final position of the image.
B

60
30. Consider an equilateral prism ABC as shown in the figure. A ray
of light is incident on the face AB and gets transmitted into the
prism. Then total internal reflection takes place at the face BC
and the ray comes out of prism through the face AC. The total
angle of deviation is 120. Find the refractive index of the
material of the prism.
60 60
A C

Level # 3
1. An object is placed 21 cm in front of a concave mirror of radius of curvature 10 cm. A glass slab of
thickness 3 cm and refractive index 1.5 is then placed close to the mirror in the space between the
object and the mirror.
Find the position of the final image formed.
(You may take the distance of the near surface of the slab from the mirror to be 1 cm). [IIT 1980]

2. The x-y plane is the boundary between two transparent media. Medium 1 with z 0 has refractive
index 2 and medium 2 with z 0. 0, has a refractive index 3 . A ray of light in medium 1 given by

the vector A 6 3 i 8 3 j 10 k is incident on the plane of separation. Find the unit vector in the direction
of the refracted ray in medium 2. [IIT 1999]

3. An object is placed in front of a convex mirror at a distance of 50 cm. A plane mirror is introduced
covering lower half of the convex mirror. If the distance between the object and the plane mirror is 30
cm, it is found that there is no parallux between the images formed by two mirrors. What is the radius of
curvature of the convex mirror? [IIT 1973]

www.physicsashok.in 38
OPTICS
4. A rectangular block of glass is placed on a printed page lying on a horizontal surface. Find the value of
the refractive index of glass for which the letters on the page are not visible from any of the vertical faces
of the block. [IIT 1979]

5. A glass lens has focal length 5 cm in air. What will be its focal length in water. (Refractive index of glass
is 1.51 and that of water is 1.33). [IIT 1977]

6. A ray of light is travelling form diamond to glass. Calculate the minimum angle of incidence of the ray as
the diamond glass interface such that no light is refracted into glass. W hat will happen if the angle of
incidence exceeds the angle? (refractive index of glass is 1.51 and that of diamond is 2.47)[IIT 1977]

7. What is the velocity of light in glass of refractive index 1.5? (Velocity of light in air = 3 x 1010 cm/sec.)
[IIT 1976]
8. Photographs of the ground are taken from an aircraft flying at an altitude of 2000 meters by a camera
with a lens of focal length 50 cm. the size of the film in the camera is 18 cm x 18 cm. What area of the
ground can be photographed by this camera at any one time? [IIT 1976]

9. A rectangular glass block of thickens 10 cm and refractive index 1.5 is placed over a small coin. A
4
beaker filled with water of refractive index to a height of 10 cm and is placed over the glass block.
3
(a) Find the apparent position of the object when it is viewed at near normal incidence.
(b) Draw a neat ray diagram.
(c) If the eye is slowly moved away from the normal at a certain position the object is found to disappear
due to total internal reflection. At which surface does this happen and why? [IIT 1975]

10. A ray of light travelling in air is incident at grazing angle


(incident angle = 90) on a long rectangular slab of a
transparent medium of thickness t = 1.0 m (see figure).
The point of incidence is the origin A (0, 0). The medium
has a variable index of refraction n (y) given by
n(y) = [Ky3/2 + 1]
3/2
where K = 1.0 (meter)
The refractive index of air is 1.0.
(a) Obtain a relation between the slope of the trajectory of
the ray at a point B (x, y) in the medium and the incident
angle at that point.
(b) Obtain an equation for the trajectory y (x) of the ray
in the medium.
(c) Determine the coordinates (x 1, y1) of the point P, where the ray intersects the upper surface of the
slab-air boundary.
(d) Indicate the path of the ray subsequently. [IIT 1995]

11. A quarter cylinder of radius R and refractive index 1.5 is


placed on a table. A point object P is kept at a distance
of mR from it. Find the value of m for which a ray from P
will emerge parallel to the table as shown in the figure.
[IIT 1999]

12. A light ray is incident on an irregular shaped slab of refractive index


2 at an angle of 45 with the normal on the incline face as shown
in the figure. the ray finally emerges from the curved surface in the
medium of the refractive index = 1.514 and passes through point E.
If the radius of curved surface is equal to 0.4 m, find the distance OE
correct up to two decimal places. [IIT 2004]

www.physicsashok.in 39
OPTICS
13. A point object O is placed at a distance of 12 cm on the axis of a convex lens of focal length 10 cm. On the
other side of the lens, a convex mirror is placed at a distance of 10 cm from the lens such that the image
formed by the combination coincides with the object itself. What is the focal length of the convex mirror?
[IIT 1976]

14. An object of height 4 cm is kept to the left of and on


the axis of a converging lens of focal length 10 cm as
shown in figure. A plane mirror is placed inclined at 45
to the lens axis 10 cm to the right of the lens (see
figure). Find the position and size of the image
formed by the lens and mirror combination.
trace the rays forming the image. [IIT 1972]

15. An object is placed at 20 cm left of the convex lens of focal


length 10 cm. If a concave mirror of focal length 5 cm is placed
at 30 cm to the right of the lens find the magnification and the
nature of the final image. Draw the ray diagram and locate the
position of the final image. [IIT 1974]

16. An object is approaching at thin convex lens of focal length 0.3 m


with a speed of 0.01 m/s. Find the magnitudes of the rates of change
of position and lateral magnification of image when the object is at a
distance of 0.4 m from the lens. [IIT 2004]

3
17. A thin biconvex lens of refractive index is placed on a horizontal
2
plane mirror as shown in the figure. The space between the lens
4
and the mirror is then filled with water of refractive index .
3
It is found that when a point object is placed 15 cm above the
lens on its principle axis, the object coincides with its own image.
On repeating with another liquid, the object and the image again coincide at a distance 25 cm from the
lens. Calculate the refractive index of the liquid. [IIT 2001]

18. A convex lens of focal length 15 cm and a concave mirror of focal length
30 cm are kept with their optic axes PQ and RS parallel but separated
in vertical direction by 0.6 cm as shown. The distance between the lens
and mirror is 30 cm. An upright object AB of height 1.2 cm is placed
on the optic axis PQ of the lens at a distance of 20 cm from the lens.
If AB is the image after refraction from the lens and reflection from the
mirror, find the distance of AB from the pole of the mirror and obtain
its magnification. Also locate position of A and B with respect to the
optic axis RS. [IIT 2000]

www.physicsashok.in 40
OPTICS
3
19. A thin equiconvex lens of glass of refractive index and
2
of focal length 0.3 m in air is sealed into an opening at one
4
end of a tank filled with water . On the opposite side
3
of the lens, a mirror is placed inside the tank on the tank wall
perpendicular to the lens axis, as shown in figure. The separation
between the lens and the mirror is 0.8 m. A small object is placed outside the tank in front of the lens at
a distance of 0.9 m from the lens along its axis. Find the position (relative to the lens) of the image of the
object formed by the system. [IIT 1997, May]

20. A thin plano-convex lens of focal length is split in to two halves:


one of the halves is shifted along the optical axis (see figure).
The separation between object and image planes is 1.8 m. The
magnification of the image formed by one of the half-lenses is 2.
Find the focal-length of the lens and separation between the two
halves. Draw the ray diagram for image formation. [IIT 1996]

21. A plano convex lens has a thickness of 4 cm. When placed on a horizontal table with the curved surface
in contact with it, the apparent depth of the bottom most point of the lens is found to be 3 cm. If the lens
is inverted such that the plane face is in contact with the table, the apparent depth of the centre of the
plane face is found to be 25/8 cm. Find the focal length of the lens. [IIT 1984]

22. The convex surface of a thin concavo-convex lens of glass of


refractive index 1.5 has a radius of curvature 20 cm. The concave
surface has a radius of curvature 60 cm. The convex side is
silvered and placed on a horizontal surface.
(a) Where should a pin be placed on the optic axis such that
its image is formed at the same place?
4
(b) If the concave part is filled with water of refractive index ,
3
find the distance through which the pin should be moved so
that the image of the pin again coincide with the pin. [IIT 1981]

23. Find the focal length of the lens shown in the figure. The
radii of curvature of both the surfaces are equal to R.
[IIT 2003]

24. The refractive indices of the crown glass for blue and red lights are 1.51 and 1.49 respectively and those
of the flint glass are 1.77 and 1.73 respectively. An isosceles prism of angle 6 is made of crown glass.
A beam of white light is incident at a small angle of this prism. The other flint glass isosceles prism is
combined with the crown glass prism such that there is no deviation of the incident light. Determine the
angle of the flint glass prism. Calculate the net dispersion of the combined system. [IIT 2001]

25. A prism of refracting angle 30 is coated with a thin film of transparent material of
refractive index 2.2 on face AC of the prism. A light of wavelength 5500 A is
incident on face AB such that angle of incidence is 60, find
(a) the angle of emergence,
[Given refractive index of the material of the prism is 3 ].

www.physicsashok.in 41
OPTICS

(b) the minimum value of thickness of the coated film on the face AC for which the light emerging
from the face has maximum intensity. [IIT 2003]

26. A right angle prism (45 90 45) of refractive index n has a plate of refractive
index n1 (n1 < n) cemented to its diagonal face. The assembly is in air. A ray is
incident on AB (see figure),
(a) Calculate the angle of incidence at AB for which the ray strikes the
diagonal face at the critical angle.
(b) Assuming n = 1.352, calculate the angle of incidence at AB for which
the refracted ray passes through the diagonal face undeviated.
[IIT 1996]

27. A right angled prism is to be made by selecting a proper material


and the angles A and B (B A), as shown in figure. It is desired
that a ray of light incident on the face AB emerges parallel to the
incident direction after two internal reflections.
(a) What should be the minimum refractive index n for this
to be possible?
5
(b) For n = is it possible to achieve this with the angle B
3
equal to 30 degrees ? [IIT 1987]

28. Monochromatic light is incident on a plane interface AB


between two media of refractive indices n1 and n2 (n2>n1)
at an angle of incidence as shown in the figure. The
angle is infinitesimally greater than the critical angle
for the two media so that total internal reflection takes place.
Now if a transparent slab DEFG of uniform thickness and of
refractive index n3 is introduced on the interface (as shown in
the figure), show that for any value of n3 all light will ultimately
be reflected back again into medium II. Consider separately the cases.
(i) n3 < n1 and (ii) n3 > n1. [IIT 1986]

4
29. A parallel beam of light travelling in water (refractive index =
) is refracted by a spherical air bubble of
3
radius 2 mm situated in water. Assuming the light rays to be paraxial,
(a) find the position of the image due to refraction at the first surface and the position of the final
image.
(b) draw a ray diagram showing the positions of both the images. [IIT 1988]

30. Light is incident at an angle on one planar end of a transparent cylindrical rod of refractive index n.
Determine the least value of n so that the light entering the rod does not emerge from the curved surface
of the rod irrespective of the value of . [IIT 1992]

31. The radius of curvature of the convex face of a plano convex lens is 12 cm and its refractive index is 1.5.
(i) Find the focal length of this lens.
(ii) The plane surface of the lens is now silvered. At what distance from the lens will parallel rays
incident on the convex face converge.
(iii) Sketch the ray diagram to locate the image, when a point object is placed on the axis, 20 cm from
the lens (polished).
(iv) Calculate the image distance when the object is placed as in (iiii). [IIT 1979]

32. A ray of light is incident at an angle of 60 on one face of prism which has an angle of 30 with the
incident ray. Show that the emergent ray is perpendicular to the face through which it emerges and
calculate the refractive index of the material of the prism. [IIT 1978]

www.physicsashok.in 42
OPTICS
33. A pin is placed 10 cm in front of a convex lens of focal length 20 cm., made of a material of refractive index
1.5. The surface of the lens farther away from the pin is silvered and has a radius of curvature are 22 cm.
Determine the position of the final image. Is the image real as virtual? [IIT 1978]

34. The refractive index of the material of a prism of refracting angle 45 is 1.6 for a certain monochromatic
ray. What should be minimum angle of incidence of this ray on the prism so that no total internal reflection
takes place as the ray comes out of the prism. [IIT
1976]
35. A prism of refractive index n1 and another prism of refractive index n2
are stuck together without a gap as shown in the figure. The angles
of the prisms are as shown n1 and n2 depend on , the wavelength
of light, according to
10.8 10 4 1.80 10 4
n1 1.20 and n2 1.45
2 2
where, is in nm.
(a) Calculate the wavelength 0 for which rays incident at any angle on the interface BC pass through
without bending at that interface.
(b) For light of wavelength 0, find the angle of incidence i on the face AC such that the deviation
produced by the combination of prisms is minimum. [IIT 1998]

36. A projector lens has a focal length 10 cm. It throws an image of a 2 cm x 1 cm slide on a screen 5 metre
from the lens. Find :
(a) the size of the picture on the screen and
(b) ratio of illuminations of the slide and of the picture on the screen. [IIT 1975]

37. A ray of light incident normally on one of the faces of a right angled
isosceles prism is found to be totally reflected as shown in the figure.
What is the minimum value of the refractive index of the material of
the prism? When the prism is immersed in water, trace the path of
the emergent rays for the same incident ray, indicating the values
4
of all the angles. . [IIT 1973]
3

www.physicsashok.in 43
OPTICS

Answer Key
Assertion & Reasion
Assertion & Reasion
Que. 1 2 3 4 5 6 7 8 9 10

Ans. A A A A B E C A C A

Que. 11 12 13 14 15 16

Ans. B D D B B D

Level # 1

Objective Type
Que. 1 2 3 4 5 6 7 8 9 10

Ans. B C A A C C B D D C

Que. 11 12 13 14 15 16 17 18 19 20

Ans. D A B C A D B CD C C

Que. 21 22 23 24 25 26 27 28 29 30

Ans. B B B A D C A A A B

Que. 31 32 33 34 35 36 37 38 39 40

Ans. C C C A D A A C D C

Que. 41 42 43 44 45 46 47 48 49 50

Ans. A D B D C C B B B C

Que. 51 52 53 54 55 56 57 58 59 60

Ans. B C A A B A BD D A C

Que. 61 62 63 64

Ans. C BC CD D

Fill in the Blanks / TrueFalse / Match Table


1. 2 x 108 m/s, 0.4 x 106 m 2. d = +15 cm 3. 4000, 5 x 1014 Hz 4. 2
25
5. 60 cm 6. 7. 35 cm 8. 1.3
9
9. Zero 10. 5 x 1014 Hz, 4000 11. 0.125 m, 0.5 m 1 2 .
15

Que. 13 14 15 16 17 18 19 20

Ans. T F T T D A C B

www.physicsashok.in 44
OPTICS

Passage Type
Que. 1 2 3 4 5 6 7 8 9 10 11 12 13 14 15

Ans. D B B C D B D D B A A C C A A

Que. 16 17 18 19 20 21 22 23 24 25 26 27 28 29 30

Ans. A B A B C A D B B A D D B B B

Que. 31 32 33 34 35 36 37 38 39 40 41 42 43 44 45

Ans. C C B D B A B C C C B B A A B

Que. 46 47 48 49 50 51 52 53 54 55 56 57 58 59 60

Ans. C D D B C A B C B A B C A A B

Que. 61 62 63 64 65 66 67 68 69 70 71

Ans. D D C A B C C A D C D

Level # 2.

2. 1.6 3. 90 cm., Yes. 4. 2.88 m 5. (a) 13.3 cm (b) 14.975 cm


0
2
A 1
x r 1 2 d 2 1 d
2 2

7. (a) A 0 0
(b) 2 2
r 0 0 1
r
(c) Ray will become parallel to y-axis.


t 1 cos i 1 cos i sin i
8. 2 Km. 9. R 12 cm . 10. 7.42 cm 11. x 1 t
2
2
12 sin 2 i sin i
2

1 1
a a
12. i A sin sin A sin 2 i cos A sin i 13. , 15
2 1 8 8
b2 b2
1 b 2
14. 2 sin 1 15. The first spot is at 12 cm on left side from the optical
R R R2

centre.
2 cos 1
16. 20 cm 17. , 0 19. b = 15 cm.
cos 1
20. At a distance of 15 cm. from the mirror 2 cm. from each other.

21. First Image at a distance of 3.33 cm from flat surface and the second at infinity. 22. 9 cm


2 1
d R 1
23. 2 2 25. 8140 26. 2 2
b b
2 2 1 2
R R
27. 25 cm from the second lens on the right side magnification m = 2. 28. 0.5 cm
7
29. 6 cm back side of unpolished lens. 30.
3

www.physicsashok.in 45
OPTICS

Level # 3
3 i 4 j 5 k
1. 7.67 cm from the mirror. 2. 3. 25 cm 4. 2
5 2
5. 18.84 cm 6. = sin1 0.6115 = 38 No ray is refracted into glass.

7. 2 x 1010 cm per second. 8. 720 m x 720 m

9. (a) 14.16 cm below the water surface. (b) No glass water interface.
4
dy 2 x
10. (a) Slop cot i (b) y K (c) (4m, 1m) (d) Ray emerges parallel to the positive x-axis.
dx 4

4
11. m 12. OE = 6.06 m 13. 25 cm.
3
14. At a perpendicular distance of 20 cms from the lens axis 8 cm is size oriented parallel to lens axis.

15. At the position of the object magnification = 1. 16. 0.09 m/s, 0.3 per second. 17. 1.6

18. Distance of AB from pole of mirror a15 cm, magnification = 1.5.


Distance of A above RS is 0.3 cm, Distance of B below RS is 1.5 cm.

19. 0.9 m from the lens (0.1 m behind the mirror) 20. (a) = 0.4 m, d = 0.6 m. 21. 75 cm
3 R
22. (a) 15 cm (b) 1.15 cm towards the lens 23. 24. 4, 0.04
3 1

1 1 2 2 1 1.352
25. (a) 0 (b) 1250 26. (a) sin n n1 n1 (b) sin 73
2 2

1
27. (a) sin B (b) No.

29. (a) Image due to first surface at a distance of 6 mm before the first surface final image at a distance of
1mm before the first surface.
30. 2 31. (a) 24 cm (b) 12 cm (d) 80 cm. 32. 3 .

33. 17 cm infront of lens, Real. 34. sin 1 0.176 10.1 35. (a) 0 600 nm (b) i sin1

slide
36. (a) 100 cm x 50 cm (b) 2401 37. 2 , Angle of refraction in water r = sin 1 .
picture

XXXX

www.physicsashok.in 46
ELECTROSTATICS

DHANALAKSHMI NAGAR

NEAR ANNAMAIAH CIRCLE,

TIRUPATI.

PH NO. 9440025125
ELECTROSTATICS
ELECTROSTATICS
ELECTRIC CHARGE
Charge is the property associated with matter due to which is produces and experiences electrical and magnetic
effects. The study of electrical effects of charge at rest is called electrostatics.
The strength of particles electric interaction with objects around it depends on its electric charge, which can be
either positive or negative. An object with equal amounts of two kinds of charge is electrically neutral, whereas
one with an imbalance is electrically charged.
In the table given below, if a body in the first column is rubbed against a body in the second column, the body
in first column will acquire positive charge, while that in the second column will acquire negative charge.
TABLE
S.No. First Column Second Column
1. Glass rod Silk rod
2. Flannes or cat skin Ebonite rod
3. Woollen cloth Amber
4. Woollen cloth Rubber shoes
5. Woollen cloth Plastic objects

Electric Charge :
Electric charge can be written as ne where n is a positive or negative integer and e is a constant of nature called
the elementary charge (approximately 1.60 1019 C). Electric charge is conserved, the (algebraic) net charge
of any isolated system cannot be changed.
Regarding charge following points are worth nothing :
(a) Like charges repel each other and unlike charges attract each other.
(b) Charge is a scalar and can be of two types; positive or negative.
(c) Charge is quantized, i.e., the charge on any body will be some integral multiple of e, i.e., q = ne.
where n = 1, 2, 3,...........
1
Charge on any body can never be e , 1.5e etc.
3
(d) The electrostatic unit of charge is stat-coulomb and electromagnetic unit is ab-coulomb in CGS system. But in
1
SI system the unit of charge is coulomb, 1 coulomb ab coulomb 3 109 stat coulomb.
10

NOTE : Recently, it has been discovered that elementary particles such as proton or
1 2
neutron are composed of quarks having charge e and e . However,,
3 3
as quarks do not exist in free state, the quantum of charge is still e.

Example-1 : How many electrons are there in one coulomb of negative charge ?
Sol. : The negative charge is due to presence of excess electrons, since they carry negative charge. Because an
electron has a charge whose magnitude is e = 1.6 1019 C, the number of electrons
q 1.0
n
e 1.6 1019
n = 6.25 1018

www.physicsashok.in 1
ELECTROSTATICS
COULOMBS LAW
The force acting between two point charges is directly proportional to the product of the charges and it is
inversely proportional to the square of distance between them. Mathematically.
q1q 2
F
r2
kq1q 2
F
r2
1
k 9.0 109 Nm 2 / C 2
4 0
where 0 = permittivity of free space
0 = 8.854 1012 C2/Nm2
Regarding Coulombs law following points are worth noting :
(a) When two charges exert forces simultaneously on a third charge, the total force acting on that charge is the
vector sum of forces that the two charges would exert individually. This important property, called the principle
of superposition of forces, holds for any number of charges. Thus,

Fnet F1 F2 F3 ..... Fn
(b) If some dielectric (K) is placed in the space between the charges, the net force acting on each charge is altered
because charges are induced in the molecules of intervenning medium.
q1 q2
Fe Fe
r
In vacuum
1 q1q 2
Thus, Fe . (in vacuum)
4 0 r 2
1 qq
F'e . 12 2 (In medium)
4 0 k r
1 q1q 2
or F'e .
4 r 2
Here 0 K is called permittivity of the medium
(c) The coulombs law expresses the force between two point charges at rest. In applying it to the case of extended
bodies of finite size care should be taken in assuming the whole charge of a body to be concentrated at its
centre as that is true only for sphercal charged body, that is too for external point.
(d) A charged paritcle under the action of coulombian forces only can never be stable. This statement is called
Earnshows theorem.

IMPORTANT FEATURE

1. Suppose the position vectors of two charges q1 and q2 are r1 and r 2 then electric force on charges q1 due to
charge q2 is
1 qq
F12
. 1 2 3 r1 r 2
4 0 r1 r 2

Similarly, electric force on q2 due to charges q1 is
1 qq
F21 . 1 2 3 (r2 r1 )
4 0 |r2 r1|

www.physicsashok.in 2
ELECTROSTATICS
Here q1 and q2 are to be subsitiuted with sign.

r 1 x1 i y1 j z1 k and r 2 x 2 i y 2 j z 2 k where (x1 , y1 , z1 ) and (x 2 , y 2 , z 2 ) are the co-ordinates of
charges q1 and q2.
2. Suppose two charges q1 and q2 are placed in vacuum at a distance r0 and the electric force between them is
1 q1q 2
F0 .
4 0 r02
Now, the same charges are placed in a dielecric medium of dielectric constant K at distance r (<r0) such that
the electric force between them
1 qq
F . 12 2
4 0 k r
remains the same or F = F0
1 qq 1 q1q 2
Then, . 12 2
4 0 k r 4 0 r02

or r0 k r
Thus, we see that for the electric force between two charges at distance r in a dielectric medium is equivalent
to a distance r0 k r in vacuum.
Example-2 : What is the smallest electric force between two charges placed at a distance of 1.0 m ?
1 q1q 2
Sol. Fe .
4 0 r 2
For Fe to be mimimum q1 q2 should be mimimum.
q1 q 2 e 1.6 1019 C
Substituting in above relation we have
(9.0 109 )(1.6 10 19 )(1.6 10 19 )
(Fe )
(1.0) 2
y
2.304 1028 N
+q x
Example-3 :Three point charges +q, q and +q are placed at the vertices P, Q and R of an P
2
1 q r r
equilateral triangle as shown in fig. If F ,
2 where r is the side of the triangle,
4 0 r
Q R
the force on charge at P due to charges at Q and R is : q r +q
(A) F along positive x-direction (B) F along negative x-direction
(C) 2 F along positive x-direction (D) 2 F along negative x-direction. S
y
Sol. Refer to Fig. The charge at Q exerts an attractive force F on charge at P F
120 P x
along PQ. The charge at R exerts a repulsive force on charge at P along Fr +q
PS of magnitude F. The angle between these two forces is 120. From F r
parallelogram law, the magnitude of the resultant force is
Fr2 = F 2 + F2 + 2F2 cos 120 Q R
q r +q
2 2 2
= 2F F = F
or Fr = F. As shown in the figure, the direction of the resultant force is along the
negative x-direction. Hence the correct choice is (b).
Example-4 : Three charges q1 = 1c, q2 = 2c and q3 = 3c are placed on the vertices of an equilateral triangle
of side1.0 m.Find the net electric force acting on charge q1.

www.physicsashok.in 3
ELECTROSTATICS
Sol. Let us assume a co-ordinate axes with q1 at origan as shown in figure. The y
q3
co-ordinates of q1, q2 and q3 in this co-ordinate system are (0,0,0), (1m, 0,0)
and (0.5m, 0.87m, 0) respectively.

Now, F1 = force on q1 due to charge q2. x
q1 q2
1 qq
. 1 2 3 (r1 r2 )
4 0 |r1 r2 |

(9.0 109 )(1.0 106 )(3.0 10 6 )



(1.0)3

[(0 0.5)i ?(0 0.87)j (0 0)k]


(1.8 102 i)N



and F2 force on q1 due to charge q3
1 qq
. 1 3 3 (r1 r3 )
4 0 |r1 r3 |

(9.0 109 )(1.0 106 )(3.0 10 6 )



(1.0)3

[(0 0.5)i (0 0.87)j (0 0)k]


(1.35i 2.349j) 102 N


Therefore net force on q1 is

F F1 F2
(0.45i 2.349j)
102 N

ELECTRIC FIELD
Electirc field is the region around an electric charge (or a group of electric charges) in which the electric force
can be experienced. If an electric charge is placed in such region, it experiences either an attractive or a
repulsive force.

Electric field at a point can be defined in terms of either a vector function E called electric field strength or a
scalar function V called electric potential. The electric field can also be visualised graphically in terms of lines of
force.

Electric Field Intensity :


The intensity of electric field at any point is defined as the force acting on a unit positive charge placed at that

point. If the electrostatic force experienced by a small test charge q0 is Fe , then intensity of electric field

Fe
E
q0

The electric field is a vector quantity and its direction is the same as the direction of the force Fe on a positive
test charge. The SI unit of electric field is N/C.

Example-5 : An electric field of 105 N/C points due west at a certain spot. What are the magnitude and direction
of the force that acts on a charge of + 2C and 5C at this spot ?
www.physicsashok.in 4
ELECTROSTATICS
Sol. Force on + 2C = qE = (2 106) (105)
= 0.2 N (due west)
Force on 5C = (5 106) (105)
= 0.5 N (due east)
Example-6 : An inclined plane making an angle 30 with the horizontal is q

placed in a uniform horizontal electric field of 100 V/m (see figure). e= 100 V/m
1m
Particle of mass 1 kg and charge 0.01 C is allowed to slide down
from rest from a height of 1 m. If the coefficient of friction is 0.2, 30
find the time it will take the particle to reach the bottom.
Sol. The different forces on the particle are shown in figure.

From figure, N = mg cos 30 + q cos 60 N co s 30
qe
N
Friction f = N = mg cos 30 + q cos 60 q
30
qe
30
Now the total force F acting along the inclined plane is sin 30 qesin 30
mg
F = mg sin 30 N qcos 30 mg cos 30
or F = mg sin 30 mg cos 30 q cos 60 qcos 30 30
Thus acceleration is
F q q
or a g sin 30 g cos 30 cos 60 cos 30
m m m
0.2 0.01 100 0.01 100 3
or
a 9.8 0.5 0.2 9.8 3 / 2 1
0.5
1

2
or = 4.9 0.98 1.732 0.10 0.551.732
or = 4.9 1.697 0.10 0.866 = 2.237
Now, distance travelled in time t is
1
s = 0 + at 2
2
2 2
or t [As s = 1/sin30 = 2]
a
4
or 1.345 sec.
2.237

Example-7 : In space horizontal Electric field (E = (mg)/q) exist as shown in figure q=45 mg
q E=
and a mass m attached at the end of a light rod. If mass m is released from the q
m
position shown in figure find the angular velocity of the rod when it passes through
the bottom most position m
+q
g 2g 3g 5g
(A) (B) (C) (D)
l l l l
Sol. According to work energy theorm :
w = T
1 q q=45 mg
WE + Wg = mv2 0 ...(1) l cosq l E=
l sinq +q q
2
WE = qE l sin , Wg = mg (l l cos ) qE m
m l l cosq
1
qE l sin + mg (l l cos) = mv2 from eqn (1) +q
2
1 mg
mg l sin + mg l mg l cos = mv2 { E = }
2 q
www.physicsashok.in 5
ELECTROSTATICS
gl gl 1
+ gl - = mv 2 v = 2gl
2 2 2
v 2g l 2g 2g
w= = = w = Ans.
l l l l

TABLE : Electric field Intensity of Various System


S.No. System Electric Field Intensity
1 q
1. Isolated charge E . 2
4 0 r
Isolated charge
r E
q p
1 qx
E .
2. A ring of charge 3/ 2

4 0 R 2 x 2

x
3. A disc of charge E 1
2 0 2 2
x R


4. Infinite sheet of charge E
2 0


5. Infinitely long line of charge E
2 0 r

www.physicsashok.in 6
ELECTROSTATICS

6. Finite line of charge E sin sin
4 0 x

E| | cos cos
4 0 x

7. Charged spherical shell (a) Inside 0 r R, E = 0


(b) Outside r R
q
E
4 0 r 2

8. Solid sphere of charge (a) Inside 0 r R


r
E
3 0
(b) Outside r R
q
E
4 0 r 2
where q = charge on sphere

Electric Lines of Force :


Faraday gave a new approach for representation of electric field in the form of electric lines of force. Electric
lines of force are graphical representation of electric field. An electric line of force is an imaginary line or curve
drawn through a region of space so that its tangent at any point is in the direction of the electric field vector at
that point.
EQ

EP Q

P
This model of electric field has the following characteristics :
(i) Electric lines of force are originated from positive charge and terminated into negative charge.

(ii) The number of electric lines of force originates from a point charge q is q/0. Electric lines of force may be
fraction.
(iii) The number of lines per unit area that pass through a surface perpendicular to the electric field lines is proportional
to the strength of field in that region.

www.physicsashok.in 7
ELECTROSTATICS
(iv) No electric lines of force cross each other. If two electric lines of force cross each other, it means electric field
has two directions at the point of cross. This is not physically possible.
Line of f E1
orce

or ce
of f
L in e E2

(v) Electric lines of force for two equal positive point charges are said to have rotational symmetry about the axis
joining the charges.

(vi) Electric lines of force for point positive charge and a nearby negative point charge that are equal in magnitude
are said to have rotational symmetry about an axis passing through both charges in the plane of the page.

(vii) Electric lines of force due to infinitely large sheet of positive charge is normal to the sheet.

(viii) No electrostatic lines of force are present inside a conductor. Also electric lines of force are perpendicular to
the surface of conductor. For example if a conducting sphere is placed in a region where uniform electric field
is present, then induced charges are developed on the sphere.

(ix) If a charged particle is released from rest in region where only uniform electric field is present, then charged
particle move along an electric line of force. But if charged particle has initial velocity, then the charged particle
may or may nor follow the electric lines of force.
(x) Electric lines of force inside the parallel plate capacitor is uniform. It shows
that field inside the parallel plate capacitor is uniform. But at the edge of
plates, electric lines of force are curved. It shows electric lines of force at the
edge of plates is non-uniform. This is known as fringing effect.
If the size of plates are infinitely large, then fringing effect can be neglected.

(xi) If a metallic plate is introduced between plates of a charged capacitor, then electric lines of force can be
discontinuous.

www.physicsashok.in 8
ELECTROSTATICS

(xii) If a dielectric plate is introduced between plates of a charged capacitor, then number of lines of forces in
dielectric is lesser than that in case of vacuum space.

(xiii) Electrostatics electric lines of force can never be closed loops, as a line can never start and end on the same
charge. Also if a line of force is a closed curve, work done round a closed path will not be zero and electric field
will not remain conservative.
(xiv) Lines of force have tendency to contract longitudinally like a stretched elastic string producing attraction
between opposite charges and repel each other laterally resulting in, repulsion between similar charges and
edge-effect (curving of lines of force near the edges of a charged conductor).

ELECTRIC POTENTIAL AND ELECTRIC POTENTIAL DIFFERENCE :


Electric Potential :
Electric potential at any point in a electric field is equal to the ratio of the work done in bringing a test charge
from infinity to that point, to the value of test charge.
Suppose, W be the work required in bringing a test charge q0 from infinity to a point b against the repulsive
force F acting on it, then potential at the point b is
W b
Vb
q0
Since, W and q0 both are scalar quantities, the potential is also a scalar quantity.

Electric Potential Difference :


The potential difference between two points in an electric field is equal to the ratio of work done in moving a
test charge from one point to the other, to the value of test charge. Suppose W work be done in bringing a small
test charge q0 from the point a to a point b against the repulsive force acting on it, then potential difference
between the points is
W
Vb Va a b
q0
Obviously, potential difference is also a scalar quantity.

IMPORTANT FEATURES
1. Following three formulae are very useful in the problems related to work done in electric field.
(Wa b)electric force = q0 (Va Vb)
(Wa b)external force = q0 (Vb Va) = (Wa b)electric force
(W a)external force = q0Va
2. Electric potential due to a point charge q : From the definition of potential,
1 qq 0
.
U 4 0 r
V
q0 q0
www.physicsashok.in 9
ELECTROSTATICS
1 q
or V .
4 0 r
Here, r is the distance from the point charge q to be point at which the potential is evaluated.
If q is positive, the potential that it produces is positive at all points; if q is negative, it produces a potential that
is negative everywhere. In either case, V is equal to zero at r = .
3. Electric potential due to a system of charges : Just as the electric field due to a collection of point charges
is the vector sum of the fields produced by each charge, the electric potential due to a collection of point
charges is the scalar sum of the potentials due to each charge.
1 qi
V
4 0 i ri
1 qi
4. In the equation V 4 r , if the whole charge is at equal distance r0 from the point where V is to be
0 i i
evaluated, then we can write,
1 q net
V .
4 0 r0
where qnet is the algebraic sum of all the charges of which the system is made.

Example-8 : The electric potential at point A is 20 V and at B is 40 V. Find the work done by a external force and
electrostatic force in moving an electron slowly from B to A.
Sol. Here, the test charge is an electron, i.e.,
q0 = 1.6 1019 C
VA = 20 V
and VB = 40 V
Work done by external force
(WB A)external force = q0(VA VB)
= (1.6 1019)[(20) (40)]
= 9.6 1018 J
Work done by electric force
(WB A)electric force = (WB A)external force
= (9.6 1018 J) = 9.6 1018 J

Example-9 : Find the work done by some external force in moving a charge q = 2C from infinity to a point where
electric potential is 104 V.
Sol. using the relation,
(W a)external force = q Va
We have (W a)external force = (2 106) (104)
= 2 102 J

Example-10 : Three point charges q1 = 1C, q2 = 2C and q3 = 3C are placed at (1m, 0, 0), (0, 2m, 0) and (0,
0, 3m) respectively. Find the electric potential at origin.
Sol. The net electric potential at origin is,
1 q1 q 2 q 3
V
4 0 r1 r2 r3
Substituting the values, we have

www.physicsashok.in 10
ELECTROSTATICS
1 2 3

V 9.0 109

1.0 2.0 3.0
10 6

V = 9.0 103 volt

TABLE : Electric Potential of Various Systems


S.No. First Column Second Column
q
1. Isolated charge V
4 0 r
q p
r
q q
2. A ring of charge V
4 0 R x2
2


3. A disc of charge V R 2 x2 x
2 0

4. Infinite sheet of charge Not defined

5. Infinitely long line of charge Not defined

sec tan
6. Finite line of charge V ln
4 0 sec tan
x
+
+
+
+ p
+
+
+
+

www.physicsashok.in 11
ELECTROSTATICS
7. Charged spherical shell (a) Inside 0 r R
q
V
4 0 R
(b) Outside r > R
q
V
4 0 r
8. Solid sphere of charge (a) Inside 0 r R
R 2r2
V 3
R2
6 0
(b) Outside r R
q
V
4 0 r

Example-11 : In a regular polygon of n sides, each corner is at a distance r from the centre. Identical charges are
placed at (n 1) corners. At the centre, the intensity is E and the potential is V. The ratio V/E has magnitude.
(A) r n (B) r(n 1) (C)(n 1)/r (D) r(n 1)/n
q
Sol. E 4 r 2
0

(n 1)q
and v
4 0 r
(n 1)q
v 4 0 r
(n 1)r
E q
4 0 r 2

Example-12 : The figure shows a nonconducting ring which has positive and negative ++

+
charge non uniformly distributed on it such that the total charge is zero. Which +
of the following statements is true ? +
O
(A) The potential at all the points on the axis will be zero. +
+ axis
(B) The electric field at all the points on the axis will be zero. +
+
(C) The direction of electric field at all points on the axis will be along the axis. +
(D) If the ring is placed inside a uniform external electric field then net torque and force acting on the ring
would be zero.
q +
Sol. V 0 ( q = 0) + R 2 + x2
4 0 R 2 x 2 +

+
There for the potential at all the points on the axis will be zero. +
O x
+
Hence (A) is correct. +
+
+
+
3/2

b qB= (2/5) q
Example-13 : Two concentric rings, one of radius a and the other of radius b have
the charges +q and (2/5)3/2 q respectively as shown in the figure. Find the ratio a z=a
b/a if a charge particle placed on the axis at z = a is in equilibrium.
qA=+q

www.physicsashok.in 12
ELECTROSTATICS
Sol. At equilibrium
F=0
QE = 0
E=0
qA z qBz qA z q Bz
3
3
0 3
3
2 2 2 2 2 2 2 2 2 2 2 2
4 0 (z a ) 4 0 (z b ) 4 0 (z a ) 4 0 (z b )
3
3
2 2 3

qa
5
qa a 2 b2 2
2


2

3
3 3
5
4 0 (a 2 a ) 2 2
4 0 (a 2 b ) 2 2 a 2
a
2 2

3/2
3 b qB= (2/5) q
3
a b
2 2 2
5

2
a 2 b2 5
3 a z=a
2
2a
2 2 2a 2 2
qA=+q

2a2 + 2b2 = 10a2 2b2 = 8a2

b2 b
4 2 Ans.
a2 a

Example-14 : A circular ring of radius R with uniform positive charge density per unit length is located in the y-
z plane with its centre at the origin O. A particle of mass m and positive charge q is projected from the point
q
P(R 3 , O, O) on the positive x-axis directly towards O, with an initial kinetic energy 4 .
0
(A) The particle crosses O and goes to infinity. (B) The particle returns to P
(C) The particle will just reach O. (D) The particle crosses O and goes to R 3 .
Sol. According to machenical energy conser
U i + Ti = Uf + Tf ...(i)
q q1 l + + + R + 3R = x
Ui 2 2

4 0 (x) + R+
+ O + q
Where q1 is the charge on ringh + + R 3
q1 = (2R) + ++
q(2 R )
Ui
4 0 (2R)
q
Ui
4 0
q
Ti
4 0
q q1
Uf
4 0 R
q(2 R ) q
Uf
4 0 R 2 0
1
Tf = mv2
2
where v is a velocity of charge putting the volues in eqn (1)

www.physicsashok.in 13
ELECTROSTATICS
q q q 1 2
mv
4 0 4 0 2 0 2
q q 1 2 1
mv mv 2 0 v 0
2 0 2 0 2 2
The particle will just reach centre of ring. Y
q ++
+ q
+
Example-15 : Find the electric field at centre of semicircular ring shown in figure. +
+
X
R
Sol. E E 20 E02 2 E0 Y
E0
But E0
2 0 R
sin
4 q + + + +q
+
K
l 2q 45 45 +
But , +
4 0 R2 45 X
E 45 O
4Kq
E directed negative x-axis. E0
R2

Example-16 : A simple pendulum of length l and bob mass m is hanging in front of a large s+
+
nonconducting sheet having surface charge density . If suddenly a charge +q is given +
+
+
to the bob & it is released from the position shown in figure. Find the maximum + l
+
angle through which the string is deflected from vertical. +
+
Sol. The electric field intensity due to nonconducting sheet is

E ...(1)
2 0
According to work energy theorem
Wnet = k = 0
Since initial and final velocity of bob is zero
Therefore, WE + Wg = 0 ...(2)
WE = qE sin s+
+
Wg = mg ( cos) +
+
+ cosq qsinq qE
Putting the values in eqn (2) +
+ v=0
qE sin mg (1cos) = 0 +
+ mg
qE sin = mg (1cos)

2 sin 2
qE 1 cos qE 2

mg
mg sin 2 sin cos
2 2
qE qE
tan tan 1
mg 2 2 mg
qE
2 tan 1
mg
Putting the value of E from eqn (1)
q q
2 tan 1 2 tan 1 Ans.
2 0 mg 2 0 mg

www.physicsashok.in 14
ELECTROSTATICS
Example-17 : A particle of mass m and negative charge q is thrown in a gravity free space with +
+
speed u from the point A on the large non conducting charged sheet with surface charge +
+
density , as shown in figure. Find the maximum distance from A on sheet where the + u
+
particle can strike. +
+A
Sol. Electric field intensity due to nonconducting sheet

E ...(1)
2 0
where ux = u sin Y
+
uy = u cos + ucos
+
qE +
ax = + u
+
m +
+
1 2 A usin X
x = uxT + a xT
2
where x = 0
1 1 qE 2
0 = u xT + a xT2 0 u sin T T
2 2 m
1 qE 2 2umsin
u sin T T T
qE
2 m
y = u yT
y = (u cos)T
2u msin u 2 msin 2
y u cos y
qE qE
2
2u m 0 sin 2 2u 2 0 m sin 2
y from eqn (1) y
q q
2u 2 0 m
y max
q
At ymax, sin2 = 1
+ +
+ +
Example-18 : The figure shows three infinite non-conducting plates of charge perpendicular ++ A +
+
B

to the plane of the paper with charge per unit area +, +2 and , Find the ratio of + +
+ +
the net electric field at that point A to that at point B. + +
2.5m 2.5m
+s +2s s
Sol. The electric field intensity at point A due to plate x, y and z 5m 5m

EA = Ex + Ey + Ez
2

2 0 2 0 2 0
EA = 0 ...(1) +s +2s s
At point B
2 A B
EB
2 0 2 0 2 0 2s s s
2 0 2 0 0
4
EB ...(2)
2 0
x y z
From eqn (1) and (2)
EA
=0
EB
www.physicsashok.in 15
ELECTROSTATICS
Example-18 : A metallic solid sphere is placed in a uniform electric field. 1 1
The lines of force follow the path(s) shown in figure as : 2 2
(A) 1 (B) 2 3 3
(C) 3 (D) 4 4 4
Sol. Electric field lines never enter a metallic conductor (E = 0, inside a conductor) and they fall normally on
the surface of a metallic conductor (because whole surface is at same potential and lines are perpendicular
to equipotential surface).

Example-19 : A non-conducting solid sphere of radius R is uniformly charged. The magnitude of electric field
due to the sphere at a distance r from its centre :
(A) increases as r increases for r < R (B) decreases as r increases for 0 < r <
(C) decreases as r increases for R < r < (D) is discontinuous at r = R
(E) both (a) and (c) are correct
r
Sol. E r<R
3 0
E r r<R
Q
E r>R
4 0 r 2
1
E 2 Hence, both (a) and (c) are correct.
r

Example-20 : A particle of mass m and charge q moves along a diameter of a uniformly charged sphere of radius
R and carrying a total charge +Q. Find the frequency of S.H.M. of the particle if the amplitude does not exceed
R.
Sol. Electric field intensity due to non conducting sphere
x
E ...(1) (where x < R)
3 0
Where is volume charge density.
The force on negative charge is opposite direction of electric field.
qE = ma
From eqn (1)
R
x q x q
q ma a ...(2) x
3 0 3 0 m
2
For S.H.M. a = x ...(3)
From eqn (2) and (3)
q q
2
3 0 m 3 0 m
q
2 f { 2 f }
3m 0
1 q 1 qQ Q
f f
2 3m 0 2 4 0 mR 3 4 / 3R 3

Example-21 : A positive charge Q is uniformly distributed throughout the volume of a dielectric sphere of radius
R. A point mass having charge +q and mass m is fired towards the centre of the sphere with velocity v from a
point at distance r (r > R) from the centre of the sphere. Find the minimum velocity v so that it can penetrate R/

www.physicsashok.in 16
ELECTROSTATICS
2 distance of the sphere. Neglect any resistance other than electric interaction. Charge on the small mass
remains constant throughout the motion.
Sol. From machenical energy conservation
U i + T i = Uf + Tf
qV i + mv2 = qVf + 0 ...(1)
2 2
Q 1 Q(3R r )
Vi , Vf
4 0 r 2 4 0 R 3 R
v
R O R
min
+q
where r = 2
2 r
2
R
Q 3R 2
Q 12R 2 R 2
1 4 Vf
Vf 3 32 0 R 3
2 4 0 R
Q(11) 11Q
Vf Vf
32 0 R 32 0 R
Putting the values of Vi and Vf in eqn. (1)
qQ 1 11qQ 1 11qQ qQ
mv2 mv2
4 0 r 2 32 0 R 2 32 0 R 4 0 r
11qQ qQ qQ 11 1
mv 2 mv 2
16 0 R 2 0 r 2 0 8R r
qQ 11 R 2kqQ 3 R
v2 v2 1
2m 0 R 8 r mR 8 r
1
2kqQ r R 3 2kqQ r R 3 2
v2 v
mR r 8
mR r 8
g
Example-22 : The diagram shows a small bead of mass m carrying charge q. The beam can x
freely move on the smooth fixed ring placed on a smooth horizontal plane. In the same +Q C
plane a charge +Q has alos been fixed as shown. The potential at the point P due P
B
a 4a
to +Q is V.
The velocity with which the bead should projected from the point P so that it can
complete a circle should
be greater than
6qV qV 3qV
(A) (B) (C) (D) none
m m m
Sol. According to conservation principal of mechanical energy.
Ui + Ti = Uf + Tf
1 qQ
qV mv 20 0 ...(1) g
2 4 0 a x
where v0 is velocity of at point p v0
+Q C
the potential at the point p due to +Q is B a P
4a
Q
v
4 0 (4a)
from eqn (1)

www.physicsashok.in 17
ELECTROSTATICS
1
qv mv02 4qV
2
1
mv 20 3qV
2
6qV 6qV
v 20 v0 = Ans.
m m

Example-23 : Two spherical, nonconducting, and very thin shells of uniformly distributed Q Q
positive charge Q and radius d are located a distance 10d from each other. A d

positive point charge q is placed inside one of the shells at a distance d/2 from d/2

the center, on the line connecting the centers of the two shells, as shown in the figure. 10 d
What is the net force on the charge q ?
qQ qQ
(A) 361 d 2 to the left (B) 361 d 2 to the right
0 0
362qQ 360qQ
(C) 361 d 2 to the left (D) 361 d 2 to the right
0 0
Sol. Electric force on charge q due to sphre A is zero.
But electric force due to sphere B on charge q is Q Q
qQ A d/2 B
F 2
towards left
19 F q
4 0 d 19
2 d
2
qQ
towards left
361 0d 2
Y
Example-24 : The diagram shows three infinitely long uniform line charges placed on the 3l
X, Y and Z axis. The work done in moving a unit positive charge from (1, 1, 1) to
X
(0, 1, 1) is equal to
2l
(A) ( ln 2) /20 (B) ( ln 2)/0 (C) (3 ln 2) /20 (D) None l Z
Sol. Here r = 1+ x2
E1 = The magintude of electric field due to wire
3
along y-axis = directed paerpendicular to y-axis
2 0 1 x 2
E
E2 = The magnitude of electric field due to wire 1+ x2
l
along z-axis = directed perpendicular to z-axis.
1+ x2
The electric field due to wire along x-axis is directed perpendicular to x-axis.
3 cos
The net component of electric field along x-axis is E x cos
2 0 1 x 2 2 0 1 x 2
where = angle made by E1 with x-axis
and = angle made by E2 with x-axis
x
But cos cos
1 x2
4 x
E
x

2 0 1 x 2
www.physicsashok.in 18
ELECTROSTATICS
dV = Ex dx
x 0
4 x
or dV dx
x 1 2 0 1
x 2

v2 x0
4 x
dV 1 x 2
dx
v1
20 x1
x 0
4 x
V1 V2
2 0 x 1 1 x 2
dx

Put 1 + x2 = z
dz
or 2x =
dx
or dz = 2 xdx
x 0
2 x dz
V1 V2
0
z 2x
x 1
x 0
dz

0 x 1 z
x 0
ln z x1
0
2 0
ln(1 x)1
0
0
ln1 ln 21
0

v2 v1 ln 2
0

1
Example-25 : A particle of mass 1 kg & charge C is projected towards V
3 from
a non conducint fixed spherical shell having the same charge uniformly 1 mm
distributed on its surface. Find the minimum initial velocity of projection
required if the particle just grazes the shall.
2 2 2
(A) m/s (B) 2 m/s (C) m / s (D) none of these
3 3 3
Sol. Apply conservation principle of angular momentum,
m vd = mv0 r
vd v r
v0 d 0.5 mm
r 2 2
Applying mechanical energy conservation principle.
Ui + T i = Uf + T f
1 2 q2 1
0 mv mv02
2 4 0 r 2
2
1 q 1
or mv 2 mv02
2 4 0 r 2

www.physicsashok.in 19
ELECTROSTATICS
2 2
1 q 1 v
or mv 2 m
2 4 0 r 2 2
1 mv 2 q2
or mv 2
2 8 4 0 r
1 2 3 q2
or mv
2 4 4 0 r
1 q2
or mv 2
2 3 0 r
8q 2
or v2
3 4 0 mr
1
8 9 109 1012
9 8
3

3 1 1 10 3
8 2
v= = 2 m/s
3 3

ELECTRIC POTENTIAL ENERGY


Two like charges repel each other while the two unlike charges attract each other. So, when the charges are
moved away from each other or they are brought near each other, either some work is obtained or some work
is done. This work is stored in the system of charges in form of electric potential energy.
The electric potential energy of a system of different point charges is equal to the work done in bringing those
charges from infinity to form the system. It is represented by U.
The electric potential energy of a system of two point charges q1 and q2 in vacuum at a separation r is given by,
1 q1q 2
U J
4 0 r

Electric Potential Energy of a System of Charges :


The electric potential energy of a system of charges is given by
1 qiq j
U J
4 0 i j rij
This sum extends over all pairs of charges. We do not let i = j, because that would be an interaction of a charge
with itself and we include only terms with i < j to make sure that we count each pair only once.
For example, electric potential energy of four point charges q1, q2, q3 and q4 would be given by,
1 q 4q 3 q 4q 2 q 4q1 q 3q 2 q 3q1 q 2q1
U
4 0 r43 r42 r41 r32 r31 r21
Here, all the charges are to be substituted with sign.
n n - 1
NOTE : Total number of pairs formed by n point charges are .
2
q4 q3
Example-26 : Four charges q1 = 1C, q2 = 2C, q3 = 3C and q4 = 4C are kept
on the vertices of a square of side 1 m. Find the electric potential energy of this
1m
system of charges.
1m
Sol. In this problem, q1 q2

www.physicsashok.in 20
ELECTROSTATICS
r41 = r43 = r32 = r21 = 1m
and r42 r31 (1)2 (1)2 2 m
Substituting the proper values with sign in the relation given above in the theory, we get
(4)(3) (4)(1) (4)(1) (3)(2) (3)(1) (2)(1)
U = (9.0 109) (106) (106)
1 2 1 1 2 1
5

9.0 103 12
2
= 7.62 102 J
NOTE : Here negative sign of U implies that positive work has been done by electrostatic
force in assembling these charges at respective distances from infinity.
+q +q +q
a
Example-27 : Consider the configuration of a system of four charges each a
+q +q
of value +q. Find the work done by external agent in changing the
configuration of the system from figure (i) to fig (ii). +q a +q +q
fig (i) fig (ii)
Sol. Ui = Electrical potential energy in square arrangement
q
4q2 2q2
Ui a
4 0 a 4 0 2a 2a
Uf = Electric potential energy of circular arrangement of charge q a q

4q2 2q2 q
Uf
4 0 2a 4 0 (2a)
w ext U
q2
Uf Ui
4 0 a

3 2
Example-28 : A cone made of insulating material has a total charge Q spread
uniformly over its sloping surface. Calculate the energy required to take a A
test charge q from infinity to apex A of cone. The slant length is L. AB = L
Sol. w ext U U f U i B
q(VA V )
wext q(VA 0) qVA ...(1)
Here VA = Electric potential at point A.
Q2 rdx
The electric charge on the considered ring dq
RL
Electric potential due to consideredring is
dq
dV
4 0 x
Q2 rdx
dV RL4 x
0 dx
x
r R r q
sin A
x L

www.physicsashok.in 21
ELECTROSTATICS
R
Rx 2 Q x dx
r L
dV
L RL4 0 x
L
Q Q
VA dv dx = 2p L
2 0 L2 0 0

qQ
w ext q VA from eqn (1)
2 0 L

RELATION BETWEEN ELECTRIC FIELD AND POTENTIAL


In case of cartesian co-ordinates

E E i E j E k
x y z
V
Here, Ex (partial derivative of V w.r.t. x)
x
V
Ey (partial derivative of V w.r.t. y)
y
V
Ez (partial derivative of V w.r.t. z)
z
V V V
E i j k
x y z
This is also written as,

E grad V V

Example-29 : The electric potential in a region is represented as, V = 2x + 3y z, Obtain expression for electric
field strength.
V V V
Sol. E i j k
x y z
V
Here, 2x 3y z 2
x x
V
2x 3y z 3
y y
V
2x 3y z 1
z z
E 2 i 3 j k
In polar co-ordinates
V 1 V
Er and E .
r r
For example, electric potential due to a point charge q at distance r is
1 q
V .
4 0 r
V
E = 0 0


1 q
and Er . 2
4 0 r
www.physicsashok.in 22
ELECTROSTATICS
dV
Thus, electric field E
dr
or E = (slope of vr graph)


NOTE : If electric field E is known, then electric potential can be determined
from the relation as given below:

dV = -E . d r
b
or Vb -Va = - E . d r
a
Here, dr dx i dy j dz k
In uniform electric field
V = Ed

Example-30 : Find Vab in an electric field


N

E 2 i 3 j 4 k
C


where
r a i 2 j k m and r b 2 i j 2 k m

Sol. Here, the given field is uniform (constant). So using,

dV E . dr
a
or Vab Va Vb E . dr
b
1, 2, 1

2, 1, 2
2i 3j 4k . dx i dy j dz k
1, 2,1

2,1, 2
2dx 3dy 4dz
1, 2, 1
2x 3y 4z 2,1, 2
=1V
y E
,d,
0)
Example-31 : A uniform electric field having strength E is existing in xy plane as A(d

shown in figure. Find the p.d. between origin O & A (d, d, 0)


(A) Ed(cos + sin) (B) Ed(sin cos)
(C) 2 Ed (D) none of these o x
z

Sol. Here E E cos i E sin j

OA d i d j

v E
v Ed cos Ed sin
Ed(cos sin )
(VA V0 ) Ed(cos sin )
V0 VA Ed(cos sin )

Example-32 : Uniform electric field of magnitude 100 V/m in space is directed along the line y = 3 + x. Find the
potential difference between point A(3, 1) & B (1, 3)
(A) 100 V (B) 200 2 V (C) 200 V (D) 0
Sol. y=3+x
tan = 1

www.physicsashok.in 23
ELECTROSTATICS
= 45

E 100cos i 100sin j
100
E i 100 j
2 2

r AB i 3j 3i j 2 i 2 j

V E r
100 100
V i j r
2 2
100 100
V
2
i
2

j 2 i 2 j
V 100 2 100 2
VA - VB = 0

Example-33 : A, B, C, D, P and Q are points in a uniform electric field. The potentials a


these points are V (A) = 2 volt. V(P) = V(B) = V(D) = 5 volt. V(C) = 8 volt. The B C

0.2m
electric field at P is P Q
(A) 10 Vm1 along PQ (B) 15 2 Vm1 along PAA A D
1 1
(C) 5 Vm along PC (D) 5 Vm along PA 0.2m
V VD 2 5 30
Sol. Ex A 15 v / m
0.2 0.2 2
y
VA VB 2 5 30
Also E y 15 v / m
AB 0.2 2 15 P

E = - 15 i - 15 j x 45 x
A 15
| E |= ( - 15) 2 + ( - 15) 2 = 15 2 v / m
E PA 15 2 Hence (B) is correct.

Example-34 : Variation of electrostatic potential along x-direction is whosn in the


graph. The correct statement about electric field is v
(A) x component at point B is maximum
(B) x component at point A is towards positive x-axis
(C) x component at point C is along negative x-axis A B C
x
(D) x component at point C is along positive x-axis
Sol. The negative slope v x graph give x component of electric field. In the given graph, slope at C is
negative.
Hence, x component of electric field is positive.

EQUIPOTENTIAL SURFACE
Equipotential surface is an imaginary surface joining the points of same
potential in an electric field. So, we can say that the potential difference +q
between any two points on an equipotential surface is zero. The electric
lines of force at each point of an equipotential surface are normal to the
surface. Fig. shows the electric lines of force due to a point charge +q.
This spherical surface will be the equipotential surface and the electrical
lines of force emanating from the point charge will be radial and normal to the spherical surface.
www.physicsashok.in 24
ELECTROSTATICS
Regarding equipotential surface, following points are worth noting :
(a) Equipotential surface may be planar, solid etc. But equipotential surface never be a point.
(b) Equipotential surface is single valued. So, equipotential surfaces never cross each other.
(c) Electric field is always perpendicular to equipotential surface.
(d) electric lines of force cross equipotential surface perpendicularly.
(e) Work done to move a point charge q between two points on equipotential surface is zero.
(f) The surface of a conductor in equilibrium is equipotential surface.
(g) equipotential surface due to isolated point charge is spherical.

(h) Equipotential surface are planar in uniform electric field.

(i) Equipotential surface due to line charge is cylindrical.

(j) Equipotential surface due to an electric dipole is shown in the figure.

q +q


Example-35 : The electric field in a region is given by : E 4axy z i 2ax z j ax y / z k,
2 2

where a is
a positive constant. The equation of an equipotential surface will be of the form
(A) z = constant/[x3y2] (B) z = constant/[xy2]
(C) z = constant/[x4y2] (D) none
Sol. At equipotential surface potential is constant.
Therefore
- dv
E=
dr

dv E . dr

.d r constant
E

r x i y j z k dr dxi dyj dzk

E . (dx i dy j dz k)
= Constant

www.physicsashok.in 25
ELECTROSTATICS
(4axy z )i (2ax 2 z)j (ax 2 y / z)k . (dxi dyj dzk)
= constant

2 ax 2 ydz
4axy z dx 2ax z dy z = constant.
4ax 2 y z
2ax2 y z 2ax 2 y z = constant
2
2
6 ax y z = constant
constant constant
z= z
6ax 2 y x 4 y2

Example-36 : The equation of an equipotential line an electric field is y = 2x, then the electric field vector at (1, 2)
may be
(A) 4 i + 3 j (B) 4 i + 8 j (C) 8 i + 4 j (D) 8 i + 4 j
Sol. Electric field is perpendicular to equipotential line
y= 2x
dy
or =2
dx
m1 = 2
m1m2 = 1
1
m2 = -
2
1
tan where is angle with x-axis.
2
4 1
In option (D), tan
8 2

ELECTRIC DIPOLE
Electric dipole is a system in which two equal and opposite point q P +q
charges are placed at a small distance. The product of any of the
charges and distance between two charge is called electric dipole 2l
moment p. It is directed from negative charge to positive charge (fig.). The line joining the two charges is called
axis of dipole.
Let charges of an electric dipole are q and +q and are separated by a small distance 2l. The dipole moment
of such a dipole is given by
p = q 2l = 2ql

Electric Potential and Field due to an Electric Dipole


Let an electric dipole is placed along y-axis and its centre is at origin, then electric potential at point A(x, y, z)
due to this dipole.
2 2 2
x + z + (yl) A (x, y, z)
y
+q
2 2 2
l x + z + (y+l)
x
l
q
z

www.physicsashok.in 26
ELECTROSTATICS
1 q q
V 2
4 0 x (y l )2 z 2 x 2 (y l ) 2 z 2
V V V
Ex , Ey and E z
x y z

Special Cases :
(i) On the axis of dipole (axial position) : x = 0, z = 0
1 p
(a) V
4 0 (y l 2 )
2

1 p
V If y = r
4 0 (r l 2 )
2

1 P
or Vaxis If r > > l
4 0 r 2
(b) Ex = 0 = Ez and
E Ey
1 2py
2
4 0 (y l ) 2 2 (along p )
1 2pr
If y = r
4 0 (r l 2 )2
2

1 2p
E axis
4 0 r3
(ii) On the perpendicular bisector of dipole (equatorical position) :
Say along x-axis (it may be along z-axis also) y = 0, z = 0
(a) V bi sec tor 0
(b) Ex = 0, Ez = 0
Ey
1 p
and 4 0 (x l 2 )3/ 2
2 (opposite to p )

Here, negative sign implies that the electric field is along negative y-direction or antiparallel to p .
Further at a distance r from the centre of dipole (x = r), then
1 p
E . 2 2 3/ 2
4 0 (r l )
1 p
or E bi sec tor if r >> l
4 0 r 3
(iii) In polar co-ordinates (r, ) :
1 p cos
(a) V
4 0 r 2
E
V 1 . 2p cos E
(b) Er
r 4 0 r3 Er
1 V 1 p sin P r A (r, )
E . 3
r 4 0 r
O
p 2
E Er E 2 2
E 1 3cos
4 0 r 3
tan
(c) In Figure, tan
2

www.physicsashok.in 27
ELECTROSTATICS
y
Example-37 : 4 charges are placed each at a distance a from origin. The dipole moment of 3q
configuration is
(A) 2qaj (B) 3qaj 2q 2q x

(C) 2aq[i + j] (D) none q

Sol. Here P1 2q 2 a y
P1 = 2 2 qa 3q
P2 q 2a 2 qa 2a P1 P2 2a
45 45 2q
P3 q 2a 2 qa 45
x
2q P3
Px = P1 cos 45 - P2 cos 45 - P3 cos 45 2a
q
Px = 2 2qa cos 45 - 2qa cos 45 - 2qa cos 45
Px = 0
Py = P1 sin 45 + P2 sin 45 - P3 sin 45
2 2qa 2qa 2qa
Py = + - = 2qa
2 2 2

P 2 q aj

Example-38 : Two point charges +3 C and 3 C are placed at a small distance 2 103 m from each other.
Find:
(a) electric field and potential at a distance 0.6 m from dipole in its equatorial position.
(b) electric field and potential at the same distance from dipole as in (a) if the dipole is rotated through 90.
Sol. Dipole moment
p = q 2l = (3 106) (2 103)
p = 6 109 C-m
(a) Electric field in equatorial position,
1 p
E . 3
4 0 r
6 109

E 9 109
(0.6)3
E = 250 N/C
Electric potential, V = 0
(b) On rotating the dipole through 90, the same point now will be in axial position.
So, electric field
1 2p
E . 500 N / C
4 0 r 3
and electric potential
1 p
V
4 0 r 2
9
9 6 10

V 9 10
(0.6)2
= 150 V

www.physicsashok.in 28
ELECTROSTATICS
Example-39 : A short electric dipole is situated at the origin of coordinate axis with its axis along x-axis and
equator along y-axis. It is found that the magnitudes of the electric intensity and electric potential due to
the dipole are equal at a point distance r = 5 m from origin find the position vector of the point.
Sol. Let P be such a point at distance r and angle from equator.
Now |EP| = |VP|
KP K sin
or 3
1 3sin 2 P 2
r r
2
1 3sin y
or sin P
5 r
Squaring both nets q
1 + 3 sin2 = 5 sin2 x
1
or sin
2
or = 45

Positive vector of r point P is
5
r=
2
( )i+j

Electric Dipole in Uniform Electric Field :


(i) Torque : When a dipole is placed in a uniform electric field as shown in Fig. the net force on it,

F qE q E 0
while the torque,
+q
qE
P
E
q l
qE
= qE 2l sin

pE sin

or pE
From this expression it is clear that torque acting on a dipole is maximum (= pE) when dipole is perpendicular
to the field and minimum (= 0) when dipole is parallel or antiparallel to the field.

NOTE :If the electric field is not uniform the dipole will experience both a resultant force
and a torque so its motion will be combined translatory and rotatory (if 0 or 180)

(ii) Work : Work done in rotating a dipole in a uniform field through a small angle d will be,
dW = d= pE sin d
So, work done in rotating a dipole from angular position 1 to 2 with respect to field,
2
W pE sin d pE [cos 1 cos 2 ]
1

So, if a dipole is rotated from field direction, i.e., 1 = 0 to position , i.e., 2 = .


W = pE[1 cos]

www.physicsashok.in 29
ELECTROSTATICS
E E
q =0 +q = 180
+q q
P P
Wmin = 0 Wmax = 2pE
(a) (b)
(iii) Potential energy : In case of a dipole (in a uniform field), potential energy of dipole is defined as work done
in rotating a dipole from a direction perpendicular to the field to the given direction.
i.e., U = (W W90)
U = pE (1 cos ) pE (1 cos 90)

U pE cos p . E
(iv) Angular SHM : A dipole when placed in a uniform electric field, align itself parallel to the field. If it is given a
small angular displacement about its equilibrium position, the restoring torque produced in it will be,
= pE sin = pE ( sin )
d 2 d2 2 2 pE
or I 2 pE or with
dt dt 2 I
2
This is standard equation of angular SHM with period T . So, dipole will execute angular SHM with

time period
I
T 2
PE

Interaction of Two Dipoles :


If a dipole is placed in the field of other dipole, then depending on the positions of dipoles relative to each other,
force, torque and potential energy are different.

TABLE : Dipole-Dipole Interaction


S.No. Relative position of Dipole Force F Torque Potential Energy U

r
1 6p1p 2 1 2p1p 2
1. (along r) zero
p1 p2 4 0 r4 4 0 r 3
F F

p1 p2 1 3p1p 2 1 p1p 2
2. r
F F 4 0 r 4 (along r) zero 4 0 r 3

p2 F
p1 1 3p1p 2 1 2p1p 2
3. r on p1 , 4 r 3 zero
4 0 r 4 0
F
1 p1p 2
(perpendicular to r) on p 2 , 4 r 3
0

Example-40 : Two point charges +3.2 1019 C and 3.2 1019 C are placed at a distance 2.4 1010 m, from
each other. This dipole is placed in a uniform electric field of 4.0 105 V/m.
(a) Find the necessary torque required to rotate the dipole through 180 from its equilibrium position.
(b) What is the work done in rotating the dipole through 180 ?

www.physicsashok.in 30
ELECTROSTATICS
(c) What is the potential energy of dipole in this position ?
Sol. Electric dipole moment
p = q 2l
Here, q = 3.2 1019 C
2l = 2.4 1010 m
p = (3.2 1019) (2.4 1010)
p = 7.68 1029 C-m
(a) necessary torque
= pE sin
= 7.68 1029 4 105 sin 180
= 0 ( sin 180 = 0)
(b) Work done in rotating the dipole through 180 is given by
W = pE (cos1 cos2)
Here, 1 = 0, 2 = 180, p = 7.68 1029 C-m
and E = 4.0 105 V/m
W = 7.68 1029 4 105 (cos 0 cos 180)
W = 7.68 1029 4 105 (1 + 1)
W = 6.14 1023 J
(c) The potential energy of dipole in this position is given by
U = pE cos
where is the angle between the axis of dipole and electric field.
In equilibrium position, = 0
U0 = pE
or U0 = (7.68 1029) (4.0 105)
U0 = 3.07 1023 J
+q
Example-41 : A wheel having mass m has charges +q and q on diametrically opposite E
points. It remains in equilibrium on a rough inclined plane in the presence of uniform
q
vertical electric field E =
mg mg mg tan q
(A) (B) (C) (D) none
q 2q 2q
Sol. The torque of electric force about centre is balanced by torque due to friction about the centre.
r f = PE sin E
P
= PE sin 90-
+q
But f = mg sin (for equilibrium) f
-q mg
mgr sin = PE sin 90- sin
mgr mgr
E
P q 2r
P = q 2r (dipole moment)
mg
E
2q

www.physicsashok.in 31
ELECTROSTATICS
Example-42 : A nonconducting ring of mass m and radius R is charged as shown. y
+
The charged density i.e. charge per unit length is . It is then placed on a rough ++
nonconducting horizontal surface plane. At time t = 0, a uniform electric +

x
field E = E 0i is swithced on and the ring start rolling without sliding.

Determine the friction force (magnitude and direction) acting on the ring,
when it starts moving.
Sol. Where f is friction surface.
f = macm ...(1)
Let d element on ring then dq d
y
d +
dq Rd { d } +
+
R + x
d dP E p

d 2RdqE sin { dp = (2Rdq)}

f
d 2RE R sin d
/2 rough non-conducting
2 surface
E 2R E sin d
0 dp
2 p- q
E 2R E
dq E
2R2 E fR = mR2 dq q
q
2RE f = mR
2RE macm = mR (from eqn (1)) dq
for pure rolling, f

acm = R
2RE = macm + mR
2RE = 2 macm
macm = RE
f ma cm RE 0

Example-43 : In the figure shown S is a large nonconducting sheet of uniform charge density S R
. A rod R of length l and mass m is parallel to the sheet and hinged at its mid point. s
xl
The linear charge densities on the upper and lower half of the rod are shown in the -l
figure. Find the angular acceleration of the rod just after it is released.

Sol. Electric field due to non-conducting sheet is 2 .
0
d = x E dq 2 = 2x Edq
/ 2 /2
x2
d 2x E dx 2E
0 2 0
2E 2 E 2

8 4 S +dq
s Edq
E2 x
I x
4
Edq dq
m 2
E 2
or
12 4

www.physicsashok.in 32
ELECTROSTATICS
E 2
12 3E 3

4 m 2
m 2 0 m

ELECTRIC FLUX
Electric flux through an elementary area dS is defined as the scalar product of area and field, i.e.,
n E
dE E . dS
dE = EdS cos

i.e., E E . dS dS
It represents the total lines of force passing through the given area.
Regarding electric flux following points are worth noting : E
n
(a) Electric flux will be maximum when cos = 1, i.e., = 0, i.e., electric field is
normal to the area with (dE)max = EdS.

(d E)max = EdS
n
(b) Electric flux will be minimum when cos = 0, i.e., = 90, i.e., field is E
parallel to the surface with
(dE)min = 0.
(d E)min = 0
(c) For a closed body outward flux is taken to be positive [Fig. (a)] while inward negative [Fig. (b)]

(a) Positive flux (b) Negative flux

GAUSSS LAW
This law gives a relation between the net electric flux through a closed surface and the charge enclosed by the
surface.
According to this law, the net electric flux through any closed surface is equal to the net charge inside the
surface divided by 0. In symbols it can be written as,
q
E E . n dS in ...(i)
S
0
but this form of Gausss law is applicable only under following two conditions :
(i) The electric field at every point on the surface is either perpendicular or tangential.
(ii) Magnitude of electric field at every point where it is perpendicular to the surface has a constant value (say E).
Here, S is the area where electric field is perpendicular to the surface.

Applications of Gausss Law :


To calculate E we choose an imaginary closed surface (called Gaussian surface) in which Eqs. (i) or (ii) can be
applied easily. In most of the cases we will use Eq. (ii). Let us discuss few simple cases.

www.physicsashok.in 33
ELECTROSTATICS
(i) Electric field due to a point charge :
q
From Eq. (ii), ES in
0
Here, S = are of sphere = 4r2 q
E
and qin = charge enclosing the Gaussian surface = q r
q

E 4r 2
0
1 q
E
4 0 r 2
It is nothing but Coulombs law. +
+
(ii) Electric field due to a linear charge distribution : +
+
Consider a long line charge with a linear charge density (charge per unit
length) . We have to calculate the electric field at a point, a distance r r
from the line charge. We construct a Gaussian surface, a cylinder of any lE E
arbitrary length l of radius r and its axis coinciding with the axis of the
line charge.
+
Hence, we can apply the Gausss law as, +
+
+
q
ES in
0

E
E

Curved Surface Plane Surface


Here, S = area of curved surface = (2rl)
and qin = net charge enclosing this cylinder = l
l
E(2l )
0

E
2 0 r
1
i.e., E
r
(iii) Electric field due to a plane sheet of charge :
From Gausss Law
q
ES in + + +
0 + + + + + + S0
+ + + + + +
( ) (S0 ) E + + + E
E(2S0 ) + + + + +
0 + + +
+ +
+ + + +

E
2 0
(iv) Electric field near a charged conducting surface :
This is similar to the previous one the only difference is that this time charges are on both sides. Hence, applying
q
ES in
0

www.physicsashok.in 34
ELECTROSTATICS
Here, S = 2S0 and qin = () (2S0) +
+ + ++++ S0
( ) (2S0 ) ++ + + ++++
E(2S0 ) ++ + +++ +
0 E ++ +
+ + E
++ + + ++
++ +
++++
E + +
0 ++ + + ++
++

Example-44 : Which of the following statements is/are correct ?


(A) Electric field calculated by Gauss law is the field due to only those charges, which are enclosed inside
the Gaussian surface.
(B) Gauss law is applicable only when there is symmetrical distribution of charge.
(C) Electric flux through a closed surface is equal to total flux due to all the charges enclosed within that
surface only.
(D) None of these

Sol. Since, electric field at a point is equal to electric flux passing per unit area, therefore,
dS is the net
E.
flux emanating from a closed surface. Though net flux through the closed surface depends upon the
charges enclosed in that surface only but electric field E at a point depends not only upon charges enclosed
but it depends upon charges lying outside the surface also. Hence (A) is wrong.
Gauss law is applicable to a closed surface. The surface may have any shape. It means, it is a general law.
Hence (B) is wrong.
q
Gauss law is E. dS . It means, net flux through a closed surface depends upon q. But it is equal
0
to net charge enclosed within the surface only. Hence (C) is correct.
Obviously (D) is wrong.
E

Example-45 : In a region of uniform electric field E, a hemispherical body is


placed in such a way that field is parallel to its base (as shown in figure).
O
The flux linked with the curved surface is :
C
2 2
R2
(A) zero (B) R E (C) R E (D) E
2
Sol. The flux linked with the body is zero as it does not enclose any charge. E
= cs + pb = 0
As field is parallel to base, the flux linked with base
pb = E R2 cos 90 = 0
paralled to base = 0
n
curved surface = 0

Example-46 : If a point charge q is placed at the centre of a cube what is the flux linked (a) with the cube (b) with
each face of the cube ?
Sol. (a) According to Gausss Law,
q q
total in
0 0
(b) Now as cube is symmetrical body with 6-faces and the point charge is at its centre, so electric flux linked with
each face will be
q
total
6 6 0

www.physicsashok.in 35
ELECTROSTATICS

NOTE : If charge is not at the centre of cube (but any where inside it), total flux will not
change but the flux linked with different faces will be different.

Example-47 : If a point charge q is placed at one corner of a cube, what is the flux linked with the cube ?
Sol. By placing three cubes at three sides of given cube and four cubes above, the charge will be in the centre. So,
the flux with each face will be one-eight of the flux (q/0).
q
Flux through the cube 8
0

Example-48 : Calculate flux through the cube and flux through the each surface of cube
when q is placed at one of its corner.

Sol. To cover charged particle completly we have to use 7 extra cubes around the charged particle.
q
So, Flux Through cube ( )
8 0
q
Flux through ABCD 3 24 F G
0 q
q H
E
Flux through ABFE 3 24 B C
0
q A D
Flux through ADEH
3 24 0
Flux through EFGH = 0
Flux through FGCB = 0
Flux through GCDH = 0

E x
0
Example-49 : The electric field in a region is given by = E i . Find the charge contained inside a cubical
l
volume bounded by the surfaces x = 0, x = a, y = 0, y = a, z = 0 and z = a. Take E0 = 5 103N/C, l = 2cm
and a = 1cm.
E x
0
Sol. E i

q
Total flux
0
q
EA where A is area of surface
0
E 0a q E0a3 q
a2
0 0
3

q
E 0a 3
0 q

5 103 8.85 10 12 1 10 2
2 10 2
5 103 8.85 104
q q 2.2 1012 C
2

www.physicsashok.in 36
ELECTROSTATICS
ELECTRIC FIELD AND POTENTIAL DUE TO CHARGED SPHERICAL SHELL OR SOLID
CONDUCTING SPHERE
Electric Field

The all points inside the charged spherical conductor or hollow spherical shell, electric field E 0, as there is
no charge inside such a sphere. We can construct a Gaussian surface (a sphere) of radius r > R.
From Gausss Law
q + q
C . ds 0
E + +
+
+
q 1 q + E
E(4r 2 ) E + r R
0 4 0 r 2
+
+
+ + +
Hence, the electric field at any external point is the same as if the total Gaussian
charge is concentracted at centre. surface
At the surface of sphere r = R,
1 q
E
4 0 R 2
Thus, we can write, Einside = 0
1 q
E surface
4 0 R 2 E
1 q
E outside 1 q =
4 0 r 2 R
2
E r2
1
The variation of electric field (E) with the distance from the
centre (r) is as shown in fig. r
O R

NOTE : At the surface graph is discontinuous.


1 q q/4R 2
Esurface = = =
40 R 2 0 0
Potential :
As we have seen,
1 q
E outside
4 0 r 2
dVoutside 1 q dV
2

E
dr 4 0 r dr
V q r dr
0 dVoutside 4 0 r 2 (V = 0)
1 q 1
V or V
4 0 r r
Thus, at external points the potential at any point is the same when the whole charge is assumed to the concentrated
at the centre. At the surface of the sphere, r = R.
1 q
V
4 0 R
At some internal point electric field is zero everywhere, therefore, the potential is same at all points which is
equal to the potential at surface. Thus, we can write,
1 q
Vinside Vsurface
4 0 R

www.physicsashok.in 37
ELECTROSTATICS
1 q V
and Voutside
4 0 r 1 q
= R 1
The potential (V) varies with the distance from the R V r
centre (r) as shown in Fig.
r
O R
Example-50 : The diameter of hollow metallic sphere is 60 cm and charge on sphere is 500 C. Find the electric
field and potential at a distance 10 cm from centre of sphere.
Sol. The point at 10 cm distance from centre of sphere will be inside the sphere. Inside hollow conducting sphere,
electric field is zero every where and potential is uniform (same as at the surface).
Hence, intensity of electric field E = 0
1 q
and potential V 4 R
0
Here, q = 500 C = 500 106 C and R = 30 cm = 0.30 m
500 106
V
9.0
10 9

0.30
7
V = 1.5 10 V

ELECTRIC FIELD AND POTENTIAL DUE TO A UNIFORMALY NON-CONDUCTING SPHERE


Electric field :
Let positive charge q is uniformly distributed throughout the volume of a solid sphere of radius R. We have to
find the intensity of electric field due to this charged sphere at point P distance r from centre O.
Applying Gausss law
q
ES in ...(i)
0
4
Here, S = 4r2 and q in () r 3
3
q
Here, = charge per unit volume =
4/3R 3
Substituting these values in Eq. (i), we have
1 q
E . 3 . r or
4 0 R Er
At the centre r = 0, so, E=0
1 q
At surface r = R, so, E
4 0 R 2
To find the electric field outside the charged sphere, we use a spherical Gaussian surface of radius r(>R). This
surface encloses the entire charged sphere, so qin = q, and Gausss law gives.
q 1 q 1
E(4r 2 ) or E E 2
0 4 0 r 2 or r
Thus, for a uniformly charged solid sphere, we have the following formulae for magnitude of electric field.
1 q
E inside . 3 .r E
4 0 R
1 q 1 q
E surface . R2 r E r12
4 0 R 2
E
1 q r
E outside .
4 0 r 2 O R

www.physicsashok.in 38
ELECTROSTATICS
The variation of electric field (E) with the distance from the centre of the sphere (r) is shown in fig.

Potential :
The field intensity outside the sphere is,
1 q
E outside . 2
4 0 r
dVoutside
E outside
dr
V 1r q

dVoutside
4
. 2 dr
0 r
1 q
V as V 0
4 0 r
1
or V
r
1 q
At r = R, V
4 0 R
1 q
i.e., at the surface of the sphere potential is VS 4 R .
0
The electric intensity inside the sphere,
1 q
E inside . 3 .r
4 0 R
dVinside
E inside
dr
V 1 q r
VS dVinside 40 R 3 R r dr
r
1 q r2
V VS
4 0 R 3 2 R
Substituting
1 q
VS . , we get
4 0 R
1 q 3 1 r2
V .
4 0 R 2 2 R 2
3 1 q 3
At the centre r = 0 and VC 2 4 R 2 VS, i.e., potential at the centre is 1.5 times the potential at
0
surface.
Thus, for a uniformly charged solid sphere we have the following formulae for potential.
1 q
Voutside
4 0 r
1 q
Vsurface V
4 0 R 3 1 q
1 q 3 1 r2 2 R
and Vinside q
4 0 R 2 2 R 2
1
R
The variation of potential (V) with distance form the
centre (r) is as shown in figure.
www.physicsashok.in 39
ELECTROSTATICS
Example-51 : The radius of a solid metallic non-conducting sphere is 60 cm and charge on the sphere is 500 C.
Find the electric field and potential at a distance 10 cm from centre of sphere.
Sol. The point 10 cm from centre of sphere will be inside the sphere. Hence,
1 qr
E inside . 3
4 0 R
6
Here, q = 500 10 C, r = 10 cm = 0.10 m
R = 60 cm = 0.6 m
500 106 0.10
E inside 9.0 109

(0.60) 3
2 106 N / C
2
1 q 3 1 r
and Vinside
4 0 R 2 2 R 2
6
) 3 1 (0.10) 2
10

9 109 (500(0.60)
2 2 (0.60) 2

3 1
7.5 106
2 72
107
7.5 106
72
7
= 1.1 10 V

Example-52 : A solid non conducting sphere of radius R has a non-uniform charge distribution of volume charge
r
density, 0 , where 0 is a constant and r is the distance from the centre of the sphere. Show that:
R
(a) the total charge on the sphere is Q = 0R3 and
KQr 2
(b) the electric field inside the sphere has a magnitude given by, E .
R4
Sol. (a) dv = 4r2dr
0r
dq dV 4 r 2dr
R
40 3
r dr
R
R
40 3 R
r
R 0
Q r dr
dr

40 R 4
Q Q 0 R 3
R 4
r
40 3
R 0
(b) q in r dr

40 r 4 40 r 4

R 4 4R
q in
E
4 0 r 2
40 r 4
E
4R 4 0 r 2
But Q = 0R3
www.physicsashok.in 40
ELECTROSTATICS
Q 1
0 and K
R3 4 0
KQr 2
E
R4

Example-53 : Three concentric metallic shells A, B and C of radii a, b and c (a < b < c) have surface charge
densities , and respectively.
(i) Find the potential of three shells A, B and C.
(ii) If the shells A and C are at the same potential, obtain the relation between the radii a, b and c.
Sol. The three shells are shown in figure.
(i) Potential of A = (Potential of A due to + on A)
+ (Potential of A due to on B)
+ (Potential of A due to + on C)
4 a 2 4 b2 4 c 2
K
a b c

a b c c
0
Potential of B = (Potential due to + on A) + (Potential due to on B) b
+ (Potential due to + on C) C B A a
4 a 2 4 b2 4 c 2 +s
K
a b c
-s
a2
b c +s
0 b
Potential of C = (Potential due to + on A) + (Potential due to on B) + (Potential due to + on C)
4 a 2 4 b2 4 c 2
K
a b c
a 2 b2
c
0 b c
(ii) Given that VA = VC
a 2 b2
or
0
a b c
0 b
c
c
2 2
a b
or abc c
b c
Solving we get, c = (a + b).

Example-54 : An electric field converges at the origin whose magnitude is given by the expression E = 100 N/C,
where r is the distance measured from the origin.
(A) total charge contained in any spherical volume with its centre at origin is negative.
(B) total charge contained at any spherical volume, irrespective of the location of its centre, is negative.
(C) total charge contained in a spherical volume of radius 3 cm with its centre at origin has magnitude
31013 C.
(D) total charge contained in a spherical volume of radius 3 cm with its centre at origin has magnitude
3109 C.

www.physicsashok.in 41
ELECTROSTATICS
Sol. Since, electric lines of forces are terminated into the sphere. E
Hence, total charge contained by should the sphere should be negative. E
So, options (A) and (B) are correct E
E
q
E
4 0 r 2
Here r = 3 102C
q = 3 1013 m Hence, option (C) is correct.

Example-55 : Figure shows a section through two long thin concentric cylinders of
radii a & b with a < b. The cylinders have equal and opposite charges per unit b
length . Find the electric field at a distance r from the axis for a

(A) r < a (B) a < r < b (C) r > b


b

Sol. a -l
l

(A) r < a b a
0
E
r
E ds
C 0

O
E 2 r
0
E=0 b
(B) a < r < b
q a
r
C E ds 0

E 2 r
0

E
2 0 r
(C) r > b


E ds
C
0
0 b
a
E 2 r 0 r

E0

THINKING TYPE PROBLEMS


1. The electric charge of macroscopic bodies is actually a surplus or deficit of electrons. Why not protons?
2. What are insulators and conductors?
3. A charged rod attracts bits of dry cork dust which, after touching the rod, often jump away from it violently.
Explain.
4. A truck carrying explosives has a metal chaing touching the ground. Why ?
5. How is the Colulomb force between two charges affected by the presence of a third charge ?
6. A person standing on an insulating stool touches a charged insulated conductor. Is the conductor completely
discharged ?
7. Electric lines of fore never cross. Why ?
www.physicsashok.in 42
ELECTROSTATICS
8. The electric field inside a hollow charged conductor is zero. Is this true or false ?
9. The electric field inside a hollow charged spherical conductor is the same at all points and is equal to the field at
the surface. Is this true or false ?
10. The electric potential inside a hollow charged spherical conductor is the same at all points and is equal to the
potential of the surface. Is this true or false ?
11. A charge A situated outside an uncharged hollow conductor experiences a force if another charge B is placed
inside the conductor, but B does not experience any force. Why? Does it not violate the third law of motion ?
12. If only one charge is available, can it be used to obtain a charge many times greater than it in magnitude?
13. In the previous question, does it make a difference which face of B is touched in order to remove the free
charge (include positive charge) from B?
14. Can a small spherical body of radius 1 cm have a static charge of 1 C?
15. A metal leaf is attached to the internal wall of an electrometer insulated from
the earth. The rod and housing of the electrometer are connected by a wire,
and then a certain charge is imparted to the housing. Will the leaves of the
electrometer be deflected ? What will happen to the leaves if the wire is
removed and the rod is then charged ?
16. A broad metal plate is connected to the earth through a galvanometer. A positively
charged ball flies along a straight line above the plate at a distance much less than
the linear dimensions of the plate. Draw an approximate diagram showing how the
current flowing through the galvanometer depends on time.
17. The work done in carrying a point charge from one point to another in an electrostatic field depends on the path
along which it is taken. Is this true of false? Briefly explain your answer.
18. The figure shows lines of constant potential in a region in which on electric
field is present. The values of the ptentials are written in brackets. Of the points
A, B and C, the magnitude of the electric field is greatest at the point...... Give
reasons for your answer in brief.
19. Can two balls with like charges be attracted to each other?
20. The housing of the electrometer of question no.15 is given a charge. Will the leaves of the electrometer be
deflected in this case? Will the deflection change if the rod is earthed ? There is no connecion between the rod
and the housing.
21. By touching different points of a metal bucket with a test ball B connected by a wire to an earthed electrometer
it can be observed that deflection of the leaves of the electrometer is the same for any position of the ball. But
if the wire is removed and the charge is transferred by the ball to the ball of the electrometer, the deflection
depends on which surface of the bucket (external or internal) is touched before that. Why ?
22. The gravitational field strength is zero inside a spherical shell of matter. The electric field strength is zero not only
inside an isolated charged spherical coonductor but inside an isolated conductor of any shape. Is the gravitational
field inside, say, a cubical shell of matter zero ? If not, in what respect is the analogy not complete ?
23. Can two equipotential surfaces intersect ?
24. An isolated, conducting spherical shell of radius R carries a negative charge Q. What will happen if a small,
positively charged metal object is placed inside and connected by a wire ? Assume that the positive charge is
(a) less than, (b) equal to and (c) greater than Q.
25. An uncharged metal sphere suspended by a silk thread is placed in a uniform external electric field E. What is
the magnitude of the electric field for points inside the sphere ? Will your answer change if the sphere carries a
charge?
26. Is charge uniformly distributed over the surface of an insulated conductor of any shape ? If not, what is the rule
for the distribution of charge over a conductor?
27. Are equipotential surfaces which arise due to a point charge and whose potentials differ by a constant amount
(say 1 volt) evenly spaced in radius ?
28. Two point charges Q and 4Q are fixed at a distance of 12cm from each other. Sketch the lines of force and
www.physicsashok.in 43
ELECTROSTATICS
locate the neutral point, if any.
29. A charge +Q is fixed at a distance d in front of an infinite metal plate. Draw the lines of force indicating the
direction clearly.
30. Indicate the surface distribution of charge on a square metal plate by using dots in
such a way that the greater the surface density of charge, the farther away the dots
are from the plate (shown in fig.).

31. A metal plate is moved with a constant acceleration a parallel to its plane. Does any charge develop on its
surfaces which are perpendicular to its motion?
32. Is an electric field of the type shown by the electric lines in the figure
physically possible?

33. Zero work is done when a charged particle is transferred from one equipotential surface to another. Is this true
or false ?
34. A potential difference V is set up between a filament emitting electrons in a vaccum tube and a thin metallic ring.
The electron beam passes past the ring through its central region without spreading. The kinetic energy of the
electrons in the beam increases while the battery producing the potential difference V performs no work, since
no current flows through the circuit.
How can his fact be reconciled with the principle of conservation of energy?
35. There is an electric field near the surface of a conductor carrying direct current. Is this true or false?
36. Ordinary rubber is an insulator. But the special rubber tyres of aircrafts are made conducting. Why is this
necessary?
37. A small sphere is charged to a potential of 50V and a big hollow sphere is charged to a potential of 100V. How
can you make electricity flow from the smaller sphere to the bigger one?
38. An electric dipole is placed in a non-uniform electric field. Is there a net force on it?
39. A point charge is placed at the centre of a spherical gaussion surface. Does electric flux e change
(a) if the sphere is replaced by a cube of the same volume,
(b) if a second charge is placed near, and outside, the original sphere, and
(c) if a second charge is placed inside the gaussion surface?
40. A spherical rubber balloon carries a charge that is uniformly distributed over its surface. As the balloon is blown
up, how does E vary for points (a) inside the balloon, (b) on the surface of the balloon, and (c) outside the
balloon?
THINKING PROBLEMS SOLUTION
1. Because protons are tightly bound in the nucleus. They cnnot be removed from there easily.
2. Conductors are materials in which there are a few free electrons per atom of the matter. Insulators are materials
in which the electrons are not as free as in conductors.
3. A charged rod first attracts the dust by producing unlike charge at the near end and like charge at the far end.
When the cork dust touches the rod, however, it acquires like charge and so is repelled strongly by the charge
on the rod.
4. To conduct away the charge produced by friction. This charge, unless conducted away to the earth, may
produce sparks causing the explosives in the truck to catch fire and explode.
5. The force on the charge due to another does not depend on the presence of other charges, that is, electrical
forces are physically independent.
6. No, the man acts as a conducting body. He shares charge with the charged body.
7. Because if they cross, the electric field at the point of intersection will have two directions simultaneously, which
is physically impossible.
8. It is true. The electric field inside a hollow charged conductor is zero. This follows from the condition that all
points on conductor have the same potential.

www.physicsashok.in 44
ELECTROSTATICS
9. It is false. The field inside is zero but on the surface, the field is finite. 10. It is true.
11. This is so because there is an electric field over A but there is no electric field over B as it lies inside a hollow
conductor. No, this is not in violation of the third law of motion. In fact, force arises between A and the hollow
sphere, the charge B is simply an internal part of the sphere.
12. Yes, by repeating the induction process. Place an insulated conductor B
close to the charged body. A. y induction, an equal unlike charge is induced
at the near end of B and equal like charge at the far end. Touch the body
B with your finger tip in the presence of A. The like charge at the far end
will flow to the earth. But the unlike charge at the near end will remain
bound due to the force of attraction. Now remove the body B to a distant place.
The unlike charge, equal in magnitude to the inducing charge, will spread over the surface of B as there is no
force of attraction to keep it on one side. Next, deliver this charge to a big body by conduction, that is, by
touching this body with B. Repeat the process after completely discharging B. To obtain a similar charge in
large magnitude, first deliver unlike charge to a body C and then repeat the induction process between C and
the body in which like charge is to be stored.
13. No. The free charge will go to the earth via the finger, irrespective of which part of B is touched, because the
free positive charge in B exists at a higher potential due to the positive charge on A. It is a fact that positive
charge always flows from higher to lower potential, irrespective of the path.
1
14. E 910
9
91013 N C1 . The field is every large. Air cannot sustain such a strong electric field, so a
0.01
small body cannot have a charge of 1 coulomb.
15. The housing and the rod connected together will have the same potential, so the leaves will not be deflected.
After the wire is removed and the rod is charged, both the leaves will be deflected because of the potential
difference between the rod and the housing.
16. As the charge approaches the plate, electrostatic induction causes the
induced positive charge to pass into the earth, while the induced
negative charge accumulates on the upper surface of the plate. A positive
current pulse passes through the galvanometer. No current is produced
when the charge moves above the plate. An opposite current is produced when the charge moves away from
the plate.
17. It is false. the work done in carrying a point charge from one point to another does not depend on the path
along which it is carried because electric field is conservative.
18. It is greatest at the point B. Since the electric field is the rate of fall of potential, the stronger the field, the closer
the equipotential surfaces. In the figure, the equipotential surfaces are closest in the neighbourhood of B, so the
field at B is the greatest.
19. They can, if the charge of one ball is much greater than that of the other. The forces of attraction caused by the
induced charges may exceed the forces of repulsion.
20. When the housing of the electrometer is given some charge, the potential of the rod cannot be the same as that
of the interior of the housing because the rod lies partly outside. On account of the potential difference between
the rod and the housing, the leaves will be deflected.
When the rod is earthed, the potential difference between the rod and the housing is increased further, so the
deflection will be greater.
21. The electrometer measures the potential diference between the body and
the housing of the electrometer. Since the surface of the bucket is
equipotential, the leaves show the same deflection wherever the testing
ball B may touch the surface of the bucket. But when the ball B is
disconnected from A, the ball B collects charge by conduction. When it
delivers this charge to A, the potential of the leaves (and also of the rod)
is raised above the potential of the housing, so there is deflection of the leaves. When B touches the inner

www.physicsashok.in 45
ELECTROSTATICS
surface, it collects no charge because there is no potential difference between it and the inner surface of the
bucket. Unless there is a potential difference, there can be no flow of charge from one body to another. So no
deflection takes place when the ball B touches the inner surface and then touches A. But when it touches the
outer surface, it collects a certain amount of charge, so there is deflection.
22. No, the field inside a cubical shell of matter is not zero. The analozy is not complete in respect of distribution of
mass and charge. Mass is uniformly distributed but charge is not distributed uniformly. It is more concentrated
at the edges and corners.
23. No, two equipotential surfaces can never intersect because if they intersect at a point, the electric field at that
point can have two directions simultaneously, which is physically impossible.
24. For the sake of simplicity, let us consider the metal body to be a sphere placed at the centre of the shell.
1 Q q
VA (potential of the surface of the shell) 4 R
0
where R = radius of the shell and q is the charge of the metal body.
1 q Q
VB (potential of the metal body) 4 r R
0
q 1 1
V V
40 r R
B A

VB VA
So charge will flow from the metal body to the shell. Since VB = VA at equilibrium, q must be reduced to zero,
i.e., the entire charge on the metal body must flow to the outer shell.
qQ
V, the common potential 4 R . Thus, (a) when q < Q, V is negative, (b) when q = Q, V = 0, (c) when q
0
> Q, V is positive.
E
25. The field inside the metal sphere at all points is zero because r is infinity for metals and r 0 , So E = 0.
E
No, the answer does not change if the sphere carries a charge.
26. Charge is not distributed uniformly over the surface of a conductor of any shape. Charge is distributed according
to the curvature of the surface. The greater the curvature, the greater the surface density of charge.
27. No.
q 1 1 q r2 r1
V1 V2
40 r1 r2 4 0 r1r2
qr
1
or 4 0 r1 r1 r where r r2 r1
Obviously, r is not constant. So the equipotential surfaces differing by 1 V are not equispaced.
28. The lines of force are shown in the figure. Here, all the lines near the
edge of the figure will appear to radiate uniformly from the point P,
the centre of gravity of the charges. At the neutral point N, the total
field is zero. Let it be at a distance x from 4Q.
1 4Q 1 Q
Then
4 0 x 2
4 0 12 x 2 or x = 8 cm
29. The lines of force are shown in the figure.
Explanation Since it is a metal plate, its surface is an equipotential surface.
So lines of force must terminate normally on it.

Note : One every important result follows from this map of electric flued due to
a point charge and the induced charges on a metal plate. If a charge Q is placed as far behind the plate as +Q

www.physicsashok.in 46
ELECTROSTATICS
is in front and the plate is removed, AB (position of the plate) is still an equipotential surface. So for interaction
between +Q and the plate or for electric fields between them, we may use the formalism of replacing the plate
by Q. This is called the electrical image of +Q.
30. The distribution of charge is shown in the figure.
Explanation Since the surface density of charge is proportional to curvature and corners have the greatest
curvature and a plane surface has zero curvature, dots are equidistant from the straight portion and far away
from the corners.
31. Since the plate is accelerated, the electrons inside are also accelerated with the same acceleration. A force may
be applied on an electron, only when an electric field is created. So there must develop charges on the faces
that are perpendicular to the direction of motion, so that there develops an electric field inside the metal plate.
The front face will be charged positively, and the rear one negatively. The intensity of the electric field is given
by eE = ma.
Since E = /0, the surface density of charge developed is given by = 0E = 0ma/e
32. No. The concentration of lines at the botom indicates a field which is stronger at the botoom than on top.
Imagine a rectangular path with its two sides perpendicular to the electric lines. Now move a charge along this
path. Some net work will be performed. But in an electric field, work done is essentially zero as it is a conservative
field. Hence, an electric field of the type represented is physically impossible.
33. False, Work is always done by an external agent when a charged particle is transferred from one equipotential
surface to another. The work done per unit charge = potential difference.
34. There is an electric field between the filament and the ring. As the electron
is emitted from the filament, it experiences a force towards the ring and so it
is accelerated. It electrical potential energy, which it acquires on emerging
from the filament, is converted into its kinetic energy. Thus, though the
battery does not supply any energy, the electron can gain velocity at the cost of
its own potential energy.
35. True. There is an electric field inside a current-carrying conductor. This is equal
to V/l, where V is the voltage across the conductor and l is the length of the
conductor. Now, consider a closed path abcda as shown in the figure. Work
done by an external agent along abcda is zero.
Einside ab 0 Eoutside cd 0 0 E inside E outside
Hence, there is a field near the surface of the conductor.
36. To conduct away electricity produced by friction.
37. Place the smaller one inside the bigger one. As potential inside a hollow conductor is the same as that of its
surface, the potential inside the hollow conductor is 100V. The potential of the smaller conductor is now
100+50 = 150 V. Connecte the two conductors by a wire. Charge will flow from the smaller one to the bigger
one as the smaller one is at 150 V and the bigger one at 100 V.
E
38. Yes, there is a net force on the dipole given by F p
l
1
39. Total flux E E . s q , by Gausss law..
0
(a) No, the flux will not change as it depends only on the total charge inside the surface and not on the extent
and shape of the surface. (b) No, it remains the same as the total flux is determined solely by the charge inside
the surface and not the charge outside. (c) Yes, flux will change as the total charge inside the surface has
changed.
40. (a) For points inside the balloon, E = 0, (b) E decreases as the surface density of charge decreases E / 0 ,
(c) E remains constant because E q / 4 0 r 2 , where r is the distance of the point from the centre of the
balloon.
www.physicsashok.in 47
ELECTROSTATICS
Reasioning Type Questions
THE NEXT QUESTIONS REFER TO THE FOLLOWING INSTRUCTIONS
A statement of assertion (A) is given and a Corresponding statement of reason (R) is given just below it
of the statements, mark the correct answer as
(A) If both A and R are true and R is the correct explanation of A.
(B) If both A and R are true but R is not the correct explanation of A.
(C) If A is true but R is false. (D) If both A and R are false. (E) If A is false but R is true.
1. Assertion (A) : If there exists colomb attraction between two bodies, both of them may not be charged.
Reason (R) : Due to induction effects a charged body can attract a neutral body.
2. Assertion (A) : A small metal ball is suspended in a uniform electric field with an insulated thread. If high
energy X-ray beam falls on the ball, the ball will be deflected in the electric field.
Reason (R) : X-rays emit photoelectrons and metal becomes negatively charged.
3. Assertion (A) : Electric current will not flow between two charged bodies when connected if their charges
are same.
Reason (R) : Current is rate of flow of charge.
4. Assertion (A) : The surface densities of two spherical conductors of radii r1 and r2 are equal. Then the
electric field intensities near their surfaces are also equal.
Reason (R) : Surface charged density = charge/area.
5. Assertion (A) : When charges are shared between two bodies, there occurs no loss sof charge, but there
does occur a loss of energy.
Reason (R) : In case of sharing of charges, the energy of conservation fails.
6. Assertion (A) : Two adjacent conductors, carrying the same positive charge have a potential difference
between them.
Reason (R) : The potential of a conductor depends upon the charge given to it.
7. Assertion (A) : Dielectric breakdown occurs under the influence of an intense light beam.
Reason (R) : Electromagnetic radiations exert pressure.
8. Assertion (A) : The tyres of aircrafts are slightly conducting.
Reason (R) : If a conductor is connected to ground, the extra charge induced on conductor will flow to
ground.
9. Assertion (A) : Metallic shield in the form of a hollow shell, can be built to block an electric field.
Reason (R) : In a hollow spherical shell, the electric field inside it is zero at every point.
10. Assertion (A) : A bird perches on a high power line and nothing happens to the bird.
Reason (R) : The level of bird is very high from the ground.
11. A metal sphere is suspended from a nylon thread. Initially, the metal sphere is uncharged . When a positively
charged glass rod is brought close to the metal sphere, the sphere is drawn toward the rod. But if the sphere
touches the rod, it suddenly flies away from the rod. Explain why the sphere is first attracted, then repelled.

Level # 1.
Objective Type Question
1. Two copper spheres of same radii one hollow and other solid are charged to the same potential then
(A) both will hold same charge (B) solid will hold more charge
(C) hollow will hold more charge (D) hollow cannot be charged

2. Through the exact centre of a hydrogen molecule, an -particle passes rapidly, moving on a line perpendicular
to the internuclear axis. The distance between the two hydrogen nuclei is b The maximum force experienced
by the -particle is

4e 2 8e 2 8e 2 4e 2
(A) (B) (C) (D)
3 3 0 b 2 3 0b 2 3 3 0 b2 3 0b 2

3. A uniform electric field pointing in positive x-direction exists in a region. Let A be the origin, B be the point on
the x-axis at x = +1 cm and C be the point on the y-axis at y = +1 cm. Then the potential at the points A, B
and C satisfy
(A) VA > VB (B) VA < VC (C) VA < VB (D) VA > VC

www.physicsashok.in 48
ELECTROSTATICS
4. On an equilateral triangle of side 1 m there are three point charges placed at its corners of 1C, 2C and 3C. The
work required to move these charges to the corners of a smaller equilateral triangle of side 0.5 m is
(A) 19.8 x 1010 J (B) 39.6 x 1010 J (C) 9.9 x 1010 J (D) 4.45 x 1010 J

5. The potential field depends on x and y coordinates as V = (x 2 y2). Correspondence electric field lines in x.y
plane as shown in figure.

(A) (B) (C) (D)

6. Two circular rings A and B, each of radius a = 130 cm are placed coaxially with
their axes horizontal in a uniform electric field E = 105 NC1 directed vertically
upwards as shown in Figure. Distance between centers of these rings A and B
is h = 40 cm, ring A has a positive charge q1 10C while ring B has a negative

charge of magnitude q2 20C . A particle of mass m = 100 gm and carrying a positive charge q 10C
is released from rest at the centre of the ring A. Calculate its velocity when it has moved a distance of 40 cm.
(A) v 6 2ms 1 (B) v 4 2ms 1 (C) v 7 ms 1 (D) v 32 ms 1

7. Three identical spheres each having a charge q and radius R, are kept in such a way that each touches the
other two. the magnitude of the electric force on any sphere due to other two is

(A)
1 3 q 2

(B)
1 2 q 2

(C)
1 2 R 2

(D)
1 3 q 2

4 0 4 R 4 0 4 R 4 0 4 q 2 0 4 R

8. A system consists of a uniformly charged sphere of radius R and a surrounding medium filled by a charge with
the volume density r , where is a positive constant and r is the distance from the centre of the
sphere. Find the charge of the sphere for which the electric field intensity E outside is independent of r.
(A) 2 0 (B) 2 0
(C) 2 R 2 (D) None of the above

9. Three charges each of +q, are placed at the vertices of an equilateral triangle. The charge needed at the
centre of the triangle for the charges to be in equilibrium is
q
(A) (B) 3q (C) 3q (D) 3q
3

10. Hollow spherical conductor with a charge of 500C is acted upon by a force 562.5 N. What is electric
intensity at its surface?
(A) zero (B) 1.125 x 106 N/C (C) 2.25 x 106 N/C (D) 4.5 x 106 N/C

11. A hemisphere of radius R is charged uniformly with surface density of charge What will be the potential at
centre?
R 4 R
(A) (B) (C) (D)
2 0 4 0 2 0 3 0

www.physicsashok.in 49
ELECTROSTATICS
12. A circular cavity is made in a conductor. A positive charge q is placed B
at the centre A
(A) The electric field at A and B are equal
(B) The electric charge density at A = the electric charge density at B
(C) Potential at A and B are equal
(D) All the above.

13. An isolated metallic object is charged in vacuum to potential v 0, its electrostatic energy being W 0. It is then
disconnected from the source of potential, its charge being left unchanged and is immersed in a large volume
of dielectric, with dielectric constant k. The electrostatic energy will be.
W0 W0
(A) kW 0 (B) (C) (D) W 0.
k 2k

14. A charge +q is fixed at each of the points x = x 0, x = 3x 0, x = 5x 0 .... ad inf. on the x-axis, and a charge
q is fixed at each of the points x = 2x 0, x = 4x 0, x = 6x 0 .... ad inf. Here x 0 is a positive constant. Take the
electric potential at a point due to a charge Q at a distance r from it to be Q/( 4 r ). Then, the potential at
the origin due to the above system of charges is.
q q loge 2
(A) Zero (B) (C) (D)
8 0 x 0 loge 2 4 0 x 0

15. A positively charged thin metal ring of radius R is fixed in the xy-plane with its centre at the origin O. A negatively
charged particle P is released from rest at the point (0, 0, z0) where z0 > 0. Then the motion of P is.
(A) Periodic, for all values of z0 satisfying 0 < z0 <
(B) Simple harmonic, for all values of z0 satisfying 0 < z0 R
(C) Such that P crosses O and continues to move along the negative z-axis towards z =
(D) None of these

16. For an infinite line of charge having charge density lying along
x-axis, the work done in moving charge from C to A arc CA is.
q q
(A) 2 loge 2 (B) 4 loge 2
0 0

q q 1
(C) 4 loge 2 (D) 2 loge 2
0 0

17. A particle A has charge +q and particle B has charge + 4q with each of them having the same mass m.
When allowed to fall from rest through same electrical potential difference, the ratio of their speeds v A : v B will
be
(A) 2 : 1 (B) 1 : 2 (C) 4 : 1 (D) 1 : 4

18. Three point charges q1, q2 and q3 are taken such that when q1 and q2 are placed close together to form a
single point charge, the force on q3 at distance L from this combination is a repulsion of 2 units in magnitude.
when q2 and q3 are so combined the force on q1 at distance L is an attractive force of magnitude 4 units. Also
q3 and q1 when combined exert an attractive force on q2 of magnitude 18 units at same distance L. The
algebraic ratio of charges q1, q2 and q3 is.
(A) 1 : 2 : 3 (B) 2 : 3 : 4 (C) 4 : 3 : 1 (D) 4 : 3 : 2

19. An electric potential is given by V = k(xy), where k is a constant. A particle of charge q0 is first taken from
(0, 0) to (0, a) to (a, a), then directly from (0, 0) to (a, a) and lastly from (0, 0) to (a, 0) to (a, a). If W 1, W 2 and
W 3 be the work done for the individual paths respectively then
(A) W 1 = W 2 = W 3 = q0ka2 (B) W 1 = W 3 = 2 q0ka2
(C) W 1 = W 3 > W 2 (D) W 1 > W 2 > W 3

www.physicsashok.in 50
ELECTROSTATICS
20. Four charges 2C, 3C, 4C and 5C respectively are placed at all the corners of a square. W hich of the
following statements is true for the point of intersection of the diagonals ?
(A) Electric field is zero but electric potential is non-zero
(B) Electric field is non-zero but electric potential is zero
(C) Both electric field and electric potential are zero (D) Neither electric field nor electric potential is zero

21. Two metallic identical spheres A and B carrying equal positive charge + q are a certain distance apart. The
force of repulsion between them is F. A third uncharged sphere of the same size is brought in contact with
sphere. A and removed. It is then brought in contact with sphere B and removed. What is the new force of
repulsion between A and B ?
3F F F
(A) F (B) (C) (D)
8 2 4

22. An electron of mass m e, initially at rest, moves through a certain distance in a uniform electric field in time
t 1. A proton of mass m p , also initially at rest, takes time t 2 to move through an equal distance in this uniform
electric field. Neglect the effect of gravity, the ratio t 2 /t 1 is nearly equal to
(A) 1 (B) (mp /me)1/2 (C) (me /mp )1/2 (D) 1836

23. Eight dipoles of charges of magnitude are placed inside a cube. The total electric flux coming out of the
cube will be
8e 16 e e
(A) (B) (C) (D) Zero
0 0 0

24. A particle of mass m and charge q is released from rest in a uniform electric field E. The kinetic energy
attained by the particle after moving a distance x is.
(A) q Ex 2 (B) q Ex 2 (C) q Ex (D) q2 Ex

25. Two point charges +q and q are held fixed at (d, 0) and (d, 0) respectively of a x-y coordinate system. Then
(A) The electric field E at all points on the x-axis has the same direction.
(B) Work has to be done in bringing a test charge from to the origin [1995]
(C) Electric field at all points on y-axis is along x-axis. (D) The dipole moment is 2qd along the x-axis.
Q
26. Three charges Q, +q and +q are placed at the vertices of a
ring-angled isosceles triangle as shown. The net electrostatic
energy of the configuration is zero if Q is equal to [2000]
q 2q
(A) (B) (C) 2q (D) +q +q +q
1 2 2 2 a
27. Three positive charges of equal value q are placed at the vertices of an equilateral triangle. The resulting lines
of force should be sketches as in [2001]

(A) (B) (C) (D)

28. Two equal point charges are fixed at x = -a and x = +a on the x-axis. Another point charge Q is placed at the
origin. The change in the electrical potential energy of Q, when it is displaced by a small distance x along the
x-axis, is approximately proportional to [2002]
(A) x (B) x 2 (D) x 3 (D) 1 x

www.physicsashok.in 51
ELECTROSTATICS
29. A metallic shell has a point charge q kept inside its cavity. Which one of the following diagrams correctly
represents the electric lines of forces? [2003]

(A) (B) (C) (D)

P Q

30. Six charges of equal magnitude, 3 positive and 3 negative are to


be placed on PQRSTU corners of a regular hexagon, such that U R
O
field at the centre is double that of what is would have been if only
one +ve charge is placed at R. [2004] T S
(A) +, +, +, , , (B) , +, +, +, ,
(C) , +, +, , +, (D) +, , +, , +,

31. A Gaussian surface in the figure is shown by dotted lime. q1 q2


The electric field on the surface will be [2004]
(A) due to q1 and q2 only (B) due to q2 only q1
(C) Zero (D) due to all

32. Three infinitely long charge sheets are placed as shown in Z


figure. The electric field at point P is [2005]
2 4
Z=a
(A) k (B) k P
0 0 2
Z=a
x
2 4
(C) k (D) k Z=2a
0 0

33. Two equal negative charges q are fixed at points (0, a) on y-axis. A positive charge Q is released from rest
at the point (2a, 0) on the x-axis. The charge Q will [1984]
(A) execute simple harmonic motion about the origin. (B) move to the origin remain at rest
(C) move to infinity (D) execute oscillatory but not simple harmonic motion

34. A charge q is placed at the centre of the line joining two equal charges Q. The system of the three charges will
be in equilibrium if q is equal to: [1987]
Q Q Q Q
(A) (B) (C) (D)
2 4 4 2

35. A solid conducting sphere having a charge Q is surrounded by an uncharged concentric conducting hollow
spherical shell. Let the potential difference between the surface of the solid sphere and that of the outer
surface of the hollow shell be V. If the shell is now given a charge of 3Q, the new potential difference between
the same two surfaces is: [1989]
(A) V (B) 2V (C) 4V (D) 2V

36. A metallic solid sphere is placed in a uniform


electric field. The lines of force follow the path(s)
shown in Figure as [1996]
(A) a (B) 2
(C) 3 (D) 4

www.physicsashok.in 52
ELECTROSTATICS
37. A non-conducting solid sphere of radius R is uniformly charged. The magnitude of the electric field due to the
sphere at a distance r from its center [1998]
(A) increases as r increases, for r < R (B) decreases as r increases, for 0 < r <
(C) is discontinuous at r = R. (D) None of these

38. An ellipsoidal cavity is carved within a perfect conductor Figure. a


positive charge q is placed at the centre of the cavity. The points A A
and B are on the cavity surface as shown in the figure. Then [1999] B
(A) electric field near A in the cavity = electric field near B in the cavity q
(B) charge density at A = charge density at B
(C) potential at A = potential at B
(D) total electric field flux through the surface of the cavity is q 0 .

More Than One Choice Questions:


39. Three concentric spherical metallic shells A, B and C of radii a, b and c (a < b < c) have charge densities of
, , and respectively. The potentials of A, B and C are:
1 1 a2
(a) VA a b c (b) VB b c
0 0 c
1 a 2 b2 1
(c) VC c (d) VA VB VC a b c
0 c c 0

40. When positively charged spheer is brought near a metallic sphere, it is observed that a force of attraction
exists between two. It means:
(a) metallic sphere is necessarily negatively charged. (b) metallic sphere may be electrically neutral.
(c) metallic sphere may be negatively charged. (d) mothing can be said about charge of metallic sphere.

41. A conducting sphere of radius R has a charge. Then:


(a) the charge is uniformly distributed over its surface, if there is no external electric field.
(b) distribution of charge over its surface will be non-uniform, if an external electric field exists in the space.
(c) electric field strength inside the sphere will be equal to zero only when no exxternal electric field exists.
(d) potential at every point of the sphere must be same.

42. A small sphere of amss m and having charge q is suspended by a light thread, then:
(a) tension in the thread may reduce to zero if anohter charged sphere is placed vertically below it.
(b) tension in the thread may increase to twice of its original value if another charged sphere is placed
vertically below it.
(c) tension in the thread is greater than mg if another charged sphere is held in the same horizontal line in
which first sphere stays in equilibrium.

43. Two point charge: Q and Q/4 are separated by a distance x. Then:
(a) potential is zero at a point on the axis which is x/3 on the right side of the charge Q/4.
(b) potential is zero at a point on the axis which is x/5 on the left side of the charge Q/4.
(c) electric field is zero at a point on the axis which is at a distance x on the right side of the charge Q/4.
(d) there exist two points on the axis, where electric field is zero.

44. Two equal and oppositely charged particles are kept some distance apart from each other. A spherical surface
having radius equal to separation between the particles and concentric with their midpoint is considered.
Then:
(a) electric field is normal to the surface at two points only.
(b) electric field is zero at no point.
(c) electric potential is zero at every point of one circle only.
(d) net electric flux through the surafce is zero.

www.physicsashok.in 53
ELECTROSTATICS
45. Two identical charges +Q are kept fixed some distane apart. A small particle P with charge q is placed mid-
way between them. If P is given a small displacement , it will undergo simple harmonic motion if:
(a) q is positive and is along the line joining the charges.
(b) q is positive and is perpendicular to the line joining the charges.
(c) q is negative and is perpendicular to the line joining the charges.
(d) q is positive and is along the line joining the charges.

46. Select the correct statement(s):


(a) Charge cannot exist without mass but mass can exist without charge.
(b) Charge is conserved but mass is not.
(c) Charge is independent of state of rest or motion. (d) None of these

47. Which of the following quantities do not depend on the choice of zero potential or zero potential energy?
(a) Potential at a point. (b) Potential difference between two points.
(c) Potential energy of a system of two charges. (d) Change in potential energy of system of two charges.

Fill in the blanks:


1. Figure shows line of constant potential in a region in which an A
electric field is present. The value of the potential are written in
B
brackets. Of the points A, B and C, the magnitude of the electric
field is greatest at the point ............. [1984] (50 V)
(40 V) C
(30 V)
(20 V)
(10 V)
2. Two small balls having equal positive charges Q (coulomb0 on each are suspended by two insulating strings
of equal length L (metre) from a hook fixed to a stand. The whole set up is taken in a satellite into space
where there is no gravity (state of weightlessness). The angle between the two string is ............ and the
tension in each string is ......... newtons. [1988]
Y
3. A point charge q moves from point P to point S along the path
PQRS (figure) in a uniform electric field E pointing parallel to
the positive direction of the X-axis. The coordinates of the points
P, Q, R and S are (a, b, O), (2a, O, O) (a, b, O) and (O, O, O) P
respectively. The work done by the field in the above process is S Q X
given by the expression ..................... [1989] R E

4. The electric potential V at any point x, y, z (all in metres) in space is given by V = 4x 2 volts. The electric field
at the point (1m, 0.2 m) is ............... V/m. [1992]

5. Five point charges, each of value +q coul, are placed on five vertices q q
of a regular hexagon of side L metres. the magnitude of the force on 5 4
the point charge of q coul. Placed at he centre of the hexagon q
3
is .......... newton. [1992] 6 q
2
1
q L q
True / False :
6. The work done in carrying a point charge from one point to another in an electrostatic field depends on the
path along which the point charge is carried. [1981]
7. Two identical metallic spheres of exactly equal masses are taken. One is given a positive charge Q coulombs
and the other an equal negative charge. Their masses after charging are different. [1983]
8. A small metal ball is suspended in a uniform electric field with the help of an insulated thread. If high energy
X-ray beam falls on the ball, the ball will be deflected in the direction of the field. [1983]

www.physicsashok.in 54
ELECTROSTATICS
9. A ring of radius R carries a uniformly distributed charge +Q. A point charge q is placed in the axis of the ring
at a distance 2R from the center of the ring and released from rest. The particle executes a simple harmonic
motion along the axis of the ring. [1988]

Table Match
10. Match List I and List II and select the correct answer using the codes given below the lists:
The electric fields due to various charge distribution are (q is the total charge on the body, is the surface
charge density, is the linear charge density)
List I List II

x
I. At a distance x from the centre of a A. E 1 2
20 x R2
uniformly charged ring of radius R. The
point is on the line passing through the centre
of the ring and perpendicular to the plane
of the ring

II. At a distance x from the centre of a uniformly B. E
2 0 x
charged ring of radius R. The point is on the
line passing through the centre of the ring and 2r
perpendicular to the plane of the disc.
r
d
E q1
III. At a distance x from an infinite sheet of C.
2 0
q2
uniform distribution of charge
1 qx
E
IV. At a distance x from an infinite line of charge D. 4 0 R 2 x 2 3 2 .

(A) I-A, II-C, III-B, IV-D (B) I-D, II-A, III-C, IV-B
(C) I-C, II-B, III-D, IV-A (D) I-B, II-D, III-A, IV-C

Passage Type Questions


THE NEXT QUESTIONS REFER TO THE FOLLOWING PASSAGE
E is the electric field created by q1. V is the voltage at a given point in the field E. Assume that the
electric field created by q2 is negligible compared to E. k is Coulombs Law constant. m 1 si the mass of q1,
and m 2 is the mass of q2.

1. E can best be described as


(A) constant (B) decreasing as r increases
(C) increasing as r increases (D) increasing as q2 increases
2. What is the work done on q2 when it is moved at constant velocity along the distance d?
(A) Zero (B) Vq1 (C) Vq2 (D) Edq2
3. Which of the following represents the work done on q2 when moved from its present position to a distance r
from q1?
1
(A) kq1q2 r (B) kq1q2 r (C) 2 kq1q2 r (D) kq1q2 r 2
2
4. If q1 and q2 have opposite charges, then when q2 is moved from its present location to a distance r from q1, the
force on q2 due to q1
(A) decreases by a factor of 4 (B) remains the same
(C) increases by a factor of 2 (D) increases by a factor of 4

www.physicsashok.in 55
ELECTROSTATICS
5. If q1 is positive then when q2 is moved from its present location to a distance r from q1, the magnitude of the
voltage experienced by q2 due to E
(A) decreases by a factor of 4
(B) remains the same
(C) increases by a factor of 2 (D) increases by a factor of 4
6. The strength of the electric field E at r is
(A) half the field strength at 2r (B) the same as the field strength at 2r
(C) twice the field strength at 2r (D) four times the field strength at 2r
7. If q1 and q2 are both positively charged and q2 is released, what is the maximum velocity that can be achieved
by q2?

kq1q2 kq1q2 2kq1q2 2kq1q2


(A) (B) (C) (D)
m2 r m2 r 2 m2r 2 m2 r
8. Electrostatic precipitators use electric forces to remove pollutant particles from smoke, in particular in the
smokestacks of coal-burning power plants. One form of precipitator consists of a vertical hollow metal cylinder
with a thin wire, insulated from the cylinder, running along its axis. A large potential difference is established
between the wire and the outer cylinder, with the wire at lower potential. This sets up a strong radial electric
field directed inward, producing a region of ionized air near the wire. Smoke enters the precipitator at the
bottom, ash and dust in the smoke pick up electrons, and the charged pollutants are accelerated toward the
outer cylinder wall by the electric field. Suppose the radius of the central wire is 80.0 m, the radius of the
cylinder is 12.0 cm, and the potential difference between the wire and the cylinder is 60.0 kV. Assume that
the wire and cylinder are both very long in comparison to the cylinder radius.

(a) What is the electric-field magnitude midway between the wire and the cylinder wall?
(b) What magnitude of charge must a 30.0- g ash particle have if the electric field computed in part (a) is
to exert a force ten times the particles weight?

9. A Geiger counter detects radiation such as alpha particles by using the fact that the radiation ionizes the air
along its path. The device consists of a thin wire on the axis of a hollow metal cylinder and insulated from it.
A large potential difference is established between the wire and the outer cylinder, with the wire at higher
potential; this sets up a strong radial electric field directed outward. When ionizing radiation enters the
device, it ionizes a few air molecules. The free electrons produced ar accelerated by the electric field toward
the wire and, on the way there, ionize many more air molecules. Thus a current pulse is produced that can be
detected by appropriate electronic circuitry and converted to an audiable click. Suppose the radius of the
central wire is 50.0 m and the radius of the hollow cylinder is 2.00 cm.
What potential difference between the wire and the cylinder is required to produce an electric field of 6.00 x
104 N/C at a distance of 1.50 cm from the wire?

(Assume that the wire and cylinder are both very long in comparison to the cylinder radius.

www.physicsashok.in 56
ELECTROSTATICS
Level # 2
1. As shown a solid spherical region having a spherical cavity R
whose diameter R is equal to the radius of spherical region
P C
that has a total charge Q. Find the potential at a point P, which
is at a distance x from C. Q
2. A spherical water drop of radius a has a charge Q spread uniformly over its face. The drop is split into two
identical spherical droplets (each of which has charge Q/2 spread uniformly over its face), which are kept
very far from one another.
(a) Compute the change in the electrostatic potential energy caused by the splitting.
(b) Repeat the above calculation for the case in which the charge is uniformly distributed in the drop
volume, before and after the splitting.
3. An electric dipole is placed at distance x from centre
O on the axis of a charged ring of radius R and charge
Q uniformly distributed over it.
(a) Find the net force acting on the dipole.
(b) What is the work done in rotating the dipole through 180.
(c) If the dipole is slightly rotated about its equilibrium position, find the time period of oscillation. Assume
that the dipole is linearly restrained. P
4. Particle 1 located far from particle 2 and possessing the kinetic
energy T 0 and mass m 1 strikes particle 2 of mass m 2 through 1
the aiming parameter , the arm of the momentum vector relative P0 r
to particle 2 as; in the figure. Each particle carries a charge +q.

Find the smallest distance between the particles when m 1 < < m 2.
2
5. An electric field line emerges from a positive point charge
+q1 at an angle to the straight line connecting it to a

negative point charge q2. At what angle will the field +
enter the charge q2 ? q1 q2

6. Two point charges each carrying a positive charge of 5e P


(e being the magnitude of the electronic charge) are sepa-
A R B
rated by a distance 2d. An electron describes a circular
path due to the attraction of the charges in a plane bise- d O d
cting perpendicularly the line joining the two point charges. 5e 5e
If the radius of the circular path described by the electron
is R, determine the orbital speed of the electron.
7. Five thousand lines of forces enter a certain volume of space, and three thousand lines emerge from it.
What is the total charge in coulombs with in the volume ?
8. Determine the strength E of the electric field at the centre of a hemisphere produced by charges uniformly
distributed with a density over the surface of this hemisphere.
9. Two small identical balls lying on a horizontal plane are connected by a
2 1
weight-less-spring. One ball (ball 2) is fixed and the other (ball 1 is free.
+
The balls are charged identically as a result of which the spring length
increases = 2 times. Determine the change in frequency.. O
10. A semi-circular ring of mass M and radius R with linear charge
density hinged at its centre is placed in a uniform electric field
as shown in the figure.
(i) Find the net force acting on the ring.
(ii) It the ring is slightly rotated about O and released, find its time
period of oscillation
(iii) What is the work done by an external agency to rotate it through an angle .

www.physicsashok.in 57
ELECTROSTATICS
11. Determine the force F of interaction between two hemispheres of radius R touching each other along the
equator if one hemisphere is uniformly charged with a surface density, 1 and the other with a surface
density 2. y
12. Suppose in an insulating medium, having di-electric constant k = 1,
volume density of positive charge varies with y-coordinate to law = ay..
A particle of mass m having positive charge q is placed in the medium A
v0
at point A(0, y0) and projected with velocity v = v 0 . i and assuming
electric field strength to be zero at y = 0, calculate slope of trajectory of
the particle as a function of y. x
o
13. A metal ball of radius r and density is charged by direct contact from the Earths surface till it acquires its
maximum value. What should be this charge Q max on the ball and the charge Q on the Earth (assuming it to
be uniform sphere of mass m and radius R) such that it may be launched from the Earths surface with zero
launch velocity.
14. A particle of mass m having negative charge q moves along an ellipse around a fixed positive charge Q so
that its maximum and minimum distances from fixed charge are equal to r1 and r2 respectively. Calculate
angular momentum L of this particle.
15. Three concentric, conducting spherical shells A, B and C have radii a = 10 cm, b = 20 cm and c = 30 cm
respectively. The innermost shell A is earthed and charge q2 = 4 C and q3 = 3 C are given to shells B and
C respectively. Calculate charge q1 induced on shell A and energy stored in the system. B


16. The system consists of a hemispherical dielectric with volume charge
density . Find the potential difference between points A and B.

3
A
r
17. A space is filled up with volume density of charge 0 e where 0 and are positive constants, r is
the distance from centre of the system. Find the magnitude of the electric field strength vector as a function
of r.
18. Two identical balls are suspended from the same point by two threads. The balls are given equal charges
and immersed in kerosene. Determine the density of the material of the balls if the threads deflect equally
in vacuum and kerosene. The density of kerosene 0 = 0.8 g/cm 3 and its relative permittivity 0 = 2.
19. A charge q is placed on the surface of an originally uncharged soap bubble of radius R0. Due to the mutual
repulsion of the charged surface, the radius is increased to a somewhat large value R. Show that q =
1/ 2
32 2
.0 PR 0R (R 2 R R 0 R 02 ) in which P is the atmospheric pressure.
3

Level # 3
1. Two fixed charges 2Q and Q are located at the points with coordinates (3a, 0) and (+3a, 0) respectively
in the x-y plane.
(a) Show that all points in the x-y plane where the electric potential due to the two charges is zero, lie on
a circle. Find its radius and the location of its centre.
(b) Given the expression V(x) at a general point on the x-axis and sketch the function V(x) on the whole x-
axis.
(c) If a particle of charge +q starts form rest at the centre of the circle, show by a short quantitative
argument that the particle eventually crosses the circle. Find its speed when it does so. [IIT 91]
2. A blank to be filled appears in each of the following statements. Write in your answer book the subquestion
number and write down against it your answer corresponding to each blank. In your answer, the sequence
of the sub-questions should be the same as given in the question paper.
(i) If 0 and 0 are respectively, the electric permittivity and magnetic permeability of free space, and
the corresponding quantities in a medium, the index of refraction of the medium in terms of the above
parameters is .....

www.physicsashok.in 58
ELECTROSTATICS
(ii) The electric potential V at any point x, y, z (all in meters) in space is given by V = 4x 2 volts. The electric
field at the point (1m, 0, 2m) is ...... V/m. q q
(iii) Five point charges, each of value +q C, are placed on five
vertices of a regular hexagon of side L metre. The magnitude q
of the force on the point charge of value q C placed at the
centre of the hexagon is ..... newton. [IIT 92] q
q

3. A circular ring of radius R with uniform positive charge density per unit length is located in the y-z plane
with its centre at the origin O. A particle of mass m and positive charge q is projected from the point P(R 3 ,
0, 0) on the positive x-axis directly towards O, with an initial speed v. Find the smallest (non-zero) value of
the speed v such that the particle does not return to P.
4. Consider the classical-model of an atom such that a nucleus of charge +e is uniformly distributed within a
sphere of radius 2 . An electron of charge ( e) at a radial distance 1 moves inside the sphere. Find the
force attracting the electron to the centre of the sphere. Calculate the frequency with which the electron
would oscillate about the centre of the sphere. [REE 95
5. A charge 109 coulomb is located at origin in free space and another charge Q at (2, 0, 0). If the
x-component of the electric field at (3, 1, 1) is zero calculate the value of Q. Is the y-component zero at (3,
1, 1) ? [REE 95]
6. A radioactive source in the form of a metal sphere of radius 10 m, emits beta particles at the rate of 5 1010
2

particles per s. The source is electrically insulated. How long will it take for its potential to be raised by 2
volt assuming that 40% of the emitted beta particles escape the source ? [REE 97]
7. A non-conducting disc of radius a and uniform positive surface charge density is placed on the ground,
with its axis vertical. A particle of mass m and positive charge q is dropped, along the axis of the disc, from
a height H with zero initial velocity. The particle has q/m = 4 0g / . [IIT 99]
(a) Find the value of H if the particle just reaches the disc.
(b) Sketch the potential energy of the particle as a function of its height and find its equilibrium position.
8. A particle of charge q and mass m moves rectilinearly under the action of an electric field E = A Bx where
B is a + ve constant and x is a distance from the point where the particle was initially at rest. Calculate
(a) Distance travelled by the particle till it comes to rest and (b) Acceleration at that moment.
9. Four point charges + 8 C, 1 C, 1 C and + 8 C are fixed at the points 27 / 2 m , 3 / 2 m , 3 / 2 m
and 27 / 2 m respectively on the y-axis. A particle of mass 6 104 kg and of charge + 0.1 C moves
along the x direction. Its speed at x = + is v 0. Find the least value of v 0 for which the particle will cross the
origin. Find also the kinetic energy of the particle at the origin. Assume that space is gravity free. Given
1/( 4 0 ) = 9 109 Nm 2/C2. [IIT 2000]
10. Eight point charges are placed at the corners of a cube of edge a as
shown in figure. Find the work done in disassembling this system of
charges. [JEE 2003]

www.physicsashok.in 59
ELECTROSTATICS
Answer Key
Assertion-Reason
Que. 1 2 3 4 5 6 7 8 9 10

Ans. A C D B C B B A A C

Level # 1

Q. 1 2 3 4 5 6 7 8 9 10
Ans. A C A C A A A C A B
Q. 11 12 13 14 15 16 17 18 19 20
Ans. A A B D A A B D A B
Q. 21 22 23 24 25 26 27 28 29 30
Ans. B B D C C B C B C C
Q. 31 32 33 34 35 36 37 38 39 40
Ans. D C D B A D A C BC BC
Q. 41 42 43 44 45 46 47
Ans. ABD ABC ABC ABCD AC ABC BD

Fill in the blanks, True-False & Match the column

1 Q2 1 qq
1. B 2. 180, 3. qEa 4. 8 5. 6. F
4 0 4 L2 4 0 L2
7. T 8. T 9. F 10. B

Passage Type
Que. 1 2 3 4 5 6 7
Ans. B A A D C D D
8. (a) 1.37 x 105 volt/m (b) 2.15 x 1011 C

Level # 2
Q 2 1
1. V
7 0 x 2 R 2 4 x 2

1
2. (a) The fractional change = 1 2 / 3 (b) The fractional change in this case is also the same,.
2

aqQ R 2 2x 2 aqQx q2 1
3. (a) 2 5/ 2 (b) 4. rmin (1 1 (2 T0 / q2 )2 ,
2
2 0 (R x ) 0 (R 2 x 2 )3/ 2 2 T0 4 0

www.physicsashok.in 60
ELECTROSTATICS
q1 5
5. sin 2 sin / 2 6. V Re 7. 17.7 nc
q2 2 0 (R d2 )3 / 2
2

3n 2 N
8. 4 9. times 10. (i) RE (ii) 2 (iii) 4ER2
0 n 2E

R2 qa
( y 3 y 30 ) 2 0 Rg 0 gR 3
11. F 1 2 12. 13. Q max 4r , Q 4 r
2 0 3 m 0 v 20 3 3
mr1 r2 Q q R 2 2 2 R 3
14. 2 0 (r1 r2 ) 15. q1 = 3 C, Energy = 9.45 J 16. V
0 3 0

17.

0 1 er
3
18. 1.6 gm/cm 3
2
3 0 r

Level # 3
Q 1 2
1. (a) radius = 4a, centre dt (5a, 0) (b) V 4 (c) At x = 9a where V = 0, the charged
0 | x 3a | | x a |

9q
particle eventually crosses the circle, V 8 0 ma

9 10 9 q2 (N) q
2. (i) 00 (ii) 8 V/m (iii) 3. V 2 0m 4. 9 1014 Hz
L2

2 10 9
5. Q = 4.27 10 10
C, E y 0
4 0 11 11

4a a 2A qA
6. 700 F 7. (a) H = and H = 0 (b) 0 8. x = 0, x = ,a=
3 3 B n

1 q2
9. Vmin = 3 m/s, K.E at origin = 2.5 10 J 4
10. W = 5.824 4 . a
0

XXXX

www.physicsashok.in 61
CURRENT ELECTRICITY

DHANALAKSHMI NAGAR

NEAR ANNAMAIAH CIRCLE,

TIRUPATI.

PH NO. 9440025125
ELECTRIC CURRENT
THEORY OF ELECTRIC CURRENT
CURRENT ELECTRICITY
The time rate of flow of charge through any cross-section is called current. Therefore, if through a cross-
section, q charge passes in time t, the average electric current through that area
q
i av
t
and instantaneous current
q dq
i lim
t 0 t dt
Regarding electric current following points are worth noting :
(a) Current is assumed to be a fundamental quantity in physics with unit ampere and dimension [A]. The CGS unit
of current is emu of current and is called biot (Bi), i.e.,
1C (1/10)emu of charge
1A = =
s s
1
1A = Bi
10
(b) The current is same for all cross-sections of a conductor of non-uniform cross-section. Similar to water flow,
charge flows faster where the conductor is smaller in cross-section and slower where the conductor is larger in
cross-section, so that charge rate remains unchanged.
(c) Though conventionally a direction is associated with current (opposite to the i1
motion of electrons) it is not a vector as the direction merely represents the
sense of charge flow and not a true direction. Further current does not obey i = i1 + i2
i2
the law of parallelogram of vectors, i.e., if two currents i1 and i2 reach a point
we always have i = i1 + i2 whatever be the angle between i1 and i2.
(d) By convention, the direction of current is taken to be that in which positive charge moves and opposite to the
direction of flow of negative charge.
(e) As charge is conserved and current is the rate of flow of charge, the charge entering at one end per second of
a conductor is equal to the charge leaving the other end per second.
(f) Current in different situations is due to motion of different charge carries. Current in conductors and vacuum
tubes is due to motion of electrons, in electrolytes due to motion of both positive and negative ions, in discharge
tube due to motion of positive ions and negative electrons and in semiconductors due to motion of electrons
and holes.

Example 1. The current in a wire varies with time according to the relation
A
i (3.0 A) 2.0 t
s
(a) How many coulombs of charge pass a cross-section of the wire in the time interval between t = 0 and
t = 4.0 s?
(b) What constant current would transport the same charge in the same time interval ?
dq
Sol. (a) i
dt
q 4
0
dq idt
0
4
q (3 2t)dt
0
4
q 3t t 2 0 12 16 28 C

www.physicsashok.in 1
ELECTRIC CURRENT
q 28
(b) i 7A
t 4

Types of Electric Current


According to its magnitude and direction current is usually divided into two types :
(i) Direct current : If the magnitude and direction of current does not vary with time, it is said to be direct current
(DC). Cell, battery or DC dynamo are its sources.
(ii) Alternating current : If a current is periodic (with constant amplitude) and has half cycle positive and half
negative, it is said to be alternating current (AC). AC dynamo is the source of it.

NOTE : It is worthy to note here that rectifier converts


AC into DC while inverter DC into AC.
Rectifier
DC AC
Inverter

Current Density

Current density J at at a point is defined as a vector having magnitude equal to current per unit area surrounding
that point and normal to the direction of charge flow and direction in which current passes through that point.

So, if at point P current I passes normally through area S as shown in Fig., current density J at P will be
given by
i + dS
J lim n
S 0 S i P n J
di
i.e., J n
dS
Regarding current density following points are worth noting :
(a) If the cross-sectional area is not normal to the current, the cross-sectional area normal to the current in
accordance with Fig., will be
dS dS cos dS

di J
J i
dScos
dS cos
i.e., di = JdS cos

or di J . d S

i.e., i J . dS
(b) In case of conductors as V = iR and by definitions,
V V
E and R
L S
L
So, EL i
S
i 1
i.e., J E
S
1
or JE

www.physicsashok.in 2
ELECTRIC CURRENT
(c) In case of uniform charge flow through a cross-section normal to it as
i = nqvS
i
So, J n (nqv) n
S

or J nq v v ()charge [()charge = nq]

Drift Velocity
The average uniform velocity acquired by free electrons inside a metal by the application of an electric field
which is responsible for current through it is called drift velocity. It is represented by vd.
The current flowing through conductor
i
i neAv d or vd
neA
where n = number of moving electrons per unit volume
A = area of cross-section

NOTE : Some books have taken average drift velocity as half of initial and final drift
velocities of an electron giving vd = (eE/2m). This is wrong as vd is the average of drift
velocities of large number of electrons at same instant and as for each electron


v d = a f with a = e E / m constant , v d av = e E / m .
Example 2. An n-type silicon sample of width 4 103m, thickness 25 105 m and length 6 102 m carries a
current of 4.8 mA when the voltage is applied across the length of the sample. What is the current density ? If
the free electron density is 1022 m3, then find how much time does it take for the electrons to travel the full
length of the sample ? Given that charge on an electron e = 1.6 1019 C.
Sol. By definition,
i i
J [as s = (b d)]
S b d
4.8 103
So, J 4.8 103 A / m 2

25 10 5 4 103
and as in case of electric conduction in metals
J = nevd
J
or vd
ne
4.8 103
So, v d 22 3 m/s
10 1.6 1019
Hence, time taken by electron to travel the length L (= 6 102 m) of the conductor
L 6 102
t 0.02 s.
vd 3

Example 3. The area of cross-section, length and density of a piece of a metal of atomic weight 60 are 106 m2,
1 m and 5 103 kg/m3 respectively. Find the number of free electrons per unit volume if every atom contributes
one free electron. Also find the drift velocity of electrons in the metal when a current of 16A passes through it.
Given that Avogadros number NA = 6 1023/mol and charge on an electron e = 1.6 1019 C.

www.physicsashok.in 3
ELECTRIC CURRENT
Sol. As according to Avogadros hypothesis,
N m N m N d m
so n N A A as d V
NA M V VM M

n

6 1023 5 103 5 10 28
/ m3

60 10 3

Now as each atom contributes one electron, the number of electrons per unit volume
ne = 1 n = 1 5 1028 = 5 1028/m3
Further as here,
i 16
J 6 16 106 A / m 2
A 10
and as J = nevd
J 16 106
vd 2 103 m / s
28

ne 5 10 1.6 10 19

NOTE : From this example it is clear that : An electron will take 1/(2 103), i.e.,
500 s = (8.3 min.) to travel 1 m length of wire if it can.
If resistivity of metal is taken to 2 108 ohmm, the electric field inside the metal
E = J = (2 108) (16 106) = 0.32 V/m and not zero as in electrostatics.

RESISTANCE AND OHMS LAW



For some materials, especially metals, at a given temperature, J is nearly directly proportional to E and the
ratio of the magnitudes E and J is constant. This ratio is called the resistivity () and this relationship is called the
Ohms law.
E
Thus, resistivity
J
SI units of resistivity are m (ohmmetre). The reciprocal of resistivity is conductivity ().
1
Thus,

A material that obeys Ohms law reasonably well is called an ohmic conductor or a linear conductor. Materials
which show substantial departures from Ohms law are called non-ohmic or non-linear.

Resistance
Suppose a conducting wire has a uniform cross-sectional area A and length l
l as shown in Fig. Let V be the potential difference between the ends of the
E
wire. If the magnitudes of the current density J and the electric field E are i A
uniform throughout the conductor, the total current i is given by i = JA and J
the potential difference V between the ends is V = El. +
V
V El l

i JA A
l
Here, is constant for ohmic materials. This is called the resistance R.
A
www.physicsashok.in 4
ELECTRIC CURRENT
V
Thus, R
i
The resistance R of a particular conductor is related to the resistivity of its material by
l
R
A
The equation, V = iR
is often called Ohms law.

V
NOTE : The equation R = defines resistance R for any conductor, whether or not it obeys ohms law, but
i
only when R is some constant we can correctly call this relationship Ohms law. V
Thus, for ohmic conductors Vi group is a straight line possing through origin.
The slope of this line is equal to the resistance of the conductor.
i
V
R = = tan
i
Reciprocal of resistance is called conductance (G),
1 i
i.e. G= =
r V
SI unit of G is ohm1 which is called mho.

Example 4. Two copper wires of the same length have got different diameters, which wire has :
(a) greater resistance and (b) greater specific resistance ?
Sol. (a) For a given wire,
l
R
A
1
i.e. R
A
So, the thinner wire will have greater resistance.
(b) Specific resistance () is a material property. It does not depend on l or A.
So, both the wires will have same specific resistance.

Example 5. A wire has a resistance R. What will be its resistance if it is stretched to double its length ?
Sol. Let V be the volume of wire, then
V = Al
V
A
l
l
Substituting this in R , we have
A
l2
R
V
So, for given volume and material, (i.e., V and are constant)
R l2
When l is doubled, resistance will become four times, or the new resistance will be 4R.

www.physicsashok.in 5
ELECTRIC CURRENT
VARIATION OF RESISTANCE WITH TEMPERATURE R
The resistance of a conductor varies with temperature. The graph of variation of R Slope = R 0
0
resistance of pure metal with temperature is shown.
Mathematically the dependence of resistance R on temperature is expressed as
R (T) = R0[1 + (T T0)] O
T
T0
In this equation R(T) is the resistance at temperature T and R0 is the resistance
at temperature T0, often taken to be 0C or 20C. The factor is temperature coefficient of resistivity.

Example 6. The resistance of thin silver wire is 1.0 at 20C. The wire is placed in a liquid bath and its resistance
rises to 1.2 . What is the temperature of the bath ?
for silver is 3.8 103 per C.
Sol. R (T) = R0[1 + (T T0)]
Here, R (T) = 1.2 , R0 = 1.0
= 3.8 103 per C and T0 = 20C
Substituting the values, we have
1.2 = 1.0[1 + 3.8 103(T 20)]
or 3.8 103 (T 20) = 0.2
Solving this, we get T = 72.6 C

Example 7. A resistance R of thermal coefficient of resistivity = is connected in parallel with a resistance = 3R,
having thermal coefficient of resistivity = 2. Find the value of eff .
Sol. The equivalent resistance at 0C is
R10 R 20
R0 ...(i)
R10 R 20
The equivalent resistance at tC is
R 1R 2
R ...(ii)
R1 R 2
But R1 = R10 (1 + t) ...(iii)
R2 = R20(1 + 2t) ...(iv)
and R = R0(1 + efft) ...(v)
Putting the value of (i), (iii), (iv), (v) in eqn (ii),
5
eff

4
Example 8. (a) The current density across a cylindrical conductor of radius R varies according to the equation
r
J J 0 1 , where r is the distance from the axis. Thus the current density is a maximum J at the axis
R 0

r = 0 and decreases linearly to zero at the surace r = R. Calculate the current in terms of J0 and the conductors
cross sectional area is A = R2.
(b) Suppose that instead the current density is a maximum J0 at the surace and decreases linearly to zero at the axis
r
so that J J 0 . Calculate the current.
R

www.physicsashok.in 6
ELECTRIC CURRENT
Sol. (a) We concider a hollow cylinder of radius r and thickness dr.
The cross-sectional area of considered element is dA = 2rdr
The current in considerd element is
r
dI JdA J 0 1 2rdr
R

r
or dI 2J 0 1 rdr
R
R r
I 2J 0 1 rdr
0 R

R 2 A
I J0 J0
3 3
R r2
(b) I 2J 0 dr
0 R
R
2J 0 r3
I 3
R 0

2J0 R 3 2A
I J
R 3 3 0

Example 9. A network of resistance is constructed with V V0


V1 = k0 V0 VN =
R1 & R2 as shown in the figure. The potential at the V0 R1 R1 V2 =
k2 VN1 kN
VN
points 1, 2, 3,......, N are V1, V2, V3,........, VN I I1 I2 I I1 R1 I2
respectively each having a potential k time smaller than R2 R2 R2 R3
previous one. Find:
R1 R2
(i) R and R in terms of k
2 3

(ii) current that passes through the resistance R2 nearest to the V0 in terms V0, k & R3.
V V0 V0
V0 R1 V1 = k0 R1 V2 = VN1 VN =
kN
k2 VN
I I1 I I1 R1 I2
Sol. R2 R2 R2 R3

(i) According to kcL,


I = I1 + I2
V0 V0 V0 V0
V0 0 2
or k k k k
R1 R2 R1

k 1 V0 V0 k 1 V0

or
kR 1 kR 2 k 2 R1

www.physicsashok.in 7
ELECTRIC CURRENT
2
R 1 k 1

R2 k
Also,
I = I1 + I2
V0 V V0 V0
N2
N01 N 1
0 N 1
0
or k k k k
R1 R2 R1 R 3
After solving,
R2 k

R3 k 1
V0
0
V1 0
(ii) Here, I1 k
R2 R2
V
I1 0
V0

k 1 V0
kR 2 k k2R 3
k R3
k 1

Example 10. A rod of length L and cross-section area A lies along the x-axis between x = 0 and x = L. The material
obeys Ohms law and its resistivity varies along the rod according to (x) = 0 ex/L. The end of the rod at x =
0 is at a potential V0 and it is zero at x = L.
(a) Find the total resistance of the rod and the current in the wire.
(b) Find the electric potential in the rod as a function of x.
Sol. (a) The resistance of considered element is
dx 0 x / L
dR e dx
A A
L L L
R 0 e x / Ldx 0 e x / L 0
A 0 A x dx
L L 1
R 0 e 1 1 0 1
A A e
0 L e 1
R
A e
V0 0 V0 AeV0
I
R R 0 L e 1
(b) E = J
or E 0 e x / L J

I 0 x / L 0 e x / L AeV0
E = Ie , E
A A A0 L e 1

e x / L V0
E

L 1 e 1
V0 Vx
E
x
V0 Vx = Ex
www.physicsashok.in 8
ELECTRIC CURRENT

Vx V0 Ex

V0 e x / L e 1
or
1 e
1

ELECTRIC CELL
An electric cell is a device which maintains a continuous flow of charge (or electric current) in a circuit by a
chemical reaction.
In an electric cell, there are two rods of different metals called as electrodes. These electrodes are kept in a
solution called electrolyte. On joining two electrodes by a wire the charge begins to flow in the wire, i.e.,
current flows in the wire. Inside the cell, a chemical reaction takes place in the electrolyte which maintains the
charge on the electrodes and the flow of charge in the wire is continuously maintained. Thus, a cell converts the
chemical energy into electrical energy. In the light of modern views in reference to a cell following terms need to
be reviewed.

Electromotive Force (EMF)


The emf of a cell is defined as work done by the cell in moving unit positive charge in the whole circuit including
the cell once. Therefore, if W is the work done by a cell in moving a charge q once around a circuit including the
cell,
W
emf E
q
SI unit of emf is joule/coulomb and is called volt. The emf of a cell in a circuit is taken to be positive, if circuit
current inside a cell, is from negative to positive terminal, (i.e., cell is discharging) otherwise negative as shown
in Fig.
i + + i i + + i i + + i
E1 E2 E1 E2 E1 E2
E = E1 + E2 E = E1 E2 E = E2 E1
(a) (b) (c)

NOTE : The term electromotive force is misleading introduced by Volta who thought it to be
force that causes the current to flow. Actually emf is not a force but work required
to carry unit charge from lower potential to higher potential inside the cell.

Internal Resistance (r) and Terminal Potential Difference


The potential difference across a real source in a circuit is not equal to the emf of the cell. The reason is that
charge moving through the electrolyte of the cell encounters resistance. We cell this the internal resistance of the
source, denoted by r. As the current moves through r, it experiences an associated drops in potential equal to
ir. Thus, when a current is drawn through a source, the potential difference between the terminal of the source
is,
V = E ir
This can also be shown as below :
E r
A B
i
VA E + ir = VB
or VA VB = E ir
Following three special cases are possible :
(i) If the current flows in opposite direction (as in case of charging of a battery), then V = E + ir
(ii) V = E, if the current through the cell is zero

www.physicsashok.in 9
ELECTRIC CURRENT
(iii) V = 0, if the cell is short circuited.
This is because current in the circuit
E
i
r
or E = ir
E ir = 0 Short Circuited

or V=0 r
E
Thus, we can summarise it as follows :

E r
V = E ir or V < E
i
E r
V = E + ir or V > E
i
E r
V=E if i = 0

E V=0 if short circuited


i= r

E r

COMBINATION OF RESISTANCES
In Series
Fig., represent a circuit consisting of a source of emf and two resistors connected in series.
i A A

R1 V1

V V R
R2 V2

B B
Let equivalent resistor is R as shown.
Then R = R1 + R2
This result can be readily extended to a network consisting of n resistors in series.
R = R1 + R2 + ....... + Rn

In Parallel
Fig., represents a circuit consisting of a source of emf and two resistors connected in paralle.
i A i A

i1 i2
V R1 R2 R
V

B B
If R be the equivalent resistance, then
1 1 1

R R1 R 2
This result can be extended to a network consisting of nwww.physicsashok.in
resistors in parallel. The result is
10
ELECTRIC CURRENT

1 1 1 1
..........
R R1 R 2 Rn

NOTE : In series combination, the same current flows through each resistance while in parallel
combination, the voltage drop across each resistor is equal to the source voltage V.

Example 11. Compute the equivalent resistance of the network shown in Fig., and find the current i drawn from the
battery.
18V
i i
6

4 3
Sol. The 6 and 3 resistances are in parallel. Their equivalent resistance is,
18V i

4 2
1 1 1
or R 2
R 6 3
Now this 2 and 4 resistances are in series and their equivalent resistance is 4 + 2 = 6 .
Therefore, equivalent resistance of the network = 6 .
18V
i

6
Current drawn from the battery is,
net emf 18
i= = =3A
net resistance 6

Star-Delta () Conversion
For Fig. (a) (Delta), to be equivalent to Fig. (b) (Star) Y,
A A
A
RA RA
R1 R3 R1 R3
RB RC
RB RC
B C B C C
R2 R2 B
(a) (b) (c)
R1R 3 R1R 2
RA , RB
R1 R 2 R 3 R1 R 2 R 3

www.physicsashok.in 11
ELECTRIC CURRENT

R 2R 3
and RC
R1 R 2 R 3

Example 12. Find the equivalent resistance between A and B in Fig. using Star-Delta Theorem.
10 C 10
A

10 10 10

B
10 D 10
Sol. Using Star-Delta theorem, the equivalent circuit can be drawn as shown in fig.
10 C 15
A RC = 5
RB = 5 5 9 5
10 A O
O B B A B
RD = 2.5
10 D 22.5

R CB R CD 20 10
RC 5
R CB R CD R DB 20 10 10

R DC R DB 10 10
RD 2.5
R DC R DB R CB 10 10 20

R BC R BD 20 10
RB 5
R BC R BD R DC 20 10 10
15 22.5 15 22.5
R 9
15 22.5 37.5
Hence, RAB = R + 5 = 9 + 5 = 14

Example 13. What will be the change in the resistance of a circuit consisting
of five identical conductors if two similar conductors are added
as shown by the dashed line in figure.
Sol.

www.physicsashok.in 12
ELECTRIC CURRENT
B
r

r r r
r r

A r

r r B
A r r r
r

2r
B
A r r
2r

A r r r B
R2 = Req = 3r
But before added the conductors, the equivalent resistance is
R1 Req 5r
R2 3

R1 5 .

KIRCHHOFFS LAW
Many electric circuits cannot be reduced to simple series-parallel combinations. For example, two circuits that
cannot be so broken down are shown in fig.
R1
A B A B
E1 C D
R1
R2 R2 R3 R4
D C
E2 E1 E2 E3
E F G
F E I H
R3 R5
(a) (b)
However, it is always possible to analyze such circuits by applying two rules, devised by Kirchhoff.
First here are two terms that we will use often.

Junction : A junction in a circuit is a point where three or more conductors meet. Junctions are also called nodes
or branch points.
For example, in figure (a) points D and C are junctions. Similarly, in figure (b) point B and F are junctions.

Loop : A loop is any closed conducting path. For example, in figure (a) ABCDA, DCEFD and ABEFA are loops.
Similarly, in figure (b), CBFEC, BDGFB are loops.
Kirchhoffs rules consists of the following two statements.
(i) Junction rule : The algebraic sum of the currents at any junction is zero. i2 i2

That is,
junction
i0 i3
4 i
This law can also be written as, the sum of all the curents directed towards a
www.physicsashok.in
point in circuit is equal to the sum of all the currents directed away from that point. 13
ELECTRIC CURRENT

Thus, in figure i1 + i2 = i3 + i4
The junction rule is based on conservation of electric charge.
(ii) Loop rule : The angebraic sum of the potential differences in any loop including those associated emfs and
those of resistive elements, must equal zero.

That is,
closed loop
V 0

The loop rule is based on the fact that the electrostatic field is conservative in nature.
In applying the loop rule, we need sign convention as discussed below : E
A B
Path
(a) When we travel through a source in the direction from to +, the emf is
considered to be positive. (a)
E
A B
(b) When we travel form + to , the emf is considered to be negative. Path

(b)

(c) When we travel through a resistor in the same direction as the assumed R i
A B
current, the iR term is negative because the current goes in the direction Path
of decreasing potential.
(c)
R i
(d) When we travel through a resistor in the direction opposite to the assumed A B
current, the iR term is positive because this represents a rise of potential. Path

(d)

Example 14. Find current in different branches of the electric circuit shown in figure.
4 B 2
A C

2V 4V 6V

F D
2 E 4
Sol. Applying Kirchhoffs first law (junction law) at junction B,
i1 = i2 + i3 ...(i)
Applying Kirchhoffs second law in loop 1 (ABEFA),
4i1 + 4 2i1 + 2 = 0 ...(ii)
4 2
Applying Kirchhoffs second law in loop 2 (BCDEB), B
A C
2i3 6 4i3 4 = 0 ...(iii) i1 i3
i2
Solving Eqs. (i), (ii) and (iii), we get 2V
1 4V 2 6V
i1 = 1A
i1 i3
8 F D
i2 A 2 E 4
3
5
i3 A
3
Here, negative sign of i3 implies that current i3 is in opposite direction of what we have assumed.

www.physicsashok.in 14
ELECTRIC CURRENT

IMPORTANT FEATURES R
1. Distribution of current in parallel connections : When
more than one resistances are connected in parallel, the i1 2R
potential difference across them is equal and the current is i i2
i3 3R
distributed among them in inverse ratio of their resistance, as
V
i
R
1
or i for same value of V
R
e.g., in the figure,
1 1 1
i1 : i 2 : i3 : : 6:3:2
R 2R 3R
6 6
i1 i i
6 3 2 11
3 3
i2 i i
6 3 2 11

2 2
and i3 i i
6 3 2 11

2. Distribution of potential in series connections : When more than one resistances are connected in series,
the current through them is same and the potential distributed in the ratio of their resistance, as
V = iR
or V R for same value of i.
e.g., in the figure,
R 2R 3R

i V1 V2 V3

V1 : V2 : V3 = R : 2R : 3R = 1 : 2 : 3
1 V
V1 V
1 2 3 6

2 V
V2 V
1 2 3 3

3 V
and V3 V
1 2 3 2

COMBINATION OF CELLS
Cells are usually grouped in following three ways :

In Series
Suppose n cells each of emf E and internal resistancer are connected in series as shown in figure. Then

www.physicsashok.in 15
ELECTRIC CURRENT
E r E r E r

R
Net emf = nE
Total resistance = nr + R
Net emf
Current in the circuit =
Total resistance
nE
or i=
nr + R

NOTE : If palarity of m cells is reversed, then equivalent emf = (n 2m)E


while total resistance is still nr + R

i=
n - 2m E

nr + R

In Parallel : Consider the following three cases : E r


Ist case : Let n cells each of emf E and internal resistance r are E r
connected in parallel.
Net emf = E E r
r
Total resistance R i i
n
net emf R
Current in the circuit i =
total resistance
E
or i=
R + r/n
IInd case : Let n cells have different E and r.
Net emf = Eeq
E/r E1 r1
= A F
1/ r
i1 E2 r2
Total resistance = Req B i2 E
1 i3 E3 r3
R
1/ r i i

E eq R
i C D
Hence, R eq

E / r
or i
1 R 1/ r
IIIrd case : This is most general case of parallel grouping in which E and r of different cells are different and the
positive terminals of few cells are connected to the negative terminals of the others as shown.
Net emf = Eeq

www.physicsashok.in 16
E1ELECTRIC
r
CURRENT
1


E1 / r1 E 2 / r2 E3 / r3 i1 E2 r2
1 1 1 i2
i3 E3 r3
r r r
1 2 3
i i
Total resistance = Req R
1
R
1 1 1
r r r
1 2 3
E eq
Hence, i
R eq

i
E1 / r1 E 2 / r2 E3 / r3
1 1 1
1 R
r1 r2 r3

In Mixed Grouping
There are n identical cells in a row and number of rows are m.
Net emf = nE
Total resistance = Req
E r
nr
R
m
nE
i
Hence, nr
R
m i i
R

Example 15. Find the emf and internal resistance of a single battery which is equivalent to a combination of three
batteries as shown in figure.
10V 2

6V

4V 2
Sol. The given combination consists of two batteries in parallel and resultant of these two in series with the third one.
For parallel combination we can apply,
E1 E 2 10 4

r1 r2
Eeq 2 2 3V
1 1 1 1

r2 r2 2 2
1 1 1 1 1
Further, 1
req r1 r2 2 2
req = 1
Now this is in series with the third one, i.e.,

www.physicsashok.in 17
ELECTRIC CURRENT
6V 3V

1 1
The equivalent emf of these two is (6 3) V or 3 V and the internal resistance will be, (1 + 1) or 2 .
E=3V

r=2

Example 16. A battery consists of a variable number n of identical cells having internal resistance connected in
series. The terminals of the battery are short circuited and the current I measured.
Which one of the graph below shows the relationship between I and n ?
I/A

I/A

I/A

I/A
I/A

(A) (B) (C) (D) (E)


O n O n O n O n O n

nE

Sol. nr

nE E I
I= =
nr r
n

Example 17. In previous problem, if the cell had been connected in parallel (instead of in series) which of the
above graphs would have shown the relationship between total current I and n ?
I/A

I/A
I/A

(A) (B) (C)


O n O n O n
I/A

I/A

(D) (E)
O n O n

E
r
Sol. n

E nE
I= =
r r
n
nE I
I=
r
n
Example 18. Under what condition current passing through the resistance R can be E1 r1 E2 r2
increased by short circuiting the battery of emf E2. The internal resistances of the
two batteries are r1 and r2 respectively.
R
(A) E2r1 > E1(R + r2) (B) E1r2 > E2(R + r1)
(C) E2r2 > E1(R + r2) (D) E1r1 > E2(R + r1)
Sol. The current through R before short circuit

www.physicsashok.in 18
ELECTRIC CURRENT

E1 + E 2
I=
r1 + r2 + R
After short circuit : E1 E2
E1 r1 r2
I' =
r1 + R
I > I R

E1 E + E2
> 1
r1 + R r1 + r2 + R
E 1r 1 + E 1r 2 + E 1R > E 1r 1 + E 1R + E 2r 1 + E 2R E1r2 > E 2 r1 + E 2 R
B
i1
WHEATSTONES BRIDGE

ig = 0
This is an arrangement of four resitances in which one resistance is unknown P i1
Q
and rest known. The Wheatstones bridge is shown in fig. The bridge is said A C
G
i2
to be balanced when deflection in galvanometer is zero, i.e., ig = 0 and hence, R S
the condition of deflection is
i i2
P R E D

Q S

NOTE : In Wheatstones bridge, cell and galvanometer arms are interchangeable.


B B
P Q P Q

A G C A C

R S R S

D D
G

P R
In both the cases, condition of balance bridge is, =
Q S
P

Example 19. A hemisphere network of radius a is made by using a conducting wire B


of resistance per unit length r. Find the equavalent resistance across OP. A O C
Sol. Point (A and C) and (D and B) are symmetrically located with respect to points
D
O and P. Hence, the circuit can be drawn as shown in figure.
This is a balanced Wheatstone bridge between P and O
Hence, r3 can be removed. And, P
r r
R PO 1 2 r1 r1
4 2 2

r1 R PB R PD
a r
Here B C
2 r2 O r2
2 2
and r2 = ROB = (a)r
r3

www.physicsashok.in 19
ELECTRIC CURRENT

R PO
2 ar
Ans.
8
2
Q
Example 20. In the circuit shown, what is the potential difference VPQ? 4V
2V 3
(A) +3V (B) +2V 1V
P
(C) 2V (D) none 1 2
Sol. In ABCPA
2I + 4 I + 2 = 0
6 2
I = = 2A A B I1
Q
3 I (II 1) I1
I
VP VQ = {algebriac sum of rise up 4V
2V 3
and drop up of voltage} 1 1V I1
= (2 2I) P
I C 2 D

= (2 4) VP - VQ = 2 V

Example 21. Two batteries one of the emf 3V, internal resistance 1 ohm and the a b
other of emf 15 V, internal resistance 2 ohm are connected in series with a
resistance R as shown. If the potential difference between a and b is zero
R
the resistance of R in ohm is
(A) 5 (B) 7 (C) 3 (D) 1
Sol. According to loop rule,
3 I IR + 15 2I = 0
3V 15V
18 a b
I=
3+ R
Va Vb = (3 + I) R
or 0=3I
I
I =3A
18
3=
3+ R
or 9 + 3R = 18
9
R 3
3

R1 R1 R1 R1 R1
Example 22. Consider an infinite ladder network shown . A
in figure A voltage V is applied between the points
R2 R2 R2 R2 R2
A and B. This applied velue of voltage is halved after each
section. B
(A) R1/R2 = 1 (B) R1/R2 = 1/2 (C) R1/R2 = 2 (D) R1/R2 = 3
Sol. I = I1 + I2
v v v v
v 0
2 2 2 4
R1 R2 R1
v v v
or = +
2R 1 2R 2 4R1

www.physicsashok.in 20
v I v/2 I2 v/4 ELECTRIC CURRENT

1 1 v R 2 I1 R 2 R2 R2 R2
or = +
R1 R 2 2R 1
1 1 v=0 v=0 v=0 v=0 v=0 v=0
or =
2R 1 R 2
R1 1
or =
R2 2
6 3
6 A
Example 23. If the switches S1, S2 and S3 in the figure are
S2
arranged such that current through the battery is minimum, S1 1
find the voltage across points A and B. 9 1 1
S3
Sol. For minimum current through battery, equivalent resistance 24V
across battery should be maximum. As we know, in series B
resistance increases, but in parallel, resistance decreases.
From these points of view, all switches should be open.
7 4.5 A
6 A
6 3 I I
1 0.5
9 1 1 24 V
24 V I

According to loop rule,


24 7I 4.5I 0.5I = 0
I = 2A
36V
VAB = VA VB = 0.5 I
= 0.5 2 = 1 V 6 3
Example 24. In the circuit shown in figure, calculate the following : S
(i) Potential difference between points a and b when switch S is open. a b
(ii) Current through S in the circuit when S is closed. 3 6
36 0
Sol. (i) Here I1 4A
9
36 0 V0 = 36V
Also I2 4A
9 I1 I2
6 3
Also, 36 Va = I1 6 = 6 I1
or 36 Va = 6 4 = 24 V a b
Va = 36 24 = 12 V 3 6
36 Vb = 3 4 = 12 V
V=0
and Vb = 36 12 = 24 V
Va Vb = 12 24 = 12 V
(ii) In loop ABCIHA.
6(I I1) + 3I1 = 0
or 6I + 6I1 + 3I1 = 0
or 9I1 = 6I

www.physicsashok.in 21
G
A I ELECTRIC CURRENT
B H
(I I1) I1
9 3 6 3
I I1 I1 ...(i)
6 2 C I
I2
In loop CDEJIC, 3 (I I1 + I2) 6
3(I I1 + I2) + 6(I1 I2) = 0 (I1 I2)
D E J
or 3I + 3I1 3I2 + 6I1 6I2 = 0 F
or 3I + 9I1 9I2 = 0 ...(ii)
In loop EFGHIJE,
36 3I1 6(I1 I2) = 0
or 9I1 6I2 = 36 ...(iii)
After solving eqn (i), (ii) and (iii)
I2 = 3A from b to a.

Example 25. An enquiring physics student connects a cell to a circuit and measures the current drawn from the cell
to I1. When he joins a second identical cell is series with the first, the current becomes I2. When the cells are
connected are in parallel, the current through the circuit is I3. Show that relation between the current is
3 I3 I2 = 2I1(I2 + I3)
Sol. Let the equivalent resistance of circuit is R. The emf and internal resistance of cell is E and r respectively.
E
I1
rR
2E
and I2
2r R
E
and I3
r/2R
3E 2E
L.H.S. = 3I3 I2 r / 2 R 2r R

6E 2
3I3 I 2
r / 2 R 2r R
R.H.S. = 2I1 (I2 + I3)
2E 2E E

r R 2r R r / 2 R

6E 2

r / 2 R 2r R
Hence, L.H.S. = R.H.S.

Example 26. Find the potential difference VA VB for the circuit shown in the figure.
1V 1V B 1V 1V

1 1 1 1
1
1 1 1 1
A
1V 1V 1V 1V
Sol. i1 + i2 = i
4i + i1 = 0

www.physicsashok.in 22
B
x+4 x+3 xELECTRIC
+2 i
x+1 CURRENT
x
i1 4i 3i 2i
4 i2 i1
x
9
A
4 22 4 3 2 1 0
VA VB 2 V
9 9
Example 27. Power generated across a uniform wire connected across a supply is H. If the wire is cut into n
equal parts and all the parts are connected in parallel across the same supply, the total power generated in the
wire is :
H H
(A) (B) n2H (C) nH (D)
n2 n

Sol. Let the resistance of wire is R = r
A
r = the resistance of each piece
r R
= =
nA n
v2
H= t ...(i)
R
when all pieces are corrected in parallel,
Then equivalent resistance is
r R
r0 = = 2
n n
v2 n 2 v2
H1 = t = t n2H from eqn. (i)
r0 R

Example 28. The ratio of powers dissipatted respectively in R and 3R, as shown is : R

(A) 9 (B) 27/4


(C) 4/9 (D) 4/27 3R
Sol. P1 = Power dissiatted in resistance R
2R
2
2I 4
R I2 R R
3 9
and P2 = Power dissipatted in 2I/3 3R
3R = I2(3R) = 3I2R I I

P1 4 I2R P1 4 2R I/3
=
P2 9 3 I2 R P2 27
y

Example 29. In the figure shown the power generated in y is maximum when y = 5. 10V,
R
(A) 2 (B) 6
(C) 5 (D) 3 2R
Sol. According to KOL :
10 2 I I y I R = 0
y
10
I= I
2+y+R
10V R
P = I 2y I
100 y I
or P=
(2 + y + R) 2
For maximum power dissipatted,
www.physicsashok.in 23
ELECTRIC CURRENT

dP
0
dy
By solving,
y=2+R
or 5=2+R R 3

Example 30. Find the current through 25V cell & power supplied by 20V cell in the figure shown.
10V 5V 20V 30V
25V

10

11
5

5
Sol. From figure,
25V 25V 25V 25V 25V
55 0 5V 20V 30V
I1 5A 10V
11 15V 30V 5V 55V 25V
50
I2 1A

10

11
5

5
5 I4 I3 I2 I1 I=I1+I2+I3+I4
30 0 4
I3 3A V=0 V=0 V=0 V=0 V=0
10
15 0
I4 3A
5
Electric current through 25V cell
= I1 + I2 + I3 + I4 = 5 + 1 + 3 + 3 = 12 A
The power supplied by 20V cell is
P = 20I2= 20 1 = 20 W

Example 31. The current I through a rod of a certain metallic oxide is given by I = 0.2 V5/2, where V is the
potential difference across it. The rod is connected in series with a resistance to a 6V battery of negligible
internal resistance. What value should the series resistance have so that :
(i) the current in the circuit is 0.44 (the value of (2.2)2/5 = 1.37)
(ii) the power dissipated in the rod is twice that dissipated in the resistance.
Sol. (i) the potential difference across rod for current 0.44 is
2
0.44 5
V
0.2
2
V 2.2 5
The potential difference across connected resistor is V = 0.44 R
E = V + V
2
or 6 2.2 5 0.44R
2
or 0.44 R 6 2.2 5
2

R
6

0.2 5
0.44 0.44
R = 13.64 3.12
R = 10.52
(ii) Total power supplied by battery is used by rod and resistor
E I = V I + I2 R
www.physicsashok.in 24
ELECTRIC CURRENT

But V I = power dissipated in rod


V I = 2I2R
EI = 2I2R + I2R = 3I2R
or E = 3IR
or 6 = 3IR
IR = 2 ...(i)
Also, V + V = E
2
I 5
IR 6
0.2
2
I 5
or 26
0.2
2
I 5
or 4
0.2
2
I
or 4 5
25 32
0.2
or I = 32 0.2 = 6.4 A
From eqn (i), IR = 2
2 20
R 0.3125
6.4 64
Remarks : In the case of rod, V I graph is not straight line. So, ohms law is not applicable in the case of the
given rod.

Example 32. A person decides to use his bath tub water to generate electric power to run a 40 watt bulb. The
bath tub is located at a height of 10 m from the ground & it holds 200 litres of water. If we install a water driven
wheel generator on the ground, at what rate should the water drain from the bath tub to light bulb ? How long
can we keep the bulb on, if the bath tub was full initially. The efficiency of generator is 90%. (g = 10 m/s2)
dm
Sol. Power P gh
dt
dm 90
or 40 gh
dt 100
dm
or 40 0.9 gh
dt
dm 40 40 4
kg / sec.
dt 0.9 gh 9 10 9

dm
m t
dt 0

m 200
t0 450 sec.
dm 4

dt 9

www.physicsashok.in 25
ELECTRIC CURRENT

3 4
Example 33. The circuit shown in figure is made of a homogeneous wire of uniform
cross-section. 1234 is a square. Find the ratio Q12/Q34 of the amounts of heat
liberated per unit time in conductor 12 and 34. 1 2
Sol. Let us represent the central junction of wires in the form of two junctions connected
by the wire 56 as shown in figure. then in follows from symmetry that there is no current through this wire.
Therefore, the central junction can be removed from the initial circuit. Further,
R12 = R13 = R34 = R24 = r
3 4 3 4
r
and R15 R 25 R 36 R 46 6 6
2
5 5
Let V be the voltage between 1 and 2. 1 2 1 2
Then the amount of heat liberated in conductor 1 2.
V2
Q12 ...(i)
r
V
i 3 4
Current through 3 4, r 2 3
2 V2
Q3 4 i 3 4 r 2
r 23
Q1 2 2
Q 3 4

2 3 11 6 2 Ans.

Metre Bridge
The metre bridge is the practical application of the Wheatstones network principle.
P Q
B

G
R J S
A C
l (100 l)
D

In such a bridge, the ratio of two resistances say R and S, can be determined from the ratio of their balancing
lengths. In Fig., AC is a 1 m long uniform wire,
Let AD = l cm, then DC = (100 l) cm
Since, resistance length
P AD l

Q DC 100 l
If P is known, then Q can be determined.

Example 34. The potentiometer wire AB is 100 cm long. When AC = 40 cm, no deflection occurs in the
galvanometer, find R.

www.physicsashok.in 26
ELECTRIC CURRENT

10 R

A B

10 AC
Sol.
R CB
CB 100 40 60
R 10 (10) 10 15
AC 40 40

10
Example 35. A battery of emf 0 = 10 V is connected across a 1 m long uniform
0 =10V
wire having resistance 10/m. Two cells of emf 1 = 2V and 2 = 4V having P
internal resistances 1 and 5 respectively are connected as shown in A B
1
the figure. If a galvanometer shows no deflection at the point P, find the
distance of point P from the point A. 1 =2V G
5
Sol. The resistance in AP is R1 = x = 10 x
2 =4V
where x is in metre.
The resistance in PB is R2 = (1 x)10
The equivalent circuit is I 10V I
According to loop rule
10 IR1 IR2 10I = 0 R1 P R2
A B
10 10 1 I
I E1 E2
E=r r + r =
5 2
+
4
= 14 V
R 1 R 2 10 10 10 2 1 2 6 1 5 6
G
But VA VP = E = IR1 r=
15
1+5
=
5
6
14 1
10x
6 2
14 2
x 0.4667 m 46.67 cm
6 10
R1 R2
Example 36. In the figure shown for which values of R1 and R2 the balance
G
point for Jockey is at 40 cm from A. When R2 is shunted by a resistance
of 10 , balance shifts to 50 cm. Find R1 and R2. (AB = 1 m) : A B
Sol. Let resistance perunit length of potentiometer is . Assume that P is
contact point of potentiometer.
The resistance in AP is R3 = x
and the resistance in PB is
R4 = (1 x) ( AB = 1 m)
According to balance condition of wheatstone bridge,
R1R4 = R2R3
or R1(1 x) = R2 x
or R1(1 x) = R2x ...(i)
when R2 is shunted.
www.physicsashok.in 27
ELECTRIC CURRENT

R 2 10 10R 2
Then R2
R 2 10 R 2 10
R1(1 x) = R2 x
10R 2 x
R1 1 x ...(ii)
R 2 10
From eqn. (i) and (ii),
10
R1 and R2 = 5.
3

IMPORTANT FEATURES
1. When battery and galvanometer arms of a Wheatstones bridge are interchanged, the balance position remains
undisturbed while sensitivity of bridge changes.
I I1 R

I1 IG G (I I 1 + IG )
A B
I P S I

(I1 I G) Q

2. Two other common forms of balanced Wheatstones bridge are shown.

G P Q
A
R B
S

POTENTIOMETER
Potentiometer is an ideal device to measure the potential difference between two points. It consists of a long
resistance wire AB of uniform cross-section in which a steady direct current is set up by means of a battery.
E1

i i
L
A B
i C
i2 = 0
G
E2 , r
Potential gradient
Potential difference across AB
k
Total length
VAB
k
L
iR AB
k i
L

www.physicsashok.in 28
ELECTRIC CURRENT

R AB
where = resistance per unit length of potentiometer wire
L
The emf of source balanced between points B and C
R
E 2 kl i CB l
l
E2 = iRCB

Applications
(i) To find emf of an unknown battery :
E1 E1

i
l1 l2
A B A B
i C1 i C2
i2 = 0 i2 = 0
G G
EK EU
We calibrate the device by replacing E2 by a source of known emf EK and then by unknown emf EU. Let the
null points are obtained at lengths l1 and l2. Then,
EK = i( l1)
and EU = i( l2)
Here, = resistance of wire AB per unit length.
E K l1

E U l2
l
or EU 1 EK
l2
So, by measuring the lengths l1 and l2, we can find the emf of an unknown battery.
(ii) To find the internal resistance of a call : Firstly the emf E of the cell is balanced against a length AD = l1.
For this the switch S is left opened and S is closed. A known resistance R is then connected to the cell as
shown. The terminal voltage V is now balanced against a smaller length AD = l2. Here, no switch S Is opened
and S is closed. Then,
l1
E l1 l2
S
V l2 E

E Rr A
D D
B
Since, { E = i(R + r) and V = iR}
V R (E, r)
R r l1 S
G
or
R l2
R
l
or r 1 1 R
l2

Example 37. A potentiometer wire of length 100 cm has a resistance of 10 . It is connected in series with a
resistance and a cell of emf 2V and of negligible internal resistance. A source of emf 10 mV is balanced against
a length of 40 cm of the potentiometer wire. What is the value of external resistance ?

www.physicsashok.in 29
ELECTRIC CURRENT
Sol. From the theory of potentiometer,
E1
VCB = E, if no current is drawn from the battery
i
E1 R
or R R R CB E
AB A B
C
Here, E1 = 2V, RAB = 10
E
40 G
R CB 10 4
100
and E = 10 103 V
Substituting in above, we get
R = 790

MOVING COIL GALVANOMETER


Moving coil galvanometer is a device used to detect small current flowing in an electric circuit. With suitable
modifications, it can be used to measure current and potential difference.

Conversion of galvanometer into an Ammeter


An ammeter is an instrument which is used to measure current in a circuit in ampere (or milli-ampere or micro-
ampere). Hence, it is always connected in series in the circuit.
Since, the galvanometer coil has some resistance of its own, S
therefore, to convert a galvanometer into an ammeter, its is
resistance is to be decreased so, to convert a galvanometer a
G
b
i
into ammeter a low resistance, called shunt (S) is connected
in parallel to the galvanometer as shown in figure.
Ammeter
S
Here, i g i
S G
RA
S G
and
G S
where RA = resistance of ammeter
S = resistance of shunt
G = resistance of galvanometer

Conversion of Galvanometer into Voltmeter


A Voltmeter is an instrument which is used to measure the potential
difference between two points of an electric circuit directly in volt
G
(or milli-volt or micro-volt). Hence, it is connected in parallel across ig R
those two points of the circuit, between which the potential difference
Voltmeter
is to be measured. When it is connected.
Since, the resistance of coil of galvanometer of its own is low, hence, to convert a galvanometer into a voltmeter,
high resistance R is series is connected with the galvanometer.
V
Here, ig
RG
where R + G = RV = resistance of voltmeter

Example 38. A moving coil galvanometer of resistance 10 produces full scale deflection, when a current of 25

www.physicsashok.in 30
ELECTRIC CURRENT

mA is passed through it. Describe showing full calculations, how will you convert the galvanometer into a
voltmeter reading upto 120 V.
Sol. Here, G = 10 , ig = 25 mA = 25 103 A
To convert the galvanometer into voltmeter reading upto 120 V : To convert a galvanometer into voltmeter
of range V, a large resistance R has to be connected in series to it. The value of R is given by
V
R G
ig
Here, V = 120 V
120
R 10 4790
25 103
, 10 mA
Example 39. A milliammeter of range 10 mA and resistance 9 is joined in
a circuit as shown. The metre gives full-scale deflection for current I
when A and B are used as its terminals, i.e., current enters at A and leaves
at B (C is left isolated). The value of I is A B C
(A) 100 mA (B) 900 mA (C) 1 A (D) 1.1 A
Sol. According to loop rule,
9 10 0.9 10 + 0.1 (I 10) = 0 10 mA
90 + 9 10 mA
or I - 10 = = 990
0.1 I10
I
I = 1000 mA 1A
I

Example 40. In the circuit shown in figure reading of voltmeter is V1 when only S1 3R
R S1
is closed, reading of voltmeter is V2 when only S2 is closed. The reading 6R
of voltmeter is V3 when both S1 and S2 are closed then S2
V
(A) V2 > V1 > V3 (B) V3 > V2 > V1
(C) V3 > V1 > V2 (D) V1 > V2 > V3 E

Sol. Step-I When S1 is closed,


R
I
4R 3R

v1 I 3R 3R
4R
3
v1
4
Step-II When S2 is closed
R

I 6R
7R
v2 I 6 R
6
v2 6R
7R 7
Step-III When S1 and S2 both are closed.
R

I 2R
3R
v3 = I 2R

www.physicsashok.in 31
ELECTRIC CURRENT

2
v3 2R
3R 3 1000
A
Example 41. In the circuit shown in figure the reading of ammeter is
R 50
the same with both switches open as with both closed. then find the
resistance R. (ammeter is ideal)
Sol. Step-I : Discuss the circuit when both switches open : +
According to loop rule : 1.5 300 I 100 I 50 I = 0 300 1.5V

1.5 15 1
I A
450 4500 300 100
A
Step-II : Discuss the circuit after closing the switch. I
In loop ABCDEA
50
IR + 1.5 300I1 = 0
or 300I1 + IR = 1.5 ...(1) I
In loop BCGFB
100I + (I1 I) R = 0 300 I I
1.5 V
or (I1 I)R = 100 I
I1R = (100 + R)I

I1
100 R I
...(2) I 100
R B A C D
no current
I1
From eqn (1) and (2)
R 50
G
300
100 R I IR 1.5 F
(I1 I)
I1
R
A
300 I1 1.5V E

300
100 R 1

R
1.5
or
R 300 300
R = 600

Example 42. The battery in the diagram is to be charged by the generator G. The R
generator has a terminal voltage of 120 volts when the charging current is +
G

10 amperes. The battery has an emf of 100 volts and an internal resistance of +
1 ohm. In order to charge the battery at 10 amperes charging current, the
resistance R should be set at
(A) 0.1 (B) 0.5 (C) 1.0 (D) 5.0
Sol. VA VB = {algebraic sum of rise up and drop up of voltage} A R

120 = {IR 1 I 100}


120 = IR + I + 100
or 20 = 10R + 10 B I
100V
I

R 1

Example 43. A galvanometer having 50 divisions provided with a variable shunt s is used to measure the current
when connected in series with a resistance of 90 and a battery of internal resistance 10 . It is observed that
when the shunt resistance are 10, 50, respectively the deflection are respectively 9 & 30 divisions. What is
the resistance of the galvanometer ?
www.physicsashok.in 32
ELECTRIC CURRENT

Sol. The electric current through galvanometer is proportional to its deflection.


I
Ig 9 3

Ig 30 10
IR S
But I
g

R Rg S
E
I
R gRS
Also, 100
Rg RS
ER S
Ig
R g RS
100 R R R g R S


g S

RS
R g RS
Ig


100
R g R S

R g RS

Ig RS
or
R g RS
100 R R R g RS


g S

10
10R g
3
100
R g 10

R g 10

10

or 50 R 50
50R g g
100

R g 50
Rg = 233.3 .

IMPORTANT FEATURES
1. (a) The reading of an ammeter is always lesser than actual current in the circuit.
i

R R
i i A G S

E E i
(a) (b) (c)
For example, in Fig. (a), actual current through R is,
E
i ...(i)
R
GS
while the current after connecting an ammeter of resistance A in series with R is,
G S
E
i ...(ii)
RA
From Eqs. (i) and (ii), we see that i < i and i = i ,When A = 0, i.e., resistance of an ideal ammeter should be
zero.
www.physicsashok.in 33
ELECTRIC CURRENT
(b) Percentage error in measuring a current through an ammeter is
1 1

i i R R A
100 1 100
i

R
A
or % error 100
R A
2. (a) The reading of a voltmeter is always lesser than true value.
r r
i i RV i
V

R
G
For example, if a current i is passing through a resistance r, the actual value is,
V = ir ...(i)
Now if a voltmeter of resistance RV(= G + R) is connected across the resistance r, the new value will be
i rR V
V
r RV
ir
V
r
or 1 ...(ii)
RV
From Eqs. (i) and (ii), we can see that
V < V
and V = V if RV =
Thus, resistance of an ideal voltmeter should be infinite.
(b) Percentage error in measuring the potential difference by a voltmeter is,

V V 1 1
100 100 % error 100
V 1 r or 1 r
RV RV

RC CIRCUITS
In preceding sections we dealt only with circuits in which the current did not vary with time. Here, were discuss
about time-varying currents. q = CV
0
C +
Charging of a Capacitor
First we consider the charging of a capacitor without resistance.
+ +
S V V
Consider a capacitor connected to a battery of emf V through
a switch S. On
closing the switch, there is no time tag between connecting and q
charging. A charge q0 = CV comes in the capacitor as soon as
switch is closed and q t graph in this case is a straight line q0
parallel to t=axis.
If we employed a resistance in the circuit, charging takes some time Fig.
r
The q t equation in this case,
q = q0(1 et/)

www.physicsashok.in 34
ELECTRIC CURRENT
C C
Here, q0 = CV
and = CR = time constant S V
q t graph is an exponentially increasing graph. The charge q increases q
expponentially from 0 to q0. q0
From the graph and equation we see that, 0.632q 0
At t = 0, q=0
and at t = , q = q0
t
At t = , q = q0(1 e1) = 0.632 q0
Here, can be defined as the time in which 63.2% charging is over in a C R circuit.
Current flows in a C R circuit during charging of capacitor. Once charging i
as over or the steady state condition is reached the current becomes zero. i0
The current at any time t can be calculated bay differentiating q with respect
to t. Hence,
Charging current is, i = i0et/
t
i.e., current decreases exponentially with time.
Time i t graph is as shown in fig.

Discharging of a Capacitor
Again we consider the discharging of capacitor with resistance.
Suppose a capacitor has a charge q0. The positive plate has a charge +q0 and negative plate q0.
q0 q=0
+

q0
+ R
S
When the switch is closed, the extra electrons on negative plate immediately C
comes to the positive plate and net charge on both plates become zero. So,
we can say that discharging takes place immediately.
S
In case of C R circuit discharging also takes time. The q t equation in this case is,
q q 0e t / c
Thus, q decreases exponentially from q0 to zero, as shown in Fig.
q
At t = 0, q = q0 q0
At t = , q=0
In case of discharging definition of is charged. 0.368q 0
At time t = ,
q = q0 e1 = 0.368q0 t
Hence, in this case can be defined as the time when charge reduces to 36.8% of its maximum value q0.
During discharging current flows in the circuit till q becomes zero. This current can be found by differentiating
q with respect to t but with negative sign because charge is decreasing with time. Hence, discharging current
is, i
i = i0et/ i0
This is an exponentially decreasing equation. Thus, i t graph decreases
exponentially with time from i0 to 0. The i t graph is as shown in Fig.
t

www.physicsashok.in 35
ELECTRIC CURRENT
C
Example 44. A capacitor C = 100 F is connected to three resistor each of resistance
1 k and a battery of emf 9V. The switch S has been closed for long time so as to 1k
charge the capacitor. When switch S is opened, the capacitor discharges with S
time constant 1k
(A) 33 ms (B) 5 ms 1k 9V
(C) 3.3 ms (D) 50 ms
Sol.

1k

1k at t = 0

R = 0.5 k
Time constant
= RC = 0.5 103 100 106
= 50 103 S = 50 m/s

Example 45. In the circuit shown in figure C1 = 2C2. Switch S is closed at time R
t = 0. Let i1 and i2 be the currents flowing through C1 and C2 at any time t, C1
then the ratio i1/i2 R
(A) is constant (B) increases with increase in time t C2
(C) decreases with increase in time t S
(D) first increases then decreases V
t
V
Sol. Here I1 e RC1
R
t
V RC 2
and I2 e
R
t
RC1 t 1 1
I1 e
R C C
t e 1 2
I2
e RC 2
t C1 C2
I1 RC1C2
e
I2
Hence option (B) is correct.
S
Example 46. In the RC circuit shown in the figure the total energy of 3.6 103 J is
dissipated in the 10 resistor when the switch S is closed. The initial charge on the 2F 10
capacitor is

www.physicsashok.in 36
ELECTRIC CURRENT

60
(A) 60 C (B) 120 C (C) 60 2 C (D) C
2
Sol. According to conservation principle of energy :
Total energy stored on capacitor appears as heat in resistor.
q 20
H 3.6 103
2C
or q02 = 2C 3.6 103
q02 = 2 2 106 3.6 103
q02 = 14.4 109
q0 = 12 105
q0 = 120 C
+
C
Example 47. The capacitors shown in figure has been charged to a potential R1
difference of V volts, so that it carries a charge CV with both the switches
S1
S1 and S2 remaining open. Switch S1 is closed at t = 0. At t = R1C switch R2
S1 is opened and S2 is closed. Find the charge on the capacitor at
E S2
t = 2 R1C + R2C.
Sol. When t < R1C
t
R 1C
q CV e
At t0 = R1C,
q0 q0
CV
q 0 CV e 1
e R1
when S2 is closed, R2
t = t0 E
At instant t (t > t0)
q
E I R1 R 2 0
C I
+q q
or (R1 + R2)CI = EC q
R1+R2
dq
or R1 R 2 C dt EC q
I E
q dq t
or R1 R 2 CCV EC q t0 R1C dt
e

q t
or R1 R 2 C
ln EC q CV t R C
1
e

CV
or R 1 R 2 C ln EC ln EC q t R 1C
e
CV
EC
R1 R 2 C ln e tR C
or 1
EC q

But t = 2R1C + R2C
Putting the value, www.physicsashok.in 37
ELECTRIC CURRENT

1 CV
q CE 1 2
e e

Example 48. In the figure shown initially switch is open for a long time.
R S
Now the switch is closed at t = 0. Find the charge on the rightmost
capacitor as a function of time given that it was initially unchanged. V R C C
Sol. Step-I : Discuss the circuit, when switch is open.
V
Here I
2R
I +q
CV
Also, q 0 CIR q0
2 I I
Step-II : Discuss the circuit after closing the switch :
V R q
I 1 0
2 2 C R
R
V
V R q q q1
Also, I 0 0
2 2 C
V R q q q
or I 0 1 0 q q 0+q q 1 q1
2 2 C C C
R V
2 (q0+qq1) q1
V R q q V IR 2
or I 0 0
2 2 C C 2 2 q
q q
or V IR 0 0
C C
or IR V 0
q q
C
dq q q
or R V 0
dt C
q Rdq t
0 q0 q 0 dt

or V
C
q
q q
RC ln V 0 t
or
C 0
q0 q
V C
RC ln t
or q0
V
C
q0 q
V t
C e RC
or q
V 0
C

www.physicsashok.in 38
ELECTRIC CURRENT
t
q0 q q 0 RC
or V
V e ...(i)
C C
But potential differ accross each capacitors are sume.
q 0 q q1 q1

C C
or q0 + q = 2q1
q0 q
q1
2
Putting the value of q,
t
CV 1 RC
q1 1 e
2 2

THINKING PROBLEMS

1. Is a current-carrying conductor electrically charged?


2. Is there an electric field near the surface of a conductor carrying direct current?
3. Is current a scallar or vector?
4. A potential difference V is applied to a copper wire of diameter d and length l. What is the effect on the
electron drift velocity of (a) doubling l, (b) doubling V, and (c) doubling d?
5. A current i enters the top of a copper sphere of radius R and leaves through the diametrically opposite
point. Are all parts equally effective in dissipating Joule heat?
6. Account for the increase in the resistance of metals with rise in temperature.
7. Answer briefly how can three resistances of values 2, 3 and 6 be connected to produce an
effective resistance of 4 ?
8. How can an electric heater designed for 220V be adopted for 110V without changing the length of the
coil and also without a change in the consumed power ?
9. The brilliance of lamps in a room noticeably drops as soon as a highpower electric iron is switched on and
after a short interval, the bulbs regain their original brilliance. Explain.
10. A current is passed through a steel wire which gets heated to a dull red. Then half the wire is immersed in
cold water. The portion out of the water becomes brighter. Why?
11. For manual control of the current of a circuit, two rheostats in parallel are preferable to a single rheostat.
Why?
12. In a hollow nonconducting pipe, there are two streams of ions in opposite directions. The ions of one
stream are negatively charged and constitute a current of strength l and those of the other stream are
positively charged and constitute a current of the same strength. What is the total current through the
pipe?
13. The drift velocity of electrons is quite small. How then does a bulb light up as soon as the switch is turned
on, although the bulb may be quite far from the switch?
14. Consider a voltaic cell in the open circuit with the copper plate at a higher potential with respect to the
zinc plate. The electrolyte between the plates is a conducting medium. So the charges on the plates
should be neutralized at once as it happens when two charged conductors are joined by a wire. Why are
the charges not neutralized immediately?
15. Does emf have electrostatic origin?
16. Why is it easier to start a car engine on a warm day than on a chilly day?
17. The resistance of the human body is about 10 k. If the resistance of our body is so large, why does one

www.physicsashok.in 39
ELECTRIC CURRENT

experience a strong shock from a live wire of 220 V supply?


18. Lay people have the notion that a person touching a high power line gets stuck to the line. Is this true?
If not, what is the fact?
19. When a direct current flows through a conductor, the amount of energy liberated is VQ, where Q is the
charge passing through the conductor and V is the potential difference, while an energy of VQ/2 is
liberated when a capacitor is discharged. Why?
20. An ordinary cell with a small emf can produce larger current than an electrostatic machine which generates
thousands of volts? Explain.

THINKING PROBLEMS SOLUTIONS


1. No, the amount of positive and negative charge in any elementary volume remains the same through the
electrons are in motion. So a current-carrying conductor is electrically neutral.
2. There is an electric field inside the conductor. This is equal to the rate of fall of potential along the conductor.
There is continuity of this field outside the surface of the conductor.
3. Current is a scalar but current density is a vector.
4. u = j/ne = E/ne where s is the conductivity and E is the electric field (by Ohms law j = E), or u = (/ne)
(V/l) ( E = V/l). Obviously from this expression (a) u is halved if l is doubled, (b) u is doubled if V is
doubled, and (c) u remains uncharged on doubling d.
5. No, the resistance of elements at right angles to the diameter varies from element to element but current is
the same through all sections and so heating effect varies from section to section.
6. metals have mobile electrons. With increase of temperature, the lattice vibrations increase in amplitude.
Thus, the probability of electrons striking ionic cores increases. This amounts to increase in resistance
with temperature.
7. Connect the 3 and 6 resistors in parallel and the 2 resistor in series with the combination of 3 and
3 6
6. Then 3 and 6 in parallel will sum up to 2 and this, with 2 in series, will work out to
36
4.
8. Join the ends of the coil and apply the supply voltage (110V) between this common
terminal and the midpoint of the coil as shown in the figure. Let P be the power of
the entire coil when the coltage applied is V volts. T he P = V 2/R where R is the
resistance of the coil. When it is connected as shown in the figure, the two parts
are in parallel and the resistance of each part becomes R/2.
Therefore,
P1 = (V/2)2/(R/2)=P/2
Total power = P/2 + P/2 = P
9. The cold resistance of the coil of the iron is much smaller than when it is hot. So when the iron is switched
on, there is a large drop of voltage and consequently the bulbs do not receive the proper voltage. As the
coil gets heated, its resistance increases and so the voltage drop is made up when it is fully heated. This
is why after some time the bulbs region their brilliance.
10. The resistance of the immersed portion decreases and so the current through the entire wire increases.
This is why the portion outside the water becomes brighter.
11. Suppose, to produce a certain current i in a circuit, the length of the single rheostat wire required is l.
Then i = E/lr where E is the emf of the cell and r is the resistance per unit length of the wire. To produce
the same current in the same circuit by using two rheostats in parallel, let l be the length of each rheostat
wire. Then i = E/(lr/2) = 2E/lr. Obviously, l = 2l. Thus more length of each wire is required to produce
the same current. This is definitely of advantage.
12. The current constituted by the negative ions moving opposite to the positive ions has the same sense as
the current constituted by the positive ions. Hence the total current is 2I and not zero.
13. For the bulb to light up, it is not necessary for the same electron to travel from the switch to the bulb.
www.physicsashok.in
When the switch is turned on, every electron in the circuit begins to move simultaneoulsy, including those
40
in the filament of the bulb.
14. The charges are prevented from being neutralized because the electrostatic field due to the charges on the
plates is opposed by the charges of ions that migrate into the solution.
ELECTRIC CURRENT

REASONING TYPE QUESTIONS


A statement of assertion (A) is given and a Corresponding statement of reason (R) is given just below it
of the statements, mark the correct answer as
(A) If both A and R are true and R is the correct explanation of A.
(B) If both A and R are true but R is not the correct explanation of A.
(C) If A is true but R is false.
(D) If both A and R are false.
(E) If A is false but R is true.

1. Assertion (A) : When a wire is stretched so that its thickness is halved, its resistance would become 16 times.
Reason (R) : The data is insufficient to predict.

2. Assertion (A) : A current flows in a conductor only when there is an electric field within the conductor.
Reason (R) : The drift velocity of electron in presence of electric field decreases.

3. Assertion (A) : A current carrying wire should be changed.


Reason (R) : The current in a wire is due to flow of free electrons in a definite direction.

4. Assertion (A) : The wires supplying current to an electric heater are not heated appreciably.
Reason (R) : Resistance of connected wires is very small and H R .

5. Assertion (A) : In meter bridge experiment, a high resistance is connected in series with the galvanometer.
Reason (R) : As resistance increases current through the circuit increases.

6. Assertion (A) : A 60 watt bulb has greater resistance than a 100 watt bulb.

V2
Reason (R) : P
R

7. Assertion (A) : The conductivity of an electrolyte is very low as compared to a metal at room temperature.
Reason (R) : The number density of free ions in electrolyte is much smaller compared to number density
of free electrons in metals. Further, ions drift much more slowly, being heavier.

8. Assertion (A) : Terminal voltage of a cell is greater than emf of cell, during charging of the cell.
Reason (R) : The emf of a cell is always greater than its terminal voltage.

9. Assertion (A) : Material used in the construction of a standard resistance is constantan or manganin.
Reason (R) : Temperature coefficient of these materials is very small.

10. Assertion (A) : If the current of a lamp decreases by 20%, the percentage decrease in the illumination of
the lamp is 40%.
Reason (R) : Illumination of the lamp is directly proportional tot he current through lamp.

11. Assertion (A) : Heater wire must have high resistance and high melting point.
Reason (R) : If resistance is high, the electrical conductivity will be less.

12. Assertion (A) : The range of given voltmeter can be increased.


Reason (R) : By adjusting the value of resistance in series with galvanometer the range of voltameter
can be adjusted.

www.physicsashok.in 41
ELECTRIC CURRENT

LEVEL # 1
1. The resistance of tungsten increases with increasing temperature. As a result, the relation between the
current, , flowing in the tungsten filament of an electric lamp and the potential difference, V, between itss
ends is of the form.
I I I I

(A) (B) (C) (D)


O O O O
V V V V
2. The current in a copper wire is increased by increasing the potential difference between its ends. Which one
of the following statements regarding n, the number of charge carriers per unit volume in the wire, and Vd , the
drift velocity of the charge carriers, is correct?
(A) n is unaltered but v d is decreased. (B) n is unaltered but v d is increased.
(C) n is increased but v d is decreased. (D) n is increased but v d is unaltered.

3. A potentiometer is to be calibrated with a standard


cell using the circuit shown in the diagram.
The balance point is found to be near L. To improve
accuracy the balance point should be nearer M.
This may be achieved by
(A) replacing the galvanometer with one of lower resistance.
(B) replacing the potentiometer wire one of higher resistance
per unit length.
(C) putting a shunt resistance in parallel with the galvanometer.
(D) increasing the resistance R.

4. When the switch 1 is closed, the current


through the 8 resistance is 0.75 A.
When the switch 2 is closed (only), the
current through the 2 resistance is 1 A.
The value of is
(A) 5 V (B) 5 2 V (C) 10 V (D) 15 V
R
5. The time constant for the shown RC circuit is
RC R
(A) RC (B) (C) 2 RC (D) not defined
2

6. Two cells of equal emf of 10 V but different internal resistances 3 and 2 are connected in series to an
external resistance R. The value of R that makes the potential difference zero across the terminals of one of
any cells is
(A) 5 (B) 6 (C) 1 (D) 1.5

7. The resistor in which the maximum heat is produced is given by


(A) 2 (B) 3
(C) 4 (D) 12

8. n resistances each of resistance R are joined with capacitors of capacity


C (each) and a battery of emf E as shown in the figure. In steady state
condition ratio of charge stored in the first and last capacitor is R C R C R
(A) n : 1 (B) (n1) : R E
2 2
(C) (n + 1) : (n 1) (D) 1 : 1

www.physicsashok.in 42
ELECTRIC CURRENT

9. What is the equivalent capacitance between A and B in the


circuit shown.
(A) 6 F (B) 1.5 F
(C) Zero (D) 2 F

10. A resistance R = 12 is connected across a source of emf as shown in the figure. Its emf changes with time
as shown in the graph. What is the heat developed in the resistance in the first four second.

R = 12 (Volt)

24

Source
4 t(s)
(A) 72 J (B) 64 J (C) 108 J (D) 100 J

11. A source of constant potential difference is connected across a A B


conductor having irregular cross-section as shown in the Figure. Then P Q
(A) electric field intensity at P is greater than that at Q
(B) rate of electric crossing per unit area of cross section at P is less than that at Q
(C) the rate of generation of heat per unit length at P is greater than that at Q
(D) mean kinetic energy if free electorn at P is greater than that at Q

12. Suppose a voltmeter reads the voltage of a very old cell to be 1.40 V while a potentiometer reads its voltage
to be 1.55 V. What is the internal resistance of the cell
(A) 20 (B) 30 (C) 10 (D) 40

13. In the above question, What is the current it would supply to a 5 resistor. Assume the voltmeter resistance
be be 280 .
(A) 44 A (B) 0.044 A (C) 4.4 A (D) None of the above.

14. If 1 , 2 and 3 are conductances of three conductors then their equivalent conductance when they are
joined in series will be
1 1 1 1 2 3
(A) 1 + 2 + 3 (B) (C) (D) None of these
1 2 3 1 2 3

15. A conductor is made of an isotropic material (resistivity ) has rectangular


cross-section. Horizontally dimension of the rectangle decreases linearly from
2x at one end to x at the other end and vertial dimension increases from y to 2y
as shown in Figure. Length of the conductor along the axis is equal to . A battery
is connected across this conductor then
(A) resistance of the conductor is equal to 4 9xy .
(B) rate of generation of heat per unit length is maximum at middle cross-section.
(C) drift velocity of conduction electrons is minimum at middle section.
(D) at the ends of the conductor, electric field intensity is same.

16. Find the current through 4 resistor just after making the circuit
(A) 0 A (B) 6 A
(C) 12 A (D) 2 A

17. 1 m long metallic wire is broken into two unequal parts P and Q. P part of the wire is uniformly extended into
another wire R. Length of R is twice the length of P and the resistance R is equal to that of Q. Find the ratio
of the resistance of P and R
(A) 1 : 4 (B) 1 : 3 (C) 1 : 2 (D) 1 : 1

www.physicsashok.in 43
ELECTRIC CURRENT
18. Which of the following statements is/are correct for potentiometer circuit
(A) Sensitivity varies inversely with length of the potentiometer wire
(B) Sensitivity is directly proportional to potential difference applied across the potentiometer wire.
(C) Accuracy of a potentiometer can be increased only by increasing length of the wire
(D) Range depends upon the potential difference applied across the potentiometer wire.

19. In the given circuit the ammeter reading is zero. What is the value
of resistance R ?
(A) R = 100 (B) R = 10
(C) R = 0.1 (D) None of these

20. What is the equivalent resistance between A and B in the given


circuit diagram.
(A) 2 (B) 12
(C) 20 (D) 10

21. A cell of emf. 1.5 V and internal resistance 0.5 is connected to a (non-linear) conductor whose -V graph is
shown in Figure. Find the current drawn from the cell and its terminal voltage

(A) 1.5 A and 2 V (B) 1 A and 1 V (C) 1 A and 2 V (D) 2 A and 1.5 V

22. You are given several identical resistances each of value R = 10 and each capable of carrying a maximum
current of 1A. It is required to make a suitable combination of these resistances to produce a resistance of
5 which can carry a current of 4 A. The minimum number of resistances required is
(A) 4 (B) 10 (C) 8 (D) 20
23. A conductor or area of cross section A having charge carriers, each having a charge q is subjected to a
potential V. the number density of charge carriers in the conductor is n and the charge carriers (along with
their random motion) are moving with a velocity v. If is the conductivity of the conductor and is the
average relaxation time, then
m m 2 m m
(A) (B) (C) (D)
2
nq nq 2
nq 2
2 nq 2
24. A vacuum diode consists of plane parallel electrodes separated by a distance d and each having an area A.
On applying a potential V to the anode with respect to the cathode a current I flows through the diode.
Assume that the electrons are emitted with zero velocity and they do not change the field between the
electrodes. The electron velocity is v and the charge density is at any point between the electrodes at a
distance x from the cathode. If I is the equivalent current, m is the mass of each charge carrier, then
2 eVx md eVx 2 md
(A) v (B) 2 (C) v (D)
md 2 e VxA 2 md e VxA 2

25. A long round conductor of cross-sectional area A is made of a material whose resistivity depends on the

radial distance r from the axis of the conductor as 2 , is a constant. The total resistance per unit
r
length of the conductor is R and the electric field strength in the conductor due to which a current I flow in
it is .
2 4 2 I 4I
(A) R 2 (B) R 2 (C) 2 (D)
A A A A2

www.physicsashok.in 44
ELECTRIC CURRENT

26. A 100 W bulb B1 and two 60 W bulbs B2 and B3 are connected to a 250 V
source as shown in figure. Now W 1, W 2 and W 3 are the output powers of the
bulbs B1, B2 and B3 respectively. Then
(A) W 1 > W 2 = W 3 (B) W 1 > W 2 > W 3
(C) W 1 < W 2 = W 3 (D) W 1 < W 2 < W 3 [JEE 2002 (Scr)]

27. Variation of current passing through a conductor as the voltage


applied across its ends is varied as shown in Fig. 9.55. If the
resistance is determined at points A, B, C and D. We will find that :
(A) Resistance at C and D are equal
(B) Resistance at B is higher than at A
(C) Resistance at C is higher than at B
(D) Resistance at A is lower than at B

28. The P. D. between the points A and B in the circuit shown here is 16 V. Which is/are the correct statement(s)
out of the following ?
9V 3V
(A) the current through the 2 resistor is 3.5 A 4 1 1 3
A B
(B) the current through the 4 resistor is 2.5 A
2
(C) the current through the 3 resistor is 1.5 A
(D) the P.D. between the terminals of the 9 V battery is 7 V.

29. A and B are two points on a uniform ring of resistance R. The ACB = , where C is the centre of the ring.
The equivalent resistance between A and B is
R 2
(A) ( 2 ) (B) R 1 2 (C) R 2 (D) R
4 2 4

30. The charge flowing through a resistance R varies with time t as Q = at bt 2. The total heat produced in R is
a3 R a3 R a3 R a3 R
(A) (B) (C) (D)
6b 3b 2b b

31. A resistance R carries a current I . The heat loss to the surroundings is (TT 0) where is a constant, T
is the temperature of the resistance, and T 0 is the temperature of the atmosphere. If the coefficient of linear
expansion is , the strain in the resistance is
2
(A) proportional to the length of the resistance wire (B) equal to I R

1 2
(C) equal to I R (D) equal to (IR)
2

32. The potential of the point O is


(A) 3.5 V (B) 6.5 V
(C) 3 V (D) 6 V

4 6
33. In the circuit shown in Figure the heat produced in the 5 ohm resistor
due to the current flowing through it is 10 calories per second.
The heat generated in the 4 ohms resistor is
(A) 1 calorie/sec (B) 2 calories/sec 5
(C) 3 calories/sec (D) calories/sec [JEE 1981]

www.physicsashok.in 45
ELECTRIC CURRENT
34. A battery of internal resistance 4 is connected to the network of R R
resistance as shown. In order that the maximum power can be delivered E 6R R
R
to the network, the value of R in should be [JEE1995] R 4R
4
4 8
(A) (B) 2 (C) (D) 18
9 3

35. A steady current flows in a metallic conductor of non-uniform cross-section. The quantity/quantities constant
along the length of the conductor is/are [JEE1997]
(A) current, electric field and drift speed. (B) drift speed only
(C) current and drift speed (D) current only.
P Q
36. In the circuit P R, the reading of the galvanometer is
S
same with switch S open or closed. Then [JEE1999]
(A) R G (B) P G R
G
(C) Q G (D) Q R
v
37. A wire of length L and 3 identical cells of negligible internal resistances are connected in series. Due to the
current, the temperature of the wire is raised by T in a time t. A number N of similar cells is now connected
in series with a wire of the same material and cross-section but of length 2L. The temperature of the wire is
raised by the same amount T in the same time t, the value of N is [JEE2001]
(A) 4 (B) 6 (C) 8 (D) 9

38. The effective resistance between points P and Q of the 2R 2R


electrical circuit shown in the figure is [JEE2002]
2R
2Rr 8R R r
(A) (B) P Q
Rr 3R r r r
2R
5R 2R 2R
(C) 2 r + 4 R (D) 2r
2

39. Express which of the following set ups can be used to verify Ohms law?

V A V
(A) (B)

V
(C) (D)
V A

40. The three resistance of equal value are arranged in the different combination shown below. Arrange them in
increasing order of power dissipation. [JEE2003]
(I) a (II) b (III) c (IV) c

(A) i (B) (C) (D) i


i i

www.physicsashok.in 46
ELECTRIC CURRENT

41. In the given circuit, no current is passing through the galvanometer. If the cross-
sectional diameter of AB is doubled then for null point of galvanometer the value
of AC would [JEE2003 (Scr)] G
A B
(A) x (B) x/2 (C) 2x (D) None x C
P
42. Six equal resitances are connected between points P, Q and R as shown in the
figure. Then the net resistance will be maximum between
[JEE2004 (Scr)]
(A) P and Q (B) Q and R
Q R
(C) P and R (D) any two points

B C D

43. For the post office box arrangement to determine the value of unknown
resistance, the unknown resistance should be connected between
[JEE2004 (Scr)]
(A) B and C (B) C and D
(C) A and D (D) B1 and C1 B1 C1

44. A capacitor is charged using an external battery with a resistance x in series. The
S
dashed line shows the variation of n with respect to time. If the resistance is
R
changed to 2x, the new graph will be [JEE2004] ln I
(A) P (B) Q Q
(C) R (D) S
P
t
45. A moving coil galvanometer of resistance 100 is used as an ammeter using a resistance 0.1 . The maximum
deflection current in the galvanometer is 100 A. Find the minimum current in the circuit so that the ammeter
shows maximum deflection. [JEE2005]
(A) 100.1 mA (B) 1000.1 mA (C) 10.01 mA (D) 1.01 mA
10V 10
46. In the figure shown the current through 2 resistor is
(A) 2 A (B) 0 A 5
2 20V
(C) 4 A (D) 6 A [JEE2005 (Scr)]

47. A galvanometer has resistance 100 and it requires current 100 A for full scale deflection. A resistor 0.1
is connected to make it an ammeter. the smallest current required in the circuit to produce the full scale deflection
is [JEE2005 (Scr)]
(A) 1000.1 mA (B) 1.1 mA (C) 10.1 mA (D) 100.1 mA

48. Consider a cylindrical element as shown in the figure. Current flowing A B


C
through the element is I and resistivity of material of the cylinder is . 4r I
2r
Choose the correct option out the following.
(A) Power loss in second half is four times the power loss in first half. l/2 l/2
(B) Voltage drop in first half is twice of voltae drop in second half.
(C) Current density in both halves are equal.
(D) Electric field in both halves is equal. [JEE2006]

www.physicsashok.in 47
ELECTRIC CURRENT
49. A resistance of 2 is connected across one gap of a meter-bridge (the length of the wire is 100 cm) and an
unknown resistance, greater than 2, is connected across the other gap. When these resistances are
interchanged, the balance point shifts by 20 cm. Neglecting any corrections, the unknown resistance is
[JEE2007]
(A) 3 (B) 4 (C) 5 (D) 6

MORE THAN ONE CHOICE MAY BE CORRECT

50. Capacitor C1 of capacitance 1 micro-farad and capacitor C2 of capacitance 2 microfarad are separately charged
fully by a common battery. The two capacitors are then separately allowed to discharge through equal resistors
at time t = 0. [JEE1989]
(A) The current in each of the two discharging circuits is zero at t = 0.
(B) The currents in the two discharging circuits at t = 0 are equal but not zero.
(C) The currents in the two discharging circuits at t = 0 are unequal.
(D) Capacitor C1, losses 50% of its initial charge sooner than C2 loses 50% of its initial charge.
3 2 2
51. In the circuit shown in Figure the current through
(A) the 3 resistor is 0.50 A. (B) the 3 resistor is 0.25 A
(C) the 4 resistor is 0.50 A (D) the 4 resistor is 0.25 A 9V 8 8 4

52. Which of the following is/are wrong?


2 2 2
(A) A current carrying conductor is electrically charged.
(B) There is an electric field inside a current carrying conductor.
(C) For manual control of the current of a circuit, two rheostats in series are preferable to a single rheostat.
(D) None of these

53. A constant voltage is applied between the two ensd of a uniform metallic wire. Some heat is produced in it.
The heat developed is doubled if:
(A) both the length and radius of the wire are halved. (B) both the length and radius of the wire are doubled.
(C) the radius of the wire is doubled.
(D) the length of the wire is doubled and the radius of the wire is halved. R1 R2

54. All the resistance in the given Wheatstone bridge have different values and the
current through the galvanometer is zero. The current through the galvanometer R3 R4
will still be zero if,
(A) the emf of the battery is doubled. (B) all resistance are doubled.
(C) resistance R1 and R2 are interchanged. (D) the battery and the galvanometer are interchanged.

55. Two cells of equal emf and having different internal resistance R1 and R2 (R2 > R1) are connected in series. If
resistance of connecting wires is equal to R, which of the following statements is/are correct?
(A) At a particular value of r, potential difference across second cell can be equal to zero.
(B) If R = 0, negative terminal of second cell will be at higher potential than its positive terminal
(C) Negative terminal of first cell can never be at higher potential than its positive terminal.
(D) None of these

56. When electric bulbs of same power but with different marked voltages are connected in series across a power
line, their brightness will be:
(A) proportional to their marked voltages. (B) inversely proportional to their marked voltages.
(C) proportional to the squares of their marked voltages.
(D) inversely proportional to the square of their voltages.

57. Two conductors made of the same material have lengths L and 2 L, but have equal resistances. The two are
connected in series in a circuit in which current is flowing. Which of the following is/are correct?
(A) The potential difference across the two conductors is the same.
(B) The electric drift velocity is larger in the conductor of length 2L.
(C) The electric field n the first conductor is twice that in the second.
(D) The electric field in the second conductor is twice that in the first.

www.physicsashok.in 48
ELECTRIC CURRENT
58. Mark correct statements:
(A) Heat is always generated in a battery whether it charges or discharges.
(B) When a battery supplies current in an external circuit, heat generated in it is always less than electrical
power developed in it.
(C) Potential difference across terminals of a battery is always less than its emf. (D) None of these.

59. A mocrometer has a resistance of 100 and a full scale range of 50 A . It can be used as a voltmeter or as
a higher range ammeter provided a resistance is added to it. Pick the correct range and resistance combinations.
(A) 50 V range with 10 k resistance in series. (B) 10 V range with 200 k resistance in series.
(C) 5 mA range with 1 resistance in parallel. (D) 10 mA range with 1 resistance in parallel.

60. An electric current is passed through a circuit containing three wires arranged in parallel. If the length and
radius of the wires are in ratio 2 : 3 : 4 and 3 : 4 : 5, then the ratio of current passing through wires would be:
(A) 54 : 64 : 75 (B) 9 : 16 : 25 (C) 4 : 9 : 25 (D) 3 : 6 : 10.

61. When a galvanometer is shunted with a 4 resistance, the deflection is reduced to one-fifth. If the
galvanometer is further shunted with a 2 wire, the further reduction in the deflection will be (the main
current remains the same).
(A) 8/15 of the deflection when shunted with 4 only. (B) 5/13 of the deflection when shunted with 4 only.
(C) 3/4 of the deflection when shunted with 4 only. (D) None of these

62. During the charging of a storage battery, the current is 22 A and the voltage is 12 V. The rate of heat generated
in the battery is 12 W. The rate of change of internal energy is:
(A) 240 J/s (B) 252 J/s (C) 264 J/s (D) 126 J/s.

63. A cell of emf 5 V, internal resistance 1 will give maximum power output to:
(A) a single resistor of 1 (B) two 1 resistors connected in series.
(C) two 1 resistors connected in parallel. (D) two 2 resistors connected in parallel.

64. In the circuit shown in figure, the current through: 3 2 2

(A) the 3 resistor is 0.50 A. (B) the 3 resistor is 0.25 A.


9V 8 8 4
(C) the 4 resistor is 0.50 A. (D) the 4 resistor is 0.25 A.

65. For the circuit shown in the figure 2 2 2

I 2k R1

[JEE2009]
24V 6k R2 RL 1.5k

(A) the current I through the battery is 7.5 mA


(B) the potential difference across RL is 18 V
(C) ratio of powers dissipated in R1 and R2 is 3
(D) if R1 and R2 are interchanged, magnitude of the power dissipated in RL will decrease by a factor of 9

www.physicsashok.in 49
ELECTRIC CURRENT

FILL IN THE BLANKS


1. An electric bulb rated for 500 watts at 100 volts is used in a circuit having a 200 volts supply. The resistance
R that must be put in series with the bulb, so that the bulb delivers 500 Watts is ........... ohms. (1997)
2. The equivalent resistance between points A and B of the circuit (Figure) given below is ........... .
3. In the circuit (Figure) shown above, each battery is 5 V and has an internal resistance of 0.2 ohm.

A 2R 2R R B V

Fig - 1 Fig - 2
The reading in the ideal voltmeter V is ......... V. [JEE 1997]

TRUE / FALSE
4. Electrons in a conductor have no motion in the absence of a potential difference across it. [JEE 1982]
T1
5. The current-voltage graphs for a given metallic wire at two different temperatures T 1 T2
and T 2 are shown in the figure. [JEE 1985]
I
The temperature T 2 is greater than T 1.

TABLE MATCH
6. Column I Column II
(A) Current (P) Mircoscopic quantity
(B) Current Density (Q) Macroscopicquantity
(C) Electric field (R) Parallel to the conductor boundaries
(D) Resistance (S) Flux associated with current density

7. Two bulbs A and B consume same power P when operated at voltage VA and VB respectively. Bulbs are
connected with a supply of d.c source then:
Column I Column II
(A) In series connection, the ratio of (P) RA/RB
potential difference across A and B
(B) In series connection, the ratio of (Q) V2A/V2B
power consumed by A and B
(C) In parallel connection, the ratio of (R) RB/RA
current in A and B
(D) In parallel connection, the ratio of (S) V2B/V2A
power consumed in A and B

8. In a R-C circuit.
Column I Column II
1
(A) Charging current at tiem t = 0 (P) CV 2
2
(B) Discharging current at t = 0 (Q) Maximum
(C) While charging energy stored (R) Capacitor becomes short circuit
(D) While charging energy dissipated as heat (S) Exponential law

www.physicsashok.in 50
ELECTRIC CURRENT
9. A battery has an emf E and internal resistance r. A variable resistor R is connected across the terminals of the
battery.
Column I Column II
(A) Current in the circuit is maximum (P) R
(B) Potential difference across the (Q) R=0
terminals is maximum
E
(C) Power delivered to the resistor is maximum (R) i
r
(D) Power delivered to the load is zero (S) r=R

10. Consider two identical cells each of emf E and internal resistance r connected to a load resistance R.
Column I Column II
E2
(A) For maximum power transfer to load (P)
4r
if cells are connected in series
E2
(B) For maximum power transfer to load if (Q)
2r
cells are connected in parallel
r
(C) For series combination of cells (R) Eeq E , req
2
(D) For parallel connection of cells (S) Eeq 2 E , req 2 r

PASSAGE TYPE QUESTIONS

PASSAGE # I
A physics instructor devises a simple electrical circuit setup in which one can easily insert various resistors
and capacitors in series and parallel combinations. One can have only resistor combinations, only capacitor
combinations, or capacitor-resistor combinations. The circuit is usually used for DC (direct current studies
but can also be used for AC (alternating current) studies. The DC battery voltage is 6 volts. The AC rms
voltage is 120 volts (at 60 Hz.) The student inserts the resistors and /or capacitors as instructed and has
available suitable ammeters and voltmeters for both DC and AC experiments. (There are also three resistors,
each of 2 ohms resistance. There are also three capacitors, each of 1 microfarad capacitance.)
1. All three resistors are connected in series and the combination is connected to the 6-volt DC battery. What
voltage drop occurs across each individual resistor as measured by the voltmeter?
(A) 0.33 V (B) 1.0 V (C) 2.0 V (D) 6.0 V
2. One capacitor and one resistor are connected in parallel. The ends of this combination are then connected t
the 6-V DC battery, what are the final current and voltage, respectively, across the 1 microfarad capacitor?
(A) 0 A, 6 V (B) 0.33 A, 3 V (C) 0.33 A, 6 V (D) 6 A, 6 V
3. Two of the 2-ohm resistors are connected in parallel and the 120-V AC voltage is applied to the ends of this
parallel combination. What current will the AC ammeter measure if connected so it measures only the current
through one of the resistors?
(A) 1 A (B) 2 A (C) 60 A (D) 120 A
4. Two resistors are connected in parallel and the set of parallel resistors is then connected in series with the
third resistor. If this series parallel combination is connected across the 6-volt battery. What total DC current
is drawn from the battery?
(A) 0.5 A (B) 2 A (C) 3 A (D) 6 A
5. One capacitor and one resistor are connected is series. The 120 V, 60 Hz, A.C. voltage is then applied to this
series RC circuit. What is the AC current through this series circuit?
(A) 0.045 A (B) 0.50 A (C) 0.72 A (D) 40.0 A

www.physicsashok.in 51
ELECTRIC CURRENT

PASSAGE # II
A set of experiments in the physics lab is designed to develop understanding of simple electrical circuit
principles for direct current circuits. The student is given a variety of batteries, resistors, and DC meters; and
is directed to wire series and parallel combinations of resistors and batteries making measurements of the
currents and voltage drops using the ammeters and voltmeters. The student calculates expected current and
voltage values using Ohms law and Kirchoffs circuit rules and then checks the results with the meters.
6. A student connects a 6-volt battery and a 12-volt battery in series and then connects this combination
across a 10-ohm resistor. What is the current in the resistor?
(A) 0.8 A (B) 0.9 A (C) 1.8 A (D) 3.6 A
7. Resistors of 4 ohms and 8 ohms are connected in series. A battery of 6 volts is connected across the series
combination. How much power, in watts, is consumed in the 8-ohm resistor?
(A) 0.67 W (B) 2 W (C) 12 W (D) 24 W
8. Two 4-ohm resistors are connected in series and this pair is connected in parallel with an 8-ohm resistor. A
12 volt battery is connected across the ends of this parallel set. What power, in watts, is consumed in the
8-ohm resistor in this case?
(A) 0.9 W (B) 2.0 W (C) 4.4 W (D) 18 W
9. A 6-votl battery is connected across a 2-ohm resistor. What is the heat energy dissipated in the resistor in
5 minutes?
(A) 430 joules (B) 560 joules (C) 4300 joules (D) 5400 joules
10. A 12-volt battery is connected across a 4-ohm resistor and the heat energy dissipated in the resistor in 10
minutes is used to heat 2 kg of water (which is thermally insulated so that no heat escapes). The initial
temperature of the water is 20C and the specific heat of the water is 4184 joule/kg-C. What is the final
temperature of the water at the end of the 10 minutes of heating?
(A) 22.6 C (B) 28.4C (C) 34.2C (D) 56.4C

PASSAGE # III
In the laboratory, the voltage across a particular circuit element can be measured by a voltmeter. A voltmeter
has a very high resistance and should be connected in parallel to the circuit element whose voltage is being
measured. Connected improperly, the voltmeter will affect the circuit, interrupting it and preventing current
from flowing through the circuit element that it is meant to measure.
An experiment is conducted in which a voltmeter is used to investigate voltages in a circuit containing a
capacitor and a light bulb. The bulb and the capacitor are connected in series with a battery and the
voltmeter is placed in different position: in the first case across the capacitor in the second case across the
light bulb, and in the third case across the battery (see figure 1). The voltmeter reading is recorded every 10
seconds. The voltage for Case 1 as a function of time is shown in figure 2.
voltmeter V capacitor
capacitor capacitor
voltmeter

V
bulb
bulb bulb
battery
battery battery V
voltmeter
Case 1 Case 2 Case 3
Voltage

Time
Fig. 2
A capacitor consists of two conducting plates separated by a nonconducting material. When a battery is
connected to a circuit containing a capacitor and a light bulb in series, a current will flow, causing positive
charge to accumulate on one capacitor plate and an equal amount of negative charge to accumulate on the
other. After the current has flowed for a finite time, the capacitor will be fully charged. The ratio of the
absolute amount of charge on one plate to the voltage across the plates is defined as the capacitance; this
is constant for a given capacitor. A light bulb is a resistor that, when enough current flows through it,
becomes hot enough to emit energy in the form of light. (Note : Assume that the battery has no internal
resistance and that the resistance of the light bulb is constant).

www.physicsashok.in 52
ELECTRIC CURRENT
11. In the circuit shows in figure 1, which of the following conditions would indicate that the capacitor was fully
charged
I. A voltmeter connected across the capacitor reads a constant voltage.
II. The light bulb in the circuit stops shining.
III. The voltage across the bulb equals the voltage across the battery
(A) I Only (B) III Only (C) I and II only (D) I, II, and III

12. Which one of the following graphs could correctly represent the voltage across the battery as a function of
time during the experiment described in the passage
Voltage

Voltage

Voltage

Voltage
(A) (B) (C) (D)
Time Time Time Time
13. After the experiment described in the passage is completed, the battery is taken out of the circuit and the
wires are reconnected. Which of the following graphs, represents the voltage across the capacitor as a
function of time
Voltage

Voltage

Voltage

Voltage
(A) (B) (C) (D)
Time Time Time Time
14. How will the voltage across the light bulb vary with time as the capacitor is charging
(A) It will decrease, because as the capacitor plates fill will charge, they will impede further charge, which
will decrease the current and the voltage across the bulb.
(B) It will remain the same, because as the capacitor plates fill with charge and impede the current, the
voltage output of the battery will increase to keep the current constant.
(C) It will increase, because as the capacitor plates fill with charge, they will induce further charge, which
will create a greater voltage across the bulb.
(D) It will increase, because as the capacitor plates fill with charge, the voltage across the capacitor will
decrease, and therefore the voltage across the light bulb will increase.
15. The light bulb shown in figure 1 is replaced first with two identical resistor in series, and then with the same
two resistors in parallel. The total time taken for the capacitor to charge is measured in both cases, and
found to be longer for the first case. It can be deduced that
(A) When the resistance of the circuit si increased, the capacitance of the capacitor increases.
(B) the presence of resistors affects the final voltage across the capacitor plates.
(C) the more charge is absorbed by the resistors as the resistance of the circuit increases.
(D) the presence of resistors hinders the flow of charge, thus reducing the current in the circuit.
16. In the diagram below, a voltmeter is connected in series to a circuit that includes a battery and two bulbs in
series. The bulbs, which had been shining in the absence of the voltmeter immediately stop shining. How
might the circuit be modified in order to make the bulbs shine steadily again with their former brilliance
without removing the voltmeter
voltmeter
V

bulb bulb
battery

V V
V V
(A) (B) (C) (D)

www.physicsashok.in 53
ELECTRIC CURRENT

LEVEL # 2
1. In the figure shown,
2 3

1 F 3 F

3 2

6V, 0.5
find the charge on 2 F each capacitor in steady state. [in C ].

2. In the given circuit diaram,


1 C 2
A E

B F
3 D 4

50 V
find the current passing through wire CD [in Ampere]

3. It is required to send a current of 8 A through a circuit whose resistance is 5 . What is the least number
of cells which must be used for their purpose and how should they be connected? Emf of each cell is 2V and
internal resistance is 0.5 .
100 V, 1
4. In the circuit shown, the capacitor is charged by a battery of emf
100 V and 1 internal resistance by closing the switch. Calculate
the heat generated across 99 resistance during the charging of s
capacitor. [in Joule].
0.1 F 99
5. In a Wheatstones bridge a battery of 2 volt and internal resistance 2 ohm is used. Find the value of the
current through the galvanometer in that unbalanced condition of the bridge when P = 1 ohm, Q = 2 ohm,
S = 30 ohm and resistance of galvanometer is 4 ohm.

6. A 20 volt battery with an internal resistance of 6 is connected to a resistor of x ohms. If an additional 6


resistance is connected across the battery find the value of x so that external power supplied by battery
remains the same.
S

7. Find how the voltage across the capacitor C varies with time t after E R C
capacitors C varies with time t after the shorting of the switch S at
the moment t = 0.
R

8. A homogeneous poorly conducting medium of resistivity fills up the space between two thin coaxial
ideally conducting cylinders. The radii of the cylinders are equal to a and b, (a < b) the length of each
cylinder is . Neglecting the edger effects. Find the resistance of the medium between cylinders.

www.physicsashok.in 54
ELECTRIC CURRENT

9. Find the current flowing through the resistance R in the


circuit shown in figure. The internal resistance of the
batteries are negligible.

A R1

10. A circuit shown in figure has resistance R1 = 20 and R2 = 30 . At


what value of the resistance Rx will the thermal power generated in it R2 Rx
be practically independent of small variation of that resistance ? The
voltage between the points A and B is supposed to be constant in this case. B

11. An ammeter and voltmeter are connected in series to a battery with an emf E = 6.0v. When a certain
resistance is connected in parallel with the voltmeter, the reading of the latter decreases = 2.0 times
whereas the reading of ammeter increases the some number of time. Find the voltameter reading after the
connecting of the resistance.

E1
10 1
12. In the circuit shown in the figure the current through 3 E2 E3
resistance is 2A. If E1 = 12V, E2 = 14V, what is the value 1
of E ? Internal resistance of each battery is 1 .
3 1.5

13. The circuit shows a capacitor C two batteries, two resistors and a switch
S. Initially S has been open for a longtime. It is then closed for a long time
by how much does the change on the capacitor change over this time
period ? Assume C = 10 , E1 = 1.0 V, E3 = 3.0v, R1 = 0.20 , R2 = 0.40 .

S 10

14. In the given network switch S is closed at t = 0. Find the current


through the 10 ohm resistor at t = 75 sec.
10V 20V 2F

15. Two coils connected in series have resistance of 600 and 300 and temperature co-efficient of 0.001
and 0.004 (C)1 respectively at 20C. Find resistance of the combination at a temperature of 50C. What is
the effective temperature co-efficient of combination. G

16. The resistance of the galvanometer G in the circuit is 25 . The meter


deflects full scale for a current of 10 mA. The meter behaves as an R1 R2 R3
ammeter of three different ranges. The range is 010 A, if the terminals
O and P are taken; range is 01 A between O and R. Calculate the
resistance R1, R2 and R3.

www.physicsashok.in 55
ELECTRIC CURRENT
17. Calculate the steady current in the 2-ohm resistor shown in the circuit in the figure. The internal resistance of
the battery is negligible and the capacitance of the conductor C is 0.2 microfarad.

OR
Two resistors, 400 ohms, and 800 ohms are connected in series with a 6-volt battery. It is desired to measure
the current in the circuit. An ammeter of a 10 ohms resistance is used for this purpose. what will be the
reading in the ammeter? similarly, If a voltmeter of 10,000 ohms resistance is used to measure the potential
difference across the 400-ohms resistor, What will-be the reading in the voltmeter.

LEVEL # 3
1. A part of circuit in a steady state along with the currents flowing in the branches, the values of resistances
etc., is shown in the figure. Calculate the energy stored in the capacitor C (4 F) [JEE 1986]

2. An infinite ladder network of resistances is constructed with a 1 ohm and 2 ohm resistances, as shown in
figure. [JEE 1987]

The 6 volt battery between A and B has negligible internal resistance:


(i) Show that the effective resistance between A and B is 2 ohms.
(ii) What is the current that passes through the 2 ohm resistance nearest to the battery?

3. An electrical circuit is shown in Figure. Calculate the


potential difference across the resistor of 400 ohm,
as will be measured by the voltmeter V os resistance
400 ohm, either by applying Kirchhoffs rules or
otherwise. [JEE 1996]

www.physicsashok.in 56
ELECTRIC CURRENT

4. Find the emf (V) and internal resistance (r) of a single


battery which is equivalent to a parallel combination of
two batteries of emfs V1 and V2 and internal resistance
r1 and r2 respectively, with polarities as shown in figure. [JEE 1997]

5. A leaky parallel plane capacitor is filled completely with a material having dielectric constant K = 5 and
electrical conductivity = 7.4 x 1012 1 m 1. If the charge on the plane at instant t = 0 is q = 8.85 mC, then
calculate the leakage current at the instant t = 12 s. [JEE 1997]

6. In the circuit shown in Figure, the battery is an ideal one, with


emf V. The capacitor is initially uncharged. The switch S is
closed at time t = 0. [JEE 1998]
(A) Find the charge Q on the capacitor at time t.
(B) Find the current in AB at time t. What is its limiting value as t :

7. A thin uniform wire AB of length 1m, an unknown resistance X


and a resistance of 12 are connected by thick conducting strips,
as shown in the figure. A battery and a galvanometer (with a sliding
jockey connected to it) are also available. Connections are to be
X 12
made to measure the unknown resistance X using the principle of
Wheatstone bridge. Answer the following questions. [JEE 2002] A B C D
(A) Are there positive and negative terminals on the galvanometer?
(B) Copy the figure in your answer book and show the battery and the galvanometer (with jockey) connected
at approxiate points.
(C) After appropriate connections are made, it is found that no deflection takes place in the galvanometer
when the sliding jockey touches the wire at a distance of 60 cm from A. Obtain the value of the resistance
of X.

8. How a battery is to be connected so that the shown rheostat


will behave like a potential divider? Also indicate the points out
which output can be taken. [JEE 2003]
A R B

9. Draw the circuit diagram to verify Ohms Law with the help of a main resistance of 100 and two galvanometers
of resistances 106 and 103 and a source of varying emf. Show the correct positions of voltmeter and
ammeter. [JEE 2004]

10. An unknown resistance is to be determined using resistances R1, R2 or R3. Their corresponding null points are
A, B and C. Find which of the above will give the most accurate reading and why? [JEE 2005]

X R
G R = R1 or R2 or R3

A B C

www.physicsashok.in 57
ELECTRIC CURRENT

ANSWER KEY
Reasoning Type
Que. 1 2 3 4 5 6 7 8 9 10 11 12
Ans. C C D A C A A C A D B A

Level # 1

Q. 1 2 3 4 5 6 7 8 9 10
Ans. C B D D A C C D D B
Q. 11 12 13 14 15 16 17 18 19 20
Ans. B B B D CD A A ABCD A C
Q. 21 22 23 24 25 26 27 28 29 30
Ans. B C B AB C D A ACD A A
Q. 31 32 33 34 35 36 37 38 39 40
Ans. B A B B D A B A A A
Q. 41 42 43 44 45 46 47 48 49 50
Ans. A A C B A B D A A BD
Q. 51 52 53 54 55 56 57 58 59 60
Ans. D AC B ABD ABC C AC AB BC A
Q. 61 62 63 64 65
Ans. B B AD D AD

Fill in the Blanks / TrueFalse / Match Table

1. 20 2. R2 3. 0 4. F 5. T
6. A Q, R and S, B P and R, C P and R, D Q
7. A P and Q, B P and Q, C R and S, D R and S
8. A Q, R and S, B Q and S, C P and S, D P and S
9. A Q and R, B P, C S, D Q, P and R
10. A Q, B Q, C S, D R

Passage Type
Que. 1 2 3 4 5 6 7 8 9 10
Ans. C A C B A C B D D A
Que. 11 12 13 14 15 16
Ans. C A B A D A

www.physicsashok.in 58
ELECTRIC CURRENT

Level # 2
72 1
1. C 2. 2 A 3. 160 4. 495 J 5. Amp 6. x = 7.5
11 9
E b ER 2 R 3 E0R 3
7. V =
2

1 e2t RC 8. n
2 a
9. I = RR R R R
2 3 2 3

R1 R 2 E
10. Rx = R R = 12 11. = 2.0 V 12. E = 7V
1 2 1

13. decreases by 13.3 c 14. I = 20 mA 15. 954 , 0.002 (C)1

16. R1 = 0.025 , R2 = 0.2275 , R3 = 2.5275 17. 0.9A or 4.96 x 103 A, 1.95 V

Level # 3
1. 2.88 x 104 J 2. (ii) 1.5 A 3. 10/3 V

V1r2 V2 r1 r1 r2 CV V V 2 t 3 RC V
4.
r1 r2 ,
r1 r2 5. 0.199 A 6. (a)
2

1 e 2 t 3 RC ; (b)
2R 6 R
e ;
2R

7. (a) No (b) (c) 8


G
X 12

A J B D
C

8. Battery should be connected across A and B. Out put can be taken across the terminals A and C or B and C.

Voltmeter
6
10
G1

100
Ammeter
9.
G2 103

E
10. This is true for r1 = r2; So R2 given most accurate value.

XXXX

www.physicsashok.in 59
MAGNETIC FIELD

DHANALAKSHMI NAGAR

NEAR ANNAMAIAH CIRCLE,

TIRUPATI.

PH NO. 9440025125
MAGNETIC FIELD
THEORY OF MAGNETIC EFFECT OF ELECTRIC CURRENT
CONCEPT OF MAGNETIC FIELD
The space around a current carrying conductor, in which its magnetic effect can be experienced, is called
magnetic field.
In magnetics, there are basically two methods of calculating magnetic field at some point. One is Biot Saverts
law which gives the magnetic field due to an infinitesimally small current carrying wire at some point and the
other is Amperes law, which is useful in calculating the magnetic field of a highly symmetric configuration
carrying a steady current.

BIOT-SAVARTS LAW

According to this law, the magnetic field dB at the point P due to the small current element of length dl is
given by
0 idl sin P
dB 2
Wb / m 2 or tesla
4 r i
r
where 0 is a constant and is called, permeability of free space.
0 = 4 107 Wb/Am

Rules to Find the Direction of Magnetic Field i


Current
(i) Right hand palm rule no. 1 : If we spread our right hand in such a way P B
carrying
that thumb is towards the direction of current and fingures are towards that conductor
point where we have to find the direction of field then the direction of field
will be perpendicular to the palm

(ii) Maxwells right handed screw rule : If a right handed cork screw is rotated so that its tip moves in the
direction of flow of current through the conductor, then the rotation of the head of the screw gives the direction
of magnetic lines of force.
Current carrying
i conductor
Magnetic line
P2 P1 of force


NOTE : By convention the direction of magnetic field B perpendicular to paper
going inwards is shown by and the direction perpendicular to the paper
coming out is shown by .

Applications of Biot-Savarts Law


Let us consider few applications of Biot-Savarts Law :
(i) Magnetic field due to a straight thin conductor is
i
2
i p
B 0 sin 1 sin 2 d 1
4d
(a) For an infinitely long straight wire,
1 = 2 = 90

www.physicsashok.in 1
MAGNETIC FIELD
0i
B
2d
(b) When wire is semi-infinite, i

1 0 and 2
2
0i
B sin 0 sin d
4d 2
i
B 0
4 d
1 B
(c) B , i.e., Bd graph for an infinitely long straight wire is a rectangular
d
hyperbola as shown in figure.
(ii) Magnetic field on the axis of a circular coil having N turns is d
0 NiR 2
B 3/ 2

2 R2 x2
O x P
Here, R = radius of the coil R
x = the distance of point P from centre
and i = current in the coil
(a) At the centre of the loop, x = 0
0 Ni
B
2R
(b) For x > > R, x + R x2
2 2

0 NiR 2 0 2NiR 2 0 2M
B
2x 3 4 x 3 4 x 3

Here, M = magnetic moment of the loop


M = NiA = Ni R2

0 2M
NOTE : This result was expected since, the magnetic field on the axis of dipole is
4 x 3

5 5
Example 1. Two circular coils A and B of radius cm and 5 cm respectively carry current 5 Amp and Amp
2 2
respectively. The plane of B is perpendicular to plane of A and their centres coincide. Find the magnetic field at
the centre.
Sol. The magnetic field due to first coil is
I 4 107 5
B1 0 1
2r1 2 5 102
2
20 3.14 107
B1
5 1.441 102
B1 = 8.88 105 web/m2

www.physicsashok.in 2
MAGNETIC FIELD
7
0 I 2 4 10 5
and B2
2r2 2 5 102 2

2 3.14 105
B2 4.44 105
1.414

B B12 B22

B (8.88) 2 (4.44) 2 10 5 weber / m 2

B 78.85 19.71 9.93 wb / m 2

Example 2. Three rings, each having equal radius R, are placed mutually perpendicular
to each other and each having its centre at the origin of co-ordinate system. If current
I is flowing through each ring then the magnitude of the magnetic field at the common
centre is

(A)
0 I
3
2R
(B) zero (C) 2RI
2 1 0
(D) 3 2 2RI
0

Sol. The magnetic field due to ring in xy plane is


0 I
B1 k
2R
the magnetic field due to ring in yz plane is
0 I
B2 i
2R
and the magnetic field due to ring in xz plane is
0 I
B3 j
2R

B B1 B2 B3
0i
B
2R

k i j

30 I
B
2R
Hence, option (A) is correct
i
(c) Magnetic field due to an arc of a circle at the centre is
i i
B = 0 or B 0
2 2R 4 R
O
R
(iii) Field along the axis of a solenoid is Inwards

2
1
O x

www.physicsashok.in 3
MAGNETIC FIELD
Ni
B 0 cos 2 cos 1
2
(a) For a long solenoid (L >>R),
i.e., 1 = 180 and 2 = 0
B = 0Ni
(b) At the ends of solenoid,
2 = 0, 1 = 90
1
we get, B 0 Ni (for L>>R)
2

Example 3. In a high tension wire electric current runs from east to west. Find the direction of magnetic field at
point above and below the wire.
Sol. When the current flows from east to west, magnetic field lines are circular round it as shown in figure (a). And
so, the magnetic field above the wire is towards north and below the wire towards south.
B
N

W E
i
S
(a) (b)

Example 4. A 0.5 m long solenoid has 500 turns and has a flux density of 2.52 103 T at its centre. Find the
current in the solenoid. Given, 0 = 4 107 Hm1.
Sol. Here, B = 2.52 103 T; 0 = 4 107 Hm1
Length of the solenoid, l = 0.5 m;
Total number of turns in the solenoid, N = 500
Therefore, number of turns per unit length of the solenoid,
N 500
n 1000 m 1
l 0.5
If i is the current through the solenoid, then
B = 0ni
B 2.52 10 3
or i 2.0 A
0 n 4 107 1000

Example 5. A rectangular polygon of n sides is formed by bending a wire of total length 2R which carries a
current i. Find the magnetic field at the centre of the polygon.
Sol. One side of the polygon is,
2R
a
n
2
d
n i

2 n
d a
cot
a / 2
a R
d cot cot
2 n n

www.physicsashok.in 4
MAGNETIC FIELD
All sides of the polygon produce the magnetic field at the centre in same direction (here ). Hence, net
magnetic field,
B = (n) (magnetic field due to one side)
i
B n 0 sin sin
4 d
in
or B n 0 tan 2 sin
4 R n n
2

sin
or 0i n n
B
2R cos / n

Example 6. Infinite number of straight wires each carrying current I


are equally placed as shown in the figure. Adjacent wires have a y
current in opposite direction. Net magntic field at point P is a
30
0 I l n 2 0 I l n 4 P 30 1 2 3 4 5 x
(A) 4 k (B) 4 k z
3a 3a

0 I l n 4
(C) 4 ( k) (D) Zero
3a

Sol. B B1 B2 B3 B4 ......... k
0 I
Here B1 sin 30 sin 30
4 a
0 I 1 1
B1
4a cos 30 2 2
20 I
B1
4 3a
20 I
Similarly, B2 and so on
8 3a
20 I 1 1 1
B 1 ........
4 3a 2 3 4
20 I
B ln 2 k
4 3a
0 I
B ln 4 k
4 3a
Hence, option (B) is correct.

Example 7. A long straight wire, carrying current I, is bent at its midpoint P


R 45 I
to form an angle of 45. Magnetic field at point P, distance R from point
I
of bending is equal to :

(A)

2 1 0 I
(B)

2 1 0 I
(C)

2 1 0 I
(D)

2 1 0I
4R 4R 4 2R 2 2 R
www.physicsashok.in 5
MAGNETIC FIELD
Sol.
0 I
B
R
sin 90 sin135
4 .
2
I
B 0
4 R

2 1 Hence option (A) is correct

Example 8. Find the magnetic field at P due to the arrangement shown 90


0i 1 20i
(A) 1 (B) 45
2 d 2 2 d d

0i 0 i 1 P
(C) (D) 1
2 d 2 d 2

Sol. B B1 B2
0 I
B sin sin 2
d 4 2
4 90
2
0 I 1
B 1
d

45
45
2


2
2 /4
d d
2 0 I 2 1 2 P 2
B
2d 2

0 I
B
2 d

2 1

0i 1
B 1 Hence option (A) is correct
2d 2

Example 9. What is the magnitude of magnetic field at the centre O of loop of radius
O
2 m made of uniform wire when a current of 1 amp enters in the loop and taken
45
out of it by two long wires as shown in the figure. 1 amp 90
1 amp
Sol. B1 = magnetic field due to left wire is
O
I
B1 0 sin sin k /4 d
4d 2 4

B2 = magnetic field due to right wire
I
0 sin sin ( k)
4 d 2 4 I1
In circular wire,
1 I1
I
R I2 2
I2
I1 I
2 2
www.physicsashok.in 6
MAGNETIC FIELD

B3 = magnetic field due to circular wire
I I (2 )
01 0 2
4r 4r
I (2 )
0 2 0 I2 0
4r 4r(2 )

B B1 B2 B3 0

AMPERES CIRCUITAL LAW



It states that the line integral of B around any closed path or circuit is equal to 0 times the total current
crossing the area bounded by the closed path provided the electric field inside the loop remains constant. Thus,

B . dl 0 inet
C
Its simplified form is Bl = 0 inet
This equation can be used only under following conditions :

(a) at every point of the closed path B || dI
(b) magnetic field has the same magnitude B at all places on the closed path.

Applications of Amperes Circulatal Law


(i) Magnetic field due to a long metal rod of radius R carrying a current i :
0i
(a) If r < R, B r , i.e., B r
2R 2
(b) If r = R (i.e., at the surface)
i
B 0
2R
(ii) Magnetic field of a solenoid wounded in the form of a helix is
B = 0Ni

NOTE : Amperes law is valid only for steady currents. Further more, it is useful
only for calculating the magnetic fields of current configurations with high degrees
of symmetry, just as gausss law is useful only for calculating the electric fields
of highly symmetric charge distributions.

Example 10. Two long conductors are arranged as shown above to form d
overlapping cylinders, each of raidus r, whose centers are separated Conductor
A y
by a distance d. Current of density J flows into the plane of the page
along the shaded part of one conductor and an equal current flows out x
of the plane of the page along the shaded portion of the other, as shown.
What are the magnitude and direction of the magnetic field at point A ? Vacuum

(A) (0/2)dJ, in the +y-direction (B) (0/2)d2/r, in the +y-direction


(C) (0/2)4d2J/r, in the y-direction (D) (0/2)Jr2/d, in the y-direction

Sol. B B1 B2
d d
B 0 j i 0 j i
2 2 2 2
www.physicsashok.in 7
MAGNETIC FIELD
0 d d
B jk i 0 jk i
2 2 2 2
0
B jd j 0 jd j
4 4
0 d j
B jd j 0 along y-axis Hence, option (A) is correct.
2 2

MAGNETIC FIELD OF A MOVING POINT CHARGE



The magnetic field vector B at point P at position vector r , from the charge q moving with a velocity v is
found to be
0 q
B
4 r 3
vr
Note down the following points regarding this equation.
(a) Magnitude of B is,
v p
qvsin
B 0 r
4 r 2 q
It is zero at = 0 and 180 and maximum at = 90

(b) Direction of B is along v r if q is

positive and opposite to v r if q is negative.

(c) Suppose a charge q1 is moving with velocity and v1 another charge q2 is moving with velocity v 2 at

position vector r relative to q1, then force on q2 will be,


F q2 v2 B
q

F v 2 0 . 31 v1 r
4 r

qq

F 0 . 1 3 2 v 2 v1 r
4 r

This corresponds to Coulombs electrical force between the charges q1 and q2 moving with velocities v1 and

v 2 respectively relative to an observer at rest.


Example 11. For a charge q moving with velocity v , find the relation between electric and magnetic fields.
q
Sol. E ...(i)
4 0 r 3

0 qv r
B ...(ii)
4 r 3
1
and c
0 0
1
or c2
0 0
www.physicsashok.in 8
MAGNETIC FIELD
From Eqs. (i) and (ii), we get

B 0 0 v E

v E
B 2
c

FORCE ON A CURRENT CARRYING CONDUCTOR IN MAGNETIC FIELD F i


When a current carrying conductor is placed in a magnetic field, the conductor B
experiences a force in a direction perpendicular to both the direction of magnetic
field and the direction of current flowing in the conductor. This force is also called
Lorentz force. i
The direction of this force can be found out either by Flemings left hand rule or by
right hand palm rule.
The magnetic force is
F = ilB sin

In vector form,
Fi l B
Where B = intesity of magnetic field
i = current in the conductor
l = length of the conductor
and = angle between the length of conductor and direction of magnetic field.
Case :
(i) If = 90 or sin = 1 then F = ilB (maximum)
Therefore, force will be maximum when the conductor carrying current is perpendicular to magnetic field.
(ii) If = 0 or sin = 0,
Then F = ilB 0 = 0
Thus, the force will be zero, when the current carrying conductor is parallel to the field.

Example 12. A straight current carrying conductor is placed in such a way that the
P
current in the conductor flows in the direction out of the plane of the paper. The
S R S N
conductor is placed between two poles of two magnets, as shown. The Q
conductor will experience a force in the direction towards.
(A) P (B) Q (C) R (D) S
Sol. The direction of magnetic field on the conductor is along SR.

But F il B

F i l k (B i)

F i l B j i l B( j)
Hence, the direction of magnetic force on the wire is towards Q. Hence option (B) is correct

Example 13. In the figure shown a semicircular wire loop is placed in uniform magnetic field B = 1.0 T. The plane
of the loop is perpendicular to the magnetic field. Current i = 2A flows in the loop in the direction shown. Find
the magnitude of the magnetic force in both the cases (a) and (b). The radius of the loop is 1.0m.

i = 2A i = 2A

1m


(a) (b)

www.physicsashok.in 9
MAGNETIC FIELD

Sol. Refer figure (a) : It forms a closed loop and the current completes the loop. Therefore, net force on the loop
in uniform field should be zero.
Refer figure (b) : In this case although it forms a closed loop, but current does not complete the loop. Hence,
net force is not zero. C B

FACD FAD

Floop FACD FAD 2 FAD
A D
| Floop | 2| FAD |

| Floop | 2ilB sin (l = 2r = 2.0 m)
= (2) (2) (2) (1) sin 90 = 8 N

Example 14. An arc of a circular loop of radius R is kept in the horizontal plane B
I
and a constant magnetic field B is applied in the vertical direction as shown

in the figure. If the arc carries current I then find the force in the arc. 90
Sol. As we know, magnetic force on a closed loop placed in uniform magnetic field is zero.

B1 B2 0
(1)


B1 B2 IlB( j)
R R
B1 Il B j /2

B1 I B( 2R) j (2)


B1 2 IBR
y(m)
Example 15. A conducting wire bent in the form of a parabola y2 = 2x carries a current A

i = 2 A as shown in figure. this wire is placed in a uniform magnetic field B 4 k 2 x(m)
Tesla. The magnetic force on the wire is (in newton) B
(A) 16 i (B) 32 i (C) 32 i (D) 16 i
Sol. The net magnetic force on closed loop is zero.
Force on parabola + force on straight wire AB = 0 A
Force on the parabola = force on straight wire AB


I j B
B

2 4 j 4 k
F 32 i Hence option (C) is correct.

Example 16. A conductor of length l and mass m is placed along the east-west line on a table. Suddenly a
certain amount of charge is passed through it and it is found to jump to a height h. The earths magnetic
induction is B. The charge passed through the conductor is : (B is horizontal)
1 2gh gh m 2gh
(A) (B) (C) (D)
Bmgh Blh Blh Bl
Sol. The magnetic force is
F = I lB
dq
or F lB
dt
www.physicsashok.in 10
MAGNETIC FIELD

Fdt
or dq
lB
Fdt mv 0
or q
lB lB
But 02 = v02 2gh
v 0 2gh

m 2gh
q Hence option (D) is correct
lB

Example 17. A U-shaped wire of mass m and length l is immersed with


B
its two ends in mercury (see figure). The wire is in a homogeneous field
of magnetic induction B. If a charge, that is, a current pulse q idt ,
m
l
is sent through the wire, the wire will jump up. i
Hg
Calculate, from the height h that the wire reaches, the size of the
charge or current pulse, assuming that the time of the current pulse is
very small in comparision with the time of flight. Make use of the fact that impulse of force equals Fdt , which
equals mv. Evaluate q for B = 0.1 Wb/m2, m = 10 gm, l = 20 cm & h = 3 meters. [g = 10 m/s2]
Sol. IlB=F
Fdt = mv
or I l Bdt = mv
1
But mv 2 mgh
2
v 2gh
mv m
Idt 2gh
lB lB

m 10 103
q 2gh 2 10 3 15 coulmb.
lB 20 102 0.1

Example 18. A metal ring of radius r = 0.5 m with its plane normal to a uniform magnetic field B of induction 0.2 T
carries a current I = 100 A. The tension in newtons developed in the ring is :
(A) 100 (B) 50 (C) 25 (D) 10
F
Sol. We consider a small portion l of the loop.
For equilibrium, Tcos /2 /2 /2 Tcos /2
sin
F 2T 2T T
2 2
or IlB = T
or IrB = T T
T
T = IrB = 100 0.5 0.2 = 10 N
+
Hence option (D) is correct B
www.physicsashok.in 11
MAGNETIC FIELD
FORCE ON A MOVING CHARGE IN MAGNETIC FIELD : LORENTZ FORCE
If a charged particle moves in a magnetic field, then a force acting on

it is given by
F = qvB sin
F
In vector form,
Fq vB
v
Cases :
(i) If v = 0, then F = 0 i.e., no force is exerted on a stationary charge, in a magnetic field.
(ii) If = 0, then F = 0 i.e., when the charge is moving parallel to the field then no force will be exerted by the field.
(iii) If = 90, then
sin = sin 90 = 1
F = qvB 1 = qvB
i.e., when the charged particle is moving parpendicular to the field, the force exerted by the field will be
maximum.

Rules of Find the Direction of Force


(i) Right hand palm rule : If we stretch the right hand palm such that the fingers and the thumb are mutually
perpendicular to each other and the fingers point the direction of magnetic field and the thumb points the
direction of motion of positive charge, the direction of force will be along the outward normal on the palm.
Force F
Field B
Current or
motion of
positive charge

(ii) Flemings left hand rule : If we spread the forefinger, central finger and thumb of our left hand in such a way
that these three are perpendicular to each other then, if first forefinger is in the direction of magnetic field,
second central finger is in the direction of current, then thumb will represent the direction of force.
Force F

Field B
Current or motion
of positive charge

NOTE : To learn this rule, remember the sequence of Father, Mother, child.
Thumb Father Force
Forefinger Mother Magnetic field
Central finger Child Current or direction of positive charge



Example 19. When a proton has a velocity v 2 i 3 j 10 m / s it experiences a force F 1.28 10 k N.
6 13

When its velocity is along the z-axis, it experiences a force along the x-axis. What is the magnetic field ?
Sol. Substituting proper values in,

Fq vB
We have,
1.28 1013 k 1.6 1019 2 i 3 j B0 j 106

www.physicsashok.in 12
MAGNETIC FIELD
1.28 = 1.6 2 B0
1.28
or B0 0.4
3.2


Therefore, the magnetic field is, B 0.4 j T

MOTION OF CHARGED PARTICLE IN MAGNETIC FIELD



The path of charged particle in uniform magnetic field depends on angle between v and B . Therefore, following
cases are possible :
Case-I . When is 0 or 180 : The magnetic force is F = Bqv sin 0 or sin 180 = 0.
Hence, path of the charged particle is a straight line (undeviated) when it enters parallel or antiparallel to
magnetic field.
Case-II. When = 90 : The magnetic force is F = Bqv sin 90 = Bqv. This magnetic force is perpendicular to the
velocity at every instant. Hence, path is circle. The necessary centripetal force is provided by the magnetic
force hence, if r be the radius of the circle, then
B B

or
q+ + q
v v

Fm = 0
mv 2 mv
qBv r
r qB
This expression of r can be written in following different ways :
mv p 2Km 2qVm
r
qB qB qB qB
Here, P = momentum of particle
p2
K = KE of particle or p 2K m
2m
Further, time period of the circular path will be
mv
2
2r qB 2m
T
v v qB
2m
or T
qB
2 qB
or the angular speed () of the particle is
T m
qB

m
Frequency of rotation is,
1 qB
f or f
T 2m

www.physicsashok.in 13
MAGNETIC FIELD
IMPORTANT FEATURES
If angle is other than 0, 180 or 90, then velocity of charged particle can be resolved in two components

one along B and another perpendicular to B . Let the two
B
v sin
components be v| | and v . Then v
v| | v cos v cos
q, m +
and v v sin
The component perpendicular to field ( v ) gives a circular path and the component parallel to field ( v| | ) gives
a straight line path. The resultant path is a helix as shown in figure.
The radius of this helical path is,

mv mvsin
r
qB qB
Time period and frequency do not depend on velocity and so they are given by
2m qB
T and f
qB 2m
There is one more term associated with a helical path, that is pitch (p) of the helical path. Pitch is defined as the
distance travelled along magnetic field in one complete cycle, i.e.,
p = v| |T
2m
or p v cos
qB
2mv cos
p
qB

Example 20. What is the smallest value of B that can be set up at the equator to permit a proton of speed
107 m/s to circulate around the earth ?
[R = 6.4 106 m, mp = 1.67 1027 kg].
Sol. From the relation
mv
R
qB
mv
We have B
qR
Substituting the values, we have

B
1.67 10 10 1.6 10
27 7
8
T
1.6 10 6.4 10
19 6

q
Example 21. A block of mass m & charge q is released on a long smooth inclined B m
plane magnetic field B is constant, uniform, horizontal and parallel to surface as
shown. Find the time from start when block loses contact with the surface.
m cos m cosec m cot
(A) (B) (C) (D) none
qB qB qB
www.physicsashok.in 14
MAGNETIC FIELD
Sol. For losing the contects
N=0
qvB = mgcos
mg cos
v
qB
But v = u + at
or v = 0 + gt sin
mg cos
gt sin
qB
m cot
t Hence option (C) is correct.
qB
Example 22. An electron having kinetic energy T is moving in a circular orbit of radius R perpendicular to a uniform

magnetic induction B . If kinetic energy is doubled and magnetic induction tripled, the radius will become

3R 3 2 4
(A) (B)R (C) R (D) R
2 2 9 3
mv P
Sol. R
qB qB
P2
But kinetic energy T P 2mT
2m
2mT
R
qB
2m(2T)
R
q(3B)
2 2
R R R Hence, option (C) is correct.
3 9

Example 23. A charged particle (charge q, mass m) has velocity v0 at origin in +x direction. In space there is a
uniform magnetic field B in z direction. Find the y coordinate of particle when is crosses yaxis.
Sol. The parth of charged particle is circular whose radius is
+
mv0 B
r
qB C
2mv0
y 2r O
qB

Example 24. A mass spectrometer is a device which select particle of equal


S
mass. An iron with electric charge q > 0 starts at rest from a source S B
and is accelerated through a potential difference V. It passes through a
V
hole into a region of constant magnetic field B perpendicular to the
plane of the paper as shown in the figure. The particle is deflected by
the magnetic field and emerges through the bottom hole at a distance
d from the top hole. The mass of the particle is
qBd qB2 d 2 qB2 d 2 qBd
(A) (B) (C) (D)
V 4V 8V 2V

www.physicsashok.in 15
MAGNETIC FIELD
Sol. The speed of charged particle just before entering the magnetic field is V0.
1
qV mv 20
2
2qV
v0
m
The radius of circular path in magnetic field is
mv0
r
qB
d mv0
or
2 qB

d 2 m 2 v02 m2 2qV
or
4 q 2 B2 q 2 B2 m

d 2 2mV
or
4 qB2

qB2d 2
m Hence, option (C) is correct
8V

Example 25. A cyclotron is operating with a flux density of 3 Wb/m2. The ion which enters the field is a proton
having mass 1.67 1027 kg. If the maximum radius of the orbit of the particle is 0.5m, find :
(a) the maximum velocity of the proton,
(b) the kinetic energy of the particle, and
(c) the period for a half cycle.
Sol. (a) As in case of motion of a charged paricle in a magnetic field,
mv qBr
r i.e., v
qB m
qBrmax
So, v max
m
1.6 1019 3 0.5
So, v max 27
1.43 108 m / s
1.67 10
1 1 2
(b)
2 2

K mv 2 1.67 1027 1.43 108
1.71 1011
i.e. K 1.71 1011 J 107 MeV
1.6 1019
(c) In case of circular motion, as
2r 2 0.5
T 2.19 108 s
v 1.43 108
1
So, time for helf cycle, T (T) 1.09 108 s
2

www.physicsashok.in 16
MAGNETIC FIELD

Example 26. The region between x = 0 and x = L is filled with uniform steady magnetic field B0 k . A particle of

mass m, positive charge q and velocity v 0 i travels along x-axis and enters the region of the magnetic field.
Neglect the gravity throughout the question
(a) Find the value of L if the particle emerges from the region of magnetic field with its final velocity at an angle 30
to its initial velocity.
(b) Find the final velocity of the particle and the time spent by it in the magnetic field, if the magnetic field now
expands upto 2.1 L.
Sol. (A) = 30,
L x=0 x=L
sin X
R + + + +
B = B0 k
mv 0 C + + +
Here R R
qB0 + + + + v0
L P
sin 30 + +L + +
mv0

qB0 A + + + +
q v0
1 qB0L + + + +
or
2 mv0 Y
mv0
L
2qB0
(b) In part (i) R

L v0
sin 30
R B
1 L v0
or or L = R/2
2 R
2.1 2.1
L = R > R
Now when L = 2.1 L or R 2
2
L > R
Therefore, deviation of the particle is = 180 is as shown.

v f v i v B
0

T m
and t AB
2 qB0
M
Example 27. A wire loop carrying a current I is placed in the xy plane
a y
as shown in fig. V
I
120

(a) If a particle with charge +Q and mass m is placed at the centre +Q


P O x
P and given a velocity v along NP (see figure), find its instan-
taneous acceleration.
N
(b) If an external unfiorm magnetic induction field B B i is applied
find the force and the torque acting on the loop due to this field.
Sol. The magnetic field at the centre P due to current in wire NM is
I
B1 0 sin 60 sin 60
4r
www.physicsashok.in 17
MAGNETIC FIELD
y
I 3 3 x
B1 0 M
4 a / 2 2 2 a V sin60
30
V
v

120
0 2I 3 Q Q 60
V cos60
B1 30 P
4 a
directed away from the reader perpendicular to the N
plane of paper.
r
sin 30
a M
a
r 30 a
2 S P
MS r
cos 30
a a

3a N
MS
2
MN 3 a
The magnetic field at the centre P due to current in arc MN is
0 2I 0 2I 2 / 3 0 2I
B2
2 a 2 4 a 2 4 3a
directed towards the reader perpendicular to the plane of paper
The net magnetic field
0 2 3I 0 2I
B B1 B2
4 a 4 3a
0 2I 2I
B 3 0 (.68)
4a 3 4 a
(directed away from the reader perpendicular to the plane of paper)
The force acting on the charged particle Q when it has a velocity v and is instantaneously at the centre is
F = QvB sin = QvB sin 90 = QvB
The acceleration produced
F QvB Qv 0 2I
A (0.68)
M m m 4 a
0.110 IQv
A
ma
y
M
x
R
Q v

B
Q 120 P

N

The direction of acceleration is given by the vecotr product v B or by applying Flemings left hand rule

www.physicsashok.in 18
MAGNETIC FIELD
RPN = 90 and MPN = 120
MPR = 120 90 = 30
Since, MPQ = 60 RPQ = 30
i.e., wthe acceleration vector makes an angle of 30 with the negative x-axis.
The torque acting on the loop in the magnetic field is giving.

MB
where M = IA where
A = (area of PMQNP) (area of trangle PMN)
1 1
A
3

a 2 MN PS
2

a 2 1 a 3
A 3a a 2
3 2 2 3 4
3
A a2 k
3 4
3
Ia 2 kiB
3 4
3
BIa 2 j 0.614 BIa J
2

3 4
The force acting on the loop is zero.

Example 28. An electron gun G emits electrons of energy 2 keV travelling in the S
B
positive X-direction. The electrons are requeired to hit the spot S where
GS = 0.1 m, and the line GS make an angle of 60 with the x-axis as shown G 60
X
in figure. A uniform magnetic field B parallel to GS exists in the region outside
the electron gun. Find the minimum value of B needed to make the electrons
hit S.
Sol. Kinetic energy of electron,
1
K mv 2 2keV
2
speed of electron, S
B
2K
v G 60
v
m
2 2 1.6 1016
v m/s
9.1 10 31
v = 2.65 107 m/s

Since the velocity (v) of the electron makes an angle of = 60 with the magnetic field B , the path will be a
helix. So, the particle will hit S if
GS = nP Here n = 1, 2, 3, .............
2 m
p pitch of helix v cos
qB

www.physicsashok.in 19
MAGNETIC FIELD
But for B to be minimum, n = 1
2m
Hence, GS p v cos
qB
2mv cos
B Bmin
q(GS)
Substituting the values, we have
1
(2)(9.1 1031 )(2.65 107 )
2
Bmin 19
tesla
(1.6 10 ) (0.1)
or Bmin = 4.73 103 tesla


Example 29. An electron moving with a velocity V1 2 i m/s at a point in a magnetic field experiences a force

F1 2jN . If the electron is moving with a velocity V 2 2j m/s at the same point, it experiences a force

F2 2 i N . The force the electron would experience if it were moving with a velocity V 3 2 k m/s at the
same point is
(A) zero (B) 2kN (C) 2kN (D) informationis insufficient

Sol. F1 qv1 B

2 j e(v1 ) B

2 j e(2 i) B

j 2e( i B)
j eB( i) ( k)

eB = 1

F2 e(2 j) ( B k)

2 i 2eB i
eB = 1

F3 e(v3 B)

F3 e(2 k) ( B k)
0 Hence option (A) is correct


Charged particle in uniform E & B

When a charged particle moves with velocity v in an electric field E and magnetic field B , then. Net force
experienced by it is given by following equation.

F qE q(v B)

Combined force F is known as lorentz force

(i) E || B || v E B v
In above situation particle passes underviated but its velocity will change due to electric field and magnetic
force on it is zero.

www.physicsashok.in 20
MAGNETIC FIELD

(ii) E || B and uniform, particle is released with velocity v0 at an angle .
y
v0 v0
v0 sin

+q
+q E, B x E, B
z v0 cos

mv 0 sin 2 m
r ; T x
qB qB
y v0 sin
1 qE 2
r v 0 cos t
2 m

t i R sin t j R R cos t k
R z
c
Cycloid motion

Suppose that B points in the x-direction, and E in the z-direction.
z
E

0
a b c y
x B

Byz
F q E v B q Ez Bzy ma m
yy zz

qB

m
E
z y
y z,
B
Their general solution is
y(t) C1 cos t C2 sin t (E / B)t C3

z(t) C 2 cos t C1 sin t C4
E E
y(t) (t sin t), z(t) (1 cos t)
B B
E
R
B
(y Rt)2 + (z R)2 = R2
E
v R
B
The particle moves as through it were a spot on the rim of a wheel, rolling down the y axis at speed, v. The

curve generated in this way is called a cycloid. Notice that the overall motion is not in the direction of E , but
perpendicular to it.

www.physicsashok.in 21
MAGNETIC FIELD
Example 30. A particle of specific charge (charge/mass) starts moving from the origin under the action of an

electric field E E i and magnetic field B B k . Its velocity at (x , y , 0) is (4 i 3 j) . The value of x is :
0 0 0 0 0

13 E0 16B0 25 5
(A) 2 B (B) (C) 2E (D) 2B
0 E0 0 0

Sol. The workdone by magnetic force is WB = 0, because, magnetic force is always perpendicular to instantaneous
displacement.

W qE . S qE i . (x i y j) qE x
E 0 0 0 0 0

The speed of particle at (x0, y0) is


v 42 ( 3) 2 5m / s
According to work energy theorm,
1 1
W T mv 2 mu 2
2 2
1
or WE WB m 52 0
2
25
or qE 0 x 0 m
2
25m 25 q
x0
2qE 0 2E 0 m
Hence, option (C) is correct.

Example 31. A particle of specific charge (q/m) is projected from the origin of coordinates with initial velocity
[u i v j] . Uniform electric magnetic fields exist in the region along the +y direction, of magnitude E and B. The
particle will definitely return to the origin once if
(A) [vB/2E] is an integer (B) (u2 + v21/2 [B/E] is an integer
(C) [vB/E] in an integer (D) [uB/E] is an integer
qE
Sol. ay
m
mu 2
quB
r
mu
r
qB
2m
also, T
qB
qE 2
y vt t
2m
For origin, x = 0, and y = 0
qE 2
0 vt t
2m

www.physicsashok.in 22
MAGNETIC FIELD

2mV
t
qE
For returning at the origin,
2mV
nT
qE
2nm 2mV

qB qE
2mVqB
n
qE(2m)
VB
n Hence, option (C) is correct
E

FORCE BETWEEN PARALLEL CURRENT CARRYING WIRES


Consider two long wires 1 and 2 kept parallel to each other at a distance
1 2
r and carrying currents i1 and i2 respectively in the same direction.
i1 i2
The force per unit length of the wire 2 due to wire 1 is :
F l
F 0 i1i 2

l 2 r
The same force acts on wire 1 due to wire 2. r

NOTE : The wires attract each other if currents in the wires are flowing in the same
direction and they repel each other if the currents are in opposite directions.

Example 32. A conductor of length 2 m carrying current of 2 A is held parallel to an infinitely long conductor
carrying current of 10 A at a distance of 100 mm. Find the force on small conductor.
Sol. We know that force per unit length of short conductor due to long conductor is given by
2i i
f 0 12
4 r
Total force on length l of the short conductor is
2i i
F fl 0 1 2 l
4 r
7
10 2 2 10 2
F 8 105 N
0.1
Force is attractive if the direction of current is same in both the parallel conductors and is repulsive if the
direction of current is opposite in two parallel conductors.
Y
Example 33. A straight segment OC (of length L) of a circuit carrying a
current i is placed along the X-axis as shown in figure. Two infinitely B

long straight wires A and B, each extending from z = to + , are O i C


X
fixed at y = a and y = + a respectively. as shown in the figure. If the
wires A and B each carry a current i into the plane of the paper, A
obtain the expression for the force acting on the segment OC. What Z
will be force of OC if the current in the wire B is reversed ?

www.physicsashok.in 23
MAGNETIC FIELD
Sol. (a) Let us assume a segment of length dx at a point P, a distance x from the centre shown in figure.
y is the positive Z-direction and
IB is the negative Z-direction and
dx
a
O i
x x
C
a
BB BA
I Bnet
A L
y
Magnetic field at P due to current in wires A and B will be in the directions perpendicular to AP and BP
respectively as shown.
i
| BA | | BB | | B | 0
2 AP
Therefore, net magnetic field at P will be along negative y = axis as shown
i x
and Bnet 2 | B | cos 2 0
2 AP AP

i.x ix
Bnet 0 2
0. 2
(AP) (a x 2 )
Therefore, force on the element will be (F = ilB)
ix
dF i 0 2 2
dx (in negative z-direction)
a x
Total force on the wire will be
xL L
i 2 xdx
F dF 0 2
x0
0 x a2
Y
0 i 2 L2 a 2 B
F ln (in negative z-axis) BB
2 a 2
0i 2 L2 a 2 i P
X
Hence F ln k Bnet C
2 a 2
(b) If current in wire B is reversed, then magnetic fields due to A BA
A and B will be in the directions shown in figure. i.e., net

magnetic field Bnet will be along positive x-axis and since
current is also along positive x-axis, force on wire OC will be zero.

Note : BA is not necessarily parallel to BP..

M
CURRENT LOOP IN UNIFORM MAGNETIC FIELD B

MB
I
| | MBsin ; where M = NIA
Work done in rotating loop in uniform field from 1 to 2
W = MB (cos 1 cos 2)

www.physicsashok.in 24
MAGNETIC FIELD

Example 34. The magnetic moment of a circular orbit of radius r carrying a charge q and rotating with velocity
v is given by
qvr qvr
(A) (B) (C) qvr (D) qvr2
2 2
Sol. The convection current is
q qv
I
2 2r
The magnetic moment is
qv 2
M Ir 2 r
2r
qvr
M Hence option (B) is correct
2

Example 35. Q charge is uniformly distributed over the same surface of a right circular cone
of semi-vertical angle and height h. The cone is uniformly rotated about its axis at
angular velocity . Calculated associated magnetic dipole moment.
Sol. I = The moment of gnertia
3
MR 2
10
3
Angular momentum L I mR 2
10
QL
But magnetic moment P
2m
Q 3
P mR 2
2m 10
3Q 2
P R
20
R
But tan
h
R = h tan
3QR 2 3Qh 2 tan 2
P
20 20

Example 36. Figure shown a square current carrying loop ABCD of side 10 cm and y

current i = 10 A. The magnetic moment M of the loop is B


(A) (0.05) i 3 k A m
2

(B) (0.05) j k A m
2

A
C
i = 10
(C) (0.05) 3 i k A m (D) i k A m 30 x
2 2

D
Sol. The magnitude of magnetic moment is z
M = Il2 = 10 (10 102)2 Am2
= 10 102 = 0.1 Am2
www.physicsashok.in 25
MAGNETIC FIELD

The normal on the loop is in x z plane. It makes 60 angle with x - axis.



M M cos 60 i M sin 60 j
M 3
M i Mj
2 2
0.1
M
2

i 3 j


M (0.05) i 3 j Am 2
2n, 2a, 2I
Example 37. A rectangular coil PQ has 2n turns, an area 2a and carries a current 2I,
(refer figure). The plane of the coil is at 60 to a horizontal uniform magnetic field
of flux density B. The torque on the coil due to magnetic force is coil 60
(A) BnaI sin 60 (B) 8 BnaI cos 60 B
(C) 4 BnaI sin 60 (D) none

Sol. MB
Here M = 2n(2I) (2a)
M = 8 nIa
= MB sin(90 60)
= MB cos 60
= 8 nIa cos 60 Hence option (B) is correct

Example 38. (a)A rigid circular loop of radius r & mass m lies in the xy plane on a flat table and has a current

I flowing in it. At this particular place, the earths magnetic field is B B x i B y j. How large must I be
before one edge of the loop will lift from table ?

(b) Repeat if, B B x i B y k.

Sol. (a) Torque due to magnetic force should be greater than torque due to weight.
Ir 2 B mgr

or Ir 2 B2x B2y mgr



(b) Since, Bz is parallel to dipolemoment
I r 2 B x mgr
mg
I
rBx

Example 39. A conducting ring of mass 2 kg and radius 0.5 m is placed on a smooth
horizontal plane. The ring carries a current i = 4 A. A horizontal magnetic field
B
B = 10T is switched on at time t = 0 as shown in figure. The initial angular
acceleration of the ring will be
(A) 40 rad/s2 (B) 20 rad/s2 (C) 5 rad/s2 (D) 15 rad/s2
Sol. As we know, if a coil or a closed of any shape is placed in uniform electric field, magnetic force on the coil or
the loop is zero.

www.physicsashok.in 26
MAGNETIC FIELD

So, the centre of mass of the coil remains in rest. the torque on the coil is
= IAB = 4r2B
mr 2
or 4r 2 B
2
8B 8 10
40rad / s 2 Hence option (A) is correct
m 2

Example 40. In the figure shown a coil of single turn is wound on a sphere of radius R
B
and mass m. The plane of the coil is parallel to the plane and lies in the equatorial
plane of the sphere. Current in the coil is I. The value of B if the sphere is in
equilibrium is
mg cos mg mg tan mg sin
(A) (B) (C) (D)
IR IR IR IR
Sol. For equilibrium, P
mg sin = f
Also, net torque should be zero. 180
The magnetic moment of the loop is perpendicualr to the plane.
The torque due to magnetic force is B = PB sin(180 ) B
B = PB sin = IR2B sin
This torque balances the torque due to friction about centre of mass.
FR = IR2Bsin
or mg sin = IRB sin
mg
B Hence option (B) is correct
IR
O+ B
Example 41. A square current carrying loop made of thin wire and having a mass m = 10 g
can rotate without friction with respect to the vertical axis OO1, passing through the centre of I
the loop at right angles to two opposite sides of the loop. The loop is placed in a homo-
geneous magnetic field with an induction B = 101 T directed at right angles to the plane
of the drawing. A current I = 2A is flowing in the loop. Find the period of small oscillations
that the loop performs about its position of stable equilibrium. O1
Sol. = PB sin
or = IAB
or I0 = IAB
IAB
or
I0

IAB
2
I0

IAB
or
I0

www.physicsashok.in 27
MAGNETIC FIELD

2 IAB
or
T I0

I0
T 2
IAB
2 2
l l m l2 m l2
Here I0 m 0 m 0 0 0
2 2 12 12

m 0 l 2 m 0l 2
I0
2 6
m
But m0 2.5 103 kg
4
l2 = A = area of loop, I = 2 amp
After putting the value,
T = 0.57 sec.


Example 42. A uniform constant magnetic field B is directed at an angle of 45 to the X-axis in X Y plane. PQRS
is a rigid square wire frame carrying a steady current I0, with its centre at the origin O. At time t = 0, the frame
is at rest in the position shown in the figure with its sides parallel to X and Y axes. Each side of the frame is of
mass M and length L. Y

(a) What is the torque about O acting on the frame due to the
I0
magnetic field ? S R
(b) Find the angle by which the frame rotates under the action of
this torque in a short interval of time t, and the axis about X
which this rotation occurs (t is so short that any variation in
the torque during this interval may be neglected). Given : the P Q
moment of inertia of the frame about an axis through its centre
4
perpendicular to its plane is ML2 .
3
Sol. Magnetic moment of the loop,

M (iA)k (I0 L2 )k

B (i j)
Magnetic Field, B (B cos 45 )i (Bsin 45 ) j
2
(a) Torque acting on the loop,

M B (I0 L2 k) B (i j)
2

I0 L2 B
(j i) or | | I0 L2B
2
(b) Axis of rotation coincides with the torque and since torque is in j i direction or parallel to QS. Therefore,
the loop will rotate about an axis passing through Q and S as shown along-side :

||
Angular acceleration,
I

www.physicsashok.in 28
MAGNETIC FIELD
Y
Where I = moment of inertia of loop about QS.
IQS + IPR = IZZ
S R
(From theorem of perpendicular axis)
But IQS = IPR X
4 2
2IQS IZZ ML2 IQS ML2
3 3 P Q
2
| | I0 L B 3 I0 b

I 2 2 2 M
ML
3
Angle by which the frame rotates in time t is
1 3IB
. (t)2 or 0 . (t) 2
2 4 M

MOVING COIL OR SUSPENDED COIL OR D ARSONVAL TYPE GALVANOMETER


Principle : When a current-carrying coil is placed in magnetic field, it experiences a torque.
Construction : It consists of a narrow rectangular coil PQRS consisting of a large number of turns of fine insulated
copper wire wound over a frame made of light, non-magnetic metal. A soft iron cylinder known as the core is
placed symmetrically within the coil and detached from it. The coil is suspended between the two cylindrical
pole-pieces (N and S of a strong permanent horse-shoe magnet) by a thin flat phosphor bronze strip, the upper
end of which is connected to a movable torsion head T. The lower end of the coil is connected to a hair-spring
s of phosphor bronze having only a few turns.
T

T1 T2
m

P S

N Core S

Q R
s

Moving coil galvanometer


In order to eliminate air-disturbance, the whole arrangement is enclosed in a brass case having a glass window
on the front. Levelling screws are provided at the base. The torsion head T is connected to a binding terminal
T1. So, the phosphor-bronze strip acts as one current lead to the coil. The lower end of the spring s is
connected to a binding terminal T2. A plane mirror or a concave mirror of larger radius of curvature is rigidly
attached to the phosphor bronze strip. This helps to measure the deflection of the coil by lamp and scale
arrangement.

Radial magnetic field : The magnetic field in the small air gap between the
cylinderical pole-pieces is radial. The magnetic lines of force within the N S
air gap are along the radii. On account of this, the plane of the coil remains
always parallel to the direction of the magnetic field i.e., the angle between Radial magnetic field
the plane of the coil and the magnetic field is zero in all the orientations of the coil.

www.physicsashok.in 29
MAGNETIC FIELD

Theory :
Let I = current flowing through the coil; B = magnetic field induction
l = length of the coil; b = breadth of the coil
N = number of turns in the coil; A(= l b) = area of the coil
Since the field is radial, therefore, the plane of the coil remains parallel to the magnetic field in all the orientations
of the coil. So, the sides SP and QR remain parallel to the direction of the magnetic field. So, they do not
experience any force. The sides PQ and RS remain perpendicualr to the direction of the magnetic field. These
sides experience forces perpendicular to the plane of the coil.
b
S
P
F
l

F
Q R

Current-carrying loop in magnetic field


Force on PQ, F = NBIl
Applying Flemings left hand rule, we find that this force is normal to the plane of the coil and directed outwards,
i.e., towards the reader.
Force on RS, F = NBIl
Applying Flemings left hand rule, we find that this force is normal to the plane of the coil and directed inwards,
i.e., away from the reader.
The forces on the sides PQ and RS are : (i) equal in magnitude; (ii) opposite in direction; and (iii) act at different
points. So, the two forces constitute a couple. This couple tends to deflect the coil and is known as deflecting
couple.
Moment of deflecting couple = NBIl b = NBIA
[The field is radial. The forces on the sides PQ and RS always remain perpendicular to the plane of the coil. So,
the perpendicular distance between the forces is always equal to b as in fig.]
F

P S
b
F
When the coil deflects, the suspension fibre gets twisted. On account of elasticity, a restoring couple is set up
in the fibre. This couple is proportional to the twist. If be the angular twist, then
Moment of restoring couple = k
where k is the restoring couple per unit angular twist. It is also known as torsional constant.
For equilibrium of the coil,
k
NBIA = k or I or I K
NBA
k
where K is the galvanometer constant.
NBA

Now, I or I
So, the deflection of the coil is proportional to the current flowing through the coil. This explains as to why we
can use a linear scale in a galvanometer. The scale is calibrated to give direct values of current.

www.physicsashok.in 30
MAGNETIC FIELD

CURRENT SENSITIVITY OF A GALVANOMETER


A galvanometer is said to be sensitive if it gives a large deflection for a small current.

The current sensitivity of a meter is the deflection of the meter per unit current, i.e., . It is given by
I
NBA
[ NBIA = k]
I k
The sensitiveness can be increased by increasing N, A and B and decreasing the value of k. But N and A
cannot be increased much because this will increase the length and consequently the resistance of the coil. In
that case, the galvanometer will not respond to weak electric currents.
B can be increased by using a strong magnet. k can be decreased by using phosphor-bronze for suspension.
k can be further reduced by using quartz suspension fibre.

VOLTAGE SENSITIVITY OF A GALVANOMETER



It is defined as the deflection of the meter per unit voltage, i.e., .
V

NBA
Now, or
V RI V kR
Advantages :
(i) The galvanometer can be made extremely sensitive.
(ii) Since the magnetic field B is very high, therefore, the external magnetic fields cannot appreciably after the
deflection of the coil. So, the galvanometer can be used in any position.
(iii) Since the deflection of the coil is proportional to current, therefore, linear scale can be used.
(iv) Since the coil is wound over metallic frame, therefore, damping is produced by eddy currents. So, the
galvanometer coil comes to rest quickly. This type of galvamometer is called aperiodic or dead beat galvanometer.
The galvanometer can be made ballistic by winding the coil on a non-conducting frame of ivory or ebonite.
(v) The lamp and scale arrangement used to measure the deflection of the coil makes the galvanometer very
sensitive.

POINTER TYPE OR WESTON OR PIVOTED MOVING COIL GALVANOMETER


The suspended type moving coil galvanometers are very sensitive. They can measure currents of the order of
109 ampere. But these require very careful handling. So, for general use in the laboratory and for those
experiments whose sensitivity is not required, pointer type galvanometers are used.

10 0 10
20 20
30 30

N S

T1 T2

www.physicsashok.in 31
MAGNETIC FIELD
In this type of galvanometer, the coil is pivoted between two ball-bearings. A light aluminium pointer is attached
to the moving coil. The controlling couple is provided with the help of a spring.

NOTE : Numerical Examples based on Moving Coil Galvanometer


Formulae used : 1. = NBIA 2. k = NBIA
NBA
3. Current sensitivity, =
I k
NBA
4. Voltage sensitivity, =
V kR
Units used : B in tesla, A in m , R in ohm, k in N m rad1.
2

EARTHS MAGNETISM
A freely suspended magnet always points in the north-south direction even in the absence of any other magnet.
This suggests that the earth itself behaves as a magnet which causes a freely suspended magnet (or magnetic
needle) to point always in a particular direction : north and south. The shape of earths magnetic field resembles
that of a bar magnet of length one-fifth of earths diameter buried at its centre.
Geographic axis
Magnetic Geographic N-pole
S-pole
Eq
ua
to r
S

M
eq agn N
ua et i
to c
r Magnetic
N-pole
Geographic
S-pole Magnetic
axis
The south pole of earths magnet is towards earths north pole (eographical north), while the north pole of
earths magnet is towards earths south pole (geographical south). Thus, there is a magnetic S-pole near the
geographical north, and a magnetic N-pole near the geographical south. The positions of the earths magnetic
poles are not well defined on the globe, they are spread over an area.
Magnetic equator : The great circle whose place is perpendicular to the earths magnetic axis is called earths
magnetic equator.
Geographical equator : The great circle whose plane is perpendicular to geographical axis is called geographical
equator.
Magnetic meridian : The line joining the earths magnetic poles is called the magnetic axis and a vertical plane
passing through it is called the magnetic meridian.
Geographical meridian : The line joining the geographical north and south poles is called the geographic axis and
a vertical plane passing through it is called the geographical meridian.

Magnetic Elements
To have a complet knowledge of earths magnetism at a place, the following three elements must be known :
(i) Angle of declination
(ii) Angle of dip or inclination
(iii) Horizontal component of earths field.
(i) Angle of declination : The angle between the magnetic meridian and geographical meridian at a place is
called the angle of declination (or simply the declination) at that place.

www.physicsashok.in 32
MAGNETIC FIELD

Geographic Geographical
north B
meridian
Magnetic
north B H
Magnetic
meridian
C
C D

In fig. ABCD is the magnetic meridian and ABCD is the geographical meridian. The angle BAB = is the
angle of declination.

(ii) Angle of dip or inclination : The angle which the axis of needle makes with the horizontal, is called angle of
dip (). In other words, the angle of dip at a place is the angle which the resultant magnetic field of earth at that
place makes with the horizontal.

B H
A
C
i V

N S D

In fig. AC shows the direction of resultant magnetic field of earth and the angle BAC (=) between it and the
horizontal AB is the angle of dip.

(iii) Horizontal component of earths field : The direction of earths field at the magnetic poles is normal to the
earths surface (i.e., in vertical direction) and at magnetic equator it is parallel to the earths surface, (i.e., in
horizontal direction). Thus, the resultant earths field can be resolved in two components as shown in fig.
(a) the horizontal component H along AB and
(b) the vertical component V, along AD.
From fig.
Horizontal component H = Be cos ...(i)
and vertical component V = Be sin ...(ii)
V Be sin
tan
H Be cos
or V = H tan
Again Eqs. (i) and (ii) give

H 2 V 2 B2e cos 2 sin 2
or Be H 2 V 2

www.physicsashok.in 33
MAGNETIC FIELD
THINKING PROBLEMS

1. Of the three vectors in the equation F qv B, which pairs are always at right angles? Which may have any
angle between them ?
2. If an electron is not deflected in passing through a certain space, can we be sure that there is no magnetic field
in that region?
3. A bream of protons is deflected side ways. Could this deflection be caused (a) by an electric field? (b) by a
magnetic field ? (c) If either is possible, how can you tell which one is present?
4. A rectangular current loop is in an arbitary orientation in an external magnetic field. Is any work required to
rotate the loop about an axis perpendicular to its plane?
5. Will a tangent galvanometer work in the polar region ?
6. At that deflections is a tangent galvanometer most sensitive?
7. Why is the magnetic needle short in a tangent galvanometer ?
8. Why is the field in a moving coil galvanometer radial in nature?
9. What is the greatest disadvantage with a suspended-type moving coil galvanometer?
10. An ammeter is a Weston galvanometer whose resistance is made negligible by shunting the coil. Is this true or
false?
11. A voltmeter is a Weston galvanometer of very high resistance. Is this true of False?
12. Which gives a more accurate value of a potential difference, a potentiometer or a voltmeter?
13. Why is an ammeter connected in series?
14. What is a faraday?
15. There is no charge in the energy of a charged particle moving in a magnetic field althrough a magnetic force is
acting on it. Is this true or false? Give reasons in support of your answer.
16. A current-carrying circular conductor is placed in a uniform magnetic field with its plane perpendicular to the
field. Does it experience any force? If it does, what is this force if its radius is a and the current passing through
it equals?
17. How do you know that the current inside a conductor is constituted by electrons and not by protons?
18. A copper pipe is filled with an electrolyte. When a voltage is applied, the current in the electrolyte is constituted
by the movement of positive and negative ions in opposite directions. Will such a pipe experience a force when
placed in a magnetic field perpendicular to the current?
19. Two parallel wires currying current in the same direction attract each other while two beams of electrons
travelling in the same direction repel each other. Explain why ?
20. Can a charged particle entering a uniform magnetic field normally from outside complete a circle?
21. A cylindrical electrolytic bath containing a solution of copper sulphate between two electrodes is mounted
above the north pole of a strong electromagnet. One electrode is at the axis () and the other electrode is at the
edge of the bath (+). What happens to the electrolyte in these circumstances?
22. A very strong current is made to flow for a short time through a solenoid. Will there be any change in its length
and diameter? Explain.
23. Cosmic rays are charged particles that strike the atmosphere from some external source. It is found that more
low-energy cosmic rays each the earth at the north and south magnetic poles than at the magnetic equator. Why
is this so ?

www.physicsashok.in 34
MAGNETIC FIELD
SOLUTION OF THINKING PROBLEMS

1. The pairs F , B and F , v are always at right angle. B and v may have any angle between them.
2. No, we cannot be sure that there is no magnetic field because the force will be zero when the direction of
motion is along the direction of the field.
3. (a) Yes, it could be due to an electric field directed perpendicular to the motion. (b) Ys, it could be due to a
magnetic field. (c) Stop the proton and keep it stationary. If it sill experiences a force in the same direction, it is
due to an electric field. If the force vanishes of stopping the motion, there is a magnetic field.
4. No, no work is done in rotating the coil because work done W = mB (1 cos). Here there is no charge in
and so no work is done.
5. No, because the earths magnetic field is vertical there.
6. A tengent galvanometer is most sensitive when the deflections are near about 0.
7. The field due to the circular coil is uniform over a very small region about the centre of the coil. So the needle
must be short so that it may be assumed to move in uniform megnetic fields.
8. The field is made radial in order to have a linear relation between the current and the deflection.
9. The greatest disadvntage is that this type of galvanometer is not portable.
10. It is true. 11. It is true.
12. A potentiometer gives a more accurate value of a potential difference than a voltmeter as it draws no current
from the cell.
13. Since the current following through the circuit has to pass through the ammeter, it must be connected in series
because it is a characteristic property of series connection that the same current passes through all parts of the
circuit.
14. A faraday is the amount of charge required to liberate 1 gram equivalent of any sustance.
15. True. A charged particle experiences a force at right angles to the velocity and so it moves in a circle with a
constant speed. So there is no charge in the energy fo the particle though a magnetic force acts on it.
16. It does not experience may force because the forces on the elements are directed radially away and sum up to
zero.
17. By the Hall effect. When a current-carrying flet conductor is placed in a magnetic field perpendicular to the flat
face, a transverse voltage is developed. From the direction of this voltage called Hall voltage, it can be determined
whether the current is due to negative or positive charge carriers.
18. The current constituted by negtive ions in the positive direction is the same as the current formed by the positive
ions and so both form current in the same direction; they do not cancel out each other. So the pipe will
experience a force.
19. A current in on wire produces only a magnetic field and on electric field (because a current-carrying conductor
is electrically neutral) over the other current-carrying conductor and so only a magnetic force which is attractive
is nature arises between them. But an electron beam is a source of both, an electric and a magnetic field. So
there arise both magnetic force (attractive) and electric force (repulsive) between them. The repulsive force
being in excess of the attractive force, they repel each other.
20. No, the boundary line of the magneitc field has to be the diameter of the circle, whatever be the velocity of the
charged particle. Hence, it will complete a semi-circle.
21. It moves counterclockwise (viewed from above)
22. The current in the adjacent turns flow in the same direction and so they attract each other. Thus there is
contraction along the length of the solenoid. As the same currents in the diametrically opposite elements flow in
opposite directions, they repel each other. On account of this repulsive force the diameter of the solenoid
increases.
23. Because at the poles, the magnetic field is parallel to the direction of motion of the cosmic ray particles, both
being vertical. Hence the cosmic ray particles do not experience any force. The force experienced is given by

F = qvb sin( v , B ). At the equator these particles experience maximum deflecting force and hence low-energy gy
particles cannot reach the earth.
www.physicsashok.in 35
MAGNETIC FIELD

ASSERTION & REASON


A statement of Statement-1 is given and a Corresponding statement of Statement-2 is given just below it of
the statements, mark the correct answer as
(A) If both Statement-1 and Statement-2 are true and Statement-2 is the correct explanation of Statement-1.
(B) If both Statement-1 and Statement-2 are true and Statement-2 is NOT correct explanation of Statement-1.
(C) If Statement-1 is true but Statement-2 is false.
(D) If both Statement-1 and Statement-2 are false.
(E) If Statement-1 is false but Statement-2 is true.
1. Statement-1 : Magnetic field interacts with a moving charge and not with a stationary charge.
Statement-2 : A moving charge produces a magnetic field.

2. Statement-1 : If an electron is not deflected while passing through a certain region of space, then only
possibility is that there is no magnetic field in this region.
Statement-2 : Force is directly proportional to magnetic field applied.

3. Statement-1 : Free electrons always keep on moving in a conductor even then no magnetic force act on
them in magnetic field unless a current is passed through it.
Statement-2 : The average velocity of free electron is zero.

4. Statement-1 : Electron cannot be accelerated by the cyclotron.


Statement-2 : Cyclotron is suitable only for accelerating heavy particles.

5. Statement-1 : The coil is wound over the metallic frame in moving coil galvanometer.
Statement-2 : The metallic frame help in making steady deflection without any oscillation.

6. Statement-1 : In electric circuits, wires carrying currents in opposite directions are often twisted together.
Statement-2 : If the wire are not twisted together, the combination of the wires forms a current loop. The
magnetic field generated by the loop might affect adjacent circuits or components.

7. Statement-1 : If an electron and proton enter in an electric field with equal energy, then path of electron
is more curved than that of proton.
Statement-2 : Electron has a tendency to form large curve due to small mass.

8. Statement-1 : If a proton and an -particle enter a uniform magnetic field perpendicularly, with the same
speed, the time period of revolution of -particle is double that of proton.
Statement-2 : In a magnetic field, the time period of revolution of a charged particle is directly proportional
to the mass of the particle and is inversely proportional to charge of particle.

9. Statement-1 : If an electron while coming vertically from outer space enter the earths magnetic field, it
is deflected towards west.
Statement-2 : Electron has negative charge.

10. Statement-1 : An electron and proton enters a magnetic field with equal velocities, then, the force
experienced by proton will be more than electron.
Statement-2 : The mass of proton is 1837 times more than the mass of electron.

11. Statement-1 : The magnetic field produced by a current carrying solenoid is independent of its length
and cross sectional area.
Statement-2 : The magnetic field inside the solenoid is uniform.

12. Statement-1 : The sensitivity of a moving coil galvanometer is increased by placing a suitable magnetic
material as a core inside the coil.
Statement-2 : Soft iron has a high magnetic permeability and cannot be easily magnetized or
demagnetized. [JEE 2008]
www.physicsashok.in 36
MAGNETIC FIELD

Level # 1
1. An electron is accelerated to a high speed down the axis of a cathode ray tube by the application of a
potential difference of V volts between the cathode and the anode. The particle then passes through a
uniform transverse magnetic field in which it experiences a force F. If the potential difference between the
anode and the cathode is increased to 2 V, the electron will now experience a force
(A) F 2 (B) F 2 (C) 2 F (D) 2 F
2. In a hydrogen atom, an electron of mass m and charge e is in an orbit of radius r making n revolutions per
second. If the mass of the hydrogen nucleus is M, the magnetic moment associated with the orbital motion
of the electron is
n er 2 m n er 2 M n er 2 m
(A) (B) (C) (D) n e r 2
M m (M m)

3. An electron of charge e moves in a circular orbit of radius r around a nucleus. The magnetic field due to
orbital motion of the electron at the site of the nucleus if B. The angular velocity of the electron is
2 0 eB 0 e B 4 r B 2 r B
(A) (B) (C) (D)
4 r r 0 e 0 e

D C G
4. Three long, straight and parallel wires C, D and G carrying currents are
arranged as shown in Figure. The force experienced by a 25 cm length
of wire C is
30 A 10 A 20 A
(A) 0.4 N (B) 0.04 N
(C) 4 x 103 N (D) 4 x 104 N

3 cm 10 cm
5. A charged particle of specific charge s passes through a region of space shown. 2

x x x xB
(A) Velocity of the particle in REGION 1 is . x x x x 0
B x x x x
(B) Work done to move the charged particle in REGION 1 is ZERO. 1
x x x x
(C) The radius of the trajectory of the charged particle in x x x x x x x x
v B x x x x
REGION 2 is . x x x x
s 0 S x x x Ex
x x x x
REGION 1 REGION 2
(D) The particle emerges from REGION 2 with a velocity '

where ' = . X
6. A wire of resistance R in the form of a semicircle lies on the top of a smooth B
table. A uniform magnetic field B is confined to the region as shown. The ends
of the semicircle are attached to springs C and D whose other ends are fixed. B=0
If r is the radius of the semicircle and k is the force constant for each spring, C below this line D
then the extension x in each spring is
2EBr 2EBR EBr EBr
(A) x (B) x (C) x (D) x E
kR kr kR 2kR

7. An infinite collection of current carrying conductors each carrying a current outwards perpendicular to
paper are placed at x = x 0, x = 3x 0, x = 5x 0, ........ ad infinitum on the x axis. Another infinite collection of
current carrying conductors each carrying a current inwards perpendicular to paper are placed at x = 2x 0,
x = 4x 0, x = 6x 0, ....... ad infinitum Here x 0 is a positive constant. The magnetic field at the origin due to the
above collection of current carrying conductors is
0 0 loge 2
(A) ZERO (B) (C) (D)
4x 0 loge 2 2x 0

www.physicsashok.in 37
MAGNETIC FIELD
Z
B
D

8. A conductor ABCDEF, with each side of length L, is bent as shown. L


It is carrying a current in a uniform magnetic induction (field) B,
parallel to the positive y-direction. The force experienced by the wire is C E F
(A) BIL in the positive y-direction. L
(B) BIL in the negative z-direction. O Y
(C) 3 BIL
(D) zero X B A
X C
B
9. The square loop ABCD, carrying a current , is placed in a uniform magnetic
Z
field B, as shown. The loop can rotate about the axis XX. The plate of the
loop makes an angle ( < 90) with the direction of B. Through what B
Y
angle will the loop rotate by itself before the torque on it becomes zero ? D
(A) (B) 90 (C) 90 + (D) 180
A
X
y
10. An electron is fired from the point A with a velocity V0 = 2 x 108 m/s.
The magnitude and direction of magnetic field that will cause the V0
electron to follow a semicircular path from A to B is
(A) 1.5 x 104 T out of the page.
(B) 1.5 x 103 T into the page x
A B
(C) 1.5 x 102 T into the page 15 cm
(D) 1.5 x 102 T out of the page
A
11. Two long straight parallel wires 2 m apart, are perpendicular to the plane of the
paper. Wire A carries a current of 10 A directed into the plane of the paper. Wire 1.6 m
B carries a current such that the magnetic field at P at a distance of 0.8 m from
this wire is zero.The magnitude and direction of the current in wire is zero. The 2m
magnitude and direction of the current in wire B is S
(A) 2.8 A into the page
1.2 m
(B) 2.8 A out of the page
B
(C) 2.8 x 106 A into the page.
(D) 2.08 A out of the page. P

12. Two particles each of mass m and charge q, are attached to the two ends of a light rigid rod of length 2 .
The rod is rotated at a constant angular speed about a perpendicular axis passing through its centre. The
ratio of the magnitudes of the magnetic moment of the system and its angular momentum about the centre
of the rod is

q q 2q q
(A) (B) (C) (D) m
2m m m
13. If a charged particle is describing a circle of radius r in a magnetic field with a time period T then,
(A) T 2 r 3 (B) T 2 r (C) T r 2 (D) T r 0


14. A non-conducting rod AB of length has a linear charge density .
The rod is rotated about an axis passing through point A with constant
angular velocity as shown in the figure. The magnetic moment of
+++
the rod is A B
2
2
3 3
3

(A) (B) (C) (D)


2 3 2 6
www.physicsashok.in 38
MAGNETIC FIELD

15. A charged particle of mass m and charge q is released from rest from (x 0, 0) along an electric field E0 j . The
angular momentum of the particle about origin
(A) is zero (B) is constant (C) increases with time (D) decreases with time

16. Two particles Y and Z emitted by a radioactive source at P made tracks


in a could chamber as illustrated in the figure. A magnetic field acted Y
downward into the paper. Careful measurements showed that both tracks
were circular, the radius of Y track being half that of the Z track. Z
Which one of the following statements is certainly true? P
(A) Both Y and Z particles carried a positive charge.
(B) The mass of Z particle was one half that of the Y particle.
(C) The mass of the Z particle was twice that of the Y particle.
(D) The charge of the Z particle was twice that of the Y particle.

17. The resistances of three parts of a circular


loop are as shown in the figure. The magnetic R B
field at the centre O is a 2R
O 120
0 I 0 I 2 0 I A
(A) (B) (C) (D) Zero I
6a 3a 3 a 120
C
18. A particle with a specific charge s is fired with a speed v towards a wall at a distance
R
d, perpendicular to the wall. What minimum magnetic field must exist in this region for the
particle not to hit the wall?
(A) v/sd (B) 2v/sd (C) v/2sd (D) v/4sd

19. Current flows through a long conducting wire bent at right


angle as shown in figure. The magnetic field at a point P on
the right bisector of the angle XOY at distance r from O is P
Y
0 20 r
(A) (B)
r r
x
20 0
(C)
4 r

2 1 (D)
2 r

2 1 O
45
X
I
20. Two circular coils X and Y having equal number of turns Y
and carry equal currents in the same sense and subtend
same solid angle at point O. If the smaller coil X is midway
d X d O
be between O and Y, then if we represent the magnetic
induction due to bigger coil Y at O as By and that due to
smaller coil X at O as Bx, then
By By By 1 By 1
(A) 1 (B) 2 (C) (D)
Bx Bx Bx 2 Bx 4
21. Two mutually perpendicular conductors carrying 1 and 2 lie in the x-y plane. Find locus of points at which
magnetic induction is zero?

22 1 1 2 2 1
(A) y x (B) x y (C) y x (D) x y
2 2 2 2

www.physicsashok.in 39
MAGNETIC FIELD

22. A coil in the shape of an equilateral triangle of side 0.02 m


is suspended from a vertex such that it is hanging in a
vertical plane between the pole pieces of a permanent
magnet producing a horizontal magnetic field of 5 x 102 T.
Find the couple acting on the coil when a current of 0.1 N S
ampere is passed through it and the magnetic field is
parallel to its plane.
(A) 5 2N m (B) 5 2N 104 N m
(C) 5 3 107 N m (D) 10 7 N m


23. A particle of charge q and mass m starts moving from the origin under the action of an electric field E E0i


and magnetic field B B0 k . It s velocity at (x, 3, 0) is 4 i 3 j . The value of x is:

36E0 B0 25m 10 m 25 E0 B0
(A) (B) (C) (D)
qm 2q E0 q E0 m

24. Two charged particles A and B enter a uniform magnetic field with velocities normal to the field. Their paths
are shown in the Figure. The possible reasons are:

(A) The momentum of A is greater than that of B (B) the charge of A is greater than that of B.
(C) The specific charge of A is greater than that of B (D) the speed of A is less than that of B.

25. Two charged particle M and N enter a space of uniform magnetic field, with velocities, perpendicular to the
magnetic field. the paths are as shown in the figure. The possible reason are:

(A) the charge of M is greater than that of N. (B) the momentum of M is greater than that of N.
(C) specific charge of M is greater than that of N. (D) the speed of M is less than that of N.

26. A current-carrying ring is placed in a magnetic field. The direction of the field is perpendicular to the plane of
the ring, then:
(A) there is no net force on the ring. (B) the ring may tend to expand.
(C) the ring may tend to contact (D) none of these.


27. A charge q is moving with a velocity v1 1i m s at a point in a magnetic field and experiences a force

F1 q 1 j 1 k N. If the charge is moving with a velocity v2 1 j ma/s at the same point, it experiences

www.physicsashok.in 40
MAGNETIC FIELD

a force F2 q 1i 1k N . The magnetic induction B at that point is:


(A) i j k Wb m
2

(B) i j k Wb m
2

(C) i j k Wb m
2

(D) i j k Wb m
2

28. A parallel beam of electrons is shot into a uniform electric field, initially parallel to and against the field with
a small initial speed. Then:
(A) the beam will pass through the field accelerating down the field without changing its width.
(B) the beam tends to spread out at the beginning and to narrow down later.
(C) the beam tends to narrow down at the beginning and to spread out later.
(D) the total energy of the beam is conserved.

29. The ratio of the energy required to set up in a cube of side 10 cm a uniform magnetic field of 4 wb/m 2 and a
uniform electric field of 106 V/m is:
(A) 1.4 x 107 (B) 1.4 x 105 (C) 1.4 x 106 (D) 1.4 x 103

30. A charged particle is fired at an angle in a uniform magnetic field directed along the x-axis. During its
motion along a helical path, the particle will:
(A) never parallel to the x-axis.
(B) move parallel to the x-axis once during every rotation for all values of .
(C) move parallel to the x-axis at least once during every rotation if 45.
(D) never move perpendicular to the x-direction.


31. A charged particle q enters a region of uniform B (out of the page) and
is deflected a distance d after travelling a horizontal distance a. The
magnitude of the momentum of the particle is:

qB a 2
(A) d (B) qBa (C) Zero (D) not possible to be determined as it keeps changing.
2 d 2

32. Two insulated rings, one of slightly smaller diameter then the other, are suspended
along their common diameter as shown in the figure, initially the planes of the rings
are mutually perpendicular. When a steady current is set up in each of them:
(A) The two rings rotate towards a common plane.
(B) The inner ring will oscillate about its initial position.
(C) The inner ring stays stationary while the outer one moves into the plane of the inner ring.
(D) The outer ring stays stationary while the inner one moves into the plane of the outer ring.

33. A particle with charge +Q and mass m enters a magnetic field of magnitude B,
existing only on the right of the boundary YZ. The direction of the motion of the
m
particle is perpendicular to the direction of B. Let T 2 . The time spent
QB
by the particle in the field will be:
2 2
(A) T (B) 2T (C) T (D) T
2 2
34. The current I flows through a square loop of a wire of side a. The magnetic induction at the centre of the loop
is:

2 0 2 2 0 2 0 2 0
(A) (B) (C) (D)
a a a a

www.physicsashok.in 41
MAGNETIC FIELD
35. Suppose a uniform electric field and a uniform magnetic field exist along mutually perpendicular directions in
a gravity free space. If a charged particle is released from rest, at a point in the space:
(A) particle can not remain in static equilibrium.
(B) first particle will move along a curved path but after some time its velocity will become constant.
(C) particle will come to rest at regular interval of time.
(D) acceleration of the particle will never become equal to zero.

36. A circular loop of radius R carries a charge q uniformly distributed on it. It is rotated at a frequency f about
one of the diameters. A uniform magnetic field B exists along its diameter . The maximum and minimum
torques acting on the loop due to the magnetic field are, respectively:
(A) qfR 2 B, 0 (B) 0, 0 (C) 2 qfR 2 B , qfR 2 B (D) None of these

37. A semi-circular wire of radius R is connected to a wire bent in the form of a sine curve
to form a closed loop as shown n the figure. If the loop carries a current and is placed
in a uniform magnetic field B, then the total force acting on the sine curve is:
(A) 2BR (downward) (B) 2BR (upward) (C) B R (upward) (D) Zero
B



38. A charged particle of unit mass and unit charge moves with velocity of v 8 i 6 j m s in a magnetic field

of B 2 k T . Choose the correct alternative(s):
(A) The path of the particle be x 2 + y2 4x 21 = 0. (B) The path of the particle may be x 2 + y2 = 25.
(C) The path of the particle may be y2 + z2 = 25. (D) The time period of the particle will be 3.14 s.

39. A proton, a deuteron and an -particle having the same kinetic energy are moving in circular trajectories in a
constant magnetic field. If rp , rd , and r denote respectively the radii of the trajectories of these particles,
then. (JEE 1997)
(A) r rp rd (B) r rd rp (C) r rd rp (D) rp rd r

40. A charged particle is released from rest in a region of steady and uniform electric and magnetic fields which
are parallel to each other. The particle will move in a (JEE 1999)
(A) straight line (B) circle (C) helix (D) cycloid

41. Two long parallel wires are at a distance 2d apart. They carry steady equal currents flowing out of the plane of
the paper, as shown. The variation of the magnetic field B along the line XX is given by (JEE 2000)
B B

X X X X
(A) (B)
d d d d
B B

X X X X
(C) (D)
d d d d
42. An infinitely long conductor PQR is bent to form a right angle as shown in figure. A current flows through
PQR. The magnetic field due tot his current at the point M is H1. Now, another infinitely long straight conductor
QS is connected at Q so that current is 2 in QR as well as in QS, the current in PQ remaining unchanged.
The magnetic field at M is now H2. The ratio H1 H 2 is given by (JEE 2000)

(A) (B) 1 (C) 23 (D) 2

www.physicsashok.in 42
MAGNETIC FIELD
43. An ionized gas contains both positive and negative ions. If it is subjected simultaneously to an electric field
along the +x direction and a magnetic field along the +z direction, then (JEE 2000)
(A) positive ions deflect towards +y direction and negative ions towards y direction
(B) all ions deflect towards +y direction.
(C) all ions deflect towards y direction
(D) positive ions deflect towards y direction and negative ions towards +y direction.

44. A non-planar loop of conducting wire carrying a current is placed


as shown in the figure. Each of the straight sections of the loop is z
of length 2a. The magnetic field due to this loop at the point y
I
P(a, 0, a) points in the direction (JEE 2001)
1 1
(A)
2

j k (B)
3

j k i x

1 1 2a
(C)
3

i jk (D)
2
i k
45. Two particles A and B of masses m A and m B respectively and having the same
charge are moving in a plane. A uniform magnetic field exists perpendicular to A
this plane. The speeds of the particles are VA and VB respectively and the B
trajectories are as shown in the figure. Then (JEE 2001)
(A) m A v A < mB v B (B) m A v A > mB v B
(C) m A < mB and v A < v B (D) m A = mB and v A = v B

46. A coil having N turns is wound tightly in the form of a spiral with inner and outer radii a and b respectively.
When a current passes through the coil, the magnetic field at the center is (JEE 2001)

0 N 2 0 N 0 N b 0 N b
(A) (B) (C) 2 b a ln a (D) ln
b b 2 b a a

47. A particle of mass m and charge q moves with a constant velocity v along the positive x direction. It enters a
region containing a uniform magnetic field B directed along the negative z direction, extending from x = a to
x = b. The minimum value of v required so that the particle can just enter the region x > b is (JEE 2002)

qbB q b a B qaB q b a B
(A) (B) (C) (D)
m m m 2m

48. A long straight wire along the z-axis carries a current in the negative z direction. The magnetic vector field

B at a point having coordinates (x, y) in the z = 0 plane is (JEE 2002)


0 y i x j 0 x i y j 0 x j y i 0 x i y j
(A) (B) (C) (D)
2 x 2
y2 2 x 2
y2 2 x 2 y 2 2 x 2 y 2

49. For a positive charged particle moving in a x-y plane initially along
the x-axis, there is a sudden change in its path due tot he presence y
of electric and/or magnetic fields beyond P. The curved path is shown
in the x-y plane and is found to be non-circular. Which one of the P x
following combinations is possible? (JEE 2003)


(A) E 0; B b i c k (B) E a i; B c k a i

(C) E 0; B c j b k (D) E a i; B c k b j

www.physicsashok.in 43
MAGNETIC FIELD
B Y
50. A conducting loop carrying a current is placed in a uniform
magnetic field pointing into the plane of the paper as shown. x
The loop will have a tendency to (JEE 2003)
(A) contract (B) expand I
(C) move towards +ve x-axis (D) move towards ve x-axis.

51. A current carrying loop is placed in a uniform magnetic field in four different orientations, I, II, III & IV, arrange
them in the decreasing order of Potential Energy (JEE 2003)
n
n B
B B
B
n II n
I III IV
(A) I > III > II > IV (B) I > II > III > IV (C) I > IV > II > III (D) III > IV > I > II

52. An electron traveling with a speed u along the positive x-axis enters y B
into a region of magnetic field where B B0 k x 0 . It comes out
e u
of the region with speed v then (JEE 2004) x
(A) v = u at y > 0 (B) v = u at y < 0
(C) v > u at y > 0 (D) v > u at y < 0

53. A magnetic needle is kept in a nonuniform magnetic field. It experiences (JEE 1982)
(A) a force and a torque (B) a force but not a torque
(C) a torque but not a force (D) neither a force nor a torque

54. A proton moving with a constant velocity passes through a region of space without any change in its velocity.
If E and B represent the electric and magnetic fields respectively. This region of space may have:(JEE 1985)
(A) E = 0, B = 0 (B) E = 0, B 0 (C) E 0, B = 0 (D) E 0, B 0

55. A rectangular loop carrying a current i is situated near a long straight wire
such that the wire is parallel to one of the sides of the loop and is in the i
plane of the loop. If steady current is established in the wire as shown I
in the figure, the loop will : (JEE 1985)
(A) rotate about an axis parallel to the wire (B) move away from the wire
(C) move towards the wire (D) remain stationary

56. Two particle X and Y having equal charges, after being accelerated through the same potential difference,
enter a region of uniform magnetic field and describe circular paths of radii R1 and R2 respectively. The ratio of
the mass of X to that of Y is (JEE 1988)
12 2
(A) R1 R2 (B) R2 R1 (C) R1 R2 (D) R1 R2

57. A particle of charge +q and mass m moving under the influence of a uniform
P V E
electric field E i and uniform magnetic field B k follows a trajectory from
P to Q as shown in Figure. The velocities at P and Q are v i and 2 j B
a
which of the following statement(s) is/are correct? (JEE 1991)
3 mv 2 Q
E
4 qa
(A) x
2a 2v

3 mv 3
(B) Rate of work done by the electric field at P is
4 a
(C) Rate of work done by the electric field at P is zero
(D) Rate of work done by the electric field at Q is zero

www.physicsashok.in 44
MAGNETIC FIELD
58. A current 1 flows along the length of an infinitely long, straight, thin-walled pipe. Then: (JEE 1993)
(A) The magnetic field at all points inside the pipe is the same, but not zero
(B) The magnetic field at any point inside the pipe is zero.
(C) The magnetic field is zero only on the axis of the pipe
(D) The magnetic field is different at different points inside the pipe.

59. H+ , He+ and O ++ all having the same kinetic energy pass through a region in which there is a uniform magnetic
field perpendicular to their velocity. The masses of H+, He+ and O 2+ are 1 amu, 4 and 16 amu respectively.
The: (JEE 1994)
(A) H+ will be deflected most. (B) O 2+ will be deflected most
(C) He+ and O 2+ will be deflected equally (D) All will be deflected equally.

60. Two very long, straight, parallel wires carry steady currents and respectively. The distance between the
wires is d. At a certain instant of time, a point charge q is at a point equidistant from the two wires, in the
plane of the wires, Its instantaneous velocity v is perpendicular to this plane. The magnitude of the force due
to the magnetic field acting on the charge at this instant is (JEE 1998)
0 qv 0 qv 20 qv
(A) (B) (C) (D) 0
2 d d d
61. An infinite current carrying wire passes through point O and in perpendicular C
to the plane containing a current carrying loop ABCD as shown in the figure. B
Choose the correct option (s) (JEE 2006)
(A) Net force on the loop is zero O O
(B) Net torque on the loop is zero. A
(C) As seen from O, the loop rotates clockwise. D
(D) As seen from O, the loop rotates anticlockwise.

B0
62. A magnetic field B B0 j exists in the region a < x < 2a and B B0 j , in


the region 2a < x < 3a, where B0 is a positive constant. A positive point charge 0 a 2a 3a x
moving with a velocity v v 0 j , where v 0 is a positive constant, enters the
magnetic field at x = 0. The trajectory of the charge in this region can be like,
B0
(JEE 2007)
z z z z
a 2a 3a a 2a 3a
(A) a 2a 3a x (B) x (C) a 2a 3a x (D) x

Region I Region II Region III


63. A particle of mass m and charge q, moving with velocity v enters
Region II normal to the boundary as shown in the figure. Region II
has a uniform magnetic field B perpendicular to the plane of the 0
v
paper. The length of the Region II is l. Choose the correct choice(s)
ql B
(A) The particle enters Region III only if its velocity v (JEE 2008) l
m
ql B
(B) The particle enters Region III only if its velocity v
m
ql B
(C) Path length of the particle in Region II is maximum when velocity v
m
(D) Time spent in Region Ii is same for any velocity V as long as the particle returns to Region I

www.physicsashok.in 45
MAGNETIC FIELD
FILL IN THE BLANKS

X X X X X X X X X
1. A neutron a proton, and an electron and alpha particle enter a region X BX X
C
X X X X X X
of constant magnetic field with equal velocities. The magnetic field is X X X X X X X X X
X X X X X X X X X
along the inward normal to the plane of the paper. The tracks of the
X X X X X X X X X
particles are labelled in Figure. The electron follows track ________
X X X X X X X X X
D
and the alpha particle follows track ________. (JEE 1984)

2. A wire of length L meters carrying a current i amperes is bent in the form of a circle. The magnitude of its
magnetic moment is ________ in MKS units. (JEE 1987)
3. In a hydrogen atom, the electron moves in an orbit of radius 0.5 making 1016 revolutions per second. The
magnetic moment associated with the orbital motion of the electron is ________ . (JEE 1988)
I
4. The wire loop PQRSP formed by joining two semicircular wires of I
radii R1 and R2 carries a current 1 as shown. The magnitude of
the magnetic induction at the centre C is ________ . (JEE 1988) R2 R
1
S R C Q P
Z D

C I
5. A wire ABCDEF (with each side of length L) bent as shown in figure F
and carrying a current is placed in a uniform magnetic induction B E
parallel to the positively-direction. The force experienced by the wire Y
is ________ in the ________ direction. (JEE 1990)
B A
X

6. A metallic block carrying current is subjected to a uniform magnetic induction as B as shown in Figure.


The moving charges experience a force F given by _____ which results in the lowering of the potential of the
face _____ Assume the sped of the carriers to be v. (JEE 1996)

7. A uniform magnetic field with a slit system as shown in figure is B


to be used as momentum filter for high-energy charged particles.
With a field B Tesla, it is found that the filter transmits -particles
each of energy 5.3 MeV. The magnetic field is increased to 2.3 B
Tesla and deuterons are passed into the filter. the energy of each
deuteron transmitted by the filter is _____ MeV. (JEE 1997)
Source Detector

www.physicsashok.in 46
MAGNETIC FIELD
TRUE / FALSE

8. No net force acts on a rectangular coil carrying a steady current when suspended freely in a uniform magnetic
field. (JEE 1981)

9. There is no change in the energy of a charged particle moving in magnetic field although a magnetic force is
acting on it. (JEE 1983)

10. A charged particle enters a region of uniform magnetic field at an angle of 85 to the magnetic line of force.
The path of the particle is a circle. (JEE 1983)

11. An electron and proton are moving with the same kinetic energy along the same direction. When they pass
through a uniform magnetic field perpendicular to the direction of their motion, they describe circular paths of
the same radius. (JEE 1985)

TABLE MATCHING
12. Column I Column II
(A) Electric field (P) Stationary charge
(B) Magnetic field (Q) Moving charge
(C) Electric force (R) Changes the kinetic energy
(D) Magnetic force (S) Does not change kinetic energy
13. Regarding the trajectory of a charged particle, match the following:
Column I Column II
(A) In electric field (P) Straight line path
(B) In magnetic field (Q) Circular path
(C) In crossed field (R) Helical path
(D) In mutually perpendicular electric and (S) Parabolic path
magnetic field, charge being at rest (T) Parabolic path
14. A current flows along length of a long thin cylindrical shell:
Column I Column II
(A) Magnetic field at all points lying inside the shell (P) Inversely proportional with distance
from axis of shell
(B) Magnetic field at any point outside the shell (Q) Zero
(C) Magnetic field is maximum (R) Just outside the shell
(D) Magnetic field on the axis of the shell
15. Column I Column II
(A) Unit of magnetic field (P) Am 2
(B) Unit of magnetic permeability (0) (Q) N Am
(C) Unit of magnetic flux () (R) N A2

(D) Unit of magnetic dipole moment (S) Nm A


16. Two long parallel wires carrying equal currents in opposite directions are placed at x = a parallel to Y-axis
with z = 0. Then:
Column I Column II
0 i
(A) Magnetic field B1 at origin O (P)
3 a
0i
(B) Magnetic field B2 at P (2a, 0, 0) (Q)
4 a
0i
(C) Magnetic field at M (a, 0, 0) (R)
a
(D) Magnetic field at N (a, 0, 0) (S) Zero

www.physicsashok.in 47
MAGNETIC FIELD
17. Two wires each carrying a steady current I are shown in four configurations in Column-I. Some of the resulting
effects are deseribed in Column-II. Match the statements is Column-I with the statements in Column-II.
Column I Column II [JEE 2007]

(A) Point P is situated P (P) The magnetic fields (B) at P due to the currents
midway between the wires. in the wires are in the same direction.
(B) Point P is situated at the (Q) The magnetic fields (B) at P due to the currents in
P
mid-point of the joining the the wires are in opposite directions.
centers of the circular
wires, which have same radii.
(C) Point P is situated at the (R) There is no magnetic field at P
mid-point of the line joining P
the centres of the circular
wires, which have same radii.
(D) Point P is situated at the (S) The wires repel each other.
P
common center of the wires.

18. Six point charges, each of the same magnitude q, are arranged in different manners as shon in Column-II. In
each case, a point M and a line PQ passing through M are shown. Let E be the electric field and V be the
electric potential at M (potential at infinity is zero) due to the given charge distribution when it is at rest. Now,
the whole system is set into rotation with a constant angular velocity about the line PQ. Let B be the
magnetic field at M and be the magnetic moment of the system in this condition. Assume each rotating
charge to be equivalent to a steady current. [JEE 2009]
Column I Column II
(A) E = 0 (P) + Charges are at the corners of a regular hexagon.
Q M is at the centre of the hexagon. PQ is

M + perpendicular to the plane of the hexagon.
P +
P
(B) V 0 (Q) + + + Charges are on a line perpendicular to PQ at
M equal intervals. M is the mid-point between the
Q two innermost charges.

(C) B=0 (R) + Q Charges are placed on two coplanar insulating


+
rings at equal intervals. M is the common centre
M
of the rings. PQ is rependicular to the plane of
P the rings.
+
(D) 0 (S) + Charges are placed at the corners of a rectangle
of sides a and 2a and at the mid points of the
P M Q longer sides. M is at the centre of the rectangle.
+ PQ is parallel to the longer sides.
P
(T) + Charges are placed on two coplanar, insulating
M rings at equal intervals. M is the mid-point bet-
+ + ween the centres of the rings. PQ is perpendi-
cular to the line joining the centres and coplanar
Q
to the rings.

www.physicsashok.in 48
MAGNETIC FIELD
PASSAGE TYPE

PASSAGE = 1.
A cyclotron is a device used by physicists to study the properties of subatomic particles. Small charged
particles are deposited at high speed into a circular pipe. electric and magnetic field then accelerate the
particle to an even higher speed. Finally, when the particle reaches the desired speed, a magnetic field keeps
it moving in a circle at constant speed.
In figure 1, a magnetic field pointing into the page keeps a charged particle travelling in a counterclockwise
circle at constant speed inside the cyclotron. The magnetic force on the particle points towards the center of
the circle, and has strength
F mag = qvB
where q is the particles charge, v is its speed, and B is the magnetic field strength. As a result of this force,
the particle moves in circles in the cyclotron at frequency
qB
f
2 m
North
particle
West East

South
where m denotes the particles mass. The frequency (in hertz) is the number of revolutions completed by the
particle per second.
In the following questions, neglect gravity.

1. In figure 1, what is the direction (if any) of the particles acceleration?


(A) North (B) East (C) West (D) It has no acceleration
2. If the magnetic field in figure were turned off, in which direction would the particle travel (until crashing)?
(A) North (B) East (C) West (D) Now here; it would stop moving
3. An alpha particle has charge 2e and mass 4 amu. A proton has charge e and mass 1 amu. Let f alpha and f proton
denote the frequencies with which those particles circle a cyclotron. If both particles experience he same
f alpha
magnetic field in the same cyclotron, what is
f proton ?
(A) 2 (B) 1 (C) 1 2 (D) 1 4
4. In order for a cyclotron to work properly, the magnetic field must make the particle move in a circle. Which of
the following particles would not work in a cyclotron?
(A) lithium atom (Li) (B) positive lithium ion (Li +)
+
(C) negative lithium ion (Li ) (D) all of the above particle would work.
5. Which of the following cannot create a magnetic field?
(A) Electrical current flowing through a well-insulated straight metal wire.
(B) A beam of electrons moving across a cathode ray tube.
(C) Electric current flowing around a superconducting ring.
(D) Static electricity (i.e., extra electrons) built up on a stationary door knob.

PASSAGE = 2.
Magnetically levitated (MAGLEV) trains are considered to be important future travelling machines. The
idea of MAGLEV transportation has been in existence since the early 1900s. The benefits of eliminating
friction between the wheel and the rail to obtain higher speeds and lower maintenance costs has great
appeal. The basic idea of a MAGLEV train is to levitate it with magnetic fields so that there is no physical
contact between the train and the rails. For comparison, bullet trains in Japan have a maximum speed of
about 250-300 km/h while a MAGLEV train under development has reached a speed of 411 km/hr. The
MAGLEV train uses powerful on board superconducting electromagnets with zero electrical resistance to
support the train above the rails.

www.physicsashok.in 49
MAGNETIC FIELD

The walls along the track contain a continuous series of vertical coils of ordinary wire. As the train
passes each coil, the superconducting magnet on the train induces a current in these coils and makes them
electromagnets. The electromagnets on the train and outside produce forces that levitate the train and keep
it centered above the rails. In addition, electric current flowing through coils outside the train propels the train
forward, as shown in Figure.
1. What is the major advantage of using super conductors in the electromagnets
(A) very low resistance for electric current (B) small size
(C) low hysteresis loss (D) zero electrical resistance.
2. Choose the correct statement
(A) MAGLEV trains have high maintenance cost
(B) They have low maintenance cost but high running cost
(C) they have low maintenance cost but high energy efficiency
(D) they have low maintenance and running cost as well as low installation cost.
3. MAGLE trains are operating in which country
(A) Japan (B) USA (C) China (D) None of these
4. Super conducting electromagnets are installed in
(A) inside the train (B) on the railroads (C) on the walls (D) on the station
5. Coils installed on the wall will carry currents
(A) only if train is moving very fast (B) all times whether train is standing or running
(C) only if train is moving irrespective of whether fast or slow. (D) None of these.

PASSAGE = 3.
A charged particle +q of mass m is placed at a distance d from
another charged particle 2q of mass 2 m in a uniform magnetic
x q, m x x -2q, 2m x
field B as shown in figure. If the particles are projected towards
each other with equal speeds v. V V
1. Find the maximum value of projection speed v m so that the two x x x x
particles do not collide.
2. Find the time after which collision occurs between the particles x x x x
if projection speed equals 2v m.
3. Assuming the collision to be perfectly inelastic find the radius d
x x x x
of particle in subsequent motion.
(Neglect the electric force between the charges)

www.physicsashok.in 50
MAGNETIC FIELD

Level # 2

1. A particle of charge q and mass m is projected from the origin with velocity v = 0 i in a nonuniform

magnetic field B B0 xk . Here 0 and B0 are positive constants of proper dimensions. Find the maximum
positive x coordinate of the particle during its motion.

2. A flat dielectric disc of radius R carries an excess charge on its surface. The surface charge density is .
The disc rotates about an axis perpendicular to its plane passing through the centre with angular velocity .
Find the torque on the disc if it is placed in a uniform magnetic field B directed perpendicular to the rotation
axis.
A y
3. A wire loop ABCDE carrying a current is placed in the x-y plane
as shown in figure. A particle of mass m and charge q is projected E
V0
from origin with velocity V
i j m/s. Find the
90
2 B x
O
(a) instantaneous acceleration.
(B) If an external magnetic field B B 0 i is applied, find the D
force and torque acting on the loop due to this field. r/2
C Z
4. A long wire of radius a is placed along z-axis and carries current i as indicated
in the figure. y-axis is taken perpendicular to the plane of paper directed into the i
paper. An electron escapes from the surface of the wire with velocity 0 directed
along x-axis. Determine the maximum distance from the wire along x-axis up to
which electron can move. e
O X
V0
5. A long horizontal wire AB, which is free to move in a vertical plane and carries
a steady current of 20 A, is in equilibrium at a height of 0.01 m over another A B
parallel long wire CD which is fixed in a horizontal plane and carries a steady
current of 30 A, as shown in figure. Show that when AB is slightly depressed, C D
it executes simple harmonic motion. Find the period of oscillations.

6. A current flows along a thin wire shaped as a regular polygon with n sides which can be inscribed into a
circle of radius R. Find the magnetic induction at the centre of the polygon.Analyse the obtained expression
at n .

7. Find the magnetic induction at the centre of rectangular wire frame whose diagonal is equal to d = 16 cm
and the angle between the diagonals is equal to = 30; the current flowing in the frame equals = 5.0 A.

8. A thin conducting strip of width h = 2.0 cm is tightly wound in the shape of a very long coil with cross-
section radius R = 2.5 cm to make a single-layer straight solenoid. a direct current = 5.0 A flows through
the strip. Find the magnetic induction inside and outside the solenoid as a function of the distance r from its
axis.

9. Find the magnetic moment of a thin round loop with current if the radius of the loop is equal to R = 100 mm
and the magnetic induction at its centre is equal to B = 6.0 T..

10. Calculate the magnetic moment of a thin wire with a current = 0.8 A,
wound tightly on half a tore (Figure). The diameter of the cross-section
of the tore is equal to d = 5.0 cm, the number of turns is N = 500.

www.physicsashok.in 51
MAGNETIC FIELD
11. A non-conducting thin disc of radius R charged uniformly over one side with surface density rotates about
its axis with an angular velocity . Find :
(a) the magnetic induction at the centre of the disc ; (B) the magnetic moment of the disc.

12. A small coil C with N = 200 turns is mounted on one end of a


balance beam and introduced between the poles of an electro-
magnet as shown in Figure. The cross-section area of the coil
is S = 1.0 cm 2, the length of the arm OA of the balance beam is
= 30 cm. W hen there is no current in the coil the balance is in
equilibrium. On passing a current = 22 mA through the coil the
equilibrium is restored by putting the additional counterweight of
mass m = 60 mg on the balance pan. Find the magnetic induction at the spot where the coil is located.

13. A square frame carrying a current = 0.90 A is located in the same plane as a long straight wire carrying a
current 0 = 5.0 A. The frame side has a length a = 8.0 cm. The axis of the frame passing through the
midpoints of opposite sides is parallel to the wire and is separated from it by the distance which is = 1.5
times greater than the side of the frame. Find :
(a) Ampere force acting on the frame ;
(B) the mechanical work to be performed in order to turn the frame through 180 about its axis, with the
currents maintained constant.

B
14. In an electromagnetic pump designed for transferring molten metals a pipe
section with metal is located in a uniform magnetic field of induction
B (Figure). A current is made to flow across this pipe section in the I
direction perpendicular both to the vector B and to the axis of the pipe.
Find the gauge pressure produced by the pump if B = 0.10 T, = 100 A, a
and a = 2.0 cm.

15. A proton accelerated by a potential difference V = 500 kV flies through


+B
a uniform transverse magnetic field with induction B = 0.51 T. The field

occupies a region of space d = 10 cm in thickness (Figure). Find the
angle through which the proton deviates from the initial direction of
its motion.
d

16. A slightly divergent beam of non-relativistic charged particles accelerated by a potential difference V propagates
from a point A along the axis of a straigth solenoid. The beam is brought into focus at a distance from the
point A at two successive values of magnetic induction B1 and B2. Find the specific charge q/m of teh
particles.

17. A non-relativistic proton beam passes without deviation through the region of space where there are uniform
transverse mutually perpendicular electric and magnetic fields with E = 120 kV/m and B = 50 mT. Then the
beam strikes a grounded target. Find the force with which the beam acts onthe target if the beam current is
equal to = 0.80 mA.
S
18. A beam of non-relativistic charged particles moves without deviation through A

the region of space A (Figure), where there are transverse mutually perpen-
dicular electric and magnetic fields with strength E and induction B. When
the magnetic field is switched off, the trace of the beam on the screen S
shifts by x . Knowing the distances a and b, find the specific charge q/m a b
of the particles.
www.physicsashok.in 52
MAGNETIC FIELD

Level # 3
B
1. An electron in the ground state of hydrogen atom is revolving in anticlock-wise n
30
direction in a circular orbit of radius R. (JEE 1996)
(i) Obtain an expression for the orbital magnetic dipole moment of the electron.

(ii) The atom is placed in a uniform magnetic induction B such that the
plane-normal of the electron-orbit makes an angle of 30 with the
magnetic induction. find the torque experienced by the orbiting electron.

2. A current of 10 A flows around a closed path in a circuit which is in the horizontal D

plane as shown in the figure. The circuit consists of eight alternating arcs of radii r2 C
r1 = 0.08 m and r1 = 0.12 m. Each arc subtends the same angle at the center.
(A) Find the magnetic field produced by this circuit at the center. (JEE 2001) r1 A
(B) An infinitely long straight wire carrying a current of 10 A is passing the
center of the above circuit vertically with the direction of the current being
into the plane of the circuit. What is three force acting on the wire at the
center due to the current in the circuit? What is the force acting on the arc AC and the straight segment
CD due to the current at the center? d

3. A wheel of radius R having charge Q, uniformly distributed on the rim of the wheel is
free to rotate about a light horizontal rod. The rod is suspended by light inextensible T0 T0
strings and a magnetic field B is applied as shown in the figure. The initial tensions in 0
the strings are T 0. If the breaking tension of the strings are 3T 0/2, find the maximum
angular velocity 0 with which the wheel can be rotated. (JEE 2003) B

y
4. A pair of stationary infinitely long bent wires are placed in the x-y plane as
shown in Figure. The wires carry currents of = 10 A each as shown. The Q
segments LR and MS are along the x- axis. the segments RP and SQ are L R O M
x
parallel to the y-axis such that OS = OR = 0.02 m. Find the magnitude and S
direction of the magnetic induction at the origin O. (JEE 1989)
P

5. A solenoid of length 0.4 m and having 500 turns of wire carries a current of 3 A. A thin coil having 10 turns of
wire and of radius 0.01 m carries a current of 0.4 A. Calculate the torque required to hold the coil in the
middle of the solenoid with its axis perpendicular to the axis of the solenoid. (JEE 1990)

6. Three infinitely long thin wires, each carrying current in the same direction are in the x-y plane of a gravity
free space. The central wire is along the y-axis while the other two are along x = d. (JEE 1997)
(a) Find the locus of points for which the magnetic field B is zero.
(B) If the central wire is displaced along the z-direction by a small amount and released, show that it will
execute simple harmonic motion. If the linear density of the wire is , find the frequency of oscillations.

7. A proton and an alpha particle, after being accelerated through same potential difference, enter a uniform
magnetic field the direction of which is perpendicular to their velocities. Find the ratio of radii of the circular
paths of the two particles. (JEE 2004)

8. In a moving coil galvanometer, torque on the coil can be expressed as b = ki, whre i is current through the
wire and k is constant. The rectangular coil of the galvanometer having numbers of turns N, area A and
moment of inertia I is placed in magnetic field B. Find
(a) k in terms of given parameters N, I, A and B.
(b) the torsional constant of the spring, if a current i0 produces a deflection of /2 in the coil in reaching
equilibrium position.
(c) the maximum angle through which coil is deflected, id charge Q is passed through the coil almost
instantaneously. (Ignore the damping in mechanical oscillations) (JEE 2005)

www.physicsashok.in 53
MAGNETIC FIELD

Answer Key
ASSERTION & REASON
Q. 1 2 3 4 5 6 7 8 9
Ans. A E A B A A D A B
Q. 10 11 12
Ans. E B C

Level # 1

Q. 1 2 3 4 5 6 7 8 9 10
Ans. C D C D ABCD C D B C C
Q. 11 12 13 14 15 16 17 18 19 20
Ans. B A D D C A D A D C
Q. 21 22 23 24 25 26 27 28 29 30
Ans. C C B BCD ACD ABC A BD C AD
Q. 31 32 33 34 35 36 37 38 39 40
Ans. A A C B ACD B B ABD A A
Q. 41 42 43 44 45 46 47 48 49 50
Ans. B C B D B C B A B B
Q. 51 52 53 54 55 56 57 58 59 60
Ans. A B A ABD C C ABD B AC D
Q. 61 62 63
Ans. AC A ACD

FILL IN THE BLANKS / TRUE-FALSE / MATCH TABLE


iL2 0 I 1 1
1. D, B 2. 3. 1.25 x 1023 Am 2 4.
4 4 R1 R2
5. lB ; +z direction 6. evB; ABCD 7. 14.0185 eV 8. True 9. True 10. False 11. False

12. [(APQ), (BQ), (CPQR), (DQS)] 13. [(APT), (BPQR), (CP), (DS)]
14. [(AQ), (BP), (CR), (DQ)] 15. [(AQ), (BR), (CS), (DP)]
16. [(AR), (BP), (CQ), (DQ)] 17. [(AQR), (BP), (CQR), (DQS)]
18. [(APRS), (BRS), (CPQT), (DRS)]

PASSAGE TYPES
Passage = 1.
1. C 2. A 3. C 4. A 5. D
Passage = 2.
1. D 2. C 3. D 4. A 5. C
Passage = 3.
Bqd m
1. Vm 2. t 3. R 3d.
2m 6Bq

www.physicsashok.in 54
MAGNETIC FIELD

Level # 2
2mv 0 BR 4
1. B0q 2.
4

0 q r 2 (2mv 0 )
3. (a) a
8 2rm
4 2 B V j i
0 0 (B) Zero,
4
1
B0
2
4. m ae 0 ei

n0 I
5. 0.2 Sec. 6. tan 7. B 4 0 / d sin 0.10 mT.
2R n
/ h (1 (h / 2R )2 0.3 mT, r R,
8. B 0 9. pm 2R 3B / 0 30 mA.m2
0 / 42 / r, r R.

10. pm 1/ 2 Nd2 0.5 A.m2 11. (a) B 1/ 2 0 R (b) pm 1/ 4 R 4 . 12. B mg / NS 0.4 T.


13. (a) F 2 0 0 / 42 1 0.40 N (b) A ( 0 a 0 / ) ln [2 1) /(2 1)] 0.10 J.

q 8 2 V
30.
14. p B / a 0.5 kPa. 15. arcsin dB 2mV 16. q / m
(B 2 B1 )2
2

a(a 2b)B 2
17. F = mE /qB = 20 N. 18. q / m
2Ex

Level # 3
eh ehB
1. (i) M (ii) directed perpendicular to the both M and B .
4m 8 m
2. (a) 6.54 x 105 Tesla (b) 0, Force on arc AC = 0, Force on segment CD = 8.1 106 N (inwards)
DT0
3. Wman 4. 1 x 104 T or Wb m 2. 5. = 5.9 x 106 Nm.
BQr 2

d 0
12
rp m p q 1
6. (a) x , z 0. (b) 7. .
3 2 d r m q p 2

2i 0 NAB NAB
8. (a) k = NAB, (b) C ; (c) Q
21i0

XXXX

www.physicsashok.in 55
ATOMIC PHYSICS

DHANALAKSHMI NAGAR

NEAR ANNAMAIAH CIRCLE,

TIRUPATI.

PH NO. 9440025125
ATOMIC PHYSICS

MASS OF A PHOTON
Rest mass of photon is zero.
Effective mass : If we assume that m is the effective mass of photons in moving conditions, then according
to Einsteins theory.
Energy of photon, E = mc2
E
m=
c2
but E = h = pc, p = photon momentum
E h p
m= 2 = 2 =
c c c

C1: Draw the graph of (a) momentum versus effective mass of photon (b) Energy versus effective mass,
(c) wavelength versus momentum, (d) Effective mass versus wavelength for a photon.
Sol:

p E


O m O m
(a) p = mc straight line tan = c (b) E = mc2 straight line tan = c2

p
h
(c)de-broglie wavelength of photon = (d) Effective mass of photon
p
E h hc
p = h Rectangular hyperbola m= 2 = 2 =
c c c 2
h
m = Rectangular hyperbola
c

Power of a light source


Suppose P = Power of the light source, = frequency of emitted photons,
Energy of single photon = h
Let, n = numbers of photons emitted per sec by the source, then
Thus, Energy emitted per sec. by the source = W = h

Intensity of light source at a point


The amount of energy incident on a point per unit area at that point in a unit time is called the intensity of light
at that point.

www.physicsashok.in 1
ATOMIC PHYSICS

Thus, intensity at a point is energy incident per unit area unit time.
Suppose, I = Intensity of light at a point
= Frequency of photon
n0 = no. of photon incident / sec / Area
I = n0h
SI unit of intensity is J/second m2.

Intensity of light at a distance r :


Let, W = Power of a point source
Amount of energy received by spherical surface per second P
= Amount of energy emitted by light source per second = W r
Hence, Intensity at distance
W W
r= =
Area 4r 2
W
Thus, I=
4r 2

Example 1: The intensity of sunlight on the surface of earth is 1400 W/m2. Assuming the mean wavelength of
sunlight to be 6000 , calculate the number of photons emitted from the sun per second assuming the
average radius of earths orbit around sun is 1.49 1011 m.
Sol: Average radius, r = 1.49 1011 m. intensity of sunlight received by earth = I = 1400 W/m2,
= mean wavelength = 6000 .
Energy emitted per second by the sum = Power of the sun = W
but, Power, W = nh, where n is number of photons emitted by sun per sec. .....(i)
Again, intensity at a distance r from a point source of power W.
W
I=
4r 2
nh
I= [From equation (i)] Sun Earth
4r 2 r
Power = P
I 4r 2 I 4r 2 hc
n= = [ h = ]
h hc
4r 2I 4 (1.49)2 1022 6 107 1400
n= =
hc 6.63 10 34 3 108
n = 1.18 1045 photons per sec.

Photon Flux (i.e. photon/sec)


Suppose, W = Power of a source.
If A is the area of the metal surface on which radiations are incident, then the power received by the plate is
W
W = IA = A
4r 2
If is the frequency of radiation, then the energy is photon is given by h.

www.physicsashok.in 2
ATOMIC PHYSICS

If n is the number of photons incident on the plate per second, then, we have
W = n h
W
A = N h
4r 2

W Area(A)
4r 2 A P
n = Plate
h

r
n is called photon flux.

Photon flow density (n0) :


The number of photons incident on unit area of the plate in one second is called the photon flow density.
n' W
i.e., Photon flow density, n0, = =
A 4 r 2 h

Photon-concentration :
The number of photons per unit volume of the space.
If n is the number of photons incident per unit area per second, then
n
Photon - concentration = , where c is the speed of light.
c

Example 2: The power of light emitted by the sun is 3.9 1026 W. Assuming the mean wavelength of sunlight to
be 6000 , calculate the photon flux arriving at 1 m2 area on earth perpendicular to light radiations. The
average radius of earths orbit around sun is 1.49 1011 m.
Sol: Power of light emitted by the sun, W = 3.9 1026 W
r = earth sun mean distance = 1.49 1011 m
= mean wavelength of sun light = 6000 = 6 107 m
W P Sun Earth
Intensity of light an earth, I = r
4r 2
Power received by area A on earth, W = I A
W
W = A
4r 2
but, W = n h, where n is the number of photon incident per sec or photon-flux.

W A
2
W' 4r
n = =
h h

WA
n = [h = hc/]
4r 2 hc
(3.9 10 26 ) (6 107 ) 1
n = photon /sec
4(1.49)2 1022 6.63 1034 3 108
= 4.22 1021 photon/sec
www.physicsashok.in 3
ATOMIC PHYSICS

PRESSURE EXERTED BY PHOTONS OR RADIATIONS PRESSURE


(a) Each photon carries energy and momentum. Hence when photons of light is incident on a surface, the light
is either absorbed or reflected or both is done by the surface.
(b) change in momentum of light takes place, which causes impulse or force on the surface resulting in a
pressure called radiation pressure.
h
Let light of intensity I is incident on a surface. Each photon carries energy h and momentum = .
c
Energy incident on unit area in unit time = I(by the definition of intensity).
Number of photons incident on unit area in unit time.
N I
i.e. = , .....(i)
At h
Momentum delivered to unit area in unit time,
P N
i.e., = (change in momentum of each photon) .....(ii)
At At
Using equation (i) & (ii), we get
P I
= (change in momentum of each photon)
At h
P Force exerted P
but, = , [ force exerted = ]
At A t
= Radiation pressure
I
Pr = Radiation pressure = (change in momentum of each photon) .....(iii)
h

Radiation pressure for perfectly absorbing surface :


h
In this case, change of momentum suffered by each photon = . hv
c c Perfectly
Using the equation (iii), we get absorbing
surface
I h I
Radiation pressure = =
h c c
I
Radiation pressure = for a perfectly absorbing surface.
c

Radiation pressure for a perfectly reflecting surface : hv


c Perfectly
h h 2h reflecting
In this case, change in momentum of each photon = = surface
c c c
hv
c
Hence, using the equation (iii), we get
I 2h 2I
Radiation pressure = =
h c c
2I
Radiation pressure = for a perfectly reflecting surface.
c
www.physicsashok.in 4
ATOMIC PHYSICS

Radiation pressure for a surface of reflection coefficient () :


h
In this case, Momentum of incident photon = hv
c c surface
h
Momentum of reflected photon = , = reflection coefficient
c hv
c
h h h
Change in momentum = = (1 + )
c c c
Thus, using equation (iii), we get,
I
Radiation pressure = (1 + ) for photons falling normally on a surface.
c

NOTE : (i) For a perfectly absorbing surface = 0


I
Radiation pressure =
c
(ii) For a perfectly reflecting surface = 1
I 2I
Radiation pressure = (1 + 1) =
c c

C2: A laser emits a light pulse of duration = 0.13 ms and energy E = 10J. Find the mean pressure exerted by
such a light pulse when it is focussed into a spot of diameter d = 10 m on a surface perpendicular to the
beam and possessing a reflection -coefficient = 0.50.
Sol: Laser energy, E = 10J
2
Plate area = d Laser d d = 10 m
4
Pulse duration, = 0.13 s
Pressure exerted by light pulse i.e.,
I
Radiation pressure = (1 + ) , by the definition of radiation pressure by photons of light .....(i)
c
Here, = reflection coefficient = 0.50
I = Intensity of light
c = speed of light = 3 108 m/s
Laser energy
By the definition of intensity of light, I = (pulse duration) (Area of plate)

E
i.e. I= .....(ii)
2
d
4 4E
From (i) & (ii) Radiation pressure = (1 + )
c d 2
4 10
= (1 + 0.50)
3 10 (0.13 103 )100 1012
8

= 4.9 107 Nm2 = 490 atm


www.physicsashok.in 5
ATOMIC PHYSICS

Example 3: 1 A current flows in an x-ray tube operated at 10,000 V. The target area is 104 m2. Find the
pressure on the target, assuming that the electrons strike the target normally and the photons leave the target
normally. Consider the ideal situation where each incident electron gives rise to a photon of the same energy.
Sol: Energy of each electron = energy of each photon = E = 104 eV
Momentum delivered by each electron = p1 = 2mE

E p1
Momentum taken away by each photon = p2 = e

c
p2
Change of momentum due to each electron-photon pair = p1 (p2) = p1 + p2 (photon)
Current incident on target = i = 106A
Number of electrons incident per second = i/e = no. of photons emitted per second. Plate

i
Total momentum change per second = force = (p1 + p2)
e
i
Pr, Pressure = force / area = (p + p2)
Ae 1
i E
= 2mE
Ae c

106 31 4 19 1/ 2 104 1.6 1019


Pr = (2 9.1 10 10 1.6 10 )
104 1.6 1019 3 108
6 2
= 3.7 10 Nm

FORCE EXERTED BY PHOTONS ON A SURFACE


We know that, when photons of light is incident on a surface, the change in momentum of photons takes
place resulting in a force exertion by the photons on the surface.
Let light of power W is incident on a surface.
h
Each photon carries energy h and, momentum = .....(i)
c
Energy incident in unit time = W(by the definition of power) Power = W Surface
But, power is given by
N
W = (h), where N = total no. of photons incident in time t.
t
N W
= .....(ii)
t h
Momentum delivered in unit time.
P N
= (change in momentum of each photon)
t t
P
= (change in momentum of each photon)
h
P
but, = rate of change of momentum
t

www.physicsashok.in 6
ATOMIC PHYSICS

P
Hence, = F = Force exerted by the photons.
t
P
Thus, force = (change in momentum of each photon) ...(iii)
h

Force exerted by photons on a perfectly absorbing surface :


h
In this case, change of momentum suffered by each photon =
c
using equation (iii), we get
hv
c Perfectly
W h W absorbing
force = =
h c c surface

W
thus, F= for a perfectly absorbing surface.
c

Force exerted by photons on a perfectly reflecting surface:


In this case, change of momentum suffered by
h h 2h hv
c surface
each photon = =
c c c
using equation ( iii), we get
hv
c
W 2h 2W
Force = =
h c c
2P
thus, F = , for a perfectly reflecting surface.
c

Force exerted by photons on a surface of reflection coefficient :


h hv
In this case momentum of incident photon = , c surface
c
h
Momentum of reflected photon = hv
c c

h h h
Change in momentum = = (1 + )
c c c
Thus, using equation (iii), we get
h W
Force = (1 + ) = (1 + )
h c c
W
i.e. F = (1 + ) for photons falling normally on a surface.
c

www.physicsashok.in 7
ATOMIC PHYSICS

C3: If a point source of light of power W is placed at the centre of curvature of W


a hemispherical surface, whose inner surface is completely reflecting, then (Source)
W
the force on the hemisphere due to the light falling on it is given by F = .
2c
C4: If a perfectly reflecting solid sphere of radius r is kept in the path of a
parallel beam of light of large aperture and having an intensity I, then the
I
r 2 I
force exerted by the beam on the sphere is given F = . r
c
Note that force is equal to the product of (I/c) and the projected area of
the sphere.
C5: If photon of light of power W falls at an angle to a perfectly reflecting

2W
surface, then net force exerted on the plate is given by Fnet = cos.
c

C6: A laser emits a light pluse of duration T = 0.10 ms and energy E = 10 J. Find the mean pressure exerted by
such a pulse when it is focused on a spot of diameter d = 10 m on a surface perpendicular to the beam and
with a reflection coefficient = 0.5.
Sol. p, momentum of a photon = hv/c = E/c
Momentum of reflected photon = (E/c)
change of momentum = (E/c) (E/c) = (1 + )E/c
Force exerted = [(1 + ) E/c]/T
Pressure = 4 [(1 + ) E/c]/Td2
4(1 )E 4(1 0.5) 10
=
cTd 2
3 108 104 1010
= 6.37 106 Nm2

C7: A plane light wave of intensity I = 0.50 W cm2 falls on a plane mirror of reflection coefficient = 0.8. The
angle of incidence = 45. Find the normal presure exerted by light on that surface.
Sol. If S is the area of the mirror on which light falls, the transverse section of the incident beam is S cos .
Momentum of the incident photons = (I/c)(S cos ). Normal component of momentum
flux = (IS cos /c) cos = IS cos 2/c.
Momentum of reflected photons = (IS cos2/c) in the opposite direction.
rate of change of momentum = force = (1 + ) IS cos2/c
normal pressure = force/area = (1 + ) I cos2/c
required pressure
= (1 + 0.5) (0.5 104 cos2 45)/3 108
= 1.25 105 Nm2

Example 4: An isotropic point source of radiation power P is placed on the axis of an ideal mirror. The distance
between the source and the mirror is n times the radius of the mirro. Find the force that light exerts on the
mirror.

www.physicsashok.in 8
ATOMIC PHYSICS

Sol. Energy flux through the annular space between two cones of half-angles A
and + d
= (P/4) (2 sin d) d R
momentum of flux = P sin d/2c
S B
rate of change of momentum along the normal. nR
2(P sin d/2c) cos = P sin cos d/c
p
c 0
force on mirror = sin cos d B

where = half-angle of the cone subtened by the mirror = tan1(1/n)


F = P sin2 / 2c = P/2(n2 + 1)c

Example 5: Figure shows a small, plane strip of mass m suspended from a fixed support through a string. A
continuous beam of monochromatic light is incident horizontally on the strip and is completely absorbed.
The energy falling on the strip per unit time is W. find the deflection of the string from the vertical, if the strip
stays in equilibrium.
Sol: Force exerted by the photons of light
= (number of incident photons per sec) (change in momentum of each photon)
Let, Number of incident photons per sec = N
h
change in momentum of each photon =
c
hence force exerted by the photons of light, Light

h (Nh)
F = N =
c c
but, W = Nhv, by the definition of power
W
F =
c
If the string makes an angle with the vertical when the strip is at equilibrium, then for the equilibrium of the
strip,
Tsin = F, in horizontal direction,
and Tcos = mg, in vertical direction. T = tension
Dividing the above equations, we get in the string

F W/c W
T
tan = = = , F m
mg mg c mg

W W
tan = = tan1 mg
c mg c mg

Example 6. A plane light wave of intensity I = 0.80 W cm2 illuminates a sphere of radius R = 5.0 cm. Find the
force that the light exerts on the sphere.
Sol. p, momentum of the incident pulse = E/c.

p (E / c) (sin i cos j) with respect to a frame of reference with the outward normal as the y-axis
i
and a line perpendicular to it and lying in the plane of the mirror as the x-axis.

www.physicsashok.in 9
ATOMIC PHYSICS

pf = E/c and pf (E / c) (sin i cos j)

p p f pi ( 1)E / c sin i ( 1) cos E / c j

| p | E / c ( 1)2 sin 2 ( 1)2 cos 2

E / c 1 2 2 cos 2


| p | 10 / 3 108 1 0.82 2 0.8 cos 60
= 5.2 108 Nm2

Impulse applied by photon on a surface


Let hn be the energy of photons of a light incident normally on a surface,
h
Momentum of an incident photon = .
c
Change in momentum of the photon takes place due to impact of the photons with the surface. This change
in momentum of the photons causes impulse.
Using impulse momentum theorem, we get
Impulse = Total change in momentum of photons
= (Total number of photons) (change in momentum of each photon)
Impulse = N (change in momentum of each photon) ......(i)

Impulse on a totally absorbing surface:


h
In this case, change in momentum of each photon =
c
using equation (i), we get
N ' h E
Impulse = = , where E is the total energy of the light
c c
E
Impulse = for a perfectly absorbing surface.
c

Impulse on a totally reflecting surface:


Impulse = N (change in momentum of each photon)
h h 2(N ' h) 2E
= N = = [ E = total energy = Nh]
c c c c
2E
Impulse = for a perfectly reflecting surface.
c

Impulse on a surface of reflection-coefficient :


h h h
In this case, change in momentum of each photon = = (1 + ) .
c c c
Using equation (i), we get

www.physicsashok.in 10
ATOMIC PHYSICS

h N ' h E
Impulse = N(1 + ) = (1 + ) = (1 + ) , [E = total energy]
c c c
E
Impulse = (1 + ) for photons falling normally on the surface.
c

Example 7: A small perfectly reflecting mirror of mass m = 10 mg is


suspended by a weightless thread of length = 10 cm as shown in the
figure. Find the angle through which the thread will be deflected when
a short laser with energy E = 13J is shot in horizontal direction at right m
Laser
angles to the mirror. (g = 10 m/s2).
m
2E
Sol: Impulse applies by laser on the mirror, Impulse = , as mirror is perfectly
c
reflecting. Initial momentum of the mirror = 0
Let, final momentum of the mirror = mv
change in momentum of the mirror due to impact = 0 (mv) = mv, where
v is the speed of mirror just after impact.
But, Impulse momentum theorem gives,
Impulse = change in momentum
2E 2E v=0 (1 cos) = H
= mv or v= .....(i)
c mc
Total mechanical energy of the mirror will be conserved after impact. v
Thus, Loss in kinetic energy = Gain in potential energy
1 2
mv = mgh v2 = 2gH
2
4E 2 2E
= 2gH, [as v = from equation (i)]
m 2c2 mc
2E 2 2E 2
= gH = g(1 cos), [ H = (1 cos)]
m 2c2 m 2c2
2E 2
1 cos = 2 2
m c g


2
2E 2
2sin = 2 2 [ 1 cos = 2sin2 ]
2 m c g 2

E2
sin = Here E = 13J, m = 10 106 kg, = 0.1 m
2 mc g

13
sin = 5 = 0.0043 or = 0.5
2 10 3 108 10 0.1

PHOTONS UNDER GRAVITY

www.physicsashok.in 11
ATOMIC PHYSICS
Photons can be considered as a particle of mass m.
If m = mass of photon
v = frequency of photon
E = energy of photon, then
E h
mc2 = E = hv m = 2 =
c c2
h
Thus, a photon of frequency v acts gravitationally like a particle of mass .
c2

C8: If a photon frequency v falls on the surface of earth from a height h, then what will be its frequency on the
surface of earth.
Sol: Change in frequency of the photon takes place. v, E = hv + mgH
Let v be the frequency of photon on the surface of earth.
H
h v, E = hv
Mass of photon = 2
c Ground
Mass of the photon depends on its frequency, but we will consider the mass to be constant as difference in
v and v is very small.
Energy conservation gives
Initial energy = final energy
hv + mgH = hv
h
hv + gH = hv
c2
gH
v = v 1 2
c

Example 8: A planet of mass M and radius R emits a photon of frequency v. What will be the frequency of
photon when it covers an infinite distance.
Sol: Let vbe the frequency photon at a infinite distance.
G = Gravitational constant v v
R at
h
Mass of the photon, m = 2 M
c
Mm GM h
Energy of photon on the surface = hv G = hv
R R c2
Energy of photon at infinity = hv
Energy must be conserved,
GM h GM
hv = hv v = v 1 2
R c2 Rc

GM
NOTE: = is called frequency shift.
Rc 2

PHOTOELECTRIC EFFECT
www.physicsashok.in 12
ATOMIC PHYSICS

Ejection of electrons from a metal plate when illuminated by light or any other radiation of suitable wavelength
(or frequency) is called photoelectric effect.
This phenomenon was first discovered by Heinrich Hertz in 1887. One year later, Hallwachs made the
important observation that when a negatively-charged zinc plate is irradiated with ultra-violet light, it loses
its negative charge. Afterwards, it was discovered that alkali metals like lithium, sodium, potassium, rubidium
and caesium eject electrons when visible light falls on them. Ten years later, J.J. Thomson and P.Lenard
showed that the action of light was to cause the emission of free electrons from metal surface. Although
these electrons are no different from all other electrons, it is customary to refer to them as photoelectrons.

Experimental Study of Photoelectric Effect


Photoelectric phenomenon can be studied with the help of a simple Quartz
apparatus shown in figure. It consists of two photosensitive surfaces C +
E and C enclosed in an evacuated quartz bulb. In the absence of E
light, there is no flow of current in the circuit and the ammeter A
A
reads zero. When plate E is exposed to some source of
monochromatic light, current starts flowing. However, no current V
is found to flow when light falls on plate C.
The explanation of the above behaviour lies in the fact that when E is irradiated with light, the incident
photons eject electrons by collision with its atoms. These photoelectrons are immediately attracted by the
collector plate C thereby starting current flow as indicated by the ammeter.
When C is irradiated, even then photoelectrons are produced but they are not allowed to leave plate C
(i) firstly, because of the pulling effect of positive potential of C and
(ii) secondly, due to repulsion from the negative plate E.
Hence, no current is found to flow in the circuit.

Einsteins Photoelectric Equation:


Following Plancks idea that light consists of photons, Einstein proposed an explanation of photoelectric
effect as early as 1905. According to this explanation when a single photon is incident on a metal surface, it
is completely absorbed an imparts its energy hf to a single electron. The photon energy is utilised for two
purposes:
(i) Partly for getting the electron free from the atom and away from the metal surface. This energy is known
as the photoelectric work function of the metal and is represented by W0.
(ii) the balance of the photon energy is used up in giving the electron a kinetic energy of 1/2mv2.
hf = W0 + 1/2mv2 .....(i)
It is known as Einsteins photoelectric equation.
In case, the photon energy is just sufficient to liberate the electron only then no energy would be available
for imparting kinetic energy to the electron. Hence, the above equation would reduce to
hf0 = W0 .....(ii)
where f0 is called the threshold frequency. It is defined as the minimum frequency which can cause photoelectric
emission. For frequencies lower than f0, there would be no emission of electrons whereas for frequencies
greater than f0, electrons would be ejected with a certain definite velocity (and hence kinetic energy).
Substituting this value of W0 in equation (i) above, the Einsteins photoelectric equation becomes
1
hf = hf0 + mv2
2
or hf = hf0 + K.E.

Long Wavelength Limit ( 0)


www.physicsashok.in 13
ATOMIC PHYSICS

It is the wavelength corresponding to the threshold frequency f0. Its physical significance is that radiations
having wavelength longer than 0 would not be able to eject electrons from a given material whereas those
having < 0 will. In other words, it represents the upper limit of wavelength for photoelectric phenomenon.
By analogy, it is also referred to as threshold wavelength.
c
Now, c = f0 0 0 =
f0

1 h
Also, W0 = hf0 =
f0 W0
ch
0 = W
0

(i) When W0 is in joules


3 108 6.625 1034
0 =
W0

19.875 1026
= metre
W0
(ii) When W0 is electron volts (eV)
3 108 6.625 1034
0 =
1.602 1019 W0

12.4 107 12, 400


= metre = W
W0 0

Kinetic energy of Photoelectrons


Einsteins photoelectric equation can be used to find the velocity and hence the kinetic energy of an ejected
photo-electron.
1
hf = W0 + mv2
2
= hf0 + K.E.
K.E. = hf hf0 = h(f f0)
c c
Now, f = and f0 =
0

1 1
K.E. = ch
0

1 1
= 3 108 6.625 1034
0

1 1
= 19.875 1026 joules and 0 in metres
0

1 1
= 19.875 1026 joules and 0 in
0
www.physicsashok.in 14
ATOMIC PHYSICS

19.875 1016 1 1
= eV and 0 in
1.602 1019 0

1 1
K.E. = 12,400 eV and 0 in
0

Incidentally, it may be noted that this also represents the maximum value of the kinetic energy a photoelectron
can have.
Emass = h(f f0) = h.f joules
1 1
= 12,400 eV and 0 in
0

Photoelectric Work Function


As explained above it is defined as the energy which is just sufficient to liberate electrons from a body with
zero velocity. Its value is given by
ch 3 108 6.625 1034
W0 = hf0 = =
0 0

19.875 1026
= joules 0 in metres
0

19.875 1016
= joules 0 in
0

19.875 1016
= eV 0 in
1.602 1019 0
12, 400
W0 = 0 eV 0 in

Laws of Photoelectric Emission

Quartz
A
C
B
V R

The apparatus shown in figure was used by Millikan to study the photoelectric effect and the various laws
governing it. S is a source of radiations of variable but known frequency f and intensity I. E is the emitting
electrode made of the photosensitive material and C is the collecting electrode. Both these electrodes are
enclosed in an evacuated glass envelope with a quartz window that permits the passage of ultraviolet and

www.physicsashok.in 15
ATOMIC PHYSICS

visible light. As shown, any potential difference can be established between the two electrodes. A reversing
switch RS enables the polarities of the two electrodes to be reversed. If the tube is in the dark, then no
photoelectrons are emitted and the microammeter A read zero. However, if ultraviolet or visible light is
allowed to fall on the emitting electrode, electrons are liberated and circuit current is set up.
From the experimental data collected by Richardson and Compton in 1912, photoelectric emission was
found to be governed by the following laws:
(i) Photoelectric current (i.e., number of electrons emitted per second) is directly proportional to the
intensity of the incident light (or radiation).
This can be verified by increasing the intensity of light and Frequency

Photoelectric
measuring the corresponding photoelectric current while holding constant

current
the frequency of the incident light frequency of the incident light
and the accelerating potential V of the collecting electrode C
constant. The graph is similar to one shown in figure. Increase in
intensity means more photons and hence greater ejection of
electrons. Light Intensity I
(ii) For each photosensitive surface, there is a minimum frequency of
radiation (called threshold frequency) at which emission begins.

Photoelectric
Light of lower frequency (or longer wavelength), however strong, Intensity
constant
will not be able to produce any electron emission. This fact can be

current
verified by keeping the light intensity constant while varying the
frequency. The graph so obtained is shown in figure. The current is
found to increase (though non-linearly) with the frequency of the
incident light.
f0
Moreover, it is seen that there is a limiting or critical frequency below Frequency
which no photoelectons are emitted. It is called threshold frequency
and its value depends on the nature of the material irradiated because for each material there is a certain
minimum energy nessecary to liberate an electron. This energy is known as photoelectric work function or
threshold energy W0. As seen from f0 = W0/h.
The wavelength corresponding to be threshold frequency f0 is called threshold wavelength or long wavelength
limit. No photo-electrons are emitted for wavelength greater than 0, no matter how long the light falls on
the surface or how greater is its intensity. The photoelectric or quantum yield (which is defined as the photo-
electric current in amperes per watt of incident light) depends on the frequency (and not intensity) of the
incident light.
(iii) The maximum velocity of electron emission (and hence kinetic energy) varies Intensity
constant
linearly with the frequency o the incident light but is independent of its intensity.
Emax

As seen from Einsteins photoelectric equation of


1
mv 2max = h(f f0) or Emax f
2 f0
Frequency
Hence, increase in the frequency of the incident light increase the velocity with which photoelectrons are
ejected. The same fact is illustrated by figure. Incidentally, it may be noted that the slope of the curve gives
the value of Plancks constant h.
If the intensity of the incident light is increased (keeping frequency constant), more photons will be incident

www.physicsashok.in 16
ATOMIC PHYSICS

on the metal surface, each photon having the same energy. Hence, more photo electrons will be ejected.
Since an electron can absorb only one photon, each photoelectron will have the same energy and will be
ejected with the same velocity. Obviously, increase in intensity only increases the number of photo-electrons
ejected but not their kinetic energy.
(iv) The velocities of emitted electrons have values between zero and a definite maximum. The proportion
of the electrons having a particular velocity is independent of the intensity of radiation.
(v) Electrons are emitted almost instantaneously even when the intensity of incident radiations is very low.
The time lag between the incidence of radiation and emission of first electrons is less than 108 second.
(vi) For a given metal surface, stopping potential V0 is directly proportional to frequency but is independent
of the intensity of the incident light.
Suppose in figure, the frequency and intensity of incident light are i
held constant but the potential difference V between the two H
electrodes E and C is increased. Up to some stage as this p.d. is b
a I
increased photoelectric current is also increased. However, soon
some value of V is reached when the current reaches a limiting or
saturation value when al the photoelectrons emitted by E are
immediately collected by C. Further increase in V hardly produces v0 O v
any appreciable increase in current as shown by the flat portion of
curve I in figure.
If V is reversed with the help of the reversing switch RS i.e. E is made positive and C negative, the current
i does not immediately drop to zero proving that electrons are emitted from E with some definite velocity.
This velocity is such that it gives enough kinetic energy to the electrons so as to surmount the retarding
electric field between the two electrodes. Hence, some electrons do succeed in reaching C despite the fact
that the electric field opposes their motion.
When reversed V is made large enough, a value V0 (called stopping or inhibiting potential) is reached when
current is reduced to zero.
Stopping potential is that value of the retarding potential difference between the two electrodes which is just
sufficient to half the most energetic photoelectrons emitted.
As seen from curve II of figure, doubling the intensity of the incident light merely doubles the current but
does not affect the value of V0.
Now, if vmax is the maximum velocity of emission of a photoelectron and V0 the stopping potential, then
1 2.eV0
mv 2max = eV0 or vmax =
2 m

or vmax = 2 1.76 1011 V0

= 5.93 105 V0 m/s


Obviously Emax = eV0 joules
= V0 electron-volt
If however, the experiment is repeated by varying the frequency of the light, it is found that the stopping
potential varies linearly with frequency as shown in figure. Below threshold frequency, no electrons are
emitted, hence stopping potential is zero for that reason. But as frequency is increased above f0, the stopping
potential varies linearly with the frequency of incident light.

www.physicsashok.in 17
ATOMIC PHYSICS

Einsteins photoelectric equation may be expressed in terms of stopping potential as given below:
1
hf = W0 + mv 2max

Stopping potential, V0
2
1
Now, W 0 = hf0 and mv 2max = eV0
2
h(f f 0 )
hf = hf0 + eV0 or V0 =
e
c c f0
Now, f = and f0 = Frequency
0

ch 1 1
V0 =
e 0

1 1
= 12.4 107 volt and 0 in metres
0

1 1
V0 = 12,400 volt and 0 in
0

C9: Photoelectrons with a maximum speed of 7 105 m/sec are emitted from a metal surface when light of
frequency 8 1014 Hz falls on it. Calyculate the threshold frequency of the surface.
Sol: Emax = h(f f0)
1
mv 2max = h(f f0)
2
1
or 9.1 1031 (7 105)2 = 6.625 1034 (8 1014 f0)
2
f0 = 4.635 1014 Hz

C10: A tungsten cathode whose threshold wavelength 2300 is irradiated by ultraviolet light of wavelength
1800 . Calculate (i) the maximum energy of the photoelectrons emitted and (ii) the work function for
tungsten, both in electron-volts.
12, 400 12, 400
Sol: (i) W0 = 0 = = 5.4 eV
2300
1 1
(ii) Emax = 12,400 eV
0

1 1
= 12,400 = 1.5 eV
1800 2300

C11: If light of = 6000 falls on a metal surface and emits photoelectrons with a velocity of 4 105 m/s, what
is photoelectric threshold wavelength ?

www.physicsashok.in 18
ATOMIC PHYSICS

1 31
(4 105 )2
Sol: K.E. of photoelectrons = 9.1 10 = 0.445 eV
2 1.6 1019
12, 000
Energy content of photon of = 6000 = = 2.07 eV
6000
W0 = 2.07 0.445 = 1.625 eV
12, 000
0 = = 7631
1.625

C12: Calculate the threshold frequency for gold having photoelectric work function equal to 4.8 eV. If light of
wavelength 2220 falls on gold, what will be maximum kinetic energy of the photoelectrons coming out?
12, 000
Sol: Energy of the light photon = = 5.58 eV. Out of this, 4.8 eV would be used for dislodging the
2220
electron and the balance would represent its kinetic energy.
Emax = 5.58 4.8 = 0.78 eV
12, 400
Alternatively, 0 = = 2583 . Hence, we may use
4.8
1 1
Emax = 12,400 eV..
0

C13: When violet light of = 4000 Au strikes the cathode of a photocell a retarding potential of 0.4 V is
required to stop emission of electrons. Calculate (i) light frequency (ii) photon energy (iii) work function (iv)
threshold frequency and (v) net energy after the electron leaves the surface.
c 3 108
Sol: (i) f = =
4000 1010
= 7.5 1014 Hz
(ii) E = hf = 6.625 1034 7.5 1014
= 4.95 1019 J = 3.1 eV
(iii) W0 = hf K.E. = hf V0 = 3.1 0.4 = 2.7 eV

W0 2.7 1.6 1019


(iv) f0 = =
h 6.625 1034
= 6.5 1014 Hz
(v) Net energy hf W0 = 3.1 2.7 = 0.4 eV = 6.4 1020 J

Example 9: A photon of wavelength 3310 falling on a photo cathode ejects an electron of energy 3 1019
J and one of wavelength 5000 ejects an electron of energy 0.972 1019 J. Calculate the value of
Plancks constant and the threshold wavelength for the photo cathode.
hc
Sol: hf = W0 + K.E. or = W0 + K.E.

www.physicsashok.in 19
ATOMIC PHYSICS

h 3 108
In the first case, = W0 + 3 1019
3310 1010

h 3 108
In the second case, = W0 + 0.972 1019
5000 1010
Subtracting one from the other,
h = 6.62 1034 J-s
Substituting this value of h, W0 = 3 1019 J
ch 3 108 6.62 10 34
0 = W =
0 3 1019
= 6620 1010 m.
vmax = 7.12 105 m/s

Example 10: A certain metal has a threshold wavelength of 6525 . Find the stopping potential when the metal
is irradiated with
(a) monochromatic light having a wavelength of 4000 .
(b) light having twice the frequency and three times the intensity of that in (a) above.
(c) If a material having double the work function were used, what would be the answer to (a) and (b)
above?
1 1
Sol: (a) V0 = 12,400 = 1.2 volt
4000 6525
(b) Stopping potential is independent of the intensity of the incident light but varies directly as frequency f
provided it is more than f0.
Since frequency is twice, the wavelength of the light is half i.e. 2000
1 1
V0 = 12,400 = 4.3 V
2000 6525
6525
(c) If work function is double, then 0 is reduced to half i.e. 0 = = 3262.5 . Since the incident light
2
has = 4000 , it would not be able to produce photoemission.
In the second case, = 2000 .
1 1
V0 = 12,400 = 2.4 volt
2000 3262.5

C14: A certain metallic surface is illuminated by monochromatic light of variable wavelength. No photoelectrons
are emitted above a wavelength of 5000 . With an unknown wavelength, a stopping potential of 3.1 V is
necessary to stop photoelectric current. Find the unknown wavelength.
Sol: Here, 0 = 5000 , V0 = 3.1 V, = ?
1 1
Now, V0 = 12,400
0

1 1
3.1 = 12,400
5000
= 2,222
www.physicsashok.in 20
ATOMIC PHYSICS

C15: Light of wavelength 2000 falls on an aluminium surface. In aluminium, 4.2 eV are required to remove an
electron. Determine (i) KE of the fastest emitted photo-electron (ii) KE of the slowest emitted photo-
electron (iii) stopping potential and (iv) cut-off wavelength for aluminium.
Sol: Photon energy of the incident light is
12400
= = 6.2 eV
2000
(i) Emax = (6.2 4.2) eV
= 2 eV = 2 1.6 1019 = 3.2 1019 J
(ii) Emin = 0
(iii) Ve = Emax = 2 V
12400 12400
(iv) 0 = W0 = 4.2 = 2952.4

C16: The stopping potential is 4.6 V for light of frequency 2 1015 Hz. When light of frequency 4 1015 Hz is
used, the stopping potential is 12.9 V. Calculate the value of Plancks constant.
Sol: eV0 = h(f f0)
Substituting the two given values, we get
4.6e = h (2 1015 f0)
12.9e = h(4 1015 f0)
Subtracting one from the other, we have
8.3e = 2h 1015
8.3 1.6 1019 = 2h 1015
8.3 1.6 10 19
h = = 6.44 1034 Js
2 1015

Example 11. 103 W of 5000 light is directed on a photoelectric cell. If the current in the cell is 0.16 A, the
percentage of incident photons which produce photoelectrons, is
(A) 0.4% (B) .04% (C) 20% (D) 10%
Sol. The percentage of incident photons which produce photoelectrons is
n / t
= 100 ...(1)
N / t
ne
I
t
n I 0.16 1016

t e 1.6 1019
n
1012 ...(2)
t
N hc N W 1016
and W ...(3)
t t hc 4
1012
Persentage = 16 4 100 [from eq. (1), (2) and (3)]
10
Persentage 0.04% Hence (B) is correct.

www.physicsashok.in 21
ATOMIC PHYSICS

Example 12. In a photo-emissive cell, with exciting wavelength , the maximum kinetic energy of electron is K.
3
If the exciting wavelength is changed to the the kinetic energy of the fastest emitted electron will be :
4
(A) 3K/4 (B) 4K/3 (C) less than 4K/3 (D) greater than 4K/3
hc
Sol. K ...(1)

hc
K ...(2)
3 / 4
Substracte eqn. (1) from eq. (2)
4hc hc
KK
3
4hc 3hc
K K
3
hc
K K
3
K
K K
3
K
K K
3
4K
K Hence (D) is correct.
3

Example 13. Let K1 be the maximum kinetic energy of photoelectrons emitted by a light of wavelength 1 and
K2 corresponding to 2. If 1= 22, then :
K2
(A) 2K1 = K2 (B) K1 = 2K2 (C) K1 < (D) K1 > 2K2
2
P2
Sol. K.E. =
2m
P 2mK
P K
h
K

1

K

1 K2

2 K1

www.physicsashok.in 22
ATOMIC PHYSICS

1 = 22
2 2 K2

2 K1

K2
4
K1
K2
K1
4
K2
K1 Hence (C) is correct.
2

Example 14. Radiation of two photon energies twice and five times the work function of metal are incident
sucessively on the metal surface. The ratio of the maximum velocity of photoelectrons emitted is the two
cases will be
(A) 1 : 2 (B) 2 : 1 (C) 1 : 4 (D) 4 : 1
Sol. E1 = 2
E2 = 5
E1 = + K1
2 = + K1
K1 =
and E2 = + K2
5 = + K2
K2 = 4
K1 1

K2 4

1
mv 2max 1 1
2
1
mv max 2 4
2

v 2max1 1
2

v max 2 4

v max 1 1
Hence (A) is correct.
v max 2 2

Example 15. When photons of energy 4.25 eV strike the surface of a metal A, the ejected photoelectrons have
maximum kinetic energy Ta eV and de-Broglie wavelength a. the maximum kinetic energy of photoelectrons
liberated from another metal B by photones of energy 4.7 eV is Tb = (Ta 1.5) eV. If the De-Broglie
wavelength of these photoelectrons is b = 2a, then find
(a) The work function of a (b) The work function of b is (c) Ta and Tb
www.physicsashok.in 23
ATOMIC PHYSICS

1
Sol. h A mv 2Amax
2
h
and a
PA
h
PA
a

1 P2
mv 2A max A where m is mass of electron.
2 2m
PA2
h A
2m
h2
or 4.25 = A ...(1)
2m 2a
For B,
4.7 = B + Tb
h2
4.7 B ...(2)
2m 2b
But b = 2a
and Tb = Ta 1.5
After solving
(a) 2.25 eV (b) 4.2 eV (c) 2 eV and 0.5 eV

C17: An isolated metal body is illuminated with monochromatic light and is observed to become charged to a
steady positive potential 1.0 V with respect to the surrounding. The work function of the metal is 3.0 eV. the
frequency of the incident light is __________.
Sol. h = + eV
or h = 3 + 1 = 4 eV
4 eV 4 1.6 10 19

h 6.63 1034
= 0.96 1015 Hz

Example 16. 663 mW of light from a 540 nm source is incident on the surface of a metal. If only 1 of each 5
109 incident photons is absorbed and causes an electron to be ejected from the surface, the total photocurrent
in the circuit is _______.
N
Sol. = no. of photon incident per second.
t
N hc
663 103
t

www.physicsashok.in 24
ATOMIC PHYSICS

N 663 103 663 103



t hc 1242 nmeV
540
n / t 1

N / t 5 109
n 1 N 1 663 103 540

t 5 109 t 5 109 1242 nmeV
ne
I 5.76 1011 A
t

Example 17. Light of wavelength 330 nm falling on a piece of metal ejects electrons with sufficient energy which
requires voltage V0 to prevent a electron from reading collector. In the same setup, light of wavelength 220
nm, ejects electrons which require twich the voltage V0 stop them in reaching a collector. Find the numerical
value of voltage V 0. (take planks constant, h = 6.6 1034 Js and 1 eV = 1.6 1019 J)
hc
Sol. eV0
1
hc
and 2eV0
2
Here 1 = 330 nm
and 2 = 220 nm
After solving,
15
V0 volt
8

Example 18. A small 10W source of ultraviolet light of wavelength 99 nm is held at a distance 0.1 m from a
metal surface. The radius of an atom of the metal is approximately 0.05 nm. Find
(i) the average number of photons striking an atom per second.
(ii) the number of photoelectrons emitted per unit area per second if the efficiency of liberation of photoelectrons is
1%.
10
Sol. (i) I = intensity =
4(0.1) 2
w = power incident on atom
10 2
2
= I r
4 10 2
0.05 10 9
n hc
w
t
n w 5

t hc / 16

www.physicsashok.in 25
ATOMIC PHYSICS

(ii) no. of photons incident per unit area per second


I

hc /
no. of ejected electrons per unit area per second
I 1 1020

hc / 100 80

supporting potential (volt)


Example 19. The surface of cesium is illuminated with monochromatic light of 2 0.49
various wavelengths and the stopping potentials for the wavelengths are 1
measured. The results of this experiment is plotted as shown in the figure. 0 v 10 15 Hz
1 0.5 1.0 1.5
estimate the value of work function of the cesium and Plancks constant.
2
Sol. h = + eVs
h
or Vs
e e
From graph
h 2

e 0.49 1015
2
h 15
1.6 1019
0.49 10
h = 6.53 1034 Js

But 2
e
= 2 eV

Example 20. In a photoelectric effect set-up a point source of light of power 3.2 103 W emits monoenergetic
photons of energy 5.0 eV. The source is located at a distance of 0.8 m from the centre of a stationary
metallic sphere of work function 3.0 eV and of radius 8.0 103 m. The efficiency of photoelectron
emission is one for every 106 incident photons. Assume that the sphere is isolated and initially neutral and
that photoelectrons are instantly swept away after emission.
(a) Calculate the number of photoelectrons emitted per second.
(b) Find the ratio of the wavelength of incident light to the de-Broglie wavelength of the fastest photoelectrons
emitted.
(c) It is observed that the photoelectron emission stops at a certain time t after the light source is switched on
why?
(d) Evaluate the time t.
Sol. (a) Energy of emitted photons
E1 = 5.0 eV = 5.0 1.6 1019 J
E1 = 8.0 1019 J
Power of the point source is 3.2 103 watt or 3.2 103 J/s
s 0.8 m r = 8.0103 m
Therefore, energy emitted per second,
E2 = 3.2 103 J.
Hence number of photons emitted per second

www.physicsashok.in 26
ATOMIC PHYSICS

E2 3.2 103
n1 or n1
E1 8.0 1019
n1 = 4.0 1015 photons/sec.
Number of photons incident on unit area at a distance of 8.0 m from the source S will be
n1 4.0 1015
n2 = = 5.0 1014 photon/sec m2.
4 (0.8)2 4(0.64)
The area of metallic sphere over which photons will fall is :
A = r2 = (8 103)2 m2 2.01 104 m2
Therefore, number of photons incident on the sphere per second are
n3 = n2 A = (5.0 1014 2.01 104) 10111 per second
But since one photoelectron is emitted for every 106 photons hence number of photoelectrons emitted per
second,
n3 1011
n = 6 = 6 = 105 per second
10 10
5
or n = 10 per second
(b) Maximum kinetic energy of photoelectrons
Kmax = Energy of incident photones work function
Kmax = (5.0 3.0) eV = 2.0 eV = 2.0 1.6 1019 J
Kmax = 3.2 1019 J
The de-Broglie wavelength of these photoelectrons will be
h h
1
p 2 K max m
(Here h = Plancks constant and m = mass of electron)
6.63 10 34
1
2 3.2 10 19 9.1 10 31
1 = 8.681010 m = 8.68
12375
Wavelength of incident light 2 (in ) = E (in eV)
1

12375
or 2 = = 2476
5
Therefore, the desired ratio is
2 2475
285.1
1 8.68
(c) As soon as electrons are emitted from the metal sphere, it gets positively charged and acquires positive
potential. The positive potential gradually increases as more and more photoelectrons are emitted from its
surface. Emission of photoelectrons is stopped when its potential is equal to the stopping potential required
for fastest moving electrons.
(b) As discussed in part (c), emission of photoelectrons is stopped when potential on the metal sphere is
equal to the stoppeing potential of fastest moving electrons.
Since Kmax = 2.0 eV

www.physicsashok.in 27
ATOMIC PHYSICS

Therefore, stopping potential V0 = 2 volt. Let q be the charge required for the potential on the sphere to be
equal to stopping potential or 2 volt. Then
1 q 9
2
4 0 r

. 9.0 109
8.0 103
q = 1.78 1012 C
Photoelectrons emitted per second = 105 [part a]
19 5
or charge emitted per second = (1.6 10 ) 10 C
= (1.6 1014) C
Therefore, time required to acquire to charge q will be
q 1.728 102
t sec sec
1.6 1014 1.6
or t 111 second

Example 21. Monochromatic radiation of wavelength 1 = 3000 falls on a photocell operating in saturation
mode. The corresponding spectral sensitivity of photocell is J = 4.8 mA/W. When another monochromatic
radiation of wavelength 2 = 1650 and power P = 5 mW is incident. It is found that maximum velocity of
photoelectrons increases to n = 2 times. Assuming efficiency of photo-electron generation per incident
photon to be same for both the cases, calculate
(i) threshold wavelength for the cell and
(ii) saturation current in second case.
[Given, h = 6.6 1034 Js, c = 3 108 ms1 and e = 1.6 1019 coul.]
hc
Sol. Maximum kinetic energy of photoelectrons is given by Ek = W0, where is wavelength of incident

radiation and W0 is work function of the surface on which radiation is incident.
Maximum kinetic energy of photoelectrons emitted by radiation of wavelength 1 is
1 hc
given by mv12 = W0
2 1
hc
or mv12 2 W0 ...(1)
1
where m is mass of an electron and v1 is maximum velocity of photoelectrons.
1 hc
Similarly, for radiation of wavelength 1, mv 22 W0 ...(2)
2 2
But v2 = 2v1, therefore from equation (2),
hc
2mv12 W0 ...(3)
2
From equations (1) and (3),
hc hc
4 W0 W0
1 2
or W0 = 3 eV

www.physicsashok.in 28
ATOMIC PHYSICS

hc
But work-function W0 = where 0 is threshold wavelength.
0
hc
0 = W = 4125 Ans. (i)
0
In saturation mode, spectal sensitivity with wavelength 1 = 3000 is J = 4.8 mA/W or 4.8 mC/J. It means
when 1 joule radiation of wavelength 1 = 3000 is incident, a charge of 4.8 mC flows in saturation mode
4.8 mC
or electrons are ejected.
e
hc
Energy of each photon of wavelength 1 is E1 =
1
Number of photons in 1 joule radiation of wavelength 1
1 1

E1 hc
J 4.8 103
No. of electrons ejected by these photons = = = 3 1016
e 1.6 10 19
Efficiency of photo-electron generation per incident photon,
3 1016
0.0198 .
(1 / hc)
Energy of each photon of wavelength 2,
hc
E2 =
2
Rate of incidence of photons of wavelength 2 in a radiation of power P
P P 2

E2 hc per second
Since, efficiency of photo-electron generation is same for both the case, therefore, rate of ejection of
electrons in later case
P
. 2 per second
hc
P 2
Rate of flow of charge in saturation mode = e Cs1 = 13.2 Cs1
hc
But rate of flow of charge is current. Hence, saturation current is second case = 13.2 A. Ans. (ii)

Example 22. A monochromatic point source S radiating wavelength A


= 6000 with power P = 2 watt, an aperture A of radius R = 1 cm
and a large screen are placed as shown in fig. A photoemissive detector S D
D of surface area S = 0.5 cm2 is placed at centre of the screen. L
Efficiency of detector for photoelectric emission per incident photon 60cm
is = 0.9.
6m
(i) calculate photon flux at centre of screen and photo current in the detector.
(ii) If a convex lens L of focal length f = 30 cm is inserted in the aperture as shown, calculate new value of
photon flux and photo current assuming a uniform average transmission of 80% from the lens.
(iii) If work function of photo-emissive surface is W0 = 1 eV, calculate value of stopping potential in two cases
(without and with the lens in aperture). Given, h = 6.625 1034 J-S, c = 3 108 ms1.
www.physicsashok.in 29
ATOMIC PHYSICS

Sol. Photon flux is rate of incidence of photons per unit are of detector. Therefore, to calculate photon flux, first
rate of emission of electrons from the source should be calculated.
hc
Energy of each photon is E

Rate of emission of energy from the source is P = 2 watt = 2 Js1
P P
Rate of emission of photons from the source is n
E hc
or n = 6.04 1018 photons per second
(i) Distance of detector from source is r1 = 6 m
Photon flux at detector,
n
1 1.33 1016 photons/m2s
4r12
Rate of incidence of photons on detector = 1 . S
Rate of emission of electrons from detector = 1S per second
Since, current is charge flowing per second, therefore photo current
= (1S)e = 9.6 108 amp
(ii) When a concave lens is inserted in the aperture, it refracts incident rays. Therefore, photon flux and hence
photo-current changes.
Distance of lens from source is r2 = 0.60 m
n
Photon flux at lens is = 4r 2 = 1.33 1018 photons/m2s
2

Considering a very small area A of the lens,


Rate of incidence of photons on this area of lens = A
Now considering refraction through the lens,
u = 60 cm, f = + 30 cm v = ?
1 1 1
Using lens formule, , v = + 60 cm
v u f
Since, average transmission from lens is 80%, therefore, rate of transmission of photons from area A of lens
= 0.8 A
But these photons are transmitted in a solid angle subtended by the area A at P as shown in fig.
A A
This solid angle, 2

v (0.6)2
Rate of photons transmitted per unit solid angle is
(0.8 A)
T = (0.8 ) (0.6)2 = 0.288

D
S P

v 4.80m

www.physicsashok.in 30
ATOMIC PHYSICS

1
Solid angle subtended by unit area of detector at P, =
(4.80)2
Photon flux at D, 2 = T = 1.67 1016 photons/m2s
Rate of incidence of photons on detector = 2S
Rate of emission of electrons from detector = 2S
Photo current = (2S)e = 1.20 107 amp
hc
(iii) Since, stopping potential V0 is given by e.V0 = W0 and and W0 both remain unchanged, therefore,

stopping potential is same for both the cases.
1 hc
V0 W0 1.07 volt
e

PHOTOELECTRIC EFFECT AND WAVE THEORY OF LIGHT


According to wave theory, when light falls on a metal surface, energy is continuously distributed over the
surface, energy is continuously distributed over the surface. All the free electrons at the surface receive light
energy. An electron may be ejected only when it acquired energy more than the work function. If we use a
low-intensity source, it may take hours before an electron acquires this much energy from the light. In this
period, there will be many collisions and any extra energy accumulated so far will be shared with the
remaining metal. This will result in no photoelectron. This is contrary to experimental observations. No
matter how small is the intensity, photoelectrons are ejected and that too without any appreciable time
delay. In the photon theory, low intensity means less number of photons and hence less number of electrons
get a change to absorb energy. But any fortunate electron on which a photon falls, gets the full energy of the
photon and may come out immediately.
In figure, we illustrate an analogy to the wave the particle behaviour of light. In part (a), water is sprayed
from a distance on an area containing several plants. Each plant receives water at nearly the same rate. It
takes time for a particular plant to receive a certain amount of water. In part (b) of the figure, water is filled
in identical, loosely-tied water bags and a particle physicist throws the bags randomly at the plants. When a
bag collides with a plant, it sprays all its water on that plant in a very short time. In the same way, whole of
the energy associated with a photon is absorbed by a free electron when the photon hits it.

(a) (b)
The maximum kinetic energy of a photoelectron does not depend on the intensity of the incident light. This
fact is also not understood by the wave theory. According to this theory, more intensity means more energy
and the maximum kinetic energy must increase with the increase in intensity which is not true. The dependence
of maximum kinetic energy on wavelength is also against the wave theory. There should not be any threshold
wavelength according to the wave theory. According to this theory, by using sufficiently intense light of any
wavelength, an electron may be given the required amount of energy to come out. Experiments, however,
show the existence of threshold wavelength.

www.physicsashok.in 31
ATOMIC PHYSICS

DUAL NATURE OF LIGHT


(a) Wave nature:
Wave nature of light can be explained on the basis of reflection, refraction, interference, diffraction and
polarization.
(b) Particle Nature:
Energy is transported by energy particles, photons. It could be explained by photoelectric effect, Zeeman
effect, Compton effect etc.

MATTER WAVE THEORY OR DE-BROGLIES THEORY


(a) This theory was given on the basis of duel nature of light.
(b) According to de-Broglie theory each and every moving particle has some wave nature associated with itself
which is called matter waves.
(c) Thus, moving particles like e, proton, neutron, -particle etc, also behave like waves.
(d) These waves are waves of probability.
(e) The wavelength associated with a moving particles is given by = h/p, where p is the momentum of the
particle.
(f) This wavelength is known as the de-Broglie wavelength of the particle.

DE BROGLIE WAVELENGTHS

For Photon For moving particle


Rest mass Zero m
E h m0
Effective mass m= 2 = m=
c c2 1 v2 / c2
hc 1 p2
Energy (kinetic) E = hv = E= mv2, E =
2 2m
E h h
Momentum p= = = p = mv = 2mE
c c
h hc h h
Wavelength = = = =
p E p 2mE
h
Wavelength for charged particle =
2mE
accelerated by V volts K.E. = qV
Speed c = 3 108 m/s v= 2E / m = 2qV / m

De-Broglies explanation for stable Bohrs orbits :
(a) De-Broglie suggested that non-radiation of energy by the electrons circling
in a Bohrs orbit can be explained on the basis on the basis of the formation r
of stationary waves by the electrons in circular motion in Bohrs orbits. O

(b) Comparing this to the vibrations of a wire loop such stationary waves would
be formed if each wave joins smoothly with the next.
Fourth Bohr-orbit

www.physicsashok.in 32
ATOMIC PHYSICS
(c) In other words the number of wavelengths must be an integer.
(d) Condition for stable orbits :
An election can circle around an atomic nucleus without radiation energy if the circumference of its orbit is
an integral multiple of the electrons wavelength.
i.e. 2r = n condition for stable orbits.
h nh nh
2r = n pr = mvr = [ p = mv]
p 2 2
nh
mvr =
2
As mvr is the angular momentum of the circling electron. Bohrs postulate is justified.

C18: Calculate the de-Broglie wavelength associated with the motion of earth (mass = 6 1024kg) orbiting
around the sun at a speed of 3 106 ms1.
h 6.63 1034 (Js)
Sol: = =
mv (6 1024 ) (3 106 )(kg ms1 )
= 3.68 1065 m

NOTE: The wavelengths associated with the motion of macroscopic objects like earth, train etc, are
negligible compared to their sizes. This is why the wave-like character of these objects is not
observable in our daily life.

C19: Calculate the de-Broglie wavelength of an -particle of mass 6.576 1027 kg and charge 3.2 1019
coulomb, accelerated though 2000 V.
Sol: E = kinetic energy of a-particle = qV
E = 3.2 1019 2000 J = 6.4 1016 J
de Broglie wavelength,
h
= , [ 2mE = momentum of photon]
2mE
6.63 10 34 (J s)
= 27 16
= 2.28 1013 m
2 6.576 10 6.4 10 (J kg)

NOTE: The wavelength associated with -particles is of the order of size of the -particle.
That is why the wave like character of a-particle is observable.

C20: A particle of mass m and charge q is accelerated through a potential difference V. Find (a) its kinetic energy
(b) momentum, and (c) de-Broglie wavelength associated with its motion.
Sol: When the particle is accelerated through a potential difference V, gain in kinetic energy is given by K = qV.
(a) Thus, kinetic energy, K = qV.
p2
(b) Momentum of the particle (p) is given by K =
2m
p = 2mK = 2m qv

Momentum = 2m qv

www.physicsashok.in 33
ATOMIC PHYSICS

(c) De- Broglie wavelength is given by,


h h
= = 2m qV
p
h
Thus, wavelength =
2m qV

Example 23. Assume that the de-Broglie was associated with an electron can form a standing wave between
the atoms arranged in a one dimensional array with nodes at each of the atomic sites. It is found that one
such standing wave is formed if the distance d between the atoms of the array is 2 . A similar standing
wave is again formed if d is increased to 2.5 but not for any intermediate value of d. Find the energy of
the electron in eV and the least value of d for which the standing wave of the type described above can
form.
Sol. From the figure it is clear that
p . (/2) = 2 2
(p + 1) . /2 = 2.5 N N
/2 = (2.5 2.0) = 0.5 p-loops
/2
or = 1 = 1010 m
(i) de Broglie wavelength is given by
(p + 1) loops
h h 2.5

p 2 km
K = kinetic energy of electron
h2 (6.63 1034 )2
K 2
31 10 2
2.415 1017 J
2m 2(9.1 10 )(10 )

2.415 1017
K eV
1.6 1019
K = 150.8 eV
(ii) N N The least value of d will be when only one loop is formed
dmin = /2 or dmin = 0.5

C21: Find the de-broglie wavelength associated with an electron accelerated through a potential difference of
30 kV.
Sol: Kinetic energy of the electron
K = qV = e(30 kV) = 30 keV = 30 103 1.6 1019 J = 4.8 1015 J
Now, momentum of the electron = 2mK = 2 9.1 1034 4.8 1015 kg.J = 2.96 1024 kg.m/s
h
De-broglie wavelength, =
Momentum
6.63 1034 J s
=
2.96 1024 Kg m / s
= 2.24 1010 m = 2.24 A

www.physicsashok.in 34
ATOMIC PHYSICS

ATOMIC STRUCTURE

ATOMIC MODEL
By now, it is well-known that matter, electricity and radiation etc. are all atomic in character. Although, no
one has so far seen individual atoms, there is no doubt that they really exist. In 1895, it was discovered by
J.Perrin in Paris that the cathode rays consist of negatively-charged practices called electrons. In 1897,
J.J. Thomson measured the e/m ratio for an electron whereas its charge was measured by Millikan in
1906 by his famous oil-drop experiment. Mass of the electron was found by dividing charge e by the ratio
e/m. Discovery of positive rays during the latter part of 19th century indicated that a normal atom consisted
of both negative and positive charges. But how these charges are distributed in an atom was not known at
that time. Consinuous efforts have been made since then to study the physical structure of an atom such as
its extra-nuclear electronic structure chiefly with the help of spectral properties of atoms. To account for the
spectroscopic data obtained experimentally over the years, several theories regarding atomic structure have
been proposed from time to time which are called the atomic models. Various atomic models proposed by
scientists over the last few decades are:
(i) Thomsons Plum pudding model, (ii) Rutherfords Nuclear model, (iii) Bohrs model (iv) Sommerfelds
Relativistic model (v) Vector model and finally (vi) Wave-mechanical model.
These different models have been suggested one after the other in an effort to get a satisfactory interpretation
of the experimental data which, it is hoped, will ultimately lead to a perfect and complete understanding of
the physical structure of an atom.
+ +
+ + +
Thomsons Plum Pudding Model +
+
+ + + +

+ + + +
+ + +

+
According to this model, the atom is regarded as a heavy sphere of positive + + + +

+ + + + + +

+
+ + + +
+ + + +

+ + + +

+
+

+ + + +
charge seasoned with enough electron plums to make it electrically neutral.

+ + +

+ + +
+ +

+ +
+ +
+ +
+ +
Thomson visualized the positive charge of an atom being spread out uniformly

+
+ +
++
++ +

+
+ +
+
throughout a sphere of about 1010 metre radius with electrons as smaller
+
+
++ + +

+
+ + + +
+ +
particles distributed in circular shells as shown in figure. Whereas the net force + +
+ + +

exerted by the positively-charged sphere on each electron is towards the centre of the sphere, the different
electrons experience mutual repulsion and get arranged in the form of circular shells.
This atomic model was given up after some time because it could not provide any satisfactory mechanism
for explaining the large deflection suffered by -particles in Rutherfords experiment.

Rutherfords Experiment on -particle Scattering


As shown in figure high-speed a-particle (i.e. helium nuclei each with a charge of +2e) from some radioactive
material like radium or radon, confined to a narrow beam by a hole in a lead block, were made to strike a
very thin gold foil G. While most of the -particles went straight through the foil as if there were nothing there
(and produced scintillations on a fluorescent screen), some of them collided with the atoms of the foil and
were scattered around at various angles -a few being turned back towards the source itself.



Radium
15
0
-particles

45
Beam of

60 +
Lead block

www.physicsashok.in 35
ATOMIC PHYSICS

Although some small angle scattering could be expected from Thomsons model, large-angle scattering was
absolutely not expected at all. Further detailed experiments in Rutherfords laboratory by Geiger and Marsden
showed conslusively that large-angle scattering can be expected only if one assumes that a massive positive
point charge exists at the centre of each gold atom as shown in figure.
According to this model proposed by Rutherford in 1911, the positive massive part of an atom is assumed
to be concentrated in a very small volume at its centre. This central core, now called nucleus, is surrounded
by a cloud of electrons which makes the entire atom electrically neutral.

The large-angle scattering of positively-charged -particles


Trajectory
could be easily explained on this atomic model as shown in of -particle P
figure. This scattering is due to the mutual repulsion (as per Nucleus
Coulombs law) between the -particles and the concentrated +Ze
positive charge on the nucleus. The -particle approaches
the positively-charged nucleus along AO. If there were no Asymptote
repulsion from the nucleus, it would have passed at a distance of trajectory
of p from it. However, due to
coulombic force of repulsion, the -particle follows a hyperbola with nucleus as its focus. The lines AO and
AO are the asymptotes of the hyperbola and present approximately the initial and final directions of the -
particle when it has passed out of the effective range of the nuclear electric field.
As seen, the -particle is deflected through an angle . The perpendicular distance from the nucleus to the
line AO is called the impact parameter and is denoted by p. The Rutherfords scattering formula is
Q1Q2
tan =
2 2pE
where Q1 = Charge of the incoming -particle
Q2 = Charge of the scattering nucleus
E = Kinetic energy of the incident -particle
p = Impact parameter

Distance of Closest Approach


Suppose that an -particle approaches a positively-charged nucleus for a head on collision with a kinetic
energy of K. As shown in figure at point A, the repulsive force of the nucleus is so strong as to stop the -
particle momentarily. At this point, all the kinetic energy of the -particle is converted into potential energy.
Let D be the distance of closest approach of the -particle. The potential at point A due to nuclear charge
Ze is
Ze
= 4 D
0

Potential energy of the -particle when at point A is
Ze.2e 2Ze 2
A
+
= 4 D = D
0 4 0 D

2Ze 2 2Ze 2
K= or D =
4 0 D 4 0 K
Since -particles are generally obtained from natural radioactive substances, their kinetic energy K is
known. Hence, value of D can be found easily from the above equation.
www.physicsashok.in 36
ATOMIC PHYSICS

C22: In Geiger-Marsden experiment on -particle scattering from gold foil, the kinetic energy of -particles
used was 7.68 MeV. Calculate the distance of closest approach of -particle if atomic number of gold in
79.
2Ze 2
Sol: D=
4 0 K
Here, Z = 79, e = 1.6 1019 C; 0 = 8.854 1012 F/m
13
K = 7.68 MeV = 7.68 1.6 10 J
2 79 (1.6 10 19 )2
D=
4 8.854 1012 7.68 1.6 1013
= 2.96 1014 m

Major Deficiencies in Rutherfords Nuclear Model


It was found later on that Rutherfords model had two serious drawbacks concerning
(i) distribution of electrons outside the nucleus and
(ii) the stability of the atom as a whole.
It can be shown that electrostatic forces between the positive nucleus and the static negative extra-nuclear
electrons are not enough to produce equilibrium in such a nuclear atom. For example, consider the case of
an atom* having two electrons and a nucleus with a charge of +2e. Suppose the electrons are symmetrically
placed at a distance of r from the nucleus and are stationary. The force of attraction between the nucleus and
2e 2e 2
each of the electrons is F = e. = while the force of repulsion between the two electrons is
4 r 2 4 r 2
e2 e2
= . Since the force of attraction is eight times the force of repulsion, the electrons will
4 4r 2 16 r 2
fall into the nucleus thereby destroying the stable structure of the atom.


+ + m v 2e
+ +
2e 2e r e +
+ + + + Photon (hv)
v


(a) (b) (c)

To overcome this difficulty, Rutherford suggested that stability can be achieved (as in a solar system) by
assuming that electrons, instead of being static, revolve round the nucleus with such a speed that the centrifugal
force balances the attractive force exerted by the nucleus on the electrons. As seen from figure (b), condition
for stability is achieved when
mv 2 2e.e 2e 2
2
= 4 r 2 or mv r =
r 0 4 0
In general, if Z is the atomic number, then nuclear charge is Ze, so that the above relation becomes
mv 2 Ze.e Ze 2
2
= 4 r 2 or mv r =
r 0 4 0

www.physicsashok.in 37
ATOMIC PHYSICS

Incidentally, it may be noted that according to the above relation, it is possible to have an infinite number of
orbits in which electrons can rotate.
But this assumption of revolving electrons led to serious difficulty from the point of view o the electromagnetic
theory according to which an accelerated charge must continuously emit electromagnetic radiation or energy.
Since an electron revolving in a circular orbit has centripetal acceleration (= v2/r), it must radiate energy as
per the laws of classical electrodynamics.
Due to this continuous loss of energy, the electrons will gradually approach the nucleus by a spiral path and
finally fall into it as shown in figure (c). Hence, it is seen that the orbital motion of the electron destroys the
very purpose for which it was postulated i.e. the stability of the atom. Obviously, either Rutherfords nuclear
atomic model with revolving electrons is defective or the classical electromagnetic theory fails in this particular
case. This dilemma was solved in 1913 by Neils Bohr who proposed an improved version of Rutherfords
atomic model.

Bohrs Atomic model


This model (first proposed for hydrogen atom but later applied to other atoms as well) retains the two
essential features of Rutherfords planetary model i.e.
(i) the atom has a massive positively-charged nucleus and
(ii) the electrons revolve round their nucleus in circular orbits the centrifugal force being balanced, as before, by
the electrostatic pull between the nucleus and electrons.
However, he extended this model further by utilizing Plancks Quantum Theory. He made the following
three assumptions:
(iii) an electron cannot revolve round the nucleus in any arbitrary orbit but in just certain definite and discrete
orbits. Only those orbits are possible (or permitted) for which the orbital angular momentum (i.e. moment of
h nh
momentum) of the electron is equal to an integral multiple of i.e. orbital angular momentum = where
2 2
n is an integer and h is Plancks constant. Such orbits are also known as stationary orbits.
(iv) while revolving in these permitted stationary (or stable) orbits, the electron does not radiate out any
electromagnetic energy. In other words, the permissible orbits are non-radiating paths of the electron.
(v) the atom radiates out energy only when an electron jumps from one orbit to another. I f E2 and E1 are the
energies corresponding to two orbits before and after the jump, the frequency of the emitted photon is given
by the relation
E2 E1 = hv or E = hv
where v is the frequency of the emitted radiations.

C 23: If I is the moment of inertia of an electron and its angular velocity, then as per assumption (iii) given
above
nh nh
I = or (mr2) =
2 2
(mr 2 )v n.h n.h
or = or mvr = v
r 2 2
Alternatively, since the momentum of the revolving electron is mv, its moment e
about the nucleus is = mvr +ze
m
n.h r
Hence mvr = .....(i)
2

www.physicsashok.in 38
ATOMIC PHYSICS

where, n = 1, 2, 3 for the first second and third orbits respectively. It is called the principal quantum number
and because it can take whole number values only, it fixes the sizes of the allowed orbits (also called Bohrs
circular orbits).

E3 E3
E2 r1 r2 E2 E
E1 r3 E1
+Ze +Ze
n=1 K
n=2 L
n=3 M

Permitted orbits Electron Jump


(a) (b)

Let the different permitted orbit have energies of E1, E2, E3 etc. as shown in figure (a). The electron can be
raised from n = 1 orbit to any other higher orbit if it is given proper amount of energy. When it drops back
to n = 1 orbit after a short interval of time, it gives out the energy difference E in the form of a radiation as
shown in figure (b). The relation between the energy released and frequency of the emitted radiation is
E2 E1 = hv or E = hv

Expressions for velocity, radius, energy of electron and orbital frequency in Bohrs orbit
Here it should be kept in mind that Bohrs model is valid only for hydrogen atom and hydrogen-like ions. In
other words, we can say that is applicable to hydrogen atom and ions having just one electron. Examples of
such ions are He+, Li++, Be+++ etc.
According to Bohrs
ze
Ze 2 2 + r e, m
mv
First postulate, 2 = v
4 0 r r
1
where 4 = 9 109 Nm2 C2 v = velocity of electron
0

and 0 = 8.854 1012 C2/Nm2 m = mass of an electron


= Absolute permittivity Z = atomic number
of vacuum or free space r = radius of the orbit
e = magnitude of charge on an electron
n = principal quantum number i.e. orbit number
Ze2
r= .....(i)
4 0 mv 2
From fourth postulate of Bohr, we have
nh
mvr = .....(ii)
2
where, n = 1, 2, 3, 4, ........ i.e., n is a positive integer.
From equations (i) and (ii), we get
Ze2 nh ze 2
mv = v =
4 0 mv 2 2 2 0 nh
Now, let us substitute the value of v in equation (ii),

www.physicsashok.in 39
ATOMIC PHYSICS

ze 2 r nh 0 n 2 h 2
m = r
2 0 nh 2 mZe 2
1
Note that for fixed n, v Z, r
Z
1
For fixed z, v, r n2
n
If V1 is the speed of the electron in the 1st orbit
Z c
Then, Vn = V1 , where V1 = , where c is speed of light.
n 137
If a0 = first Bohr radius = 0.53 , then
n2
rn = a0
Z

1 mZ2 e 4 kZe 2
2
K.E. of the electron = mv = =K= ,
2 8 20 h 2 n 2 2r

Ze 2 kZe 2
Potential energy of the atom = =U= [k = 1/40]
4 0 r r

mZ2 e 4
U=
4 02 h 2 n 2
Total energy of the atom, E = K + U
mZ2 e 4 Z2
E= In general, En = 13.6 in eV..
8 20 h 2 n 2 n2
Orbital frequency for the electron,
V Ze 2 mZe 2 mZ2 e 4
v= = v=
2r 2 2 0 hn0 h 2 n 2 4 02 n 3 h 3
Time period of revolution (T) is given by
n3
T = T1
Z2
2a 0
where T1 = time period of revolution in the 1st orbit = 1.52 1016 s = v1
NOTE : It is assumed that the acceleration of the nucleus is negligible on account of its large mass.

Some important results for H-atoms when n = 1


1. Bohr radius, a0 = 0.53
rn = a0n2/Z
2. v1 = 2.18 106 ms1 c/137
U1
3. E1 = 13.6 eV =
2
www.physicsashok.in 40
ATOMIC PHYSICS

U1
4. K 1 = 13.6 eV =
2
5. U1 = 27.2 eV
6. v1 = 6.6 1015 Hz
7. 1 rydberg = 13.6 eV
8. B1 = 12.5 tesla, where B1 is the magnetic field at the centre of Bohr atom due to the current generated by
the motion of electron in 1st orbit.

Ground state and excited states


The state of an atom with the lowest energy is called its ground state or normal state. For ground state, n =
1.
The states with higher energies are called excited states. For the first excited state, n = 2; for the 2nd excited
state, n = 3 and so on.
For mth excited state, n = m + 1

Ionization energy and ionization potential


The minimum energy needed to ionize an atom is called ionization energy.
The potential difference through which an electron should be accelerated to acquire the value of ionization
energy is called ionization potential.
The value of ionization energy of H-atom in ground state is 13.6 eV that of ionization potential is 13.6 eV.

Binding energy
Binding energy of a system is the energy needed to separate its constituents to large distances or it may be
defined as the energy released when its constituents are brought from infinity to form the system.
The value of binding energy of H-atom is 13.6 eV, identical to its ionization energy.

Excitation energy and excitation potential


The energy needed to take the atom from its ground state to an excited state is called the excitation energy
of that excited state.
The potential difference through which an electron should be accelerated to acquire the value of excitation
energy is called excitation potential.

NOTE : (A) I.E. = E E1 = E1 = Binding energy of the H-atom.


(B) Movement of electron in circular orbits in a Bohr atom causes electric current in the orbit. This
current will lead to self-generated magnetic field in the atom and also magnetic current (m)
(a) Magnetic field(B)
If B is the magnetic field generated at the centre of atom, then e
0i a0
B = 2a , where i is the current due to motion of the electron
0
+e i
and a0 is the 1st Bohr-radius. H-atom
Hence, i = ev
0 e
hence B = 2a
0

www.physicsashok.in 41
ATOMIC PHYSICS

Putting the values of v and a0, we get B = 12.5 Tesla



(b) Magnetic movement vector ( )

= iA
= iA = ev a 20

(c) Relation between & L (angular momentum vector)
= iA = ev a 20 , [i = ev, A = a 20 ]
L = mV1a0 = m2v a 20 [V1 = 2va0]

ea 20 e e
= 2 = =
L m2a 0 2m L 2m
Vectorially,
e
= L
2m

Example 24: The quantum number of Bohr orbit in H-atom whose radius is 0.01 mm is
(a) 223 (b) 435 (c) 891 (d) none of these
Sol: (b) We know that
rn = a0n2
rn
n2 = a
0

0.01 10 3
n = n = 435
0.529 1010

Example 25: The quantum number n in the Bohrs model of H-atom specifies:
(a) radius of the electron (b) energy of the electron
(c) angular momentum of the electron (d) all of these
0 h 2
Sol: (d) rn = me 2 n2

me 4 , h
En = Ln = n
8 20 h 2 n 2 2

Example 26: The radius Bohrs orbit in ground state for H-atom is
(Take 0 = 8.86 1012 C2/Nm2, h = 6.6 1034 J-s)
(a) 0.528 (b) 0.0528 (c) 5.28 1010 m (d) 5.28 1010 cm
0 h 2
Sol: (a) a0 = (in this case, Z = 1, n = 1)
me 2
(6.6 1034 )2 (8.86 10 12 )
a0 = = 0.528
3.14 (9.11031 ) (1.6 1019 )2

www.physicsashok.in 42
ATOMIC PHYSICS

Example 27: The speed of the electron in the first Bohr orbit of H-atom is (Take c = speed of light in vacuum)
c c
(a) c (b) (c) (d) 137 c
13.6 137
Sol: (c) For H-atom, we know that
e2
v = [since n this case, Z = 1 and n = 1]
2 0 h
e2 c
=
2 0 hc
e2 1
But = = fine structure constant
2 0 hc 137
c
v =
137

Example 28: In a H-atom, binding energy of the electron in the ground state is E1. Then the frequency of
revolution of the electron in the nth orbit is
2E1 2E1n 3 2mE1
(a) 3 (b) (c) (d) none of these
nh h n 3h
Sol: (b) The frequency of revolution of the electron in the nth orbit is given by
mZ2 e 4 2E1 mZ2 e 4
v = = where E1 =
4 02 h 3 n 3 hn 3 8 20 h 2

C24: Calculate the energy of a He+ ion in its first excited state.
(13.6eV)Z2
Sol: En =
n2
Here, Z = 2, n = 2
13.6 2 2
En = eV = 13.6 eV
n2

C25. An electron in a hydrogen like atom is in an excited state. It has a total energy of 3.4 eV. Calculate :
(i) the kinetic energy
(ii) the de-Broglie wavelength of the electron
Sol. (i) Kinetic energy of electron in the orbits of hydrogen and hydrogen like atoms
= | Total energy |
Kinetic energy = 3.4 eV
(ii) The de Broglie wavelength is given by
h h
(K = kinetic energy of electron)
P 2 Km
Substituting the values, we have
(6.6 1034 J s)

2(3.4 1.6 1019 J)(9.1 1031 kg)
= 6.63 1010m or = 6.63

www.physicsashok.in 43
ATOMIC PHYSICS

Example 30. The electron in a hydrogen atom makes transition from M shell to L. The ratio of magnitudes of
initial to final centripetal acceleration of the electron is
(A) 9 : 4 (B) 81 : 16 (C) 4 : 9 (D) 16 : 81
2
Sol. rn n
1
and vn
n
v2n
an
rn
1
an
n n2 2

1
an 4
n
K
an
n4
K
a M a3 { M = 3}
81
K
aL a2 { L = 2}
16
a M 16
Hence (D) is correct.
a L 81

3h
Example 31. The angular momentum opf an electron in the hydrogen atom is . Here h is Plancks constant.
2
The kinetic energy of this electron is :
(A) 4.53 eV (B) 1.51 eV (C) 3.4 eV (D) 6.8 eV
nh 3h
Sol. L
2 2
n=3
In the electronic third orbit, the energy of electron
Z2
E 13.6
n2
1
E 13.6
9
13.6
E 1.51 eV Hence (B) is correct.
9

Example 32. A particle of mass m moves along a circular orbit in a centrosymmetrical potential field
kr 2
U(r) = . Using the Bohrs quantization condition, find the permissible orbital radii and energy levels of
2
that particle.

www.physicsashok.in 44
ATOMIC PHYSICS

dU
Sol. F= = kr
dr
Negative sign implies that force is acting towards centre. The necessary centripetal force to the particle is
being provided by this force F. Hence
mv 2
= kr ...(1)
r
h
and mvr = n h where h ...(2)
2
solving equations (1) and (2) we get
nh k
r = rn = where =
m m
kr 2 1
and total energy E = U + K = + mv2
2 2
Substituting the values, we get
E = n h = En

Example 33. In a hypothetical system a particle of mass m and charge 3q is moving around a very heavy
particle having charge q. Assuming Bohrs model to be true to this system, the orbital velocity of mass m
when it is nearest to heavy particle is
3q 2 3q 2 3q 3q
(A) (B) (C) 2 h (D) 4 h
2 0 h 4 0 h 0 0

mv 2
Sol. Fe
r
q 3q mv2

4 0 r 2 r

3 q2
mvr v
4 0
h
mvr
2
3 q2 h
v
4 0 2

3 q2
v Hence (A) is correct.
2 0 h

www.physicsashok.in 45
ATOMIC PHYSICS

SPECTRUM
Dispersed light arranging itself in a pattern of different wavelength is referred to as a spectrum. Light coming
from a source may be dispersed by a prism or by any other dispersing medium.
When white light falls on a prism and the transmitted light is collected on a white wall or white paper then a
spectrum is obtained which consists of different colours from red to violet.

Kinds of spectra :
(A) Emission spectra: When a light beam emitted by certain source is dispersed to get the spectrum, it is
called an emission spectrum.
An emission spectrum can be three types :
(a) Continuous spectrum: That emission spectrum which is obtained by continuously varying wavelength, is
called continuous emission spectrum. In this case, when light is dispersed, a bright spectrum continuously
distributed on a dark background is obtained. Light emitted from an electric bulb, a candle or a red hot iron
piece comes under this category.
(b) Line spectrum: The atoms and molecules can have certain
fixed energies. An atom or molecule, in an excited state, can
emit light to lower its energy. Light emitted in such a process

Energy
has certain fixed wavelengths. When such a light is dispersed,
certain sharp bright lines on a dark background is obtained.
Such a spectrum is called line emission spectrum. Line spectrum
Atomic energy levels

For example, when electric discharge is passed through sodium vapour, they vapour emits light of the
wavelength 589.0 nm and 589.6 nm. When this light is dispersed by a high resolution grating , one obtains
two bright yellow lines on a dark background.
(c) Band spectrum : The wavelengths emitted by the molecular energy levels which are generally grouped into
several bunches, are also grouped; each group being well separated from the other. The spectrum looks like
separate bands of varying colours. Such a spectrum is called band emission spectrum.
Energy

Molecular energy levels Band spectrum

(B) Absorption spectrum : When white light is passed through an absorbing material, the material may absorb
certain wavelengths selectively. When the transmitted light is dispersed, dark lines or bands at the positions
of the missing (absorbed) wavelengths are obtained. Such type of spectrum is called absorption spectrum.

White light
Absorbing material
Absorption
spectrum

www.physicsashok.in 46
ATOMIC PHYSICS
An absorption spectrum may be of two types :
(a) Line Absorption spectrum : Light may be absorbed by atoms to take them from lower energy states to
higher energy states. In the similar way when white light is passed through a gas, the gas is found to absorb
light of certain wavelength. The absorption spectrum consists of dark lines on bright background. Such a
spectrum is called a line absorption spectrum. When light coming from the sun is dispersed, it shows certain
sharply defined dark lines. This shows that certain wavelengths are absent. There missing lines are called
Fraunhofer lines.
(b) Band Absorption spectra : If absorbing media is polyatomic such as H2, CO2 or KMnO4 solution,
instead of dark lines we get few characteristic dark bands (against coloured background) called band
absorption spectra.
Hydrogen Spectra :
If hydrogen gas enclosed in a sealed tube is heated to high temperature, it emits radiation. This radiation
consists of components of different wavelengths which deviate by different amounts. The radiation with
different amounts of deviation forms H-spectrum.
Explanation of hydrogen spectra by Bohr
(a) The electron in a H-atom if not disturbed remains in the ground state (i.e. n = 1 state). When the electron
receives energy from outside, it is elevated to any one of the higher permitted states (excited state).
(b) The electron remains only for a short interval of time (generally in the order of 108 s) in the excited state and
comes back to the ground state finally.
(c) The electron can reach the ground state from any one of the excited states in many ways. As a result, many
electron transitions take place.
(d) According to Bohr, all electron transitions terminating at a particular state give rise to a particular spectral
series.
n= E=0
n=5
n=4
n=3

n=2
Energy

Series limit

n =1
Lyman series Balmer Paschen Brackett
series series series
1 E 1 1
(e) Lyman: nf = 1: = 1 2 2 n = 2, 3, 4,..........
ch 1 n
1 E 1 1
Balmer: nf = 2: = 1 2 2 n = 3, 4, 5, ..........
ch 2 n
1 E 1 1
Paschen: nf = 3: = 1 2 2 n = 4, 5, 6, ..........
ch 3 n
1 E 1 1
Brackett: nf = 4: = 1 2 2 n = 5, 6, 7, .........
ch 4 n
1 E1 1 1
Pfund: nf = 5: = 2 2 n = 6, 7 , 8,......
ch 5 n

www.physicsashok.in 47
ATOMIC PHYSICS

(f) If an electron makes a jump from the nith to nfth orbit (ni > nf), the extra energy Ei Ef is emitted as a photon
of electromagnetic radiation. The corresponding wavelength is given by
1 Ei Ef
= where c = speed of light in vacuum.
hc
According to Bohr, we can write

1 mZ2e4 1 1
= 2 3 2 2
8 0 ch n f n i

1 1 1
= RZ2 2 2
nf ni

me 4
where R = is called the Rydberg constant.
8 02 ch 3
(g) The value of R is 1.0973 107 m1

RhcZ2
NOTE: En = , 1 rydberg = 13.6 eV, Rhc = 13.6 eV
n2

1 2
(h) is called wave number ( ) of the line and is called angular wave number of the line.

(i) Photon energy = Ep = hv
Photon Energy
(j) Momentum of a photon = p = speed of light

Ep
p=
c

Important points regarding H-spectra


(a) The sharply defined, discrete wavelengths exist in the emitted radiation in the H-spectrum.
(b) A hydrogen sample emits radiation with wavelengths less than those in the visible range (i.e. uv light) and
also with wavelengths more than those in the visible range (i.e. infrared).
(c) the lines may be grouped in separate series.
(d) In each series, the separation between the consecutive wavelengths decreases as we move from higher
wavelength to lower wavelength.
(e) A particular minimum wavelength in each series approach a limiting value known as series limit.
(f) The series corresponding to uv region, visible region and infrared region are known as Lyman, Balmer and
Paschen series respectively.
(g) In the Balmer series of hydrogen, the H line (3 2) is red, the H line (4 2) is blue, the H ( 5 2) and
H(6 2) lines are violet, and the other lines are in the near ultraviolet (uv).
n(n 1)
(h) Theoretically possible no. of emission spectral lines = .
2
(i) Theoretically possible no. of absorption spectral lines = (n 1)

www.physicsashok.in 48
ATOMIC PHYSICS

(j) Approximate range of wavelength for different colours of visible light


Colour Wavelength Range
Violet + Indigo 3800 to 4500
Blue 4500 to 5000
Green 5000 to 5500
Yellow 5500 to 6000
Orange 6000 to 6500
Red 6500 to 7200
Infrared rays: 720 nm to 50 mm
Ultraviolet light: 10 to 3800

Limitations of Bohrs Model


(a) It is valid only for one electron atom and hydrogen-like ions e.g. : H, He+, Li+2, Na+10 etc.
(b) Orbits were taken as circular but according to SOMMER field these are elliptical.
(c) Intensity of spectral lines could not be explained.
(d) Nucleus was taken as stationary but it also rotates on its own axis.
(e) It could not be explained the minute structure in spectrum line.
(f) This does not explain the ZEEMAN effect (splitting up of spectral lines in magnetic field) & Stark effect
(splitting up in electric field).
(g) This does not explain the doublets in the spectrum of some of the atoms like sodium (5890 to 5896 ).

C26. Total number of emission lines from some excited state n1 to another energy state n2(< n1) is given by
(n1 n 2 )(n1 n 2 1) n(n 1)
. For example total number of lines from n1 = n to n2 = 1 are .
2 2
C27. As the principal quantum number n is increased in hydrogen and hydrogen like atoms, some quantities
are decreased and some are increased. The table given below shows which quantities are increased and
which are decreased.
Table
Increased Decreased
Radius Speed
Potential energy Kinetic energy
Total energy Angular speed
Time period
Angular momentum
1
C28. Whenever the force obeys inverse square law F 2 , and potential energy is inversely proportional
r
to r, kinetic energy (K), potential energy (U) and total energy (E) have the following relationships.
|U| U
K= and E = K = .
2 2
1 1
If force is not proportional to 2 or potential energy is not proportional to , the above relations do not
r r

www.physicsashok.in 49
ATOMIC PHYSICS

hold good. In JEE problems, this situation arises at two places, in an atom (between nucleus and electron)
and in solar system (between sun and planet).
C29. Total energy of a closed system is always negative and the modulus of this is the binding energy of the
system. For instance, suppose a system has a total energy of 100 J. It means that this system will separate
if 100 J of energy is supplied to this. Hence, binding energy of this system is 100 J. Thus, total energy of an
open system is either zero or greater than zero.
C30. Kinetic energy of a particle cant be negative, while the potential energy can be zero, positive or negative.
It basically depends on the reference point where we have taken it zero. It is customary to take zero
potential energy when the electron is at infinite distance from the nucleus. In some problem suppose we take
zero potential energy in first orbit (U1 = 0), then the modulus of actual potential energy in first orbit (when
reference point was at infinity) is added in U and E in all energy states, while K remains unchanged.

Example 33: The value of series limit in Lyman series is:


(a) 121.6 nm (b) 91.2 nm (c) 656.3 nm (d) 365.0 nm
1 1
Sol: = R 1
min
1
min = R[1 0]

1
min = = 91.2 nm
1.097 107

C31: Find the longest wavelength present in the Balmer series of hydrogen:
Sol: In the Balmer series, nf = 2. The longest wavelength in this series corresponds to the smallest energy
difference between energy levels. Hence the initial state must be ni = 3.
1 1 1 1 1 1 1
= R n2 n2 = R 2 2 = R
f i 2 3 4 9
1 5R
=
36
36
= = 6.56 107 m
5 1.097 107
= 656 nm (near the red end of the visible spectrum)

C32: A hydrogen atom emits uv radiation of 102.5 nm. Calculate the quantum numbers of the states involved in
the transition.
Sol: The uv radiation 102.5 nm lies in the lyman region of spectrum. Thus nf = 1
1 1
= R 1 n 2
i

1 1
2 = 1
ni R
1 1 1
2 = 1
ni 9 7 = 1
102.5 10 1.097 10 1.124

www.physicsashok.in 50
ATOMIC PHYSICS

1
n i2 = 0.1
n i2 = 10
ni 3
Hence transition is from 3 1.

C33: What is the unit of reciprocal of Rydberg constant in S.I. units ?

1 1 1
Sol: We know that = R n 2 n 2 the term in the bracket is unitless.
f i

Now, we can write


1 1
=R =
R
1
Hence, the unit of will be metre i.e. m.
R

C34: How many different wavelengths may be observed in the spectrum from a hydrogen sample if the atoms
are excited to states with principal quantum number n ?
Sol: The total number of possible transitions is
n(n 1)
(n 1) + (n 2) + (n 3) + ............. + 2 + 1 =
2
C35: Consider the following two statements:
(A) Line spectra contain information about atoms only
(B) Band spectra contain information about molecules
(a) Both A and B are wrong (b) A is correct but B is wrong
(c) B is correct but A is wrong (d) Both A and B are correct
Sol: (c) Line spectra contain information about atoms and molecules both.

C36. Ultraviolet light of wavelength 800 and 700 when allowed to fall on hydrogen atoms in their ground
state is found to liberate electrons with kinetic energy 1.8 eV and 4.0 eV respectively. Find the value of the
Planck constant.
Sol. hv = E0 + T where E0 = ground level energy and T = kinetic energy of electron
hc
E0 T

hc
= E0 + 1.8 1.6 1019
800 1010
hc
and = E0 + 4.0 1.6 1019
700 1010
hc 1 1
Subtracting = 2.2 1.6 1019
108 7 8
2.2 1.6 10 27 56
or h= = 6.57 1034 Js
3 108

www.physicsashok.in 51
ATOMIC PHYSICS

C37: The excitation energy of a hydrogen like ion in its first excited state is 40.8 eV. Find the energy needed to
remove the electron from the ion.
Sol: The excitation energy in the first excited state is
1 1
E = 13.6Z2 2 2
1 2
3
40.8 = (13.6 eV) Z2
4
Z=2
13.6Z2
Now, ionization energy = = 4 (13.6 eV)
12
Eion = 54.4 eV

C38: The first ionization potential of some hydrogen like Bohr atom is x V. Then the value of the first excitation
potential for this atom will be:
x 3
(a) xV (b) V (c) xV (d) 20 xV
2 4
Sol: The value of first excitation potential is given by
1
x = x 1 V
4
3x
x = V
4

Example 34: A doubly ionised lithium atom is hydrogen like with atomic number 3. Find the wavelength of the
radiation required to excite the electron in Li++ from the first to the third Bohr orbit. (Take ionization energy
of H-atom equal 13.6 eV). E3 n=3
Sol: For E1, Z = 3, n = 1 E n=2
2

13.6Z 2
13.6 9 E
E1 = 2 = = 122.4 eV
n 1
E1 n=1
For E3, Z = 3, n = 3
13.6 32
E3 = = 13.6eV
32
We have E + E1 = E3
E = E3 E1 = 108.8 eV
hc 12400
= = = 114
E 108.8

Example 35: In Bohrs model of hydrogen atom when the electron is moving in one of the stationary orbits then:
(a) velocity of the electron is fixed and no emission of energy takes place e
v2
(b) velocity changes continuously but no emission of energy takes place v1
(c) energy is emitted but the velocity does not change
(d) energy is emitted and the velocity also changes. e e
Sol: (b) According to the second postulate of Bohr.

| v1| = | v 2 | = | v3| = | v 4 | = v v3
v4
e
www.physicsashok.in 52
ATOMIC PHYSICS

Example 36. An electron in an unexcited hydrogen atom acquired an energy of 12.1 eV. To what energy level
did it jump? How many spectral lines may be emitted in the course of transition to lower energy levels?
Calculate the shortest wavelength.
Z2
Sol. E = E0 2 where E0 = 2.18 1018 J = 1 rydberg
n
E (energy gap between unexcited state (n = 1) and an excited state (n = m))
1 1
E 0 Z2 2 2
1 m n=3

12.1 1.6 1019 n=2


= 2.18 1018 12(1 1/m2)
1 n=1
or 1 2 = 0.888 or m=3
m
Three lines are emitted. The shortest wavelength corresponds to the greatest energy gap.
hc 1 1
min = E E = E 2 2
3 1 0 1 3

6.6 1034 3 108 9


or min = = 1.02 107 m = 1020
2.18 1018 8

Example 37. A hydrogen like atom (atomic number Z) is in a higher excited state of quantum number n. The
excited atom can make a transition to the first excited state by successively emitting two photons of energy
10.2 and 17.0 eV respectively. Alternatively, the atom from the same excited state can make a transition to
the second excited state by successively emitting two photons of energies 4.25 eV and 5.95 eV respectively.
Determine the values of n and Z. (lonization energy of H atom = 13.6 eV)
Sol. From the given conditions
En E2 = (10.2 + 17) eV = 27.2 eV ...(1)
and En E3 = (4.25 + 5.95) eV = 10.2 eV ...(1)
Equation (1) and (2) gives
E3 E2 = 17.0 eV
2
or Z (13.6)(1/4 1/9) = 17.0
Z2 (13.6) (5/36) = 17.0 Z2 = 9 Z=3
From equation (1)
Z2 (13.6) (1.4 1/n2) = 27.2
or (3)2(13.6) (1/4 1/n2) = 27.2
or 1/4 1/n2 = 0.222 or 1/n2 = 0.0278
or n2 = 36 n=6

ATOMIC EXCITATION WITH THE HELP OF COLLISION


(a) An atom can be excited to an energy above its ground state by a collision with another particle in which part
of their joint kinetic energy is absorbed by the atom.
(b) An excited atom returns to its ground state in an average of 108 s by emitting one or more photons.

www.physicsashok.in 53
ATOMIC PHYSICS

(c) Energy transfer is a maximum when the colliding particles have the same mass.
hc
(d) The energy used in this process will be of discrete nature. (E = E2 E1 = hv = ).

NOTE: If the joint kinetic energy of colliding particles is less than 20.4 eV (considering particles as H-
atoms or one neutron and one H-(atom) then the nature of collision will be necessarily elastic.

Example 38: A neutron moving with speed v makes a head-on collision with a hydrogen atom in ground state
kept at rest. Find the minimum kinetic energy of the neutron for which inelastic (completely or partially)
collision may take place. The mass of neutron = mass of hydrogen = 1.67 1027 kg.
Sol: Let us suppose that neutron and H-atom move at speeds v1 and v2 after the collision. Suppose an energy
E is used in this way.
On the basis of conservation of linear momentum and energy, we can write
mv = mv1 + mv2 .....(i)
1 1 2 1 2
mv2 = mv1 + mv 2 + E .....(ii)
2 2 2
From equation (i) we have
v2 = v12 + v 22 + 2v1 v2 .....(iii)
Now, from equation (ii)
2E
v2 = v12 + v 22 + .....(iv)
m
From equation (iii) and (iv)
2E
2v1v2 =
m
Hence, (v1 v2) = (v1 + v2)2 4v1v2
2

4E
= v2
m
4E
Since v1 v2 is always real, v2 0
m
mv2 4E
1
mv2 2E
2
The minimum kinetic energy of the neutron needed for an inelastic collision corresponds to transition from
n = 1 to n = 2
1
Kmin = mv2min = 2 10.2 eV
2
Kmin = 20.4 eV

Example 39. Consider an excited hydrogen atom in state n moving with a velocity v (v << c). It emits a photon
in the direction of its motion and changes its state to a lowr state m. Apply momentum and energy conservation
principle to calculate the frequency v of the emitted radiation. Compare this with the frequency v0 emitted if
the atom were at rest.
Sol. Let En and Em be the energies of electron in nth and mth states. Then
www.physicsashok.in 54
ATOMIC PHYSICS

En Em = hv0 ...(1)
In the second case when the atom is moving with a velocity v. Let v be the velocity of atom after emitting
the photon. Applying conservation of linear momentum,
m m
v v v

hv
mv = mv + (m = mass of hydrogen atom)
c
hv
or v = v ...(2)
mc
Applying conservation of energy
1 1
En + mv2 = Em + mv2 + hv
2 2
1
or hv = (En Em) + m(v2 v2)
2
2
1 2 h
hv = hv0 + m v v mc
2

1 2 2 h 2 2 2hv
hv = hv0 + m v v
2 m2c2 mc

h v h 2 2
hv = hv0 +
c 2mc2
h 2 2
Here the term is is very small. So, can be neglected
2mc 2
h v h v
hv = hv0 +
c c
1
v v v
or v 1 v 0 or v = v0 1 ; v v0 1 as v<<c
c c c

Example 40: The wavelength of D1 and D2 lines of sodium are 5890 and 5896 respectively, if their mean
wavelength is 6000 then find the difference of excited energy states.
hc hc
Sol: E= E =
2
6.62 1034 3 108 6 1010
E =
6000 6000 1020
E = 3.31 1022 J
3.311022
E = , 2 103 eV
1.6 1019

C39: A lithium atom has three electrons. Assume the following simple picture of the atom. Two electrons move
www.physicsashok.in 55
ATOMIC PHYSICS

close to the nucleus making up a spherical cloud around it and the third moves outside this cloud in a circular
orbit. Bohrs model can be used for the motion of this third electron but n = 1 states are not available to it.
Calculate the ionization energy of lithium in ground state using the above picture.
Sol: In this picture, the third electron moves in the field of a total charge +3e 2e = +e. Thus, the energy are the
same as that of hydrogen atoms. The lowest energy is
E1 13.6eV
E2 = = = 3.4 eV
4 4
Thus , the ionization energy of the atom in this picture is 3.4 eV.

Example 41: Find the wavelengths in a hydrogen spectrum between the range 500 nm to 700 nm.
Sol: The energy of a photon of wavelength 500 nm is
hc 1242 eV nm
= = 2.44 eV
500nm
The energy of a photon of wavelength 700 nm is
hc 1242 eV nm
= = 1.77 eV
700nm
The energy difference between the states involved in the transition should, therefore, be between 1.77 eV
and 2.44 eV.
Figure shows some of the energies of hydrogen states. It is clear that only those transitions which and at n
= 2 may emit photons of energy between 1.77 eV and 2.44 eV. Out of these only n = 3 to n = 2 falls in the

proper range. The energy of the photon emitted in the transition n = 3 to n = 2 is E = (3.4 1.5)eV = 1.9
eV. The wavelength is
hc
=
E
1242eV nm
= = 654 nm.
1.9eV

Example 42: Calculate the (a) velocity, (b) energy, and (c) frequency of the electron in first Bohr orbit of
hydrogen atom.
Ze 2
Sol. (a)We have, vn = ; but here Z = 1 and n = 1
2 0 nh

e2
v1 =
2 0 nh

(1.6 1019 ) 2 1 36109


=
2 1 6.62 1034
= 2.18 106 m/sec
me4 Z2
(b) We have, En = 2 2 2
8 0 n h
Again here, z = 1 and n = 1

www.physicsashok.in 56
ATOMIC PHYSICS

9.11031 (1.6 1019 ) 4 12 (4 9 109 )2


E1 =
8 12 (6.62 1034 )2
= 21.758 1019 joule
21.758 1019
=
1.6 1019
= 13.6 eV
(c) We have,
Z2 me 4
v = ; here also n = 1 and Z = 1
4 02 n 3 h 3

me 4
= = 6.57 1015 Hz.
4 02 h 3
C40: Find out the radius of the hydrogen atom in ground state.
h 2 n 2 0
Sol. We have, rn = ; here Z = 1 and n = 1
mZe2
(6.62 1034 )2 1
r1 =
4 9 109 1 (1.6 1019 )2 (9.11031 )
r1 = 0.53

Example 43: If the wavelength of the first member of the Balmer series of hydrogen spectrum is 6562 , then
calculate the wavelength of first member of Lymen series in the same spectrum.
Sol. We have, for the first member of the Balmer series
1 1 5
v1 = R 2 2 = R
2 3 36
and for the first member of Lyman series,
1 1 3R
v2 = R 2 2 =
1 2 4
1 5R 4 5
= 1 = =
2 2 36 3R 27

51 5 6552
2 = = = 1215.18
27 27

Example 44. The hydrogen atom in its ground state is excited by means of monochromatic radiations of wavelength
975 . How many different lines are possible in the resulting spectrum? Calculate the longest wavgelength
among them. You may assume the ionization energy for hydrogen atom to be 13.6 eV, the Planck constant
= 6.63 1034 Js.
Sol. En = E0Z2/n2. The energy required to take the electron from n = 1 to infinity is the ionization energy of the
hydrogen atom.
13.6 = E0(1/12 1/) or E0 = 13.6 eV

www.physicsashok.in 57
ATOMIC PHYSICS

Therefore, for hydrogen En = 13.6/n2 eV


The energy of the photon incident on hydrogen is
hc 6.63 1034 3 108
E h 10
2.04 1018 J
975 10
Let the electron jump from n = 1 to n = m after absorbing the incident photon.
Then E = Em E1 = 13.6(1/12 1/m2)eV
= 13.6(1 1/m2) 1.6 1019 J
13.6(1 1/m2) 1.6 1019 = 2.04 1018
(1 1/m2) = 0.9375 or m=4
The resulting transitions are shown in the figure. So there are six possible lines. The longest wavelength
corresponds to the minimum energy gap. Hence longest wavelength corresponds to transition from m = 4 to
m=3
1 1 m=4
h E 0 2 2 m=3
3 4

hc 1 1 m=2
E0
9 16

hc 144 E0 m=1

7E 0

6.63 1034 3 108 144



7 13.6 1.6 1019
= 1.88 106 m = 18800 .

Example 45: The energy of an excited hydrogen atom is 3.4 eV. Calculate the angular momentum of the
electron according to Bohrs theory.
Ans: 2.11 102 joule sec.
Since the energy of an electron in nth level in hydrogen atom is
RCh 13.6
En = 2 or, 3.4 = 2
n n
RCh = 13.6 eV
n=2
h
From Bohrs theory, L = n
2
0.6 1034
L=2 = 2.11 103 joule sec.
2 3.14

REDUCED MASS
.In our earlier discussion we have assumed that the nucleus (a proton in case of hydrogen atom) remains at
rest. With this assumption the values of the Rydberg constant R and the ionization energy of hydrogen
predicted by Bohrs analysis are within 0.1% of the measured values.
Rather the proton and electron both revolve in circular orbits about their common centre of mass. We can
www.physicsashok.in 58
ATOMIC PHYSICS

take the motion of the nucleus into account simply by replacing the mass of electron m by the reduced mass
of the electron and nucleus.
Mm
Here = .....(i)
Mm
where M = mass of nucleus. The reduced mass can also be written as,
m
= 1 m
M
m
Now, when M > > m, 0 or m
M
For ordinary hydrogen we let M = 1836.2 m. Substituting in
v
equation (i), we get = 0.99946 m when this value is used instead
of the electron mass m in the Bohr equations, the predicted values
are well within 0.1% of the measured values. m+ m
cm
The concept of reduced mass has other applications. A positron
has the same rest mass as an electron but a charge +e. v
A positronium atom consists of an electron and a positron, each
with mass m, in orbit around their common centre of mass. This Separation r
6 Applying
structure lasts only about 10 s before two particles annihilate The electron and the Bohr model to positronium.
the positron revolve about
(combine) one another and disappear, but this is enough time to their common centre of mass, which is located
midway between them because they have equal mass
study the positronium spectrum. The reduced mass is m/2, so the
energy levels and photon frequencies have exactly half the values
for the simple Bohr model with infinite proton mass.
Now, let us prove why m is replaced by the reduced mass when
motion of nucleus (proton) is also to be considered. r1 r2
M m
In figure both the nucleus (mass = M, charge = e) and electron CM
(mass = m, charge =e ) revolve about their centre of mass (CM)
with same angular velocity () but different linear speeds. Let r1
and r2 be the distance of CM from proton and electron.
Let r be the distance between the proton and the electron. Then,
Mr1 = mr2 .....(ii)
r1 + r2 = r .....(iii)
mr Mr
r1 = and r2 = .....(iv)
Mm Mm
Centripetal force to the electron is provided by the electrostatic force. So,
1 e2
2
mr2 =
40 r 2

Mr 2 1 e2
or m = .
Mr 40 r 2

Mm 3 2 e2
or r =
Mm 4 0

www.physicsashok.in 59
ATOMIC PHYSICS

e2
3 2
or r = .....(v)
4 0
Mm
where =
Mm
Moment of inertia of atom about CM,
Mm 2
I = Mr12 mr22 = r = r2 .....(vi)
MM
According to Bohrs theory,
nh nh
= I r2= .....(vii)
2 2
Solving equations (v) and (vii) for r, we get
0 n 2 h 2
r = .....(viii)
e 2
Electrical potential energy of the system,
e2
U =
4 0 r
1 2 1 2 2
and kinetic energy, K = I = r
2 2
e2 e2
2
From equation (v), = , K=
40 r3 8 0 r

e2
Total energy of the system, E= K+U=
8 0 r
Substituting value of r from equation (viii), we have
e4
E = 2 2 2 .....(ix)
80 n h

me 4
The expression for En without considering the motion of proton is En = 2 2 2 , i.e., m is replaced by
80 n h
while considering the motion of proton.

NOTE :(i) Variation of rn, vn and En with mass of election is as under,


1
rn , vn = independent of m and En m
m
Sometimes the electron is replaced by some another particle which has a charge e but mass different
from the mass of electron. Here, two cases are possible.
Case 1: Let say mass of the replaced particle is x times the mass of the electron and nucleus is still very
heavy compared to the replaced particle, i.e., the motion of the nucleus is not to be considered. In this case

www.physicsashok.in 60
ATOMIC PHYSICS

1
rn will become times, vn will remain unchanged and En becomes x times.
x
Case 2: In this case motion of nucleus is also to be considered, i.e., mass of the replaced particle is
comparable to the mass of the nucleus. In this case the mass of the electron is replaced by the reduced mass
of the nucleus and the replaced particle. Let say the reduced mass is y time the mass of the electron.
1
Then, rn will become times, vn remains unchanged and En becomes y-times.
y
m1m 2
(ii) Reduced mass m = m m of m1 and m2 is less than both the masses.
1 2

C41: A positronium atom is a system that consist of a positron and an electron that orbit each other. Compare
the wavelengths of the spectral lines of positronium with those of ordinary hydrogen.
Sol: In this case reduced mass will be given by
mM m2 m
m = = =
mM 2m 2
where m = mass of the electron.
Hence, the energy levels of a positronium atom are
m' E E
En = 21 = 12
mn 2n
It means that the Rydberg constant for positronium is half as large as it is for H-atom. As a result the
wavelength in the positronium spectral lines are all twice those of the corresponding lines in the H-spectrum.

Example 46: Bohrs theory assumes that nucleus is of infinite mass and so electron rotates round the stationary
nucleus. Assuming the nucleus to be of finite mass MH, the value of correct Rydberg constant will be
(consider hydrogen atom)
mM H e 4 MH m e4
(a) M m 8 2 ch 3 (b) mM 8 2 c 2 h 3
H 0 H 0

mM H e4 e4
(c) M m 8 2 ch 3 (d)
H 0 8 02 ch 3
Sol: (c) In this case both electron and nucleus will rotate about a common centre of mass, say O. Suppose that
the radii of the electron and nucleus orbits are re and rn respectively then by definition of centre of mass; the
electron, the centre of mass O and the nucleus are always in a straight line.
MHrn = mre when m is the mass of electron
rn m
=
re rn m MH
mr MH r
Let r = re + rn then rn = m M and re = m M
H H

From Bohrs quantization rule, we have

www.physicsashok.in 61
ATOMIC PHYSICS

nh
mre2 M H rn2 = [ is the angular velocity about the centre of mass]
2
Putting the values of rn and re, we get

mM H nh
r2 =
m M H 2
nh
The above equation can be compared with mr2 =
2
The Rydberg constant will be given by
mM H e4
R = mM 2 3
H 8 0 ch

mM H
Obviously the reduced mass of the electron is m M .
H

Example 47. A positromium atom is a bound system of an electron (e) and its antiparticle positron (e+) revolving
about their centre of mass. Find the wavelength of the radiation when the system de-excites from its first
excited state to the ground state.
Sol. This problem can be solved by Bohrs theory of the hydrogen atom by replacing the mass of the electron by
its reduced mass.
me me m
e
me m e 2
m
Z2e4 e
2
En 2 2 2
80h n
e4me Z2
En 2 2 2
80 h 2n
En = 2.18 1018(12/2n2) ( e4me/802h2 = 2.18 1018)
When n = 1, E1 = 2.18 1018/2
When n = 2, E1 = 2.18 1018/8
E = E1 E2 = 2.18 1018(1/2 1/8)
= 2.18 1018 3/8
E = hc/ = 2.18 1018 0.375


6.63 10 34

3 108
, = 1.2165 107 m = 2433
2.18 10 18
0.375

Example 48: The mass of muon ( ) is 207 times that of the electron and charge = 1.6 1019 C. A muon can
be captured by a nucleus to form a muonic atom. Calculate the value of ionization energy of the muonic
atom.

www.physicsashok.in 62
ATOMIC PHYSICS

Sol: The ionization energy of the muonic atom is obtained by replacing me in H-atom formula by the reduced
mass m of the proton-muon system. This reduced mass is
m p m
m = m m
p

1836me 207m e
= 1836m 207m
e e
186 me
Thus the ground state energy is (n = 1, Z = 1).
2 2 mk 2 e 4
E1
h2
m 2 2 m e k 2 e 4
E1
me h2
E1 = 186 13.6 eV
The ionization energy
= E1 = 186 13.6 eV = 2.53 KeV

PRODUCTION OF X-RAYS
X-rays were accidentally discovered by Wilhelm Rontgen in 1895 during the course of some experiments
with a discharge tube. At present, it is well known that these rays are produced whenever fast moving
electrons strike a high atomic weight solid like tungsten kept in vacuum.

(a) X-ray Tube:


The essential elements of a modern Coolidge X-ray vacuum tube which is widely used for commercial and
medical purposes are shown in figure. Electrons are produced thermionically from a tungsten filamentary
cathode F which is heated to incandescence either by a storage battery or by a low-voltage alternating
current from a stepdown transformer T2. These electrons are focussed on the target T with the help of a
cylindrical shield S which surrounds F and is maintained at a negative potential. The electrons are accelerated
to very high speeds (upto 10% of velocity of light) by the d.c. potential difference (of about 50 kV - 100
kV) applied between F and the anode (also called anticathode). This high d.c. potential is obtained from a
step-up transformer T1 whose output is converted into direct current by full-wave rectifier and a suitable
filter.
+
50 kV
Electrons
S A
F T
B
Cooling
Tube Fins

X-rays
The target T usually employed in X-ray tubes is a massive block of tungsten or in many cases, a molybdenum
www.physicsashok.in 63
ATOMIC PHYSICS

plug embedded in the face of a solid copper anode. The face of the copper anode is sloped at about 45 to
the electron beam. Being very good conductor of heat, copper helps to conduct heat efficiently to the
external cooling fins or the water-cooling system. Under the terrific bombardment of the target by so many
electrons, most metals will melt. That is why metals like tungsten, platinum and molybdenum etc. are used
which have high melting points and also have a high atomic weight (which is essential for abundant production
of X-rays).
When the electrons strike the tungsten target, they give up their kinetic energy and thereby produce X-rays.

Tube
Rectifier
T1 S A
T2
F T

Cooling
Water

X-rays

(b) Control of Intensity and Quality


The intensity of X-rays depends on the number of electrons striking the target. This number is determined
by the temperature of the electron-emitting filament which itself is proportional to the heater current. Hence
by controlling the filament current with the help of a rheostat R, thermionic emission and hence intensity of
X-rays can be controlled.
The quality of X-rays is measured in terms of their penetrating power which is dependent on the potential
difference between filamentary cathode and the anode. Greater this accelerating voltage, higher the speed
of the striking electrons and consequently, more penetrating the X-rays produced. It is customary to refer to
highly penetrating X-rays (i.e. those possessing high frequency) as hard X-rays and to those less penetrating
(i.e. of low frequency) as soft X-rays. Obviously, the quality or penetrating power of X-rays can be controlled
by varying the potential difference between the cathode and anode.
It will be noticed from above explanation that in coolidge X-ray tube, it is possible to achieve separate
control of the intensity and quality of X-rays independent of each other.
It has been found that apart from the intensity and quality of X-rays, their abundance depends on the atomic
weight of the target material. Target materials of higher atomic weights yield a greater abundance of X-rays
than those of lower atomic weights.

ORIGIN OF X-RAYS
X-rays are produced when high-speed electrons strike some material object. However, majority of the
electrons that strike a solid target, do nothing spectacular at all. Most of them undergo glancing collisions
with the matter particles, lose their energy a little bit at a time and thus merely increase the average kinetic
energy of the particles of the target material. It is found that nearly 99.8 percent of the energy of the electron
beam goes into heating the target.
But a small number of the bombarding electrons produce X-rays by losing their kinetic energy in the following
two ways:
(i) Some of the high-velocity electrons penetrate the interior of the atoms of the target material and are attracted

www.physicsashok.in 64
ATOMIC PHYSICS

by the positive charge of their nuclei. As an electron passes close to the positive nucleus, it is deflected from
its path as shown in figure. The electron experiences deacceleration during its deflection in the strong field of
the nucleus. The energy lost during this de-acceleration is given off in the form of X-rays of continuously
varying wavelength (and hence frequency). These X-rays produce continuous spectrum when analysed by
Bragg spectrometer. This spectrum has a sharply defined short-wavelength limit min (or high-frequency limit
fmax) which corresponds to the maximum energy of the incident electron.

v X-Ray

Continuous
1 mv2

spectrum
2 X-Ray
+ +
+ + X-Ray

1 mv2 v
2

If, as shown in figure, the striking electron has its velocity reduced from v to v during its passage through the
1 1
atom of the target material, then its loss of energy is = ( mv2 mv2 ). This must equal the energy of the
2 2
X-ray photons emitted.
1
m(v2 v2) = hv
2
The highest or maximum frequency of the emitted X-rays corresponds to the case when the electron is
completely stopped i.e. when v = 0. In that case
1
mv2 = hvmax .....(i)
2
If the electron is accelerated through a potential of V volts, then
1
mv2 = eV .....(ii)
2
From (i) and (ii), we get
hvmax = eV; vmax = eV/h
Now, hvmax = hc/min [ c = v .]
hc/min = eV or min = hc/eV
Substituting the values of e = 1.602 1019 C
h = 6.62 1034 J-s and c = 3 108 m/s, we get
6.62 1034 3 108 1.24 106
min = = m
1.602 1019 V V
12400
or min = [1 = 1010 m]
V
Such X-rays are very aptly called braking radiations because they are due to braking or slowing down of
high-velocity electrons is the positive field of a nucleus. These radiations constitute, as said earlier, the
continuous spectrum of the X-rays because they consist of a series of uninterrupted wavelengths having a
sharplydefined short-wavelength limit min. These X-rays are independent of the nature of the target material
but are determined by the potential difference between the cathode and anode of the X-ray tube.
(ii) Some of the high-velocity electrons while penetrating the interior of the atoms of the target material, knock

www.physicsashok.in 65
ATOMIC PHYSICS

off the tightly bound electrons in the innermost shells (like K, L-shells etc) of the atoms. When electrons
from outer orbits jump to fill up the vacancy so produce, the energy difference is given out in the form of X-
rays of definite wavelength (and frequency). These wavelengths constitute the line spectrum which is
characteristic of the material of the target.

e X-Ray

e
K-line

K K
L e L
M M

(a) (b)
Figure (a) shows the case when the high-velocity incident electron knocks off one electron from the K-
shell. As shown in figure (b), this vacancy in K-shell is filled by a nearby electron in the L-shell. During the
jump an X-ray radiation is emitted whose frequency is given by
Ek E = hv
where Ek is the energy required to dislodge an electron form the K-shell and El is that required for L-shell.
Since this energy difference is comparatively very large, the X-rays emitted have very large energy content
and hence are highly penetrating.
If, however, this vacancy in K-shell is filled up by an electron jumping from M-shell, the X-rays emitted
would be still more energetic and would consequently possess still higher frequency because E = (Ek Em)
is more than E = (Ek EL). Such X-rays arising from millions of atoms produce the K-lines as shown in
figure. Usually, two lines K and K of this series are detected although there are many more.
Similarly, when the incident electron carries some what lesser amount of energy, it dislodges an electron
from the L-orbit and an electron either from M-orbit or other outer orbits takes its place so that X-rays of
frequency lower than that of the K-series are produced. This gives the L-series of the X-ray spectrum as
shown by K, L and L lines in figure (a).
Energy in eV
SHELL
K E=0
2 O
K 20
L N
L L M M
L 200 M
M L L
K
L 2000 L
M M
K K
N
O 20,000 K
(a)
(b)
Spectral lines of M-series are produced in a similar way as shown in the energy-level diagram of figure (b).
As stated earlier, these K, L and M series constitute the line spectra of the X-rays which are characteristic
of the material used as target in the X-ray tube.
Hence, the X-rays produced by an X-ray tube consist of two parts:
(i) one part consists of a series of uninterrupted wavelengths having a short cut-off wavelength min. This

www.physicsashok.in 66
ATOMIC PHYSICS

constitute the continuous spectrum and


(ii) the other part consists of a number of distinct and discrets wavelengths which constitute the line or
discontinues spectrum of the X-rays.

X-ray Spectrum
As explained in X-ray spectrum consists of
(i) continues spectrum and (ii) line spectrum. These two are shown in figure.
(a) Continuous Spectrum
(i)It is produced due to the de-acceleration of high-velocity electrons when they are deflected while passing
near the positively, charged nucleus of an atom of the target material.
K L
K L
L
Intensity

Continuous
Continuous Spectrum
Spectrum K K L L L

min f
(a)
(ii) It has a sharply-defined short wavelength limit given by
12.400 12, 400
min = 1010 m or
V V
(iii) The cut-off wavelength min is independent of the nature of the target material but is inversely proportional
to the potential difference between the cathode and anode of an X-ray tube. The value of min decreases as
this potential difference is increased.
(iv) The intensity of the continuous spectrum (given by the area Tungsten
enclosed by the curve of figure (b) is found very nearly proportional Target
to the square of the applied voltage for a given target and to the K L L
atomic number of the target material when a constant potential K 25 kV L
difference is applied.
X-ray Intensity

(v) There is a shift of the maximum intensity position towards the 20 kV


short wavelength side as voltage is increased.
15 kV
(b) Line Spectrum
10 kV
(i) It is produced when electrons are dislodged from the innermost
5 kV
orbits of the atoms of the target material followed by electron jumps
from outer orbits. 0 1.0 2.0 3.0 4.0
(ii) It consists of discrete spectral lines which constitute K-series, L-series Wave length(A.U.)
and M-series etc. K-series consists of those lines for which electron jumps end at K-level. (b)
(iii) K-series being most energetic constitute the hard X-rays whereas L- and M-series form the soft X-
rays.
(iv) Line spectrum is characteristic of the target material used. In

www.physicsashok.in 67
ATOMIC PHYSICS

fact, X-rays constituting the line spectrum are known as K K


characteristic X-rays. The number of lines present in the spectrum Cr 24

Mass No. (Z)


depends both on the nature of target material and the excitation Cu 28
voltage. Mo 42
(v) There is a regular shift towards shorter wavelength in the K- Sn 50
spectrum as the atomic number of the target is increased figure (c). Au 79
The exact relationship, as found by Moseley, is Cf 90

2 1 (Z1 1) 2 (c)
1 = 2 = (Z2 1)2
where v1 is the frequency of the K line for a target material having an atomic number of Z1 and v2 and Z2
are similar quantities for some different target material.

C42: An X-ray tube works on 60,000 V. What will be the wavelength of X-ray emitted in it.
12, 400
Sol: min =
V
Here, V = 60,000 V
12, 400
min =
60, 000
= 0.2
C43: If the potential difference applied across an X-ray tube is 12.4 kV and the current through it is 2 mA,
calculate:
(i) the number of electrons striking the target per second
(ii) the speed with which they strike it
(iii) the shortest wavelength emitted
Take e = 1.6 1019 C and m = 9.1 1031 kg
Sol: (i) If n is the number of electrons striking the anode per second, then I = ne
I 2 10 3
n= = = 1.25 1016 s1
e 1.6 1019

2eV
(ii) v 5.93 105 V
m
v = 5.93 105 v = 5.93 105 12, 400
= 6.6 107 m/s
12, 400 12400
(iii) min = = = 1
V 12400

C44: Calculate the minimum applied potential required to produce X-rays of 1 wavelength.
12400
Sol: min =
V
12400 12400
V= = = 12.4 kV
min 1
C45: An X-ray tube passes 5 mA at a potential difference of 100 kV. Calculate the maximum speed of the

www.physicsashok.in 68
ATOMIC PHYSICS

electrons striking the target and the rate of production of heat at the target if only 0.1 percent of the incident
energy is converted into X-radiations. Take e/m = 1.76 1011 C/kg and J = 4.18 joules/cal.
2eV
Sol: v 5.93 105 V
m
v = 5.93 105 100, 000 = 1.88 108 m/s
Incident power = 100,000 5 103 = 500 W
Power converted into heat = 99.9% of 500 = 499.5 W 5 mA
499.5 +
Heat produced / second = = 119 cal/s. 100 kv
4.18
C46: An X-ray tube operated at 30 kV emits a continuous X-ray spectrum with a short wavelength limit
min = 0.414 . Calculate Plancks constant h if e = 1.602 1019 C and c = 3 108 m/s.
ch
Sol: min =
eV
eV min
h =
c
1.602 1019 30 103 0.414 1019
=
3 108
= 6.63 1034 J-s

MOSELEYS LAW
In 1913-14, Moseley carried out a systematic study of the characteristic X-ray spectra of various elements
used as targets in an X-ray tube. By using Braggs spectrometer for the purpose, the remarkably similar to
each other in the sense that each consists of K-L and M-series. However, there is one very important
difference. The frequency of lines (in every series) produced from an element of higher atomic number is
greater than that produced by an element of lower atomic number. It is due to the fact that binding energy of
electrons increases as we go from one element to another of higher atomic number. Because there is greater
positive charge on the nucleus of an element of higher atomic number, larger amount of energy is required to
liberte an electron from the K, L and M shells of that element.
Consider the K line of the characteristic X-ray spectrum of any element. It is found that higher the atomic
number of the target material, higher is the frequency of the K line produced by it. The exact mathematical
relationship between frequency and atomic number is given by
v (Z b)2
or (Z b)
or = a(Z b)
where Z is the atomic number of the element and a and b are constants for a particular series but vary from
one series to another i.e. their values for K-series are different from those for L-series etc. The constant b
is known as nuclear screening constant. For lines of K-series, b = 1. Its values for lines of L-series is more.
The above relation is known as Moseley law for the characteristic or line X-ray spectrum. It may be stated
as follows:
The frequency of a spectral line in the characteristic X-ray spectrum varies directly as the square of the
atomic number of the element emitting it.
Figure shows Moseley diagram for K and K lines which is

www.physicsashok.in 69
ATOMIC PHYSICS
Sn-50 K
obtained by plotting versus atomic number of different

Atomic number
Ze-40
K
elements of the periodic table. As expected, the graph is linear. Zn-30
An exact form of Moseleys law is
Ca-20
1 1 1
= R(Z )2 n 2 n 2 Al-13
1 2 10
5 8
10 15 20 25 10
where R is Rydbergs constant, Z the atomic number, a correction Frequency
factor and n1 and n2 the principal quantum numbers of the energy levels between which the transition
occurs.

Importance of Moseley law


The great significance of Moseley law lies in the fact that it proves for the first time that it is the atomic
number and not the atomic weight of an element which determines its characteristic properties (both physical
and chemical). It provides the proper guideline that elements must be arranged in the periodic table according
to their atomic numbers and not their atomic weights.
Accordingly, Moseley law has been used to place elements in their proper sequence in the periodic table in
certain questionable cases. For example, if we go by the atomic weight, potassium (19K39) should come
before argon (18A40) and similarly, nickel (28Ni58.7) should precede cobalt (27Co58.9). But Mosely law dictates
that as per their atomic numbers, their order should be just opposite of the above. This fact is further
supported by the chemical properties of these elements.
Moseley law has led to the discovery of new elements like hafnium (72), promethium (61), technetium (43)
and rhenium (75) etc. by the indication of gaps in Mosely diagram. This law has been also helpful in determining
the atomic number of rare earths thereby fixing their position in the periodic table.
It can be shown that Moseley law is in accordance with Bohrs theory of spectral emission from atoms. As
shown in when an electron jumps from an orbit n2 to the orbit n1, the frequency of the radiation given out is,
me 4 1 1
v= 2 3 .Z 2
2 2
8 0 h n1 n 2
This may be put as
me 4 1 1
v = 8 2 h 3 n 2 n 2 Z2
0 1 2
2
or vZ
or Z
Bohr did not take into account the screening effect of electrons whereas Moseley did. That is why the
expression becomes
(Z b)

Formulae for K- and L-series of X-ray Spectrum


The frequencies of the various lines in the K- and L-series of the X-ray line spectrum are given by the
following empirical formulae.
K-series: The general formula is
1 1
= R(Z 1)2 1 2 where n = 2, 3 etc
n
Here, the nuclear screening constant is unity.

www.physicsashok.in 70
ATOMIC PHYSICS

(i) For K line, n = 2.


1 1 3R
= R(Z 1)2 1 = (Z 1)2
4 4
(ii) For K line, n = 3
1 1 8R
= R(Z 1)2 1 = (Z 1)2
9 9
L-Series: The general formula is
1 1 1
= R(Z 7.4)2 2 where n = 3, 4 etc.
4 n
Here, screening constant is 7.4.
(i) For H line, n = 3
1 1 1 5R
= R(Z 7.4)2 = (Z 7.4)2
4 9 36
(ii) For H line, n = 4
1 1 1 3R
= R(Z 7.4)2 = (Z 7.4)2
4 16 16

C47: Find the nuclear screening constant for the L-series of X-rays if it is known that X-rays with a wavelength
of = 1.43 are emitted when an electron in a tungsten atom (Z = 74) is transferred from the M-level to
L-level. Take Rydberg constant = 10.97 106 m1.
Sol: When electron jumps from M to L-level, the first member of the L-series i.e. L line is given out. Its
wavelength as given by Moseleys law is
1 1 1 5R
= R(Z b)2 2 2 = (Z b)2
2 3 36
Substituting the given values, we have
1010 5
= 10.97 106 (74 b)2
1.43 36
(74 b)2 = 4589.8; (74 b) = 67.75 ; b = 6.25.

Example 49: The K and L absorption edges of copper occur at wavelengths 1.380 and 11.288
respectively. Calculate the atomic number of copper.
Sol: It should be remembered that absorption ages are found in the absorption spectrum of X-rays. The absorption
edge of each series (of line spectrum) represents the limit of that series. In other words, the short-wavelength
limit of each series is called its absorption edge and is written as . It may be obtained by putting n = in
the formulae. Moreover, for K-series, the value of screening constant for K member is 3.3 (instead of 1 for
other members). Corresponding value for L-series is 11(instead of 7.4).
1 1
K-Series: = R(Z 3.3)2 1 2 = R(Z 3.3)2

10
10
= (Z 3.3)2
1.38
1 1 1 R
L-Series: = R(Z 11)2 2 = (Z 11) 1)2
4 4
www.physicsashok.in 71
ATOMIC PHYSICS

1010 R
= (Z 11) 1)2
11.288 4
Dividing one by the other, we get
(Z 3.3)2 11.288
2 = = 2.045
(Z 11) 4 1.38
(Z 3.3)
or = 1.43
(Z 11)
or Z = 29

Example 50: An impure tungsten target emits a strong K line of = 0.21 and a weak K line of = 1.537
. Can you identify the impurity taking the nuclear screening constant as unity. Given for tungsten, Z = 74,
for Ni = 28.
Sol: The wavelength for K line is given by the relation
1 1 3
= R(Z 1)2 1 = R(Z 1)2
4 4
For tungsten
1 3
10 = R(74 1)2
0.21 10 4
15,987R
=
4
For impurity
1 3
10 = R(Z 1)2
1.537 10 4
Dividing one by the other, we get
1.537 5329
= ; Z = 28
0.21 (Z 1) 2
Obviously, impurity is nickel whose atomic mass number is 28.

Absorption of X-rays
When a narrow and monochromatic beam of X-rays passes through matter, part of it is absorbed and the
remaining part is transmitted. Absorption of x-rays can be studied with the help of the apparatus shown in
figure. The X-rays produced by an X-ray tube are first made into a well defined narrow beam by passing
them through two fine slits S1 and S2 in the two lead plates. The beam is then monochromatised by Bragg
reflection from a crystal (not shown in the figure) and allowed to enter the ionization chamber which measures
the ionization current. The strength of the ionization current is a measure of the intensity of the X-ray.
Next, a sheet of the absorbing material is interposed in the path of the X-ray beam before it enters the
ionization chamber. It is found that the ionization current and hence the intensity of X-rays is reduced by
I
their passage through the absorber sheet. The ratio I of the two ionization currents can be used to
0

measure the absorption coefficient of the material.

www.physicsashok.in 72
ATOMIC PHYSICS
L1 L2

Absorber
Ionization
S1 S2 Chamber
Let I0 be the initial intensity of a homogeneous X-ray beam incident normally on an absorber sheet and I the
intensity after the beam has travelled a thickness x of the absorber.
If dI is the further decrease in intensity over a thickness dx of the absorber
I0
dI
figure, then gives the rate of decrease of intensity with thickness.
dx
Assuming that this rate is proportional to the intensity I, we have
dI I (I dI)
I
dx
x dx
dI
or = I
dx
where is a constant of proportionality and is called the linear absorption coefficient of the absorber (it is
also known as macroscopic absorption coefficient or linear attenuation coefficient).
dI
Now, = .dx .....(i)
I
Integrating both sides of the above equation, we get

dI / I = dx
logeI = x + K .....(ii)
The value of the integration constant K can be found from the known initial conditions which are that when
z = 0, I = I0. Substituting these values in equation (ii) above, we have
I0
logeI0 = K
Hence, equation (ii) becomes I
logeI = x + logeI0
log e I
or I0 = x
O x
I
x
or I0 = e
or I = I0ex .....(iii)
It is seen that intensity of the X-ray beam decreases exponentially with the thickness of the absorbing
material as shown in figure.

BRAGGS LAW
Figure gives a 3-dimensional view of how a beam of P
S
monochromatic X-rays undergoes Braggs reflection from
different planes in a NaCl crystal. Figure gives a
A
www.physicsashok.in
Q R73
E
H
M N
G
F B
ATOMIC PHYSICS

2-dimensional view of the same diagram. It shows a beam


of monochromatic X-rays incident at a glancing angle on a
set of parallel planes of NaCl crystal. The beam is partically
reflected at the successive layers rich in atoms Ray no.1 is
reflected from atom A in plane 1 whereas ray no. 2 is reflected
from atom B lying plane 2 immediately below atom A.
Whether two reflected rays will be in phase or antiphase
with each other will depend on their path difference.
This path difference can be found by drawing perpendiculars AM and AN on ray No. 2. Since the two rays
travel the same distance from points A and N onwards, it is obvious that ray no. 2 travels an extra distance
1
= MB + BN 1
2 2
Hence, the path difference between the two reflected beams is A Plane 1
= MB + BN 3
M N d
Plane 2
B
d
= d sin + d sin = 2d sin Plane 3
where d is the interplanar spacing i.e. vertical distance between two adjacent planes belonging to the same
set.
The two reflected beams will be in phase with each other if this path difference equals an integral multiple of
la dn will be antiphase if it equals an odd multiple of /2.
Hence, the condition for producing maxima becomes
2d sin = n
where n = 1, 2, 3 etc., for the first order, second order and third order maxima respectively. This equation
is known as Braggs Law.

PROPERTIES OF X-RAYS
Main properties of X-rays may be summarised as under:
(i) Like visible light, X-rays consist of electromagnetic waves of very short wavelength (or of very high frequency)
and show reflection, refraction, interference, diffraction and polarisation etc.
(ii) They are not deflected by electric and magnetic field.
(iii) They posses high penetrating power and can pass through many solids which are opaque to visible light.
The transparency depends on the density of the material. Higher the density of the substance, the less
transparent it is to the X-rays. For example, sheet of lead 1 cm thick can absorb X-rays whereas aluminium
sheet of same thickness cannot.
The penetrating power of X-rays depends upon (a) the voltage applied across the cathode and anode of
the X-ray tube and (b) the atomic number of the material of the cathode. Greater the accelerating potential
of the X-ray tube and higher the atomic number of its target material, the more penetrating the X-rays
produced.
(iv) They ionize a gas and also eject electrons from metals on which they fall.
(v) They cause fluorescence in many substances like barium, cadmium, tungstate and zinc sulphide etc.
(vi) They suffer compton scattering.
(vii) They have a destructive effect on living tissue. Exposure of human body of X-rays causes the reddening of
skin and surface sores.

Practical Applications of X-rays


On account of their diverse and distinctive properties, X-rays have been put to many uses in different fields
of our daily life i.e. in industry, medicine and research.
(a) Industrial applications: Some of these applications are as under:

www.physicsashok.in 74
ATOMIC PHYSICS

(i) to detect and photograph defects within a body i.e. in its internal structure such as metals, machine parts
intended for with standing high pressures, cracks in wood, porcelain and other insulators, defects in diamonds
and other precious stones, in moulds, forgings and castings etc.
(ii) to analyses the structure of alloys and other rcomposite bodies by determining the crystal form in an ingot
with the help of diffraction of X-rays. In this way, alloys like cobalt-nickel, steel, bronze, duraluminium,
artificial pearls and old paintings have been analysed.
(iii) to study the structure of rubber, cellulose and plastics. The diffraction of X-rays by these substances leads
to valuable information about their molecular grouping.

(b) Applications for pure scientific research:


(i) for investigating the structure of the atom.
(ii) for studying the structure of the crytalline solids and alloys (X-ray crystallography).
(iii) for identification of chemical elements including determination of their atomic numbers.
(iv) for analysing the structure of coraplex organic molecules by examining their X-ray diffraction patterns.

(c) Medical Applications: These can be broadly divided into two classes; one for diagnosis purposes
(radiography) and the other for curative purposes (X-ray therapy).
(i) Radiography:
X-rays are being widely used for detecting fractures, tumours, the presence of foreign matter like bullets
etc. in the human body as well as diseased organs of the boy. It is due to differential absorption of X-rays
between bones, tissues and metals. Radiographs or X-ray photos are used for this purpose. Since bones
are more dense and hence more opaque to X-rays then flesh, a contrasting radiograph of human body can
be obtained for leisurely study by interposing it between the X-ray tube and photo film. Where the organs
do not provide contrast as, for example, the intestines or other fleshy parts of the human body, a artificial
means are adopted to create sufficient contrast between them. In such cases, before taking radiograph,
barium or bismuth meal is given to the patient. This meal consists of milk to which some amount of barium
sulphate or bismuth carbonate has been added. A few hours after the meal has been taken, these powders
settle in the gastrointestinal tract and if a radiograph is taken at that item, the intestines stand out in sharp
contrast to the surrounding tissues due to the fact that absorption coefficient of barium is greater on account
of its high atomic number. In this way, pepticulcers and ruptures etc. in the internal organs of the human body
can be accurately located. Similarly, radiographs are routinely used for the diagnosis of tuberculosis, stones
in kidneys and gall-bladders etc.

(ii) X-ray or Rontgen Therapy


Many types of skin diseases, malignant sores, cancers and tumours have been cured by controlled exposure
to X-rays of suitable quality. This curative power of X-rays is due to the fortunate fact that diseased tissue
is more susceptible to destruction than the surrounding healthy tissue. Unnecessary long exposure of human
body to X-rays produces many injurious effects including the loss of white cells in the blood, sterility and
harmful genetic changes.
X-rays have been used for the identification of different types of cells and tissues and for bringing about
genetic mutations.

www.physicsashok.in 75
MODERN PHYSICS

THINKING PROBLEMS
X-rays
1. What leads you do believe that X-rays are electromagnetic waves ?
2. The occurrence of a lower bound of wavelengths of X-rays produced in an X-ray tube lends support to the
quantum concept of radiation. Explain how.
3. If Youngs experiment is repeated with electron beams interference is observed. Does this mean that an
electron gets divided into two while passing through the slits ?
4. Water irradiated with X-rays is unsafe for drinking. Is this true or false ?
5. An electron moves through a gas-filled region in the presence of a transverse magnetic field. Describe its
motion.
6. Fluerescence is produced by ultraviolet rays but never by infrared rays. Explain why.

7. A neutron, a proton, an electron and an alpha particle enter a region of constant


magnetic field with equal velocities. The magnetic field is along the inward normal
to the place of the paper. The tracks of the particles are labelled in the figure.
Which tracks do the electron and alpha particle follow ?

8. A charged and an uncharged particle have the same momentum. Will they have the same de Broglie wavelength?
9. The electrical conductivity of a gas increases when X-rays or -rays pass through it. Explain this phenomenon.
10. Why are tungsten or platinum widely used as the target the X-ray tubes ?
11. A uniform electric field acts normally on a moving charge. Can the charge be deflected through 90 ?
12. A uniform electric field acts normally on a moving charge. Is work done by the field on the charge (a) as it
enters the field, (b) later ?
13. Does the speed of a charged particle change when (a) a magnetic field, (b) an electric field, acts on it for
some time ?
14. Parallel electric and magnetic fields act on on a charged particle moving perpendicular to these fields.
Describe its subsequent motion.
15. How do you conclude that cathode rays are fast moving negatively charged particles ?
16. Magnetic mirror is a term for the region of a magnetic field in which there is an
intense concentration of lines of induction as shown in the figure. Suppose a charged
particle approaches a magnetic mirror. What will happen to it ?
17. How is a monoenergetic, slightly diverging beam of charged particles focussed by a magnetic field ?
18. X-rays are produced when a fast electron hits a proper target. What happens to the electron ?
19. Why does the target in an X-ray tube become hot ?
20. X-rays can be produced in cathode ray tubes and also in Coolidge tubes. Why are the latter preferred in
actual use ?
21. Why is the wave nature of matter not apparent in our daily lives ?
22. A neutral pion decays into two gamma photons.
0
Why cannot a single photon be born? What conservation law is in contradiction with it ?
2 1
23. Quarks inside protons and neutrons are thought to carry fractional charges e, e . Why are they not
3 3
evidenced in Millikans oil drop experiment ?

www.physicsashok.in 123
MODERN PHYSICS

Atomic Structure, Radioactivity, Nuclear Fassion and Fusion


1. Distinguish between excitation and ionization by collision.
2. According to the theory of electron transitions, the spectral lines from a glowing gas should be sharp i.e.,
of one particular wavelength each. In practice, they are found to be somewhat diffuse, i.e., spread over a
small range of wavelengths. Suggest a reason for this.
3. Can it be concluded from -decay that electrons exist inside the nucleus ?
4. Why are -rays emitted only in nuclear processes and not in orbital electron transitions ?
5. What are the principles that are obeyed in filling the orbits of an atom ?
6. How is the radioactivity of an element affected when it forms chemical compounds ?
7. How can Becquerel ray, i.e., the combination of -, - and -rays, be separated ?
8. When a nucleus undergoes -dcay, is the product atom electrically neutral ? In -decay ?
9. Do -decay and -decay cause a change of element, called transmutation ?
10. Experimental results in radioactivity show small variations from the results predicted by theory. Explain this.
11. Does the relation E = mc2 suggest that mass can be converted to energy only when it is in motion ?
12. What is a thermal neutron ?
13. Does a nucleus have to be bombarded with fast or show neutrons in order for it to undergo fission ?
14. Why has it not been possible so far to control the fusion process and obtain usable energy from it ?
15. An atom has a continuous distribution of mass in..... (Thomson model, Rutherford model) but has a highly
non-uniform distribution in ..... (Thomson model, Rutherford model).
16. Which level of the doubly-ionized lithium (Li++)ion has the same energy as the ground state energy of the
hydrogen atom ?
17. If the a-decay of U238 is allowed from the point of view of energy (the decay products have a total mass less
than the mass of U238) what prevents U238 from decaying all at once ? Why is its half-life so large ?
18. Can a spectral line belong to both the Lyman and Balmer series ?
19. Although the Lyman series involves transitions to the ground level, and the Balmer series to the second orbit,
the latter was discovered earlier. Why ?
20. Bohrs principle of quantization of angular momentum is not a postulate but an essential condition. Explain
how.
21. Why are -particle tracks as observed in a cloud chamber much shorter than -particle tracks though they
emerge from a radioactive sample with almost the same speed ?
22. Cathode rays and -particles are streams of electrons. In what respect do they then differ from each other?
23. When a radioactive substance emits an -particle its position in the periodic table is lowered by two places.
Is this true or false ?
24. When Rutherford bombarded a thin foil of gold by -particles, he found that 1 in 2500 are deflected
through very large angles. What inference did he draw from this result ?
25. A uranium nucleus (atomic number 92, mass number 238) emits an -particle and the resultant nucleus
emits a -particle. What are the atomic and mass numbers of the final nucleus ?
26. Among -particles, -particles, protons and neutrons which have the greatest penetrating power through
matter and why ?
27. The isotope of hydrogen H13 (tritium) is radioactive. What would be its decay process and the product?
28. What is meant by the disposal of radioactive waste in a nuclear reactor ?
29. - and -particles suffer equal and opposite deflections in an external electric field. Is this true or false?
30. It requires infinite time for all the atoms in a radioactive sample to decay, whatever be the half-life of the
material. Is this true or false ?
31. What is mean by enrichment of uranium ?
32. Which yields greater energy per atom fission or fussion ? .... per unit mass ?
33. Explain the statement The moderator in a nuclear reactor thermalizes the neutrons.

www.physicsashok.in 124
MODERN PHYSICS
34. When is a chain reaction said to be critical ?
35. If a nucleus emits only a -ray photon, does its mass number change? Does its mass change ?
36. A classical atom based on ..... (Thomson model, Rutherford model) is doomed to collapse. Why ?
37. What is Bohrs correspondence principle ?
38. Bohrs quantization principle, i.e., angular momentum = nh/2 is a basic law in nature. Why do we never
speak of quantization of the angular momentum of a planet, around the sun ?
39. A photon is emitted by a dense star. Scientists say there is a change in the frequency of the photon as it
moves away from the star and call the difference in frequcney gravitational shift. Can you explain this?

SOLUTION OF THINKING PROBLEMS

X-rays
1. X-rays cannot be deflected by electric and magnetic fields. They are reflected, refracted, diffracted like
ordinary light waves. All these facts lead us to believe that X-rays are electromagnetic waves.
2. The quantum theory predicts a lower bound (hv = Ve) of wavelengths of X-rays produced in an X-rays
tube. This is found to be in good agreement with experimental observations. So this supports the quantum
theory of light.
3. No. In fact electrons have associated waves. At the accelerating voltage of the experiment the wave behaviour
of electrons becomes quite prominent and interference fringes are observed.
4. False, X-rays damage living cells and hence kill the bacteria present in water. Hence, the water actually
becomes safer for drinking.
5. The electron loses energy due to collisions with the gas atoms. It therefore describes a circle with decreasing
radius, i.e., it spirals inwards.
6. The phenomenon of fluorescence consists of absorption of higher energy photons and re-radiation of lower
energy visible light. This is possible with ultraviolet rays as these have greater photon energy than visible
light. Infrared rays have lower photon energy, so fluorescence cannot occur.
7. D, B. Alpha particles are heavy and positively charged, and so they are deflected the least to the left
according to Flemings left-hand rule. The electrons are deflected to the right.
8. Yes. The de Broglie wavelength does not depend on charge, only on momentum.
9. X-rays or -rays cause ionization by collision in the gas atoms. The free electron and ion pairs produced can
now move and conduct electricity. Thus, the gas becomes more conducting.
10. They have large atomic numbers and high melting points.
11. No. The electric field will not affect the initial velocity. It will only produce an additional accelerated motion
perpendicular to the initial velociyt. A 90 deflection requires that the initial velocity be reduced to zero.
12. (a) No work is done, since the displacement due to the motion is perpendicular to the field. (b) Work iis
done, as the displacement how has a component parallel to the force.
13. (a) The speed does not change since the force is perpendicular to the displacement at every point. The field
does no work on the particle. The energy and speed of the particle remain unchanged.
(b) When an electric field acts for some time, the particle will acquire a displacement parallel to the force.
Work is done by the field on the particle. The energy and speed of the particle will change.
14. Due to the magnetic field, the particle will move in a circle with the field as axis. Due to the electric field, a
force will act along this axis, producing an accelerated motion perpendicular to the plane of the circle. The
result will be a helical path of gradually increasing pitch.
15. Their negatively charged character is shown by the direction of deflection in an electric field. Their streaming
character is shown by deflection in a magnetic field because a magnetic field can produce deflection only
when charged particles are in motion.

www.physicsashok.in 125
MODERN PHYSICS
16. The charged particle will move along a helix winding around the lines of
induction. Let us resolve the velocity along the field and perpendicular to
it. The resolved part along the field vII is called orbital velocity. Since the
magnetic moment due to the orbital motion is opposite to the magnetic field
is tends to push the charged particle out of the field, i.e., the charged particle is strongly decelerated and so
its drift velocity decreases, becoming zero in the case of a sufficiently high field gradient. From this place it
begings to move in the opposite direction.
17. The charged particles follow a helical path of period T = 2m/Bq which is independent of the velocity of the
particles and pitch p = (2m/Bq) v cos = (2mv)/Bq when small. Thus after a path of length p all the
particles come down to the same point whatever be the angle of inclination of their initial motion with the
field.
18. It is absorbed by the target, which is also the anode of the X-ray tube. Subsequently, the electron returns to
the cathode via the external voltage circuit.
19. Less than 1% of the incident electron energy is actually converted to X-rays. The balance is lost in inelastic
collisions be between the electrons and the target atoms. This energy appears as heat in the target.
20. In a Coolide tube, the hardness and intensity of the X-rays can be controlled independently. The hardness
is controlled by the applied voltage. The intensity is controlled by the filament temperature, i.e., by the
filament current. Such independent control is not possible in the cathode ray tube.
21. The wavelength of a matter wave is given by = h/p. The momentum of ordinary material bodies at ordinary
speeds is very large and so the associated wavelength is extremely small because of the very small value of
h = 6.6 1034 Js. This is why the wave nature of matter is not apparent in our daily lives.
22. A single photon cannot be born because the principle of conservation of momentum would then be violated.
The meson is at rest and so if a single photon is created, that photon must also be at rest to conserve
momentum. But it is not possible for a photon to be at rest. This is why two photons are born which move
in opposite directions after creation, in conformity with the principle of conservation of momentum.
23. Because they are held together by a strong force, they are not exhibited separately.

Atomic Structure, Radioactivity, Nuclear Fission and Fusion


1. When a moving electron collides with an atom, one orbital electron in the atom absorbs part or all of the
kinetic energy of the incident electron. As a result, the orbital electron may move to an outer orbit, of higher
energy (excitation), or become completely free from the attractive field of the nucleus (ionization).
2. The gas atoms emitting light due to electron transitions are in motion. They behave like fixed frequency
sources in motion. Due to Doppler effect, the observed frequency (and hence wavelength) becomes different
from the emitted frequency. This difference depends on the velocity of the atom, causing the small spread of
the spectral line.
3. No. The -particle, although an electron, is actually created at the instant of -decay and ejected at once.
It cannot exist inside the nucleus as its de Broglie wavelength is much larger than the dimensions of the
nucleus.
4. The energy of a -ray photon is of the order of MeV. Energies of this magnitude occur in nuclear processes
but not in orbital electron transitions.
5. The universal principle of stability of a system, that is, a system lies in stable equilibrium when its energy is
at the lowest possibel value and Paulis exclusion principle, i.e., no two electrons can have all their quantum
numbers identical.
6. In no way. Chemical bonds involve only orbital electrons, whereas radioactivity is a nuclear process.
7. By passing them through transverse electric or magnetic fields.
8. No. In -decay, the atomic number decreases by 2, hence the atom is left with two extra orbital electrons.
It therefore has a double negative charge. In -decay, the atom is left with a net single positive charge.

www.physicsashok.in 126
MODERN PHYSICS
9. Yes. In -decay, the element moves back two places on the periodic table. In -decay, the element moves
forward one place on the periodic table.
10. The law of radioactive decay is statistical in nature. Hence, individial experimental results will show slight
variations. The averages over a large number of experimental results conform exactly with the theory.
11. No. Here c2 appears only as a constant and does not suggest motion.
12. This is a neutron with energy of the order of (3/2) kT, where T is the absolute temperature of the surroundings
and k is Boltzmanns constant. This follows from comparison with the law of equipartition of energy as
applied to gas molecules. Thermalisation of a neutron brings down its energy from a high value of about
(3/2) kT.
13. For fission, the neutron must be absorbed by the fissionable nucleus. This is possible only with slow neutrons.
14. Fusion occurs only at temperatures of the order of 106 K. This makes it extremely difficult to control fusion
processes.
15. Thomson model, Rutherford model.
16. E Z 2 / n 2 . When E is constant, Z2 n 2 Z n
3/1 = n/1 n = 3
17. The emission of a-particles is caused by quantum mechanical tunnelling through the repulsive Coulomb
barrier. They bounce to and fro in the potential well bounded by the barrier before tunnelling through it.
Hence the probability of escape is not the same for all the a-particles because all are not born inside the
nucleus at the same time.
18. Spectral lines from hydrogen arise from the relation hv = E0(1/n2 1/m2) with m > n. For the Lyman series
n = 1, m = 2, ...., .
(hv)max = E0 and (hv)min = (3/4) E0
3
or E /h < v < E0/h
4 0
For the Balmer series, n = 2, m = 3, ....., .
(hv)max = E0/4 and (hv)min = 5E0 /36
or 5E0/36h < v < E0/4h. Clearly, the same value of v cannot satisfy both the series. Hence, a spectral line
cannot belong to both series.
19. Because the Balmer series lies in the visible region and the Lyman series in the ultraviolet region.
20. Bohrs principle of quantization of angular momentum is seen to be an essential condition while considering
de Broglies matterwave principle, that is, in nature all moving bodies have an associated wave. We have p
= h/. In the stationary orbits the waves associated with the particle must form a stationary wave. If r is the
radius of the stationary orbit, its circumference (equal to 2r) must be in multiples of wavelength . Therefore,
2r = n or 2r = n . h/p or pr = nh/2. Moment of momentum is angular momentum. Hence pr is the
angular momentum (L) of the electron. Thus finally we find that I = nh/2.
21. -particles have greater ionizing power than -particles and so they lose their energy much earlier than
-particles because of collisions. This is why their track lengths are shorter than those of -particles.
22. They differ in respect of their origin. -particles originate in the nucleus when a neutron is converted into a
proton, whereas electrons in cathode rays are orbital electrons.
23. Yes. Since -particles are helium atoms, their emission lowers the atomic number by 2 and mass number by
4. Elements are arranged in the periodic table according to their atomic number. So emittion of -particles
lowers the position of the element by two places in the periodic table.
24. An atom consists of a small centralized mass containing positively charged particles. Otherwise the atom as
a whole in empty.
25. 92 and 234.
92
U238 2He4 2 1e0 92U234

www.physicsashok.in 127
MODERN PHYSICS
26. Neutrons, because they are electrically neutral and so they do not interact with matter electrically.
27. The only possible decay process is -decay. The decay product would be 2He3.
28. In nuclear fission, two nuclides with Z of the order of 40 to 50 are created. These are highly radioactive,
with half-lives or thousands of years. These are called radioactive waste. They have to be disposed of in
sealed containers which can contain their radioactive emissions.
29. False. -particles are deflected more due to their larger specific charge.
30. True. This follows from the exponential nature of the decay. In
N = N0et, for N = 0, t =
31. Natural uranium contains less than 1% of U235 mixed with U238. The latter is not fissionable, while the former
is fissionable. The proportion of U235 must be increased artificially for the uranium to be used in fission. This
is called enrichment of uranium.
32. Fission yields greater energy per atom. Fusion yields greater energy per unit mass.
33. The neutrons emitted in fission must be slowed down in order to cause further fission in other nuclei. This is
called thermalization, and is performed by the moderator.
34. When exactly one neutron, of the several produced by the fission of one nucleus, is permitted to cause
further fission. This happens in a controlled chain reaction, e.g., in a nuclear reactor.
35. The mass number does not change. The mass is reduced.
36. Thomsons model.
37. For large quantum numbers, quantum mechanical results reduce to classical results.
38. Because large n corresponds to a very large value at which classical and quantum results are identical by
Bohrs correspondence principle.
39. As the photon moves out against strong gravitational attraction its energy decreases and so its frequency is
expected to decrease. This is gravitational shift.

www.physicsashok.in 128
MODERN PHYSICS

ASSERTION-REASON
(A) Statement-1 is True, Statement-2 is True; Statement-2 is a correct explanation for Statement-1
(B) Statement-1 is True, Statement-2 is True; Statement-2 is NOT a correct explanation for Statement-1
(C) Statment-1 is True, Statement-2 is False
(D) Statement-1 is False, Statement-2 is True

1. Statement-1 : Electron capture occurs more often than positron emission in heavy elements.
Statement-2 : Heavy elements exhibit radioactivity.

2. Statement-1 : In a hydrogen atom energy of emitted photon corresponding to transition from n = 2 to


n = 1 is much greater as compared to transition from n = to n = 2.
Statement-2 : Wavelength of photon is directly proportional to the energy of emitted photon

3. Statement-1 : Ionisation energy of atomic hydrogen is greater than atomic deuterium.


Statement-2 : Ionisation energy is directly proportional to reduced mass

4. Statement-1 : For pair production, energy of ray is greater than 1.02 MeV.
Statement-2 : In pair production, energy is converted into mass.

5. Statement-1 : The ratio of rate of production (R) of neutrons to the rate of leakage of neutron from a
spherical body of 92U235 is directly proportional to radius (r)
Statement-2 : Rate of production of neutron is directly proportional to volume but rate of leakage of
neutrons is directly proportional to area.

6. Statement-1 : The nuclear energy can be obtained by the nuclear fission of heavier nuclei as well as nuclear
fusion of lighter nuclei.
Statement-2 : The binding energy per nucleon with increase in atomic number first increases and then
decreases.

7. Statement-1 : A small metal ball is suspended in a uniform electric field with an insulated thread. If high
energy X-ray beam falls on the ball, the ball will be deflected in the direction of electric field.
Statement-2 : Wavelength of L X-ray must be greater then the wavelength of K X-ray for the same
material.

8. Statement-1 : The difference in the frequencies of series limit of Lyman series and Balmer series is equal to
the frequency of the first line of the Lyman series.
Statement-2 : Difference in energy of two atomic levels is proportional to the energy of emitted or absorbed
photon.

9. Statement-1 : Work function of aluminium is 4.2 eV. If two photons of each of energy 2.5 eV strike on an
electron of aluminium, the electron is not emitted.
Statement-2 : In photoelectric effect, electron is emitted only if energy of each of incident photon is greater
than the work function.

10. Statement-1 : If the accelerating potential in an X-ray tube is increased, the wavelengths of the characteristic
X-rays do not change.
Statement-2 : When an electron beam strikes the target in an X-ray tube, part of the kinetic energy is
converted into X-ray energy. [JEE, 07]
www.physicsashok.in 129
MODERN PHYSICS

Match the Column


1. Column I Column II
(A) Work function of copper is 4 eV. If two (P) 13.6 Z2/n2eV
photons each of energy 2.5 eV strike an
electron of copper emission of electrons
1 1
(B) Cathode rays get deflected by (Q) 13.6 Z 2 2
2 eV
n1 n 2
(C) Ionisation energy of H like atom is (R) both electric and magnetic field
(S) 1H1
(D) Greater wavelength in transition from n = 2 (T) 1H3
to n = 1 is for (U) Not possible
(V) Possible
2. Column I Column II
(A) Radius of orbit depend on principal quantum (P) Increase
number as
(B) Due to orbital motion of electron, (Q) decrease
Magnetic field arises at the centre of Nucleus
is proportional to principal quantum no. as (R) is proportional to 1/n2
(C) If electron is going from lower energy level to
higher energy level then velocity of electron will (S) is proportional to n2
(D) If electron is going from lower energy level to
higher energy level, then total energy of (T) is proportional to 1/n5
electron will
3. Column I Column II
(A) Rate of disintegration, i.e. dN/dt is (P) Greater than Half life
proportional to
(B) Mean life of radioactive substance is (Q) Less than Half life
(C) Intensity I of ray of initial intensity I 0 after (R) Number of atoms of parent radioactive
transversing the thickness of x of the absorber is substance still undecayed at time t
(related as)
(D) The radioactive decay rate is not affected by (S) I = I 0/x
(T) I = I 0ex
(U) Temperature, pressure, volume
4. Column I Column II
(A) Binding energy per nucleon for middle order (P) Optical Model
or elements is
(B) Nuclear force depends on (Q) Shell model
2
Z
(C) For nuclear fission is (R) 8.8 MeV
A
(D) Magic numbers are 2, 8, 20, 28, 50, 82, 126 (S) 2.5 eV
explained by
(T) Charges of Nucleons
(U) Spin of Nucleons
(V) Greater than 15
(W) Less than 15
5. Column I Column II
(A) Radius of orbit is related with atomic number (Z) (P) is proportional to Z
(B) Current associated due to orbital motion (Q) is inversely proportion to Z
electron with atomic number (Z)
(C) Magnetic field at the centre due to orbital (R) is proportional to Z 2
motion of electron related with Z.
(D) Velocity of an electron related with atomic (S) is proportional to Z 3
number (Z)

www.physicsashok.in 130
MODERN PHYSICS
6. Match the following Columns [JEE, 06]
Column I Column II
(A) Nuclear fusion (P) Converts some matter into energy
(B) Nuclear fission (Q) Generally occurs for nuclei with low atomic number
(C) -decay (R) Generally occurs for nuclei with higher atomic number
(D) Exothermic nuclear reaction (S) Essentially proceeds by week nuclear forces
7. In the following, column I lists some physical quantities & the column II gives approx, energy values
associated with some of them. Choose the appropriate value of energy from column II for each of the
physical quantities in column I and write the corresponding letter A, B, C etc. against the number (i), (ii),
(iii), etc. of the physical quantity in the answer book. In your answer, the sequence of column I should be
maintained.
Column I Column II [JEE, 97]
(A) Energy of thermal neutrons (P) 0.025 eV
(B) Energy of X-rays (Q) 0.5 eV
(C) Binding energy per nucleon (R) 3 eV
(D) Photoelectric threshold of metal (S) 20 eV
(T) 10 keV
(U) 8 MeV
8. Some laws/processes are given in Column I. Match these with the physical phenomena given in Column II.
Column I Column II
(A) Transition between two atomic energy levels (P) Characteristic X-rays [JEE, 07]
(B) electron emission from a material (Q) Photoelectric effect
(C) Mosleys law (R) Hydrogen spectrum
(D) Change of photon energy into kinetic (S) -decay
energy of electrons.
9. Column - II gives certain systems undergoing a process. Column-I suggests changes in some of the parameters
related to the system. Match the statements in Column I to the appropriate process(es) from Column-II.
Column I Column II [JEE, 09]
(A) The energy of the system is increased (P) System : A capacitor, initially uncharged
(B) Mechanical energy is provided to the system, Process : It is connected to a battery
which is converted into energy of random (Q) System : A gas in an adiabatic container fitted
motion of its parts with an adiabatic piston
(C) Internal energy of the system is converted into Process : The gas is compressed by pushing
its mechanical energy. the piston
(D) Mass of the system is decreased (R) System : A gas in a girid container
Process : The gas gets cooled due to colder
atmosphere surrounding it
(S) System : A heavy nucleus, initially at rest
Process : The nucleus fissions into two fragments
of nearly equal masses and some
neutrons are emitted.
(T) System : A resistive wire loop
Process : The loop is placed in a time varrying
magnetic field perpendicular to its plane

www.physicsashok.in 131
MODERN PHYSICS

Level 1
1. An energy of 24.6 eV is required to remove one of the electrons from the neutral helium atom. The energy
(in eV) required to remove both the electron from a neutral helium atom is
(a) 38.2 (b) 49.2 (c) 51.8 (d) 79.0

2. The K X ray emission line of tungsten occurs at = 0.021 nm. The energy difference between K and
L levels in this atom is about
(a) 0.51 MeV (b) 1.2 MeV (c) 59 keV (c) 136 eV

3. The electron in a hydrogen atom makes a transition n1 n 2 where n1 and n2 are the principal quantum
numbers of the two states. Assume the Bohr model to be valid. The time period of the electron in the
initial state is 8 times that in the final state. The possible values of n1 and n2 are
(a) n1 = 4, n2 = 1 (b) n1 = 8, n2 = 2 (c) n1 = 8, n2 = 1 (d) n1 = 6, n2 = 3

4. X-rays are produced in an X-ray tube operating at a given accelerating voltage. The wavelength of the
continuous X-rays has values from
(a) 0 to (b) min to where min > 0
(c) 0 to max where max < (d) min to max where 0 < min < max <

5. A particle of mass M at rest decays into two particles of masses m 1 and m 2, having non zero velocities.
The ratio of the de Broglie wavelengths of the particles, 1 2 , is
m1 m2 m2
(a) m (b) m (c) 1.0 (d) m1
2 1

6. The electron in a hydrogen atom makes a transition from an excited state to the ground state. Which of
the following statements is true ?
(a) Its kinetic energy increases and its potential and total energies decrease.
(b) Its kinetic energy decreases, potential energy increases and its total energy remains the same.
(c) Its kinetic and total energies decrease and its potential energy increases.
(d) Its kinetic, potential and total energies decrease.
7. Imagine an atom made up of a proton and a hypothetical particle of double the mass of the electron but
having the same charge as the electron. Apply the Bohr atom model and consider all possible transitions
of this hypothetical particle to the first excited level. The longest wavelength photon that will be emitted
has wavelength (given in terms of the Rydberg constant R for the hydrogen atom) equal to
(a) 9/5R (b) 36/5R (c) 18/5R (d) 4/R
8. Electrons with energy 80 keV are incident on the tungsten target of an X-ray tube. K shell electrons of
tungsten have 72.5 keV energy. X-rays emitted by the tube contain
(a) a continuous X-ray spectrum (Bresmsstrahlung) with a minimum wavelength of about 0.155 A
(b) a continuous X-ray spectrum (Bremsstrahlung) with all wave-lengths
(c) the characteristic X-ray spectrum of tungsten
(d) a continuous X-ray spectrum (Bremsstrahlung) with a minimum wavelength of about 0.155 A and the
characteristics X-rays spectrum of tungsten.

9. For a photoelectric cell, the graph in Figure. showing the variation of the cut-off voltage V0 with frequency
(v) of incident light is

V0 V0 V0 V0

O v O v O v O v
(a) (b) (c) (d)

www.physicsashok.in 132
MODERN PHYSICS
10. Monochromatic light of frequency v 1 irradiates a photocell and the stopping potential is found to be V1.
What is the new stopping potential of the cell if it is irradiated by monochromatic light of frequency v 2?
h h h h
(a) V1 v 2 v 1 (b) V1 v 2 v 1 (c) V1 v 1 v 2 (d) V1 v 1 v 2
e e e e

11. When a certain photosensitive surface is illuminated with monochromatic light of frequency v, the stop-
ping potential for photoelectric current is V0/2. When the same surface is illuminated by monochromatic
high of frequency v/2, the stopping potential is V0. The threshold frequency for photoelectric emission is
2v 3v 3v 5v
(a) (b) (c) (d)
3 2 5 3

12. The energy of a photon of frequency v is E = hv and the momentum of a photon of wavelength is p = h
/ . From this statement one may conclude that the wave velocity of light is equal to
2

(a) 3 x 10 ms8 1
(b) (c) p (d)
p

13. When a centimeter thick surface is illuminated with light of wavelength , the stopping potential is V..
When the same surface is illuminated by light of wavelength 2 , the stopping potential is V/3. The
threshold wavelength for the surface is
4 8
(a) (b) 4 (c) 6 (d)
3 3

14. A star of mass M0, radius R0 contracts to radius R. Energy radiated by the star assuming uniform density
in each case while temperature remains unchanged is
R 2 R 3

R
1
(a) 0 c
(b) 0 c 1
(c) 0 c 1
(d) 0 c
R 0 R
R 0
0

15. A sensor is exposed for time t to a lamp of power P placed at a distance . The sensor has an opening
that is 4d in diameter. Assuming all energy of the lamp is given off as light, the number of photons
entering the sensor if the wavelength of light is is (l >> d)
P d2 t 4 P d2 t P d2 t P d2 t
(a) N (b) N (c) N (d) N
h c 2 h c 2 4 hc 2 16 h c 2

16. If elements of quantum number greater than n were not allowed, the number of possible elements in
nature would have been
2
1 n n 1 1 1
(a) n n 1 (b) (c) n n 1 2n 1 (d) n n 1 2n 1
2 2 6 3

17. An electron is lying initially in the n = 4 excited state. the electron de-excites itself to go to n = 1 state
directly emitting a photon of frequency v 41 . If the same electron first de-excites to n = 3 state by emitting
a photon of frequency v 43 and then goes from n = 3 to n = 1 state by emitting a photon of frequency v 31 ,
then
(a) 41 43 31 (b) 41 43 31 (c) 41 43 2 31 (d) Data Insufficient

18. A photon of energy 10.2 eV corresponds to light of wavelength 0 . Due to an electron transition from n =
2 to n = 1 in a hydrogen atom, light of wavelength is emitted. If we take into account the recoil of the
atom when the photon is emitted,
(a) = 0 (b) 0 (c) 0 (d) the data is not sufficient to reach a conclusion.

www.physicsashok.in 133
MODERN PHYSICS
19. When an electron moving at a high speed strikes a metal surface, which of the following are possible?
(a) The entire energy of the electron may be converted into an X-ray photon.
(b) Any fraction of the energy of the electron may be converted into an X-ray photon.
(c) The entire energy of the electron may get converted to heat.
(d) The electron may undergo elastic collision with the metal surface.

20. A star converts all of its 2He4 nuclei completely into oxygen nuclei. The energy released per oxygen
nuclei is (given mass of nucleus = 4.0026 amu, mass of oxygen nucleus = 15.994 amu)
(a) 7.26 MeV (b) 7 MeV (c) 15.252 MeV (d) 23.9 MeV

21. The graph showing the energy spectrum of particles is :


Y Y Y

(a) n(E)
(b) n(E)
(c) n(E)
(d) n(E)

E
X E
X E
X E

22. Binding energy per nucleon of 1H2 and 2He4 are 1.1 eV and 7.0 MeV respectively. Energy released in
the process 1H2 + 1H2 = 2He4 is :
(a) 20.8 MeV (b) 16.6 MeV (c) 25.2 MeV (d) 23.6 MeV

23. Two electrons are moving with the same speed V. One electron enters a region of uniform electric field
while the other enters a region of uniform magnetic field . After sometime if the de Broglie wavelength of
the two are 1 and 2 then :
(a) 1 = 2 (b) 1 > 2 (c) 1 > 2 (d) 1 > 2 or 1 < 2

24. In a characteristic X-ray spectra of some atom superimposed on a continuous X-ray spectra :
P Q
Relation
intensity

min
(a) P represents K line (b) Q represents K line
(c) Q and P represents K and K lines respectively
(d) Position of K and K depend on the particular atom

25. Difference between nth and (n + 1)th Bohrs radius of H atom is equal to its (n 1)th Bohrs radius. The
value of n is
(a) 1 (b) 2 (c) 3 (d) 4

26. A hydrogen atom is in an excited state of principal quantum number n. It emits a photon of wavelength
while returning to the ground state. The value of n is :
(R 1) R
(a) R(R 1) (b) (c) (d) (R 1)
R R 1

27. The binding energies of nuclei X and Y are E1 and E2 respectively. Two atoms of X fuse to give one atom
of Y and an energy Q is released. Then :
(a) Q = 2E1 E2 (b) Q = E2 E1 (c) Q < 2E1 E2 (d) Q > E2 2E1

28. Two radioactive materials x1 and x2 have decay constant 10 and respectively. Initially they have the
same number of nuclei. The ratio of the number of nuclei x 1 to that of x 2 will be 1/e after time :
(a) 1/10 (b) 1/11 (c) 11/10 (d) 1/9

www.physicsashok.in 134
MODERN PHYSICS
29. At t = 0 activity of radioactive substance is 1600 Bq and t = 8 sec activity remains 100 Bq. The activity
at t = 2 sec is :
(a) 200 Bq (b) 400 Bq (c) 600 Bq (d) 800 Bq

30. There is a stream of neutrons with a kinetic energy of 0.0327 eV. If the half life of neutrons is 700 s. The
fraction of neutrons will decay before they travel a distance 10 m. m n = 1.675 1027 kg :
(a) 3.96 105 (b) 3.96 106 (c) 2.96 104 (d) None

31. The count rate for 10 gram radioactive material was measured
time (hrs) at different times and this had been shown in figure 100
scale given. The half life of material and the total count in the
first half value period respectively are : 75
(a) 4 hrs. and 9000 approximately
(b) 3 hrs and 14100 approximately 50
(c) 3 hrs and 235 approximately 25
12.5
(d) 10 hrs and 157 approximately 3 6 9 time in hr.

32. Assuming that about 200 MeV energy is released per fission of 92U235 nuclei. What would be mass of
92
U235 consumed per day in the fission of reactor of power 1 MW approximately ?
(a) 10 kg (b) 100 kg (c) 1 gram (d) 102 gm

33. The energy, the magnitude of linear momentum and orbital radius of an electron in a hydrogen atom
corresponding to the quantum number n are E, P and r, according to the Bohrs theory of hydrogen
atom:
(a) EPr is proportional to 1/n (b) P/E is proportional to n0
(c) Er is not constant for all orbits (d) Pr is proportional to n.

34. An electron is excited from a lower energy state to a higher energy state in a hydrogen atom. Which of
the following quantities decrease in the excitation.
(a) Potential enrgy (b) Angular speed (c) Kinetic energy (d) Angular momentum

35. The correct statement is l are :


(a) density of nucleus is independent of mass number (A)
(b) Radius of nucleus increases with mass number (A)
(c) Mass of nucleus is directly proportional to mass number (A).
(d) Density of nucleus is directly proportional to mass number.

36. A hydrogen like atom of atomic number Z is an excited state of quantum number 2n. It can emit a
maximum energy photon of 204. eV. It makes a transition to quantum state n, a photon of energy 40.8
eV is emitted, then
(a) Z = 2 (b) Z = 4 (c) n = 1 (d) n = 2

37. An electron with kinetic energy varying from 5 eV to 50 eV is incident on a hydrogen atom in its ground
state. The collision :
(a) may be elastic (b) may be partially elastic
(c) must be completely inelastic (d) from zero to 13.6 eV be elastic and more than 27.2 eV be
inelastic.

38. The wavelength of first Balmer line for 1H1 , 1H2 and 2He4 and 1, 2 and 3 respectively. The correct
statements are
(a) 2 1 (b) 3 2 (c) 1 2 (d) 1 2 > 3

39. Regarding X ray spectrum which of the following statement


(a) The characteristic X ray spectrum is emitted due to excitation of inner electrons of atom
(b) Wavelength of characteristic spectrum depend on the potential difference across the tube.
(c) Wavelength of continuous spectrum is dependent on the potential difference across tube
(d) None of these

www.physicsashok.in 135
MODERN PHYSICS
40. If the wavelength of light in an experiment on photo electric effect is doubled :
(a) The photoelectric emission will not take place.
(b) The photoemission may or may not take place. (c) The stopping potential will increase
(d) The stopping potential will decrease under the condition that energy of photon of doubled. Wavelength
is more than work function of metal.

41. the binding energy per nucleon is :


(a) Maximum for middle order element (b) Minimum for lighter elements
(c) Binding energy per nucleon suddenly increases for some mass number called magic numbers.
(d) Binding energy per nucleon is minimum for middle order elements

42. When Z is doubled in an atom, which of the following statement are consistent with Bohrs theory :
(a) Energy of a state is double. (b) Radius of an orbit is double.
(c) Velocity of electrons in an orbit is doubled (d) Radius of orbit is halved.

43. Photons of wavelength 6620 are incident normally on a perfectly reflecting screen. Calculate the
number of photons per second falling on the screen as total power of photons such that the exerted
force is 1N :
(a) 5 1026 (b) 5 1025 (c) 1.5 108 (d) None of these

44. The energy of particles emitted by 210Po is 5.3 MeV. What mass 210Po is needed to power a
thermoelectric cell of 13 watt output, What would be power output after 1 year : (The half life of 210Po is
138 days)
(a) 8.85 102 gram (b) 0.159 watt (c) 0.179 watt (d) 8.85 104 gram

45. The atomic masses of 7N15, 8O15 and 8O16 are respectively 15.0001 a.m.u., 15.0030 a.m.u. and 15.9949
a.m.u. Then :
(a) Binding energy per nucleon in 8O 16 is 7.97 MeV
(b) Energy is needed to remove one proton 8O 16 is 12.13 MeV
(c) Energy needed to remove one proton from 8O 16 is 15.61 MeV.
(d) All the above

46. A sample contains 102 kg each of two substances A and B with half lives 4 sec and 8 sec respectively.
Their weights are in the ratio of 1 : 2. The amounts of a and B after an interval of 16 sec.
(a) 6.25 104 kg (b) 12.5 104 kg (c) 2.5 103 kg (d) 1.25 105 kg

47. The wavelength and frequency of photons in transition 1,2 and 3 for H like atom are 1, 2, 3 and 1, 2,
3. then :
1 C
1

2 B
2 3
3 A

1 2 1 2
(a) 3 = 1 + 2 (b) 3 = 1 + 2 (c) 3 (d) 3
1 2 1 2

48. Which of the following pair constitute very similar radiations ?


(a) Hard U.V. ray and soft X ray. (b) Soft U.V. ray and hard X ray
(c) Very hard X ray and low frequency Y ray (d) Soft X ray and Y ray

49. The correct option are :


(a) In uranium ore, the ratio of U235 to U238 is 1 : 40 (b) Critical mass of uranium is 10 kg
(c) 92U235 : 92U238 = 1 : 4 (d) All the above

www.physicsashok.in 136
MODERN PHYSICS

50. A star initially has 1040 deutrons. It produces energy via, the processes 1H2 + 1H2 3
1H + p &
1
H2 + 1H3 4 16
2He + n. If the average power radiated by the star is 10 W, the deuteron supply of the
star is exhausted in a time of the order of : [JEE, 93]
6 8 12 16
(A) 10 sec (B) 10 sec (C) 10 sec (D) 10 sec

51(i). Fast neutrons can easily be slowed down by :


(A) the use of lead shielding (B) passing them through water
(C) elastic collisions with heavy nuclei (D) applying a strong electric field

(ii). Consider particles, particles & -rays, each having an energy of 0.5 MeV. Increasing order of
penetrating powers, the radiations are : [JEE, 94]
(A) , , (B) , , (C) , , (D) , ,

52. Which of the following statement(s) is (are) correct ? [JEE, 94]


(A) The rest mass of a stable nucleus is less than the sum of the rest masses of its separated nucleons.
(B) The rest mass of a stable nucleus is greater than the sum of the rest masses of its separated nucleons.
(C) In nuclear fusion, energy is released by fusion two nuclei of medium mass (approximately 100 amu).
(D) In nuclear fission, energy is released by fragmentation of a very heavy nucleus.

53. The binding energy per nucleon of 16O is 7.97 MeV & that of 17O is 7.75 MeV. The energy in MeV
required to remove a neutron from 17O is : [JEE, 95]
(A) 3.52 (B) 3.64 (C) 4.23 (D) 7.86

54. The maximum kinetic energy of photoelectrons emitted from a surface when photons of energy 6 eV fall on
it is 4 eV. the stopping potential in Volts is : [JEE, 97]
(A) 2 (B) 4 (C) 6 (D) 10

55. Select the correct alternative(s). [JEE, 98]


20
(i) Let mp be the mass of a proton, mn the mass of a neutron, M1 the mass of a 10 Ne nucleus & M2 the mass
40
of a 20 Ca nucleus. Then :
(A) M2 = 2M1 (B) M2 > 2M1 (C) M2 < 2M1 (D) M1 < 10(mn + mp)

(ii) The half-life of 131I is 8 days. Given a sample of 131I at time t = 0, we can assert that :
(A) no nucleus will decay before t = 4 days (B) no nucleus will decay before t = 8 days
(C) all nuclei will decay before t = 16 days (D) a given nucleus may decay at any time after t = 0
Binding Energy/nucleon

Y
8.5 X
56(a). Binding energy per nucleon vs. mass number curve for 8.0
in MeV

nuclei is shown in the figure. W, X, Y and Z are four nuclei 7.5 W

indicated on the curve. The process that would release


5.0 Z
energy is [JEE, 99]
(A) Y 2Z (B) W X + Z
(C) W 2Y (D) X Y + Z 30 60 90 120
Mass Number of Nuclei

www.physicsashok.in 137
MODERN PHYSICS

(b) Order of magnitude of density of Uranium nucleus is, [mp = 1.67 1027 kg]
(A) 1020 kg/m3 (B) 1017 kg/m3 (C) 1014 kg/m3 (D) 1011 kg/m3

22
(c) Ne nucleus, after absorbing energy, decays into two -particles and an unknown nucleus. The unknown
nucleus is
(A) nitrogen (B) carbon (C) boron (D) oxygen

(d) Which of the following is a correct statement ?


(A) Beta rays are same as cathode rays (B) Gamma rays are high energy neutrons.
(C) Alpha particles are singly ionized helium atoms
(D) Protons and neutrons have exactly the same mass (E) None

(e) The half-life period of a radioactive element X is same as the mean-life time of another radioactive element
Y. Initially both of them have the same number of atoms. Then
(A) X & Y have the same decay rate initially (B) X & Y decay at the same rate always.
(C) Y will decay at a faster rate than X (D) X will decay at a faster rate than Y

57. A particle of mass M at rest decays into two particles of masses m1 and m2, having non-zero velocities. The
radio of the de-Broglie wavelengths of the particles, 1/2, is [JEE, 99]
(A) m1/m2 (B) m2/m1 (C) 1.0 (D) m 2 / m1

58.(a) Imagine an atom made up of a proton and a hypotherical particle of double the mass of the electron but
having the same charge as the electron. Apply the Bohr atom model and consider all possible transitions of
this hypothetical particle to the first excited level. The longest wavelength photon that will be emitted has
wavelength (given in terms of the Rydberg constant R for the hydrogen atom) equal to [JEE, 2000 Scr.]
(A) 9/(5R) (B) 36/(5R) (C) 18/(5R) (D) 4/R

(b). The electron in a hydrogen atom makes a transition from an excited state to the ground state. Which of the
following statements is true ? [JEE, 2000 Scr.]
(A) Its kinetic energy increases and its potential and total energies decrease.
(B) Its kinetic energy decreases, potential energy increases and its total energy remains the same.
(C) Its kinetic and total energies decrease and its potential energy increases.
(D) Its kinetic, potential and total energies decrease.

59. The potential difference applied to an X-ray tube is 5 kV and the current through it is 3.2 mA. Then the
number of electrons striking the target per second is [JEE, 02 Scr.]
16 16 17
(A) 2 10 (B) 5 10 (C) 1 10 (D) 4 1015

60. A Hydrogen atom and Li++ ion are both in the second excited state. If lH and lLi are their respective
electronic angular momenta, and EH and ELi their respective energies, then [JEE, 02 Scr.]
(A) lH > lLi and |EH| > |ELi| (B) lH = lLi and |EH| < |ELi|
(C) lH = lLi and |EH| > |ELi| (D) lH < lLi and |EH| < |ELi|

61. Given a sample of Radium-226 having half-life of 4 days. Find the probability, a nucleus disintegrates within
2 half lives. [JEE, 06]
(A) 1 (B) 1/2 (C) 3/4 (D) 1/4
www.physicsashok.in 138
MODERN PHYSICS
62. The graph between 1/ and stopping potential (V) of three metals
having work functions 1, 2 and 3 is an experiment of photoelectric V metal 1 metal 2 metal 3
effect is plotted as shown in the figure. Which of the following
statement(s) is/are correct? [Here is the wavelength of the incident
ray].
(A) Ratio of work functions 1 : 2 : 3 = 1 : 2 : 4
0.001 0.002 0.004 1/ nm1
(B) Ratio of work functions 1 : 2 : 3 = 4 : 2 : 1
(C) tan is directly proportional to hc/e, where h is Plancks constant and c is the speed of light
(D) The violet colour light can eject photoelectrons from metals 2 and 3. [JEE, 06]

63. In the options given below, let E denote the rest mass energy of a nucleus and n a neutron. The correct
option is : [JEE, 07]
(A) E 236
92 I E Y 2E(n)
U E 137
53
97
39 (B) E 236
92 I E Y 2E(n)
U E 137
53
97
39

(C) E U E Ba E Kr 2E(n) (D) E U E Ba E Kr 2E(n)


236 140 94 236 140 94
92 56 36 92 56 36

64. The largest wavelength in the ultraviolet region of the hydrogen spectrum is 122 nm. The smallest wavelength
in the infrared region of the hydrogen spectrum (to the nearest integer) is [JEE, 07]
(A) 802 nm (B) 823 nm (C) 1882 nm (D) 1648 nm

65. Electrons with de-Broglie wavelength fall on the target in an X-ray tube. The cut-off wavelength of the
emitted X-rays is [JEE, 07]
2mc 2 2h 2m 2 c 2 2
(A) 0 (B) 0 (C) 0 (D) 0 =
h mc h2

66. Which one of the following statements is WRONG in the context of X-rays generated from a X-ray tube?
(A) Wavelength of characteristic X-rays decreases when the atomic number of the target increases
(B) Cut-off wavelength of the continuous X-rays depends on the atomic number of the target. [JEE, 08]
(C) Intensity of the characteristic X-rays depends on the electrical power given to the X-rays tube.
(D) Cut-off wavelength of the continuous X-rays depends on the energy of the electrons in the X-rays tube.

67. Assume that the nuclear binding energy per nucleon (B/A) versus mass B/A
number (A) is as shown in the figure. Use this plot to choose the correct 8

choice(s) given below : Figure 6

(A) Fusion of two nuclei with mass numbers lying in the range of 4

1 < A < 50 will release energy. 2


(B) Fusion of two nuclei with mass numbers lying in the range of 0
A
51 < A < 100 will release energy. 100 200
(C) Fission of a nucleus lying in the mass range of 100 < A < 200 will release energy when broken into two
equal fragments.
(D) Fission of a nucleus lying in the mass range of 200 < A < 260 will release energy when broken into two
equal fragments. [JEE, 08]

68. The quantum number n of the state finally populated in He+ ions is [JEE, 08]
(A) 2 (B) 3 (C) 4 (D) 5

www.physicsashok.in 139
MODERN PHYSICS

69. The wavelength of light emitted in the visible region by He+ ions after collisions with H atoms is [JEE, 08]
(A) 6.5 107 m (B) 5.6 107 m (C) 4.8 107 m (D) 4.0 107 m

70. The ratio of the kinetic energy of the n = 2 electron for the H atom to that of He+ ion is [JEE, 08]
1 1
(A) (B) (C) 1 (D) 2
4 2

71. A radioactive sampel S1 having an activity of 5Ci has twice the number of nuclei as another sample S2
which has an activity of 10 Ci. The half lives of S1 and S2 can be : [JEE, 08]
(A) 20 years and 5 years, respectively (B) 20 years and 10 years, respectively
(C) 10 years each (D) 5 years each

72. Photoelectric effect experiments are performed using three different metal plates p, q and r having work
functions p = 2.0 eV, q = 2.5 eV and r = 3.0 eV, respectively. A light beam containing wavelengths of
550 nm, 450 nm and 350 nm with equal intensities illuminates each of the plates. The correct IV graph for
the experiment is [JEE, 09]
I I I I
p r
q q
(A) r (B) p q (C) p (D) r q
r p
V V V V

Passage
PASSAGE : 1

Figure 1 below depicts three hypothetical atoms. Energy levels are represented as horizontal segments. The
distance between the segments is representative of the energy difference between the various levels . All
possible transitions between energy levels are indicated by arrows

Atom #1 Atom # 2
Atom # 3

Scientists can observe the spectral lines of atoms that are dominant in far-away galaxies. Due to the speed at
which these galaxies are travelling, these lines are shifted, but their pattern remains the same. This allows
researchers to use the spectral pattern to determine which atoms they are seeing. Table 1 below shows
spectroscopic measurements made by researchers trying to determine the atomic makeup of a particular far-
away galaxy. Light energy is not measured directly, but rather is determined from measuring the frequency of
light. Which is proportional to the energy.

www.physicsashok.in 140
MODERN PHYSICS
Table 1
Frequencies Measured
868440
880570
879910
856390

1. For each of three hypothetical atoms (Atom 1, Atom 2 and Atom 3), Figure 1 depicts the
(A) number of electrons and the amount of energy the atom contains
(B) distance an electron travels from one part of the atom to another
(C) energy released by the atom as an electron as it moves from one energy state to another
(D) frequency with which the atoms electrons move from one energy state to another
2. In which of the three hypothetical atoms depicted in Figure 1 does the energy of the light released by
the atom vary the least
(A) Atom 1 (B) Atom 2 (C) Atom 3 (D) It is impossible to
tell
3. Scientist observing an actual atom similar to hypothetical Atom 1 in the figure might observe
(A) three spectral lines close together and two other spectral lines close together
(B) light blinking at six different frequencies
(C) a much brighter light emanating from one electron than from any other.
(D) four distinct spectral lines emanating from six different electrons.
4. Based on the spectroscopic measurements shown in Table 1, which of the atoms in Figure 1 (Atom 1,
Atom2, or Atom 3) is most similar to the one the scientists were observing, and why ?
(A) Atom 2, because it contains four different energy levels
(B) Atom 3, because it contains four different energy levels
(C) Atom 1, because the frequencies listed in Table 1 indicate a high level of atomic activity.
(D) Atom 3, because there is a comparatively small difference between exactly two of the four frequencies
listed in Table 1
5. The laws of atomic physics prohibit electron movements between certain energy states. In atomic
physics. these prohibitions are called forbidden transitions. Based on Figure 1, which of the following
is most accurate ?
(A) Atom 2 has the same number of forbidden transitions than Atom 3
(B) Atom 2 has more forbidden transitions than atom 3
(C) Atom 3 has the same number of forbidden transitions as Atom 1
(D) Atom 1 has fewer forbidden transitions than Atom 2

PASSAGE : 2
In quantum mechanics, some quantities are discrete and cannot be continuous. One of these quantities
is the energy. Energy can only take certain values E1, E2, E3, E4......., which are called energy levels.
The energy cannot take any values between E1 and E2, or E2 and E3 or E3 and E4 etc. Certain transitions
from one energy level to another result in the emission of a photon of radiation, whereas others can
only take place if a photon is absorbed. The energy levels in a newly discovered gas are expressed as:
E1z 2
En
n2
in which E1z2 is the ground state energy. Take z = 1 for simplicity, but do not assume that the gas is
hydrogen. An experiment is designed to measure the energy as a functions of the level. The results
obtained are as follows :
n En(eV)
2 144
3 64
4 36
6. The ionization energy of the gas must be
(A) 244 eV (B) 576 eV (C) 144 eV (D) +13.6 eV
7. The ground state energy is
(A) 144 eV (B) + 144 eV (C) 244 eV (D) none of the above

www.physicsashok.in 141
MODERN PHYSICS
8. Which of the following shapes is most likely to represent the graph of En versus 1/n2 ?

(A) (B) (C) (D)

9. A transition from the n = 2 state to the n = 3 state results :


(A) in emission of a photon of energy 144 eV (B) in emission of a photon of energy 80 eV
(C) in emission of an ultraviolet photon (D) only accomplished if a photon is absorbed
10. A transition from the n = 4 state to the n = 3 state results :
(A) in emission of a photon of energy 28 eV (B) in emission of a photon of energy 13.6 eV
(C) in emission of an infrared photon (D) only accomplished if a photon is absorbed.

PASSAGE : 3
The power per unit area reaching the Earths surface from cathode Anode
the sun, averaged over 24 hours, is 0.2 kW/m 2. This solar (emitter) g ht (collector)
e n t li
energy can be converted directly into electrical energy via inc id
photocell
the photoelectric effect. For example, in the photoelectrical
cell shown in figure 1, a cathode emits electrons when A
illuminated by light of a high enough frequency. The ejected Ammeter
electrons travel to the anode, and a small electric current flows. An electron within the cathode requires
a minimum energy to break free from the cathode surface. This minimum energy is known as the work
function, W, and is a constant intrinsic to the material of which the cathode is composed. An individual
photonincident on the cathode collides with an electron and is absorbed, transferring all of its energy to
the electron. The energy of each incident photon is given by Ep = hf, where f is the frequency of incident
light and h is Plancks constant. If Ep is less than W, then no electrons will be ejected from the
cathode at all. The maximum kinetic energy Emax of an electron liberated from the cathode is given by
: Emax = Ep W
A voltage source can be connected across the photoelectric cell to oppose the current flow. At a
critical applied voltage, called the stopping voltage, even an electron ejected from the cathode with a
kinetic energy of Emax will not be able to reach the anode therefore the current will stop altogether. The
value of this stopping voltage is dependent only on Emax.
11. The most efficient modern photovoltaic cells can covert the Suns energy into electrical energy with an
efficiency of 35%. Approximately what area would have to be covered by such cells in order to supply
a household with 20 kW-hourse of electrical energy per day
(A) 0.5 m 2 (B) 12 m 2 (C) 285 m 2 (D) 6850 m 2
12. Light intensity is defined as the energy flowing per unit area per unit time for an area perpendicular to
the direction of energy flow. In an experiment, the frequency of light incident on the cathode of a
photoelectric cell is held constant, but the intensity is varied. As the intensity of the incident light is
increased, the stopping voltage.
(A) increases, because more electrons are ejected from the cathode as the number of photons striking
it increases.
(B) remains the same, because the energy supplied to one electron depends only on the energy of an
individual photon.
(C) increases, because the electrons in the cathode absorb more energy per unit time
(D) remains the same, because each incident photon shares its energy between several electrons in
the cathode.
13. The behavior of light is sometimes explained in terms of particles and sometimes in terms of waves.
Which of the following CANNOT be explained by a theory that refers to light in terms of waves alone
(A) Current flow in a photoelectric cell can be stopped by reducing the intensity of the incident light
while maintaining the same frequency.
(B) An electron requires energy to escape from the surface of a photosensitive cathode.
(C) Current flow in a photoelectric cell can be stopped by reducing the frequency of the incident light
while maintaining the same intensity.
(D) The angle through which light is refracted when it moves from one medium to another is a function of
frequency, rather than intensity.

www.physicsashok.in 142
MODERN PHYSICS
14. Under which of the following conditions will the stopping voltage across a photoelectric cell be greatest-
(A) The wavelength of the incident light is short, and the work function of the cathode material is low.
(B) The wavelength of the incident light is short, and the work function of the cathode material is high.
(C) The wavelength of incident light is long, and the work function of the cathode is low.
(D) The wavelength of the incident light is long, and the work function of the cathode material is high.

PASSAGE : 4
An x- ray tube consists of two metal electrodes, a heated filament cathode,
and an anode containing the metal target sealed under high vaccum in a glass
envelope. The heated filament in the cathode emits electrons which are
accelerated by a high DC voltage and collide with the positive anode target. Kp
Two different types of x-ray spectra may be seen. The continuous or I
bremsstrahlung spectrum that is always present is produced when the electron
Ka
penetrates through the outer electron cloud and is abruptly accelerated by the
large positive charge on the nucleus of a heavy atom. The production of x-rays
increases with increasing atomic number but is typically no more than 1% min
efficient, the remaining energy appearing as heat in the target metal. The sharp line spectra that can be
seen at higher voltages occur when the incident electrons eject an inner shell electron, such as n = 1 shell
electron. The spectral line is produced when an electron, say from n = 2, fills the vacancy in the n = 1 shell,
emitting an x-ray photon whose energy corresponds to the energy difference between the n = 2 and n = 1
shells. The intensity of x-rays is proportional to the number of photons created. The photon energy E = hf
= hc/ where h is Plancks constant and c is the speed of light. Figure 1 is a sketch of intensity versus
wavelength for a molybdenum target with an accelerating voltage of 35,000 V.
15. Figure shows that the continuous x-ray spectrum has a minimum (cut-off) wavelength. No shorter
wavelengths are emitted from the tube. This minimum wavelength corresponds to :
(A) the smallest number of emitted photons (B) the highest energy photons emitted
(C) the type of cathode used (D) the type of anode material used.
16. The sharp K peak in figure 1 corresponds to an electron transition from state n = 2 to n = 1, where K
corresponds to a transition from state n = 3 to n = 1. The K line peak is higher than the K because
(A) it is due to a higher atomic number target metal (B) K is the more energetic transition
(C) the K transition is more probable than the K transition, so its intensity is higher
(D) the K transition occurs in the valence shells instead of the inner shells
17. The current to the heated filament in the cathode is increased while the accelerating voltage is kept
constant. This increased current increases the number of electrons striking the target increasing the
overall intensity. What effect does this have on the minimum wavelength value ?
(A) The minimum value will move to shorter wavelength values.
(B) It will move to longer wavelength values
(C) There will no longer be a cutoff wavelength.
(D) The minimum wavelength will remain the same.
18. If the accelerating voltage, V0, increased while keeping the filament current constant, the overall intensity
will also increase. What effect will this have on the wavelength position where the two peaks are
observed?
(A) They will occur at the same wavelengths
(B) The peaks occur at shorter wavelengths due to the higher wavelengths due to the higher energies
available.
(C) They may diappear because all energies may exceed those of the n = 3 to n = 1 transition
(D) The K occur at longer wavelengths but the K occur at shorter wavelengths.
19. Because the efficiency of x-ray production increases with increasing atomic number, Z, it would seem
that lead (Z = 82) would be a better target material than tungsten (Z = 74). However, tungsten (also
used in ordinary light bulb filaments) is the most common target meal, whereas lead is never used. this
is because-
(A) lead has too many electrons (B) lead becomes radioactive
(C) lead would melt, whereas tungsten has a very high melting point
(D) lead will not get hot enough to produce and x-rays.

www.physicsashok.in 143
MODERN PHYSICS
PASSAGE : 5
Student in a medical physics class are given the assignment of planning a nuclear medicine facility.
They not only design the rooms and choose the major equipment they also will have to solve elementary
problems dealing with treatment, doses, radiation protection and safety, and the general principles of physics
of typical isotopes that might be used in diagnostic nuclear medicine. They are required to be familiar with
concepts of half life, half-thickness for shielding and the decay schemes of representative isotopes.
20. The most common isotope used in diagnostic work is Technicium. It is furnished from a generator or
cow in which the negative beta decay of Molybdenum -99 produces the desirable metastable isotope
of Technicium according to the following decay scheme ?
42
Mo99 Z TcA + 1e0
What are the atomic number , Z and mass number A, of the Tc isotope ?
(A) 41, 99 (B) 42, 99 (C) 43, 98 (D) 43, 99
21. If the Mo99 has a physical half-life of 67 hours, about what fraction is left after 5.5 days ?
(A) 0.10 (B) 0.25 (C) 0.40 (D) 0.45
22. This isotope of Technicium has a physical half life of 6 hours. When it is tagged onto a polyphosphate
carrier used for a certain procedure, there is a biological half-life of 12 hours (for the biological excretion
of the carrier). What is the effective half life in this case ?
(A) 4.0 hours (B) 7.5 hours (C) 10 hours (D) 14 hours
23. The cow was milked at 8.00 A.M. At 2.00 PM the activity of the milked sample is measured by a
technician and found to be 200 millicuries. What was the activity of the Technicium at 8.00 A.M.
(A) 100 mCi (B) 150 mCi (C) 300 mCi (D) 400 mCi
24. Thallium -201 is used for myocardial prefusion studies of the heart. It decays by electron capture when
the nucleus captures one of the atoms own orbital electrons (converting a proton in the nucleus into an
uncharged neutron), with the emission of gamma rays used for the imaging. What are the atomic
number Z, the mass number, A, of the Mercury isotope produced in the decay of the Thallium -201 ?
81
Tl 201 + 1e0 Z HgA + Is
(A) 80, 201 (B) 80, 202 (C) 81, 201 (D) 81, 203
25. The Thallium -201 half-life is 74 hours. If the sample has an activity of 80 millicuries initially, what will
be the activity after 9.25 days ?
(A) 2.5 mCi (B) 5 mCi (C) 10 mCi (D) 20 mCi
26. One advantage of the Thallium isotope is the low whole body absorbed dose per millicurie, which for
Tl-201 is 70 millirads/millicurie. If the recommended amount to be injected for a heart scan is 10
microcurie per kg of body mass, what would be the whole body dose in millirads for a 70 kg person ?
(A) 34 mrad (B) 49 mrad (C) 72 mrad (D) 134 mrad
27. Another feature that makes Technicium a desirable isotope for diagnostic nuclear medicine use is that
it is a pure gamma emitter. What is the meaning of the terminology pure gamma emitter?
(A) The gamma radiation stays in the patients tissue while the electrons are detected.
(B) Particles emitted cannot escape tissue while the gama radiation escapes
(C) The isotope decay emits only penetrating gamma radiation that can escape from tissue and be
detected.
(D) The isotope decay emits energetic electrons that act like gamma rays.
28. A patient is injected with Technicium and is estimated to have received a whole body dose of 400
millirads. If the Quality Factor is 1 for gamma radiation and 3 for low energy neutrons, what was the
doese received by the patient in rem units ?
(A) 4 mrems (B) 400 mrems (C) 1200 mrems (D) 4000 mrems
29. The half-thickness of lead for the absorption of the gamma radiation from a particular isotope is 0.4 cm
of lead. How many half thicknesses are necessary to reduce the radiation penetration to less than 1%
and how thick would the lead be ?
(A) 2 half - thickness, 1.2 cm (B) 4 half-thicknesses, 3.2 cm
(C) 7 half-thicknesses, 2.8 cm (D) 11 half-thicknesses , 4.4 cm

PASSAGE : 6
Medical researchers and technicians can track the characteristic radiation patterns emitted by certain
inherently unstable isotopes as they spontaneously decay into other elements. The half life of a radioactive
isotope is the amount of time necessary for one-half of the initial amount of its nuclei to decay. the decay
curves of isotopes 39Y90 and 39Y91 are graphed below as functions of the ratio of N, the number nuclei remaining
after a given period to N0, the initial number of nuclei.

www.physicsashok.in 144
MODERN PHYSICS
1.0 1.0
0.9 0.9
0.8 90 0.8 91
Y
39 39 Y
0.7 0.7
0.6 0.6
N/N0 0.5
N/N0 0.5
0.4 0.4
0.3 0.3
0.2 0.2
0.1 0.1
1 2 3 4 5 6 30 60 90 120 150 180
Time (days) Time (days)
30. The half-life of 39Y90 is approximately :
(A) 2.7 days (B) 5.4 days (C) 27 days (D) 48 days
31. What will the approximate ratio of 39Y90 to 39Y91 be after 2.7 days if the initial samples of the two
isotopes contain equal numbers of nuclei ?
(A) 1 : 1 (B) 1 : 2 (C) 2 : 1 (D) 10 : 1
32. When inhaled by humans, 39Y90 accumulates in the gastrointestinal tract, whereas 39Y91 accumulates in
the bones. If the total amount of each isotope inhaled goes to the specified area, which of the following
situations will exist three days after a patient inhales these substances, assuming none of the isotopes
leave the assuming none of the isotopes leave the specified areas due to physiological factors ?
(A) The amount of 39Y91 in the gastrointestinal tract will be approximately equal to the total amount
inhaled.
(B) The amount of 39Y90 in the bones will be approximately one-half of the total amount inhaled
(C) The amount of 39Y90 in the gastrointestinal tract will be approximately one-half of the total amount
inhaled
(D) None of the 39Y91 inhaled will be left in the bones.
33. Approximately how many 39Y91 nuclei will exist after three half -lives have passed, if there are 1,000
nuclei to begin with ?
(A) 50 (B) 125 (C) 250 (D) 500
34. Which of the following conclusions is / are supported by the information given in the passage ?
I. 39Y90 is less stable than 39Y91
II. Only one-quarter of the original amount of 39Y90 will remain after 116 days.
III. 39Y90 and 39Y91 are both radioactive
(A) I only (B) III only (C) I and II only (D) I and III only

PASSAGE : 7
Scientists are working hard to develop nuclear fusion reactor. Nuclei of heavy hydrogen, 12 H , known as
deuteron and denoted by D, can be thought of as a candidate for fusion reactor. The DD reaction is
2
1 H + 12 H 32 He + n + energy. In the core of fusion reactor, a gas of heavy hydrogen is fully ionized into
deuteron nuclei and electrons. This collection of 12 H nuclei and electrons is known as plasma. The nuclei
move randomly in the reactor core and occasionally come close enough for nuclear fusion to take place.
Usually, the temperatures in the reactor core are too high and no material wall can be used to confine the
plasma. Special techniques are used which confine the plasma for a time t0 before the particles fly away
from the core. If n is the density (number/volume) of deuterons, the product nt0 is called Lawson number. In
one of the criteria, a reactor is termed successful if Lawson number is greater than 5 1014s/cm3.
e2
5
It may be halpful to use the following : Boltzmann constant k = 8.6 10 eV/K; = 1.44 109
4 0
eVm. [JEE, 09]

www.physicsashok.in 145
MODERN PHYSICS
35. In the core of nuclear fusion reactor, the gas becomes plasma because of
(A) strong nuclear force acting between the deuterons
(B) Coulomb force acting between the deuterons
(C) Coulomb force acting between deuteron-electron pairs
(D) the high temperature maintained inside the reactor core.

36. Assume that two deuteron nuclei in the core of fusion reactor at temperature T are moving towards each
other, each with kinetic energy 1.5 kT, when the separation between them is large enough to neglect Coulomb
potential energy. Also neglect any interaction from other particles in the core. The minimum temperature T
required for them to reach a separation of 4 1015 m is in the range.
(A) 1.0 109 K < T < 2.0 109 K (B) 2.0 109 K < T < 3.0 109 K
(C) 3.0 109 K < T < 4.0 109 K (D) 4.0 109 K < T < 5.0 109 K

37. Results of calculations for four different designs of a fusion reactor using DD reaction are given below.
Which of these is most promising based on Lawson criterian ?
(A) deuteron density = 2.0 1012 cm3, confinement time = 5.0 103 s
(B) deuteron density = 8.0 1014 cm3, confinement time = 9.0 101 s
(C) deuteron density = 4.0 1023 cm3, confinement time = 1.0 1011 s
(D) deuteron density = 1.0 1024 cm3, confinement time = 4.0 1012 s

www.physicsashok.in 146
MODERN PHYSICS

Level 2
1. When a surface is irridated with light of = 4950 A , a photo current appears which vanishes if a retarding
potential greater then 0.6V is applied across the photo tube. When a different source of light is used, it
is found that the critical retarding potential is changed to 1.1 V. what is the work function of the surface
and the wavelength of the second source ? If the photo-electrons (after emission from the source) are
subjected to a magnetic field of 10 tesla what changes will be observed in the above two retarding
potentials?

2. A particle of charge equal to that of an electron and mass 208 times the mass of the electron (called a
mu-meson) moves in a circular orbit around a nucleus of charge +3e. (Take the mass of the nucleus to
be infinite.) Assuming the Bohr model of the atom to be applicable to this system,
(i) derive an expression for the radius of the nth Bohr-orbit,
(ii) find the value of n for which the radius of the orbit is approximately the same as that of the first
Bohr orbit for the hydrogen atom, and
(iii) find the wavelength of the radiation emitted when the meson jumps from the third orbit to the
first orbit. (Rydbergs constant = 1.097 x 107 m 1 )

3. Hydrogen atom in its ground state is excited by means of monochromatic radiation of wavelength 975 A .
How many different lines are possible in the resulting spectrum ? Calculate the longest wavelength
amongst them. You may assume the ionization energy for hydrogen atom as 13.6 eV.

4. An X-ray tube works at a potential difference of 100,000 V. Only 0.1% of the energy of cathode rays is
converted into X-ray radiation and heat is generated in the target at the rate of 120 calorie per second.
What current does the tube pass and what is the energy and velocity of an electron when it reaches the
target?

5. A laser emits a light pulse of duration 0 .13 ms and energy E = 10 J. Find the mean pressure exerted
by such a light pulse when it is focussed into a spot of diameter d = 10 m on a surface perpendicular
to the beam and possessing a reflection coefficient = 0.5

6. A short light pulse of energy E = 7.5 J falls in the form of a narrow and almost parallel beam on a mirror
plate whose reflection coefficient is = 0.60. The angle of incidence is 30. In terms of the corpuscular
theory find the momentum transferred to the plate.

7. The binding energy of an electron in the ground state of He atom is equal to E0 = 24.6 eV. Find the energy
required to remove both electrons from the atom.

8. A stationary He+ ion emitted a photon corresponding to the first line of the Lyman series. That photon
liberated a photoelectron from a stationary hydrogen atom in the ground state. Find the velocity of the
photoelectron.

9. Taking into account the motion of the nucleus of a hydrogen atom, find the expressions for the electrons
binding energy in the ground state and for the Rydberg constant. How much (in per cent) do the binding
energy and the Rydberg constant, obtained without taking into account the motion of the nucleus, differ
from the more accurate corresponding values of these quantities?

10. Calculate the separation between the particles of a system in the ground state, the corresponding
binding energy, and the wavelength of the first line of the Lyman series, if such a system is
(a) a mesonic hydrogen atom whose nucleus is a proton ( in a mesonic atom an electron is replaced
by a meson whose charge is the same and mass is 207 that of an electron);
(b) a positronium consisting of an electron and a positron revolving around their common centre of
masses.

www.physicsashok.in 147
MODERN PHYSICS
11. In accordance with classical electrodynamics an electron moving with an acceleration a loses its energy
d 2 e2
due to radiation as : 3 a 2 Estimate the time during which an electron moving in a hydrogen
dt 3c
atom along a circular orbit of radius r = 50 pm would have fallen onto the nucleus. Assume a to be
directed permanently towards the nucleus.

12. The Earth revolves round the Sun due to gravitational attraction. Suppose that the Sun and the Earth are
point particles with their existing masses and that Bohrs quantization rule for angular momentum is
valid in the case of gravitation.
(a) Calculate the minimum radius the Earth can have for its orbit.
(b) What is the value of the principal quantum number n for the present radius? Mass of the Earth =
6.0 x 1024 kg, mass of the Sun = 2.0 x 1030 kg, Earth-Sun distance = 1.5 x 1011 m.

13. A parallel electron beam is obtained from the application of


accelerating voltage V0 = 2 x 104 V. These electrons are sent
travelling in the direction normal to an infinitely long straight
copper wire of radius r0 = 108 m as shown in figure. The wire
carries uniform positive charge with charge density b
= 4.4 x 10111 c/m, the distance of the electrons closest
approach to the wire if uncharged is represented by bmax = 104 m.
The electrons after passing the charged copper wire land on the L
screen located at distance L = 0.3 cm ( L >.b) from the wire. Electron Beam Screen
At the beginning of the experiment the electron beam is confined
within the normal distance to the wire of b ( collision parameter) (see Figure).
(a) Determine electric field E due to the charged copper wire and sketch a graph of electric field E as
a function of distance measured from the axis of the wire to inside as well as outside the wire.
(b) Calculate the angle of deflection of electron beam which has the collision parameter b and the
electron does not collide with the wire. If f is a small angle between the direction of the original
velocity of the electron and the direction of final velocity of the electron arriving at the screen.
Calculate f .
(c) Show that two plane waves representing deflected beams in the upper and lower parts give
interference pattern on the screen. Calculate the number of bright bands in the interfence pattern.

14. To stop the flow of photoelectrons produced by electromagnetic radiation incident on a certain metal, a
, what is
negative potential of 300 volts is required. If the photoelectric threshold of the metal is 1500 A
the frequency of the incident radiation? Is this radiation visible?

15. A toy truck has dimensions as shown in Figure and its width normal to
the plane of this paper is d. Sun rays are incident on it as shown in b
h
figure. If intensity of sun rays is and all surfaces of truck are perfectly
black, calculate tension in thread used to keep truck stationary. Neglect
fraction. Thread

a
16. A monochromatic beam of light ( = 4900 A) incident normally upon a surface produces a pressure of 5
x 107 N/m 2 on it. Assuming that 25% of the light incident is reflected and the rest absorbed, find the
number of photons falling per second on a unit area of thin surface.

17. A beam of light consists of four wavelength 4000 A , 4800 A , 6000 A and 7000 A , each of intensity 1.5
x 103 wm 2. The beam falls normally on an area 104 m 2 of a clean metallic surface of work function 1.9
eV.Assuming no loss of light energy calculate the number of photo electrons liberated per second.

18. A single electron orbits around a stationay nucleus of charge +Ze, where Z is a constant and e is the
magnitude of the electric charge. It requires 47.2 eV to excite the electronfromthe second Bohrorbit
tothe third Bohr orbit. Find :

www.physicsashok.in 148
MODERN PHYSICS
(i) the value of Z
(ii) the energy required to excite the electron from the third to the fourth Bohr orbit.
(iii) The wavelength of the electromagnetic radiation required tomove the electron from.
(iv) Thekinetic energy, the potentialenergy and the angular momentumof the electron in the first Bohr
ortib.
(v) the radius of the first Bohr orbit.

19. Radiations from hydrogenic C gas corresponding to 2nd line of Lyman series falls on a hydrogenic atom
where rotating particles mass and charge are unknown. Nucleus contains one proton only. This atom is
excited to 4th excited energy level.
(a) Find a relation for valid values of mass and charge of the rotating particles.
(b) Find the maximum K.E.of the electron ejected due to aforesaid radiation falling on a hydrogenic
potassium having some of the atoms in the ground energy level & some in the 4th energy level.

20. Stopping potentials of 24, 100, 110 and 115 kV are measured for photoelectrons emitted from a certain
element when it is radiated with monochromatic x-ray. If this element is used as a target in an x-ray
tube, what will be the wavelength of K line?

21. Light from a discharge tube containing hydrogen atoms falls on the surface of a piece of sodium. The
kinetic energy of the fastest photo-electrons emitted form sodius is 0.73 eV. The work fraction for sodium
is 1.82 eV. Find :
(i) the energy of the photons causing the photoelectric emission.
(ii) the quantum numbers of the two levels involved in the emission of these photons.
(iii) the charge in the angular momentum of the eletron in the hydrogen atom in the above transition.
(iv) the recoil speed of the emitting atom assuming it to be at rest before the transition.
(Ionization potential of hydrogen is 13.6 eV).

22. A gas of hydrogen like atoms can absorb radiations of 68 eV. Consequently, the atoms emit radiations of
only three different wavelengths. All the wavelengths are equal or smaller than that of the absorbed
photon.
(a) Determine the initial state of the gas atoms.
(b) Identify the gas atoms.
(c) Find the minimum wavelength of the emitted radiation.
(d) Find the ionization energy and the respective wavelength for the gas atoms.

23. According to the Thomson model, a helium atom consists of a cloud of positive charge, within which two
electrons sit at equlibrium positions. Assume that the positive cloud has a charge +2e uniformly distributed
over the volume of a sphere of radius 0.50 A .
(a) Find the equilibrium position of the two electrons. Assume that the electrons are symmetrically
placed with respect to the centre.
(b) What is the frequency of small radial oscillations of the electrons about their equilibrium positions.
Assume that the electrons move symmetrically with identical amplitudes.

24. The Rydberg constants for hydrogen and singly ionized helium are R1 and R2 respectively. Find the ratio
of the mass of the proton to that of the electron in terms of R1, R2 and R .

25. A sample contains 102 kg each of two substances A and B with half lives 4 second and 8 second
respectively. Their weights are in the ratio of 1.2. Find the amounts of A and B after an interval of 16
second.

26. A sample of uranium is a mixture of three isotopes 92U234, 92U235 and 92U238 present in the ratio of 0.006%,
0.71% and 99.284% respectively. The half-lives of these isotopes are 2.5 x 105 years, 7.1 x 108 years
and 4.5 x 109 years respectively. Calculate the contribution to activity (in %) of each isotope in this
sample.

www.physicsashok.in 149
MODERN PHYSICS
27. Polonium ( 84Po210 ) emits 2He4 particles and is converted into lead (82Pb206). This reaction is used for
producing electric power in a space mission. Po210 has half life of 138.6 days. Assuming an efficiency of
10% for the thermoelectric machine, how much Po210 is required to produce 1.2 x 107 J of electric energy
per day at the end of 693 days. Also find the initial activity of the material. (Given : masses of nuclei Po210
= 209.98264 a.m.u., Pb206 = 205.97440 a.m.u., 2He4 = 4.00260 a.m.u., 1 a.m.u. = 931 MeV and Avogadro
number = 6x1023/mol.

28. 103 kg of radioactive isotope (atomic mass 226) emits 3.72 x 1010 -particles in a second. Calculate the
half-life of the isotope. If 4.2 x 102 joule is released in one hour in this process, what is the average
energy of the -particle?

29. In an ore containing uranium, the ratio of U238 to Pb206 nuclei is 3. Calculate the age of the ore, assuming
that all the lead present in the ore is the final stable product of U238. Take the half-life of U238 to be 4.5 x
109 year.

30. A7 Li target is bombarded with a proton beam current of 104 A for 1 hour to produce 7Be of activity 1.8 x
108 disintegrations per second. Assuming that one 7Be radioactive nuclei is produced by bombarding
1000 protons, determine its half-life.

Level 3

1. A small quantity of solution containing 24Na radionuclide (half life 15 hours) or activity 1.0 microcurie is
injected into the blood of a person. A sample of the blood of value 1 cm3 taken after 5 hours shows an
activity of 296 disintegrations per minute. Determine the total value of blood in the body of the person.
Assume that the radioactive solution mixes uniformly in the blood of the person. (1 Curie = 3.7 1010
disintegrations per second) [JEE, 94]

2. At a given instant there are 25% undecayed radio-active nuclei in a sample. After 10 sec the number of
undecayed nuclei remains to 12.5%. Calculate : [JEE, 96]
(i) mean-life of the nuclei and
(ii) The time in which the number of undecayed nuclear will further reduce to 6.25% of the reduced number.

3. Consider the following reaction ; 2H1 + 2H1 = 4He2 + Q. [JEE, 96]


Mass of the deuterium atom = 2.0141 u; Mass of the helium atom = 4.0024 u
This is a nuclear _______ reaction in which the energy Q is released is _______ MeV.

4. The element Curium 96 248


Cm has a mean life of 1013 seconds. Its primary decay modes are spontaneous
fission and decay, the former with a probability of 8% and the later with a probability of 92%. Each fission
releases 200 MeV of energy. The masses involved in decay are as follows :
248 244
96 Cm = 248.072220 u, 94 Pu = 244.064100 u & 42 He = 4.002603 u.
Calculate the power output from a sample of 1020 Cm atoms. (Iu = 931 MeV/C2) [JEE, 97]

5. Nuclei of a radioactive element A are being produced at a constant rate . the element has a decay constant
. At time t = 0, there are N0 nuclei of the element. [JEE, 98]
(a) Calculate the number N of nuclei of A at time t.
(b) If = 2N0, calculate the number of nuclei of A after one halflife of A & also the limiting value of N as
t .

www.physicsashok.in 150
MODERN PHYSICS
6. Photoelectrons are emitted when 400 nm radiation is incident on a surface of work function 1.9 eV. These
photoelectrons pass through a region containing -particles. A maximum energy electron combines with an
-particle to form a He+ ion, emitting a single photon in this process. He+ ions thus formed are in their fourth
excited state. Find the energies in eV of the photons, lying in the 2 to 4eV range, that are likely to be emitted
during and after the combination. [Take, h = 4.14 1015 eVs] [JEE, 99]

7(a). A hydrogen-like atom of atomic number Z is in an excited state of quantum number 2 n. It can emit a
maximum energy photon of 204 eV. If it makes a transition to quantum state n, a photon of energy 40.8 eV
is emitted. Find n, Z and the ground state energy (in eV) for this atom. Also, calculate the minimum energy
(in eV) that can be emitted by this atom during de-excitation. Ground state energy of hydrogen atom is
13.6 eV. [JEE, 2000]

(b). When a beam of 10.6 eV photon of intensity 2 W/m2 falls on a platinum surface of area 1 104 m2 and
work function 5.6 eV, 0.53% of the incident photons eject photoelectrons. Find the number of photoelectrons
emitted per sec and their minimum and maximum energies in eV. [JEE, 2000]

8. A hydrogen like atom (described by the Bohr model) is observed to emit six wavelengths, originating from
all possible transition between a group of levels. These levels have energies between 0.85 eV and 0.544
eV (including both these values) [JEE, 02]
(a) Find the atomic number of the atom.
(b) Calculate the smallest wavelength emitted in these transitions.

9. Two metallic plates A and B each of area 5 104 m2, are placed at a separation of 1 cm. Plate B carries
a positive charge of 33.7 1012 C. A monochromatic beam of light, with photons of energy 5 eV each,
starts falling on plate A at t = 0 so that 1016 photons fall on it per square meter per second. assume that one
photoelectron is emitted for every 106 incident photons. Also assume that all the emitted photoelectrons are
collected by plate B and the work dunction of plate A remains constant at the value 2 eV. Determine
(a) the number of photoelectrons emitted up to t = 10 sec.
(b) the magnitude of the electric field between the plates A and B at t = 10 s and
(c) the kinetic energy of the most energetic photoelectron emitted at t = 10 s when it reaches plate B.
(Neglect the time taken by photoelectron to reach plate B) [JEE, 02]

10. Frequency of a photon emitted due to transition of electron of a certain elemrnt from L to K shell is found to
be 4.2 1018 Hz. Using Moseleys law, find the atomic number of the element, given that the Rydbergs
constant R = 1.1 107 m1. [JEE, 03]

11. In a photoelectric experiment set up, photons of energy 5 eV falls on the cathode having work function 3
eV.
(a) if the saturation current is iA = 4A for intensity 105 W/m2, then plot a graph between anode potential
and current.
(b) Also draw a graph for intensity of incident ratiation of 2 105 W/m2 ? [JEE, 03]

12. A radioactive samle emits n -particles in 2 sec. In next 2 sec it emits 0.75 n -particles, what is the mean
life of the sample ? [JEE, 03]

www.physicsashok.in 151
MODERN PHYSICS
13. The potential energy of a particle of mass m is given by
E 0 x 1
V(x) 0
0 x 1
1 and 2 are are the de-Broglie wavelengths of the particle, when 0 x 1 and x > 1 respectively. If the
total energy of particle is 2E0, find 1/2. [JEE, 05]

14. Highly energetic electrons are bombarded on a target of an element containing 30 neutrons. The ratio of
radii of nucleus to that of helium nucleus is (14)1/3. Find [JEE, 05]
(a) atomic number of the nucleus
(b) the frequency of K line of the X-ray produced. (R = 1.1 107 m1 and c = 3 108 m/s)

15. In hydrogen-like atom (z = 11), nth line of Lyman series has wavelength equal to the de-Broglies wavelength
of electron in the level from which it originated. What is the value of n ? [Take : Bohr radius (r0) = 0.53
and Rydberg constant (R) = 1.1 107 m1] [JEE, 06]

Answer Key
Assertion-Reason

Q. 1 2 3 4 5 6 7 8 9 10
Ans. B C D B A A B A A B

Match the Column

1. (A) (U), (B) (R), (C) (P), (D) (S)

2. (A) (S), (B) (T), (C) (Q), (D) (P)

3. (A) (R), (B) (P), (C) (T), (D) (U)

4. (A) (R), (B) (U), (C) (V), (D) (Q)

5. (A) (Q), (B) (R), (C) (S), (D) (P)

6. (A) (PQ), (B) (PR), (C) (PS), (D) (PQR)

7. (A) (P), (B) (T), (C) (U), (D) (R)

8. (A) (PR), (B) (QS), (C) (P), (D) (Q)

9. (A) (PQT), (B) (Q), (C) (S), (D) (S)

www.physicsashok.in 152
MODERN PHYSICS

Level 1

Q. 1 2 3 4 5 6 7 8 9 10
Ans. D C D B C A C D D A
Q. 11 12 13 14 15 16 17 18 19 20
Ans. B B B D A D A C ABC C
Q. 21 22 23 24 25 26 27 28 29 30
Ans. C D D C D C B D D B
Q. 31 32 33 34 35 36 37 38 39 40
Ans. B C AD BC ABC BD ABD AB AC BD
Q. 41 42 43 44 45 46 47 48 49 50
Ans. ABC CD AC AB AB AC AD AC AB C
Q. 51 52 53 54 55 56
Ans. (i-B), (ii-A) AD C B (i-CD), (ii-D) (a-C), (b-B), (c-B), (d-E), (e-C)
Q. 57 58 59 60 61 62 63 64 65
Ans. C (a-C), (b-A) A B C AC A B A
Q. 66 67 68 69 70 71 72
Ans. B BD C C A A A

Passage

Q. 1 2 3 4 5 6 7 8 9 10
Ans. C B A D B B D A D A
Q. 11 12 13 14 15 16 17 18 19 20
Ans. B B C A B C D A C D
Q. 21 22 23 24 25 26 27 28 29 30
Ans. B A D A C B C B C A
Q. 31 32 33 34 35 36 37
Ans. B C B D D A B

Level 2
1.
0 1.9 eV , = 4125 A 2. (i) 8.426 x 1014 n2 (ii) 25 (iii) 5.478 x 1011 m


3. n = 4; = 18800 A 4. 50.6 A 5. (p) 41 d2 c 50 atm

6. P E / c 1 2 2 cos 2 35 nN.s 7. 0 4R 79 eV.

8. 2 R/ m 3.1106 m/s, where m is the mass of the electron.


9. b e 4 / 2 3 , where is the reduced mass of the system. If the motion of the nucleus is not taken into
account, these values (in the case of a hydrogen atom) are greater by m/M 0.055%, where m and M
are the masses of an electron and a proton.

www.physicsashok.in 153
MODERN PHYSICS
10. (a) 0.285 pm, 2.53 keV, 0.65 nm; (b) 106 pm, 6.8 eV, 0.243 m.

11. t m 2e c 2 r 3 / 4 e 2 13 ps 12. (a) 2.3 x 10138 m (b) 2.5 x 1074

q L q
13. (a) E for r > r0 = 0; (b) f ; (c) 500 14. v = 7.45 x 1016 cycles/s, no
2 0r 4 0 V0

d
15. b cos a cos h sin sin 16. 2.93 x 1020 17. 11.72 x 1011
c
18. (i) x = 5 ; (ii) 16.53 eV (iii) 36.36 A ; (iv) 1.056 x 1034 kg m 2 s1 (v) 1.06 x 1011 m

19. (a) 1.6 x 1017 02 h 2 (b) The radiation connot eject electron.

20. 0.163 A
21. 2.55 eV, electron jumps from 4th to 2nd orbits, 2.11 x 1034 Js 0.8144 m/s

22. (a) n = 2 (b) z = 6


(c) 28.5 A (d) 25.32 A

0.75
1 1
23. (a) 0.25 A (b) 1.76 x 1016 Hz 24. R
R
1 R 2

25. NA = 6.25 x 104 kg, NB = 2.5 x 103 26. 51.41%, 2.13%, 46.45%

27. A0 = 4.57 x 1021 per day, 320 gm 28. T = 1573 year, Energy per particle = 19.5 MeV
29. 1.868 x 109 year

30. T = 8.63 x 106 s.

Level 3

1. 6 litre 2. (i) t1/2 = 10 sec., tmeans = 14.43 s; (ii) 40 seconds 3. Fusion, 24


1 3N0
4. 33.298 W [ (1 e t ) N 0 e t ] ; (b)
5. (a) N , 2N0
2
6. during combination = 3.365 eV; after combination = 3.88 eV (5 3) & 2.63 eV (4 3)

7.(a) n = 2, z = 4; G.S.E. 217.6 eV; Min. energy = 10.58eV; (b) 6.251019per sec, 0, 5 eV

8. (a) 3; (b) 4052.3 nm 9. (a) 5 107, (b) 2000 N/C, (c) 23 eV


I
8A
I=2105 W/m2
4A
2
10. z = 42 11. I = 105W/m2 12. 1.75n = N0(1e4), 6.95 sec,
ln(4 / 3)
2V VP
18
13. 2 14. v = 1.546 10 Hz 15. n = 24

XXXX

www.physicsashok.in 154
NUCLEAR PHYSICS

DHANALAKSHMI NAGAR

NEAR ANNAMAIAH CIRCLE,

TIRUPATI.

PH NO. 9440025125
NUCLEAR PHYSICS

RADIO ACTIVITY
INTRODUCTION OF ATOMIC NUCLEUS
(a) The atomic nucleus consists of two types of elementary particles, viz. protons and neutrons. These particles
are collectively called nucleons.
(b) The electrons surround this nucleus to form the atom.
(c) This structure of atom was revealed by the experiments of Rutherford in which a beam of -particles were
made to strike a thin gold foil.
(d) Nucleus can be regarded as a small spherical volume situated at the centre of atom.
(e) Most of the mass of the atom is concentrated in the nucleus.
(f) Nucleus has a positive charge.

PROPERTIES OF AN ATOMIC NUCLEUS


Composition:
(a) All nuclei are composed of two types of particles protons and neutrons.
(b) The only exception is the ordinary hydrogen nucleus which is just a single proton.
(c) A proton has a mass of 1.6726 1027 kg and charge +e (= 1.6 1019 C).
(d) A neutron has a mass of 1.6750 1027 kg and is electrically neutral.
The atomic number Z: This is equal to the number of protons in the nucleus.
The neutron number N: This is equal to the number of neutrons in the nucleus.
The mass number A: This is equal to the number of nucleons (protons + neutrons) in the nucleus.
Thus, A = Z + N
Symbolically a nucleus X shall be represented as Z XA .

Types of Nuclei : Nuclei are of three types:


(i) Isotopes: Nuclei having the same Z value but different N and A values are called isotopes of the element.
eg. 1H1, 1H2 and 1H3 are isotopes of hydrogen.
(ii) Isobars: Nuclides (nuclei of different elements) having the same A value but different Z and N values are
called isobars.
e.g. 1H3 and 2He3 are isobars.
(iii) Isotones: Nuclides having the same N value but different A and Z values are known as isotones. e.g. 1H3
and 2He4 are isotones.

Radius of atomic nucleus:


The size of the nucleus is of the order of fermi (fm).
1 fm = 1015.
Most nucleus are almost spherical with an average radius R given by
R = R0 A1/3 where A is the mass number and R0 is a constant.
R0 ~ 1.2 fm = 1.2 1015 m

www.physicsashok.in 1
NUCLEAR PHYSICS
Mass :
(a) Nuclear masses have been accurately determined with the help of the mass spectrometer.
(b) It is expressed in a.m.u. i.e. atomic mass unit.
(c) This unit is such that mass of the carbon isotope 6C12 is exactly 12 amu.
(d) 1 a.m.u. = 1.66056 1027 kg
(e) Matter can be viewed as a condense form of energy.
(f) The energy corresponding to the mass of a particle when it is at rest is called rest mass energy.
(g) Rest mass energy is E = mc2, from Einstein relation.
(h) Energy equivalent of 1 a.m.u. is equal to 931.5 MeV. i.e. 1 a.m.u. = 931.5 MeV/c2.

Example 51: (a) Calculate the value of 1 a.m.u. from Avogadros number.
(b) Determine the energy equivalent of 1 a.m.u.
Sol: (a) One mole of C12 has a mass of 12 g and contains Avogadro number, NA of atoms.
By definition each C12 atom has a mass of 12 a.m.u.
Thus, 12 g corresponds to (12 NA) a.m.u. which means,
1g 103
1 a.m.u. = N = kg
A 6.022055 1023
or 1 a.m.u. = 1.66056 1027 kg
(b) From Einstein relation, rest mass energy
E = mc2
Hence, energy equivalent of 1 a.m.u.,
E = (1 a.m.u.) c2
= (1.66056 1027 kg) (3 108 m/s)2 = 1.4924 1010 J
Since, 1 eV = 1.6 1019 J,
E = 931.5 MeV
Hence, 1 a.m.u = 931.5 MeV/c2

Density:
Mass of a nucleus can be taken approximately Am, where m is the mass of proton or a neutron and A is
mass number
Mass = Am
Also, assuming the nucleus to be a sphere of radius R, its volume is
4 4 3
V= R3 = R 0 A , [ R = R0A1/3]
3 3
The nuclear density is thus given by,
Am
mass 3m
= = 4 R 3 A = 4R 3
volume 3
0 0

It is thus independent of the mass number A and is therefore nearly the same for all nuclei. Putting R 0 =
1.2 fm = 1.2 1015 m and m = 1.67 1027 kg, we get, = 2.3 1017 kg/m3.
This is almost 1014 times the density of water.
(a) Nuclear density = 2.3 1017 kg/m3
(b) It is nearly the same for all nuclei.
www.physicsashok.in 2
NUCLEAR PHYSICS
C48: Find the mass density of the oxygen nucleus 8O16.
4 4 3
Sol: Volume = R3 = R 0 A [ R = R0A1/3]
3 3
4
= (1.2)3 16 1045 m3, [ R0 = 1.2 1015, A = 16]
3
= 1.16 1043 m3
Mass of oxygen atoms (A = 16) is approximately 16 a.m.u.
mass 16 amu
Therefore density is = =
volume 1.16 1043 m 3
16 1.66 1027
= kg/m3
1.16 1043 m3
= 2.3 1017 kg/m3

NUCLEAR STABILITY
(a) High density of the nucleus suggests a very tight packing of protons and neutrons in it.
(b) The Coulombs repulsive force between two protons in a nucleus is about 1036 times as large as the
gravitational force between them. It is therefore surprising that a nucleus should be so stable.
(c) Nuclei are stable because of the presence of another force, called the nuclear force.

Nuclear force :
It arises due to interaction between protons, proton with neutrons, and neutron with neutrons. This force is
essentially a very strong attractive force and overcomes the electrostatic repulsion between the proton
inside the nucleus.

Properties of Nuclear forces :


(a) These are strong attractive forces.
(b) These are about 100 times stronger than the Coulombs force.
(c) These are short ranged forces (effective upto 10 fm).
(d) They contain a small component of repulsive force which is effective up to a distance of the order of
0.5 fm or 0.5 1015 m or less. This repulsive component prevents the collapse of the nucleus.
(e) These forces are charge independent.
(f) Let d be the range of the effectiveness of nuclear forces, then
0.5 fm d 10 fm
(g) Let Fpp, Fpn and Fnn denote the magnitude of the nuclear force by a proton on a proton, by a proton on a
neutron and by a neutron on a neutron respectively.
Then for a separation of 1 fm, Fpp = Fpn = Fnn.

N
e

N/Z ratio :
lin
Z

(a) N/Z ratio inside a nucleus is responsible for stability of a nucleus.


=
N

(b) It must be greater than or equal to unity and less than 1.6 Z
N N v/s Z curve
i.e. 1 1.6.
Z
www.physicsashok.in 3
NUCLEAR PHYSICS
(c) Reason for N/Z ratio to be greater than unity is due to the fact that protons are positively charged and repel
on another electrically. This repulsion becomes so great in nuclei with more than 10 protons or so that an
excess of neutrons, which produce only attractive nuclear forces, is required for stability. Thus N/Z ratio
increases with increase in Z.
(d) When excess of neutrons or protons in a nuclide is there then the nuclide -decay or -decay to achieve the
required N/Z ratio for stability. This causes radioactive disintegrations of nuclides.
Binding Energy (B.E.) :
The binding energy is equal to the work that must be done to split the nucleus into particles constituting it.
Hence, Energy of nucleus + B.E. = Energy of each nucleon individually.
Let Mass of nucleus = M,
Mass of neutron = mn,
and, Mass of proton = mp
Rest mass energy of nucleus = Mc2,
Rest mass energy of neutron = mnc2,
Rest mass energy of proton = mpc2.
Thus, Mc2 + B.E. = Z mpc2 + (A Z)mn c2
B.E. = [M {Z mp + (A Z)mn}]c2
The quantity, M {Z mp + (A Z)mn} is called mass-defect (m)
Mass defect = m = M {Z mp + (A Z)mn}
B.E. = m.c2
If m is in a.m.u., then
B.E. = m 931.5 MeV
or B.E. = [M {Z mp + (A Z)mn}] 931.5 MeV

NOTE: Negative sign of B.E. represents boundedness of nucleons inside the nucleus.
C49: Find the binding energy of 56 56 1
26 Fe . Atomic mass of Fe is 55.9349 u and that of H is 1.00783 u. Mass of

neutron = 1.00867 u.
56
Sol: Z = The number of protons in 26 Fe = 26 and the number of neutrons, A Z = 56 26 = 30
Then binding energy of 5626 Fe = [M {Zmp + (A Z))mn}]c
2

= [26 1.00783 u + 30 1.00867 u 55.9349 u]c2


= (0.52878 u)c2
= (0.52878 u) (931 MeV/u)
= 492 MeV
56
Negative sign indicates boundedness of nucleons. 26 Fe
Binding energy per

10
B.E. = 492 MeV in magnitude.
nucleon, MeV

8 4
He
Binding energy per nucleon : 6
6
Binding energy per nucleon is obtained by dividing 4
Li
the binding energy of the nucleus by the number A
2
of nucleons in the nucleus.
B.E. 0 50 100 150 200 250
i.e. B.E. per nucleon =
A Mass number, A
www.physicsashok.in 4
NUCLEAR PHYSICS
(a) The adjacent figure shows the dependence of
the B.E. per nucleon, B.E./A on the mass number
A of the nucleus.
(b) Nucleons in nuclei with mass number from 50 to 60 have the highest B.E. The B.E./A for these nuclei
amounts to 8.7 MeV and gradually decreases with increasing A.
56
(c) B.E. per nucleon is highest for 26 Fe .

Example 52: Find the binding energy of 126 C ? Also find the binding energy per nucleon.
Mass of 6C12 atom = 12 a.m.u.
Mass of proton = 1.00759 a.m.u.
Mass of neutron = 1.00898 a.m.u.
Sol: M = mass of C12 atom = 12 a.m.u.
mp = mass of proton = 1.00759 a.m.u.
mn = mass of neutron = 1.00898 a.m.u.
Z = number of proton =6
A Z = number of neutrons = 12 6 = 6
Mass-defect, m = M {Zmp + (A Z)mn}
= 12 (6 1.00759 + 6 1 1.00898) in a.m.u.
= (12 12.009) a.m.u = 0.099 a.m.u.
B.E. = m 931.5 MeV = 0.099 931.5 MeV = 92.22 MeV
Hence, Binding energy = 92.22 MeV (Negative sign indicates boundedness of the nucleon)
B.E. 92.22
Binding energy per nucleon = = MeV = 7.68 MeV
A 12

NUCLEAR COLLISIONS
(a) A nuclear reaction in which a collision between particle a and nucleus X produces Y and particle b is
represented as
a+X Y+ b
(b) The reaction is sometimes expressed in the shorthand notation X (a, b)Y.
(c) Reaction are subjected to the restrictions imposed by
(i) The conservation of charge,
(ii) The conservation of energy,
(iii) The conservation of momentum, and
(iv) The conservation of angular momentum.

Q-Value or Energy of a reaction :


Let m2, m3 are nuclear masses of X and Y respectively.
a X Y X

m1 K1 m2 K2 m3 K3 m4 K4
After collision
Before collision
Initial energy: Ei = m1c2 + m2c2 + K1 + K2
Final energy: Ef = m3c2 + m4c2 + K3 + K4

www.physicsashok.in 5
NUCLEAR PHYSICS
Since, Ei = Ef, (from energy conservation)
[(m1 + m2) (m3 + m4)]c2 = (K3 + K4) (K1 + K2)
The energy, [(m1 + m2) (m3 + m4)]c2, that is released or absorbed in a nuclear reaction is called the Q-
Value or disintegration energy of the reaction.
Hence, Q = [(m1 + m2) (m3 + m4)]c2 J
or, Q = [(m1 + m2) (m3 + m4)] 931.5 MeV, when masses are in a.m.u.

Mass defect:
The quantity [(m1 + m2) (m3 + m4)] is called the mass defect of the reaction and is given by
m = (m1 + m2) (m3 + m4) in a.m.u.
Q = m. 931.5 MeV

Example 53: A neutron breaks into a proton and electron. Calculate the energy produced in this reaction in
MeV. Mass of an electron = 9 1031 Kg, Mass of proton = 1.6725 1027 kg,
Mass of neutron = 1.6747 1027 Kg. Speed of light = 3 108 m/s.
Sol: 0n1 1
H1 + 1e0
Mass defect,
m = [mass of neutron (mass of proton + mass of 1e0)]
= 1.6747 1027 kg (1.6725 + 0.0009) 1027 kg
= 0.0013 1027 kg
Energy released,
Q = m.c2
= 1.3 1030 (3 108)2 kg m2/s2
= 1.17 1013 J
1.17 1013
= eV
1.6 1019
= 0.73 MeV

Example 54. Find the Q-value of the reaction


1
H2 + 3Li6 3Li7 + 1H1
The rest masses of 1H2, 3Li6 3Li7, and 1H1are, respectively, 2.01410 amu, 6.01513 amu, 7.01601 amu
and 1.00783 amu.
Sol. Suppose 1H2 + 3Li6 3Li7 + 1H1 + Q.
Total mass of left-hand side = 2.01410 + 6.01513 = 8.02933 amu
Total mass on right-hand side = 7.01601 + 1.00783 = 8.02384 amu
8.02933 = 8.02384 + Q
or Q = 0.00549 amu = 0.00549 931 MeV ( 1 amu = 931 MeV)
Q = 5.1 MeV

INTRODUCTION OF RADIOACTIVITY
(a) In 1896, Becquerel discovered accidentally that uranium salt crystals emit an invisible radiation which
affected a photographic plate even though it was properly covered.
(b) In 1898, Marie and Pierre Curie and other workers showed that many other substances also emitted similar
radiations.

www.physicsashok.in 6
NUCLEAR PHYSICS
(c) The property of spontaneous emission of radiation from the substance is called radioactivity and such type
of substance is called radioactive substance.
(d) Radioactivity is due to the decay or disintegration of unstable nuclei.
(e) The radiations are being emitted from the nuclei hence it is a nuclear phenomenon, not an atomic phenomenon.
(f) Some examples of radioactive substances are: U, Ra, Th, Po and Np.
(g) Electronic configuration of atom does not have any relationship with radioactivity.
(h) Radioactivity is explained on the basis of quantum mechanics.
(i) No single phenomenon has played so significant role in the development of nuclear physics as radioactivity.
(j) It is not influenced by external parameters such as pressure, temperature, chemical reaction (combination)
or phase of matter.

C50: Uranium salt crystals emit


(a) visible radiation (b) invisible radiation
(c) any type of electromagnetic radiation (d) sound waves
Sol: This is according to discovery of radioactivity of Becquerel.

C51: The radioactivity is a/an


(a) optical phenomenon (b) Atomic phenomenon
(c) nuclear phenomenon (d) photoelectric phenomenon
Sol: Nuclear radiations are obtained from the nuclei hence it is a nuclear phenomenon.

RADIOACTIVE DECAY
(a) The decay of radioactive substance means disintegration of nuclei of the substance by emission of different
radiations.
(b) Despite the strength of the forces that hold nucleons (protons and neutrons) together to form an atomic
nucleus, many nuclides are unstable and spontaneously change into other nuclides by radioactive decay.
(c) The energy liberated during radioactive decay comes from within individual nuclei without external excitation,
unlike the case of atomic radiation.
(d) It is statistical process that obeys the laws of chance.
(e) The decay of nucleus takes place to achieve the stable end products.

Kinds of Decay
There are five kinds of radioactive decays.
When radioactivity was discovered, only three kinds of radioactive decays alpha(), beta() and gamma)()
were known. Which were eventually identified as 42 He nucleus, electron and high energy photon respectively..
Later two more kinds of radioactive decays namely positron emission and electron capture were added.

Alpha decay
(a) In this type of decay, the unstable nucleus emits an alpha particle.
(b) It reduces the proton number Z of the nucleus by 2.
(c) It reduces the neutron number N of the nucleus by 2.

www.physicsashok.in 7
NUCLEAR PHYSICS
(d) It reduces the mass number (i.e. Z + N) of the nucleus by 4.
(e) It changes the element itself hence the chemical symbol of the residual nucleus is different from that of the
original nucleus.
(f) The alpha decay process may be represented as
A A 4
Z X Z 2 Y 42 He
(g) The nucleus before the decay is called the parent nucleus and that obtained after the decay is called the
daughter nucleus.
Ex: Let us consider the example given below:
212 208
83 Bi 81 Tl 42 He
In the above example of -decay, the parent nucleus is bismuth (Bi) and the daughter nucleus is thallium
(Tl).
(h) Alpha decay occurs in all nuclei with mass A > 210.
(i) In this decay, the nucleus decreases its mass number to move towards stability.
(j) On emission of -particle, the binding energy per nucleon increases and the residual nucleus tends towards
stability.

Q-Value for -decay:


If -decay process is given by
A A 4
Z X Z 2 Y + 42 He , then

Q-value = m Z X m Z 2 Y m 2 He c2
A A4 4

NOTE: The quantity m AZ X represent atomic mass of the particle X.

Example 55. A radon nucleus Rn86222, of mass 3.6 1025 kg, undergoes -decay. The -particle has mass
6.7 1027 kg and energy 8.8 1031 J.
(a) What is the resulting nucleus? (b) Find the velocity of recoil of the nucleus.
Sol. (a) The atomic number will be reduced by 2 and the mass number by 4.
A = 222 4 = 218 and Z = 86 2 = 84
218
The resulting nucleus is 84Po .
(b) mass of resulting nucleus = m1 = 3.6 1025 6.7 1027
= 3.533 1025 kg
Let v1 = its velocity of recoil
Mass of -particle = m2 = 6.7 1027 kg
1/ 2
2E
Velocity of -particle = v2 = with E = 8.8 1013 J
m 2
Now m1v1 = m2v2
(2m 2 E)1/ 2 (2 6.7 1027 8.8 1013 )1/ 2
v1 , v1 = 3.1 10+5 ms1
m1 3.533 1025

www.physicsashok.in 8
NUCLEAR PHYSICS
Beta Decay
(a) Beta decay is a process in which a neutron is converted into a proton.

n p + e +
(b) It increases the atomic number (Z) of nucleus by 1.
(c) It does not alter the mass number (A).
(d) If a nucleus is formed with more number of neutrons than needed for stability, a neutron will convert itself
into a proton to move towards stability.
(e) When a neutron is converted into a proton, an electron and a new particle named antineutrino are created
and emitted from the nucleus.

(f) The antineutrino is denoted by the symbol . It is supposed to have zero rest mass like photon. It is charge-
less and has quantum number .
(g) The electron emitted from the nucleus is called a beta particle and is denoted by the symbol or 1e0.
(h) A stream of beta particles coming from bulk of unstable nuclei is called beta ray.
(i) It is also called beta minus decay as negatively charged beta particles are emitted.
(j) The beta decay process may be represented by
A A
Z A Z 1 Y + e + ( n p + e + )
A A
Z X Z 1 Y + +

(k) An example of beta decay is 6C14 7


N14 + e + (antineutrino)

Q-value for -decay:


(a) decay: If -decay process is given by AZ X A
Z 1 Y + + , then

Initial rest mass energy, REi = m Z X Zm e c2


A

Final rest mass energy, REf = m ZA1 Y Zm e (Z 1) m e c2

Q = REi REf = m Z X m Z1 Y c2
A A

Because of the large mass, the residual nucleus Z A1 Y does not share appreciable kinetic energy. Thus, the
energy Q is shared by the antineutrino and the beta particle. Depending on the fraction taken away by the
antineutrino, the kinetic energy of the beta particle can be anything between zero and a maximum value Q.

Positron emission ( + decay)


(a) + decay is a process in which a proton is converted into a neutron with emission of positron (e+) and
neutrino (v).
p n + e+ + v
(b) It reduces the atomic number (Z) of nucleus by 1.
(c) It does not alter the mass number (A).
(d) An isolated proton does not beta decay to a neutron. On the other-hand, an isolated neutron decays to a
proton.

www.physicsashok.in 9
NUCLEAR PHYSICS
(e) If the unstable nucleus has excess protons than needed for stability, a proton converts itself into a neutron.
(f) When a proton is converted into a neutron, a positron and a neutrino are created and emitted from the
nucleus.
(g) The neutrino is denoted by the symbol v. It is charge-less particle.
(h) The positron (e+) has a positive electric charge equal in magnitude to the charge on an electrons and has a
mass equal to the mass of an electron.
(i) Positron is called the antiparticle of electron.
(j) When an electron and a positron collide, both the particles are destroyed and energy is made available.
(k) Neutrino and antineutrino are antiparticles of each other.
(l) The + decay process is represented as
A A
Z X Z 1 Y + e+ + v [ p n + e+ + v]
A A +
Z X Z 1 Y + + v

If the unstable nucleus has excess protons than needed for stability, a proton converts itself into a neutron.
In the process, a positron and a neutrino are created and emitted from the nucleus,
p n + e+ + v .....(iv)
+
The positron e has a positive electric charge equal in magnitude to the charge on an electron and has a mass
equal to the mass of an electron. Positron is called the antiparticle of electron. When an electron and a
positron collide, both the particles are destroyed and energy is made available. Similarly, neutrino and
antineutrino are antiparticles of each other. When a proton in a nucleus converts itself into a neutron, the
decay process is represented as
A A
Z X Z 1 Y + e+ + v
A A +
or ZX Z 1 Y + + v .....(v)
This process is called beta plus decay. The positron so emitted is called a beta plus particle.

+ - decay or position - emission :


If the +-decay or position-emission is given by
A A
Z X Z 1 Y + + + v, then

R.Ei = m Z X Zm e c2
A

R.Ef = m Z1 Y (Z 1)m e m e c2
A

= R.Ei R.Ef = m Z X m Z1 Y 2me c2


A A
Q

Q = m Z X m Z1 Y 2me c2
A A
.....(vi)
Can an isolated proton decay to a neutron emitting a positron and a neutrino as suggested by equation (iv)?
The mass of a neutron is larger than the mass of a proton and hence the Q-value of such a process would be
negative. So, an isolated proton does not beta decay to a neutron. On the other hand, an isolated neutron
decays to a proton as suggested by equation (i).
(m) The positron is also called beta plus particle.
(n) An example of + decay is 29
Cu64 28
Ni64 + e+ + v

www.physicsashok.in 10
NUCLEAR PHYSICS
A similar process, known as electron capture, takes place in certain nuclides. In this process, the nucleus
captures one of the atomic electrons (most like an electron from the K shell). A proton in the nucleus
combines with this electron and coverts itself into a neutron. A neutrino is created in the process and emitted
from the nucleus.

Electron capture
(a) When the nucleus has too many protons relative to the number of neutrons, the nucleus captures one of the
atomic electrons (most likely an electron from the K-shell).
(b) A proton in the nucleus combines with this electron and converts itself into a neutron.
(c) A neutrino is created in this process and emitted from the nucleus.
p + e n+v
(d) In this process, atomic number (Z) of the nucleus decreases by 1.
(e) This process does not alter the value of mass number (A).
(f) When an atomic electron is captured, a vacancy is created in the atomic shell and X-rays are emitted
following the capture.
(g) This process is also called K-capture.
(h) The process may be represented as
A A
Z X + e Z 1 Y +v [ p + e n + v]
(i) An example of electron capture is
29
Cu64 + e 28
Ni64 + neutrino

Q-Value of K-capture process:


If K -capture process is given by
A A
Z X + e Z 1 Y + v, then
Q = m Z X m Z1 Y c2
A A

Example 56: Calculate the Q-value in the following decays-


(a) 19O 19
F+e+ [ -decay]
(b) 25Al 25
Mg + e+ + v, [+-decay]
The atomic masses of 19O = 19.003576 a.m.u. 19F = 18.998403 a.m.u, 25Al = 24.990432 a.m.u., 25 M g
= 24.985939 a.m.u.
Sol: (a) The Q-value of -decay is
Q = [m (19O) m(19F)]c2
= [19.003576 a.m.u. 18.998403 a.m.u.] (931.5 MeV/a.m.u.)
= 4.819 MeV
(b) The Q-value of + -decay is
Q = [(mass of 25Al nucleus) (mass of 25Mg nucleus) (mass of positron)]c2
= [(24.990432 a.m.u 13me) (24.985939 a.m.u. 12 me) me]c2
= [(0.004593 a.m.u.) 2me]c2
MeV
= 0.004593 a.m.u. (931.5 MeV/a.m.u.) 2 0.511 2 .c2
c
= 4.276 MeV 1.022 MeV = 3.254 MeV
www.physicsashok.in 11
NUCLEAR PHYSICS
Gamma Decay
(a) Nucleus has also energy levels like atoms have.
(b) This decay process is related to the transitions between two nuclear energy levels.
(c) The protons and neutrons inside a nucleus move in discrete quantum states with definite energies.
(d) In the ground state, the nucleons occupy those quantum states which minimise the total energy of the
nucleus.
(e) The higher energy states are also available to the nucleons and if appropriate energy is supplied, the nucleus
may be excited to higher energies.
(f) The energy differences in the allowed energy levels of a nucleus are generally large (in the order of MeV).
(g) It is difficult to excite the nucleus to higher energy levels by usual methods of supplying energy like heating
etc.
(h) When an alpha or a beta decay takes place, the daughter nucleus is generally formed in one of its excited
states. The daughter nucleus in an excited state eventually comes to its ground state by emitting one photon
or more than one photon of electromagnetic radiation.
(i) The process of a nucleus coming down to a lower energy level by emitting a photon is called gamma decay.
(j) In this decay, atomic number (Z) as well as mass number (A) of the nucleus remain constant.
(k) In this decay, the quantum states of the nucleons vary.
(l) The electromagnetic radiation emitted in nuclear transitions is called gamma ray.
(m) The wavelength of this radiation is given by the common relation.
hc
= where E = energy of the photon.
E
(n) An example of gamma decay is shown in figure below:
57
Co +

136 keV 2nd excited state


14 keV 1st excited state



0 keV 57
Ground state
Fe
When 57Co is taken in bulk, we can observe a stream of + particles, 136 keV photons, 122 keV photons
and 14 keV photons coming from the 57Co source.

NOTE: The , and rays are collectively called nuclear radiation.

Comparison among the kinds of decay


(a) The velocity of -particles is relatively low:
v = (c/30 c/15), when c is the velocity of light
Mass m = 4 amu
charge q = +2e
For -particles: 0 v < c
m = mass of an electron
q = e
www.physicsashok.in 12
NUCLEAR PHYSICS

For -rays:
Since -rays are electromagnetic waves, hence they propagate with the speed of light.
i.e. v = c
Rest mass,m = 0(like photon)
q = 0 (like photon)

(b) The penetrability of -rays is 0 - 100 times higher than the penetrability of b-rays and 1000 - 10000 times
higher than the penetrability of -rays. The penetrability of -rays also exceeds the penetrability of x-rays.
Al Lead
Carboard

- particles from radioactive materials are stopped by a piece of cardboard.


-particles penetrate the cardboard but are stopped by a sheet of aluminium.
Even a thick slab of lead may not stop all the -rays.
-rays
(c) In a magnetic field, a beam of , and -rays splits into
three parts. In a magnetic field, -rays are undeviated and
-particles are most deviated -particles -particles

Magnetic field
(d) Table for various decay directed into paper

Decay Transformation Example


238 234
-decay A
Z X A4
Z 2 Y 42 He 92 U 90 Th 42 He
A A 14 14
-decay Z X Z1 Y + e + 6 C 7 N + e +
A A 64 64
Positron emission Z X Z 1 Y + e+ + v 29 Cu 28 Ni + e+ + v
A A 64 64
Electron capture Z X + e Z 1 Y +v 29 Cu + e 28 Ni + v
A
Gamma decay Z X* A
Z X + 87
38 Sr * 87
38 Sr +

The *denotes an excited nuclear estate, -denotes a gamma-ray photon and v and denotes neutrino and
antineutrino particles respectively.

C52: An a-particle is bombarded on 14N. As a result, a 17O nucleus is formed and a particle is emitted. This
particle is a/an
(a) neutron (b) proton (c) electron (d) positron

www.physicsashok.in 13
NUCLEAR PHYSICS
14
Sol: 7 N 42 He 17
8 O 11p
1
1p is equivalent to 11H .

C53: In a radioactive decay, neither the atomic number nor the mass number changes. Which of the following
particles is emitted in the decay ?
(a) Proton (b) Neutron (c) - particle (d) photon
Sol: Photon is equivalent to -radiation. In this decay, only quantum states of the nucleons vary.

C54: -rays emitted by a radioactive material are


(a) electromagnetic waves (b) electrons orbiting around the nucleus
(c) charged particles emitted by the nucleus (d) neutral particles
Sol: A neutron in the nucleus decays emitting an electron.

C55: Give an equation representing the decay of a free neutron.


1 1
Sol: 0 n 1 H 01e v

C56: How many electrons, protons and neutrons are there in a nucleus of atomic number 11 and mass number
24 ?
Sol: Number of electrons or protons, Z = 11
and number of neutrons, N = mass number Atomic number
= 24 11 = 12

C57: Fill up the blanks


(i) hv e + ..............
(-photon) (electron)
(ii) 90
Th234 B01
+ 91Pa234 + .............
(Thorium) (-particle) (Protactinium)
(iii) U238
92
He4 + ................
2
(Uranium) (-particle)
Sol: (i) e+(Positron) (ii) (Antineutrino) (iii) 90Th234(Thorium)
27
C58: Following the origin of gamma decay, calculate the value of 12Mg

wavelength of resulting photon when the nucleus of 27
13 Al

reaches the ground state (Eg = 0) from the state in which Eex. = 1.015 MeV
1.015 MeV. The related figure is shown adjacent.
12400 12400 27
0 MeV
Sol: (in ) = = 13Al
E(eV) (1.015 0) 106
= 0.012217

LAW OF RADIOACTIVE DECAY


When the radioactive substance is only disintegrating:
Radioactive decay is a random process. Each decay is an independent event, and one cannot tell when a
particular nucleus will decay. When a given nucleus decays, it is transformed into another nuclide which may

www.physicsashok.in 14
NUCLEAR PHYSICS

or may not be radioactive. When there is a very large number of nuclei in a sample, the rate of decay
dN dN dN
i.e. is proportional to the number of nuclei, N, that are present i.e. N = N
dt dt dt
where is called the decay constant.
N t
dN dN
This equation may be expressed in the form
dt
= dt and integrated to get N
N0
= 0 dt , to yield, ln
N
N 0 = t, where N0 is the initial number of parent nuclei at t = 0. The number that survive at time t is
therefore
N = N0et

Definition of decay constant :


The probability of decay per second for a particular process for a sample is called the decay constant for
that process for that sample.
dN
Radioactivity law of decay gives, =
Ndt
Half life :
The time period for the number of parent nuclei to fall to 50% is called the half-life, T, and may be related
to .
if we put N = N0/2 at t = T, the exponential decay equation gives,
0.5N0 = N0eT
0.693
T = ln|2| = 0.693 , T =

(a)It takes one half-life to drop to 50% of any starting value.
(b) The half-life for the decay of the free neutron is 12.8 min.
(c) Other half lives range from about 1020s to 1016 years.

Mean life(Tm ) :
Mean life of a radioactive sample is defined as the average of the lives of all nuclei.
0

tdN 1 T
Tm = N0 = =
N0 0.693
1
Tm = and T = 0.693 Tm

NOTE : (i) Radioactive decay equation, N = N0et, can also be written as


t/T
1
N = N0 , where T is half-life of the sample.
2
2.303 N0
(ii) The decay constant is also given as, = log
t N

www.physicsashok.in 15
NUCLEAR PHYSICS
Activity of radioactive substance:
Since the number of atoms is not directly measurable, we measure the decay rate or activity (A)
dN
A= , but N = N0et
dt
dN
A= = (N0et) = N
dt
A = N = A0et, where A0 = N0 is the initial activity.
Both N and A decrease exponentially with time. The activity is characterized by the same half-life.

Units of activity:
(a) The SI unit for the activity is the becquerel (Bq), but the curie (Ci) is often used in practice.
1 becquerel (Bq) = 1 disintegrations per second (dps)
1 curie (Ci) = 3.7 1010 dps
1 rutherford = 106 dps
(b) Rate of decay of 1 gm substance is called specific activity.
(c) Activity of 1 gm Ra226 is 1Ci.

Example 57: The half-life of 198Au is 2.7 days. Calculate (a) the decay constant, (b) the average life and (c) the
activity of 1.00 mg of 198Au. Take atomic weight of 198Au to be 198 g/mol.
0.693
Sol: (a) The half-life and the decay constant are related as T =

0.693 0.693 0.693
= = 2.7 days = 2.7 24 3600 s
T
= 2.9 106 s1.
T
(b) Tm =
0.693
2.7
= days = 3.9 days.
0.693
(c) Activity is given as, A = N, where N is the number of nuclei present in 1 mg of 198Au.
Atomic mass of 198Au = 198 g
1 mg
N = 198 g Avogadro no

103
N = 6 1023 atoms
198
N = 3.03 1018 atoms
Thus, A = N = (2.9 109 s1) 3.03 1018 (atoms)
= 8.8 1012 disintegrations per sec.
8.8 1012
= Ci = 240 Ci
3.7 1010

www.physicsashok.in 16
NUCLEAR PHYSICS

Example 58. Two radioactive material A1 and A2 have decay constants of 10 0 and 0. If initially they have
same number of nuclei, the ratio of number of their undecayed nuclei will be (1/e) after a time
1 1 1
(A) (B) 9 (C) 10 (D) 1
0 0 0

Sol. N1 N 0 e 1t N 0 e100t

N 2 N 0e 2t N 0 e 0 t

N1 N 0 e 10 0 t

N2 N 0e 0 t
N1
e 9 0 t
N2
1
e 9 0 t
e
e1 e9 0 t
1 = 90t
1
t Hence (B) is correct.
9 0

C59. A certain element has a density of 10 g cm3 and half-life of 140 days. Over a period of 140 days, the
average number of -emissions per day is found to be 12 1012, from a sample of initial mass 1 g.
Estimate the number of atoms in 1 cm3 of the element.
Sol. We assume that only one emission takes place per atom.
In 140 days, no. of emissions = 140 12 1012
initial no. of atoms present = 2 140 12 1012
(since 140 days is the half-life)
no. of atoms in 1 g = 28 12 1013
no. of atoms in 1 cm3, i.e., 10g = 3.36 105 107
= 3.36 1022

C60: A radioactive sample has 3.2 1016 active nuclei at certain instant. How many of these nuclei will still be in
the same active state after four half-lives ?
Sol: In one half-life the number of active nuclei reduces to half the original number. Thus in four half-lives the
1111 1
number is reduced to i.e. th of the original number..
2222 16
The number of remaining active nuclei is,
1
= 3.2 1016 = 2 1015 nuclei
16

www.physicsashok.in 17
NUCLEAR PHYSICS
C61: The activity of a radioactive sample falls from 1200 s1 to 1000 s1 in 60 minutes. Calculate its half life.
Sol: We have, A = A0et
(1000 s1) = (1200 s1) et 5/6 = et
ln(6 / 5) ln(6 / 5)
t = ln(6/5) = = 60 min
t
ln 2 ln 2 ln(6 / 5)
but, T= = 60 min
T
ln 2 0.693
T= 60 min = 60 min
ln(6 / 5) 0.182
half-life = 228 min

C62. At a given instant there are 25% undecayed radioactive nuclei in a sample. After 10 second the mumber of
undecayed nuclei reduces to 12.5% Calculate :
(i) mean life of the nuclei
(ii) the time in which the number of undecayed nuclei will further reduce to 6.25% of the reduced number.
Sol. (i) In 10 second, number of nuclei has been reduced to half (25% to 12.5%). Therefore, its half life is
t1/2 = 10 s
Relation between half life and mean life is
t 10
t mean 1/ 2 s
ln (2) 0.693
tmean = 14.43 s
(ii) From initial 100% to reduction till 6.25%, it takes four half lives.
100% t1/ 2
50%
t1/ 2
25% t1/ 2
12.5% t1/ 2
6.25%
t = 4 t1/2 = 4(10) s = 40 s

Example 59. A sample initially contains 1020 radioactive atoms of half-life 130 days. Calculate the activity of the
sample after 260 days have elapsed. Also find the total energy released during this period if the energy
released per disintegration is 8 1013 J.
In 260 days, i.e., two half-lives, the number of undisintegrated atoms will reduce to 1/4th.
1
Sol. no. of atoms present after 260 days = N = 1020
4
Also, disintegration constant = = 0.6931/T
where T = 130 days = 130 86400 s
0.693 1
Now, activity = N = 1020 = 1.54 1012 s
130 86400 4
Number of atoms present initially = 1020
1
Number of atoms present after 260 days = 1020
4
number of disintegrated atoms = 3/4 1020
Energy per disintegration = 8 1013 J
3
total energy released = 1020 8 1013 = 6 107 J
4
www.physicsashok.in 18
NUCLEAR PHYSICS

Example 60. A body of mass m0 is placed on a smooth horizontal surface. The mass of the body is decreasing
exponentially with disintegration constant . Assuming that the mass is ejected backwards with a relative
velocity u. Initially the body was at rest. Find the velocity of it after time t.
Sol. Mass of the body left after time t is
m = m0et
dm
So = m0et
dt
and thrust force on the body is
dm
Ft = ur (in forward direction)
dt

dv
or m = u(m0et) (ur = u)
dt
dv
or (m0et) = m0 uet or dv = udt
dt
v t
or 0
dv u dt
0
or v = ut

Example 61. A radio nuclide with half life T = 69.31 second emits -particles of average kinetic energy E =
11.25 eV. At an instant concentration of -particles at distance, r = 2 m from nuclide is n = 3 1013 per m3.
(i) Calculate number of nuclie in the nuclide at that instant.
(ii) If a small circular plate is placed at distance r from nuclide such that -particles strike the plate normally and
come to rest, calculate pressure experienced by the plate due to collision of -particles.
[Mass of -particle = 9 1031 kg)]
Sol. Let activity (rate of decay) of the nuclide be A nuclie per second. It means
A -particles are emitted per second. If a spherical surface of radius r with
centre at position of nuclide be considered then A -particles cross this
surface
per second. It means during an elemental time interval dt a number (A . dt) (v dt)
r
of -particles cross this surface. If velocity of -particles be v then above
calculated (A . dt) -particles are in a space having shape of a spherical
shell of radius r and radial thickness (v dt) as shown in figure.
Volume of this space = 4r2 (v dt)
Concentration of -particles at distance r from nuclide is
A dt
n
4r 2 (v dt)
or activity of the nuclide, A = 4r2 vn
But activity, A = N where N is number of nuclei
4r 2 vn log 2
Hence, N= but decay constant =
T
4r 2 vnT
N ...(1)
0.6931

www.physicsashok.in 19
NUCLEAR PHYSICS

1 2E
Kinetic energy of -particles, E mv 2 , v
2 m
substituting this value in equation (1),
4r 2 nT 2E
N 9.6 1022
0.6931 m
(ii) At distance r from the nuclide (A/4r2) -particles cross unit area per second. Let area of small circular
plate be S, then
number of -particles striking the plate per second
A N 0.6931 NS
2
S 2
S
4r 4r 4 r 2 T
Momentum of each particle just before collision is mv and after collision particles come to rest or momentum
becomes zero.
Momentum transferred to plate due to collision is
p = mv 0 = mv
Due to transfer of momentum, the plate experiences a force which is equal to rate of transfer of momentum.
Force, F = p no. of particles striking per second
0.6931 NS
or F = mv
4r 2 T
Pressure, P = Force per unit area
F 0.6931 N 2E
P= = mv but v =
S 4r 2 T m
0.6931 N
P= 2mE = 1.08 104 Nm2
4r 2 T

RADIOACTIVE DATING OR CARBON DATING


Radiocarbon dating, also called carbon dating, is used to estimate the age of organic samples. The technique
is based on the -activity of the radioactive isotope C14;
14
6 C 14
7 N + + beta particles, anti neutrino]
(a) High energy particles for outer space, called cosmic rays, induce nuclear reactions in the upper atmosphere
and create carbon - 14.

(b) The carbon dioxide molecules of the earths atmosphere have a constant ratio ( ~ 1.4 1012) of C14 to C12
isotopes.
(c) All living organisms also show the same ratio as they continuously exchange CO2 with their surroundings.
(d) However after its death, an organism can no longer absorb CO2 and the ratio C14/C12 decreases due to the
-decay of C14.
(e) Thus by measuring the -activity per unit mass, it is possible to estimate the age of a material.
(f) Half life of 14C is 5739 y.

www.physicsashok.in 20
NUCLEAR PHYSICS

Example 62: When charcoal is prepared from a living tree it shows a disintegration rate of 15.3 disintegrations
of 14C per gram per minute. A sample from an ancient piece of charcoal shows 14C activity to be 12.3
disintegrations per gram per minute. How old is this sample ? Half life of 14C is 5730 y.
Sol: 14C-activity of a living tree,A0 = 15.3 dis. per min.
14
C-activity of the old sample, A = 12.3 dis. per min.
Suppose, the sample is t year old, then
(14C-activity of sample) = (14C-activity of living tree )et
i.e. A = A0et .....(i) where is decay constant of 14C-activity
Putting values of A and A0 in equation (i), we get
15.3
12.3 = 15.3 et t = ln t = ln(1.24) = 0.218
12.3
0.218 0.218 0.693
t= = T,, [as, = ]
0.693 T
0.218
t= 5730 y,, [T = half-life of 14C -activity = 5730 y]
0.693
t = 1805 y
Thus, the sample is 1805 y old.

Example 63: The relative radiocarbon activity in a piece of charcoal from the remains of an ancient camp fire is
0.18 that of a contemporary specimen. How long ago did the fire occur ? Half-life of 14C-activity is 5730 y.
Sol: Here, ratio between the 14C-activity of burnt charcoal and that of a living tree is given.
14
C activity of charcoal (say, A) A
14 = 0.18 A = 0.18
C activity of a living tree (say, A 0 ) 0

Suppose, t year ago fire occurred and let be the decay constant of 14C-activity. We have, using radioactive
decay law,
A0
A = A0et et =
A
A0 1 A0
t = ln t= ln
A A
T A0 0.693
t = ln , [ = ]
0.693 A T
5730 1
= ln
0.693 0.18
= 8268.3 ln|5.56|
= 1.4 104 y
Thus, fire occurred 1.4 104 years ago.

Radioactivity law for different types of disintegration of the radioactive substance


It is seen that radioactive disintegration of a radioactive substance is not only ruled by the radioactive law,
N = N0et (which is applicable only when the radioactive substance is only disintegrating), but the radioactive
law changes for various types of disintegration of the substance. Let us first enlist these types of disintegration
as:
www.physicsashok.in 21
NUCLEAR PHYSICS
(1) Radioactive substance only disintegrates:
dN
For this radioactive law is = N or N = N0et
dt
(2) Disintegration with continuous production of the radioactive substance.
It deals with the case when production and the decay of the radioactive substance are taking place
simultaneously.
Formation A decays B
( )

(3) Successive disintegrations of the products:


It deals with the case when a substance A decays into a substance B and B successively decays into a third
substance C with the same or different decay rates.
decays
A B decays C
( 1) ( 2)

(4) Simultaneous disintegrations of parent nuclei:


It deals with the case when a parent nucleus may disintegrate in a number of ways into different products.
1 B
A Parent nucleus A may decay in B or C with decay constants 1 and 2 respectively..
2 C

(5) Radioactive equilibrium:


In a radioactive series, after a period of time, successive daughter nuclei decay at the same rate as it is
formed. This situation is called radioactive equilibrium.
(6) Disintegration of isotopes:
A preparation may have a number of radioactive isotopes. Here we will deal with the net rate of disintegration
of the preparation.
Let us discuss these types of disintegrations.

Radioactive substance only disintegrates :


Suppose disintegration of A into B is taking place with decay constant , then decay rate,
dN
= N N = N0et
dt
Disintegration with continuous production:
Suppose a substance A decays into B with decay constant and simultaneously the production of A is
taking place at a constant rate q.
Formation decays
A B
(q) ( )
Let, N is the number of nuclei of A present at time t.
q = constant rate of formation of A.
Disintegration rate will be given by
dN
= q + N, where N is the rate of decay of A.
dt
Rearranging, we get,

www.physicsashok.in 22
NUCLEAR PHYSICS

dN
= dt,
q N
N t
dN
Integration gives, q N dt , where N0 is the no. of nuclei of A initially present.
N0
=
0

1
Finally, N = [q + (N0 q)et]

Example 64. A radionuclide with disintegration constant is produced in a reactor at a constant rate nuclei
per second. During each decay energy E0 is released. 20% of this energy is utilised in increasing the temperature
of water. find the increase in temperature of m mass of water in time t. Specific heat of water is s. Assume
that there is no loss of energy through water surface.
Sol. Let N be the number of nuclei at any time t. Then

N Rate of decay = N

Rate of formation =
net rate of formation of nuclei at time t is
dN N dN t

dt
= N or 0
dt
N 0

or N= (1 et)

Number of nuclei formed in time t = t

and number of nuclei left after time t


1 e t

Therefore, number of nuclei disintegrated in time t t


1 e t
t
energy released till time t E 0 t 1 e .



But only 20% of it is used in raising the temperature of water.

So


0.2 E 0 t 1 e t Q

where Q = ms

Q
= increase in temperature of water =
ms



0.2 E0 t 1 e t



ms

Example 65. Nuclei of a radioactive element A are being produced at a constant rate . The element has a
decay constant . At time t = 0 , there are N0 nuclei of the element.
(a) Calculate the number N of nuclei of A at time t
(b) If = 2 N0, calculate the number of nuclei of A after one half life of A, and also the limiting value of N
as t .

www.physicsashok.in 23
NUCLEAR PHYSICS
Sol. (a) Let at time t, number of radioactive nuclei are N. A Rate of decay = N
Net rate of formation of nuclei of A. Rate of formation =

dN dN
= N or = dt
dt N
NdN t t=t
or N0 N
dt
0
N=N

Solving this equation, we get


1
N N 0 e t ...(1)

ln (2)
(b) (i) Substituting = 2N0 and t = t1/2 = in equation (1), we get

3
N N0
2
(ii) Substituting = 2N0 and t in equation (1), we get

N= = 2N0 or N = 2N0

Example 66. A radionuclide with half life T is produced in a reactor at a constant rate q nuclei per second.
During each decay, energy E0 is released. If production of radionuclide is started at t = 0, calculate.
(i) rate of release of energy as function of time t and
(ii) total energy released upto time t.
Sol. To calculate rate of release of energy at time t and total energy released upto time t, rate of decay at that
instant and total number of decays upto that instant must be known.
Since, nuclei produced are radioactive, therefore, their decay starts as soon as their production is started.
Let at some instant number of nuclei in the radionuclide be N. Then rate of its decay = N where is decay
log e 2
constant which is equal to .
T
Since, rate of production is q nuclei per second, therefore, at instant t, net rate of increase of nuclei
dN N log e 2
q N q
dt T
dN qT Nloge 2
or
dt T
dN dt
...(1)
qT N log e 2 T
Integrating above equation with limits at t = 0, N = 0 and at t, N = ?
N dN t dt
0

qT N log e 2 0 T
t loge 2
qT
T

N 1 e
log e 2

www.physicsashok.in 24
NUCLEAR PHYSICS

t log e 2

Hence, rate of decay, A N q 1 e T

Since, energy E0 releases during each decay, therefore, rate of release of energy at time
t log e 2

t AE 0 qE 0 1 e T

Total number of nuclei produced upto time t = q . t
But the number of nuclei remaining undecayed at that instant is N. Therefore, total number of nuclei which
decayed upto time t = (qt N)
Hence, total energy released upto this time = (qt N)E0
t log e 2
qTE 0
T

qtE 0 1 e
log e 2

Successive disintegration :
Suppose a radioactive substance A decays into B with decay constant 1 and B successively decays into
another stable product C with a decay constant 2.
decays decays
A B C (stable product)
1 2
Let N0 be the number of nuclei of A present at t = 0, N1, N2, N3 be the number of nuclei of A, B and C
respectively at any instant t.
Decay rate of A is given by
dN
= 1N1 N1 = N0e1t .....(i)
dt
Rate of change of no of nuclei of B is
dN 2
= (Rate of decay of A) (Rate of decay of B)
dt
But,
Rate of decay of A = 1N1
dN 2
and rate of decay of B = 2N2 hence, = 1N1 2N2 .....(ii)
dt
dN 3
Rate of change of number of nuclei of C is = (rate of decay of B) = 2N2
dt
dN 3
= 2N2 .....(iii)
dt
Solving (i), (ii) and (iii), we get
N 1 = N0e1t
N 0 1
N 2 = [e1t e2t]
2 1

1e2 t 2e 1t
N 3 = N0 1
2 1
www.physicsashok.in 25
NUCLEAR PHYSICS

NOTE : In this case total number of nuclei remains constant,


hence, N1 + N2 + N3 = N0 at any time.

Example 67: A radioactive nucleus A decays to a nucleus B with a decay constant 1. B further decays to a
stable nucleus C with a decay constant 2. Initially there are only A nuclei and their number is N0. Set up the
rate equations for the populations of A, B and C. The population of B nucleus as function of time is given by
N 01 1t 2 t
N2(t) = ( ) (e e ) . Calculate the population of C as a function of time t.
2 1

Sol: Let N1, N2 and N3 be the number of A, B and C nuclei respectively present at a time t.
Decay rate for A nuclei will be
dN1
= 1N1 N1 = N0e1t
dt
Rate of change of the number of nuclei of B is
dN 2
= 1N1 2N2 where 2N2 = decay rate of B and 1N1 = decay rate of A
dt
dN 3
Rate of change of the number of nuclei of C is = (Rate of decay of B) = 2N2
dt
N3 t
dN 3
dt
= 2N2 dN3 = 2 N 2dt , after integrating.
0 0
t
N 01 1t 2 t
N3 = 2 N 2 dt but, N2 is given as, N2 = (e e )
0 ( 2 1 )
t t
N 0 1 2 1t 2 t N 0 1 2 e1t e2 t
(e e ) dt
2 1 0
N3 = =
2 1 1 2 0

N 0 1 2 e 1t e 2 t 1 1
=
2 1 1 2 1 2

N0
N3 = [1e2 t 2 e 1t ] + N0
2 1

1e2 t 2e 1t
N3 = N0 1
2 1

Simultaneous disintegration:
A radioactive nucleus can decay by two different processes. For example a nucleus A may either a-decay
to a nucleus B or -decay to nucleus C.
1 B
A Let 1 and 2 be the decay constants for these two decay processes.
2 C

www.physicsashok.in 26
NUCLEAR PHYSICS
The probability that an active nucleus decays by the first process in a time interval dt is 1dt . As decay
constant, is defined as the probability of decay per second for a particular process for a sample. Similarly,
the probability that it decays by the second process is 2dt.
Hence, the probability the it either decays by the first process or by the second process is
1dt + 2dt.
If the effective decay constant is eff, this probability is also equal to effdt.
Thus, effdt = 1dt + 2dt
eff = 1 + 2
For a number of different process for decay, eff = 1 + 2 + ............

Example 68: A radioactive nucleus can decay by two different processes. The half life for the first process is t1
and that for the second process is t2. Show that the effective half-life t of the nucleus is given
1 1 1
= + .
t t1 t2
ln 2
Sol: The decay constant for the first process is 1 =
t1
ln 2
For the second process, 2 = t
2

Probability of decay by the first process = 1dt


Probability of decay by the second process = 2dt
Probability that it either decay by the first process or the second process = 1dt + 2dt
This probability also equals to effdt, where eff is the effective decay constant.
Thus, effdt = 1dt + 2dt
ln 2 ln 2 ln 2
eff = 1 + 2 = +
t t1 t2
1 1 1
= + Proved.
t t1 t2

Example 69. A number N0 of atoms of a radio active element are placed inside a closed volume. The radiactive
decay constant for the nucleus of this element is 1. The daughter nucleus that form as a result of the decay
process are assumed to be radioactive too with a radioactive decay constant 2. Determine the time variation
of the number of such nucleus. Consider two limiting cases 1 >> 2 and 1 << 2.
Sol. In time interval dt, number of increase of daughter nuclei are
dN2 = 1N1dt 2N2dt
or dN2 = 1N0 e 1t dt 2N2dt (N1 = N0 e 1t )
dN 2
or + 2N2 = 2N0 e 1t ...(1)
dt
Case-1 : When 1 > >2 i.e. (t1/2)1 < < (t1/2)2 (t1/2 = half life)
We can assume that N20 N0 so that N2 = N0 e 2 t
(N20 = number of daughter atoms at time t = 0)

www.physicsashok.in 27
NUCLEAR PHYSICS
Physically this means that parent nuclei practivally instantly transform into daughter nuclei, which then decay
according to the law of radioactive decay with decay constant 2.
Case-2 : When 1 < <2 i.e. (t1/2)1 > > (t1/2)2
In this case number of parent nuclei can be assumed to remain constant over a sizable time interval and is
equal to N0.
dN 2
This transforms equation (1) into = (2N2 1N0)
dt
N2 t
dN 2
or 1N0 2 N 2 dt

0 0
Which after integration gives
1
N2 = N0(1 e 2 t )
2

Example 70. A radio nuclide consists of two isotopes. One of the isotopes decays by -emission and the other
by -emission with half lives T1 = 405 second and T2 = 1620 second, respectively. At t = 0, probabilities
of getting and particles from the radionuclide are equal. Calculate their respective probabilities at t =
1620 second.
If at t = 0, total number of nuclie in the ratio-nuclide are N0, calculate time t when total number of nuclie
N0
remained undecayed becomes equal to .
2
Given log10 2 = 0.30103, log10 13 = 1.11394
Sol. Since, at t = 0, probabilities of getting and particles from the radionuclide are equal, therefore, initial
activities of two isotopes are equal. Let it be A0.
Activity of first isotope at t = 1620 sec.
t / T1
1 A0
A1 A 0
2 16
t / T2
1 A0
That of second isotope, A2 A0
2 2
9
Total activity of radionuclide at t = 1620 sec, A = A1 + A2 = A
16 0
A1 1
Probability of getting -particle, P1
A 9
A2 8
and that of getting -particle, P2
A 9
Let at t = 0, number of nuclei of two isotopes be N01 and N02, respectively.
log 2
Initial activity of first isotope, A1 N 01 1 N 01
T1
log 2
That of second isotope, A 2 N 02 2 N 02
T2

www.physicsashok.in 28
NUCLEAR PHYSICS

N 01 N 02
Since, A1 = A2, therefore
T1 T2

N 01 T1 1
or
N 02 T2 4
Initially, total number of nuclei, N0 = N01 + N02
1 4
N 01
N0 and N 02 N0
5 5
At time t, number of nuclei of first isotope that remain undecayed,
t / T1 t / 405
1 1 1
N1 N 01 N0
2 5 2
t / T2 t /1620
1 4 1
That of second isotope, N 2 N 02 N0
2 5 2
Total number of nuclei remaining undecayed at time t,

N0 1
t / 405
4 1
t /1620
N0 1 4 1 t /1620
N = N1 + N2 = + N0 = 5 4
5 2 5 2 2 2

N0
But it is equal to .
2

N0 1 4 1 t /1620 N 0
4
5 2 2 2

t /1620
1 8
or
2 13
t
Taking log, log 2 = log 8 log 13
1620

t
log13 log 8 1620 s
or
log 2

t
log13 3log 2 1620 s 1134 s
or
log 2

Radioactive equilibrium :
Decay of 238 206
92 U into a stable end product 82 Pb is a radioactive series which contains a number of intermediate

members. The intermediate members of each decay series have much shorter half-lives than their parent
nuclide. As a result, if we start with a sample NA nuclei of a parent nuclide A, after a period of time an
equilibrium situation will come about in which each successive daughter B, C, ........... decays at the same
rate as it is formed. Thus the activities RA, RB, RC, ........... are all equal at equilibrium, and since R = N we
have.

www.physicsashok.in 29
NUCLEAR PHYSICS
ANA = BNB = CNC = ...........
This situation is called radioactive equilibrium. The above equation can be used to establish the decay
constant (or half-life) of any member of the series if the decay constant of another member and their relative
proportions in a sample are known.

Example 71: The atomic ratio between the uranium isotope 238U and 234U in a mineral sample is found to be 1.8
104. The half-life of 234U is 2.5 105 year. Find the half-life of 238U.
Sol: The two isotopes are in radioactive equilibrium. Hence activities of the two will be equal.
Thus, 1N1 = 2N2,
where, 1, 2 are decay constant of 238U and 234U respectively.
N1 and N2 are number of atoms of 238U and 234U respectively, we have
ln 2
2 = (year)1,
2.5 105
N1
4
N 2 = 1.8 10 ,
ln 2
1 = , where T is half life of 238U.
T

ln 2 N1 ln 2
Now, 1N1 = 2N2 = (year)1
T N2 2.5 105

N1
T = N 2.5 105 year
2
= 1.8 104 2.5 105 year
half life of 238U = 4.5 105 year

Disintegration of isotopes :
Suppose a sample is a mixture of three radioactive isotopes A, B and C.
Let 1, 2 and 3 be the decay constant of A, B and C respectively.
N1, N2 and N3 be the number of nuclei of isotopes A, B and C respectively at any instant.
If A1, A2, A3 are decay rates of A, B and C respectively then, the net decay rate is
Anet = A1 + A2 + A3
but, A = N net N = 1N1 + 2N2 + 3N3
1 N1 2 N 2 3 N 3
net =
N
N is the total no. of nuclei in the sample at any instant
N = N1 + N2 + N3
1 N1 2 N 2 3 N 3
hence net decay constant, net = N1 N 2 N3

Example 72: A sample of uranium is a mixture of three isotopes 234 235 238
92 U , 92 U and 92 U present in atomic ratio

of 0.006 %, 0.71% and 99.284% respectively. The half life of these isotopes are 2.5 105 years, 7.1 108
years and 4.5 109 years respectively. Calculate the contribution of activity (in %) of each isotope in this
sample.

www.physicsashok.in 30
NUCLEAR PHYSICS
Sol: Let N1, N2 and N3 be the number of the three isotopes in the sample.
we have
N1 : N2 : N3 = 0.006 : 0.71 : 99.284 .....(i)
If 1, 2, 3 are decay constants of these isotopes and as activity A = N, we have
A1 : A2 : A3 = 1N1 : 2N2 : 3N3
(ln 2)N1 (ln 2)N 2 (ln 2)N 3 ln 2
A1 : A2 : A3 = T1 : T2 : T3 , [as = ]
T
N1 N 2 N3
A1 : A2 : A3 = T1 : T2 = T3 .....(ii)

where, T1, T2 and T3 are half lives of the isotopes.


Comparing (i) and (ii) we get
0.006 0.71 99.284
A1 : A2 : A3 = T1 : T2 : T3
0.006 0.71 99.284
= 5 : 8 :
2.5 10 7.1 10 4.5 109
60 99.284
= :1:
2.5 4.5
= 24 : 1 : 22.06
A1 : A2 : A3 = 24 : 1 : 22.06
24 100 1 100 22.06 100
= : :
24 1 22.06 24 1 22.06 24 1 22.06
2400 100 2206
= %: %:
47.06 47.06 47.06
= 51.00% : 2.12% : 46.88%
Activity ratio = 51% : 2.12% : 46.88%
234
Contribution of 92 U = 51%;
235
Contribution of 92 U = 2.12%;
238
and Contribution of 92 U = 46.88% in activity of the sample.

Example 73. The isotopes of uranium U238 and U235 occur in nature in the ratio 128 : 1. Assuming that at the
time of the earths formation they were in equal ratio, make an estimate of the age of the earth. The half-lives
of U238 and U235 are 4.5 109 years and 7.13 108 years, respectively.
Sol. Let N0 be the initial number of atoms. Then from N = N0et, N1 N 0 e (ln 2 / T1 ) t and N 2 N 0 e (ln 2 / T2 ) t

N1 / N 2 e 2 1
ln 2 1/ T 1/ T t

128 e

ln 2 1/ 7.13108 1/ 4.5109 t

9 9
128 eln 21.1810 t or 27 e ln 21.1810 t
7
or t 109 = 5.9 109 years
1.18

www.physicsashok.in 31
NUCLEAR PHYSICS

Types of Nuclear Collision:


Exoergic collision / reaction: If Q is positive, rest mass energy is converted to kinetic mass energy (K3, K4
etc), radiation energy or both, and the reaction is called exoergic.

Note: (i) The kinetic energy KE of the emitted -particle is never quite equal to the disintegration energy Q
because the nucleus recoils with a small amount of kinetic energy when the -particle emerges (since
momentum must be conserved).
*
A4
KE Q , A is the mass number of the parent nucleus.
A
(ii) The mass numbers of nearly all alpha-emitters exceed 210, and hence most of the disintegration energy
appears as the KE of the -particle.
(iii) In the decay process, the energy Q is shared by the antineutrinos and the beta particle. The kinetic
energy of the beta particle can be anything between zero and maximum value of Q.

* K + Ky = Q & p = py
p 2 p 2y
+ 2m = Q
2m y

p 2 m
1 = Q
2m my
4
K 1 =Q
A4
A4
K = Q
A

C63: Find the kinetic energy of the -particle emitted in the decay 238Pu 234
U + .
The atomic masses of Pu = 238.04955 a.m.u.; of U = 234.04095 a.m.u; of 4He = 4.002603 a.m.u.
238 234

Neglect any recoil of the residual nucleus.


Sol: Using energy conservation,
m(238Pu)c2 = m(234U)c2 + m(4He)c2 + K
or K = [m(238Pu) m(234U) m(4He)]c2
= (238.04955 a.m.u. 234.04095 a.m.u. 4.002603 a.m.u.]c2
= 0.0059970 a.m.u. (931.5) MeV/a.m.u.
= 5.59 MeV

C64: Neon-23 beta decay in the following way:


23 23 0
10 Ne 11 Na + 1 e +
0
Find the minimum and maximum kinetic energy that the beta particle 1 e can have. The atomic masses
of 23Ne and 23Na are 22.9945 u and 22.9898 u, respectively.
Sol: Reactant Product

www.physicsashok.in 32
NUCLEAR PHYSICS
23 23
10 Ne 22.9945 10me 11 Na 22.9898 11me
0
e me 1

Total 22.9945 10 me Total 22.9898 10 me


Mass defect, m = 22.9945 u 22.9898 u = 0.0047 u
Q = m c2 = (0.0047 u) 931.5 MeV/u) = 4.4 MeV

The -particle and neutrino share this energy. The maximum kinetic energy of a beta particle in this decay is,
therefore, 4.4 MeV when the antineutrino does not get any share. Energy of particle can range from 0 to
4.4 MeV.

Endoergic collisions
(a) If Q is negative, the reaction is endoergic.
(b) For endoergic reaction to take place a minimum energy has to be supplied.
(c) Threshold energy Eth: The minimum amount of energy that a bombarding particle must have in order to
initiate an endoergic reaction, is called threshold energy Eth.
Using momentum conservation also, we get
m1
Eth = Q m 1 , where m1 = mass of the bombarding particle, m2 = mass of the target nucleus.
2

m1c2 + m2c2 + K1 = (m3 + m4)c2 + K3 + K4


& p1 = p3 + p4; Q + K1 = K3 + K4
p12 p32 p 24 p32 (p1 p 3 )2
Q+ = + = +
2m1 2m 3 2m 4 2m 3 2m 4

p12 2 1 1 2p1p3 p12


2Q + p
= 3m m m +
2m1 3 4 4 m4

1 1 2p1p3 1 1
p32 + p 2
2Q = 0
m3 m 4 m4 1
m 4 m1

4p12 1 1 2 1 1
4 p1 2Q 0
m 24 m3 m 4 m 4 m1

p12 m3 m4 m3 m4
Q m m m ; K1 Q m m m
2m1 3 4 1 3 4 1

m1
If m1 + m2 ~ Q 1 m
2

C65: How much energy must a bombarding proton possess to cause the reaction
7
3Li + 11 H 7 1 7 1 7 1
4 Be + 0 n atomic masses of Li, H, Be and 0 n are 7.01600 u, 1.0783

u, 7.01693 u and 1.0866 u respectively.


Sol: Since the mass of an atom include the masses of the atomic electrons, the appropriate number of electron

www.physicsashok.in 33
NUCLEAR PHYSICS
masses must be subtracted from the given values to get masses of nuclei.
Q-value, = [m( 37 Li ) 3me + m( 11 H ) me]c2 [m( 74 Be ) 4me + m( 10 n )]c2
= [m( 37 Li ) + m( 11 H ) m( 74 Be ) m( 10 n )]c2
= (8.02383 8.02559)u.c2
= 0.00176 u (931.5 MeV/c2)
= 1.65 MeV
Negative sign of Q indicates endoergic reaction. Energy must be supplied for this reaction to take place.
The energy is supplied as kinetic energy of the bombarding proton. The incident proton must have more
than this energy because the system must posses some kinetic energy even after the reaction, so that
momentum is conserved. With momentum conservation taken into account, the minimum kinetic energy of
the incident can be found with the formula.
m 1
Eth = 1 Q = 1 (1.65 MeV) = 1.89 MeV..
M 7

C66. Making use of the table of atomic masses find the energies of the following reachings Li7(, n) B10
Sol. 3
Li7 + 2He4 5B10 + 0n1
Q = (7.01601 + 4.00260) (10.0124 + 1.00867)
Q = 0.00300 amu = 2.79 MeV

Example 74: Find the energy of the reaction N14(, p) O17, if the TO
kinetic energy of the incoming -particle is T = 4.0 MeV Y
and proton outgoing at an angle = 60 to the motion direction
of the alpha-particle has a kinetic energy Tp = 2.09 MeV. X N
T = 60
Sol: T= 4.0 MeV, Tp = 2.09 MeV
Let TO is kinetic energy after collision of oxygen. p Tp
Reaction is, 7N14 + 2He4 8
O17 + 1p1
Let Q - value of reaction Q.
Energy conservation gives, Q + T = TO + Tp .....(i)
Momentum conservation along x-direction gives,
2m T = 2mO TO .cos + 2m p Tp .cos

2m T 2m p Tp .cos = 2mO TO .cos .....(ii)


Momentum conservation along y-direction gives,
2m p Tp .sin= 2mO TO .sin .....(iii)
Squaring the equation (ii) and (iii) on both sides and adding the result, we get,
( 2m T 2m p Tp .cos)2 + 2mpTp sin2 = 2mOTO

2mT 2 (2m T )(2m p Tp ) .cos + 2mpTp(cos2 + sin2) = 2mOTO

1
TO = m [m T + mpTp 2 m m p T Tp .cos] .....(iv)
O

From equation (i), Q = TO + Tp T .....(v)


Putting the value TO from equation (iv) in equation (v), we get,

www.physicsashok.in 34
NUCLEAR PHYSICS

1
Q = Tp T + m [m T + mp Tp 2 m m p T Tp . cos]
O

1
= 2.09 MeV 4.0 MeV + [1 4 + 1 2.09 2 4 1 4 2.09 .co60] MeV
16
Q = 1.14 MeV

NUCLEAR FISSION
Continuous research or artificial transmuttion and especially the study of induced radioactivity, culminated in
the discovery of nuclear fission which is accompanied by the release of enormous amounts of energy. In
ordinary nuclear disintegrations, both natural and artificial, the nucleus is only chipped off rather than broken
and accordingly, the amount of energy released is comparatively less i.e. from about 10 to 23 MeV.
It was discovered in 1939 that the heavy unstable uranium nucleus when bombarded by neutrons splits into
two almost equal fragments which fly apart with great speed and the amount of energy released per fission
is about 200 MeV.
This division of a nucleus into two approximately equal parts as called nuclear fission.

Discovery of fission
The starting point in the discovery of nuclear fission can be traced to the attempts of Fermi in 1934, to
produce transuranic elements by bombarding uranium with neutrons. However, the fission process itself
was discovered in 1939 by German radio chemists Otto Hahn and his two associates Meitner and
Strassmann. After bombarding uranium with neutrons, they performed a series of chemical separations to
identify the products. To their great surprise, they found that the atoms produced by the bombardment of
uranium belonged to elements which lie near the centre of the periodic table. Obviously, a uranium nucleus
after capturing neutron had become so unstable that instead of disintegrating by ejecting one or two particles,
it had split up into two parts.
The actual fission process can be understood with the help of figure which shows a uraniumnucleus capturing
a neutron.
51 Sb133
-rays
92 U
236
51p Unstable
236 82n Antimony
92 U Nuclide
1
0 n

-rays
92p
143n Unstable
Niobium
Nuclide
(a) (b) 41p
48n 99
41 Nb

(c)
The newly-formed nucleus of figure (b) is unstable and starts breaking up into two parts. In breaking up, the
uranium nucleus, behaving like a liquid drop, splashes out small droplets, i.e. neutron and -rays. So great
is the release of energy that the two fission fragments fly apart in opposite directions with tremendous
speeds. It amy, however, be noted that not all uranium nuclei break into Sb and Nb as shown in figure.

www.physicsashok.in 35
NUCLEAR PHYSICS
There are at least 30 different ways in which a fissile nuclide can divide itself. The experimental evidence
seems to favour pairs of fission fragments of unequal masses (asymmetrical fission) accompanied by one to
five or some time more neutrons. In general, fission fragments are unstable nuclei containing an excess
number of neutrons. After a series of -emissions in which neutrons are converted into protons in the
nucleus, a stable nuclide results.
Out of all the neutrons ejected during the fission of uranium, about 99 per cent are ejected in an extremely
short interval of time and are called prompt neutrons. The remaining one per cent of neutrons are emitted a
little later and are called delayed neutrons. The delayed neutrons originate from unstable fragments that
decay by neutron emission on their way to becoming stable nuclei.
It may be noted that division of a fissile nucleus into three fragments of comparable sizes (ternary fission) has
been observed although it is a rare event, occurring about 5 times per million binary fissions.

Types of Fission Reactions


Historically speaking, uranium was the first element to undergo fission. However, soon after it was found
that other elements of high atomic weight could also be made to undergo fission and that particles other than
neutron could be equally effective in this respect.
Natural uranium contains three principal isotopes with the following relative abundance:
U238 99.28% 4.51 109 Y
U235 0.714% 7.1 108 Y
U234 0.006% 2.48 105 Y
It is found that slow neutrons cause fission of U235 but not of more abundant isotope U238 which requires fast
neutrons with energies exceeding 1 MeV. Similarly, Th232 and Pa231 undergo fission when bombarded with
fast neutrons. Fission can also be produced in uranium and thorium by high-energy -particles, protons,
protons, deuterons and -rays etc. Two other nuclides which do not occur in nature but have proved to be
fissionalbe by neutrons of all energies are 92U233 and 94Pu239. In 1947, successful fission of bismuth, lead,
thallium, mercury, gold, platinum and tantalum was achieved in USA by means of -particles, deutrons and
neutrons of 100 MeV and more. With bismuth (Z = 83) fission was detected with 50 MeV deuterons
whereas tantalum (Z = 73) required -particles of 400 MeV energy.
It is worth nothing that only three fissile materials U233, U235, Pu239 are important in the large-scale application
of nuclear energy.
Finally, some heavy nuclei have been found to undergo spontaneous fission. In this process, nucleus divides
in the ground state without bombardment by particles from outside.

Mass distribution of Fission Products


During uranium fission, a large number of nuclides of intermediate charge and mass are found. Their study
is a promising source of information about the mechanism of the fission process itself and also offers the
possibility of discovering hitherto unknown nuclides. Investigations of the fission products of U235 have
shown that the range of their mass numbers is from 72 to 158. About 97% of the U235 nuclei undergoing
fission give fragments which fall into two groups as shown in the fission yield curve of figure .
(i) light group with mass numbers from 85 to 104 and
(ii) heavy group with mass numbers from 130 to 149.
The most probable type of fission which occurs in about 7% of the total cases, gives fission products with
mass numbers 95 and 139.

www.physicsashok.in 36
NUCLEAR PHYSICS
10000
139

95
1000

Number of Fragments
100

10

60 80 100 120 140 160 180


Mass Number (A)
As mentioned earlier, fission fragments have too many neutrons in their nuclei for stability. Consequently,
most of them decay by electron emission. Each fragment starts a short radio-active series involving many
emission of -particles. These series are called fission decay series and chain has three members on the
average although longer and shorter chains occur frequently. One such fission decay chain is shown in figure
which starts with one of the unstable fragments of the fission of U235 nucleus.
O O O O
51 Sb133 52 Te133 53 I133 54 Xe133 55 Cs133
51 52 53 54 55
82 81 80 79 78
5m 60m 5d 5d
Unstable Unstable Unstable Unstable Stable
Antimony Tellurium Iodine Xenon Cesium

Energy Distribution of Fission Products


Energy distribution among the fission products can
be found by measuring their kinetic energy with the 400
Number of frangments

help of suitable ionization chambers. The results of 100


such study on U235 fission have shown that the energy 300
distribution curve is not uniform; rather it is a double- 67
200
peaked curve with maxima at 67 MeV and 100 MeV.
It is seen that while the greatest probability is for a 100
fragment of 100 MeV, the areas under the two peaks
0
which represent the total number of particles in the 40 60 80 100 120
two groups are approximately equal. Energy (MeV)

Neutron Emission in Nuclear Fission


One of the notable features of nuclear fission is that while it is initiated by neutrons it is also accompanied by
the emission of fast-moving neutrons. The number of neutrons released depends on the mode of fission and
on the energy of the neutrons which induce fission. The average values for the number of neutrons emitted
per thermal neutron absorbed by the three important fissile materials are given below:
U235 2.43
U233 2.50
Pu239 2.89
These neutrons are emitted by the fission fragments and not by the compound nucleus.

www.physicsashok.in 37
NUCLEAR PHYSICS
The neutrons emitted as a result of fission process (i.e. fission neutrons) can be divided into two groups:
(i) Prompt Neutrons: These make up about 99.36% of the total fission neutrons and are ejected by the
product nuclei within 1014 second of the fission process. Prompt -rays are also emitted at the same time.
(ii) Delayed Neutrons: These constitute about 0.64% of the total neutrons from the fission of U235. These
are emitted with gradually decreasing intensity for several minutes after actual fission process. Although the
number of delayed neutrons is small, they have a strong influence on the time-dependent behaviour of chain-
reacting systems based on fission and play an important role in the control of nuclear-fission reactors.
Fissile and Fissionable Nuclides
Elements like U235, U233 and Pu239 undergo fission by neutrons of energy from almost zero upwards. Such
nuclei are referred to as fissile nuclides. On the other hand, U238 and Th232 nuclei which have a fission
threshold at 1 MeV are said to be fissionable nuclides.
In general, fissile nuclides have either an even number of protons and an odd number of neutrons or odd
numbers of both. Fissionalbe nuclides, on the other hand, have either even number of protons and neutrons
or an odd number of protons and an even number of neutrons.
Fission Energy
One of the striking features of the fission process is the magnitude of the energy released which is about 200
MeV per fission of U235 nuclide. Before 1939, the largest known nuclear reaction energy was 22.2 MeV
associated with Li6 (d, ) He4 reaction.
The amount of energy released per fission of U235 nuclide may be calculated by the following three methods:
(i) Binding-energy method: As mentioned above all stable fission products have mass numbers in the range
72 to 158 where the average binding energy per nucleon is about 8.5 MeV. However, in the neighbourhood
of uranium, its value is 7.6 MeV. Hence, average binding energy per nucleon is (8.5 7.6) = 0.9 MeV
greater in the fission products than in the compound nucleus of U235. The excess energy is released as fission
energy. Its value is 235 0.9 ~ 200 MeV per fission of U235 nuclide (which has 235 nucleons).
(ii) Mass Defect Method: The energy released per fission can also be estimated by comparing the mass of the
interacting particles and the final fission products.
As mentioned, U235 splits in many ways and the nuclei obtained in the greatest yield in fission by slow
neutrons have mass numbers of 95 and 139. The fission products being initially radioactive, undergo many
-emissions to form ultimately stable nuclides. If molybdenum-95 and lanthanum-139 are taken as pair of
stable products from fission of U235, the fission reaction can be written as
92
U235 + 0n1 62
Mo95 + 57La139 + 20n1
Comparing masses on both sides of the above equation we get,
mass of U235 nuclide = 235.124 amu
mass of one neutron = 1.009 amu
Total = 236.133 amu
mass of Mo95 nuclide = 94.946 amu
mass of La139 nuclide = 138.955 amu
mass of two neutrons = 2.018 amu
Total = 235.919 amu
mass of defect = 236.133 235.919 = 0.214 amu
Therefore, energy released per fission of U235 nucleus
= 0.214 931 ~ 200 MeV
(iii) Kinetic energy measurement method: The total amount of energy released per fission is equal to the sum
of the following energies:
(a) the kinetic energy of fission fragments. As seen from figure the average value of this energy for U235 is
167 MeV.

www.physicsashok.in 38
NUCLEAR PHYSICS
(b) the kinetic energy of fission neutrons. Since the average number of neutrons emitted per fission of U235
is 2.43 or say 2.5 and the average kinetic energy of these neutrons is 2 MeV, total kinetic energy of fission
neutrons is 2.5 2 = 5 MeV.
(c) the kinetic energy of prompt -rays. Its value is about 7 MeV.
(d) the total energy of the decay process in the fission decay chains.
This includes the energy carried away by radiations like -rays, -rays and neutrons. Its value is nearly 21
MeV.
The total of all the above energies is
= 167 + 5 + 7 + 21 = 200 MeV

C67: A U235 nucleus is fissioned by a thermal neutron and two fission fragments and two neutrons are produced.
Compute the fission energy released if the average binding energy per nucleon is 7.8 MeV in fissioned U235
nucleus and 8.6 MeV in the fission fragments.
Sol: Greater binding energy of the fission fragments indicates that there has been release of energy during fission
of low-binding energy nucleus U235.
Fission energy released is = (234 8.6 236 7.8)
= 171.6 MeV

Theory of Fission Process


The first attempt to explain the mechanism of fission process was made by Bohr and Wheeler who accounted
for many of the properties of fission on the basis of the liquid-drop model of the nucleus.

1 2 3 4 5 6
The shape of the drop depends on a balance between the surface tension forces and Coulombic repulsive
forces. The excitation energy given to the drop during the capture of the slow or thermal neutron sets up
oscillations within the drop. These oscillations tend to distort the spherical shape so that the drop becomes
ellipsoid in shape. The surface tension forces try to make the drop return to its original spherical shape while
the excitation energy tends to distort the shape still further. If the excitation energy and hence oscillations are
sufficiently large, the drop pattains the dumbbell shape as shown in figure. The Coulombic repulsive forces
then push the two bells further apart until the dumbbell splits into two similar drops each of which assumes
a spherical shape. The sequence of steps leading to fission is shown in figure.
However, if the excitation energy is not large enough, the ellipsoid will return to the spherical shape. In that
case, the excitation energy is given out in the form of -rays and the process becomes a radioactive capture
process rather than fission process.

LECTURE 5
Nuclear Reactors :
(a) A nuclear rector is a system designed to control the chain reaction of fission with continuous energy production.
(b) A useful factor for describing the level of operation of a reactor is the reproduction constant K. It is defined
as the average of neutrons available from each fission that will cause another fission. For a controlled or self
sustained chain reaction K must be maintained close to unit.
i.e. K 1 for controlled chain reaction.

www.physicsashok.in 39
NUCLEAR PHYSICS
(c) Fuel: This is the fissionable material. Commonly used materials
are U238 enriched in U235 and plutonium (Pu239).
(d) Moderator: Fast moving neutrons cannot trigger the fission Control
rods
of U235 and have a very high chance of being captured by
U238 which is not fissionable. It is therefore necessary to
use moderators to slow down the neutrons. Shield

(e) Coolant: Air, water or CO2 are used as a coolant to


remove the heat released inside the reactor.
(f) Control Rods: Cd(cadmium) which is a good absorber
Fuel
of neutrons is used to control the rate of fission and also to elements
shut down the reactor in case of emergency.
(g) Types of reactors: Moderator
(i) Thermal reactors: In these reactors fission is produced by Nuclear Reactor

slow neutrons ro thermal neutrons.


(ii) Breeder reactor: Breeder reactors generally use fast neutrons in these reactors U238 is converted into
Pu239 by capture of neutrons. Pu239 is fissionable. Thus such reactors also produce fuel in addition to the
energy released through fission.
(h) Critical mass: For a fuel there is a critical mass below which the fissionable material is completely safe.
But for a mass above the critical mass more neutrons are produced than are lost so that the chain reaction
builds up rapidly and the system explodes.
The atomic bomb are therefore stocked as subcritical mass such that the combined mass is greater than the
critical mass resulting in a spontaneous explosion.

Example 75: In a nuclear reactor, fission is produced in 1 g for U235 (235.0439u) by a slow neutron (1.0087 u).
Assume that 35Kr92 (91.8973 u) and 56Ba141 (140.9139 u) are produced in all reaction and no energy is lost.
(a) Write the complete reaction,
(b) Calculate the energy released per fission,
(c) Calculate the total energy produced in kilowatt hour. Given 1 u = 931.5 MeV/c2.
Sol: (a) The nuclear fission reaction is
92
U235 + 0n1 56
Ba141 + 36K92 + 3 0n1
(b) Mass defect, m = [(mu + mn) (mBa + mKr + 3mn)
m = 256.0526 u 235.8373 u = 0.2153 u
Energy released per fission,
Q = 0.2153 u 931.5 MeV/u = 200.6 MeV
6.02 1023
(c) Number of atoms in 1g = = 2.56 1021
235
Energy released in fission of 1 g of U235
= 200.6 2.56 1021 MeV
= 5.14 1023 MeV
= (5.14 1023) (1.6 1013)J
= 8.2 1010 J = 8.2 1010 W-s
8.2 1010
= KWh
3.6 106
= 2.28 104 KWh
www.physicsashok.in 40
NUCLEAR PHYSICS
Example 76: In neutron-induced binary fission of 92U235 (235.044) two stable end-products usually found are
42
Mo98 (97.905) and 54Xe136 (135.917). Assuming that these isotopes have come from the original fission
process, find (i) what elementary particles are released (ii) mass defect of the reaction (iii) the equivalent
energy released.
Sol: (i) The reaction can be written as
0
n1 + 93U235 = 43Mo98 + 51Xe136
It is seen that the total Z-value of the two stable fission products is (42 + 54) = 96. It is 4 units more than
that of 92U235. For balance, the original unstable fission products must have got Z = 92. Obviously, the
original unstable products must have emitted 4 -particles before becoming stable. Now, mass number on
right-hand side is 2 units less than on the left-hand side. It means that tow fission neutrons must have been
produced. Hence, the fission reaction can be represented by the following equation:
0
n1 + 92U235 = 42Mo98 + 54Xe136 + 4 1e0 + 20n1
(ii) m = LHS mass RHS mass
LHS mass = (1.009 + 235.055) = 236.053 amu
RHS mass = (97.905 + 135.917 + 4 0.0055 + 2 1.009)
= 235.842 amu
m = 236.053 235.842
= 0.211 amu
(iii) Energy released = 0.211 931 = 196 MeV

Example 77. About 180 MeV energy is released when one nucleus of 92U235 undergoes fission. estimate the
energy released from 1 kg of U235, assuming that each nucleus undergoes fission.
1000
Sol. 1 kg of U235 = 1000 g = mole
235
1000
number of atoms = 6.02 1023
235
6.02
total energy released = 1026 180 MeV
235
6.02 18
= 1027 106 1.6 1019 J = 7.37 1013 J
235

C68. Calculate the energy released in slow neutron capture by Pu239. Mass of Pu239 = 239.127 amu, Pu240 =
240.1291 amu, 0n1 = 1.008665 amu.
Sol. Energy released = mass defect in energy units
= (239.127 + 1.008665 2401) amu
= 0.006565 931 MeV = 6.1 MeV

Example 78. A nuclear reactor generates P = 20 MW power at efficiency = 60% by nuclear fission of a radio-
nuclide whose half life is T = 2.2 years. If each fission releases energy E = 200 MeV, calculate time during
which = 10 mole of the radionuclide will be consumed completely.
(Avogadro number, N = 6 1023, loge 2 = 0.693, 1 year = 3.15 107 s)

www.physicsashok.in 41
NUCLEAR PHYSICS

Sol. To operate the nuclear reactor, let the number of fissions required per second be n0. Then energy released
per second by fission reactions = n0E
Since, efficiency of the reactor is , therefore, power output from the reactor = n0E.
But it is equal to P therefore, P = n0E
P
or n0
E
Let at an instant number of nuclei of radionuclide be n then rate of decay = n where is decay constant
log e 2
which is equal to .
T
dn
Hence, net rate of decrease of nuclei n n 0
dt

dn n log e 2 P n E log e 2 PT
or
dt T E ET
dn dt
...(1)
PT n E log e 2 Et
At t = 0, number of nuclei are n = N and time t is to be calculated when all the nuclei are consumed or
when n = 0, t = ?
Integrating equation (1) with these limits,
0 dn t dt
N PT n E log 2
e

0 ET

T N E log e 2
t log e 1 108 log e 1.0576 s
log e 2 PT

Example 79. The element Curium 248


96 Cm has a mean life of = 1013 second. Its primary decay modes are
spontaneous fission and -decay, the former with a probability of P1 = 8% and latter with a probability of
P2 = 92%. each fission releases E = 200 MeV energy. The masses involved in -decay are as follows
248 244
96 Cm = 248.072220 u, 94 Pu = 244.06400 u and 42 He = 4.002603 u.
Calculate the power output from a sample of N = 1020 Cm atoms. (1 u = 931 MeV/c2)
1
Sol. Decay constant, 1013 sec1
mean life ()
Rate of decay from a sample of N atom, A = N = 107 sec1
Since, probabilities of fission and -decay are P1 and P2 respectively, therefore, rate of decay due to
fission, A1 = P1A
5 1
or A1 = 8 10 sec
and rate of decay due to -emission, A2 = P2A = 9.2 106 sec1.
Since, each fission releases energy E, therefore, rate of release of energy due to fission
= A1 . E

www.physicsashok.in 42
NUCLEAR PHYSICS

248 244
Equation of -decay is 96 94 Pu 42 He
Cm
Mass most during each -decay,
m = [248.072220 (244.06400 + 4.002603)] u
= 5.617 103 u
Energy released during each -decay, E = 5.617 103 931 MeV
E = 5.23 MeV
Rate of release of energy due to -decay = A2 . E
Total rate of release of energy = A1E + A2E
But total rate of release of energy is equal to power output.
Therefore, power output, P = A1E + A2E = 3.33 105 W

Nuclear fusion
It is the process of combining or fusing two lighter nuclei into a stable and heavier nuclide. In this case also,
large amount of energy is released because mass of the product nucleus is less than the masses of two nuclei
which are fused.
Many reactions between nuclei of low mass numbers have been brought about by accelerating one or the
other nucleus in a suitable manner. These are often fusion processes accompanied by release of energy.
However, reactions involving artificially-accelerated particles cannot be regarded as of much significance
for the utilization of nuclear energy. To have practical value, fusion reactions must occur in such a manner as
to make them self-sustaining, i.e. more energy must be released than is consumed in initiating the reaction. It
is thought that the energy liberated in the Sun and other stars of the main sequence type is due to the nuclear
fusion reactions occurring at the very high stellar temperature of 30 million K. Such processes are called
thermonuclear reactions because they are temperature-dependent.

Steller Thermonuclear Reactions:


Following two sets of thermonuclear reactions have been proposed as sources of energy in the Sun and
other stars of the main sequence:
(i) proton-proton (p - p) chain and (ii) carbon-nitrogen (C-N) cycle.
At low temperatures corresponding to those in the Sun when it was first formed, the proton-proton chain
was predominant. In the present state of the Sun with its higher central temperature and larger He4
concentration, the C-N cycle is supposed to be the main source of its energy.

Proton-Proton Chain
It is so called because the step involves the combination of two protons. When two protons fuse, they
produce a deuteron nucleus, a positron and a neutrino thus:
1
H1 + 1H1 = 1D2 + 1e0 + v 2
The deutron then combines with another proton to yield helium-3.
1
D2 + 1H1 = 2He2 + 2
The two helium-3 nuclei fuse to produce helium-4 thus
2
He3 + 2He3 = 2He4 + 21H1 + 1e0
It should be noted that for the third reaction to occur, each of the first two reactions must occur twice. The
net effect of the reactions is
41H1 = 2He4 + 21e0 + 2 + 2v

www.physicsashok.in 43
NUCLEAR PHYSICS
Obviously, four hydrogen atoms are fused to produce one helium atom with a total energy release of about
26.7 MeV. When the kinetic energy of neutrions is substracted, the energy is 26.2 MeV. The emitted
positrons are annihilated by free electrons with the production of -rays.

Carbon-Nitrogen Cycle
It was proposed by H.A. Bethe to account for the energy production in the Sun and other stars of main
sequence. In this cycle, carbon acts as a nuclear catalyst. The cycle starts when a proton (hydrogen atom)
first interacts with carbon-12 nucleus with the release of fusion energy thus
6
C12 + 1H1 = 7N13 +
The product N13 is known to be radioactive, emitting a positron with a half-life of 10 minutes. Hence, it
decays in a very short time according to the relation
7
N13 = 6C13 + 1e0 + v
The stable C13 nucleus reacts with another proton, thereby liberating more energy
6
C13 + 1H1 = 7N14 +
The stable product N14 combines with third proton thus
7
N14 + 1H1 = 8O15 +
The O15 nucleus is a positron emitter with a half-life of 2.06 minutes which decays by the process
8
O15 = 7N15 + 1e0 + v
Finally, the resulting N15 nucleus interacts with the fourth proton thus:
7
N15 + 1H1 = 6C12 + 2He4
By adding up the above six equations and cancelling out those nuclei which appear on both sides, it is seen
that four hydrogen atoms are consumed and, in return, 2 positrons, 3 -rays and one helium nucleus are
created. In other words, hydrogen is burned and helium is created.
The overall process may be written as
41H1 = 2He4 + 21e0 + 24.7 MeV
The annihilation of positrons supplies an additional energy of 2 MeV so that total energy released is 26.7
MeV.
The fusion energy released may also be found by the loss of mass during the above reaction:
41H1 = 4 1.008144 = 4.032576 amu
2
He4 = 4.003873 amu
21e 0 = 2 0.000558 = 0.001115 amu
mass loss = 4.032576 (4.003873 + 0.001115)
or m = 0.028857 amu
energy released = 931 0.028857 = 26.7 MeV
It is worthnoting that the above energy release is less than that in nuclear fission. However, its value is
26.7/4 = 6.7 MeV per nucleon as compared to less than 1 MeV per nucleon in fission process.

Controlled Thermonuclear Reactions


The fact that nuclear fusion reactions release large amounts of energy, as in stars, has attracted much
attention and continuous search is being made for finding practical means or controlled release of such
energy. It has however, been found that reactions of C-N cycle and proton-proton chain occur too slowly
to be of any practical use. Other thermonuclear reactions which occur much more rapidly and depend on
aboundant hydrogen isotopes like deuteron (1D2 or 1H2) and tritium (1T3 or 1H3) and hence seem more
practical proposition, are as under:
(i) 1H2 + 1H2 = 2He3 + 0n1 + 3.3 MeV www.physicsashok.in 44
NUCLEAR PHYSICS
or 1D2 + 1D2 = 2He3 + 0n1 + 3.3 MeV
(ii) 1H2 + 1H2 = 1H3 + 1H1 + 4 MeV
or 1D2 + 1D2 = 1T3 + 1H1 + 4 MeV
(iii) 1H2 + 1H3 = 2He4 + 0n1 + 17.6 MeV
or 1D2 + 1T3 = 2He4 + 0n1 + 17.6 MeV
The most important aspect of the above nuclear fusion reactions is that deuterium is available easily and
aboundantly. It occurs in nature with an aboundance of one part in six thousands of hydrogen and can be
separated from the lighter isotope quite cheaply. Five litres of water contain about 1/8 gram of deuterium
but its energy content if it could be used as a fuel in a thermo-nuclear reactor, would be equivalent to 130
litres of petrol! The more than 5 1019 kg of water present in the oceans could thus supply worlds power
requirement for several million years at negligible cost if the deuterium could be utilized to provide energy by
fusion reactions. However, as discussed below, there are some difficult problems to be solved before man-
made controlled fusion reactors can become a reality.
Condition for Controlled Fusion
In order to provide useful energy, the fusion process must be self-sustaining. Once the temperature of
deuterium (or a mixture of D2 and T3) has been raised to the point at which fusion occurs at an appreciable
rate, the energy released must be sufficient, at least, to maintain that temperature. The minimum temperature
is known as critical ignition temperature and may be defined as that temperature above which the rate of
energy production by fusion exceeds the rate of energy loss. Its value is about 5 keV (i.e. 50 million K) for
a D-D- reaction. At these temperatures, the atoms are entirely stripped of their electrons. The result is a
completely ionized gas or plasma consisting of atomic nuclei (like deuterons, tritons and protons) and
electrons in rapid random motion. It is practically impossible to contain such plasma in walls of ordinary
materials. In the Sun, the fusion reactions are contained by a tremendous gravitational pressure. Such a high
pressure is yet not available for controlled thermo-nuclear reactions on earth although the plasma can be
contained in a magnetic field. Hence, main problem is to devise an apparatus in which plasma can be
obtained by means of a magnetic field at the kinetic temperatures required for the fusion reactions to proceed.
Another necessary condition for a self-sustaining thermonuclear system is known as Lawson criterion. It is
based on the requirement that in the operation of a fusion reactor, the total useful recoverable energy should
be at least sufficient to maintain the temperature of te reacting nuclei. Lawson criterion can be expressed in
terms of the product nt where n is the number of reacting nuclei per m3 and t is the time in seconds in which
the thermonuclear reaction takes place. The minimum value of nt for D-T system is 7 1019 and D-D
system is 2 1021. In controlled thermonuclear reactions, t is taken as the time during which the high-
temperature plasma can be confined.
It will be seen from above that both the critical ignition temperature and Lawson criterion are much more
favourable for D-T system than for D-D system. But the former system has the draw-back that it requires
tritium which has to be obtained by nuclear reactions (because it does not occur in nature). Tritium can,
however, be made by bombarding lithium with slow neutrons in a reactor thus:
3
Li6 + 0n1 = 1H3 + 2He4 + 4.8 MeV
This reaction can be made to serve two useful purposes. In the thermonuclear reactor using D-T reaction,
the escaping neutrons carry off much of the energy (about 14 MeV per neutron). This energy can be
converted into heat by slowing down these fast neutrons in a blanket of beryllium surrounding the reactor in
which plasma is produced. The slow neutrons are captured by lithium which produces tritium. The blanket
could thus consist of moderator (i.e. beryllium), coolant and lithium. The heat generated by the moderation
and absorption of neutrons could thus be transferred by coolant to external heat exchangers and then to
turbines which could run alternators.

www.physicsashok.in 45
NUCLEAR PHYSICS
Hydrogen bomb
This bomb is 1000 times more powerful than the atomic bomb which is based on nuclear fission. Hydrogen
bomb is based on the fusion if the hydrogen atoms into heavier ones by the thermonuclear reactions with
release of enormous energy. The essential conditions for the operation of the hydrogen bomb are extremely
high temperatures and pressures required for the fusion to start. Once started, the fusion itself maintains the
temperatures to keep the process going. For this purpose, the atom bomb (fission bomb) is used as a
primer which, by first exploding, provides the high temperature and pressure necessary for the successful
working of the hydrogen bomb (fusion bomb).
A fusion bomb is superior to a fission bomb because of the following reasons:
(i) The energy release in a hydrogen bomb is open-ended i.e. it has no upper limit. It depends on how much
fusible material is present in the bomb.
(ii) It has no limitation of a critical size of the fusible material unlike an atomic bomb. If the active material in
an atomic bomb exceeds the critical size, spontaneous explosion results. Hydrogen bomb cannot explode
unless ignited i.e., heated to critical ignition temperature and any amount of fusible material is safe until
ignited. Thus the amount of fusible material in a hydrogen bomb is not limited.
Cobalt Bomb
It consists of a hydrogen bomb which is encased in a sheath of metallic cobalt and is more lethal and
destructive than a simple uncased hydrogen bomb. When the hydrogen bomb explodes, it gives off neutrons
which act on the cobalt cover and render it intensively radioactive due to the formation of Co60 that is 300
times more powerful than radium. During explosion, the radioactive cobalt is pulverised and converted into
a gigantic radioactive cloud which can spread over thousands of kilometres killing everything living in that
area.
Fission and Fusion:
One thing common between the two nuclear processes si that they release very large amounts of energy.
But there are many differences in the mechanisms of the two processes.
(i) Fission involves breaking up of a heavy nucleus into lighter nuclei. Fusion, on the other hand, involves
combining of two lighter nuclei into on heavy nucleus.
(ii) The links of the fission process are neutrons while the links of a fusion process are protons.
(iii) Fission proceeds best with thermal neutrons where thermal means room temperature. Fusion proceeds
best with thermal particles where thermal means temperatures of millions of K.

Example 80: the masses of 1H1 and 2He4 atoms are 1.00813 amu and 4.00386 amu respectively. How much
hydrogen must be converted to helium if solar constant is 1.35 kW/m2 an the earth is 1.5 1011 m from the
Sun.
Sol: This thermo-nuclear reaction may be written, in its essentials as
41H1 = 2He4 + 1e0
Neglecting the two positrons, it is seen that 4 hydrogen atoms fuse to produce one atom of helium.
Mass of 4 hydrogen atoms = 4 1.00813 = 4.03252 amu
Mass of one helium atom = 4.00386 amu
Decrease in mass, m = 4.03252 4.00386
= 0.002866 amu
energy produced = 0.02866 931 = 26.68 MeV
This is the energy released when four hydrogen atoms fuse. Hence, energy produced by one hydrogen
atom.
26.68
= = 6.67 MeV = 6.67 1.6 1013 = 10.67 1013J
4
= 6.02 1026 10.67 1013 = 6.42 1014 J

www.physicsashok.in 46
NUCLEAR PHYSICS
The solar constant represents the amount of energy received per second
by 1 m2 area held perpendicular to the Suns rays at a distance equal to the
mean distance of the earth from the Sun. Sun 1.5 1011
Given value of solar constant
= 1.35 kW/m2
= 1.35 1000 W/m3
= 1350 J/s/m2
Total energy emitted by the Sun is equal to the energy received by the inner surface of the imaginary sphere
drawn with Sun as centre and radius = 1.5 1011 m.
Surface area of the sphere
= 4R2 = 4 (1.5 1011)2 = 28.28 1022 m2
energy received by this surface area per second is
= 1350 28.28 1022 = 38.18 1025 J/s
It also represents the energy emitted by the Sun per second.
Mass of hydrogen consumed is
38.18 1025
= = 5.59 1011 kg/s = 5.95 108 tones/second
6.42 1014
C69: Calculate the energy liberated when a Helium nucleus is formed by the fusion of two deuterium nuclei. The
mass 1H2 = 2.01478 a.m.u. and mass of 2He4 = 4.00388 amu.
Sol: The reaction may be written as
1
H2 + 1H2 = 2He4 + Q
2.01478 + 2.01478 = 4.00388 + Q ,
Q = 0.02568 amu = 0.02568 931 = 23.9 MeV
C70. In the fusion reaction 1H2 + 1H2 2He3 + 0n1, deuteron, helium and the neutron have masses 2.015 amu,
3.017 amu and 1.009 amu, respectively. Estimate the total energy released if 1 kg of deuterium undergoes
complete fusion.
Sol. Mass difference = 2 2.015 (3.017 + 1.009) = 0.004 amu
energy released = 0.004 931 MeV = 3.724 MeV
1
Energy released per deuteron = 3.724 MeV
2
6.02 1026
No. of deuterons in 1 kg =
2
energy released per kg = 1.862 3.01 1026 MeV = 5.6 1026 106 1.6 1019 9 1013 J
C71. In some stars, three 2He4 nuclides fuse together to form 6C12 of mass 12.0000 amu. How much energy is
released per fusion of 6C12 ? Rest mass of 2He4 = 4.002603 amu. Find also the rate of consumption of
helium to maintain the radiative power of the star at 4 1021 MW. mass of 2He4 atom = 6.9 1027 kg.
Sol. Energy released per fusion = mass defect in amu
= (3 4.00263 12.0000) amu = 0.007809 931 MeV = 7.27 MeV
energy released by n atoms
7.27 1.6 1019 106
nJ
3
= 4 1021 106 (given) , n = 1.103163 1040
mass of helium atoms burnt per second
= 1.03163 1040 6.9 1027 kg = 7.1 1013 kg

www.physicsashok.in 47

You might also like